Download as pdf or txt
Download as pdf or txt
You are on page 1of 869

Baffour Ba Series

ELECTIVE

MATHEMATICS
for

Schools and Colleges

By:

BAFFOUR ASAMOAH
CERT „A‟– OTC, Dip(Edu) – UCC, B.Ed (Maths) – UEW, BAT(Admin) – CSUC, BSc (Maths & Stats) - PUC
Copyright© 2018
All rights reserved under copyright law.
No part of this publication may be published, produced or stored on a print or any retrievable media
without approval or authorization.

For suggestions & views / copies;


Please call any of the following numbers:
0208211966 / 0244282977

ISBN : P0002417952

Baffour Ba Series , Further Mathematics for Schools Page i


PREFACE
Baffour Ba Series, is a series of Mathematics, covering the contents of Senior High Schools and
Colleges syllabus.

In this book, each chapter is broken down into short, manageable sections intended to completely
alleviate or reduce to a large extent, the canker that is termed in Ghanaian context as “Math‟s
Phobia” by providing in its content, concise notes and commentaries with well – illustrated
diagrams on each topic and subtopic of the Senior High Schools and Colleges Syllabuses.

The book reflects the authors experience that students work better from work examples than
abstract discussion of principles, hence the provision of numerous and comprehensive range of
worked examples. A numbered, step-by-step approach to problem solving, trial test and challenge
problems intended to cater for gifted students are greatly featured in this book. Not left out in this
book is “exercises” on each subtopic which contains an extensive selection of Multiple- choice,
Fill-ins, True or False, Essay – type questions similar in standard to the WASSCE questions.
Tackling these exercises is no doubt, an excellent form of revision. Answers to all exercises are also
provided to help students assess their level of progress.

Taking into accounts, full analysis of the pattern and level of difficulty of examination questions,
the last part of the book is “some solved past questions”, in its content.

Baffour Ba Series , Further Mathematics for Schools Page ii


ACKNOWLEDGEMENTS
A myriad of services including spiritual, moral, inspirational, motivational and financial support,
typing, editing, printing, designing and what have you, were deeply rendered by friends, family
members, students, staff members, some co - tutors of J.H.S, S.H.S and Colleges of Education to
make this work a success. To mention a few: Mrs. Joyce Mensah, Mr. Kwabena Agyeman (Check
Yourself), Mr. Asamoah Koduah Thick, Mr. Stephen Amoateng, Mr. Frank Owusu, Mr. Kwaku
Boateng, Mr. Augustine Boadi, Mr. Isaac Owusu Mensah, Mr. Owusu Blessing, Mr. K.D. Bosiako,
Mr. Brownson Adusei, Mr. Bernard Kusi Knambers, Mr. Attakorah Jacob, Mr. Sampson Brobbey,
George Osei Mensah (Kin – George), Cherish Asamoah Baffour – Ba, Prize Asamoah Agyei
Baffour Ba Snr, Prize Asamoah Agyei Baffour Ba Jnr, Grace Adwoa Pomaa, Benjamin Osei –
Mensah (Benjii), Appiah Collins, Maybel Owusu Ansah, Antwi Gloria, Priscilla Mensah, Ernest
Baffoe, Oteng Kwabena, Mrs Diana Amankwah, Mr. Opoku John, the Department of Mathematics,
AGASS, and the Department of Mathematics, OKESS .

I also wish to acknowledge the fact that the book is not absolutely free from errors of typing,
grammar and inaccuracy. These occurred as a result of oversight but not ignorance and
incompetence on the part of the author and editors. These should therefore, not undermine the
credibility of the book, for they say “to err is human”. However, your comments, corrections,
suggestions and criticism are warmly welcomed for consideration and rectification in the next
edition.

Baffour Asamoah
(Baffour Ba)

Baffour Ba Series , Further Mathematics for Schools Page iii


Table of Contents
1. Sets …………………………………… 1 – 34  Types of Function
 Meaning of Sets  Proof of one - to - one functions
 Representation of Sets  Onto Functions
 Types of Set  Increasing Function and Decreasing Function
 Cardinal Number of Set  Even Functions and Odd Functions
 Pair of Sets  Quadratic Function and Linear Function
 Joint or Overlapping Set  Graphs of Function
 Power Set  Domain and Range of a Function
 Union of sets and intersection of sets  Evaluating a Function
 Difference of Two Sets  Word Problems
 Complement of a Set  Operations on Functions
 De Morgan‟s Law  Composite Function and its Decomposition
 Algebra of Sets  Inverse of a Function
 Properties of Complements of a Set  Graph of f and
 Diagrams for Three Sets  Quadratic Function
 Three Set Problems  Drawing the Graph of f(x) = ax2 + bx + c = 0,
 Equation of Axes or Line of Symmetry
2. Surds………………………………….. 35 – 50  The Minimum and Maximum Points
 Meaning and Properties of Surds  Quadratic Function f(x) = ax2 + bx+ c in the
 A simplified Surd as single Root from f(x) = a(x - h)2 + k
 Addition and Subtraction of Surds  Quadratic Equations
 Multiplication and Division of Surds  Word Problems
 Rationalizing Denominators  Sum and Product of Roots of a Quadratic
 Radical Equation Equation
 Equality of Surds  Forming Quadratic Equations from its roots
 The Square Root of an Irrational Number  Finding Two Unknowns from the Roots of
the equation
3. Binary Operations………………………51 – 72  Nature of Root of a Quadratic Equation
 Involving surds
 Combining Two Binary Operators 5. Polynomials………………………. 130 – 153
 Truth Set under a given Binary Operation  Degree of a polynomial
 Table of Binary Operation  Types of Polynomials
 Properties of Binary Operations  Evaluating Polynomials
 Identity Element and the Binary table  Addition and Subtraction of Polynomials
 The Inverse Property  Multiplication of Polynomials
 The Inverse Element on the Binary Table  Opposite of a Polynomial
 Word Problems
4. Relations and Functions…………………73 – 129  Division of Polynomials :
 Idea of a Relation  The Synthetic Method
 Relations as Ordered Pairs  The Remainder Theorem
 Range of a relation  Application of Remainder Theorem
 Types of Relations  Factor Theorem and its Application
 Function Notation  Using a Combination of the Factor and
 Representation of Functions Remainder Theorems to find Unknowns

Baffour Ba Series , Further Mathematics for Schools Page iv


 Finding a Polynomial with Prescribed Zeros  Maximum or Minimum Values
 Solving Cubic Equations  Maximizing and Minimizing a Linear
Function
6. Rational Functions…………………….154 – 167  Using the Search Line ax + by = k
 Rational Functions and their Domain  Quadratic Inequalities
 The Range  Sign Graph of Factors and Test Point Method
 Evaluating Rational Functions
 Reducing Rational Functions 9. Coordinate Geometry…………….…….235 – 269
 Rational Functions of Two Variables  A Straight Line
 Identity  Distance between Two Points
 Opeartions of Rational Functions  Three Points Forming a Right Triangle
 Partial Fractions : Linear, repeated linear  Area of a Triangle
factor, irreducible quadratic factor and a  Unknown Coordinate given the Area of a
repeated quadratic factor Triangle
 Mid-point of a Line
7. Binomial Theorem ……………………168 – 201  Finding an Unknown Endpoint of a Line
 Binomial Expansion  Collinear Points
 The Pascal‟s Triangle  Division of a Line Segment in a given Ratio
 Factorials and Combinatorial Numbers  Finding the Ratio of Division
 Binomial Expansion of (a + b)n and (1 + a)n  Gradient of a Line from Two Points, a Linear
 Evaluating Expansions of Sums and Equation and from a Linear Graph
Difference  Equation of a Line (from two given Points,
 Substitution from gradient and a point on the line and from
 Related Application the gradient and the intercept on the y – axis)
 Unknown Coefficients or an Unknown Index  Perpendicular Lines
 Selecting a Particular Term  Equation of a parallel Line
 Sums of Binomial Coefficients  The Equation of a Line which Passes through
 Negative and Rational Exponents the Point ( , ) and Perpendicular to another
 Conditions for Convergence of Binomial Line
Series
 Equation of Perpendicular Bisector of a line
 Unknowns of Expansions with Rational
 Application to Parallelograms
Exponents
 Point of Intersection of Two Lines
 Perpendicular Distance from a Line
8. Linear Programming…………………202 – 234
 Distance between Two Parallel Lines
 Inequalities
 Slope of Two Lines Which Meet at a Point at
 Meaning of - and
a given distance from a given Point.
 Basic Interval Notation
 Points on the same Side or Opposite sides of a
 Solving Linear Inequalities
Line
 Graphing Linear Inequality in one variable
 Angles between Two Lines
 Linear Inequalities in Two variables
 Congruencies of a Triangle;
 Graph of Linear Inequality in Two Variables
 Circle Centres
 Systems of Linear Inequalities:
 Linear Programming
10. Probability 1…………………………...270 – 300
 Graphing the constraints
 Equally Likely Events
 (Mathematical model)
 Complementary Events
Baffour Ba Series , Further Mathematics for Schools Page v
 Compound Events  Concept of Motion
 Coin and a Die Experiments  Displacement
 Other Related Experiments  Equations of Motion
 Mutually Exclusive Events:  Velocity and Acceleration
 Independent and Dependent Events  Motion Under Gravity
 Conditional Probability  Motion along Inclined Planes
 Selection with and without Replacement  Friction and Coefficient of Friction
 Probability and Tree Diagrams  Momentum and Impulse
 Conservation of (Linear) Momentum:
11. Vectors………………………………...301 – 334  Direct Impact
 Velocity and Force Vector  Velocity-Time graph
 Unit Vector
 Unit Vector in the Opposite Direction to a 14. Circles…………………………………..396 – 436
given Vector  Equation of a Circle with Center (a, b)
 The Dot (scalar) Product and its Properties  Equation of a Circle with Center at the Origin
 Finding the Constants of a Vector given the  The General Equation of a circle
Angle Between two Vectors.  Center and Radius, from a given equation
 Parallel and Orthogonal Vectors  Equation of a Circle given the Diameter
 Sine and Cosine Rules  Equation of a Circle through 3 Points
 The Triangular Law of Vectors  Tangent and Equation of Tangent to a Circle
 Relating Free Vectors and Position Vectors  Verifying that a Point Lies on a Circle
 Applications  Finding the Gradient of the Tangent
 Parallelogram Law of Vectors  Length of a Tangent from an External Point
 Position Vector of a Point Dividing a Straight  Normal and Equation of a Normal to a Circle
line in a Given Ratio  Condition of Tangency
 Using Vector Approach to Solve Bearings  Points Inside , On or Outside a Circle
 Definition of Compb a  Intersection of a Circle and a Line
 Point of Intersection of Two Circles
12. Statics…………………………………..335 – 372  Equation of Locus
 Scalar and Vector Quantities
 The Resultant Vector, Parallelogram Law and 15. Parabola……………………………..437 – 443
Triangular Law of Vectors  Meaning of a Parabola
 Magnitude of the Resultant Vector  Standard Equation of a Parabola
 Force and Resultant Force  Other Forms of the Equation of a Parabola
 Laws of Forces  Equations of Tangent and Normal to a
 Resolution of Forces Parabola
 Two forces acting at a point
 Law of Polygon of Forces 16. Sequence and Series…………….…….444 – 472
 Resultant of Coplanar Forces  Arithmetic Progressions
 Equilibrium of Forces  Finite Arithmetic Sequence
 Polygon of Forces  Finding the nth Term from Two given Terms
 Lami‟s Theorem  Inserting Arithmetic Means
 Principles of Moments  Sum of Arithmetic Series
 Finding the Number of Terms or the Common
13. Dynamics……………………………….373 - 395 Difference Given the Sum of Terms

Baffour Ba Series , Further Mathematics for Schools Page vi



 Sum of A.P. from Two Given Terms Trigonometric Ratios of 300, 600, 450 and 900
 Word Problems Involving A.P.  Trigonometry Ratios for Any Angle
 Geometric Progressions  Negative Angles
 The sequence of numbers  Angles between 00 and 3600
 a and r of a G.P. from Two given Terms  Simplifying and Evaluating Trigonometric
 Geometric Series and Geometric Means Expressions
 A.P. combined with G.P.  Inverse of Trigonometric Ratios
 Word Problems Involving G. P.  Domain and Range of the Inverse of
 The Sum to Infinity of a Geometric Series. Trigonometric functions
 Reccurence Relation  Application to Right Triangles
 Recuring Decimals as Geometric Series  Finding Trigonometric function value given
 Word Problems Involving Geometric series one trigonometric function
 The Sigma Notation (Σ)  The Pythagorean identities:
 Application of fundamental identities
17. Indices and Logarithms…………….....473 – 514  Compound Angles
 Positive Integral Indices  Addition and Subtraction Formulas for
 Laws of indices and its Applications Cosine Sine and Tangent
 Rational Indices and Negative Indices  Proof of the cosine formula
 Exponential Equations 1 and 2  Co-function Formulas
 Relating Indices and Logarithms  Addition and subtraction formulas for sine
 Laws of Logarithms and its Application  Addition and subtraction formula for Tangent
 Logarithmic Expressions  Multiple Angles, Half – Angle Identities and
 Simplifying / Evaluating Log Expressions formulas
 Operations on Logarithms  Product to Sums and Differences Formulas
 Involving Cubic Equations  Sums and Differences to Product Formulas
 Substitution  Trigonometric Equations 1 and 2
 Involving Change of Base  Maximum and Minimum values for simple
 Complex Logarithmic Equations 1 and 2 Trigonometric equations, Multiples and Sub-
 Involving Simultaneous Equations multiples
 Logarithmic Functions  Involving Quadratic Equations
 Word Problems Involving Logarithms  Domain and Range of Trigonometric
 Experimental Laws Function and Inverse Trigonometric Function.
 Application to Other Triangles
18. Trigonometry…………………………..515 – 572
 Sine and Cosine Rule, Area of a Triangle
 Radian Measure
 Trigonometry and Bearings
 Relationship Between Radians and Degrees
 Graphs of Trigonometric Functions
 Changing from Degrees to Radians
 Amplitude and a Period
 Changing from Radians to Degrees
 The expression Rcos (x − α)
 Changing Radians to Degrees, Minutes and
 Application to solving an equation
Seconds
 Finding maximum and minimum values of a
 Changing Minutes and Seconds as Decimal
trigonometric function
Degrees
 Length of a Circular Arc 19. Limits of a Function…………………...573 – 578
 Area of a Circular Sector  Limits of Functions at Infinity
 Reciprocal Identities  Rules for Limits of Functions at Infinity

Baffour Ba Series , Further Mathematics for Schools Page vii


 Rules for Limits of Functions at a Point  The General Solution
 Indeterminate Forms of Limit  The Equation of a Curve
 Evaluating Limits that Contain Surds  Value of the Constant of Integration
 Limit of Simple Trigonometry Functions  Definite Integral and Negative Integral
 Equations Involving Limits  Finding an Unknown Limit
 Integration by Substitution
20. Differentiation…………………………579 – 637  Change of Limits
 Differentiation from the first Principles  Integration of Simple Trigonometric Function
 Slope of a line and a Curve  Kinematics
 Differentiation by Rule  Motion in a Straight Line
 Derivative of a Constant  Motion under Constant Acceleration
 Power Rule for Differentiation  Area under a Curve
 Derivative of Identity Function  Area of a region bounded by the horizontal
 Constant Multiple Rule for Differentiation lines and the y – axis.
 Derivative of Sum or Difference  Volumes of Revolution
 Evaluating Derivatives  Rotation about the x-axis and y – axis
 Second Derivatives  The Trapezium Rule
 Product Rule, Quotient Rule and Chain Rule
 Product, Quotient and Chain Rules Combined 22. Permutation and Combination……….675 – 701
 Implicit Differentiation  Arrangements with and Without Restrictions
 Evaluating Implicit Functions at a Point  Application to Forming Digits
 Application of Differentiation  Forming Even, Odd and Multiples of
 Evaluating Gradient at a Point Numbers
 Equation of a Tangent to a curve at a Point on  Circular and Linear Arrangements;
the Curve.  Combination and the ( ) notation
 Equation of a Tangent at a Curve at the point
 Restricted Combination
where the Curve meets the line y = mx + c
 Choosing Objects from Two Different Groups
 Equation of the Tangent Parallel to the x or y
– axis
23. Probability II………………………..702 – 715
 Unknown Coefficients given the value of  The Binomial Probability Distribution
 Equation of Normal to a Curve  A Bernoulli trial
 Rates of Change 1 (Linear Kinematics)  Permutations and Combinations
 Displacement (Position), Velocity and  Rings and Roundabouts
Acceleration  Combination Probability Formula
 Rate of Change 2  Selection Without Replacement
 Small Change and Percentage Error
 Greatest and Least Values 24. Statistics………………………………..716 – 778
 Maxima and Minima  Types Sources and Collection of Data
 Stationary Points and their Nature  Measure of Central Tendencies
 Second Derivative test  Frequency Diagrams for Ungrouped Data
 Application of Maxima and Minima  Class Limits and Class Boundaries
 Curve Sketching  Class Mark and Class Size/Width
21. Integration ………..…………………..638 – 674  Format of a Grouped Frequency Table
 Integration of Sum or Difference  Mean and Median of a Grouped Data

Baffour Ba Series , Further Mathematics for Schools Page viii


 Histogram for Ungrouped and Grouped Data  A Mapping
 Mode and Median from a Histogram  Finding the Transformation given the Matrix
 Class Boundary, Class Mark/Mid-point of a  Translation and Properties of Translation
Histogram  Reflection and Properties
 Frequency Table for Histogram with Unequal  Rotation through an angle θ, anticlockwise
Interval about the origin:
 Measures of Positions  Anticlockwise and Clockwise Rotations
 Deciles and Percentiles through an Angle
 Cumulative Frequency Table and Ogive  Properties of Rotation
 Estimating the Quartiles from a Cumulative  Rotation about a Point
Frequency Curve  Enlargement and Properties of Enlargement
 Mean Deviation of a Raw and Grouped Data  Reflection in a Line through the Origin
 Measures of Dispersion  Inverse of a Linear Transformation
 The Range and Semi - Interquartile Range  Composition of Linear Transformation
 Variance and Properties of Variance
 Standard Deviation of a Raw / Grouped Data 27. Logic……………………………….825 – 836
 Properties of Standard Deviation  Statements and Notation of Statements
 The Assumed Mean  Negation of a Statement
 The Range and Interquartile Range from  Statements and Venn Diagrams
Frequency Distribution  Compound Statements
 Interquartile and Semi - inter quartile range  Implication amd Converse of an Implication
 Equivalent Statements
25. Matrices……………………………..779 – 809  The Chain Rule of Implication
 Describing a Matrix  Truth Tables
 A Square Matrix and a Zero Matrix  Valid Arguments
 Operations on Matrices
 Matrix Algebra 28. Correlation and Regression…………837 – 848
 The Unit (Identity) Matrix  Correlation and Regression Analysis
 Solving Problems Involving Identity  Univariate and Bivariate Distributions
 Matrix Equality  The Scatter Diagram
 Determinants and inverse of a 2 × 2 matrices  Forms of Correlation and Type of Scatter
 The Adjoint and Minors of a Matrix Diagrams
 Determinant of a 3 × 3 Matrix  The Line of Best Fit
 Equations Involving Determinants  Equation of the Line of Best Fit
 Singular Matrix  A line of regression of x on y, which gives an
 Transpose of a Matrix estimate of x, given a value of y.
 Inverse of a 3 × 3 Matrix  A line of regression of y on x, which gives an
 Word Problems Involving Matrices estimate of y, given a value of x.
 Matrix Equation  The Least Square Method
 Solving Simultaneous Equations  Spearman‟s Rank Correlation Coefficient
 Cramer‟s Rule
Answers to exercises………………….849 – 857
26. Matrices and Transformations…….…810 – 824
References………………………………858

Baffour Ba Series , Further Mathematics for Schools Page ix


1 SETS Baffour Ba Series

Meaning of Sets Statement Form


A set is a well – defined collection of distinct In this method, a well-defined description of
objects. Set are usually denoted by capital the elements of the set is given, listed and
letters such as A, B, C, D…etc enclosed in curly brackets. For example, the set
of even numbers less than 8 is written as: {even
Elements of a set are represented by small numbers less than 8}
letters such as a, b, c, d…etc.
2. Tabular or Roster Form.
If “a” is an element of set A, then we say “a In this method, the elements of the set are
belongs to A”, mathematically expressed as listed, separated by commas and enclosed in
“a A, where the symbol “ ” means“belongs curly brackets. For example, let N denote the
to”. set of first five natural numbers. Therefore,
N = {1, 2, 3, 4, 5}. Similarly, if M denotes the
If “b” is not an element of set A, then we say “b set of the months beginning with letter “J”, then
does not belongs to A” mathematically M = {January, June, July}
expressed as “b ∉ A” where the symbol “∉”
means “does not belong to”. 3. Rule or Set Builder Form
In this method, the elements of the set are
Examples of sets used in mathematics are: described by using the variable “x” or any other
N : the set of natural numbers . variable, followed by a colon. The symbol “:”
= {1, 2, 3, 4,…} or” “/” as used in this case is read “such that”.
Thereafter, the property possessed by the
Z : the set of integers . elements is written in braces (called “set of
= {…-3, -2, -1, 0, 1, 2, 3,…} all”). For example, P is the set of counting
Q : the set of rational numbers . numbers greater than 12. In set builder notation
or form, P is expressed as:
R : the set of real numbers .
P = {x : x is a counting number greater than
Z+ : the set of positive integers 12}. This is read as “P is the set of elements of
W: the set of all whole numbers. x such that x is a counting number greater than
12.
Representation of Sets
Sets are commonly represented by any of the Likewise, if A is the set of even numbers
following methods: between 6 and 14, then A is written in set
1. Statement form. builder notation as:
2. Tabular or roaster form. A = {x / x is even, 6 < x < 14} or
3. Rule or set builder form. A = {x : x P, 6 < x < 14, P is an even number}

Baffour Ba Series , Further Mathematics for Schools Page 1


Also, if K = {4, 5, 6, 7} then set builder form of 2. Unit Set or Singleton Set
K is: It is a set which contains only one element. For
K = {x : x is a natural number and 3 < x < 8} example,
R = {x : x is neither prime nor composite}
Exercises 1 .1 R = {1}
A. Write the following in roaster form. Other examples are:
1. The set of whole number less than 20 and S = {x : x is an even prime} = {2}, n(S) = 1
divisible by 3. T = {x : x N and x2 = 4} = {2}, n(T) = 1
2. The set of integers greater than -2 and less
than 4. 3. Finite Set
3. The set of integers between - 4 and 4. It is a set which contains a definite number of
4. The squares of the first four natural numbers. elements. A finite set can contain all its
5. Set of prime factors of 36. elements. For example,
6. The counting number between 15 and 35, A = {set of days of the week}
each of which is divisible by 6. B = {2, 3, 5, 7….19}
C = {x : x N and x < 7}
B. Write each in set builder form:
1. A = {16, 25, 36, 39, 64} 4. Infinite Set
2. B = {2, 4, 6, 8, 10} 3. C = {2, 3, 5, 7, 11} It is a set whose element cannot be listed. In
4. D = {1, 3, 5, 7, 9} 5. E = {a, e, i, o, u} other words it a set containing never – ending
elements. For example,
Types of Set A = {x : x N, x > 1}
1. Null Set or Empty Set B = {Set of all prime numbers}
It is a set which does not contain any element.
It is denoted by (read as phi). Sometimes, Cardinal Number of Set
is also denoted by { }. It can be concluded that The total number of elements in a set P, is call
the number of elements in an empty set is 0. An the cardinal number of P, denoted by n(P). For
empty set is therefore an example of a finite. example,
For example, the set of whole numbers less 1. A = {x : x N, x < 5}
than 0. Since there is no whole number less
than 0, the set is said to be a null set. A = {1, 2, 3, 4}
n(A) = 4
Likewise M = {x : x is a composite number
less than 4}. M is an empty set because there is 2. B = {set of elements in the word
no composite number less than 4. {“MATHEMATICS”}
B = {M, A, T, H, E, I, C, S} n(B) = 8
Note:
1. ≠ {0} Note that n ( ) = 0
2. {0} is a set which has one element 0

Baffour Ba Series , Further Mathematics for Schools Page 2


Pair of Sets Subsets of a Sets
1. Equivalent set and equal sets A set “M” is said to be the subset of another set
2. Joint set and disjoints sets “N” if all the elements of “M” can be found in
“N”.
Equivalent Set Subset is denoted by ⊂ or ⊃. For example, if N
Two sets are P and Q said to be equivalent if = {1, 2, 3, 4, 5, 6} and M = {1, 2, 3}, then M is
they have the same cardinal number. That is a subset of N, because all the elements of M can
n(P) = n(Q). The symbol “ or ⇔” is used. For be found in N. This is written as M ⊃N or
example, given that P = {m, e, n}and Q = {1, 2, N⊂M.
3}, n(P) = 3 and n(Q) = 3. Therefore, P Q.
Worked Examples
Equal Sets 1. If Y = {house, tree} and X = {cat, house, tree.
Two sets P and Q are said to be equal if they Find the relationship between Y and X.
contain the same elements. This means that
every element of P is an element of Q and Solution
every element of Q is an element of P. For Since all the elements in Y can be found in X,
example, given that P = {t, h, e} and Q = {e, t, we say Y is a subset of X, i.e. Y⊂X
h}, then A = B or set A and set B are equal sets.
2. If P = {2, 4, 8} and Q = {even counting
Note: numbers less than 12}. What is the relationship
1. Equal sets are always equivalent. between P and Q?
2. Equivalents set may not be equal.
Solution
Disjoint Set P = {2, 4, 8} and Q = {2, 4, 6, 8, 10}
Two sets A and B are said to be disjoint if they Since all the elements of P can be found in Q, P
do not have any element in common. For is a subset of Q. That is P ⊂ Q.
example:
A = {x : x is a prime} 3. P = {x : 20 < x < 30, where is odd} and Q =
B = {x : x is a composite} {23, 29}. Establish a relationship between sets
A and B do not have any element in common. P and Q.
That is A ∩ B = {}
Solution
Joint or Overlapping Set P = {21, 23, 25, 27, 29} and Q = {23, 29}
Two sets A and B are said to be overlapping if Since all the members of Q can be found in P,
they contain at least one element in common. we say Q is a subset of P. i.e. Q⊂P
For eg; A = {a, b, c, d} and B = {a, e, i, o, u}
A and B have the element {a} in common, that Power Set
is A∩B = {a} If A is a set, then the collection or family of all
subsets of A, is called power of A, denoted by

Baffour Ba Series , Further Mathematics for Schools Page 3


P(A). Thus, if A = {i, s}, then the power of A is 2. Let A ={ 0, 1, 2, 3, 4, 5 } B = {2, 4, 6, 8}
given by P(A) = {{i},{s}, {i, s}, } and C = {1, 3, 5, 7}. Verify that;
(A∪B)∪C = A ∪ (B∪C)
Note
1. The null set or empty set having no elements Solution
of its own is an element of the power set. A ={ 0, 1, 2, 3, 4, 5}, B = {2, 4, 6, 8} and
2. The set being a subset of itself is also an C = {1, 3, 5, 7}.
element of the power set. (A∪B) ∪C
⇒(A∪B) ={0, 1, 2, 3, 4, 5, 6, 8}
Worked Examples (A∪B) ∪C = {0, 1, 2, 3, 4, 5, 6, 7, 8}
1. Given that S = {1, 2, 3}, find P(S).
Now,
Solution A∪ (B∪C);
S = {1, 2, 3},
(B∪C) = {1, 2, 3, 4, 5, 6, 7, 8}
P(S) = {{1}, {2}, {1, 2}, {2, 3}, {1, 3} S}
A∪ (B∪C) = {0, 1, 2, 3, 4, 5, 6, 7, 8}
2. If S = {a}, find P(S) ⇒(A∪B) ∪C = A∪ (B∪C)
= {0, 1, 2, 3, 4, 5, 6, 7, 8}
Solution
If S = {a}, then P(S) = {{a}, } 3. Let X = {1, 2, 3, 4}, Y = {2, 3, 5} and
Z = {4, 5, 6}.Verify that:
Union of sets i. X∪Y = Y∪X
The union of two or more sets is a set which ii. (X∪Y) ∪Z = X∪(Y∪Z)
contains a combination of all the elements in
those sets. It is denoted by “∪” Solution
i. X = {1, 2, 3, 4}, Y = {2, 3, 5}, Z = {4, 5, 6}.
Worked Examples X∪Y = {1, 2, 3, 4, 5}
1. Let A = {x : x is a natural number and a Y∪X = {1, 2, 3, 4, 5}
factor of 18} and B = {x : x is a natural number X∪Y = Y∪X = {1, 2, 3, 4, 5}
less than 6}. Find A∪B
ii. (X∪Y)∪Z = {1, 2, 3, 4, 5, 6}
Solution X∪(Y∪Z) = {1, 2, 3, 4, 5, 6}
A = {x : x is a natural number and a factor of (X∪Y)∪Z = X∪(Y∪Z) = {1, 2, 3, 4, 5, 6}
18}
A = {1, 2, 3, 6, 9, 18} Intersection of sets
B = {x : x is a natural number less than 6}. The intersection of two or more sets is the set
B = {1, 2, 3, 4, 5} which contains elements which are common to
A∪B = {1, 2, 3, 4, 5, 6, 9, 18} those sets. It is denoted by “∩”

Baffour Ba Series , Further Mathematics for Schools Page 4


Worked Examples Find the following sets:
1. Let A = {x : x is a natural number and a i. A∪B ii. A∩C
factor of 18} and B = {x : x is a natural number iii. B∩C iv. (A∪B) ∩ B
less than 6}. Find A∩B. v. (A∩C) ∪ (B∩C) vi. (A∪B)1
vii. A1∩B1
Solution
A = {x : x is a natural number and a factor of Solution
18} = {1, 2, 3, 6, 9, 18} A = {2, 3, 5, 7, 11}
B = {x : x is a natural number less than 6}. B = {2, 4, 6, 8, 10, 12}
B = {1, 2, 3, 4, 5} C = {3, 6, 9, 12}
A ∩ = {1, 2, 3}
i. A∪B = {2, 3, 4, 5, 6, 78, 10, 11, 12}
2. Given that P = {multiples of 3 between 1 and ii. A∩C = {3}
20} and Q = {even natural numbers up to 15}. iii. B∩C = {6, 12}
Find the intersection of set P and set Q. iv. (A∪B) ∩ B = {2, 3, 4, 5, 6, 78, 10, 11,
12}∩ {3, 6, 9, 12}
Solution = {3, 6, 12}
P = {multiples of 3 between 1 and 20} v. (A∩C) ∪ (B∩C) = {3} ∪ {6, 12}
P = {3, 6, 9, 12, 15, 18} = {3, 6, 12}
Q = even natural numbers up to 15} vi. (A∪B) = {2, 3, 4, 5, 6, 78, 10, 11, 12}1
1

Q = {2, 4, 6, 8, 10, 12, 14} = {1, 9}


1 1
P ∩ Q = {6, 12} vii. A ∩B = {1, 4, 6, 8, 9, 10, 12}∩{1, 3, 5,
7, 9, 11}
3. Let A = {0, 1, 2, 3, 4, 5}, B = {2, 4, 6, 8 and = {1, 9}
C = {1, 3, 5, 7}. Verify that:
(A∩B)∩C = A ∩ (B∩C) Exercises 1.2
A. Find the intersection of the sets:
Solution 1. A = {x: n N, x = 3n, n < 3}
A = {0, 1, 2, 3, 4, 5} B = {2, 4, 6, 8} and B = {x : x N x < 7}
C = {1, 3, 5, 7}.
(A∩B)∩C = {2, 4} ∩ {1, 3, 5, 7} = { } 2. P = {x : x = n2 when n N}
A ∩ (B∩C) = {0, 1, 2, 3, 4, 5}∩{ } = { } Q = {x : x = 4n, when n W, n < 5}
(A∩B)∩C = A∩(B∩C) = { }
B. Find the union of the following sets.
4. Let ∪ = {1, 2, 3…12} 1. A = {x: n N, x = 2n, n < 4}
A = {x ∪ : x is a prime number} B = {x : x is an even number less than 10}
B = {x ∪ : x is an even number}
2. P = {x : x is a natural number and multiple of 3}
C = {x ∪ : x is divisible by 3}.
Q = {x : x is a prime number less than 19}

Baffour Ba Series , Further Mathematics for Schools Page 5


3. M = {x : x N, n < 7} i. the difference between sets P and Q
N = {x : x Z, -2 ≤ x ≤ 3} is an even number ii. Q – R iii. R – P iv. Q – P
less than 10}
Solution
4. X = {x : x is an integer – 3 ≤ x ≤ -3} P = {x : x is a natural number between 10 and 16},
Y = {x : x is a factor of 8} P = {11, 12, 13, 14, 15}
Q = {x : x is an even number between 8 and 20}
C. Let A = {x ∪ : x ≤ 10}, B = {x ∪ : x is a Q = {10, 12, 14, 16, 18}
prime number}, C = {x ∪ : x is an even R = {7, 9, 11, 14, 18, 20}
number}, where ∪ = {1, 2, 3…..19} is the i. P – Q = {11,13, 15}
universal set. Find the following: ii. Q – R = {{10, 12, 16}
i. A∪B∪ C ii. A∩B∩C iii. R – P = {7, 9, 18, 20}
iii. A∪B) ∩ C iv. A∩B)∪C iv. Q – P = {10, 16, 18}
1
v. A∩B vi. A1∩B1
Complement of a Set
Difference of Two Sets If is a universal set and A is a subset of ,
If P and Q are two sets, then their difference is then the complement of A is the set of all
given by P – Q or Q – P. elements of , which are not the elements of A.
P – Q means elements of P which are not
elements of Q. For example, if P = {2, 3, 4} Mathematically, the complement of a set A is
and Q = {4, 5, 6}, then P – Q = {2, 3} denoted as .

Generally, if P and Q are disjoint sets, then Note:


P – Q = P and Q – P = Q 1. The complement of a universal set is an
empty set.
Worked Examples 2. The complement of an empty set is a
1. If A = {a, b, c, d, e, f} and B = {b, d, f, g} universal set.
Find the difference between the sets: 3. The set and its complement are disjoint.
i. A and B ii. B and A
Proofs
Solution 1. The complement of a universal is an empty
A = {a, b, c, d, e, f} and B = {b, d, f, g} set.
i. A – B = {a, c, e} Let represent the universal set.
ii. B – A = {g} Then = the set of elements not in
= (empty set)
2. Given the sets P, Q and R such that P ={x : x
is a natural number between 10 and 16}, 2. A set and its complement are disjoint.
Q = {x : x is an even number between 8 and 20} Let A be any set. Then A1 is the set of element
and R = {7, 9, 11, 14, 18, 20}. Find: of which are not in A1

Baffour Ba Series , Further Mathematics for Schools Page 6


Let x∉A, then x is an element of not 1. (A∪B∪C)1 = A1 ∩ B1∩ C1
contained in A , so x ∉ A . Therefore A and A1
1 1
2. (A∩B∩C)1 = A1 ∪ B1∪C1
are disjoint sets
Illustration
Worked Examples Let = {natural numbers which are multiples
1. If = {1, 2, 3, 4, 5, 6, 7} and A = {1, 3, 7}. of 3 less than or equal to 20};
Find A = {even numbers} and B = {prime numbers},
where A and B are subsets of ∪.
Solution
= {1, 2, 3, 4, 5, 6, 7} By listing the elements of ∪, A and B,
A = {1, 3, 7}. ∪ = {3, 6, 9, 12, 15, 18}
= {2, 4, 5, 6}. A = {6, 12, 18}
B = {3}
2. Let = {Natural numbers} and A = {even
natural numbers}. Find Verifying law 1;
A∪B = {3, 6, 12, 18}
Solution (A∪B)1 = {9, 15}
= {Natural numbers}
= {1, 2, 3, 4, 5, 6, 7, 8, 9, 10 …} A1 = {3, 9, 15}
A = {even natural numbers} B1 = {6, 9, 12, 15, 18}
A = {2, 4, 6, 8, 10} A1 ∩ B1 = {9, 15}
= {1, 3, 5, 7, 9…} ⇒(A∪B)1 = A1 ∩ B1 = {9, 15}
= {x : x is a set of odd natural numbers}
Verifying law 2;
De Morgan‟s Law A∩B = { }
De Morgan‟s law states that: (A∩B)1 = {3, 6, 9, 12, 15, 18}
1. the complement of the union of two sets A A1 = {3, 9, 15}
and B is equal to the intersection of the B1 = {6, 9, 12, 15, 18}
complements of the sets A and B. A1∪B1 = {3, 6, 9, 12, 15, 18}
(A∪B)1 = A1 ∩ B1 ⇒ (A∩B)1 = A1 ∪ B1 = {3, 6, 9, 12, 15, 18}

2. the complement of the intersection of two Algebra of Sets


sets A and B is equal to the union of the 1. Commutative Laws
complements of the sets A and B. For any two finites sets A and B;
(A∩B)1 = A1 ∪ B1 i. A∪B = B∪A
ii. A∩B = B∩A
Note:
This De Morgan‟s law is applicable to three Consider the sets, A = {1, 3, 5, 15} and
sets A, B and C, such that: B = {1, 2, 4, 5, 10, 20}

Baffour Ba Series , Further Mathematics for Schools Page 7


A ∪ B ={1, 2, 3, 4, 5, 10, 15, 20} 3. Idempotent Laws
B ∪ A = {1, 2, 3, 4, 5, 10, 15, 20} For any finite set A;
A ∪ B = B ∪ A. i. A∪A = A
Hence, the union of sets is commutative ii. A∩A = A
Consider the set, P = {odd factors of 18}
Similarly, A∩B = {1, 5}, P = {1, 3, 9}
B ∩ A = {1, 5} Now, P∩P = {1, 3, 9}= P
A∩B=B∩A P∪P = {1, 3, 9}= P
Hence, the intersection of sets is commutative
4. Distributive Laws
2. Associative Laws For any three finite sets A, B and C;
For any three finite sets A, B and C; i. A∪(B∩C) = (A∪B) ∩ (A∪C)
i. (A∪B)∪C = A∪(B∪C) ii. A∩(B∪C) = (A∩B) ∪ (A∩C)
ii. (A∩B)∩C = A∩(B∩C) Union and intersection are distributive over
Union and intersection are associative. intersection and union respectively.

Consider the sets A = {1, 5, 6, 7}, B = {2, 5, 7, Consider the sets, A = {1, 2, 5, 8, 9}, B = {2,
6} and C = {1, 7, 8, 10} 5, 8, 9, 10} and C = {5, 9, 12, 13}
(A∪B)∪C = {1, 2, 5, 6, 7}∪{1, 7, 8, 10} A ∪ (B ∩ C) = {1, 2, 5, 8, 9} ∪ {5, 9}
= {1, 2, 5, 6, 7, 8, 10} = {1, 2, 5, 8, 9}
A∪ (B∪C) = {1, 5, 6,7}∪{1, 2, 6, 7, 8,10}
= {1, 2, 5, 6, 7, 8, 10} (A ∪ B) ∩ (A ∪ C)
(A∪B)∪C = A∪ (B∪C) = {1, 2, 5, 8, 9, 10} ∩{1, 2, 5, 8, 9, 12, 13}
= {1, 2, 5, 8, 9}
Hence for any three sets A, B and C,
(A∪B)∪C = A∪(B∪C). A ∪ (B ∩ C) = (A ∪ B) ∩ (A ∪ C)
The operation of union of sets is associative. Hence for any three sets A, B and C,
A ∪ (B ∩ C) = (A ∪ B) ∩ (A ∪ C).
Similarly, (A∩B)∩C = {5, 6, 7} ∩{1,7, 8, 10} The union is said to be distributive over
= {7} intersection.

A ∩ (B ∩ C) = {1, 5, 6, 7} ∩ {7} In a similar manner,


= {7} A ∩ (B ∪ C)
(A ∩ B) ∩ C = A ∩ (B ∩ C) = {1, 2, 5, 8, 9}∩{2, 5, 8, 9, 12, 13}
= { 2, 5, 8, 9}
Hence, for any three sets A, B and C,
(A ∩ B) ∩ C = A ∩ (B ∩ C). The operation of (A ∩ B) ∪ (A ∩ C)
intersection of sets is associative. = {2, 5, 8, 9} ∪ {5, 9}
= {2, 5, 8, 9}

Baffour Ba Series , Further Mathematics for Schools Page 8


A ∩ (B ∪ C) = (A ∩ B) ∪ (A ∩ C) = {1, 2, 3, 5, 10, 15}

Hence for any three sets A, B and C, A∪B = {1, 3, 5, 15}∪ { 1, 2, 5, 10}
A ∩ (B ∪ C) = (A ∩ B) ∪ (A ∩ C). The = {1, 2, 3, 5, 10, 15}
intersection is said to be distributive over the
union. (A – B)∪B = A∪B = {1, 2, 3, 5, 10, 15}

5. For any two finite set A and B; iv. (A – B) ∩B = { }


i. A – B = A∩B1 {3, 15}∩ { 1, 2, 5, 10} = { }
ii. B – A = B∩A1
iii. A – B = A ⇔A∩B = { } v. (A – B) ∪ (B – A) = (A∪B) – (A∩B)
iv. (A – B)∪B = A∪B (A – B)∪(B – A) = {3, 15} ∪ {2, 10}
v. (A – B) ∩B = { } = {2, 3, 10, 15}
vi. (A – B)∪(B – A) = (A∪B) – (A∩B)
vii. A⊆B ⇔ B1⊆A1 (A∪B) = {1, 3, 5, 15}∪{ 1, 2, 5, 10}
= {1, 2, 3, 5, 10, 15}
Consider the sets; (A∩B) = {1, 3, 5, 15}∩{ 1, 2, 5, 10}
A = {1, 3, 5, 15}, B = {1, 2, 5, 10} and C ={} = {1, 5}
i. A – B = A∩B1
A – B = {1, 3, 5, 15} - { 1, 2, 5, 10} = {3, 15} (A∪B) – (A∩B) = {1, 2, 3, 5, 10, 15} - {1, 5}
A∩B1 = {1, 3, 5, 15} ∩ {3, 15} = {2, 3, 10, 15}
= {3, 15}
(A – B) ∪ (B – A) = (A∪B) – (A∩B) = {2, 3, 10, 15}
A – B = A∩B1 = {3, 5}

ii. B – A = B∩A1 vii. Consider the sets;


B – A = { 1, 2, 5, 10} - {1, 3, 5, 15} = {2, 10} A = { factors of 5}and B = { factors of 15}
B∩A1 = {1, 2, 5, 10} ∩ {2, 10} A = {1, 5}
= {2, 10} B = { 1, 3, 5, 15}
B – A = B∩A1 = {2, 10} Since all the elements of A is contained in B,
A⊆B is a true statement.
iii. Considering sets A and C;
A – C = A ⇔A∩C = { } Now B1 = {} and A1 = {3, 15}. Since B1 is a
A – C = {1, 3, 5, 15} – { } = {1, 3, 5, 15} subset of A1, then B1⊆A1
A–C=A
It follows that if A⊆B ⇔ B1⊆A1
A∩C = {1, 3, 5, 15} ∩ { } = { }
A – C = A ⇔A∩ C = { } 6. For any three sets A, B and C;
iv. (A – B)∪B = A∪B i. A – (B∩C) = (A – B) ∪ (A – C)
(A – B)∪B = {3, 15}∪ { 1, 2, 5, 10} ii. A – (B∪C) = (A – B) ∩ (A – C)

Baffour Ba Series , Further Mathematics for Schools Page 9


iii. A∩ (B – C) = (A∩B) – (A∩C) A – (B∪C) = (A – B) ∩ (A – C) = {2, 36}
iv. A∩ (B ⋄ C) = (A∩B) ⋄ (A∩C)
iii. A∩ (B –C) = (A ∩B) – (A∩C)
Consider the sets; A∩ (B – C) = {2, 4, 6, 12, 18, 36}∩ {4, 8, 12, 24}
A = {even factors of 36} = {4, 12}
B = {composite factors of 24}
C = {factors of 18 that are multiples of 3} (A ∩B) – (A∩C) = {4, 6, 12} – {6, 18}
= {4, 12}
A = {2, 4, 6, 12, 18, 36} A∩ (B – C) = (A ∩B) – (A∩C) = {4, 12}
B = {4, 6, 8, 12, 24}
C = {3, 6, 9, 18} iv. A∩ (B ⋄ C) = (A ∩B) ⋄ (A∩C), where ⋄ could
mean +, – , ∪ or ∩. This is already proven as in
i. A – (B∩C) = (A – B) ∪ (A – C) iii.
A – (B∩C) = {4, 6, 12, 18, 36} – {6}
= {2, 4, 12,18, 36} Properties of Complements of a Set
1. Complement laws
(A – B) ∪ (A – C) The union of a set A and its complement A1
A – B = {2, 4, 6, 12, 18, 36} – {4, 6, 8, 12, 24} gives the universal set U, of which A and A1 are
= {2, 18 , 36} subsets of U. That is A∪A1 = U
A – C = {2, 4, 6, 12, 18, 36} – {3, 6, 9, 18}
= {2, 4, 12, 36} Also the intersection of a set A and its
complement A1 gives an empty set. That is
(A – B) ∪ (A – C) = {2, 18, 36}∪{2, 4, 12, 36} A∩A1 = { }
= {2, 4, 12, 18, 36}
A – (B∩C) = (A – B) ∪ (A – C) 2. Double complementation law
{2, 4, 12, 18, 36} = {2, 4, 12, 18, 36} The complement of a complemented set A,
written as (A1)1, gives the set A.
ii. A – (B∪C) = (A – B) ∩ (A – C)
A – (B∪C) 3. Law of empty set and universal sets
= {2, 4, 6, 12, 18, 36} – {3, 4, 6, 8, 9, 12 , 18, 24} The complement of the universal set gives an
= {2, 36} empty set and the complement of the empty set
gives the universal set. That is ∪1 = and 1
=∪
(A – B) ∩ (A – C)
A – B = {2, 4, 6, 12, 18, 36} – {4, 6, 8, 12, 24} Worked Examples
= { 2, 18 , 36} Let D(n) denotes the factors of a natural
A – C = {2, 4, 6, 12, 18, 36} –{3, 6, 9, 18} number n, including n itself but 1.
= {2, 4, 12, 36} e.g. D(8) = {2, 4, 8}.
(A – B) ∩ (A – C) = {2, 36} i. List the elements of D(12) and D(18)
ii. Find the integer, r such that;

Baffour Ba Series , Further Mathematics for Schools Page 10


D(12) ∩ D(18) = D(r) 6. What can you say about (A∪B)1 = A1∩B1

iii. If P = D(8) ∪ [D(12) ∩ D(18)] and B. Find the complement of the following sets
Q = [D(8) ∪ D(12)]∩,D(8)∪D(18)] if the universal set is the set of natural
numbers;
Solution 1. {x : x is a prime number}
i. D(12) ={2, 3, 4, 6, 12} 2. {x : x is a multiple of 2}
D(18) = {2, 3, 6, 9, 18} 3. {x : x ≥ 100}
4. {x : x N, 5x + 1 > 20}
ii. D(12) ∩ D(18) = D(r)
{2, 3, 4, 6, 12} ∩ {2, 3, 6, 9, 18} = D(r) C. Let A, B and C be the subset of the universal
{2, 3, 6} = D(r) set U. For each of the following statements,
But D(6) ={2, 3, 6} determine whether it is true or not.
⇒ D(r) = D(6) = {2, 3, 6} i. A – B = A1∩B
r=6 ii. (A∪B) ∩ C = A∪(B∩C)
iii. (A1∪B1) ∩ B = B – A
iii. P = D(8) ∪ [D(12) ∩ D(18)]
P = {2, 4, 8} ∪ {2, 3, 6} Challenge Problems
P = {2, 3, 4, 6, 8} The universal set is the set of integers p such
that 10 ≤ p ≤ 100.
Q = [D(8) ∪ D(12)]∩[D(8)∪D(18)] A = {p : p divided by 12 leaves a remainder 5}.
D(8) ∪ D(12) = {2, 4, 8}∪{2, 3, 4, 6, 12} B = {p : p divided by 16 leaves a remainder 5}.
= {2, 3, 4, 6, 8, 12} Four positive integers a, b, c and d are such
D(8) ∪ D(18) = {2, 4, 8} ∪{2, 3, 6, 9, 18} that:
= {2, 3, 4, 6, 8, 9, 18} a = 12c + 5
Q = {2, 3, 4, 6, 8, 12}∩ {2, 3, 4, 6, 8, 9, 18} b = 16d + 5
Q = {2, 3, 4, 6, 8} a + b = 78
Find a and b.
P = Q = {2, 3, 4, 6, 8}
Illustration of Diagrams
Exercises 1.3 A. Joint Sets
A. Let U = {1, 2, 3, 4, 5, 6, 7, 8, 9, 10}; U
A B
A = {1, 2, 4, 6, 8, 10};
B = {1, 3, 5, 7, 8, 9}.
Find:
1. B1 2. A1
3. A1∪ B1 4. A1 ∩ B1 The shaded regions represent A∪B.
5. (A∪ B)1 The non-shaded region represents (A∪B) 1

Baffour Ba Series , Further Mathematics for Schools Page 11


2. A B U 2. U
A B

The shaded portion represents: The shaded portion represents:


A ∩B1 = A only A∩B1 = A only

3. 3. A B U
A B U

The shaded portion represents:


The shaded portion represents A∩B and the A1∩B = B only
non-shaded portions represent (A∩B)1
4. A B U
4. A B U

The shaded portion represents:


The shaded portion represents: A1∩B1 = (A∪B)1. That is outside A or B or
A1∩B = B only both.

5. U
C. Subsets (B ⊂ A)
A B
1. U
A
B

The shaded portion represents:


A1∩B1 = (AUB)1. That is outside A or B or
both. The shaded regions represent A∪B
The non-shaded region represents (A∪B) 1
B. Disjoint sets
1. U 2. U
A B A
B

The shaded regions represent A∪B The shaded portion represents:


The non-shaded region represents (A∪B) 1 A∩B1 = A only

Baffour Ba Series , Further Mathematics for Schools Page 12


3. Solution
A U
n(A∪B) = n(A – B ) + n(A∩B) + n(B – A)
B
70 = 18 + 25 + n(B – A)
70 = 43 + n(B – A)
n(B – A) = 70 – 43
n(B – A) = 27
The shaded portion represents A∩B and B Now,
only. The non-shaded portions represent n(B) = n(A∩B) + n(B – A)
(A∩B)1 = 25 + 27
U = 52
4. A
B
B
3. In a group of 60 people, 27 like cold drinks
and 42 like hot drinks and each person likes at
least one of the two drinks. How many people
like both drinks?
The shaded portion represents:
A1∩B1 = (AUB)1. That is outside A or B or
Solution
both
Let A represent people who like cold drink
and B represent people who like hot drink
D. Single Sets
n(A∪B) = 60, n(A) = 27, n(B) = 42
1. U
A n(A∪B) = n(A) + n(B) – n(A∩B)
= 27 + 42 – 60
= 69 – 60 = 9
Therefore, 9 people like both drinks.
The shaded region is AI.
4. There are 35 students in Art class and 57
The non-shaded region is A.
students in Business class. Find the number of
students who are in Art or Business class;
Word Problems
i. when two classes meet at different hours and
1. Let A and B be two finite sets such that n(A)
12 students are enrolled in both activities.
= 20, n(B) = 28 and n(AUB) = 36. Find n(A∩B).
ii. when two classes meet at the same hour.
Solution
Solution
n(AUB) = n(A) + n(B) – n(A∩B)
i. Let A represent students in Arts class and B
n(A∩B) = n(A) + n(B) – n(AUB)
represent students in Business class.
= 20 + 28 – 36 = 12
When two classes meet at different hours and
12 students are enrolled in both activities.
2. If n(A – B) =18, n(A∪ B) = 70 and n(A∩B) =
n(A) = 35, n(B) = 57, n(A∩B) = 12
25, then find n(B).
⇒ n(AUB) = n(A) + n(B) – n(A∩B)

Baffour Ba Series , Further Mathematics for Schools Page 13


= 35 + 57 – 12 = 80 iii. The number of people speaking French
only;
ii. When two classes meet at the same hour n(B – A) = n(B) – n(A∩B)
A∩B = = 43 – 15 = 28 people
n (AUB) = n(A) + n(B) – n(A∩B) E = 50
= n(A) + n(B) A B
= 35 + 57 6. In the diagram below,
= 92 /A/ = 15, /B/ = 25,
/A∩B/ = 5 and /E/ = 50
5. In a group of 100 people, 72 can speak a. Insert the number of elements to each of the
English and 43 can speak French. four regions.
i. How many can speak English only? b. Hence find /A∪B/ and /A∩B1/
ii. How many can speak English only?
E = 50
ii. How many can speak both English and Solution (15)
A B(25)
French? a. A∩B = 5
A only = 10 10 5 20
Solution B only = 20 15
A = {persons who speak English}
B = {persons who speak French} E = A only + B only + A∩B + (A∪ B)1
A – B = {persons who speak English and not 50 = 10 + 5 + 20 + (A∪ B)1
French} (A∪B)1 = 50 – 10 – 5 – 20 = 15
B – A = {persons who speak French and not
English} b. From the diagram;
A∩B ={persons who speak both English and A∪B = 10 + 5 + 20 = 35
French}
Alternatively;
n(A) = 72, n(B) = 43 , n(AUB) = 100 A∪B = A + B – (A∩B)
n(A∩B) = n(A) + n(B) – n(AUB) = 15 + 25 – 5 = 35
= 72 + 43 – 100
= 15 people A∩B1 = 10
The umber of people who speak both English 7. A and B are two sets and the numbers of
and French is 15. elements are shown in the diagram below;

A B
ii. The number of people speaking English
only; 14 + x x 3x
n(A) = n(A – B ) + n(A∩B)
⇒ n(A – B) = n(A) – n(A∩B)
= 72 – 15 Given that n (A) = n (B), calculate:
= 57 people i. x ii. n(A∪ B)

Baffour Ba Series , Further Mathematics for Schools Page 14


Solution a. How many cars needed both repairs?
i. 14 + x + x = x + 3x b. How many cars needed new brakes but
14 + 2x = 4x not exhaust system?
14 = 4x – 2x
14 = 2x 4. Twenty – four people go on holidays. If 15
x= =7 go swimming 12 go fishing and 6 do neither;
a. How many go swimming and fishing?
ii. n ( ∪ ) b. Draw a Venn diagram and fill in the number
= n(A) only + n(B) only + n(A ∩ B ) of people in all four regions.
But n(A) only = 14 + x
5. A travel agent surveyed 100 people to find
= 14 + 7 = 21
out how many of them had visited the cities of
n( ) only = 3x = 3(7) = 21 Accra and Kumasi. 31 people had visited
n(A ∩ B ) = 7 Accra, 26 people had been to Kumasi, and 12
people had visited both cities. Draw a Venn
⇒n(A∪ B) = 21 + 21 + 7 = 49 diagram to find the number of people who had
visited;
Exercises 1.4 a. Accra or Kumasi.
1. M = {x : x is an integer and 2 ≤ x ≤ 20} b. Kumasi but not Accra.
N = {Prime numbers less than 30} c. Only one of the two cities.
a. Draw a Venn diagram and illustrate the d. neither city.
information above.
b. Copy and complete the statement: 6. Draw a Venn diagram of two sets and .
i. M∩N = {…} ii. (M∩N)1 = {…} Given that / / = 15, / / = 20, / ∪ / = 25 and
/ / = 50;
2. In a group of 100 customers at a shopping a. Insert the number of elements to each of the
mall, 80 of them ordered Milo and 72 of them four regions.
ordered Nido. 60 customers ordered both Milo b. Hence find / ∩ / and / ∪Q1/.
and nido;
a. How many customers ordered Milo but not 7. Let the universal set be E = {whole numbers
Nido? less than 20} and let A = {squares less than 20},
b. How many customers ordered Nido but not B = {even number less than 20} and C = {odd
Milo? squares less than 20}
c. How many customers ordered neither of the a. Draw A and C on a Venn diagram, and place
two? the numbers in the correct regions.
b. Draw B and C on a Venn diagram, and place
3. At a certain automotive shop, 50 cars were the numbers in the correct regions.
inspected. 23 of them needed brakes, 34 of c. Shade A∩B on a Venn diagram, and place the
them needed exhaust system? numbers in the correct region.

Baffour Ba Series , Further Mathematics for Schools Page 15


d. Shade ∪ on a Venn diagram, and place C. If P = { p : p is prime and 3 ≤ p ≤ 31} and
the numbers in the correct region. A = {a :a is an odd number and 3 ≤ a ≤ 35},
where p and a are integers, which of the
8. In a class of 35 boys, is the set of boys following are true and which are false?
who take athletics and is the set who play i. A⊂P ii. P⊂A
cricket. n (A) = 15, n( ) = 16, n( ∩ ) = 5.
iii. 25 P iv. 25 A
Using the whole class as the Universal set,
draw a Venn diagram and mark the numbers in v. ∩A = P, vi. P∪A = A
their appropriate regions. vii. 17 (A∩ ) viii. 27 (P∪A)
ii. How many boys take neither athletics nor ix. If C is the set of all even numbers what can
cricket? you say about A∩C?
Sketch Venn diagrams to illustrate (vii) and
9. and are two sets and the numbers of (viii)
elements are shown in the diagram below;
Three Set Problems
P Q Three set problem arises when a number of
10 + 2x x 5x –8 people are made to choose between three items.
The choice could be for one item only, two
items only, all the three items or none of the
Given that n (P) = n ( ), calculate: three items.
i. n(P ∩Q) ii. n( ) iii. n(P∪ )
Diagrams for Three Sets
10. In an examination, x pupils take the history A. Shading One Region
paper and 3x pupils take the mathematics paper. 1.
Given that 6 pupils take both papers, illustrate U
A B
the data on a Venn diagram indicating the
number of pupils in each region. If the number C
of pupils taking the examination is 46, find x.
The shaded region is A∩B∩C
B. Use true or false for the following;
1. If two sets are equal, they are also 2.
equivalent. U
A B
2. 4 {x : 4 < x ≤ 10
3. An empty set is a finite set C
4. If P = {x : x = 2n, n N} and Q = {x : x = 2n
+ 1, n N}, then P and Q are disjoint sets
The shaded region represents A only
5. A = {x : x is a factor of 24} and B = {x : x is
= A∩B1∩C1 = A∩ (B∩C)1
a multiple of three less than 30}, then P and Q
are overlapping sets

Baffour Ba Series , Further Mathematics for Schools Page 16


3. U C. Shading Three Regions
A B 1. U
A B
C
C

The shaded region represents B only


The shaded region represents A∩ (BUC)
= A1∩C1∩B = (AUC)1∩B

4. 2. U
U B
A B A

C
C

The shaded region represents C only The shaded region represents B∩ (A∪C)
= A1∩B1∩C = (AUB)1∩C
3. U
B. Shading Two Regions A B
1. U
A B C

C
The shaded region represents C∩ (A∪B)

The shaded region represents A∩B 4. U


2. A B
U
A B
C
C

The shaded region represents (A∪B) ∩C1


The shaded region represents B∩C

5. U
3. U A B
A B

C C

The shaded region represents A∩C The shaded region represents (B∪C) ∩A1

Baffour Ba Series , Further Mathematics for Schools Page 17


6. The shaded regions represent exactly two items
U
A B
8.
U
C B
A

The shaded region represents (AUC) ∩B1 C

7. U
A B The shaded regions represent exactly one item

Solving Problems Involving Three Overlapping Sets

U U
B B
A A

V II VI A only (A∩ B) - x B only

I x
III IV (A∩C) - x (B∩C) - x

VII VIII C only


C 1
C
(AUBUC)

Fig. I Fig. II

For three sets A, B, and C,


1. a. n(AUBUC) = ( I + II + III + IV + V + VI + VII )
= A only + B only + C only + (A∩B) only + (B∩C) only + (A∩C) only + (A∩B∩C), if the values of
exactly two items only and exactly one item are given
b. n(ABUC) = n(A) + n(B) + n(C) – n(A∩ B) – n(B∩C) – n(A∩C) + n(A∩B∩C), if each person like
all the three items (without complement)

Baffour Ba Series , Further Mathematics for Schools Page 18


c. n(AUBUC) = ( I + II + III + IV + V + VI + VII + VIII)
= n(A) + n(B) + n(C) – n(A∩B) – n(B∩C) – n(A∩C) + n(A∩B∩C) + n(AUBUC)1, if some people do
not like all the three items (with complement)
2. Number of people in exactly one set: = (V + VI + VII)
= n(A) + n(B) + n(C) – 2n(A∩B) – 2n(A∩C) – 2n(B∩C) + 3n(A∩B∩C)
3. Number of people in exactly two of the sets = (II + III + IV)
= n(A∩B) + n(A∩C) + n(B∩C) – 3n(A∩B∩C)
4. Number of people in exactly three of the sets = n(A∩B∩C) = I
5. n(A∩B) only = (A∩B)∩C1 = II
6. n(A∩C) only = (A∩C) ∩B1 = III
7. n(B∩C) only = (B∩C) ∩ A1 = IV
8. Number of people in set A only = A ∩ (B1∩C1) = V
9. Number of people in set B only = B ∩ (A1∩C1) = VI
10. Number of people in set C only = C ∩ (A1∩B1) = VII
11. Number of people who do not like any of the three = n(A∪B∪C)1 = VIII
12. Number of people in two or more sets = (II + III + IV + I) = (at least 2 sets) :
= n(A∩B) + n(A∩C) + n(B∩C) – 2n(A∩B∩C)
13. n(A) = V+ I + II + III
14. n (B) = I + II + IV + VI
15. n(C) = I + III + IV + VII
16. n(A∩B) = I + II
17. n(A∩C) = I + III
18. n(B∩C) = I + IV

Note: Questions must be carefully read in order to place values at their respective regions in the
diagram. Any region without a given value must be represented by a preferred variable.

Baffour Ba Series , Further Mathematics for Schools Page 19


Type 1 v. n(Art or Chemistry or both)
It involves the situation whereby the values of = 3 + 7 + 20 + 5 + 6 + 12 = 53
all the regions of the Venn diagram is given to
answer some related questions. vi. n(Art or Chemistry (but not both)
= 3 + 7 + 6 + 12 = 28
Worked Examples
1. The number of students at Asaaman S.H.S vii. Neither Arts nor Biology nor Chemistry
1A, offering the various combinations of Arts, = 10
Biology and Chemistry is shown on the
diagram below; viii. Number of students in the class
= 7 + 3 + 5 + 20 + 2 + 6 + 12 + 10
U= 65
= 65
A = 35 C = 43

7 5 12 2. Use the diagram below to answer the


20 6 questions that follows.
3 U (102)
A B
2
10 B = 31 7 14 12
15
9 14 6
How many students study: 20
i. Art and Biology but not Chemistry 19 C
ii. Biology but not Art
iii. Chemistry but neither Art nor Biology
1. How many students do not like either A or
iv. Biology and Chemistry
B?
v. Art or Chemistry (or both)?
2. How many students like B or C?
vi. Art or Chemistry but not both?
3. How many students like both A and C but
vii. Neither Arts nor Biology nor Chemistry
not B?
vii. How many students are in the class?
4. How many students only like A?
5. How many students do not like either B or
Solution
C?
i. n(Art and Biology but not Chemistry) = 3
6. How many students like A or C but not B?
ii. n(Biology but not Art) = 2 + 6 = 8 7. How many students like B or C but not A?
8. How many students like A or C?
iii. n(Chemistry but neither Art nor Biology) = 9. How many students like A and C?
12 10. How many students do not like A and B?
iv. n(Biology and Chemistry)
= 20 + 6 = 26 Solution
1. (A∪B)1 = 19 + 29 = 39

Baffour Ba Series , Further Mathematics for Schools Page 20


2. B∪C = 14 + 12 + 15 + 6 + 9 + 20 = 76 Exercises 1.5
3. A∩C only = 9 1. The venn diagram below shows the number
4. A only = 7 of students in form 3 who has registered for
5. ( B∪C)1 = 7 + 19 = 26 keyboard (K), guitar (G) and drum (D)
6. (A∪C) ∩B1 = 7 + 9 + 20 = 36
7. (B∪C) ∩ A1 = 12 + 6 + 20 = 38 K G
8. A∪C = 7 + 14 + 15 + 9 + 6 + 20 = 71 7
8 11
9. A∩C = 9 + 15 = 24 24
1 2
10. (A∩B)1 = 7 + 9 + 20 + 6 + 12 + 19 = 73
5
3. Answer the questions below using the Venn 6 D
diagram.
B
A a. How many students have enrolled for guitar
3 classes?
8 16 5
b. Find the number of students who have not
9
14 12 enrolled in any of the classes.
15 17 13

7 10 2. The venn diagram below represents the


18 C number of students willing to study the subjects
Physics (P), History and Social studies (S).
List the elements of the following regions.
P H
1. A1 ∪(B∩C)1 6. C1 ∩ (A∩B)1
28 17 19
2. (B∪C)1∪ A 7. A1 ∩ (B – C)1
10 12
3. C∪(A – B) 8. (A ∪ B)1 ∪ C 4
4. A – (B∪C) 9. (A∪ C) ∩ B 34
5. C – (A∪B) 10. (A ∪B)1 ∪ C1 2 S

Solution a. How many students study all the three


1. {3, 5, 7, 8, 9, 10, 12, 13, 14, 15, 16, 18} subjects.
2. {3, 7, 8, 9, 12, 14, 15, 18} b. How many students study at least two
3. {5, 8, 9, 12, 13, 14, 15} subjects
4. {8, 10, 13, 14}
5. {3, 5, 8, 10, 14, 16} Type 2: Finding the universal set
6. {5, 7, 8, 14, 18} This involves finding the value of the universal
7. {7, 13, 18} set (total).
8. {7, 9, 10, 12, 13, 18}
9. {3, 9, 12, 13, 16} Note that for three sets A, B, and C,
10. {3, 5, 7, 8, 10, 14, 16, 18} 1. a. n(A∪B∪C)

Baffour Ba Series , Further Mathematics for Schools Page 21


= A only + B only + C only + (A∩B) only + n(M∩S∩V) = 4, n(M∪S∪V)1 = {}
(B∩C) only + (A∩C) only + (A∩B∩C), if the n(M∪S∪V) = x
U(x)
values of exactly two items only and exactly
S(30)
one item are given. M(20)
5–4 b
b. n(A∪B∪C) = n(A) + n(B) + n(C) – n(A∩ B) a
4 6–4
9–4
– n(B∩C) – n(A∩C) + n(A∩B∩C), if each
person like all the three items (without c
V(40)
complement)
c. n(A∪B∪C) = ( I + II + III + IV + V + VI + For set M;
VII + VIII) a + 9 – 4 + 4 + 5 – 4 = 20
= n(A) + n(B) + n(C) – n(A∩B) – n(B∩C) – a + 10 = 20
n(A∩C) + n(A∩B∩C) + n(A∪B∪C)1, if some a = 20 – 10 = 10
people do not like all the three items (with
complement) For set S;
b + 6 – 4 + 4 + 5 – 4 = 30
In each case, substitute the given values (or b + 7 = 30
without if not given) in their respective regions b = 30 – 7 = 23
and find n(A∪B∪C) = n(U).
For set V;
Worked Examples c + 9 – 4 + 4 + 6 – 4 = 40
1. Workers are grouped by their areas of c + 11 = 40
expertise, and are placed on at least one team. c = 40 – 11 = 29
20 are on the marketing team, 30 are on the
sales team, and 40 are on the vision team. 5 Now,
workers are on both the marketing and sales x = a + b + c + (9 – 4) + 4 + (6 – 4) + (5 - 4)
team, 6 workers are on both the sales and
Substitute the values of a, b and c;
vision teams, 9 workers are on both marketing
and vision teams and 4 workers are on all the x = 10 + 23 + 29 + 5 + 4 + 2 + 1
three teams. How many works are there in x = 74
U(x)
total?
S(30)
M(20)
Solution 1 23
10
Let U ={Total number of workers} 4 2
5
M = {marketing team}
29
S = {sales team}
V(40)
V = {vision team}
n(U) = ?, n(S) = 30, n(V) = 40, n(M∩S) = 5,
n(S ∩V) = 6, n(M∩V) = 9.

Baffour Ba Series , Further Mathematics for Schools Page 22


Type 3: n(MUPUC) = 200,
Finding the intersection of all the three or n(M) = 120,
exactly three n(P) = 90, n(C) = 70,
It involves finding the value of the intersection n(M∩P) = 40, n(P∩C) = 30, n(M∩C) = 50,
of the three sets. This can be done by either (MUPUC)1 = 20, n(M∩P∩C) = x
using the formula, the diagram or the cover – U= 200
up method as shown in the examples below. M = 120 P= 90
Take note of the fact that for all three sets A, B
f 40 - x g
and C,
1. n(A∪B∪C) = n(A) + n(B) + n(C) – n(A∩ B) x 30 - x
50 - x
– n(B∩C) – n(A∩C) + n(A∩B∩C), if each h
person like all the three items (No complement) 20 C = 70

2. n(A∪B∪C) = n(A) + n(B) + n(C) – n(A∩B)


– n(B∩C) – n(A∩C) + n(A∩B∩C) + Method I
n(A∪B∪C)1, if some people do not like all the For set M,
three items (Complement) f + 40 – x + x + 50 – x = 120
f = 30 + x …………… (1)
In each case, substitute the known values and
workout for n(A∩B∩C) For set P,
g + 40 – x + x + 30 – x = 90
Worked Examples g = 20 + x …………….( 2)
1. In a survey of 200 students of a school, it
For set C,
was found that 120 study Mathematics, 90
h + 50 – x + x + 40 – x = 70
study Physics and 70 study Chemistry, 40
h = -10 + x …………… (3)
study Mathematics and Physics, 30 study
Physics and Chemistry, 50 study Chemistry
For three sets M, P and C, n(M∪P∪C)
and Mathematics and 20 study none of these
= (I + II + III + IV + V + VI + VII + VIII)
subjects.
= x + 40 – x + 50 – x + 30 – x + f + g +h + 20
a. Find the number of students who study all
three subjects.
By substitution,
b. How many students study;
200 = x + 40 – x + 50 – x + 30 – x + 30 + x
i. Physics only? ii. Chemistry only?
+ 20 + x – 10 + x + 20
Solution 200 = 180 + x
Let U = {Students in the school}
x = 200 – 180
M = {Mathematics students}
P = {Physics students} x = 20
C = {Chemistry students}

Baffour Ba Series , Further Mathematics for Schools Page 23


Method II: (Cover – up method) n(T∪H∪F) = 118, n(T) = 56,
On the diagram, cover up one of the circles and n(H) = 44, n(F) = 67,
add the value of that covered circle to the sum n(T∩F) = 23, n(F∩H) = 18, n(H∩T) = 20,
of the values of the other regions uncovered n(M∪P∪C)1 = 0, n(M∩P∩C) = n
and equate them to the value of the universal U= 118
set. T = 56 F = 67

a 23 - n b
Covering set C,
70 + f + 40 – x + g + 20 = 200 n 18 - n
20 - n
But f = 30 + x and g = 20 + x
c
H = 44
By substitution,
70 + 30 + x + 40 – x + 20+ x + 20 = 200
180 + x = 200 Method I
x = 200 – 180 For set T,
x = 20 a + 23 – n + n + 20 – n = 56
a = 56 – 23 – 20 + n
b. i. n(P) only = g = 20 + x, but x = 20 a = 13 + n……………………..(1)
= 20 + 20
= 40 For set F,
b + 23 – n + n + 18 – n = 67
ii. n(C) only = h = - 10 + x, but x = 20 b = 67 – 23 – 18 + n
= - 10 + 20 b = 26 + n……………………..(2)
= 10
For set H,
2. In a certain school , there are 118 boys in c + 18 – n + n + 20 – n = 44
form three. Of these, 56 play table – tennis, 67 a = 44 – 18 – 20 + n
play football and 44 play hockey. 23 play table a = 6 + n……………………..(1)
tennis and football, 18 play football and
hockey, and 20 play hockey and table tennis. For three sets T, F and H, n(T∪F∪H)
Everybody play at least one game and n boys = (I + II + III + IV + V + VI + VII + VIII)
play all the three. Express these facts in a Venn = n + 23 – n + 18 – n + 20 – n + a + b + c
diagram and hence find the value of n.
By substitution,
Solution 118 = n + 23 – n + 18 – n + 20 – n + 13 + n +
Let U = {Boy in form three} 26 + n + 6 + n
T = {Boys who play table tennis} 118 = 106 + n
H = { Boys who play hockey} x = 118 – 106
F = { Boys who play football} x = 12

Baffour Ba Series , Further Mathematics for Schools Page 24


Method II 12 owned exactly two types of the three pet
On the diagram, cover up one of the circles and ⇒ p + q + r = 12…………(i)
add the value of that covered circle to the sum Now,
of the values of the other regions uncovered For set C,
and equate them to the value of the universal a + p + q + x = 30
set. a = 30 – x – p – q ………..(1)

Covering set H, For set D,


44 + a + b + 23 – n = 118 b + q + r + x = 40
But a = 13 + n and b = 26 + n b = 40 – x – q – r…………..(2)

By substitution, For set C,


44 + 13 + n + 26 + n + 23 – n = 118 c + p + r + x = 10
106 + n = 118 c = 10 – x – p – r ………(3)
n = 118 – 106
n = 12 Now,
a + p + q + x + b + r + c = 60
2. In the city of Kwadaso, 60 people own cats, a + b + c + p + q + r + x = 60
dogs or rabbits. If 30 people owned cats, 40
Substitute a, b and c;
owned dogs, 10 owned rabbits and 12 owned
30 – x – p – q + 40 – x – q – r +10 – x – p – r +
exactly two of the three types of pet. How
x + p + q + r = 60
many people own all the three?
80 – 2x – 2p – 2q – 2r + p + q + r = 60

Solution But p + q + r = 12;


Let U ={Owners} ⇒80 – 2x – 2p – 2q – 2r + 12 = 60
C = {Cat Owners}
80 + 12 – 2x – 2p – 2q – 2r = 60
D = {Dog Owners}
92 – 2x – 2p – 2q – 2r = 60
R = {Rabbit Owners}
92 – 60 – 2p - 2q – 2r = 2x
n(U) = 60, n(C) = 30, n(D) = 40, n(R) = 10,
32 – 2 (p + q + r) = 2x
n(C∩D) = p, n(C ∩R) = r , n(D∩R) = q.
32 – 2(12) = 2x
⇒ p + q + r = 12 32 – 24 = 2x
n(C∩D∩R) = x, n(C∪D∪R)1 = { } 8 = 2x
U(60) x=4
D(40) Therefore, 4 people own all the three pets.
C(30)
a q b
x r 3. An examination was held for the filling of
p
vacancies in three branches A, B and C of a
c
R(10) certain service. There were 75 candidates, all of
whom were asked to name the branch or
Baffour Ba Series , Further Mathematics for Schools Page 25
branches in which they were willing to accept a ⇒ 3 + q + 3 + r + x + p + 2 = 75
vacancy, if it were offered. The results of this
inquiry was as follows; 3 candidates would Substitute p = 5, q = 28 and r = 10
accept A only, 3 B only and 2 C only; there 3 + 28 + 3 + 10 + x + 5 + 2 = 75
were altogether 15 candidates who would not x + 51 = 75
accept A, 10 who would not accept B and 34 x = 75 – 51 = 24
who would not accept C.
With the aid of a Venn diagram , or otherwise, Type 4
find how many were prepared to accept a Finding exactly two items
vacancy in any of the three branches. This involves finding the number of items in
every two intersections and adding up. Note
Solution that for three given sets A, B and C, the number
n(A) only = 3, n(B) only = 3, n(C) only = 2 of people in exactly two of the sets
A1 = 15, B1 = 10, C1 = 34 = (II + III + IV)
n(A∩B) only = q, n(A∩C) only = p = n(A∩B) + n(A∩C) + n(B∩C) –3n(A∩B∩C)
n(B∩C) only = r, n(A∩B ∩ C) = x
n(A∪B ∪ C) = 75 Worked Examples
1. In a competition, a school awarded medals in
U(75)
different categories. 36 medals in dance, 12
B
A medals in dramatics and 18 medals in music. If
q 3 these medals went to a total of 45 students and
3
x r only 4 persons got medals in all the three
p
categories, how many received medals in
2
C exactly two of these categories?

Solution
A1 = r + 3 + 2
Method 1:Without the Venn diagram
A1 = 15
A = {persons who get medals in dance}
⇒ r + 3 + 2 = 15
B = {persons who get medals in dramatics}
r = 15 – 2 – 3 = 10
C = {persons who get medals in music}
n(A) = 36, n(B) =12 , n(C) =18
B1 = p + 3 + 2 = 10
n(A∪B∪C) = 45
⇒ p + 3 + 2 = 10
n(A∩B∩C) = 4
p = 10 – 2 – 3 = 5
Exactly two of the three sets A, B and C
C1 = q + 3 + 3= 34
= n(A∩B) + n(B∩C) + n(A∩C) – 3n(A∩B∩C)
⇒ q + 3 + 2 = 34
= n(A∩B) + n(B∩C) + n(A∩C) – 3 (4)……(1)
q = 34 – 3 – 3 = 28
n(A∪B∪C) = n(A) + n(B) + n(C) – n(A∩B) –
n(B∩C) – n(A∩C) + n(A∩B∩C)
n(A∪B ∪ C) = 75

Baffour Ba Series , Further Mathematics for Schools Page 26


But, Now,
n(A∩B) + n(B∩C) + n(A∩C) a + p + q + 4 + b + r + c = 45
= n(A) + n(B) + n(C) + n(A∩B∩C) – n(AUBUC) a + b + c + p + q + r + 4 = 45
= 36 + 12 + 18 + 4 – 45 = 25
Substitute a, b and c;
By substitution, eqn (1) now becomes; 32 – p – q + 8 – q – r +14 – p – r + 4+ p + q +
= n(A) + n(B) + n(C) + n(A∩B∩C) – r = 45
n(A∪B∪C) – 12 58 – p – q – r = 45
= 25 – 12 58 – 45 = p + q + r
= 13 people 13 = p + q + r
Therefore, 13 students received medals in
Method 2: Using the Venn diagram
exactly two categories.
Let A = {persons who got medals in dance}
B = {persons who got medals in dramatics}
2. This semester, each of the 90 students in a
C = {persons who got medals in music}
certain class took at least one course from A, B
n(A) = 36, n(B) =12 , n(C) =18
and C. If 60 students took A, 40 students took
n(ABUC) = 45, n(A∩B∩C) = 4
1
B, 20 students took C and 5 students took all
n(A∪B∪C) = { } the three, how many students took exactly two
U(45)
B(12) courses?
A(36)
a q b
Solution
p 4 r
Let U = {Students}
c A = {Students studying course A}
C(18)

For set A, B = {Students studying course B}


a + p + q + 4 = 36 C = {Students studying course C}
a = 34 – 4 – p – q n(U) = 90, n(A) = 60, n(B) = 40, n(C) = 20,
a = 32 – p – q ……………(1) n(A∩B) = p, n(B∩ C) = r , n(A∩C) = q.
n(A∩B∩C) = 5, n(A∪B∪C)1 = { }
For set B, U(90)

b + q + r + 4 = 12 B(40)
A(60)
b = 12 – 4 – q – r q b
a
b = 8 – q – r ……………(2)
p 5 r
For set C, c
C(20)
c + p + r + 4 = 18
c = 18 – 4 – p – r
For set A,
c = 14 – p – r ……………(3) a + p + q + 5 = 60
a = 60 – 5 – p – q
Baffour Ba Series , Further Mathematics for Schools Page 27
a = 55 – p – q ……………(1) which represents the complement of the three
sets can be calculated.
For set B,
b + q + r + 5 = 40 Worked Examples
b = 40 – 5 – q – r 1. a. The Venn diagram below shows choices
b = 35 – q – r ……………(2) of subject by 40 students. 18 chose Chemistry
(C), 20 chose Physics (P) and 15 chose Biology
For set C, (B), 2 chose Chemistry only, 8 chose Physics
c + p + r + 5 = 20 only and1 chose Biology only, 4 chose all
c = 20 – 5 – p – r subjects. Find the number of students who
c = 15 – p – r ……………(3) chose none of the three subjects.
U= 40
Now, C = 18 P = 20
a + p + q + 5 + b + r + c = 90 x
2 8
a + b + c + p + q + r + 5 = 90 4

Substitute a, b and c; y 1 B = 15
55 – p – q + 35 – q – r +15 – p – r + 5+ p + q
+ r = 90 Solution
110 + p + q + r = 90 Let x be the number of students who chose P
p + q + r = 110 – 90 and C only
p + q + r = 20 Number of students who offer Physics and
Biology only;
Therefore, the number of students who took
= 20 – (8 + x + 4)
exactly two courses = 20
=8–x

Type 5: Number of students who offer Chemistry and


Finding the complement of the three Sets Biology only;
It involves finding the value or number of
= 18 – (2 + x + 4)
items that does not belong to any of the three
= 12 – x
sets. It is called the complement of the set.
Therefore, for biology;
For any three given sets A, B and C,
(12 – x) + 4 + 1 + (8 – x ) = 15
n(AUBUC) = n(A) + n(B) + n(C) – n(A∩B) –
25 – 2x = 15
n(B∩C) – n(A∩C) + n(A∩B∩C) + n(A∪B∪C)1,
25 – 15 = 2x
2x = 10
From the above relation, n(A∪B∪C)1 = n(∪1),
x=5

Baffour Ba Series , Further Mathematics for Schools Page 28


Let y be number of students who chose none of b = 5 – q – r ……………(2)
the three subjects;
⇒ 2 + x + 8 + 4 + 1 + (8 – x) + (12 – x) + y = 40 For set C,
c+p+r+2=9
But x = 5 c=9–2–p–r
2 + 5 + 8 + 4 + 1 + (8 – 5) + (12 – 5) + y = 40 c = 7 – p – r ……………(3)
20 + 3 + 7 + y = 40
y = 40 – 30 = 10 But Adults belonging to exactly two
organizations;
2. Of 20 Adults, 5 belong to A, 7 belong to B ⇒ n(A∩B) + n(B ∩C) + n(A∩C) = 3.
and 9 belong to C. If 2 belong to all the three ⇒ p + q + r = 3…………(i)
organizations, and 3 belong to exactly 2
organizations, how many belong to none of From eqn (i);
these organizations. p=3–q–r
q=3–p–r
Solution r=3–p–q
Let U ={Adults}
Now, solving for x;
A = {Adult belonging to A}
p + q + r + 2 + a + b + c + x = 20…..(ii)
B = {Adult belonging to B}
C = {Adult belonging to C}
But from eqn (i);
n(U) = 20, n(A) = 5, n(B) = 7, n(C) = 9,
p+q+r=3
n(A∩B) + n(B ∩C) + n(A∩C) = 3.
⇒3 + 2 + a + b + c + x = 20
n(A∩B∩C) = 2, n(A∪B∪C)1 = x
U(20) Substitute the values of a, b and c in
B(7) 3 + 2 + a + b + c + x = 20
A(5)
q b ⇒ 5 + (3 – p – q) + (5 – q – r) + (7 – p – r) + x
a
= 20
2 r
p (3 – p – q) + (5 – q – r) + (7 – p – r ) + x = 15
c 15 – 2p – 2q – 2r + x = 15
x C(9)
x = 15 – 15 + 2p + 2q + 2r
x = 2(p + q + r)
For set A,
x = 2(3) = 6
a+p+q+2=5
a=5–2–p–q
Type 6
a = 3 – p – q ……………(1)
Finding one or only one intersection
For set B, Worked Examples
b+q+r+2=7 1. Each student in a class of 40 plays at least
b=7–2–q–r one indoor game; chess, carom and scrabble.

Baffour Ba Series , Further Mathematics for Schools Page 29


18 play Chess, 20 play scrabble and 27 play For set A;
carom. 7 play chess and scrabbles, 12 play 3 + 4 + x + a = 18
scrabble and carom and 4 play chess, carom 3 + 4 + 6 + a = 18
and scrabble. Find the number of students who a = 18 – 3 – 4 – 6 = 5
play;
i. chess and carom, Students who play chess and carom;
ii. chess, carom but not scrabble. = x + 4, but x = 6
=6+4
Solution = 10 students
Method 1 : Using the Venn diagram
i. Let A = {students who play chess} ii. Number students who play chess and carom
B = {students who play scrabbles} but not scramble. This is the same as the
C = {students who play carom} number students who play chess and carom
n(A) = 18, n(B) = 20 , n(C) =27 only;
n(A∩B) = 7, n(C∩B) = 12, n(A∩C) = x = n(A∩C) only = x = 6
n(A∪B∪C) = 40, n(A∩B∩C) = 4
n(A∪B∪C)1 = 0 Method 2 : Without Venn diagram
U(40) Let A = {students who play chess}
B(20) B = {students who play scrabbles}
A(18)
3 b C = {students who play carom}
a
n(A) = 18, n(B) =20 , n(C) =27
x 4 8
n(A∩B) = 7, n(C∩B) = 12, n(A∩B∩C) = 4
c n(A∪B∪C) = 40, n(A∪B∪C)1 = 0
C(27)

For set B; n(A∪B∪C) = n(A) + n(B) + n(C) – n(A∩B) –


8 + 3 + 4 + b = 18 n(B∩C) – n(A∩C) + n(A∩B∩C)
b + 15 = 20 By substitution;
b = 20 – 15 = 5 40 = 18 + 20 + 27 – 7 – 12 – n(A∩C) + 4
40 = 69 – 19 – n(A∩C)
Covering up set A, 40 = 50 – n(A∩C)
18 + 8 + b + c = 40 n(A∩C) = 50 – 40 = 10 students
But b = 5 Therefore, 10 students who play chess and carom
⇒ 18 + 8 + 5 + c = 40
ii. Number students who play chess and carom
c = 40 – 18 – 8 – 5 = 9
but not scramble. This is the same as the
For set C; number students who play chess and carom
4 + 8 + c + x = 27 only;
4 + 8 + 9 + x = 27 = n(A∩C) – n(A∩B∩C)
x = 27 – 4 – 8 – 9 = 6 = 10 – 4 = 6

Baffour Ba Series , Further Mathematics for Schools Page 30


Type 7 But x = 4
It involves the situation where two sets n(F∩C) = 10 – 4 = 6
intersect with each other but not all the three
sets. n(F) only + n(F∩C) + n (C) only + n(C∩M) +
n(M) only + n(F∪C∪M)1= U
Worked Examples
1. In a school, 50 students were asked their ⇒8 + 6 + 17 + 4 + 9 + y = 50
preference for three brands of soft drinks 44 + y = 50
namely; Fanta, coke and malt. None liked all y = 50 – 44
the three brands, 8 liked only Fanta, 17 liked y=6 U = 50
only coke and 9 liked only malt. 10 liked two
C M =13
brands, 13 liked malt and none liked Fanta and F
malt 6 17 4 9
8
i. Illustrate the information on a Venn diagram
ii. How many students liked; 6
a. fanta, b. coke,
c. at least one soft drink, d. one soft drink, ii. a. Number of students who liked Fanta;
e. none of the drinks. = 8 + 6 = 14

Solution b. Number of students who liked coke;


= 6 + 17 + 4 = 27
i. U = {Students}
F = {students who like Fanta} c. Number of students who liked at least one
C = {students who like coke} soft drink = 8 + 6 + 17 + 4 + 9 = 44
M = {students who liked malt}
n(U) = 50, n(F) only = 8, n (C) only = 17 d. Number of students who liked one soft drink
n(M) only = 9, n(M) = 13, n(C∩M) = x = 8 + 17 + 9 = 34
n(F∩C) = 10 – x n(F∪C∪M)1= y
n(F∩C ∩M) = {} e. none of the drinks = 6
U = 50
Some Solved Past Question
C M =13
F 1. A number of school children were asked
8 10 – x 17 x 9 whether they liked rice, fufu or kenkey. Twelve
children said they liked rice, 16 liked fufu and
y 21 liked kenkey. Only three children said they
liked all the three foods. 5 children liked rice
For set M, and fufu, eight children liked rice and kenkey
x + 9 = 13 and 12 children liked kenkey and fufu. How
x = 13 – 9 = 4 many liked;
n(F∩C) = 10 – x i. Rice only ii. Fufu only
iii. Kenkey only iv. Rice or fufu or kenkey
Baffour Ba Series , Further Mathematics for Schools Page 31
Solution both pig and cattle, 28 keep both cattle and
n(U) = a, n(R) = 12, n (F) = 16, n(K) = 21, sheep and 8 keep both pigs and sheep.
n(R∩F∩K) = 3, n(R∩F∩K1) = 5, a. Draw a Venn diagram to illustrate the above
n(R∩F1∩K) = 8 n(R1∩F∩K) = 12 information.
b. Calculate n(P ∩ C ∩S).
U(a)
2. Total of 14 advocates, 8 can handle criminal
F(16)
R(12) cases, 9 civil cases, 10 constitutional cases. 6
2 y can handle criminal and civil cases, 5 civil and
x
3 9
constitutional cases, 5 criminal and
5
constitutional cases. How many can handle;
z
K(21)
i. All three cases
ii. 2 cases
n(R∩F1∩K1) = x + 2 + 3 + 5 = 12 iii. Only constitutional cases
x + 10 = 12
x = 12 – 10 = 2 3. The universal set is the set of integers {x : 1
≤ x ≤ 20}. The sets A, B, C are defined by A =
b. n(R1∩F∩K1) = 2 + 3 + 9 + y = 16 {p : p is a multiple of }; B = {q : q is a multiple
y + 14 = 16 of 3}; C = { r : r is a multiple of 9}
y = 16 – 14 = 2 i. List the elements of :
a. A∩ C b. A∩B∩C1 c. B ∩ A1∩ C1
c. n(R1∩F1∩K) = 5 + 3 + 9 + z = 21 ii. Illustrate the relationship of A, B and C in a
z + 17 = 21 Venn diagram and list in each region the
z = 21 – 17 numbers which it contains.
z=4
4. A survey of 85 students asked them about
d. n(R∩F∩K) = x + 2 + 3 + 5 + 9 + y + z = a the subject they liked to study. Thirty – five
x + y + z + 19 = a students liked mathematics, thirty – seven liked
history and 26 liked physics. 20 liked
But x = 2, y = 2 and z = 4 mathematics and history, 14 liked mathematics
a = 2 + 2 + 4 + 19 and physics, and 3 liked history and physics. 2
a = 27 students liked all the three subjects.
a. How many of these students like
Exercises 1.6 mathematics or physics?
1. In a region of mixed farming, farms keep b. How many of these students did not like any
pigs, cattle or sheep. There are 77 farms of the three subjects?
altogether. 19 farms keep only pigs, 8 keep c. How many of the students liked mathematics
only cattle and 13 keep only sheep. 13 keep and history but not physics?

Baffour Ba Series , Further Mathematics for Schools Page 32


5. Each of the 59 members in a high school liked all the three product, what percentage of
class is required to sign up for a minimum of the survey participants liked more than one of
one and a maximum of three clubs. The three the three products.
clubs to choose from are the poetry club, ,
27students for the history club and 28 students 9. In a class of 50 students, 20 play Hockey, 15
for the writing club. If 6 students sign up for play crickets and 11 play football. 7 play but
exactly two clubs, how many students sign up hockey and cricket, 4 play cricket and football
for all the three clubs. and 5 play hockey and football. If 18 students
do not play any of these given sports, how
6. In a certain school, there are 180 pupils in many students play exactly two of these sports.
the final year. 110 pupils study French, 88
study German and 65 study Indonesian. 40 10. When Mrs. Blay looked at the rosters for
pupils study both French and German, 38 study terms classes, she saw that the roster for her
German and German only. Find the number of economic classes (E) had 26 names, the roster
pupils who study; for her marketing class (M) had 28, and the
a. all the three languages. roster for her statistics class (S) had 18. When
b. Indonesian only. she compared the roster, she saw that E and M
c. none of the languages. had 9 names in common, E and S had 7, and M
d. at least one language and S had 10. She also saw that 4 names were
e. either one or two of the three languages. on all 3 rosters. If the roster for Mrs. Blay 3
classes are combined with no student‟s name
7. In a class of 40 students, 17 has ridden an listed more than once, how many will be on the
air plane, 28 have ridden a boat, 10 has ridden a combined roster.
train, 12 have ridden both an airplane and a
boat, 3 have ridden a train only, and 4 have 11. A form three science class contains 50
ridden an airplane only. Some students of the students all of whom take Mathematics. 18
class have not ridden any of the three modes of study Chemistry, 17 study Biology and 24
transportation and an equal number have taken study Physics. Of those taken three subjects 5
all the three. study physics and chemistry, 7 study Physics
a. How many students have used all three and biology and 6 study Chemistry and
modes of transportation? biology, while 2 take all four subjects.
b. How many students have taken only the a. Draw a Venn diagram to illustrate this
boat? information.
b. Use your diagram to find:
8. In a consumer survey, 85% of those i. how many students study only mathematics.
surveyed liked at least one of the three ii. how many students study only twos subjects.
products; 1, 2 and 3. 50% of those asked like
product 1, 30% liked product 2 and 20% liked 12. The number of farmers growing rice, maize
product 3. If 5% of the people are in the survey and yams is illustrated in the diagram below:

Baffour Ba Series , Further Mathematics for Schools Page 33


13. In a consumer survey, 85% of those
U
M R surveyed liked at least one of the products A, B
Y
and C. 50% of those asked liked product A,
55 100 150 25 65 30% liked product B and 20% liked product C.
If 5% of the people in the survey liked all three
200
of the products, what percentage of the survey
U = {farmers} R = {rice farmers} participants liked more than one of the three
M = {maize farmers} Y = {yam farmers} products.
i. How many farmers grow only one crop?
ii. How many grow exactly two crops 14. There are 50 employees in the office of
iii. How many grow all the three crops ABC company. Of these, 22 have taken an
iv. How many grow only yam or only maize? accounting course, 15 have taken a course in
v. How many grow maize but not rice? finance and 14 have taken a marketing course.
vi. How many grow yam and rice? 9 of the employees have taken exactly two of
vii. How many grow rice and maize but not the courses and 1 employee has taken all three
yam? of the courses. How many of the 50 employees
viii. How many grow at least one of the three have taken none of courses?
crops.

Baffour Ba Series , Further Mathematics for Schools Page 34


2 SURDS Baffour Ba Series

Meaning of Surds 8. √ – √ ≠ √
A surd is defined as the root of a number that √ – √ ≠ √ since √ = √
cannot be reduced to a whole number. For (undefined)
example, √ and √8 = 2 are not surds,
as the respective answer is a whole number. On Exercises
the other hand, √ (on A. Show whether statements are true or not.
calculator) is a surd because the answer is 1. √ + √ = √
irrational. Other examples of surds are√ , √ , 2. √ – √ = √
√ .
B.1. Prove that √ √ = √ for all positive
Properties of Surds numbers a and b.
1. √ ×√ = (√ ) = a Hint: Consider (√ √ ) = (√ )
e.g.√ ×√ = (√ ) = 5 √
2. Prove that = √ for a 0 and > 0.

2.√ × √ = √ Hint: Consider (√ √ ) = (√ )


e.g. √ × √ = √ =√
3. Are √ +√ = √ and √ – √ =

3. =√ √ possible law for radical numbers?


e.g. =√ =√ A simplified Surd as Single Root

Surds of the form a√ are said to be in their
4. a × √ = a√ simplest form. Such surds can be expressed as
single surd by going through the steps below:
e.g. 5 × √ = 5√
1. Find the square of the coefficient of the root
sign to obtain a perfect square. That is in a√ ,
5. a√ × c√ = a √
find a2
e.g. 2√ × 3√ = 2 × 3√ = 6√
II. Put the perfect square obtained in a root and
multiply by the surd. That is:√ ×√ .
6. a√ × c√ = a (√ ) = acb III. Simply and express the product as a single
e.g. 2√ × 5√ = 2 × 5(√ ) = 2 × 5 × 3 = 30 root. a√ =√ ×√ .

7. √ + √ ≠ √ Worked Examples
e.g. √ + √ ≠ √ since √ ≠5 Express 5√ as a single root.

Baffour Ba Series , Further Mathematics for Schools Page 35


Solution 4. Simplified √ +√ +√
5√ =√ ×√ =√ ×√ =√
Solution
Addition and Subtraction of Surds √ +√ +√
Two or more surds with a common surd factor =√ +√ +√
are called like surds. For example, in √ and =√ +√ ×√ +√
√ the common surd factor is √ . Like surds = √ + 2√ + √
can be added and subtracted.
= (1 + 2)√ + √
= 3√ + √
However, two or more surds with no common
surd factor acre called unlike surds. Example
5. √ –√ + √ 8 = k√ . Find the value of
√ and √ are unlike surds. Unlike surds 2
k
cannot be simplified by addition nor
subtraction. Solution
√ –√ + √ 8 = k√ .
Worked Examples
√ – √ +√ = k√ .
1. Add √ + √
5√ – 10√ + 7√ = k√ .
Solution (5 – 10 + 7) √ = k√
√ + √ = (3 + 4 )√ = 7√ 2√ = k√
⇒k=2
2. Simplify √ + √
k2 = 22 = 4

Solution Exercises 2.1


√ + √ = (7 + 1)√ = 8√ A. Simplify:
1. 2√ + √ 4. 6√ 8 + 5√8
3. Simplify 5√ –√ 8
2. 7√ – 3√ 5. √ + √ 8
3. 3√ + 2√ 6. √ + √8
Solution
5√ – √ 8
B. Simplify:
= 5√ –√ 1. 4√ + √ – √
= 5√ –√ ×√ 2. 9√8 + √ – 9√8
3. 7√ – 2√ +√
= 5√ – 3√
4. 5√ 8 – 2√ – √
= (5 – 3) √ 5. √ + √ – 9√
=2√ 6. √ + 2√ + √ – 2√

Baffour Ba Series , Further Mathematics for Schools Page 36


Multiplication of Surds
B. 1. √ 2. √ 3. √ √
A. Multiplying a surd by a fraction
To multiply a surd by a fraction:
I. Express the given surd in its simplest form B. Multiplying a surd by a surd
and multiply by the given fraction. Method 1
II. Cross out common fractions if possible. To multiply two or more surd:
I. Express each surd in its simplest form.
Worked Examples II. Group like terms, if any and multiply.
1. Simplify √
Method 2
Solution I. Find the product of the given surd to obtain a
single surd.

II. Simplify the single surd obtained in act I.
= √ ×√ = ×3×√ =2√
Worked Examples

2. Simplify 1. Simplify √ 8 × √

Solution Solution
√ Method 1
= × √88
√ 8×√
= ×√ ×√ =√ ×√
= ×2×√ =√ ×√ ×√ ×√
= ×√ = 3√ × 2√

= (3 × 2 ) √ ×√
= = 6√

3. √ Method 2
√ 8×√
=√
Solution
=√

√ =√ = = =√ ×√

= 6√
Exercises 2.2
Simplify the following: Exercises 2.3
√ √ √ Simplify the following expressions:
A. 1. 2. 3.
√ √ √ 1. √ × √ 4. √ × √
√ √ √
4. 5. 6. 2. 4√ × 2√ 5. √
√ √ √

Baffour Ba Series , Further Mathematics for Schools Page 37


3. √ =√ –5√

C. Expansion of surds Exercises 2.4


Reminders Multiply:
1. a2 – b2 = (a + b ) (a – b) 1. √ (√ + √ )
2. (a + b)2 = a2 + 2ab + b2 2. √ (√ – √ )
3. a(b + c) = ab + ac
3. 2√ (√ + 3√ )
4. (a + b)(c + d)
4. 2√ (3√ + √ )
= a(c + d ) + b (c + d )
= ac + ad + bc + bd 5. 3√ (2√ – 3 √ + 5√ )
5. 4. (a – b)(c – d)
= a(c – d ) – b (c – d ) Type 2
= ac – ad – bc + bd Expansions of the form:
1. (a + b)(c + d)
Type I = a(c + d ) + b (c + d )
Expansion of the form a(b + c) = ac + ad + bc + bd
a(b + c) = ab + bc
2. (a – b)(c – d)
Worked Examples = a(c – d ) – b (c – d )
Simplify the following: = ac – ad – bc + bd
1. √ (√ + √ )
2. √ (√ + √ ) Worked Examples
3. √ (√ – √ ) 1. Multiply ( 2 + √ ) ( 5 – 4 √ )

Solution Solution
1. √ (√ + √ ) (2 + √ ) ( 5 – 4 √ )
=√ ×√ +√ ×√ ) = 2 (5 – 4 √ ) + √ (5 – 4 √ )
=√ +√ = (2)(5) – (2 )(4√ ) + (√ ) (5) – (√ ) (4√ )
= 10 – 8√ + 5√ – 4 × 3
2. √ (√ + √ ) = 10 – 8√ + 5√ – 12
=√ ×√ +√ ×√ ) = 10 – 12 – 8√ + 5√
=√ +√ = –2 – 3√

3. √ (√ – √ ) 2. (4 + √ ) (2 +√ )
=√ ×√ –√ ×√ ) = 4 (2 +√ ) + √ (2 +√ )
=√ –√ = (4)(2) + (4) (√ ) + (2) (√ ) + (√ ) (√ )
=√ –√ ×√ = 8 + 4√ + 2√ + 7

Baffour Ba Series , Further Mathematics for Schools Page 38


= 8 + 7 + 4√ + 2√ 3. (√ – √ ) (√ + √ )
= 15 + 6 √ 4. (√ + √ ) (√ – √ )
5. (√ + 3) (√ – 3)
Exercises 2.5
Simplify the following: Expansions of the form: (a + b)2
1. (3 + √ ) (4 + √ ) 1. (a + b)2 = a2 + 2ab + b2
2. (5 + √ ) (3 + √ ) 2. (a – b)2 = a2 – 2ab + b2
3. (√ + 4 ) (√ – √ )
4. (√ – 2 ) (√ – 5) Worked Examples
Expand the following:
5. (4 + 3√ ) (1 –√ )
1. (7 + √ )2 2. (2 + √ )2
6. (8 – √ ) (3 + 2√ )
3. (6 – 3√ )2 4. (√ – √ )2
Type 3
Solution
Expansions of the from: (a + b) (a – b)
Method 1
(a + b) (a – b) = a2 – b2
1. (7 + √ )2
Worked Examples = 72 + 2(7)(√ ) + (√ )
1. Multiply (2 – √ ) (2 + √ ) = 49 + 14√ + 3
= 52 + 14√
Solution
Method 1 Method 2
(2 – √ ) (2 + √ ) (7 + √ )(7 + √ )
= (2)2 – ( √ )2 = 7(7 + √ ) + √ (7 + √ )
=4–5 = (7)(7) + (7)(√ ) + (7)(√ ) + (√ ) (√ )
= -1 = 49 + 14√ + 3
= 52 + 14√
Method 2
(2 – √ ) (2 + √ ) 2. (2 + √ )2
= 2 (2 + √ ) – √ (2 + √ )
= 22 + 2(2)(√ ) + (√ )
= (2)(2) + (2)(√ ) – (2)(√ ) – (√ ) (√ )
= 4 + 4√ + 5
=4–5
= 9 + 4√
= -1

Exercises 2.6 3. (6 – 3√ )2
Expand and simplify: = 62 – 2(6)( √ ) + ( √ )
1. (√ + 4) (√ - 4) =36 – 36√ + (9)(5)
2. (1 + √ ) (1 – √ ) = 36 – 36√ + 45
Baffour Ba Series , Further Mathematics for Schools Page 39
= 36 + 45 – 36√ Worked Examples
= 81 – 36√ Simplify the following:
√ √ √ √
1. 2. 3. 4.
√ √ √ √ √
4. Express (2 – √ )2 in the form a + b √ ,
where a, b, c Z Solution
√ √ √ √
1. = × = × =
Solution √ √ √ √ √ √ √
√ √ √
(2 – √ )2 2. = =√
√ √
2
= (2) – 2(2)(√ ) + (√ ) 3.

=
√ √
=√ =√ = 2√
√ √
= 4 – 4√ + 3
√ √ √ √
= 4 + 3 – 4√ 4. = = =
√ √ √ √ √
= 7 – 4√
⇒ a = 7, b = - 4 and √ = √ Type 2
1. Write the following in simplest radical form
5. (√ – √ )2 a + b√ , where a , b Q, n Z
√ √
= (√ )2 – 2(√ )(√ ) + (√ ) a. b.
= x – 2√ + 10
Solution

Exercises 2.7 a.
Simplify: √ √ √
= + =2+ =2+ =2+√
1. ( 1 – 2 √ )2 2. (√ – √ )2
⇒ a = 2, b = 1 and n = 2
3. (5√ + 3√ )2 4. (3√ – √ )2
5. Express (7 + √ )2 – (7 – √ )2 in the form

k√ , k N b.
√ √ √ √
= + =2+ =2+ =2+
Division of Surds
=2+√
Type 1

⇒ a = 2, b = 1 and n = 2
If a is divisible by b, then = √ , where √ is

√ Exercises 2.8
a simplified surd. e.g. =√ ,

A. Simplify:

Type 2 1. 2. 15√ ÷ 3√

√ √ 3. √ ÷√ 4. √ ÷√
If a is not divisible by b, then = , where k
√ √
√ √ √
≠ 0. e.g. = = B. Write in simplest radical form a + b√ ,
√ √ √

Baffour Ba Series , Further Mathematics for Schools Page 40


where a , b Q, n Z Worked Examples
1.

2.

3.
√ Rationalize the denominators:
√ √ √
√ √ √ 1. 2. 3. 4. 5.
4. 5. 6. √ √ √ √ √

Solution
Rationalizing Denominators
√ √
Rationalizing the denominator of a surd is the 1. = × =
√ √ √
act of getting rid of any surd in the
√ √ √
denominator. Usually, the word “simplify” is 2.

=

×

= =
used in place of rationalizes.
√ √ √ √
3. = × =
√ √ √
Type 1 : Surds of the form: √ √ √ √ √

4. = × = =
If the denominator of a surd of the form: √ √ √

√ √ √ √ √
(simple surd) is irrational, it has to be made 5. = = =
√ √ √ √ √
rational through the process called
Now rationalize the denominator:
rationalizing the denominator.
√ √ √ √
= × = =
√ √
Steps:
I. In , identify √ , as the irrational Exercises 2.9

denominator. A. Simplify the following:
II. Multiply the numerator and the denominator √ √
1. 2. 3. 4. √. /
√ √ √ √
of the fraction by (meaning the same value is

√ √
maintained since

= 1). That is:

×

B. 1. Express – in the form k √
√ √
III. Multiply numerators and denominators, not
forgetting the fact that √ × √ = b 2. simplify




√ √ √ √ √ √ √ √
That is : × =
√ √
IV. The denominator of the R.H.S. is now a √
3. If a = , find the value of a2 +
rational number, hence, the process is
completed.
√ √ √ √
4. If x = and y = , find the value of
√ √ √ √ √

= x2 + y2

Type 2 Surds of the form :



Irrational Rational Expressions such as √ and √ are
denominator denominator
said to be conjugates. Once the conjugate of a

Baffour Ba Series , Further Mathematics for Schools Page 41


surd is found, it can be used to rationalize the =
(√ √ )
denominator.
(√ √ )
=
Steps: √ √
=
I. Identify the conjugate of the denominator.
II. Multiply both the denominator and
numerator by the conjugate. That is: √
3.


= × √ √
√ √ √ = ×
√ √
III. Simplify the numerator and simplify the
( √ )( √ )
denominator using the property: =
( – √ )( √ )
(A + B) (A – B) = A2 – B2 to complete
( √ ) ( √ ) √ ( √ )
rationalization. That is : ( =
√ )( √ ) – (√ )
√ √ (√ )(√ )
=
Worked examples –

Rationalize and simplify: =
√ √
1. 2. 3. 4. =

√ √ √ √ √ √

Solution √
4.
1. √ √
√ √ √ √
√ = ×
= × √ √ √ √
√ √
√ (√ √ )
( √ ) =
= (√ √ )(√ √ )
( √ )( √ )
(√ )(√ ) (√ )(√ )
√ =
= (√ ) (√ )
(√ )
√ √
√ =
=
√ √ √
= =

= - 6 + 3√
5. Write the following in the form a + b√
2. √ √
√ √ a. b. c.
√ √ √
√ √
= ×
√ √ √ √
(√ √ )
Solution
=( √
√ √ )(√ √ a. = ×
√ √ √
√ √
=

Baffour Ba Series , Further Mathematics for Schools Page 42


=(
( √ ) Consider the L. H. S.
√ )( √ )

√ √ √
= (√ )
√ Rationalize the denominator;
=
√ √
√ = × – ×
= √ √ √ √
√ √
= - 6 – 3√ = (√ – (√
) )
⇒ a = -6 and b = -3 √ √
= –
√ √
√ √ √ == –
b. = ×
√ √ √
( √ )(√ ) = √ + 1 – (√ – 1)
= (√ )(√ ) =√ +1 – √ +1
( ) √
= =√ –√ +1+1
(√ )
=2

=
√ L. H. S = R.H. S
=

– =2
√ √
= –

= – Exercises 2.10
A. Rationalize each denominator and
√ √ √ simplify if possible:
c. = ×
√ √ √ 1. 2. 3.
√ √ √
( √ )( √ )
=( 4. 5. 6.
√ )( √ ) √ √ √ √ √

( √ ) √ ( √ ) 7. 8. 9.
= √ √ √
( √ ) √ ( √ )

√ √ B. Write with integer denominator;


=
√ √ √ √ √
1. 4. 7.
√ √ √ √

=
√ √ √
2. 5. 8.
√ √ √ √
= -3 – 2√
√ √ √
3. 6. 9.
√ √ √ √ √
6. Show that – =2
√ √
C. Write in the form a + b√ , where a, b Q
Solution √
1. 3.
√ √
– =2
√ √

Baffour Ba Series , Further Mathematics for Schools Page 43


2. 4.
√ Worked Examples
√ √ √
1. Simplify +
√ √
D. Write in the form a + b√ , where a, b Q
√ √ Solution
1. 2. 3. 4.
√ √ √ √ √ √ √
+ =. /+
√ √ √ √ √
Challenge Problems = + = =
√ √ √ √
A. Rationalize and simplify:
√ √ √ √ √
1. 2. 3. Rationalizing the denominator;
√ √ √ √ √
√ √
√ √ √ = × =
4. 5. 6. √ √
√ √ √

√ √ √
7.
√ √
8.

9.

2. Simplify –
√ √

B. Rationalize and simplify if possible: Solution


1.



2. + –
√ √ √ √ √ √
√ √ √
3. – 4.
√ √

√ √ =. /– = – =
√ √ √ √ √ √ √ √ √ √ √ √ √ √

Rationalizing the denominator;


C. 1. Express - as a single fraction.
√ √
= ×
Hence, or otherwise, express - in the form – √ √
√ (√ )
k √ , where a = 1 – √ and b = 1 + √ = (√ ) (√ )
√ √ )
=
2. Show that + =√ )
√ √ √ √
=
√ √ )
3. If √ = 1.414 and √ = 1.732, find the value =
of +
√ √ √ √
Exercises 2.11
Type 2 Simplify the following:
√ √ √
Simplifying Fractions with Surds 1. + 2. –
√ √ √ √
To simplify radicals expressions involving
fractions, the intent is to produce a single
Radical Equation
fraction without radicals in the denominator An equation in which a variable appears in a
Create the lowest common denominator to radicand (square root, cube root or some other
enable addition of numerators. root) is called a radical equation. The
following are examples:
Baffour Ba Series , Further Mathematics for Schools Page 44
1. √ – 4 = 7, When x = 2,
2. 2√ =√ ⇒2 – 5 = √
3. 3 + √ =x -3 = √
-3 = 3 (FALSE)
A radical equation is solved by using the The number 9 checks but 2 does not. Thus, the
principle of squaring which states that if a = b, solution set is 9.
then a2 = b2. The principle does not states that if
a2 = b2, then a = b. 2. Solve 3 + √ =x
To solve a radical equation;
1. Isolate a radical term. Solution
2. Use the principle of squaring (square both 3+√ =x
sides). √ =x–3
3. Solve the new equation. (√ ) =( – )
4. Check all possible solution in the new 27 – 3x = (x – 3) (x – 3)
equation. 27 – 3x = x2 – 6x + 9
0 = x2 – 6x + 3x + 9 – 27
Worked Examples
0 = x2 – 3x – 18
1. Solve x – 5 = √
(x – 6) (x + 3) = 0 (Solving by factorization)
x - 6 = 0 or x + 3 = 0
Solution
x = 6 or x = -3
x–5=√
Check
Using the principle of squaring;
When x = 6;
( – ) = (√ ) 3+√ =x
(x – 5) (x – 5) = x+ 7 3+√ ( ) =6
x2 – 10x + 25 = x + 7
3+√ =6
x2 – 11x + 10 = 0
3+3=6
(x – 9) (x – 2) = 0 (Solving by factorization)
6 = 6 (TRUE)
x – 9 = 0 or x – 2 = 0
x = 9 or x = 2 When x = -3
3+√ =x
Check
3+√ ( ) = -3
x–5=√
When x = 9, 3 + √ = -3
⇒9 – 5 = √ 3 + 6 = -3
9 = -3 (FALSE)
4=√
Therefore, the solution set is 6.
4 = 4 (TRUE)

Baffour Ba Series , Further Mathematics for Schools Page 45


3. Solve √ –5=0 When x = -3, R.H.S = -3
√ 8 = -3
Solution √ ( ) 8 = -3
√ –5=0 8 = -3

2
(√ ) – (5) = 0 √ = -3 (TRUE)
2x – 3 – 25 = 0 Therefore, the solution set is x = 6 or x = -3
2x – 28 = 0
2x = 28
x = 14 Exercises 2.12
A. Solve the following:
4. Solve √ =√ 1. x4 – 1 = 80 2. √ –7=0
3. √ –6=0 4. 2 √ =5
Solution 5. 3√ =6 6. 3 – √ =0
√ =√
(√ ) = (√ ) B. Solve:
3x + 5 = x – 1 1. √ =√
3x – x = -1 – 5 2. √ =√
2x = - 6 3. √ =√
x = -3 4. √ =√
5. √ +x=7
5. Solve √ 8=x 6. √ + 1 = 2√

Solution C. Solve:
√ 8=x 1. √ = 2x
(√ 8) = ( ) 2. √ =x
3x + 18 = x2 3. √ =3
x2 – 3x – 18 = 0 4. √ =4
(x – 6) (x + 3) = 0 (Solving by factorization)
5. √ =x
x = 6 or x = -3
6. √ =x
Check 7. √ =x
When x = 6, R.H.S = 6 8. √ = 2x
√ 8=6 9. √ – √ =3
√ ( ) 8=6
Type 2
√ 8 8=6 This is usually an equation involving operation
√ = 6 (TRUE) of two different radicals. Eg. √ +√ =3

Baffour Ba Series , Further Mathematics for Schools Page 46


i. Isolate one of them by transferring one to the √ =1+√
other side of the equation. (√ ) =( √ )
ii. Square both sides and expand where 2
x + 7 = 1 + (2) (1) √ + (√ )
possible.
x+7=1+2√ + 3x – 2
iii. Group like terms and square again to
x – 3x + 7 – 1 + 2 = 2 √
remove all radicals.
iv. Solve the equation to obtain the value (s) of -2x + 8 = 2 √
the variable. -2x + 8 = 2 √
v. Verify your answer in the original equation. √
+ =
Worked Examples
-x+4=√
1. Solve √ +√ =3
( ) = (√ )
2 2
Solution (-x) – 2(x)(4) + (4) = 3x – 2
x2 – 8x + 16 = 3x – 2
√ +√ =3
x2 – 8x – 3x + 16 + 2 = 0
√ =3–√
x2 – 11x + 18 = 0
(√ ) =( – √ ) x2 – 11x + 18 = 0
2
x + 2 = 3 – (2) (3) √ + (√ ) (x2 – 2x) – (9x + 18) = 0 (Solving by factorization)
x+2 =9–6√ + x–1 x (x – 2) – 9 (x – 2) = 0
x–x +2 -9+1 = –6√ (x – 2) (x – 9) = 0
-6 = – 6 √ x = 2 or x = 9
1= √
Check;
= (√ )
When x = 2, R.H.S = 1
1=x–1
x=1+1 √ –√ ( ) =1
x=2 √ –√ =1
3 – 2 = 1 (TRUE)
Check:
When x = 2; R.H.S = 3 When x = 9, R.H.S = 1
√ +√ =3 √ –√ ( ) =1
√ +√ =3 √ –√ =1
2 + 1 = 3 (TRUE) 4–5=1
Therefore, the solution set is x = 2 -1 = 1 (FALSE)
Therefore, the solution set is x = 2
2. Solve √ +√ =1
3. Solve √ – √ =1
Solution
√ –√ =1 Solution

Baffour Ba Series , Further Mathematics for Schools Page 47


√ – √ =1 3. √ –√ =2
√ =1+√ 4. √ –√ =1
(√ ) =( √ )
2 5. √ –√ =1
3x + 1 = 1 + (2) (1) √ + (√ )
3x + 1 = 1 + 2 √ + 2x – 1 6. √ –√ =1
3x – 2x + 1 = 2 √
x+1=2√ Equality of Surds
( ) =( √ ) When two or more expressions involving surds
2 2
(x) + 2(x) + (1) = 4(2x – 1) are separated by an equal sign, they are said to
x2 + 2x + 1 = 8x – 4 be equal. Mathematically, if a + √ = c + √ ,
x2 + 2x – 8x + 1 + 4 = 0 thena = c, =
x2 – 6x + 5 = 0
(x2 – x) – (5x + 5) = 0 Worked Examples
x (x – 1) – 5 (x – 1) = 0 1. If x – 3 + √ = -1 + √ , find the values
(x – 1) (x – 5) = 0 of x and y.
x = 1 or x = 5
Solution
Check;
x–3+√ = -1 + √
When x = 1, R.H.S = 1
x – 3 = -1 and √ =√
√ – √ =1
x = -1 + 3 = 2
√ ( ) – √ ( ) =1
√ =√
√ – √ =1
2–1=1 (√ ) = (√ )
1 = 1 (TRUE) y+2=5
y=5–2
When x = 5, R.H.S = 1 y=3
√ –√ =1 (x, y) = (2, 3)
√ ( ) –√ ( ) =1
Exercises 2.13
√ –√ = 1
A. Solve for x and y given that they are
4–3=1
rational:
1 = 1 (TRUE)
Therefore, the solution set is x = 1 or x = 5 1. x + y√ = 3 + 2 √
2. -x + y√ = 11 – 3 √
Exercises 2.12B 3. x + y√ = -3 √
Solve the following: 4. 15 - 4√ = x + y√
1. √ –√ =1 5. + √
2. √ –√ =2 6. + y√ = 0

Baffour Ba Series , Further Mathematics for Schools Page 48


B. Solve for x and y given that they are Solution
rational: Let √ √ = (√ + √ ),
1. (x + y√ ) (2 – √ ) = 1 + √ 7 + 4√ = a + 2√ +b (Squaring both sides)
2. (2 – 3 √ )(x + y√ = √ 7 + 4√ = a + √( ) +b
3. (x + y√ ) (3 + √ ) = 1
7+√ = a + √( )+b
4. (x + y√ ) (3 + √ ) = - 4 √
7 + √ 8 = a + √( )+b
c. Find rational a and b such that: 7 + √ 8 = a + b + √( )
1. (a + √ ) (2 – √ ) = 4 – b√
Therefore:
2. (a + √ ) (3 – √ ) = 6 + b√ a + b = 7………….(1)
3. ( √ ) = 33 + 20 √ 4ab = 48
ab = 12………..…(2)

Challenge Problems From eqn (1)


Use binomial expansion for (a + b)3 to write b=7–a
the following in simplest radical form:
1. (3 + √ )3 2. (√ + √ )3 Substitute b = 7 – a into eqn (2)
a(7 – a) = 12
The Square Root of an Irrational Number 7a – a2 = 12
Consider the square of; a2 – 7a + 12 = 0
(√ + √ )2 = a + 2√ + b, where a and b are (a – 3) (a – 4) = 0
positive rational numbers. ⇒ a = 3 or a = 4 and b = 4 or b = 3
(√ + √ )2 = a + 2√ + b The required number is (√ +√ ) or
= (a + b) + 2√ (√ + √ ) = (2 + √ ),
= (a + b) + √ ⇒√ √ = (2 + √ ),

Let (a + b ) = p and 4ab = q 2. Find the square root of 18 – 12√ .


By substitution,
(√ + √ )2 = p + q, where p and q are both Solution
rational. Hence, the square root of the irrational
Let √ 8 √ = (√ + √ ),
number p + √ can be represented by :
18 – 12√ = a – 2√ + b (Squaring both sides)
(√ + √ ), where p, q and b are positive
18 – 12√ = a – √( ) +b
rational numbers.
18 – √ = a – √( )+b
Worked Examples 18 – √ 88 = a – √( )+b
1. Find the square root of 7 + 4√ . 18 – √ 88 = a + b – √( )

Baffour Ba Series , Further Mathematics for Schools Page 49


Therefore: 4(12 – n)n = 24
a + b = 18………(1) 48n – 4n2 = 24
4ab = 288 0 = 4n2 – 48n +24
ab = 72…………(2) n2 – 12n + 6 = 0

From eqn (1) Solving by quadratic formula;


b = 18 – a √

Substitute b = 18 – a into eqn (2) a = 1, b = -12 and c = 6


( ) √( ) ( )( )
a(18 – a) = 72
( )
18a – a2 = 72

a2 – 18a + 72 = 0
(a – 12) (a – 6) = 0 (Solving by factorization) √

⇒ a = 12 or a = 6 and b = 6 or b = 12 √

Since a > b, it follows that a = 12 and b = 6 √ √


or
√ 8 √ = (√ – √ )
√ √
= ( √ –√ ) + or –

3. Find the square root of 12 + 2√ expressing n=6+√ and m = 6 – √ or


your answer in the form √ + √ n=6–√ and m = 6 + √

Solution Exercises 2.14


Let x = √ √ =√ +√ Find the square root of the following:
Then x2 = .√ √ / = (√ √ ) 1. 14 + 6√ 4. 33 – 12√
2. 28 – 10√ 5. 33 + 20√
x2 = √ = m + n + 2√
3. 17 + 12√ 6. 17 – 4√
x2 = √ = m + n + 2√
=m+n+√
Challenge Problems
1. Suppose x and y are positive numbers. Show
Therefore;
m + n = 12……………….(1) that √ + √ = √ and hence
4mn = 24…………………(2) simplify √ √

From (1)
2. Express as rational denominator.
m = 12 – n √

Substitute m = 12 – n into eqn (2)

Baffour Ba Series , Further Mathematics for Schools Page 50


3 BINARY OPERATIONS Baffour Ba Series

Definition Now;
The word “binary” means composed of two 49 * 2 = 49 + 2 + 2(49)(2)
pieces. A binary operation is simply a rule for = 49 + 2 + 196
combining two values to create a new value. = 247

A binary operation on a set is a calculation iii. * = + + 2. / . /


involving two elements of the set to produce
another element of the set. = + + =2

In a binary operation, mathematical symbols 2. Let a ⋄ b = a2 – 2ab, where a, b R. Find


like ●, ◘, ■, ♠, ♣, ▲, ♥, ♦, * etc are used on the (3 ⋄ 5) ⋄ (5 ⋄3)
L.H.S. and define at the R.H.S. with
mathematical operators like +, - , ÷ , ×. Solution
Example of a binary operation is a ♦ b = 2ab – a ⋄ b = a2 – 2ab, where a, b R.
a, defined under a given set. (3 ⋄ 5) ⋄ (5 ⋄ 3)

Worked Examples Now,


1. A binary operation is defined on the set of (3 ⋄ 5) = 32 – 2(3)(5)
rational numbers Q, as follows: = 9 – 30
a* b = a + b + 2ab = - 21
Evaluate :
(5 ⋄ 3) = 52 – 2(5)(3)
i. 3 * 7 ii. (5 * 4) * 2 iii. *
= 25 – 30
=-5
Solution
i. a* b = a + b + 2ab (3 ⋄ 5) ⋄ (5 ⋄ 3) = -21 ⋄ - 5
3 * 7 = 3 + 7 + 2(3)(7) -21 ⋄ - 5 = (-21)2 – 2(-21) (-5)
= 3 + 7 + 42 = 441 – 210
= 52 = 231

ii. (8 * 4) * 2
3. Let a * b = , a + b ≠ 0, and a, b R, find;
Evaluate bracket first;
(5 * 4) = 5 + 4 + 2(5)(4) i. 7 * 4 ii. (4 * 7) * 5 iii. (4 * 7) * (7 * 5)
= 5 + 4 + 40
= 49 Solution
i. a * b = ,
(5 * 4) * 2 = 49 * 2 ( )( )
7*4= =

Baffour Ba Series , Further Mathematics for Schools Page 51


ii. (4 * 7) * 5 a. i. Evaluate √ * and *√
( )( ) √ √
(4 * 7) = = ii. Use your results in (i) to evaluate;
.√ / * .√ /
√ √
(4 * 7) * 5 = *5
Solution
⁄ a. i. p * q = p2 + q2 – 2pq
*5= = ⁄
=
√ *√ (√ ) + . / – 2(√ ) . /
√ √
=3+ – 2(√ ) . /
iii. (4 * 7) * (7 * 5) √ √
=3+ . /
( )( )
(4 * 7) = = =3+ =

( )( )
(7 * 5) = = *√ . / + (√ ) – 2. / (√ )
√ √ √
= + 3 – 2. / (√ )
√ √
(4 * 7) * (7 * 5) = * = +3–1=

⁄ ⁄ ⁄ ii..√ / * .√ /
* = = = √ √
⁄ ⁄ ⁄
* = . / + . / – 2. / . / = 6.14
4. x * 4 denotes the integer part when x is
divided by 4 and y ○ 7 denotes the remainder 2. The binary operation ○ is defined on the set
when y is divide by 7. Calculate 81 * 4 and 117 of no zero real number by a ○ b = ab – .
○ 7.
i. Evaluate (1 + √ ) ○ (2 – √ ), giving your
Solution answer in the form x + y √
x * 4 denotes the integer part when x is divided
Solution
by 4.
a ○ b = ab –
81 * 4 = = 20 = 20
(1 + √ ) ○ (2 – √ )

y ○ 7 denotes the remainder when y is divided = (1 + √ )(2 – √ ) -

by 7. √ √
= 2 – √ + 2√ – 2 – ×
117 ○ 7 = = 16 = 5 √ √
( √ )( √ )
= 2√ – √ – ( √ )( √ )
Involving surds √ √
=√ –
1. The operation * is defined on the set of real (√ )
numbers by p * q = p2 + q2 – 2pq, where p, q R =√ –

Baffour Ba Series , Further Mathematics for Schools Page 52


=√ –
√ (2○ 5) = 22 – 2(2) (5)
= 4 – 20
√ √
= – = - 16
√ √
=
(2○ 3) * (2 ○ 5) = -8 * -16
√ ( )( )
= -8 * - 16 = = =
( )

=
√ iii. 2 * (3 ○ 5)
= –
(3 ○ 5) = 32 – 2(3) (5)
=2– √ = 9 – 30
= - 21
⇒ x = 2 and y =
( )( )
Combining Two Binary Operators 2 * -21 = = =
( )
1. Identify the given operators and their
definitions. iv. (2 * 3) ○ (2 * 5)
2. Perform the brackets first before you ( )( )
2*3= =
combine the second operator.
( )( )
Worked Examples 2*5= =
1. Let a ○ b = a2 – 2ab, where a, b R and
a*b= , a + b ≠ 0, and a, b R, evaluate; (2 * 3) ○ (2 * 5) = ○
i. 2 ○ (3 * 5) ○ = . / - 2. / . /
ii. (2○ 3) * (2 ○ 5)
iii. 2 * (3 ○ 5) = –
iv. (2 * 3) ○ (2 * 5) =

Solution 2. Two binary operations are defined as


i. a ○ b = a2 – 2ab and a * b = , a + b ≠ 0, follows: p * q = + and p q = –
2 ○ (3 * 5)
a. If p = and q = , evaluate ;
( )( )
(3 * 5) = = i. p * q ii. p q iii.
2
2○ = 2 – 2 (2) . / = 4 – = b. If p * q = , evaluate p2 q2

ii. (2○ 3) * (2 ○ 5) Solution


(2○ 3) = 22 – 2(2) (3) a. i. p * q
= 4 – 12
p*q= +
=-8

Baffour Ba Series , Further Mathematics for Schools Page 53


* = + = + = = and of what is left”. Thus:
⁄ ⁄

= +. /=
ii. p q
i. Simplify of a cake
p q= –
ii. Evaluate:
= – = – = =– a. b.
⁄ ⁄
iii. Use your answers in i. and ii. to suggest a
⁄ property for the operation
iii. = =–
⁄ iv. Evaluate:
a. ( ) b. ( )
b. From p * q = , v. Evaluate
By substitution,
+ = Truth Set under a Given Binary Operation
⁄ ⁄
1. Identify the binary operation and its
= definition.

= 2. Substitute the given operation with the


variable and solve for the value of the variable.

By cross multiplication
Worked Examples
(p + q) (q – p) = (pq)(pq)
1. A binary operation is defined on the set of
p2 – q2 = p2q2…………..(1)
rational numbers Q, a* b = a + b + 2ab. Find
the value of z if z * 3 = 9
But p q= –
Solution
2 2
p q = – = a* b = a + b + 2ab.
z * 3 = z + 3 + 2(z) (3)
From eqn (1); = z + 3 + 6z
p2 – q2 = p2q2 = 7z + 3

p2 q2 = =1
But z * 3 = 9
⇒ z
Exercises 3.1 7z = 9 – 3
1. Suppose a * b = 3a + 2b, find √ * √ 7z = 6
z=
2. In this question “ of a cake” means the
fraction of a cake formed by taking of a cake 2. Let a ⋄ b = a2 – 2ab, where a, b R. Find a
if a ⋄ 4 = 9

Baffour Ba Series , Further Mathematics for Schools Page 54


Solution Solution
a ⋄ b = a2 – 2ab, where a, b R. a. 3 * (5 * 2)
a⋄4=9 m*n=
a ⋄ 4 = a2 – 2(a)(4) (5 * 2) = =
= a2 – 8a ⁄ ⁄
⇒ a2 – 8a = 9 ⇒3 * (5 * 2) = 3 * = ⁄
= ⁄
=1
a2 – 8a – 9 = 0
(a + 1) (a – 9) = 0 b. i. 8 * k = 12 * 3
a = -1 or a = 9 =
=
3. If a * b = ab + a + b, solve the equations
a * 3 = 19 and ( a * 3) + ( 2 * a) = 4m =3
8 – k = 3k
Solution 8 = 3k + k
a * b = ab + a + b 8 = 4k
a * 3 = 19 k=
a * 3 = 3a + a + 3
k=2
= 4a + 3
⇒ 4a + 3 = 19
ii. k * 8 = 12 * 3
4a = 19 – 3
=
4a = 16
a=4 =

( a * 3) + (2 * a) = 4m =3
Substitute a = 4 k–8=3×8
(4 * 3) + (2 * 4) = 4m k – 8 = 24
[(4)(3) + 4 + 3] + [(2) (4) + 2 + 4] = 4m k = 24 + 8
(12 + 7) + (8 + 6) = 4m k = 23
19 + 14 = 4m
33 = 4m
6. x * y denotes . Given that a * b = 4b * a,
m=
find the two possible values of a * b.

4. The operation * is defined on the set of real Solution


numbers by m * n = ,n x*y=
a. Evaluate 3 * (5 * 2)
a * b = 4b * a
b. Find the truth set of :
a * b = 4(b * a)
i. 8 * k = 12 * 3
ii. k * 8 = 12 * 3 =4. /

Baffour Ba Series , Further Mathematics for Schools Page 55


= 4. Given that a * b = , for every a, b R, and
a2 = 4b2 that 2 * (x * 5) = 10. Find the value of x.
=4
Challenge Problems
=±√ 1. p and q are integers and p * q denotes the
= 2 or – 2 remainder when the product pq is divided by
a * b = 2 or - 2 10. Find a value p, greater than 4, such that
p * p = (8 * p) + 3.
7. If x * y = 2x – y where x and y are real,
determine the value of y for which y * ( 3 * y) = 6 Table of Binary Operation
Given a well-defined binary operation, a table
Solution of values can be constructed for a given set of
x * y = 2x – y values.
y * ( 3 * y) = 6 I. Identify the definition of the binary and the
given set of values
3 * y = 2(3) – y II. If there is n number of elements in the set,
=6–y prepare n × n square table
III. Place the operator at the top left corner of
y * (6 – y) = 2y – (6 – y) the table
2y – 6 + y = 6 IV. Occupy the first row and column of the
3y = 12 table with the elements of the given set as
y=4 shown below;

* n1 n2 n3
Exercises 3.2
n1
1. The operation * on the integers a and b is
n2
defined by a * b = ab + b. Find x given that n3
4*x=x*4
IV. Operate each element of the first row
2. The operation ○ is defined over the set of against each element of the second row under
real numbers, R, by a ○ b = a + b2. Find: the binary definition and record your answers in
a. (3 ○ 2) ○ 5 the cells or boxes until it is completed.
b. k if 3 ○ k = 7
Worked Examples
3. A binary operation * is defined on R, the set 1. The operation ‡ is defined on the set R of
real numbers by p*q = p2 + 2pq. Find; real numbers by x ‡ y = x2 + y2 – 4
i. 3 * 5 ii. 5 * 3 a. Copy and complete the table below for the
iii. (3 * 5) * (5 *3) iv. p if p * 4 = 9 operation ‡ on the set M = {1, 2, 3, 4}

Baffour Ba Series , Further Mathematics for Schools Page 56


b is divided by 10. Construct a combination
‡ 1 2 3 4 table for *
1 2 1 6
2 4 9 16 Solution
3 6 9 21
S = {1, 3, 7}.
4 13 28
By definition,
b. Use the table to; a*b=R+
i. evaluate 2 ‡ 4 and 4 ‡ 2
ii. what can you say about 2 ‡ 4 and 4 ‡ 2? 1*1= , R1
iii. find (1 ‡ 2) ‡ 4
1*3= , R3
Solution 1*7= , R7
a. M = {1, 2, 3, 4}
x ‡ y = x2 + y2 – 4 3*1= , R3
1 ‡ 4 = 12 + 42 – 4 = 13
2 ‡ 1 = 22 + 12 – 4 = 1 3*3= , R9
2 ‡ 3 = 22 + 32 – 4 = 9 3*7= , R1
3 ‡ 3 = 32 + 32 – 4 = 14
4‡ 2 = 42 + 22 – 4 = 16
7*1= , R7
4 ‡ 3 = 42 + 32 – 4 = 21
7*3= , R1
‡ 1 2 3 4 7*7= , R9
1 2 1 6 13
2 1 4 9 16
3 6 9 14 21 * 1 3 7
4 13 16 21 28 1 1 3 7
3 3 9 1
b. From the table; 7 7 1 9
i. 2 ‡ 4 = 16 and 4 ‡ 2 = 16
Exercises 3.3
ii. 2 ‡ 4 = 4 ‡ 2 = 16. 1. A binary operation * is defined on the set R by
The operation ‡ is commutative because it is a * b = the remainder when ab is divided by 4.
symmetrical along the leading diagonal. a. Draw a table for * on the set S = {0, 1, 2, 3}
b. Use your table to find:
iii. From the table (1‡ 2) = 1
(1 ‡ 2) ‡ 4 = 1 ‡ 4 = 13 i. 3 * 3 ii. (1 * 3) * 3 iii. (3 * 2) * (1 * 2)

2. Let S = {1, 3, 7}. The operation * is defined 2. 1. A binary operation * is defined on the set
on x as follows: For all a, b S, a * b is the R by a * b = the remainder when a + b is
remainder when the results of multiplying a by divided by 5. Draw a table for * on the set S =
{1, 2, 3, 4}

Baffour Ba Series , Further Mathematics for Schools Page 57


b. Use your table to find: b * a = b + 2a
i. (2 * 4) * (3 * 3) a + 2b ≠ b + 2a
a * b ≠ b * a.
3. Consider the binary operation * on the set {1, Therefore, the operation is not commutative
2, 3, 4, 5} defined by a * b = HCF of a and b.
i. Write a table for the operation. 3. The operation * is defined on the set Q , of
ii. Is * commutative? rational numbers by a * b = , a + b ≠ 0.
iii. Compute :
Show whether * is commutative or not.
a. (2 * 3) * 5 b. (2 * 3) * (4 * 5)
Solution
Properties of Binary Operations
a*b= ,
The Commutative Property
Let * be any binary operation defined over the b*a=
set S. We say the operation * is commutative if But ab = ba (multiplication is commutative)
for any two numbers a and b belonging to S, a a + b = b + a (Addition is commutative)
*b=b*a ⇒a*b=b*a
Therefore , the operation * is commutative
Worked Examples
1. If a * b = a + b + ab, show whether the The Commutative Property on a Table
operation * is commutative or not.
A binary operation, * defined over a set S, is
Solution said to be commutative if;
The operation * is commutative if; 1. A table of values constructed is symmetrical
a*b=b*a about the leading diagonal.
a * b = a + b + ab
Consider the table below;
b * a = b + a + ba
a + b + ab = b + a + ba * e f g h
Therefore a * b = b * a e e f g h
The operation * is commutative f f f h h
g g h g h
2. Let a * b = a + 2b be defined over the set of h h h h h
real numbers R. Verify whether or not, * is
commutative. It can be seen that the table is symmetrical
about the leading diagonal. That is to say that
Solution each set of entries is a “reflection” of the other
For the operation to be commutative, in the leading diagonal. Therefore the
a*b=b*a operation* is said to be commutative.
a * b = a + 2b

Baffour Ba Series , Further Mathematics for Schools Page 58


2. Test two values on the table by interchanging 1. The operation ○ is defined over the set of
their positions to see if the operation gives the real numbers, R, by a ○ b = ab + a + b. Show
same answer. For e.g. from the above table: whether ○ is associative or not.
e*f=f
Solution
f *e=f Let a, b and c R, then
⇒ e * f = f * e = f. a ○ (b ○ c) = (a ○ b) ○ c
The operation* is said to be commutative But a ○ b = ab + a + b
L. H. S: (a ○ b) ○ c = a ○ (bc + b + c)
Worked Examples
Let b ○ c = bc + b + c = m
The operation * is defined over the set M ={1,
2, 3, 4} as a * b = a + b – 4. Construct a table
a ○ (b ○ c) = a ○ m = am + a + m
of values and show whether the operation * is
By substitution,
commutative or not.
a ○ (b ○ c) = a(bc +b + c) + a + (bc +b+ c)
= abc + ab + ac + a + bc + b + c
Solution
= a + b + c + ab + bc + ac + abc
a* b = a + b – 4,
Set M = { 1, 2, 3, 4}
R. H.S: a ○ (b ○ c) = (ab + a + b) ○ c
* 1 2 3 4 Let ab + a + b = n
1 -2 -1 0 1 ⇒ (a ○ b) ○ c = n ○ c
2 -1 0 1 3
3 0 1 2 3 By substitution,
4 1 2 3 4 (a ○ b) ○ c =(ab + a + b) c + (ab +a + b) + c
= abc + ac + bc + ab + a + b + c
The operation * is commutative because the = a + b + c + ab + bc + ac + abc
table is symmetrical about the leading diagonal Comparing L.H.S results to R.H.S results, it is
seen that a ○ (b ○ c) = (a ○ b) ○ c
Alternatively, Therefore, the operation ○ is associative.
From the table;
2 * 3 = 1 and 3 * 2 = 1 2. The operation * is defined on the set Q, of
⇒2 * 3 = 3 * 2 = 1 rational numbers by a * b = , a + b ≠ 0.
Therefore, the operation is commutative. Show whether * is associative or not.

The Associative Property


Solution
If a, b and care members of a set of real
numbers R, then the operation * defined over a*b= ,a+b≠0
R is associative if: (a * b) * c = a *(b * c) By the associative property,
(a * b) * c = a * (b * c)
Worked Examples
. /*c=a*. /

Baffour Ba Series , Further Mathematics for Schools Page 59


Consider the L. H.S; Solution
. / * c ……………………(1) Let a, b and c Q, then if :
a * (b * c) = (a * b) * c, the operation * is
Let . / =x associative.

By definition, Now, consider the L.H.S.


x*c= ……………………(2) By the definition,
a * b = a + ab
Substitute x = . / in eqn (2)
a * (b * c)
( ⁄ )
. /*c= b * c = b + bc
( ⁄ )

= Let b * c = x

a * (b * c) = a * x
= But a * x = a + ax
a * (b * c) = a + a(b + bc)
Consider the R. H.S; = a + ab + abc
a*. / ……………………(3)
Consider the R. H. S
Let . /=y (a * b) * c
By the definition;
By definition; a * b = a + ab
a*y= ……………….(4) Let (a * b) = y
⇒ y = a + ab
y * c = y + yc
Substitute y = . / in eqn (4)
. ⁄ / Substitute y = a + ab
a*. / =
. ⁄ / ⇒ (a * b) * c = a + ab + (a + ab)c
⁄ = a + ab + ac + abc
=

= L. H. S ≠ R.H.S
a + ab + abc ≠ a + ab + ac + abc
L.H.S. = R. H. S. Therefore , the operation * is associative.
(a * b) * c = a * (b * c) = 4. The binary operation ○ is defined on the set
Therefore, the operation * is associative. of real numbers by x ○ y = x2 – y. Show
whether or not ○ is associative.
3. The operation * is defined on the set Q, of
rational numbers by a * b = a + ab. Show Solution
whether * is associative or not. x ○ y = x2 – y

Baffour Ba Series , Further Mathematics for Schools Page 60


For associative property, Consider the R. H. S. ;
(x○y) ○ z = x○(y ○ z) a ○ (b ○ c)
(b ○ c) = b
Consider the L.H.S. ⇒ a ○ (b ○ c) = a ○ b = c
(x○y) ○ z Now, L. H. S = R. H. S
(x○y) = x2 – y (a ○ b) ○ c = a ○ (b ○ c) = c
(x○y) ○ z = (x2 – y) ○ z Therefore, the operation ○ is associative.
=(x2 – y)2 – z
= (x2 – y) (x2 – y) – z On the other hand, if (a ○ b) ○ c ≠ a ○ (b ○ c),
= x2 (x2 – y) – y (x2 – y ) – z then we conclude that the operation ○ is not
= x4 – x2y – x2y + y2 – z associative.
= x4 – 2x2y + y2 – z
Worked Examples
Consider the R.H.S. 1. The operation * is defined on the set P = {1,
x○(y ○ z) = x ○(y2 – z ) 2, 3, 4, 5} by a * b = , a + b ≠ 0. Copy and
x ○(y2 – z )= x2 – (y2 – z)
complete the table below for * to the nearest
= x2 – y2 + z
whole number.
L.H.S ≠ R. H. S
* 1 2 3 4 5
x4 – 2x2y + y2 – z ≠ x2 – y2 + z
1 1
Therefore, the operation ○ is not associative.
2 1 1 1
3 1 1 2 2
Associative Property on the Table 4 1 1 2 2
Consider the table below for the operation ○ 5 1 1 1 2

○ a b c d ii. From the table show whether * is


a b c a d associative or not.
b c d b a
c a b c d Solution
d d a d c
i. P = {1, 2, 3, 4, 5}

For the operation ○ to be associative, then a*b=


(a ○ b) ○ c = a ○ (b ○ c)
1*1= =1 1*2= =1
Consider the L. H. S. ;
1*3= =1 1*4= =1
(a ○ b) ○ c
(a ○ b) = c
⇒ (a ○ b) ○ c = c ○ c = c 2*4= = 1.3 2*5= =1
3*4= =2 4*3= =2

Baffour Ba Series , Further Mathematics for Schools Page 61


5 *5 = =3 6 * 8 = ( 2(8) – 6) = × 10 = 5
8 * 2 = ( 2(2) – 8) = × (-4) = -2
* 1 2 3 4 5
1 1 1 1 1 1 8 * 4 = ( 2(4) – 8) = × 0 = 0
2 1 1 1 1 1
3 1 1 2 2 2 * 2 4 6 8
4 1 1 2 2 2 2 1 3 5 7
5 1 1 2 2 3 4 0 2 4 6
6 -1 1 3 5
ii. From the table, 8 -2 0 2 4
2 * (3 * 4)
=2*2=1 From the table, investigate 2 * 4 * 6
2 * ( 4 * 6)
(2 * 3) * 4 =2*4
=1*4=1 =3
⇒ 2 * (3 * 4) = (2 * 3) * 4 = 1
Therefore, from the table, * is associative. (2 * 4) * 6
= 3 * 6 (Not defined on the table)
2. i. Copy and complete the table below for
⇒2 * ( 4 * 6) ≠ (2 * 4) * 6
operation * defined by a * b = ( 2b – a) on the
Therefore, the operation * is not associative.
set P = {2, 4, 6, 8}
Exercise 3.4
* 2 4 6 8
A. 1. The binary operation * is defined on Z by
2 1 3 5
4 2 4 6 x * y = 1 – 2xy. Show that * is commutative and
6 1 3 associative.
8 2 4
2. The operation * on R is defined by a * b =
ii. From the table, show whether or not * is ( )
associative. a. Simplify the right hand side and calculate:
– 5 * (– 1) and *0
Solution
b. Prove that the operation * is commutative
a * b = ( 2b – a) if * is a binary operation on Q. Find if a * b =
P = {2, 4, 6, 8} a2 + b2 is commutative and associative.

2 * 8 = ( 2(8) – 2) = × 14 = 7 3. The binary operation ∆ is defined over the


4 * 2 = ( 2(2) – 4) = × 0 = 0 set of real numbers as m ∆ n = . Find:
a. (2 ∆ 3) ∆ (4 ∆ 8) b. (2 ∆ 3) + (4 ∆ 8)
6 * 2 = ( 2(2) – 6) = × (-2) = -1

Baffour Ba Series , Further Mathematics for Schools Page 62


5. The binary operation ○ is defined on the set
of real numbers by x ○ y = x2 – y. Find: ○ 2 4 6
i. 3 ○ 5 ii. 5 ○ 3 2 2 4 6
iii. What can you say about i and ii? 4 4 4 4
iv. Show whether the operation ○ is associative 6 2 4 6
or not.
ii. From the table, a ○ b K., therefore, the
B. In each of the following, the operation * is table is closed.
defined on the set of real numbers. Determine
whether or not * is commutative and iii. 2 ○ 4 = 4 ○ x
associative. =
b
1. a * b = a 2. a * b = 4ab 3. a * b = 4=x
x=4
The closure Property
Given that a and b are members of set S and the 2. The binary operation * is defined on the set A
operation * is defined over S. If a * b always = {1, 2, 3, 4} by x * y = 3x – xy. Show whether
gives an answer which is also in S, then the set * is closed under A in each of the following;
S is said to be closed with respect to *. That is a i. 1 * 2 ii. 3 * 4
* b S, a, b S, the * is said to be closed.
Solution
Worked Examples i. x * y = 3x – xy.
1. The operation ○ is defined on the set K = {2, But x = 1 and y = 2
4, 6} by a ○ b = 1 * 2 = 3(1) – (1)(2)
=3–2
i. Draw a table for ○ on the set K.
=1
ii. Determine whether or not the operation ○ is
1 * 2 = 1 A, therefore set A is closed under
closed on K.
the operation*
iii. Find the truth set of 2 ○ 4 = 4 ○ x
ii. x * y = 3x – xy
Solution 3 * 4 = 3(3) – (3)(4)
K = {2, 4, 6} = 9 – 12
a○b= =-3
2○2= =2 2○4= =4
3 * 4 = - 3 ∉ A, therefore set A is not closed
2○6= =6 4○2= =4 under the operation*
4○4= =4 4○6= =4
Exercises 3.5
6○2= =2 6○4= =4
1. The binary operation ○ is defined as:
6○6= =6 x○ y = x + y + 2xy.

Baffour Ba Series , Further Mathematics for Schools Page 63


a. Find the values of t, k, r and s in the table for By the definition;
the operation ○ on the set P = {1, 2, 3, 4} a ○ x = a + x – ax
But x = (b + c)
○ 1 2 3 4
1 4 7 10 13 ⇒ a ○ (b + c) = a + (b + c) – a(b + c)
2 t 12 17 s a ○ (b + c) = a + b + c – ab – ac
3 10 k r 31
4 13 22 31 40 (a ○ b) + (a ○ c) = (a + b – ab) + (a + c – ac)
= a + b – ab + a + c – ac
b. Show whether set P is closed or not with = 2a + b + c – ab – ac
respect to the operation ○
c. Is the operation ○ is commutative or not? Since a ○ (b + c) ≠ (a ○ b) + (a ○ c), the
operation „○‟ is not distributive.
2. The binary operation* is defined on the set
M = {5, 6, 7, 8, 9} by x * y = xy – 4y. Show ii. a ○ a = - 3………………………(1)
whether * is closed under M in each of the By the definition,
following; a ○ a = a + a – aa…………………(2)
i. 5 * 6 ii. 5 * 8 iii. 7 * 9 iv. 6 * 9
eqn (1) = eqn(2)
The Distributive Property a + a – aa = -3
If a, b and c are three members of the set of real 2a – a2 = -3
numbers, R and the binary operations * and ○ a2 – 2a - 3 = 0
are defined over S, then * is distributive over ○
if a * (b ○ c) = (a * b) ○ (a * c) Solving by factorization;
(a + 1) (a – 3) = 0
However, if a * (b ○ c) ≠ (a *b) ○ ( a * c), then a + 1 = 0 or a – 3 = 0
* is not distributive over ○ a = -1 or a = 3

Worked Examples 2. Two binary operations * and ∆ are defined as


1. i. If a ○ b = a + b – ab, where a, b are real a * b = 2ab and a ∆ b = a + b + 2 for all a, b
numbers, show by evaluating a ○ (b + c) and R. Evaluate:
(a ○ b) + (a ○ c) that the operation „○‟ is not i. 4 * (5 ∆ 6) ii. (4 * 5) ∆ (4 * 6)
distributive. iii. What can you say about (i) and (ii)
ii. Find the truth set of the equation a ○ a = - 3
Solution
Solution i. a * b = 2ab and a ∆ b = a + b + 2
i. a ○ b = a + b – ab In 4 * (5 ∆ 6), taking the bracket first,
For a ○ (b + c)……….....(1) (5 ∆ 6) = 5 + 6 + 2 = 13
Let (b + c) = x 4 * 13 = 2(4)(13) = 104
eqn (1) now becomes a ○ x

Baffour Ba Series , Further Mathematics for Schools Page 64


ii. (4 * 5) ∆ (4 * 6), The Identity Element
Taking the first bracket, Consider the addition statements below;
(4 * 5) = 2(4)(5) = 40 5+ = 5, -14 + = -14, + =
In each case, zero satisfies the equation, hence
Taking the second bracket
the truth set. We can therefore conclude that
4 * 6 = 2(4)(6) = 48
when zero is added to any number, the value of
the number is unchanged. In this case, zero is
⇒(4 * 5) ∆ (4 * 6) = 40 ∆ 48 called the identity element of addition.
= 40 + 48 + 2 Consider the multiplication statements below;
= 90
5× = 5, -14 × = -14, × =
iii. The operation * is not distributive over ∆ In each case, one satisfies the equation, hence
because 4 * (5 ∆ 6) ≠ (4 * 5) ∆ (4 * 6) the truth set. The conclusion is that when one is
multiplied to any number, the answer is that
Exercises 3.6 number. In this case, one is called the identity
1. Two binary operations * and ○ are defined element of multiplication.
over the set, R, of real numbers by a * b = a –
2b and a ○ b = a + b + ab, where a, b R. Some binary operations may have identity
i. Find the values of 3 * - 5 , - 5 * 3: elements under some given sets. Let the
ii. Is the operation commutative on R? identity element of the binary operation be e,
iii. Determine whether or not 4 ○ (3 * - 5) = (4○ and x be a member of a set S. If * is defined
3) * (4 ○ – 5. over the set S, then we can equally say that
iv. Is the operation ○ distributive over *? x * e = x or e * x = x

2. Two binary operations * and ○ are defined Steps:


over the set, R, of real numbers by x * y = xy 1. Use the definition of the given binary
and x ○ y = x + y. Find: operation.
i. x * (y ○ z) ii. (x * y) ○ (x * z) 2. Use the definition of e.
iii. Show whether the operation * is distributive 3. Solve for the value of e.
over ○ or not
Worked Examples
3. Let ∪ = {1, 2, 3… 4}, A = {2, 3, 5, 7, 8} 1. A binary equation is defined over the set of
B = {1, 3, 5, 9} and C = {4, 9, 10} where A, B real numbers R by, a * b = a + b + 2. Find the
and C are subsets of ∪. Find: identity element under the operation * and the
i. A ∩ (B∪C), element a.
ii. (A∩B) ∪ (A ∩ C),
iii. What can you say about the operations ∩ Solution
and ∪ over the set? a*b=a+b+2
a * e = a ……………………(1)

Baffour Ba Series , Further Mathematics for Schools Page 65


a * e = a + e + 2…………….(2) eqn (1) = eqn (2);
⇒ a + 2ae = a
eqn (1) = eqn (2); 2ae = a – a
⇒a+e+2=a 2ae = 0
e=
Solving for e,
e=0
e+2=a–a
e+2=0
4. The operation * is defined over the set R of
e = -2
real numbers by a * b = ab. Show that the real
Therefore, the identity element of the operation
* defined by a * b = a + b + 2 is -2. number 4 , is the identity element for the
operation,
2. A binary operation ○ is defined by x ○ y = xy
+ x + y. Find the identity element e, under the Solution
operation ○ and the element x. a * b = ab
a * e = a…………………..(1)
Solution
a * e = ae……………….(2)
x ○ y = xy + x + y
x ○ e = x………………………(1)
eqn (1) = eqn (2);
x ○ e = xe + x + e……………..(2)
a = ae
eqn(1) = eqn(2); 4a = ae
⇒xe + x + e = x e=

Solving for e, e=4


xe + e = x – x
5. The binary operation * is defined on R by
e(x + 1) = 0
(a, b) * (c, d) = (ac – bd, ad + bc). Find the
e= identity element under *
e=0
Solution
(a, b) * (c, d) = (ac – bd, ad + bc)
3. An operation * is defined on the set S of real
Let the identity element be (x, y)
numbers such that a * b = a + 2ab. Find the (a, b) * (x, y) = (ax – by, ay + bx) = (a, b)
identity element.
⇒ ax – by = a………………..(1)
Solution ay + bx = b………………..(2)
a * b = a + 2ab
a * e = a ……………………(1) From eqn (1);
a * e = a + 2ae……………….(2) ax = a + by
x = …………..………...(3)

Baffour Ba Series , Further Mathematics for Schools Page 66


Put eqn (3) in eqn (2); Identity Element on the Binary table
ay + b. / = b…………..(2) 1. Carefully complete the table if the need be.
2 2 2. Carefully read the rows and column to
ay + ba + b y = ab
observe the identity property. That is:
ay2 + b2y = ab – ab
a * e = a, where e, is the identity element. The
ay2 + b2y = 0
row that gives the orderly arrangement of the
y(ay + b2) = 0
main set is the identity element.
y=0

Worked Examples
Put y = 0 in eqn (1);
1. The combination table for the set S = {a, b,
ax – b(0) = a
ax = a c, d}under the operation * is given below.
x=
* a b c d
x=1 a b c a d
The identity element is (1, 0)
b c d b a
c a b c d
Exercises 3.7
d d a d c
A. Find the identity element of ecah.
1. a * b = 3ab 2. a * b = a + b + 1
State the identity element.
3. a * b = ab + 1
Solution
B. 1. An operation * on gthe set of real
( )
From the table,
numbers is define by a * b = – 1 for all a*c=c*a=a
a, b R. What is the identity element in R b*c=c*b=b
under * c*c=c
d*c=c*d=a
2. What is the identity element for a * b = (a – Therefore, c is the identity element.
b) 2 for all a, b Q
Method 2
3. Let * be a binary operation defined on the set Consider the third row :
Z of integers by a * b = a + b – 5 for all a, b Z.
i. Show that * is commutative and associative. c a b c d
ii. Find the identity element if it exist.
Compared to the main set S = {a, b, c, d}, the
4. Let * be a binary operation defined on N × arrangements are the same (orderly). Hence, c
N, by (a, b) * (c, d) = (ac, bd). is the identity element.
i. Show that * is commutative and associative.
ii. Find the identity element for * 2. The operation * is defined on the set S = {0,
1, 2, 3, 4} by a * b = /a – b/, where /x/ means

Baffour Ba Series , Further Mathematics for Schools Page 67


the numerical value of x. 3 1 3 5
i. Draw a combination table for *. Hence find the 5 5 5 5
identity element for *.
ii. Of the four arithmetic operations, addition, Find the identity element under the operation ○.
subtraction, multiplication and division, which
are associative, commutative and identity? 3. Find if there is an identity element in I = {-1,
0, 1} if the binary operation is addition defined
Solution on I.
S = {0, 1, 2, 3, 4}
a * b = /a – b/, 4. The operation * is defined by a * b = a(b + 2)
i. Solve the equation 6 * 3 = 3 * x
* 0 1 2 3 4 ii. What can be said about a and b if a * b = b *
0 0 1 2 3 4 a?
1 1 0 1 2 3 iii. Solve the equation a * x = a
2 2 1 0 1 2 iv. For what value of a is it impossible to solve
3 3 2 1 0 1 the equation in (iii)
4 4 3 2 1 0
5. Define a binary operation * on A = {1, 2, 3,
From the table, observe that a * 0 = 0 * a = a, 4, 5 as a * b = ab(mod 5) for all a, b Z. Show
for all a S. Therefore, the identity element is that 1 is the identity element with respect to *
zero.
ii. The Inverse Property
Operation Associative Commutative
Consider the statements below;
Addition Yes Yes
5+ = 0, -14 + = 0, + = 0, a + = 0,
Multiplication Yes Yes
In all cases, the sum of the addends is zero
Subtraction No No
under the operation +. This means that the
Division No No
known number added to the unknown is zero,
where zero is the identity element of addition.
Exercises 3.8 The truth sets are as follows;
1. The operation * is defined by a * b denotes 5 + , - = 0, -14 + , - = 0,
, where a, b {-2, 0 2} +0 1 = 0, a+, -=0
i. Construct the operation table for *
ii. State the identity element for * Hence, it is conclusive that for each given
number, there is another number which
2. The operation ○ is defined on the set S = combines with it to give the identity element
{ 1, 3, 5} by the table below. zero. That special number which combines with
a given number to give the identity element is
○ 1 3 5 called the additive inverse of that number.
1 3 1 5
Baffour Ba Series , Further Mathematics for Schools Page 68
Thus, the additive inverse of 5 is – 5, - 14 is 14, i. 7 ii. - 11
a is – a and vice – versa.
Solution
Likewise, consider the following; a. a * b = a + b + 5.
5× = 1, -14 × = 1, × = 1, a × =1 a * e = a …………………..(1)
a * e = a + e + 5…………..(2)
In each case, the product of the known and the
unknown number is one under the operation ×,
eqn (1) = eqn (2);
where 1 is the identity element of
⇒a+e+5=a
multiplication.
The truth sets are as follows;
Solving for e,
5 × 0 1 = 1, -14 × 0 1 = 1, e+5=a–a
× 0 1 = 1, a×0 1=1 e+5=0
e=-5
Hence, we can conclude that for each given
Now,
number, there is another special number which
a* = e………………(3)
multiplies with it to give the identity element
a* =a+ + 5…..(4)
one, where the special number is called the
multiplicative inverse of that number. Thus, the eqn (3) = eqn (4);
multiplicative inverse of 5 is , - 14 is , a is ⇒a+ +5=e
and vice – versa. But e = - 5

Solving for
These special properties can be extend to binary
a+ +5=-5
operations. Now, if is the inverse of a, then
a+ = - 10
for the operation *, a * = e, where e is the
= - 10 – a
identity element.
b. = - 10 – a
To find the inverse of a binary operation,
i. = - 10 – 7 = - 17
1. find the identity element of the binary
operation a * e = a ii. = - 10 – (-11) = 1
2. substitute the value of the identity
2. The operation * is defined on the set R, of
element, e, in a * =e
real numbers by a * b = a + b – ab. Find the
3. Workout for
inverse element and use your results to find the
inverse of 4.
Worked Examples
1. a. A binary operation * is defined by a * b = Solution
a + b + 5. Find the inverse element. a * b = a + b – ab.
b. Use your results to find the inverse of ; a * e = a……………….(1)
Baffour Ba Series , Further Mathematics for Schools Page 69
a * e = a + e – ae………(2) x + e + xe = x

eqn (1) = eqn (2); Solving for e,


a + e – ae = a x + e + xe = x
e + xe = x – x
Solving for e, e(1 + x) = 0
e – ae = a – a
e=
e – ae = 0
e(1 – a) = 0 e=0

e= =0
b. Let be the inverse of x.
Now, By definition,
a* = e……………………(3) x* = e…………………(3)
a* =a+ –a ….…(4) x* =x+ +x ………………..(4)

eqn (3) = eqn (4); eqn (3) = eqn (4);


a+ –a =e x+ +x =e
But e = 0
But e = 0
x+ +x =0
a+ –a =0
Solving for ,
Solving for ,
+x =-x
–a =-a
(1 + x) = - x
(1 – a) = - a
= , x ≠ -1
=
= =
= =

3. A binary operation * is defined on R, the set


Exercises 3.9
of real numbers by x * y = x + y + xy for all x,
1. A binary operation ○ is defined on R, the set
y R. Find:
of real numbers by a ○ b = ab + a – b , for all
a. the identity element.
a, b R. Find:
b. the inverse element of 3.
i. the identity element.
Solution ii. the inverse of 8 under this operation.
a. x * y = x + y + xy for all x, y R. Find:
2. Let * be a binary operation on Z defined by
x * e = x………………………(1)
a * b = a + 3b – 1
x * e = x + e + xe……………(2)
i. Prove that the operation is binary.
ii. Determine whether the operation is
eqn (1) = eqn (2);
associative or commutative.

Baffour Ba Series , Further Mathematics for Schools Page 70


iii. Determine whether the operation has State the inverses of the elements a, b, c, d
identities.
iv. Discuss inverses. Solution
Carefully read the table below.
3. Let * be a binary operation defined on the set Z
of integers by a * b = a + b + 2 for all a, b Z. * a b c d
i. Show that * is commutative and associative. a b c a d
ii. Find the identity element with respect to Z. b c d b a
iii. Find the inverse of a Z. c a b c d
d d a d c
4. The operation * is defined on the set of real
numbers by (a, b) * (c, d) = (a + c, b + d). From the table,
i. Prove that * is both commutative and a*c =a
associative. b*c=b
ii. Find the identity element for * c*c=c
iii. Find the inverse of (-3, 5) under * d*c=a
5. A binary operation * is dfined on Q by (a, b) Therefore, c is the identity element.
* (c, d) = (ac, ad + b). Find :
i. the identity element of * Now,
ii. the invertible (inverse) element of * From the first row,
a * b = c.
The Inverse Element on the Binary Table Therefore, the inverse of a is b
1. Carefully read the rows and columns to
identify the identity element. From the second row,
2. Read the inverse of an element x, by b * a = c.
selecting a row that contains the element x. Therefore, the inverse of b is a.
3. Check if this row contains the identity
element. From the third row,
4. Read off the element which combines with x c * c = c.
to give the identity element as the inverse of x. Therefore, the inverse of c is c.

Worked Examples From the fourth row,


The combination table for the set S = {a, b, c, d * d = c.
d} under the operation * is given below. Therefore, the inverse of d is d.

* a b c d 2. Given the set E = {1, 3}; F = {1}, G = {3};


a b c a d H = { }.
b c d b a i. Copy and complete the table below for the
c a b c d
operation ∪
d d a d c

Baffour Ba Series , Further Mathematics for Schools Page 71


∪ E F G H 2. The combination table for a set of 4 elements
E E {a, b, c, d} under the operation * is given:
F E
G G
H H * a b c d
a b c a d
b. Determine with reason; b c d b a
c a b c d
i. the identity element for the operation ∪;
d d a d c
ii. the inverse of the element G.
i. State the identity element
Solution ii. State the inverses a and of d.
E = {1, 3}; F = {1}, G = {3}; H = { }. iii. Is the operation commutative?
iv. By considering a * b * d, show that the set is
∪ E F G H not associative under *.
E E E E E
F E F E F
2. For I = {-2, -1, 0, 1, 2}and the binary
G E E G G
H E F G H operation on addition defined on I, find unique
inverse element for each of the set.
b. i. The identity element for the operation ∪
x∪e =x 3. A set P = {2, 4, 6, 8, 10, 12}. The operation *
From the table, there is no identity element for is defined as follows: if a and b are any
the operation ∪, none of the rows or columns elements of P, then a * b denotes the remainder
obeys the identity rule given by: when the results of multiplying a by b is
x∪e =x divided by 14; For example, 6 * 12 = 2
a. Find the identity element of P, that is the
ii. The inverse of the element G element e, such that e * a = a for every element
of the set.
Exercises 3.10 b. Find the inverse of 10, that is the element y
1. The operation * is defined by a * b denotes such that y * 10 = e where e is the identity
element.
, where a, b {-2, 0, 2}
c. Solve the equation 10 * x = 4, by multiplying
i. Construct the operation table for * both sides of the equation by the value of y
ii. State the identity element for * above.
iii. For each element, state its inverse or write d. Use similar technique to solve the equation
“none” as the case may be . 6*x=2
e. Find two solutions of the equation x * x = 4

Baffour Ba Series , Further Mathematics for Schools Page 72


4 RELATIONS AND FUNCTIONS Baffour Ba series

Idea of a Relation Consider the mapping below;


A relation is a rule that help us to match two x a b c d
different sets. Among the two sets, the first set, ↓ ↓ ↓ ↓ ↓
usually represented by x is called the domain y q r s t
and the second set, usually represented by y is As ordered pairs, we have (a, q), (b, r), (c, s)
called co – domain. and (d, t)

Consider the rule “is one – third of” for the Here, the first number is an element of the
domain {6, 12, 33, 45}, the co-domain is domain and the second number, the
obtained by planting the elements of the corresponding element in the range.
domain in the rule. Thus, the rule “is one –
third of” is mathematically expressed as : Exercises 4.1
y= x 1. If A = {10, 14, 12} and B = {2, 3, 4, 5}.
Draw an arrow diagram to show the relation
When x = 6, y = (6) = 2
“has as factor” from set A to set B. Also, write
When x = 12, y = (12) = 4 the relation as a set of ordered pairs of
When x = 33, y = (33) = 11 numbers.

When x = 45, y = (45) = 15 2. Given that set P = {8, 10, 14} ans Q = {4, 5,
Co domain = {2, 4, 11, 15} 7}. The relation from set P to set Q is “a factor
of”. Represent the above relation by using:
This is shown in the diagram below: i. an arrow diagram;
x y ii. ordered pairs.

( , ( , 3. Draw an arrow diagram to represent R,


where R = {(3, 2), (5, 2), (5, 4), (7, 2), (7, 4),
(7, 6).
x 6 12 33 45
OR 4. Let A = {2, 4, 7} and B = {1, 3} be a set and
y relation R = {(2, 1), (2, 3), (4, 1), (4, 3), (7, 1),
2 4 11 15
(7, 3)}, then draw an arrow diagram.
Relations as Ordered Pairs
In relations, each elements of the domain has a Range
corresponding element in the range. When each The domain of a relation is the set of x –
element in the domain is paired with its coordinates of the ordered pairs and the range
respective element of the range, we have of the relation is the set of y coordinates of the
ordered pairs: ordered pairs.
Baffour Ba Series , Further Mathematics for Schools Page 73
y= x–5
x y Co- domain = {1, 3, 5, 15}
Domain is = {9, 12, 15}
( , ( , Let x represent the elements of the domain
From the rule, y = x – 5,
When x = 3, y = (9) – 5 = 1
Range
When x = 4, y = (12) – 5 = 3
The range is always a subset of the codomain.
When x = 6, y = (15) – 5 = 5
Worked Example The range of the relation is {1, 3, 5}
1. A relation is defined by the rule “the square
of the odd prime factors of 30”. Exercises 4.2
i. Represent this in the diagram, for the Determine the range;
codomain {1, 4, 9, 16, 25, 36} 1. Rule: “one – fifth decreased by twenty”
ii. From the diagram, find the range. Domain = * , , , +
2. Rule: “ ”
Solution Domain = * , , , , +
i. Odd prime factors of 30 = {2, 3, 5} 3. Rule: “three less than one – fourth”, Domain
Domain = {2, 3, 5} =* , , , +

“the square of the odd prime factors of 30”. Types of Relations


= {4, 9, 16} 1. One – to – one Relation
It is the type of relation in which each element
of the domain matches to only one element of
( + the co-domain. This is represented as in the
diagram below for the relation “is increased by
( ) four”
x y
ii. Range = {4, 9, 16}
( , ( ,
2. Find the range given the following;
8
Rule: „5 less than two - thirds of’
Domain = {multiples of 3 from 9 to 15}
2. One – to – many Relations
Co-domain = {factors of 15}
It is the type of relation in which one or more
element(s) of the domain matches to many
Solution
elements in the co-domain. This is
2. Rule = “5 less than two- thirds of”

Baffour Ba Series , Further Mathematics for Schools Page 74


diagrammatically represented as shown below 2. x y
for the relation “is square root of”;
( , ( +

. / ( ,
3. x y

3. Many – to – one Relation 8


. / ( ,
It is the type of relation in which more than one
element of the domain match to one element of
4. x y
the co-domain. This is represented in the
diagram below for the relation “is a product of”
x y : ; ( ,

( , . /
B. Identify the rule of relations;
1. x y
Domain Co-domain

( + ( +
4. Many – to – many Relations
It is the type of relation in which more than one
member of the domain match to many members 2. x y
of the co-domain as shown below for the
relation, “is less than” ( , ( ,

x y
3.
( + ( +
( , ( ,
Domain Co-domain 8

4.
Exercises 4.3
Identify the type of relation:
8
1. ( , ( ,
x y

( , ( ,

Baffour Ba Series , Further Mathematics for Schools Page 75


Finding the Domain and Co-domain domain. Draw a diagram of the relation.
Given the rule of a relation and the domain, the
co-domain can be found. Similarly, with a Solution
given rule and co-domain, the domain can also Rule =“is one-third of”
be found. Domain ={12, 15, 18, 24},
According to the rule, y = . /,
Likewise, given the rule and the co-domain, the
domain can as well be found. For example, if when x = 12, y = × 12 = 4,
the rule of a relation is given as “is 5 times of” when x = 15, y = × 15 = 5
and the co-domain is * , , , +, then it
when x = 18, y = × 18 = 6,
means that, 5 times a certain group of numbers
will give the respective elements of the co- when x = 24, y = × 24 = 7 ,
domain. The rule is mathematically expressed The co-domain = {4, 5, 6, 7} respectively.
as, y = 5x,⇒ x = x y

Put the elements of the codomain in x = to ( , ( ,


8
obtain the elements of the domain.
When y = 20, x = =4
3. The rule of a relation is “four exponent of”.
When y = 30, x = =6 If the domain = {1, 2, 3, 4}, find the co-domain
When y = 40, x = =8 and show the relation in a diagram.
When y = 50, x = = 10
Solution
The rule is “is four exponent of” : y = 4x
Domain = {4, 6, 8, 10} Domain is * , , , 8, , +.
Codomain = {20, 30, 40, 50} Let x represent the elements of the domain.
Rule : y = 4x,
This can be represented diagrammatically When x = 1, y = 4 = 4
as; When x = 2, y = 42 = 16
“5 times of” When x = 3, y = 43 = 64
x y When x = 4, y = 44 = 256

( , ( ,
8
( , ( ,

Worked Examples
1. The rule of a relation is “is one-third of”. Domain Co-domain
Given the domain, {12, 15, 18, 24}, find the co-

Baffour Ba Series , Further Mathematics for Schools Page 76


Exercises 4.4
1. The rule of a relation is “is four less than
twice”. Given the domain {5, 6, 7, 11, 13}, ( , . /
find the co-domain. Draw a diagram of the
relation. Function
2. Given the domain {2, 4, 6, 8, 10}, find the
co-domain, if the rule of the relation is “five
more than thrice”. Represent the relation in a . / ( ,
diagram.
Not a function
3. Given the co domain {13, 14, 15, 17} for the
relation “is seven more than of”;
Function Notation
i. find the respective elements of the domain,
In mathematics, functions are often given by
ii. show the diagram of the relation
rules or equations to define the relationships
between two or more variables. For example,
4. The co-domain of a relation is {25, 49, 81,
the equation y = 2x defines the set of ordered
100}. If the rule of the relation is “is a square
pairs such that the y – value is twice the x –
of”, determine the domain of the relation, for
value.
all positive integers and draw a diagram of the
relation.
When a function is defined by an equation, a
function notation is used. For example, the
Functions
equation y = 2x can be written in function
A function is a special kind of relation in which
notation as: f(x) = 2x, where f is the name of the
each element of the domain matches to one and
function, x is an input from the domain of the
only one element of the co – domain. Thus,
function, and f(x) is the function value (y –
function f from set x to a set y is a
axis) corresponding to x.
correspondence that assigns to each element of
x, exactly one element of y. For example, to
f(x) is read as “f of x” or the “value of the
each book in the library, there corresponds a
function f at x”.
number of pages in the book and to each human
being there corresponds a birth date. The names of functions are often given by a
lowercase or uppercase letters, such as f, g, h, p,
Each correspondence in the above examples k and M.
involves two sets x and y.
 x denotes the set of books in the library Representation of Functions
and y, the number of pages. The element 1. Formulas (rule).
of x is the argument of f. The set y is the 2. Ordered pairs (x, y).
domain of the function. 3. Set builder notations. e.g. {x, y: x + y = 6}

Baffour Ba Series , Further Mathematics for Schools Page 77


Note: Solution
As ordered pairs, no two different pairs can f(x) = 3x + 7
have the same x coordinate. f(y) = 3y + 7
For one – to – one function;
Types of Function f(x) = f(y) ⇒ x = y
One – to– one Function 3x + 7 = 3y + 7
A one to one (injective) function f from set X to 3x = 3y
set Y is a function such that x in X is related to x=y
a different y in Y. The funcition is one – to – one.

In a one – to – one function, each member of


2. Show that the function f(x) = is one – to
the domain corresponds to just one member of
the range, and each member of the range one.
corresponds to just one member of the domain.
Functions that are one – to – one are called Solution
invertible functions. f(x) =
f(y) =
One to one simply means we never get the
same “y” value twice. For one – to – one function
f(x) = f(y) ⇒ x = y
Illustrative Examples =
The function below is one – one.
(x – 3) (y + 2) = (x + 2) (y – 3)
xy + 2x – 3y – 6 = xy – 3x + 2y – 6
2x – 3y = – 3x + 2y
( ,
2x + 3x = 3y + 2y
5x = 5y
( )
This function below is not one – one. x=y
The functuion is one- to – one.
( + . /
3. Why is f : R → R given by f(x) = x2 not a one
This is because as ordered pairs, we have: – to – one?
(1, a), (2, a), (3, b). “a” appears twice as y
Solution
value and that is a violation of the one – to –
f(x) = x2
one rule.
f(y) = y2
Proof of one - to - one functions
Worked Examples For one – to – one function;
1. If f : R →R is given by f(x) = 3x + 7, proof it f(x) = f(y) ⇒ x = y
is one to one. x2 = y2

Baffour Ba Series , Further Mathematics for Schools Page 78


x = y or x = - y at most one point. For example, the function
x = - y violates the definition of onoe – to – one whose graph is sketched below is not one – to –
function. Therefore f(x) = x2 is not one to one. one, since a ≠ b, but f(a) = f(b). The horizontal
line y = f(a), intersects the curve in more than
4. Given that A = {3, 4, 5} and B = {1, 3, 5, 6} one point, and as such y = f(x ) is not a one – to
Show that f : A → B for f (x) = 2x – 5 is a one – one function.
to one mapping. y
y= f(x)
Solution y = f(a)

A = {3, 4, 5} and B = {1, 3, 5, 6}


f (3) = 2(3) – 5 = 1
f (4) = 2(4) – 5 = 3 f(a) f(b)
f (5) = 2(5) – 5 = 5
x
a b
f(x) = 2x - 5
( + ( , Exercises 4.6
1. If f(x) = 3x + 2, proof graphically and
algebraically that f is one – to – one.
A B
As ordered pairs, the function can be expressed 2. If g(x) = x2 – 3, proof graphically and
as (3, 1), (4, 3), (5, 5). Since no y value appears algebraically that g is not one – to – one.
twice, the function, f(x) = 2x – 5 is a one – to –
one function. Onto Functions
A function f : X →Y is said to be onto
Exercises 4.5 (subjective) if for every y in Y, there is an x in
Determine whether f is one – to one. X such that f(x) = y. This can be restated as a
1. f(x) = 3x – 7 2. f(x) = 3 function f from a set X to a set Y is surjective
2
3. f(x) = x – 9 4. f(x) = x2 + 4 (or onto) if every element y in Y has a
5. f(x) = 2x3 – 4 6. f(x) = corresponding element x in X, such that f(x) = y

B. Determine whether f is one – to one. Generally, a function is onto when its image
1. f(x) = 3
2. f(x) = - x + 3x – 22 equals to its range, meaning all elements in the
co domain are used .i. e. f(X) = Y.
3. f(x) = √ 4. f(x) = √
5. f(x) = √ 5. f(x) = ( )
: ; ( +
Horizontal Line Test for One – to – one
A function f is one – to – one if and only if A Onto B
every horizontal line intersects the graph of f in

Baffour Ba Series , Further Mathematics for Schools Page 79


The fig I above represents a function that is 6.
onto because all elements in B are used.
( + ( ,
Co domain = (4, 2, 5) and Range = (4, 2, 5)
Co domain = Range
Increasing Function
A function is increasing when the y – value
4 5 increases as the x – value increases.
x
( )
A “Not onto” B
f(x2)
The fig II. above represents a function that is +
not unto because all elements in B are not used. f(x1)

Co domain = (4, 8, 2, 5, 1), Range = (4, 2, 5)


x
Co domain ≠ Range x1 x2
+
Exercises 4.7
Which of the following diagrams represents If we can‟t plot the graph to see whether it is
a one to one function or onto function or increasing or decreasing, then we need an
both? algebra test.
1.
( + . / For a function y = f(x);
When x1 < x2 then f(x1) ≤ f(x2), then the function
2. is an increasing function..
( + ( + When x1 < x2 then f(x1) < f(x2), then the function
is strictly increasing function.

3. A function that is increasing throughout its


: ; ( +
domain is one – to – one.
A function may also increase within a certain
4. interval.
( + ( , y
f(x)
5.

: ; ( ,
x
Interval

Baffour Ba Series , Further Mathematics for Schools Page 80


Decreasing Function 2. f is decreasing on I if f(a) > f(b) for all a < b
A function is decreasing when the y – value on I.
decreases as the x –value increases. 3. f is constant on I if f(a) = f(b) for all a and b
on I.
x

Note
1. A function is increasing on an interval if it
f(x1 f(x) goes “uphill” from left to right.
- 2. A function is decreasing on an interval if it
)f(x2
) goes “downhill” from left to right.
x1 x2 x
3. A function is constant on an interval if it is
+
“level” or “flat”.
For a function y = f(x);
When x1 < x2 then f(x1) ≥ f(x2), then the function Even Functions
is a decreasing function.. A function f is even if the graph of f is
symmetric with respect to the y – axis. This
When x1 < x2 then f(x1) > f(x2), then the function means the graph remains unchanged after
is strictly decreasing function. reflection about the y – axis.
A function that is decreasing throughout its
domain is one – to – one. f(x) = x2 is an example of an even function.
A function may also decrease within a certain
interval.

Constant Function
A constant function is a horizontal line.
y
Algebraically, f is even if and only if f(-x) = f(x)
C f(x) = C or f(x) – f(-x) = 0, for all x in the domain of f.
This test can also be done with numbers. If f(x)
and f(-x) gives the same value , then the
x x function is even.
2
Intervals over which a function is Increasing, Other examples of even functions are /x/, x4,
Decreasing or Constant. cos x, cosh (x)
Let I be an open interval in the domain of a
function, f. then; Odd Functions
1. f is increasing on I if f(a) < f(b) for all a < b A function, f is odd if the graph of f has
on I. rotational symmetry with respect to origin. This
means that its graph remains unchanged after

Baffour Ba Series , Further Mathematics for Schools Page 81


rotation of 1800 about the origin. Algebraically, Therefore, the function is even.
f is odd if and only if f(-x) =
-f(x) for all x in the domain of f. Method 3
f(x) = -3x2 + 4
f(x) = x3 is an example of an odd function. f(2) = -3(2)2 + 4 = -8
f(-2) = -3(-2)2 + 4 = -8
f(2) = f(-2) = -8
The function is even.

2. Determine algebraically whether f(x) = 2x3 –


4x is even, odd or neither.
Algebraically, f is odd if and only if f(-x) =
-f(x) or f(x) + f(-x) = 0, for all x in the domain of f. Solution
Method 1
This test can also be done with numbers. If f(x) f(x) = 2x3 – 4x
and f(-x) gives the same value, but opposite f(-x) = 2(-x3) – 4(-x)
signs then the function is odd. = -2x3 + 4x

Other examples of odd functions are x, x3, sin x, f(x) ≠ f(-x)


sinh (x) or any linear combination of these. 2x3 – 4x ≠ -2x3 + 4x
But all the signs switched. Therefore, f(x) is
Worked Examples odd.
1. Determine algebraically whether f(x) = -3x2 +
4 is even, odd or neither.
Method 2
f(x) + f(-x) = 0 is odd
Solution f(x) = 2x3 – 4x
Method 1 f(-x) = 2(-x3) – 4(-x)
f(x) = -3x2 + 4
= -2x3 + 4x
f(-x) = -3(-x)2 + 4
f(-x) = -3x2 + 4
f(x) + f(-x) = [2x3 – 4x] + [-2x3 + 4x] = 0
⇒ f(x) = f(-x) = -3x2 + 4
Therefore, f(x) is odd.
Therefore, the function is even.
Method 3
Method 2 f(x) = 2x3 – 4x
f(x) – f(-x) = 0 is even f(-3) = 2(-33) – 4(-3)
f(x) = -3x2 + 4 f(-3) = -54 + 12
f(-x) = -3(-x)2 + 4 f(-3) = - 42
= -3x2 + 4
f(x) – f(-x) = [-3x2 + 4] – [-3x2 + 4] = 0 f(3) = 2(33) – 4(3)

Baffour Ba Series , Further Mathematics for Schools Page 82


= 54 – 12 a. f(x) = 3x4 – 2x2 + 5
= 42 f(-x) = 3(-x)4 – 2(-x)2 + 5
= 3x4 – 2x2 + 5
f(3) + f(-3) = 42 – 42 = 0 ⇒ f(x) = f(-x) = 3x4 – 2x2 + 5
Therefore, f(x) is odd. Therefore, f(x) is even.

3. Determine algebraically whether f(x) = 2x3 - b. f(x) = 2x5 – 7x3 + 4x


3x2 – 4x + 4 is even, odd or neither. f(-x) = 2(-x)5 – 7(x)3 + 4(-x)
f(-x) = -2x5 + 7x3– 4x
Solution
f(x) = 2x3 -3x2 – 4x + 4 f(x) + f(-x) = [2x5 – 7x3 + 4x] + [-2x5 + 7x3 – 4x]
f(-x) = 2(-x)3 -3(-x)2 – 4(-x) + 4 =0
= -2x3 – 3x2 + 4x + 4 Therefore, f(x) = 2x5 – 7x3 + 4x is an odd
function.
f(x) ≠ f(-x)
2x3 -3x2 – 4x + 4 ≠ -2x3 – 3x2 + 4x + 4 c. f(x) = x3 – x2
So the function is not even. f(-x) = (-x3 ) – (-x)2
f(-x) = - x3 – x2
Comparing f(x) and f(-x), the signs are not Comparing f(x) and f(-x),
switched. Therefore the function is not odd. f(x) ≠ f(-x). Therefore , the function is not even.
Also, f(x) + f(-x) ≠ 0, therefore the function is
Method 3 not odd.
f(x) = 2x3 – 3x2 – 4x + 4
f(-2) = 2(-2)3 –3(-2)2 – 4(-2) + 4 Exercises 4.8
= -16 – 12 + 8 + 4 Determine algebraically whether f(x) is even,
= -16 odd or neither.
1. f(x) = 2. f(x) = x6 + x4 – x2
3 2
f(2) = 2(2) – 3(2) – 4(2) + 4 3. f(x)= x /x/ 4. f(x) = /x/ – 1
= 16 – 12– 8 + 4
= -32 5. f(x) = 6. f(x) =
f(x) is neither even nor odd.
B. Determine whether f is even, odd, or
4. Determine whether f(x) is even, odd or neither even nor odd.
neither. 1. f(x) = 5x3 +2x 6. f(x) = 7x5 – 4 x3
a. f(x) =3x4 – 2x2 + 5 2. f(x) = 8x3 +3x2 7. f(x) = 12
b. f(x) = 2x5 – 7x3 + 4x 3. f(x) = 3x + 2x – 5 8. f(x) = 3x2 – 5x + 1
4

c. f(x) = x3 – x2 4. f(x) = x3 – 9. f(x) √


5. f(x) = √
Solution

Baffour Ba Series , Further Mathematics for Schools Page 83


Linear Function Worked Examples
A function that can be written in the form: Determine if the function is linear, quadratic or
f(x) = mx + c is a linear function. constant.
1. The domain of every linear function is R. 1. f(x) = - 4 2. f(x) = x2 + 3x + 2
2. The range is also R. 3. f(x) = 7 – 2x 4. f(x) =
3. The graph is a straight line with gradient m
and y – intercept (0, c).
Solution
1. f(x) = - 4 is a constant function because it is
written in the form f(x) = c , where c = - 4

2. f(x) = x2 + 3x + 2 is a quadratic function


because it is of the form f(x) = ax2 + bx + c

3. f(x) = 7 – 2x is a linear function because it


Quadratic Function
is written in the form f(x) = mx + c. The
A function that can be written in the form:
function can also be written as f(x) = -2x + 7
f(x) = ax2 + bx + c , where a, b and c are
where m = -2 and c = 7.
constants and a ≠0 . The graph of a quadratic
function is in the shape of a parabola. The
4. f(x) = is a linear function because it is
leading coefficient, a, determines the direction
of the parabola. written in the form f(x) = mx + c. The function
can also be written as f(x) = x + where m =
If a > 0, the parabola opens upward. In normal and c = 1.
language, it is said to have a ∪ shaped curve or
a minimum curve. For example, f(x) = x2 Exercises 4.9
Determine if the function is linear, quadratic
If a < 0, the parabola opens downward. In or constant,
normal language, it is said to have an ∩ shaped 1. f(x) = -x2+ 4x + 12 2. f(x) = 2x2+ 3x + 11
curve or a maximum curve. For eg. f(x) = -x2 3. f(x) = - 3x – 7 4. f(x) = -x – 3
a<0 a>0 5. f(x) = -x2+ 4x + 12 6. f(x) =
Maximum curve Minimum curve
7. f(x) = + 8. f(x) = –3
9. f(x) = x + 10. f(x) = √

Constant Functions
1. The domain of every quadratic function is R. A constant function is a function f that is given
2. The range is [k, ) if a > 0 and (- , k] if by f(x) = c, where c is a constant (real number).
a < 0, where k is the y – coordinate of the 1. The domain of every constant function is R
vertex. 2. The range is a singleton: {c}

Baffour Ba Series , Further Mathematics for Schools Page 84


3. The graph is a horizontal line whose y – The principal square root function , denoted by
intercept (0, c). sqrt, is the function given by;
y Sqrt (x) = √
1. The domain of a square root function is [0,
C f(x) = C )
2. The range is also [0, ).

x x x Exponential Function
1 2 Let b be a positive real number different from
Polynomial Functions 1. The exponential function with base b,
A function f given by f(x) = anxn + an - 1 xn - 1 denoted by expb, is the function given by;
+… + a1x + a0, where a0, a1…an are constants expb (x) = bx
with an ≠ 0, is called a polynomial function of 1. The domain of every exponential function is R.
degree n. 2. The range of every exponential function is
(0, ).
If n = 0, f is a constant function. 3. The y – intercept of the graph of every
If n = 1, f is a linear function. exponential function is (0, 1). This is because
If n = 2, f is a quadratic function. b0 = 1.
1. The domain of every polynomial function is R.
2. The are three possibilities for the range Graphs of Function
a. if the degree is odd, then the range of f is R. Every graph represents a set of ordered pairs,
b. if the degree is even and positive, then; but not every graph is a graph of a function. For
i. ran(f) = [k, ) if an > 0; example, the circle below is not a graph of a
ii. ran(f) = (- , k] if an < 0; function.
where k is the y – coordinate of the lower point y

for case (i) or the highest point for case (ii), of 3


the graph, to be treated later. 2

1
Rational Functions x
-3 -2 -1 1 2 3
A rational function is a function f in the form; -1
( ) -2
f(x) = ;
( )
-3
where p and q are polynomial functions.
e.g. f(x) = This is because the ordered pairs (0, 3) and
(0, -3) on the graph have the same first
Square - root Function coordinate and different second coordinate.
The principal square root of x , denoted by √ , is
defined to be the positive square root of x. On the other hand, the graph below represents a
function.

Baffour Ba Series , Further Mathematics for Schools Page 85


y
4.
y = 2x - 1
x

5.
y

This is because no two ordered pairs have the x


same x coordinate.

Identifying the Graph of a Function 6. y


The Vertical Line Test
If there is a vertical line that crosses a graph
twice (or more), then we have two points (or
x
more) with the same x coordinate and different
y coordinate and so the graph is not a graph of a
function. If every possible vertical line does not
cross the graph more than once, then the graph
is a graph of a function. Domain and Range of a Function
Consider a function defined by the equation
Worked Examples y = f(x). The domain of f is the set of all x –
Which of the following is a graph of a values that when substituted into the function,
function? produce a real number. The range of f is the set
y
1. of all y – values corresponding to the values of
x in the domain.

x
To find the domain of a function defined by
y = f(x);
y I. Exclude values of x that make the
2. denominator of a fraction zero.
x II. Exclude the value of x that makes a negative
value within the square root. That is to say that
y the number under a root sign must be positive.
3.
Worked Examples
x
Type 1 : Without root sign
1. Find the domain of the function and write
your answer in interval notation.
a. f(x) = b. h(x) = c. g(t) = t2 – 3t
Baffour Ba Series , Further Mathematics for Schools Page 86
Solution ii. g(x) =
a. The function will be undefined when the
x–2=0
denominator is zero.
x=2
2x – 1 = 0
When x = 2, g(x) ceases to exist.
2x = 1
Therefore, the domain of g(x) is all real
x= numbers except x = 2
Domain of f is {x : x R, x ≠ } Domain of g is {x : x R, x ≠ 2}

Interval notation : . , /∪. , / Type 2 : Domain of a Squqre Root Function


I. Set the expression inside the square root
-3 -2 -1 0 1 2 greater than or equal to zero.
II. Solve the equation found in step I. In some
b. The function will be undefined when the cases, divide through by a negative number to
denominator is zero. reverse the direction of the inequality.
But the quantity x2 is greater than or equal to 0
for all real numbers x, and the number 9 is Worked Examples
positive. The denominator of the function, 1. Find the domain of the function k(t) = √
h(x) = can never be zero.
The domain is the set of all real numbers. Solution
Domain of h is {x : x R} 1. The function defined by k(t) = √ will
not be a real number when t + 4 is negative;
Interval notation : ( , ) hence the domain is the set of all t values that
make the radicand greater than or equal to
-3 -2 -1 0 1 2 zero.
t+4≥0
c. The function defined by g(t) = t2 – 3t has no t≥-4
restrictions on its domain because any real Domain = {t : t R, t ≥ - 4 }
number substituted for t will produce a real Interval notation [- 4, ]
number. The domain is the set of all real
numbers. [
-4 -3 -2 -1 0 1
2. For each of the following functions, find its 2. Determine the domain of the function
domain. f(x) = - √
i. f(x) = x2 + 3 ii. g(x) =
Solution
Solution f(x) = - √
i. Since f(x) = x2 + 3 is defined for all real Set the inside greater than or equal to zero,
numbers x, the domain of f is R. -2x + 3 ≥ 0

Baffour Ba Series , Further Mathematics for Schools Page 87


-2x ≥ -3 Steps to find the Range of a Function
x≤ To find the range of a function f described by
the formula, where the domain is taken to be
Domain of f is {x : x R, x ≤ }
the natural domain;
1. Put y = f(x)
3. For the function h(x) = √ , find the 2. Solve x in terms of y.
domain. 3. The range of f is the set of all real y such that
x can be solved.
Solution
h(x) = √ Worked Examples
h(x) is defined if 1 + 5x ≥ 0; For each of the following functions, find its
1 + 5x ≥ 0 range.
5x ≥ - 1 1. f(x) = x2 + 2 3. h(x) = √
x≥ 2. g(x) = 4. f(x) =
Domain of h is {x : x R, x ≥ }
Solution
Interval notation = [ , )
1. f(x) = x2 + 2
Put y = f(x) = x2 + 2
Exercises 4.10
y = x2 + 2; (Solve for x)
A. Write the domain of the function.
x2 = y – 2
1. f(x) = 4. p(x) =
x=±√
2. f(x) = 5. h(x) = x + 6
Note that x can be solved if and only of
3. k(x) = 6. f(p) =
y – 2 ≥ 0.
y≥2
B. Write the domain in of each function. The range of f is {y R : y ≥ 2}
1. f(p) = 4. g(x) = √
2. f(x) = √ 5. f(x) = √ 2. g(x) =
3. f(t) = 2t2 + t – 1 6. h(t) = t3 + t – 1 Put y = g(x) =
y= (Solve for x)
C. Find the domain of each .
1. f(x) = x2 – 5 5. f(x) = x–2 =

2. f(x) = 6. f(x) = –√ x = +2

3. f(x) = 7. f(x) = +√ Note that x can only be solved if and only if


y≠0
4. f(x) = 8. f(x) = √ The range of g is {y R : y ≠ 0}

Baffour Ba Series , Further Mathematics for Schools Page 88


3. h(x) = √ Express the given function in vertex form by
Put y = h(x) = √ completing the square.
y=√ ,y≥0 (Solve for x)
f(x) = - 2x2 + 4x – 7
f(x) = – 2[(x2 – 2x + (1)2 – (1)2] – 7
y2 = 1 + 5x
f(x) = -2[(x2 – 1)2 – 1] – 7
5x = y2 – 1
f(x) = -2 (x – 1)2 + 2 – 7
x= f(x) = - 2(x – 1)2 – 5
The range of h is {y: y R}
4. f(x) = Now, the domain of the function is on all R.
This implies that (x – 1)2 is either zero or
Put y = f(x) =
positive (because of the square). Hence set :
y= (x – 1)2 ≥ 0
y(x2 + 1) = 2x + 1 (Solve for x)
yx2 + y = 2x + 1 Multiply both sides by -2;
yx2 – 2x + (y – 1) = 0 - 2 (x – 1)2 ≤ 0

Solving by quadratic formula; Add -5 to both sides of the inequality;


a = y , b = -2 and c = y - 1 - 2 (x – 1)2 – 5 ≤ - 5
√ f(x) ≤ - 5
x= Thus, the range of the function is (x : x R,
x=
( ) √( ) ( ) x ≤ - 5 ) or ( , )

√ ( )
x= Exercises 4.11
A. Find the range of the following.

x= 1. f(x) = x2 – 5 4. f(x) = 3 -

x= 2. f(x) = 5. f(x) =

√ 3. f(x) = 6. f(x) =
x=

Note that x can be solved if and only if B. Find the range defined by the following
1 – y2 – y ≥ 0. That is y2 – y – 1 ≥ 0 1. f(x) = x2 – 2x – 3
The range of f is {y R, y2 – y – 1 ≥ 0 or y ≥ 0} 2. f(x) = 2x2 – 10x + 19
3. f(x) = 2x2 +10x + 16
4. Find the range of the function defined by
f(x) = - 2x2 + 4x – 7 Evaluating a Function
A function may be evaluated at different values
Solution of x, by substituting x – values from the domain
f(x) = - 2x2 + 4x – 7 into the function. For example, to evaluate the

Baffour Ba Series , Further Mathematics for Schools Page 89


function defined by f(x) = 2x at x = 5, substitute 3. Given that f(x) = , find:
x = 5 into the function.
i. find f(2), f(- 4), f(x – 1)
ii. for what value of x is f(x) = 3?
f(x) = 2x
Solution
f(5) = 2(5) i. f(x) =
( )
f(5) = 10 f(2) = =

Worked Examples ( )
f(- 4) = = =7
1. Given the function defined by g(x) = x – 1,
find the function values; ( )
i. g(4) ii. g(-2) f(x – 1) = = =
( )

Solution ii. =3
i. g(x) = x – 1, 4x + 1 = 3(x + 2)
g(4) = (4) – 1, 4x + 1 = 3x + 6
=1 4x – 3x = 6 – 1
x=5
ii. g(-2) = (-2) – 1
4. Given that f(x) = (x – 1)2 (x + 3) – 30,
= -2 evaluate ;
i. f(3) ii f(4)
2. Given the functions defined by f(x) = x2 – 2x iii. Hence estimate the value of x for which f(x) = 0
and g(x) = 3x + 5, find the function values:
i. f(t) ii. g(w + 4) iii. f(-t) Solution
f(x) = (x – 1)2 (x + 3) – 30 , evaluate ;
Solution f(3) = (3 – 1)2 (3 + 3) – 30 ,
i. f(x) = x2 – 2x = 22 × 6 – 30
f(t) = t2 – 2t = 4 × 6 – 30
=-6
ii. g(x) = 3x + 5
g(w + 4) = 3(w + 4) + 5 f(4) = (4 – 1)2 (4 + 3) – 30 , evaluate ;
= 3w + 12 + 5 = 32 × 7 – 30
= 3w + 17 = 9 × 7 – 30
= 33
iii. f(x) = x2 – 2x
f(-t) = (-t)2 – 2(-t) f(x) = (x – 1)2 (x + 3) – 30
= t2 + 2t f(x) = 0
Baffour Ba Series , Further Mathematics for Schools Page 90
⇒ (x – 1)2 (x + 3) – 30 = 0 i. g(-x) ii. g(x + h)
(x – 1)2 (x + 3) = 30
(x – 1)2 = 30 or (x + 3) = 30 2. Given that f(x) = x2, express as simply as
x–1=±√ or x + 3 = 30 possible:
( ) ( )
x = 1 + √ or x = 1 – √ or x = 30 – 3 = 27 a. f(5 + h) b.

5. Let g(x) = x2 – 3x + 7. Find the following: 3. Given that f(x) = x2, find :
i. g(10) ii. g(a +1 ) iii. g(r2) i. f. / ii. f(5a) iii. f. /
( ) ( )
iv. g(x + h) v.
4. A. f(x) is defined by f(x) = 1 + – x. Write
Solution down the values of ;
g(x) = x2 – 3x + 7. i. f(1) ii. f(1.5) iii. f(2)
i. g(10) = 102 – 3(10) + 7 iv. Hence, estimate correct to one decimal
= 77 place, the value of x between 1 and 2 for which
f(x) = 0
ii. g(a +1 ) = (a + 1)2 – 3(a + 1) + 7
= (a + 1) (a + 1) – 3(a + 1) + 7
= a2 + 2a + 1 – 3a – 3 + 7 B. 1. Let f(x) = , find the following;
= a2 + 2a – 3a + 1 – 3 + 7 a. f(2) b. f(3.5) c. f(a + 1)
= a2 – a + 5 d. f(√ ) e. f(a2) f. f(a) + f(1)

iii. g(r2) = (r2)2 – 3r2 + 7 2. Let f(x) = and g(x) = √ . Find the
= r4 – 3r2 + 7
following:
iv. g(x + h) = (x + h)2 – 3(x + h) + 7 a. f(1) + g(1) d. f(a – 1) + g(a + 1)
= (x + h) (x + h) – 3(x + h) + 7 b. f(2)g(2) e. f(a2 + 1)g(a2 + 1)
( )
= x2 + 2xh + h2 – 3x – 3h + 7 c. ( )

( ) ( ) ( )
v. = 3. Let f(x) = x2 – 3x + 4. Find and simplify the
following:
= ( ) ( )
i. f(a + b ) ii.
= ( ) ( )
iii.
= 2xh + h – 3

Exercises 4.12 C. 1. Let f(x) = and g(x) = ,


A. 1. Given a function defined by g(x) = 4x – 3, find:
find the function values: i. f(1)g(1) ii. f(3) g(3)
iii. h(x) = f(x).g(x) iv. h(1) v. h(3)

Baffour Ba Series , Further Mathematics for Schools Page 91


2. Let f(x) = and f(x) = , find: This means 1.5 seconds after the ball was
dropped; it bounced back a height of 18 feet
i. f(1)g(1) ii. f(-2) g(-2)
from the ground.
iii. h(x) = f(x).g(x) iv. h(1) v. h(-2)
2. f(x) denotes the sum of all positive integers,
3. Let f(x) = and f(x) = , find: except x itself which divides exactly into x. For
( ) ( ) ( ) example, f(4) = 1 + 2 = 3. Find:
i. ii. iii. h(x) =
( ) ( ) ( )
i. f(6) ii. f (28) iii. f (29)
iv. the values of x for which h(x) is undefined.
Solution
4. Let f(x) = and f(x) = , find: i. f (6) = 1 + 2 + 3 = 6
( ) ( ) ( ) ii. f (28) = 1 + 2 + 4 + 7 + 14 = 28
i. ii. iii. h(x) =
( ) ( ) ( ) iii. f (29) = 1
iv. the values of x for which h(x) is undefined.
3. R(x) denotes the remainder when x is divided
Word Problems by 5. Given that k = 4, write down the value of:
Sometimes, a function may be stated and i. R(3k) ii. R(5k) iii. R(k2)
defined in words. Carefully identify the
function and work out for the required value. Solution
i. R(3k)
Worked Examples When k = 4, R(3×4) = R(12)
1. The height (in feet) of a ball that is dropped R(12) denotes the remainder when 12 is divided
from an 80 – ft building is given by; by 5. Hence, R(12) = R(3k) = 2
h(t) = -16t2 + 18, where t is the time in seconds
after the ball is dropped. ii. R(5k)
a. Find h(1) and h(1.5). when k = 4, R(5 × 4) = R(20)
b. Interpret the meaning of the function R(20) denotes the remainder when 20 is divided
values obtained in (a). by 5. Hence, R(20) = R(5k) = 0

Solution iii. R(k2) = R(42) = R(16)


a. h(t) = -16t2 + 18 By the definition;
h(1) = -16(1)2 + 18 = 2 R(k2) = 1
h(1.5) = -16(1.5)2 + 18 = -18
Exercises 4.13
b. h(1) = 2 A. 1. A ball is dropped form a 50 – m building.
This means 1 second after the ball was The height in meters after t seconds is given by
dropped; it reached a height of 2 feet from the h(t) = - 4.9t2 + 50.
building. a. Find h(1) and h(1.5).
h(1.5) = -18

Baffour Ba Series , Further Mathematics for Schools Page 92


b. Interpret the meaning of the function
values obtained in (a). Terminology Function Value
Sum f + g (f + g)(x) = f(x) + g(x)
2. If Jimmy rides a bicycle at an average speed Difference f – g (f – g)(x) = f(x) – g(x)
of 11.5mph, the distance that he rides can be Product fg (fg)(x) = f(x)g(x)
( )
represented by d(t) = 11.5t, where t is the time Quotient . /(x) = , g(x) ≠ 0
( )
in hours.
a. Find d(1) and d(1.5). Note:
b. Interpret the meaning of the function 1. The domains of f + g, f – g and fg are the
values obtained in (a). intersection, I of the domains of f and g (that is,
the numbers that are common to both domains.
3. An object is thrown up and its height h(t) in
meters after t seconds is given by h(t) = 1 + 4t – 2. While it is true that (f + g)(x) = f(x) + g(x),
5t2. remember that f(a +b) ≠ f(a) + f(b)
a. When will the object hit the ground?
b. Find the maximum height obtained by Worked Examples
the object. 1. If f(x) =3x – 2 and g(x) = x2, find;
i. (f + g)(2) ii. (f – g)(2)
4. Michael walks an average of 5.9km/hr. The
iii. (fg)(2) iv. . / (2)
distance that he walks can be represented by
d(t) = 5.9t, where t is the time in hours.
a. Find d(1) and d(2) Solution
b. Interpret the meaning of the function If f(x) =3x – 2 and g(x) = x3,
values obtained in (a). i. (f + g)(2) = f(2) + g(2)
= [3(2) – 2] + [23]
B. 1. F(x) denotes the number of factors of a = 6 – 2 + 8 = 12
positive integer x, including 1 and x itself. For
example, 1, 2, 3, 6 are the factors of 6. Hence ii. (f – g)(2) = f(2) – g(2)
F(6) = 4. Write down any x, 10 < x < 100, for = [3(2) – 2] – [23]
which: i. F(x) = 2 ii. F(x) = 3 = 6–2–8=-4

2. R(x) denotes the remainder when x is divided iii. (fg)(2) = f(2) g(2)
by 7. Given that k = 9, write down the value of = [3(2) – 2] [23]
= (6 – 2)(8) = 32
i. R(3√ ) ii. R(8k) iii. R(2k2)
( ) ( )–
Operations on Functions iv. . / (2) = = = = =
( )
In general, if f and g are any functions, the
terminology and notations in the following 2. If f(x) = √ and g(x) = 3x + 1, find;
chart are used. i. (f + g)(2) ii. (f – g)(2) iii. (fg)(2)

Baffour Ba Series , Further Mathematics for Schools Page 93


iv. . / (2), and state the domain of the 4. . / (x), 5. (f + g)(3) 6. (f + g)(2)
respective functions. 7. (f-g)(-3) 8. (f-g)(-2) 9. (fg)(-1)
10. (fg)(-2) 11. . / (4) 12. . / (-2),
Solution
Consider f(x) = √ B. Find:
Domain = {x : -2 ≤ x 2} a. i. (f + g)(x) ii. (f – g)(x)
iii. (fg)(x) iv. (f/g)(x)
Consider g(x) = 3x + 1
Domain = {x : x R} b. the domain of f + g, f – g , and fg
c. the domain of f/g
The intersection of these domains is {x : -2 ≤ 1. f(x) = x2 + 2 , g(x) = 2x2 – 1
x 2}, which is the domain of (f + g)(2), 2. f(x) = x2 + x , g(x) = x2 – 3
(f – g)(2) and (fg)(2). 3. f(x) = √ , g(x) = √
4. f(x) = √ , g(x) = √
For the domain of , we exclude each number 5. f(x) = , g(x) =
x in -2 ≤ x 2 such that g(x) = 3x + 1 = 0
6. f(x) = , g(x) =
(namely x = . Thus we have the following;
i. (f + g)(x) Composite Function
= f(x) +g(x) The composite function f○g of two functions
=√ + (3x + 1), -2 ≤ x 2 f and g is defined by (f○g)(x) = f(g(x)).

ii. (f – g)(x) The domain of f○g is the set of all x in the


= f(x) – g(x) domain of g such that g(x) is in the domain of f.
=√ – (3 x + 1), -2 ≤ x 2 f○g
g
iii. (f g)(x) 
= f(x)g(x) x f

=√ (3 x + 1), -2 ≤ x 2 g(x)
Domain of g 
Domain of f
f(g(x))

iv. . / (x) = , -2 ≤ x 2 and x ≠ The diagram illustrates relationships among f, g
and f○g. Note that for x in the domain of g, first,
Exercises 4.14 find g(x) (which must be in the domain of f)
A. Let f(x) = 4x – 3 and g(x) = x2 – 2x. Find then, second, find f(g(x)).
the following:
For the composite function g○f, the order is
1. (f + g)(x) 2. (f – g)(x) 3. (f.g)(x)
reversed, first finding f(x) and second finding

Baffour Ba Series , Further Mathematics for Schools Page 94


g(f(x)). The domain of g○f is the set of all x in a. Find (f○g)(x) and the domain of f○g.
domain of f such that f(x) is in the domain of g. b. Find (g○f)(x) and the domain of g○f.
Since the notation g(x) is read “g of x” we c. Find (f○g)(2)
sometimes say that g is a function of x. For the
composite function f○g, the notation f(g(x) is Solution
read “f of g of x” and f is regarded as a function a. f(x) = x2 – 1 and g(x) = 3x + 5
of g(x). In this sense, a composite function is a (f○g)(x) = f(g(x))
function of a function, or more precisely, a = f(3x + 5)
function of another function‟s value. = (3x + 5)2 – 1
= (3x + 5) (3x+ 5) – 1
Worked Examples = 9x2 + 30x + 24
1. Let f(x) = 3x – 2 and g(x) = x2 + 2x. Find : Domain of f○g = {x : x R}
a. (g○f )(2) b. (f○g )(2) c. (g○f )(x) d. (f○g )(x)
b. (g○f)(x) = g(f(x))
Solution = g(x2 – 1)
a. (g○f )(2) = g(f(2)) = 3(x2 – 1) + 5
f(2) = 3(2) – 2 = 4 = 3x2 – 3 + 5
(g○f )(2) = g(f(2)) = 3x2 + 2
= g(4) Domain of g○f = {x : x R}
= 42 + 2(4)
= 24 c. Method 1
f(g(2))
b. (f○g )(2) = f(g(2)) First, find g(2)
g(2) = 22 + 2(2) = 8 g(2) = 3(2) + 5 = 11

(f○g )(2) = f(g(2)) = f(8) = 3(8) – 2 = 22 Second, find f(11)


f(g(2)) = f(11) = 112 – 1 = 120
c. (g○f )(x) = g(f(x))
= g(3x – 2)
Method 2
= (3x – 2)2 + 2(3x – 2)
(f○g)(x) = f(g(x))
= 9x2 – 12x + 4 + 6x – 4
= f(3x + 5)
= 9x2 – 6x
= (3x + 5)2 – 1
= (3x + 5) (3x+ 5) – 1
d. (f○g )(x) = f(g(x))
= 9x2 + 30x + 24
= f(x2 + 2x)
= 3(x2 + 2x) – 2 ⇒(f○g)(2) = f(g(2))
= 3x2 + 6x – 2 = 9(2)2 + 30(2) + 24
= 120
2. Let f(x) = x2 – 1 and g(x) = 3x + 5

Baffour Ba Series , Further Mathematics for Schools Page 95


3. Let f(x) = x2 – 16 and g(x) = √ ii. f ○ g = 6x + 1
a. Find (f○g)(x) and the domain of f○g. f ○g(7) = 6(7) + 1
b. Find (g○f)(x) and the domain of g○f. = 43

Solution 5. If f(x) = 3x + 1, g(x) = 4x – 3, find;


a. (f○g)(x) = f(g(x)) i. f(f(2)) ii. g(g(2))
= f(√ ) iii. Show that f(x + y) = f(x) + f (y) – 1 and that
g(f(x)) – f(g(x)) = 9
= (√ ) – 16
= x – 16
Solution
By definition, the domain of f○g is the domain
f(x) = 3x + 1, g(x) = 4x – 3
of g.
i. f(2) = 3(2) + 1 = 7
Therefore; domain of f○g = {x : x R, x is
f(f(2)) = f(7) = 3(7) + 1 = 22
nonnegative}
ii. g(2) = 4(2) – 3 = 5
b. (g○f)(x) = g(f(x))
g(g(2)) = g(5) = 4(5) – 3 = 17
= g(x2 – 16)
=√
iii. f(x + y) = f(x) + f(y) – 1
By definition, the domain of g○f is the domain
Consider the L.H.S.
of g. Therefore;
f(x + y) = 3 (x + y) + 1
domain of g○f = {x : x R, x 4 or x - 4}
= 3x + 3y + 1
4. Two functions f and g are defined on the set of
Consider the R.H.S.;
real numbers by f : x → 2x + 3 and g : x →3x – 1
f(x) + f(y) – 1
i. Find the composite function of ;
(3x + 1) + (3y + 1) – 1
a. f○g b. g○f
3x + 3y + 1 + 1 – 1
ii. Find the image of 7 under the composite
3x + 3y + 1
function of f○g

Solution L.H.S. = R. H. S
f : x → 2x + 3 and g : x →3x – 1 . f(x + y) = f(x) + f (y) – 1 = 3x + 3y + 1
i. a. f ○ g = 2(3x – 1) + 3 g(f(x)) – f(g(x)) = 9
= 6x – 2 + 3 g(f(x)) = 4(3x + 1) – 3
= 6x + 1 = 12x + 4 – 3
= 12x + 1
b. g ○ f = 3(2x + 3) – 1
= 6x + 9 – 1 f(g(x)) = 3(4x – 3) + 1
= 6x + 8 = 12x – 9 + 1
= 12x – 8

Baffour Ba Series , Further Mathematics for Schools Page 96


g(f(x)) – f(g(x)) = (12x + 1 ) – (12x – 8) 8. Let F : R → R and G : R → R be given by :
= 12x – 12x +1 + 8 f(x) = sin x and g(x) = x2. Find g○f(x)
=9
Hence, g(f(x)) – f(g(x)) = 9 Solution
g○f(x) = g(f(x)).
6. f and g are the functions f : x → , = (sin x)2
= sin2 x
g : x → – 3 , write down :
i. f(-1) ii. g. / iii. gf(-1) iv. fg. / 9. Let f(x) = x + 1 and g(x) = √ . Find the
domain of g○f.
Solution
Solution
f:x→ , and g : x → – 3 ,
f(x) = x + 1 and g(x) = √ .
g(f(x)) = – 3;
⁄ g○f = √
( ( )) = x + 3 – 3 Where x + 1 ≥ 0;
( ( )) = x x ≥ -1
Domain = {x : x R, x ≥ -1}
f (g (x)) = = [-1, )
. /

f(g(x)) = =x Exercises 4.15


A. In each of the following find;
i. f(-1) = =
a. i. (f○g)(x) ii. (f○g)(-2)
ii. g. / = – 3 = 2 – 3 = -1 b. i. (g○f)(x) ii. (g○f)(3)

1. f(x) = 2x – 5 g(x) = 3x-1
iii. gf(-1) = - 1
2. f(x) = 5x + 2 g(x) = 6x + 7
iv. fg. / = 2
3. f(x) = 3x + 4 g(x) = 5x
4. f(x) = 3x – 1 g(x) = 4x2
7. If f : x → x + 4 and g : x → x2, find: 5. f(x) = 2x2+ 3x – 4 g(x) = 2x- 1
i. gf(x) ii. gf(-3) 6. f(x) = 5x – 7 g(x) = 3x2 – x + 2
7. f(x) = 4x g(x) = 2x3 – 5x
Solution 8. f(x) = x3 + 2x2 g(x) = 3x
If f : x → x + 4 and g : x → x2, find:
i. gf(x) = (x + 4)2 B. In each of the following find;
gf(x) = (x + 4) (x + 4 ) a. (f○g)(x) and the domain of f○g .
gf(x) = x2 + 8x + 16 b. (g○f)(x) and the domain of g○f .
1. f(x) = x2 – 3x g(x) = √
ii. gf(-3) = (-3)2 + 8(-3) + 16
2. f(x) = √ g(x) = x2 + 2x
gf(-3) = 9 – 24 + 16
3. f(x) = x2 – 4 g(x) = √
gf(-3) = 1

Baffour Ba Series , Further Mathematics for Schools Page 97


4. f(x) = -x2 + 1 g(x) = √ ii. If the domain is {x : 1 ≤ x ≤ 2}, find the
range.
C. In each of the following find;
a. (f○g)(x) and the domain of f○g . 2. Let f(x) = x2 and g(x) = 2x + 1. Find;
b. (g○f)(x) and the domain of g○f . i. (f○g)(x) ii. (g○f)(x)
1. f(x) = √ g(x) = √
Challenge Problems
2. f(x) = √ g(x) = √
For each of the following, find f(x) and g(x) ,
3. f(x) = √ g(x) = √
where g(x) is in the form xr with r ≠ 1 such that
4. f(x) = √ g(x) = √ (g○f)(x) equals the given expressions:
a. √ b.
D. In each of the following find;
a. (f○g)(x) and the domain of f○g .
b. (g○f)(x) and the domain of g○f . Composite Function Values from Tables
Special values of composite functions may
1. f(x) = g(x) =
sometimes be obtained from table as illustrated
2. f(x) = g(x) = x – 1 in the example below.
3. f(x) = x2 g(x) =
Worked Examples
4. f(x) = g(x) = 1. Several values of two functions f and g are
5. f(x) = g(x) = listed in the following tables.

6. f(x) = g(x) =
x 1 2 3 4
f(x) 3 4 2 1
E. Let f(x) = 2x – 3 and g(x) = x2 + 3x and
h(x) = . Find the following; x 1 2 3 4
1. (g○f)(1) 5. (f○f)(4) 9. (h○f)(5) g(x) 4 1 3 2
2. (f○g)(-2) 6. (g○h)(-1) 10. (f○h)(0)
3. (f○g)(1) 7. (h○g)(-1) 11. (g○h)(-1) Find:
4. (g○f)(-2) 8. (h○h)(3) 12. (f○g)(2.36) i. (f○g)(2) ii. (g○f)(2) iii. (f○f)(2) iv. (g○g)(2)

F. Let f(x) = 2x – 3 and g(x) = x2 + 3x and Solution


h(x) = . Find the following; From the tables,
i. (f○g)(2) = f(g(2)) = f(1) = 3
1. (g○f)(x) 2. (h○g)(x) 3. (h○f)(x) 4. (f○h)(x)
5. (g○g)(x) 6. (h○h)(x) 7. (f○f○f)(x) 8. (f○f)(x) ii. (g○f)(2) = g(f(2)) = g(4) = 2
iii. (f○f)(2) = f(f(2)) = f(4) = 1
G. 1. If f(x) = 2x – 1, g(x) = x2 – 1,
iv. (g○g)(2) = g(g(2)) = g(1) = 4
i. find g[f(x)]

Baffour Ba Series , Further Mathematics for Schools Page 98


Exercises 4.16 = 2(3x – 2) + 1
1. Several values of two functions f and g are = 6x – 4 + 1
listed in the following tables: = 6x – 3

x 5 6 7 8 (f○g)○h = 6(x + 5) – 3
f(x) 8 7 6 5 = 6x + 30 – 3
= 6x + 27
x 5 6 7 8
g(x) 7 8 6 5 Consider the R.H.S;
f○ (g○h)
Find:
g○h = g{h(x)}
i. (f○g)(6) iii. (f○f)(6)
= 3(x + 5) – 2
ii. (g○f)(6) iv. (g○g)(6)
= 3x + 15 – 2
= 3x + 13
2. Several values of two functions T and S
are listed in the following tables:
f○ (g○h) = 2(3x + 13 ) + 1
= 6x + 26 + 1
x 0 1 2 3
T(t) 2 3 1 0 = 6x + 27

x 0 1 2 3 Since L.H.S = R.H.S


S(x) 1 0 3 2 ⇒ (f○g)○h = f○ (g○h) = 6x + 27

Find: 2. Composition of functions is distributive on


1. (T○S)(1) 2. (T○T)(1) 3. (S○T)(1) 4. (S○S)(1) the left over addition. That is:
(f + g)○h = ( f○h) + (g○h)
Properties of Composite functions
1. Composition of functions is associative. That Worked Examples
is : (f○g)○h = f○ (g○h) Three functions are defined on the set of real
numbers by f(x) = 2x + 1, g(x) = 3x – 2 and
Worked Examples h(x) = x + 5. Show that (f + g)○h =( f○h) + (g○h)
Three functions are defined on he set of real
numbers by f(x) = 2x + 1, g(x) = 3x – 2 and Solution
h(x) = x + 5. Show that (f○g)○h = f○ (g○h) f(x) = 2x + 1, g(x) = 3x – 2 and h (x) = x + 5

Solution Consider the L.H.S.;


f(x) = 2x + 1, g(x) = 3x – 2 and h (x) = x + 5. (f + g)○h
(f○g)○h = f○ (g○h) (f + g) = (2x + 1) + (3x – 2)
Consider the L. H. S. = 2x + 3x + 1 – 2
f○g = f{(gx)} = 5x – 1

Baffour Ba Series , Further Mathematics for Schools Page 99


(f + g) ○ h = 5(x + 5) – 1
= 5x + 25 – 1
.
(f g)○h = 6(x + 5)2 – (x + 5) – 2

= 5x + 24 = 6(x + 5) (x + 5) – (x + 5) – 2
= 6(x2 + 10x + 25) – (x + 5) – 2
Consider the R.H.S; = 6x2 + 60x + 150 – x – 5 – 2
( f○h) + (g○h) = 6x2 + 59x + 143
f ○h = 2(x + 5) + 1
= 2x + 10 + 1 Consider the R. H. S;
= 2x + 11 ( f○h) . (g○h)
( f○h) = 2(x + 5) + 1
g○h = 3(x + 5) – 2 = 2x + 10 + 1
= 3x + 15 – 2 = 2x + 11
= 3x + 13
g○h = 3(x + 5) – 2
= 3x + 15 – 2
( f○h) + (g○h) = (2x + 11) + (3x + 13)
= 3x + 13
= 2x + 3x + 11 + 13
= 5x + 24 ( f○h) . (g○h) = (2x + 11) (3x + 13)
= 2x (3x + 13) + 11 (3x + 13)
Since L. H. S. = R. H. S; = 6x2 + 26x + 33x + 143
⇒(f + g)○h = ( f○h) + (g○h) = 5x + 24 = 6x2 + 59x + 143
Composition of functions is distributive on the left
over multiplication. That is; Since L.H.S. = R. H. S.
(f . g) ○ h = ( f ○ h) . (g ○ h)
. .
(f g)○h =( f○h) (g○h) = 6x2 + 59x + 143
Worked Examples
Three functions are defined on the set of real Decomposing a Composite Function
numbers by f(x) = 2x + 1, g(x) = 3x – 2 and h(x) We can go the other way and break up a
= x + 5. Show that (f . g)○h =( f○h).(g○h) function into a composition of other functions.
This is called decomposing a function. For
Solution
f(x) = 2x + 1, g(x) = 3x – 2 and h (x) = x + 5 example, in . / , the function can be
made from these two functions:
Consider the L.H.S.;
f(x) = g(x) = x2
(f . g)○h =
f . g = (2x + 1) (3x – 2) g○f (x) = g(f(x) = g . /=. / ,
= 2x(3x – 2) + 1(3x – 2)
= 6x2 – 4x + 3x – 2 Worked Examples
= 6x2 – x – 2 1. If f(x) = 2x – 3 and f(g(x) = 2x + 1, find g(x)

Baffour Ba Series , Further Mathematics for Schools Page 100


Solution Decompose f(x) as h(x) = 5 – x2 and g(x) = √
f(x) = 2x – 3, f(g(x) = 2x + 1, g(x) = ? Thus, g(h(x) = g(5 – x2) = √
f ○g (x) = f(g(x)
But f(x) = 2x – 3 Exercises 4.17
A. 1. Decompose the following functions into
Substitute g(x) for x the forms g f;
⇒ f(g(x) = 2 g(x) – 3 a. 6x + 3 b. 4x2
2x + 1 = 2 g(x) – 3 2. If h(x) = f(g (x)) , find the decomposition of
2x + 1 + 3 = 2 g(x) h(x) = √
2x + 4 = 2 g(x)
g(x) = + 3. Find two possible decompositions of h(x) = √

g(x) = x + 2 4. Find f(x) and g(x) such that h(x) = f ○g (x)


and h (x) = (8 – 4x)2
2. Given f(x) = x2 + 1 and (g○f ) (x) = , find 5. If fog(x) = 16x + 7 and g(x) = 2x – 1. Find
f(x)
g(x).

Solution B. 1. Find g(x) if f(x) = x + and fog(x) = 3x + 1


(g○f ) (x) = g(f(x) =
2. Find two functions f and g such that fog (x) =
g(f(x) = ( )
h(x); h(x) = Ans f(x) = g(x) = x + 1
But f(x) = x2 + 1
g(f(x) = ( ) 3. Find two functions f and g such that h(x) can
be expressed as a function h(x) = 4x + x2; f – g
Substitute f(x) = x Ans: f(x) = x2, g(x) = - 4x

g(x) =
Functional Equations
⇒ g(x) = I. Identify the given functions.
II. Workout the composite functions at the
3. Write f(x) = √ as composition of two L.H.S and equate to the value at the R.H.S.
functions. III. Solve the equation to obtain the value of the
involving variable.
Solution
f(x) = √ Worked Examples
For g and h, f(x) = g(h)(x) 1. a. Given that f(x) = (x – 2) (x – 3), solve the
equation f(x) = 6.
Now look for a function inside f(x); b. Describe briefly the range of values of x for
5 – x2 and the square root function. which f(x) 0.

Baffour Ba Series , Further Mathematics for Schools Page 101


Solution 3. Given that f(x) = + and that f(5) = 8,
a. f(x) = (x – 2) (x – 3),
calculate the value of ; i. k ii. f(4)
f(x) = 6.
⇒(x – 2) (x – 3) = 6 Solution
x(x – 3) – 2 (x – 3) = 6
f(x) = +
x2 – 3x – 2x + 6 = 6
x2 – 5x + 6 – 6 = 0 f(5) = 8
x2 – 5x = 0 f(5) = +
x(x – 5) = 0 ⇒ + =8
⇒ x = 0 or x – 5 = 0
x = 0 or x = 5 + =8
(12) + (12) = (12) 8
b. f(x) = (x – 2) (x – 3),
3k + 4(6) = 96
f(x) 0
3k + 24 = 96
⇒(x – 2) (x – 3) 0
3k = 96 - 24
(x – 2) 0 or (x – 3) 0
3k = 72
x 2 or x 3
k=
2. The function f is given by f(x) = 3x2 and the k = 24
function g is given by g(x) = x – 4
i. Write down f{g(x)} ii. Put k = 24 in f(x) = +
ii. Write down g{f(x)}
f(x) = +
iii. Find the value of p if f{g(p)} = g{f(p)}
f(4) = + = + = = 11
Solution
i. f {g(x)} = 3(x – 4)2 Exercises 4.18
f {g(x)}= 3(x – 4) (x – 4) Solve the equation (f○g)(x) = 0
f {g(x)}= 3(x2 – 8x + 16) 1. f(x) = x2 – 2 g(x) = x + 3
f {g(x)}= 3x2 – 24x + 48 2
2. f(x) = x – x – 2 g(x) = 2x – 1
ii. g{f(x)} = 3x2 – 4
Inverse of a Function
iii. f{g(p)} = g{f(p)} The inverse of a one – to – one function f is the
3(p – 4)2 = 3p2 – 4 function , which is obtained from f by
3(p – 4) (p – 4) = 3p2 – 4 interchanging the coordinates of each ordered
3(p2 – 8p + 16) = 3p2 – 4 pair of f. For example given f = (2, 8), (3, 27)
3p2 – 24p + 48 = 3p2 – 4 (4, 64) the following is true:
- 24p = - 4 – 48 1. f is a function because no first (x) coordinates
- 24p = - 52 are the same.
p=

Baffour Ba Series , Further Mathematics for Schools Page 102


2. f is a one – to – one function because no Identifying Inverse Functions
second (y) coordinates are the same. Functions f and g are inverses of each other if
3. If f is a one – to – one function, then f is and only if:
invertible. (g○f)(x) = x for every number x in the domain
4. If f is invertible, then the coordinates of x of f and (f○g)(x) = x for every number x in the
and y can be interchanged to obtain the inverse domain of g.
of f. That is: = (8, 2), (27, 3), (64, 4)
Worked Examples
Worked Examples Determine whether the functions f and g are
Determine whether each function is invertible. inverses of each other.
If it is invertible, find the inverse. 1. f(x) = 3x and g(x) =
1. f = {(2, 4), (-2, 4), (3, 9)}
2. f(x) = 2x – 1 and g(x) = x + 1
2. g = 2. , / , . , / , . , /3
3. f(x) = x2 and g(x) = √
3. h = {(3, 5), (7, 9)}
Solution
Solution
1. f(x) = 3x and g(x) =
1. In f = {(2, 4), (-2, 4), (3, 9)}
{(2, 4) and (-2, 4) have the same y – coordinate. (g○f)(x) = g(f(x) = g(3x) = =x
Therefore f is not one- to – one and as such not
(f○g)(x) = f(g(x) =f. / 3× =x
invertible.
Therefore g = or (x) =
2. In g = 2. , / , . , / , . , /3, no y value is
the same.
2. f(x) = 2x – 1 and g(x) = x + 1
Therefore g is invertible and one – to – one.
(g○f)(x) = g(f(x)
= 2. , / , . , / , . , /3,
= g(2x – 1)
= (2x – 1) + 1
3. In h = {(3, 5), (7, 9)}, y values are not the
same. Therefore h is invertible and one – to – =x+
one. = {(5, 3), (9, 7)} Since (g○f)(x) ≠ x, the functions f and g are not
inverse of each other.
Exercises 4.19
Which of the following is invertible? 3. f(x) = x2and g(x) = √
1. f = {(-3, 3), (-2, 2), (0, 0), (2, 2)} (g○f)(x) = g(f(x)
2. f = {(1, 1), (2, 8), (3, 27)} = g(x2) = √ = /x/
3. f = {(16, 4), (9, 3), (0, 0)} The functions f and g are not inverses of each
4. f = {(0, 5), (5, 0), (6, 0)} other. Note f(x) = x2 has no inverse.
5. f = {(-1, 1), (-3, 81), (3, 81)}

Baffour Ba Series , Further Mathematics for Schools Page 103


Exercises 4.20 ⇒ y = 2x + 1
A. Determine whether the following pair of x = 2y + 1 ( Interchange x and y)
functions are inverses of each other. x = 2y + 1 (Solve for y)
1. f(x) = 2x – 10 and g(x) = x + 5 x – 1 = 2y
=y
2. f(x) = 3x + 7 and g(x) =
y=
3. f(x) = and g(x) =
4. f(x) = and g(x) = ⁄ (x) =
5. f(x) = 3x – 2 and g(x) =
2. If f(x) = , find (x)
B. Proof that f and g are inverse functions of
each other. Solution
1. f(x) = + 5, x ≤ 0 g(x) = - √ ,x≥5 f(x) =
2. f(x) = - + 3, x ≥ 0 g(x) = √ ,x≤3 Let f(x) = y
3. f(x) = – 4, x ≤ 0 g(x) = √ , ⇒y=
x= (Interchange x and y and solve for y)
Switch – and – solve Strategy x(y – 3) = y + 1
If an invertible function is defined by a xy – 3x = y + 1
formula, then the inverse function must undo or xy – y = 3x + 1
reverse what the function does. Because the y(x – 1) = 3x + 1
inverse function interchanges the roles of x and
y=
y, we interchange the x and y in the formula and
then solve the new formula for y to undo what (x) =
the original function did.
3. If f : x → 3x + 2, find;
The switch – and – solve strategy for finding i. f(1) ii. f(- 4)
is as follow. iii. determine the inverse function
1. Replace f(x) by y.
2. Interchange x and y. Solution
3. Solve the equation for y. i. f : x → 3x + 2
4. Replace y by (x). f(1) = 3(1) + 2 = 5

Worked Examples ii. f(- 4) = 3(-4) + 2 = -10


1. Find the inverses of h(x) = 2x + 1
iii. Let y = 3x + 2
Solution x = 3y + 2 (Interchange x and y and solve for y)
h(x) = 2x + 1 3y = x – 2
Let y = h(x)

Baffour Ba Series , Further Mathematics for Schools Page 104


y= Inverse of a Function with Even Exponent
Worked Example
=
Find the inverse of the function f(x)= (x – 3)2
for x ≥ 3
4. If f(x) = 3x + 1, g(x) = 4x – 3, find;
i. f(f(2)) ii. g(g(2)) iii. (x) iv. (-10) Solution
f(x)= (x – 3)2
Solution y = (x – 3)2
f(x) = 3x + 1, g(x) = 4x – 3 x = (y – 3)2 (Interchange x and y and solve for y)
i. f(2) = 3(2) + 1 = 7
√ =y–3
f(f(2)) = f(7) = 3(7) + 1 = 22
y–3=√
ii. g(2) = 4(2) – 3 = 5 y=3+√
g(g(2)) = g(5) = 4(5) – 3 = 17 =3+√

iii. f(x) = 3x + 1 Exercises 4.22


Let y = 3x + 1 Find the inverse of each function.
x = 3y + 1 (Interchange x and y and solve for y) 1. f(x)= (x – 2)2 for x ≥ 2
2
3y = x – 1 2. f(x)= (x + 5) for x ≥ -5
2
3. f(x)= x + 3 for x ≥ 0
y= 2
4. f(x)= x – 5 for x ≥ 0
= 5. f(x) = √
6. f(x) = √
iv. (- 10) = =
Challenge problems
Exercises 4.21 Find the inverse of each function.
A. Determine for each function. 1. f(x)= x2 – 6x, x≥3
2
1. f(x) = 2. f(x) = 2. f(x)= x – 4x+3, x≤2
3. f(x)= ( )
3. f(x) = 4. f(x) =
4. f(x)= √ , 0≤x≤2
5. f(x) = 6. f(x) =
7. f(x) = 8. f(x) = Type 2
If two functions are inverses of each other, then
9. f(x) = 10. f(x) =
their composite function is always x ( if x is the
involving variable). (f○f 1)(x) = x
B. Find the inverse of each function
1. f(x) = √ 2. f(x) = √ Worked Examples
3. f(x) = √ 4. f(x) = √ 1. Show that (f○g)(x) = x for each pair of
5. f(x) = √ 6. f(x) = √ + 1 functions.

Baffour Ba Series , Further Mathematics for Schools Page 105


a. f(x) = 2x – 1 and g(x) = 1.f(x) = g(x) =

b. f(x) = x3 + 5 and g(x) = ( ) 2. f(x) = g(x) =

Solution 3. f(x) = g(x) =



a.(f○g)(x) = f( ( )) 4. f(x) = 2x3 g(x) = . /
f( ( )) = f. /=2. /–1 =x+1–1=x
Graph of f and
b. (f○g)(x) = f( ( )) If a point (a,b) is on the graph of a function f,

(f○g)(x) = f(( ) ) then (b, a) must be on the graph of (x). The

(f○g)(x) = f(( ) )+5 points (a, b) and (b, a) lie on opposite sides of
(f○g)(x) = x – 5 + 5 the diagonal line y = x and are the same
(f○g)(x) = x distance from it. For this reasons, the graphs of
f and (x) are symmetric with respect to the
3. Determine algebraically whether f(x) = 3x – 2 line y = x.
and g(x) = x + 2 are inverses of each other.
For example, consider f(x) = x2 for x 0 and
Solution (x) = √ . Their graphs are shown below:
f(x) = 3x – 2 and g(x) = x + 2 y
(f○g)(x) = f( ( )) y=x

= f. / 5
4 f
(b, a)
=3. / 2 3 (a, b)
=x+6–2 2
=x+4 1
Since the answer is not x, f(x) and g(x) are not x
-3 -2 -1 1 2 3 4 5
inverses of each other
-2

Exercises 4.23 y
A. Show that (f○g)(x) = x and (g○f)(x) = x for
y=x
each pair of functions. 2 5
f(x) = x ,
1. f(x) = 3x + 5 and g(x) = x≥ 0 4
3
2. f(x) = 3x – 7 and g(x) =
2
3. f(x) = x3 – 9 and g(x) = √ 1 f--1(x) = √
4. f(x) = x3 + 1 and g(x) = √ x
-3 -2 -1 1 2 3 4 5
B. Determine whether f(x) and g(x) are -2
inverses of each other.
Baffour Ba Series , Further Mathematics for Schools Page 106
Exercises 4.21 Solution
A. Find the inverse of each function and f(x – 1) = 2x + 3
sketch graph of f and on the same pair of Let y = x – 1
axes.
Inverse of y ,
1. f(x) = 2x + 3 4. f(x) = x2+ 3 for x ≥ 0 x= y–1
2. f(x) = -3x + 2 5. f(x) = √ y=x+1 (Interchange x and y)
3. f(x) = x2– 1 for x ≥ 0 6. f(x) = √ y=x+1 (Solve for y )

f (x) = f (x + 1)
B. For each pair of functions, find: = 2(x + 1) + 3
( ○ f)(x) = 2x + 2 + 3
1. f(x) = x3 – 1 and (x) = √ = 2x + 5

2. f(x) =2x3+1 and (x) = √ 2. If f(2x + 1) = 6x + 2,


i. what is f(x) ?
3. f(x) = x3 – 3 and (x) = 2x + 6 ii. find x when f(x) = 2
4. f(x) =3x – 9 and (x) = x + 3

Solution
C. For each pair of functions, find: i. f(2x + 1) = 6x + 2
( ○ f)(x)

1. f(x) = + 2 and = Let y = 2x + 1


To find inverse of y
2. f(x) = 4 – and = x = 2y + 1
x – 1 = 2y
3. f(x) = and =
y=
4. f(x) = and =
f (x) = f . /
The Parent Function from a Transformed
= 6. /+2
Function
= 3 (x – 1) + 2
The function f(x – 1) is a direct transformation
= 3x – 3 + 2
of the parent function f(x) = 3x – 1

To find the parent function, f(x) from the ii. f (x) = 3x – 1


transformed function f(x + a), find the inverse f (2) = 3 (2) – 1
of (x + a) and substitute in the function given =5
by f(x + a)
Exercises
1. Given f(2x – 1) = 8x2 – 3x + 2, find f(x)
Worked Examples
1. If f(x – 1) = 2x + 3, find f(x) 2. If f(x + 2) = 2x + 1, find f(x)

Baffour Ba Series , Further Mathematics for Schools Page 107


3. Given the function f(x) = 2x – 3, find g(x) = Solution
f(x + 1) + f(x – 1) Ans 4x - 6 In y = 2x2 – 8x + 3, a = 2 and b = – 8
( )
At the turning point, x = – =– =4
Quadratic Function
A function is a function f given by f(x) = ax2 +
bx + c , where a, b and c are constants and a ≠0 Put x = 4 in y = 2x2 – 8x + 3
y = 2(4)2 – 8(4) + 3
The graph is a parabola which opens up if a > 0 y=3
and downward if a < 0 At the turning point of 2x2 – 8x + 3 = 0,
a<0 a>0 x = 4 and y = 3
Maximum curve Minimum curve
Graphical Solution of Quadratic Function
The graph of f(x) = ax2 + bx + c = 0, a > 0 is a
parabola of U – shape and the graph of ax2 + bx
+ c, a < 0 is a parabola of ∩ – shape. The points
The Minimum and Maximum Points at which the parabola cuts the x – axis is the
The vertex of a parabola is the point of roots of the function or the truth set or the
intersection of the line of symmetry and the solution set or the zeros of the function.
parabola itself. The vertex is the turning point:
y
either maximum (highest) or minimum (lowest)

Line of symmetry
4
point of the parabola. ax2 + bx + c
When the parabola is U – shaped, it is said to =0
have a minimum or least turning pointand x
-m n
when it is ∩ – shaped, it is said to have a
Minimum point
maximum or greatest turning point.
Fig. I

Values of x and y at the Turning Point From the diagram above (Fig. I), the intercept
For all equations of the form: on the x – axis is -m and n. Therefore the truth
y = ax2 + bx + c, a , at the turning point, set is x = - m or x = n
y
x=–
To get the value of y at the turning point, Maximum point
Line of symmetry

substitute x = – in y = ax2 + bx + c. The


f x
value of y obtained is called the maximum or e
minimum value, depending on the nature of the 8-ax2 + bx +2 c = 0
parabola.
Fig. II

Worked Examples From the diagram above (Fig. II), the intercept
1. What is the value of x and y at the turning on the x – axis is -e and f. Therefore the truth
point of y = 2x2 – 8x + 3 set is x = e or x = f
Baffour Ba Series , Further Mathematics for Schools Page 108
Drawing the Graph of f(x) = ax2 + bx + c = 0, to have no roots or no zeros as shown below;
To draw the graph of f(x) = ax2 + bx + c = 0, a ≠ 0 y
I. Prepare a table of values for a given range of
values of x. x
II. Plot the points (x, y) on a graph sheet, using
a given or a convenience scale;
III. Join the points to make a free hand sketch
of the required parabola/curve; In both diagrams, the functions have no roots

The Roots or Zeros of Quadratic Function Equation of Axes or Line of Symmetry


The roots of a quadratic function is the value(s) Parabolas can be described as being
of x for which the function, f(x) = 0. The roots symmetrical, meaning that a line can be drawn
of the quadratic function are also called the through a parabola, to divide it into two equal
zeros of the function. parts, creating mirror images of each other. The
straight line bisecting the parabola is called a
From the graph, the roots of a function is line of symmetry. For all quadratic equations
determine with cognizance to the nature of the of the form, y = ax2 + bx +c, a ≠ 0, the line of
parabola in relation to the x – axis;
symmetry has the equation, x = –
I. Whether U or ∩ – shaped, the points at which
the parabola cuts the x – axis is the roots or
The Minimum and Maximum Points
zeros of the equation as shown below.
y The vertex of a parabola is the point of
intersection of the line of symmetry and the
parabola itself. The vertex is the turning point:
x either maximum (highest) or minimum (lowest)
a b point of the parabola.

When the parabola is ∩ – shaped, it is said to


The zeros or truth set of the function is x = a
have a minimum or least turning point and
or x = b
when it is ∩ – shaped, it is said to have a
maximum or greatest turning point.
a b
x Expressing the Quadratic Function f(x) = ax2
+ bx + c in the from f(x) = a(x - h)2 + k,
The standard form for the quadratic function
The zeros or truth set of the function is x = a
f(x) = ax2 + bx+ c is f(x) = a(x + h)2 + k,.
or x = b
The standard form is obtained by completing
II. Whether ∪ or ∩ – shaped, if the parabola the square of the quadratic function.
does not cut the x – axis, the equation is said

Baffour Ba Series , Further Mathematics for Schools Page 109


In the standard form; f(x) = a(x – h)2 + k,. 3. Complete the square in f(x) = -2x2 + 6x – 1.
1. The vertex is at (-h, k) Hence, find the maximum or minimum value of
2. The maximum value is k, if a is negative. the function.
3. The minimum value is k, if a is positive
Solution
Worked Example f(x) = -2x2 + 6x – 1
1. For the quadratic function, f(x) = 2x2 + 6x + f(x) = -2(x2 – 3x) – 1
5, find the vertex and maximum or minimum f(x) = -2[x2 – 3x + . / –. / ]– 1
value.
f(x) = -2[x2 – 3x + . / – ]–1
Solution
f(x) = - 2[. / – ]–1
f(x) = 2x2 + 6x + 5
f(x) = 2(x2 + 3x) +5 ( Factorize 2 out;) f(x) = -2. / + –1

Complete the square of the bracket; f(x) = -2. / +


f(x) = 2[x2 + 3x + . / – . / ] + 5 ⇒ The vertex (h, k) is at the point . , /

f(x) = 2[x2 + 3x + . / – ] + 5 . / The minimum value k, is

f(x) = 2[. / – ]+5 Exercises 4.24


f(x) = 2. / – +5 ( *
A. Find the vertices of the parabola formed
by the following functions.
f(x) = 2. / + 1. f(x) = x2 + 4x + 2 2. f(x) = 2 + x – x2
3. f(x) = -2x2 – 6x 4. f(x) = -3x2 – x + 1
⇒ The vertex (h, k) is at the point. , /
5. f(x) = x2 – 16x + 10 6. f(x) = 6x2 + 6x + 15
The minimum value k, is 7. f(x) = -x2 + 5x + 6 8. f(x) = 3 – 6x + 10x2

2. Find the vertex and maximum or minimum B. i. Complete the square in the following,
value of the function f(x) = -2x2 + 4x + 1 ii. find the maximum or minimum value of
the function,
Solution iii. sketch the graph of the function f
f(x) = -2x2 + 4x + 1 1. f(x) = x2 – 6x + 8
f(x) = -2(x2 – 2x) + 1 2. f(x) = x2 + 16x – 36
f(x) = -2[(x2 – 2x + ( ) – ( ) ] + 1 3. f(x) = (x – 4)(x + 2)
f(x) = -2[( ) –1]+1 4. f(x) = (3 – x)(x – 5)
f(x) = -2( ) +2 +1 5. f(x) = (x – 1)(x + 2) (-1)
f(x) = -2( ) +3
The vertex is at the point (1, 3) and the Quadratic Equations
minimum value is 3. Any equation that can be written in the form:

Baffour Ba Series , Further Mathematics for Schools Page 110


y = ax2 + bx + c, where a, b and c are constants IV. At this point, the implication is that either x
and a ≠ 0 is called a quadratic equation. + n = 0 or x + m = 0
x = − n or x = − m
The values of x that satisfy a quadratic equation
Therefore, the truth set or solution set of the
is called the truth set or solution set or the
roots of the quadratic equation. equation is * or +

The truth set or solution set or the root of the Worked Examples
1. Find the truth set of x2 – x − 6 = 0
quadratic equation is found by using any of the
following methods.
Solution
1. Factorization
x2 – x − 6 = 0,
2. Completing the square
x2 + 2x – 3x – 6 = 0
3. The quadratic formula
(x2 + 2x) – (3x – 6) = 0
4. Graphical method
x (x + 2) – 3(x + 2) = 0
Method of factorization (x + 2) (x – 3) = 0
This method makes use of the idea of the x + 2 = 0 or x – 3 = 0
product of real number. x = – 2 or x = 3
Let a and b be real numbers. If ab = 0 ⇔ a = 0 Truth set = * or +
or b = 0
2. Solve x2 − 13x + 36 = 0
Equations the form: x2 + b x + c = 0 , Solution
To solve quadratic equations of the form: x2 – 9x – 4x + 36 = 0
x2 + b x + c = 0 , (x2 – 9x) − (4x + 36) = 0
I. Find two factors of the constant term, c, that x ( x − 9 ) – 4( x − 9) = 0
sum up to the coefficient of x, as illustrated (x – 4) (x – 9) = 0
below = (x − 4) = 0 or (x − 9) = 0
x2 + b x + c = 0… …….. (1) x = 4 or x = 9
The truth set = {x : x = 4 or x = 9}

m + n mn 3. Solve 2x2 + 5x − 3 = 0
II. Substitute mx + nx = bx in eqn (1) to obtain
four terms as: Solution
x2 + mx + nx + mn = 0 2x2 + 5x – 3 = 0
2x2 – x + 6 x – 3 = 0
III. Group the terms and factorize completely as (2x2 – x) + (6x – 3) = 0
shown below; x (2x – 1 ) + 3 (2 x – 1 ) = 0
(x2 + mx) + (nx + mn) = 0 (2 x – 1) = 0 or x = - 3
x(x + m) + n(x + m) = 0 2x = 1 or x = -3
(x + n) (x + m) = 0.
x = or x = − 3

Baffour Ba Series , Further Mathematics for Schools Page 111


The truth set = 2 or 3 Solution
Comparing the statements
Solving Related Problems ( ) = 25 + + and
Worked Examples ( ) = + 2ab + , it is seen that:
i. = 25
1. Write the equation x – 11 + = 0 in the form
a=√
ax2 + bx + c = 0, and hence find the truth set
a = 5x
Solution
ii. 2ab = 70x
x – 11 + = 0 (Multiply through by x)
But a = 5x
x2 – 11x + 24 = 0 2(5x)b = 70x
(x – 3) (x – 8) = 0 10bx = 70x
x – 3 = 0 or x – 8 = 0 b = 7 so = = 49
x = 3 or x = 8 Therefore, the complete statement is
( ) = 25 + + 49
2. If (p, q) is the truth set of the equation
x2 + 10x = 96, evaluate (p + q) 2. Copy and complete the statement

Solution ( ) = – +
x2 + 10x = 96
x2 + 10x – 96 = 0 Solution
x2 + 16x – 6x – 96 = 0 Comparing the statements:
(x2 + 16x) – (6x – 96) = 0 ( ) = – + and
x(x + 16) – 6( x + 16) = 0 ( ) = – 2ab + it is seen that
(x – 6) (x + 16) = 0
i. ( ) =( )
x – 6 = 0 or x + 16 = 0
x = 6 or x = -16 Therefore a = and =
Truth set (6, -16) = (p, q)
⇒( p + q ) = 6 + (-16) = - 10 ii. – 2ab = – x, but a =

Method of Completing the Square – 2( ) b = – x


The method of completing the square depends – b = –1
on the identities: b = 1, so =1
1. ( ) = + 2ab + Therefore the complete statement is:
2. ( ) = – 2ab + ( ) = – +1

Worked Examples
3. If m, n Z, find m and n such that
1. Complete the squares in the following;
x2 + 12x + m = ( )
( ) = 25 + +

Baffour Ba Series , Further Mathematics for Schools Page 112


Solution IV. Complete the squares at the left side of the
Comparing the statements:
equation to get . / =–c+. /
x2 + 12x + m = ( ) and
2 V. Introduce a square root sign on both sides of
a + 2ab + = ( )
2 2
i. x + 12x + m = a + 2ab + the equation i.e. . /= √ . /
Therefore a = x
6. Make x the subject of the equation
ii. 2ab = 12x, but a = x x= √ . /
2bx = 12x
Find the value or values of x that satisfies the
b=6
equation.
But b = n and =m
6 = n and =m Worked Examples
1. Find the truth set of x2 + 8x + 15 = 0
By substitution,
x2 + 12x + m = ( ) Solution
2
= x + 12x + 36 = ( ) x2 + 8x + 15 = 0
Therefore m = 36 and n = 6 x2 + 8x = – 15
x2 + 8x + . / = – 15 + . /
Exercises 4.25
1. The truth set of the equation ax2 + bx = 4 is x2 + 8x + = – 15 +
{-3, 2}. Find the numerical values of the ( ) = – 15 + 16
constants a and b. ( ) =1
x+4= √
Solving Quadratic Equations by Method of x = – 4 + √ or x = – 4 – √
Completing Squares x = – 4 + 1 or x = – 4 – 1
A. Quadratic equations of the form: x = – 3 or x = – 5
x2 + b x + c = 0 Truth set = * or +
To complete the squares of quadratic equations
of the form: x2 + bx + c = 0 B. Quadratic equations of the form:
I. Transpose the constant, c, to the right side of ax2 + b x + c = 0, a > 1
the equation to assume the opposite sign i.e. x2 To complete the squares of quadratic equations
+ bx = – c of the form: ax2 + bx + c = 0,
II. Divide the coefficient of x (number attached i. Transpose the constant, c, to the right side of
to x) by 2. i.e. the equation to assume the opposite sign
i.e. ax2 + bx = – c
III. Square to get . / and add . / to both ii. Divide through the equation by a, which is
sides of the equation the coefficient of ,
i.e. x2 + bx + . / = – c + . / i.e. + =–

Baffour Ba Series , Further Mathematics for Schools Page 113


⇒ x2 + =– x=–
iii. Find of i.e. × = x=– or x = –

iv. Square to get . / and add . / to both x= or x =


sides of the equation. x=– or x = –
i.e. x2 + +. / = –c+. / x = – 1 or x = –
v. Complete the squares at the left side of the Truth set = 2 or – 3
equation to get;
. / =–c+. / 2. Find the truth set of 3x2 + 2x – 5 = 0
vi. Introduce the square root sign on both sides
Solution
of the equation;
3x2 + 2x – 5 = 0
i.e.. /= √– c . / , 3x2 + 2x = 5

vii. Make x the subject to get the value or x2 + =


values of x that satisfies the equation; x2 + +. / = +. /
x=– √– c . / x2 + +. / = +. /

Worked Examples . / = +
1. Find the truth set of 3x2 + 8x + 5 = 0 . / =

Solution x+ = √
3x2 + 8x + 5 = 0
3x2 + 8x = – 5 x+ =

x2 + =– x=–

x2 + +. / = – +. / x=– or x = –
x= or x = –
x2 + +. / = – +. /
x = 1 or x = –
2
x + +. / = – +. /
Truth set = 2 – 3
. / =– +
Exercises 4.26
. / =
A. Complete the squares in the following;
x+ = √ 1. ( ) = + x+9
2. ( ) = – +
x+ = 3. ( ) = – +

Baffour Ba Series , Further Mathematics for Schools Page 114


4. ( ) = – + quadratic equations which cannot readily be
5. ( ) = – +4 solved by factorization. In other words, to solve
6. ( ) = – + the quadratic equation ax2 + bx + c = 0
I. Attempt to factorize the quadratic expression
B. Solve by completing the square; on the left - hand side.
1. x2 + 7x – 3 = 0 4. x2 + 9x + 20 = 0 II. If factorization is not possible, use the

2. 10 + 3x – 2 = 0 5. x2 + 4x – 21 = 0 formula: x =
3. 4x2 – 6x – 1= 0 6. 3x2 + 12x + 6 = 0
Note
The Quadratic Formula b2 – 4ac is called the discriminant of ax2+ bx + c
The quadratic formula is derived from the 1. If b2 – 4ac > 0 , then the equation has two
quadratic equation ax2 + bx + c = 0 as follows: distinct solutions.
In ax2 + bx + c = 0 2. If b2 – 4ac = 0, then the equation has one
I. Subtract c from both sides; solution.
ax2 + bx = – c 3. If b2 – 4ac < 0 , then the equation has no
II. Divide through by a; (real) solutions.
+ . /x = – . /
Worked Examples
III. Add . / to both sides of the equation; 1. Find the truth set of 2x2 – 9x + 10 = 0

+ . /x + . / = . / – . / Solution
2x2 – 9x + 10 = 0 and ax2 + bx + c = 0
+ . /x + . / = –
compared, a = 2, b = -9 and c = 10

IV. Factorize the left – hand side; Substitute in x =
√( ) ( )( )
. / = x= ( )

x+ = √. / ( * x=

√( ) x=
x+ = 4 5

√( )
x=
x=–
x= or x =
So if ax2 + bx + c = 0, then
x= or x =

x= x = or x = 2
Truth set = {x : x = or x = 2}
Using the Quadratic Formula

The formula x = is used to solve 2. Solve the quadratic equation x2 + 2x + 3 = 0

Baffour Ba Series , Further Mathematics for Schools Page 115


Solution B. Solve with the quadratic formula.
x2 + 2x + 3 = 0 1. 20x2 – 81x = - 40
a = 1, b = 2 and c = 3 2. 18x2 – 55x + 8 = -1
Substitute in b2 – 4ac 3. x2 – 26x = 60
22 – 4 (1) (3) 4. 6x2 – 66x – 9 = 79
-8<0 5. 6x2 + 17x – 88 = 0
This means that the equation has no roots.
C. Solve the following equations.
3. Find the value(s) of k such that the equation 1. 4x – 4x2 = 0 2. 2 + x – 3x2 = 0
x2 + kx + (k + 3) = 0 has only one solution. 3. 4x(x – 4) = x – 15 4. x2 – 2√ x + 2 = 0
5. x2 – 2√ x + 3 = 0 6. x3 – 7x2 + 3x = 0
Solution
If b2 – 4ac = 0, then the equation has one Word Problems
solution. In solving word problems involving quadratic
From x2 + kx + (k + 3) = 0 equations, write the mathematical equation for
a = 1, b = k, c = (k + 3) the problem and solve it, taking note of the fact
k2 – 4 (1) (k + 3) = 0 that the problem does not end with solving the
k2 – 4(k + 3) = 0 quadratic equation. You should therefore go
k2 – 4k – 12 = 0 (Solving by quadratic formula) back to the word problem and answer the
a = 1, b = - 4 and c = -12 question it demands.

Substitute in x =
Worked Examples
( ) √( ) ( )( )
x= 1. The sum of two numbers is 18. The sum of
( )
√ the squares of the numbers is 194. Find the
x=
numbers.

x=
x= or x = Solution
Let x be the number
x= or x =
Then the other number is (18 – x).
x = 6 or x = - 2 Sum of squares = x2 + (18 – x)2.
Truth set = {x : x = 6 or x = - 2} But this is given as 194
x2 + (18 – x )2 = 194
Exercises 4.27 x2 + x2 – 36x + 324 = 194
A. Solve with the quadratic formula 2x2 − 36x + 130 = 0
1. 24x2 + 108x + 120 = 0 x2 − 18x + 65 = 0
2. x2 + 13x –7 = 15 x2 – 5x – 13x + 5 = 0
3. 18x2 + 61x = 50 (x2 − 5x) – (13x + 65) = 0
4. x2 + 30x = -132 x ( x − 5) – 13 ( x − 5 ) = 0
5. x2 + x – 72 = 0 (x −13) (x − 5) = 0

Baffour Ba Series , Further Mathematics for Schools Page 116


x − 13 = 0 or x – 5 = 0 its length. If the area is 42cm2, find the
x = 13 or x = 5 breadth.
Therefore, the numbers are 5 and 13.
Solution
2. A certain rectangle has perimeter of 48cm Let the breadth be x cm. It means that the
and area of 128cm2. Find the length and breadth length is (x +1)cm
of the rectangle.
But area = length × Breadth
Solution 42cm2 = x (x + 1)
Let the length be L cm and breadth be B 42cm2 = x (x + 1)
P = 2(L + B) „ 41= x2 + x
But P = 48 x2 + x − 42 = 0
⇒ 2(L + B) = 48 x2 + 7 x – 6 x − 42 = 0
L+B= (x2 + 7x) – (6 x – 42) = 0
x( x + 7) – 6 ( x + 7) = 0
L + B = 24cm
(x – 6) (x + 7) = 0
⇒B = 24 – L……….(1)
x – 6 = 0 or x + 7 = 0
But area of rectangle x = 6 or x = -7
= L × B = 128cm2 Since the breadth of a rectangle cannot be
L × B = 128cm2…….(2) negative, the breadth of the rectangle is 6cm.

Put eqn (1) into eqn (2); 4. The present ages of a man and his son are 44
⇒L(24 – L) = 128cm2 and 13 years. How many years ago was the
24L – L2 = 128 product of their ages 140?
L2 – 24 L + 128 = 0
Solution
L2 − 8L – 16L + 128 = 0
Let the number of years ago be x
(L2 − 8L) – (16L+ 128) = 0 (44 – x ) (13 – x ) = 140
L(L – 8) – 16 (L – 8) = 0 44(13 – x ) – x (13 – x ) = 140
(L – 16) (L – 8) = 0 572 – 44x – 13x + x2 = 140
L − 16 = 0 or L− 8 = 0 x2 – 57x + 572 = 140
L = 16 or L = 8 x2 – 57x + 572 – 140 = 0
x2 – 57x + 432 = 0
When L =16, B = 24 – 16 = 8 Now, a = 1, b = -57 and c = 432
When L = 8, B = 24 – 8 = 16 √
Substitute in x =
Therefore, the length and breadth of the
( ) √( ) – ( )( )
rectangle are 16cm and 8cm respectively.
Substitute in x = ( )

3. The breadth of a rectangle is 1cm less than √ –


x=

Baffour Ba Series , Further Mathematics for Schools Page 117


√ – x = -1.5 11.06 or x = -1.5 11.06
x=
x = 9.56 or x = -12.56

x= Therefore the original speed is 9.56km/h
x=
Exercises 4.28
x= or x =
1. The sum of a number and its reciprocal is .
x = 48 or x = 9
Since the ages must be positive number of Find all such numbers.
years, x = 9 years ago.
2. The sum of two numbers is 23 and their
5. A man cycles 20 km from C to B. If he product is 132. Find the two numbers
increases his speed by 3 km/h, he saves 30
minutes on the journey. Find his original speed 3. Find two numbers such that their product is
in km/h. 104 and their sum is 21.

Solution 4. Find two consecutive positive integers whose


Let x km/h be the original speed product is 168.

Time taken for the journey at this speed is hr


5. The sum of a number and its reciprocal is .
Time taken for the journey at this increased Find all such numbers.
speed would be = . / hours.
6. The area of a rectangle is 84cm2. If the
Thus, we have – =
length is 5cm greater than the width, find the
20(x + 3) – 20(x) = x (x + 3) (
( )
* length of the rectangle.
20x + 60 – 20x =
7. 18 added to the square of a number is equal
x2 + 3x – 120 = 0
to eleven times the number. Find the number.
⇒ a = 1, b = 3 and c = -120
√ 8. My brother‟s age is 7 less than mine. If I
Substitute in x =
square his age and subtract one, I get the same
√ ( )( )
x= results as when I multiply my age by 5. How

old am I?
x=
√ B. 1. A plot of land for sale has a width of x m
x=
and a length that is 8 ft less than its width . A

x= farmer will only purchase the land if its
x= measues 240 m2. What value of x will cause the
farmer to buy the land?
x = -1.5 11.06

Baffour Ba Series , Further Mathematics for Schools Page 118


2. The length of a rectangle is 5 cm more than x = and x =
twicw a number. The width is 4 cm less than x – and x –
the same number. If the area of the rectangle is (x – ) (x – ) = 0
15, find the number. x2 – x – x + =0
2
x – ( + )x + =0
3. Find two consecutive positive integers such x2 – ( + )x + = x2 + x + = 0.
that the square of the first decreased by 17
Equating the coefficient of x and the constant
equals 4 times the second.
terms:
4. If the measure of one side of a square is + = and =
increased by 2 cm and the measure of the The quadratic equation can be written:
adjacent side is decreased by 2 cm, the area of
the resulting rectangle is 32 cm2. Find the x2 – ( + )x+ =0
measure of one side of the original square.
OR
Challenge Problems x2 - (sum of roots)x + (product of roots) = 0
1. A number on base ten has two digits, the
second one being two more than the first. The
number itself is 22 more than the product of its This can be used to obtain a new quadratic
digits. Find the number. equation whose roots are known or are given as
functions of and .
2. Due to the fault, the speed of train was
reduced by 10km/h over a journey of 100km. Proofs
The journey took 20 minutes longer. Find the 1. + = =
usual speed of the train.
A
2. α2 + β2
3. What value must be
⇒( ) = α2 + β2 + 2αβ
chosen for x so that < 2x+
B 2 x+8
is a right angle?
α2 + β2 = ( ) – 2αβ
B x C

Sum and Product of the Roots of a Quadratic 3. + = ( )


Equation
The quadratic equation ax2 + bx + c can be ( )
4. + = =
written as : x2 + x + = 0.
The roots of this equation are usually denoted
5. ( ) =( )( )
by and . 2 2
⇒( ) = α + β – 2α β
Now we can write down a quadratic equation
But α + β2 = (
2
) – 2αβ
with roots and
Baffour Ba Series , Further Mathematics for Schools Page 119
⇒( ) =( ) – 2αβ - 2α β = √
=( ) – 4αβ = √
= (α + β2) – 2α β
2
/α– /=√

6. = √( )
vi. + = ( )
But ( )=( ) –2α β
– . /
α = √( ) = = = 32
. / ⁄
α = √( )

vii. α3 + β3
7. α3 + β3
=( ) (α2 – αβ + β2)
=( ) (α2 – αβ + β2)
=( ) [(α2 + β2) - αβ]
=( ) [(α2 + β2) – αβ]
= (3)[(8) – ]
Worked Examples = (3)(7.5)
1. If α and β are the roots of the equation = 22.5
2x2 – 6x + 1 = 0, find the value of :
i. α + β ii. α β iii. α2 + β2 2. One root of the equation ax2 + bx + c = 0 is
iv. α3 β + α β3 v. /α – β/ vi. + five times the other. Show that 5b2 = 36ac, a ≠ 0

vii. α3 + β3 Solution
Let the roots be α and 5α
Solution ax2 + bx + c = 0
2x2 – 6x + 1 = 0
2
x2 + x + = 0 (let coefficient of x2 equal to 1)
x – 3x + = 0
α + 5α = and (α ) (5α) =
i. α + β = 3
6α = ……….(1)
ii. α β = 5α2 = ……….(2)

iii. α2 + β2 = ( ) –2αβ α is in both equations and not in the solution


=( ) –2. /=9–1=8 required. Therefore, get α on its own from eqn
(1) and substitute in eqn (2)

iv. α3 β + α β3 = α β(α2 + β2 ) From 6α = ……….(1)

= (8) = 4 α=

v. α = √( ) Put α = in eqn (2)


5α2 =
= √( ) . /

Baffour Ba Series , Further Mathematics for Schools Page 120


3 3
5. / = iv. + v. + vi. +
vii. (1 + ) + (1 + )
5. /=
viii. . /. /
= (Cross product)
5ab2 = 36a2c ( Divide through by a)
3. The roots of the quadratic equation 2x2 +
5b2 = 36ac
6x + 3 = 0 are α and β. Find the value of:
i. α ii. iii. 3 + 3 iv. 2 + 2
3. If α and β are the roots of the equation;
B. 1. One root of the equation 2x2 + bx + c = 0
x2 – px + q = 0, show that :
2 is three times the other root. Show that 3b2 =
i. + 2 = p2 – 2q
32c.
ii. (α )2 = p2 – 4q
2. Given that one root of the equation 2x2 – 12 x
Solution + k = 0 is twice the other root, find the value of k.
i. x2 – px + q = 0
= p and αβ = q 3. One root of the equation x2 – px + q = 0 is
twice the other. Show that 2p2 = 9q.
α2 + β2 = ( ) – 2αβ
2 2
α +β = – 2q 4. The equation x2 – 2px + q = 0 has roots α and
α + 2. Verify that p2 = q + 1
ii. ( ) =( ) – 2αβ –2α β
=( ) – 4αβ 5. The equation x2 – 12x + k = 0 has roots α and
2
= p – 4q α2. Find two possible values of k.

Exercises 4.30 6. The equation x2 – ax + 16 = 0 has roots α and


1. If α and β are the roots of the equation: α3 find the two possible values of a.
x2 – 2x + 5 = 0, find the value of :
i. α ii. Challenge Problems
2 2
iii. α + iv. 2 + 2 1. One root of the equation ax2 + bx + c = 0 is
v. α3 + 3 vi. (α )2 three times the other show that 3b2 = 16ac
vii. 3 + 3 viii. ( 2 – )( 2 – )
ix. + x. + 2. One root of the equation px2 + qx + r = 0 is
four times the other. Show that 4q2 – 25pr = 0
xi. xii. – –
3. For what values of k is the roots of x2 – 4(k +
2. If α and β are the roots of the equation: 1) x + (k2 – k + 7) = 0 equal to three times the
x2 – 4x – 3 = 0, find the value of : other.
i. 3 + 3 ii. α2 β2 iii. 2 + 2

Baffour Ba Series , Further Mathematics for Schools Page 121


4. i. The quadratic equation x2 + (2k + 2)x + (2k = = =

+ 5) = 0 has roots α and β. Express in terms of
Substitute in:
k,α + β b. αβ c. 2α2β + 2αβ2
x2 – (sum of roots)x + (product of roots) = 0
ii. The equation x2 – px + q = 0 has roots 2α +
αβ and 2β + βα, = x2 – x+ =0
2
a. Show that p = 6 = 2x – 5x + 3 = 0
b. Express q in terms of k.
c. Find the values of k for which q = 0. ii. α2, β2
Sum of roots;
Forming Quadratic Equations ⇒( ) = α2 + β2 + 2αβ
A. Forming a Rational Roots α2 + β2 = ( ) – 2αβ
When forming a quadratic equation given the
= . / – 2. / = – =
roots;
1. Identify the roots of the equation.
2. Find the sum of the roots and the product of Product of roots;
the roots respectively. α2 β2 = ( ) =. / =
3. Then respectively substitute in:
x2 – (sum of roots)x + (product of roots) = 0 to
Substitute in:
obtain the quadratic equation.
x2 – (sum of roots)x + (product of roots) = 0
x2 – x+ =0
Worked Examples
1. If the roots of 3x2 – 5x + 2 = 0 are α and β, = 9x2 – 13x + 4 = 0
form equations whose roots are The equation is 9x2 – 13x + 4 = 0
i. , ii. α2 , β2 iii. + , +
3 3
iii. + , +
iv. +1, +1 v. ,
Sum of roots;
= + + +
Solution
3x2 – 5x + 2 = 0 = + + +
2
x – x+ =0 = + +
α = and = ⁄
= + = + =

i. ,
Sum of roots;
Product of roots;

= + = = = = . /. /

= . / . /
Product of roots;

Baffour Ba Series , Further Mathematics for Schools Page 122


= + + + αβ = +
( ⁄ ) ( ⁄ )
+1 =
( ⁄ ) ( ⁄ )
= + 1 + 1 + αβ

= + αβ + 2 x2 – (sum of roots) x + (product of roots) = 0


x2 – x+ =0
2
By substitution, 4x – 21x + 26 = 0
= + +2 = + +2 = The equation is 4x2 – 21x + 26 = 0

Substitute in: v. 3, 3
x2 – (sum of roots) x + (product of roots) = 0 Sum of roots;
x2 – x+ =0 3
+ 3
2
= 6x – 25x + 25 = 0 =( ) (α2 – αβ + β2)
The equation is 6x2 – 25x + 25 = 0 =( ) [(α2 + β2) – αβ]
=( ) [( ) – αβ]
iv. + 1, +1 =( ) [( ) ]
=( ) [( ) ]
Sum of roots; = [. / – 3. / ]
+1+ +1 = . /
= + +2 =
= +2 =
( )
( )
= +2
( ) Product of roots;
( ⁄ ) . / 3 3
= +2 =( ) =. / =
. /

= Substitute in;
x2 – (sum of roots) x + (product of roots) = 0
Product of roots; x2 – x+ =0
. /. / 27x2 – 35x + 8 = 0
The equation is 27x2 – 35x + 8 = 0
= . / . /

=( + + +1 3. If α and β are the roots of the equation:


)
ax2 + bx + c = 0, obtain in terms of a, b and c ,
=( )
+ ( )
+1 the values of :
( )
=( + +1 i. + ii.
) ( )

Baffour Ba Series , Further Mathematics for Schools Page 123


Solution x2 – (sum of roots) x + (product of roots) = 0
i. ax2 + bx + c = 0 x2 – 2√ + 1 = 0
= , =
⁄ 2. Find the quadratic equation whose one root is
+ = = =
⁄ 3 + 2√

ii. = √( ) Solution
Sum of roots;
= √. ⁄ /
(3 + 2√ ) + (3 – 2√ )
3 + 2√ + 3 – 2√ = 6
= √. ⁄ /

= √ Product of roots ;
(3 + 2√ ) (3 – 2√ )
B. Forming from an Irrational root –( √ )
In a quadratic, irrational and imaginary roots 9 – (4) (3)
occur in conjugate pairs. Thus, if a root of a 9 – 12 = - 4
quadratic equation is given as: Substitute in;
1. a – b √ , then the other root is a + b √ x2 – (sum of roots) x + (product of roots) = 0
2. a + i, then the other root is a – i x2 – 6 x – 4 = 0

Find the sum of roots and the product of roots 3. Determine the quadratic equation whose one
and substitute in : root is - 4 – √
x2 – (sum of roots)x + (product of roots) = 0
Solution
Worked Examples Sum of roots;
1. Form an equation with roots √ + 2, √ – 2, = (- 4 – √ ) + (- 4 + √ )
=-4–√ –4+√
Solution =-8
Let = √ + 2, and = √ - 2,
+ =√ +2+ √ –2 Product of roots ;
= 2√ = (- 4 – √ ) (- 4 + √ )
= ( ) – (√ )
= (√ ) (√ – ) = 16 – 6
= √ (√ – ) (√ – ) = 10
Substitute in;
= 5 – 2√ + 2√ – 4
x2 – (sum of roots) x + (product of roots) = 0
=1
x2 + 8x + 10 = 0
Substitute in;

Baffour Ba Series , Further Mathematics for Schools Page 124


4. Find the quadratic equation with real Solution
coefficients which -2 + i as a root (i = √ )? x= and x = 3
x – = 0 or x – 3 = 0
Solution
-2 + i and -2 – i 3x – 2 = 0 or x – 3 = 0
Sum of roots; (3x – 2)(x – 3) = 0
= (-2 + i ) +( -2 – i ) 3x(x – 3) – 2(x – 3) = 0
= -2 + i – 2 – i 3x2 – 9x – 2x + 6 = 0
=-4 3x2 – 11x + 6 = 0
x2 – x+ =0
Product of roots;
= (-2 + i )(-2 – i ) Comparing x2 + ax + b = 0 and x2 – x+ =0
= (-2)2 – (i)2
⇒ax = - and b =
= 4 – (-1)
=5 a=- and b = 2
Substitute in;
x2 – (sum of roots) x + (product of roots) = 0 2. The roots of the equation 2x2 + 6x + 3 = 0 are
x2 + 4 x + 5 = 0 and . Find the values of:
i. + ii. iii. 3 + 3 iv. 2 + 2
Exercises 4.31 The roots of the equation 2x2 + px + q = 0 are
1. What is the quadratic equation that has 2 + and 2 + . Find the values of p and q.
one of the following as its root;
1. 5 + √ 2. 4 + √ 3. 4 – 5i Solution
4. -2 – √ 5. 3 – 4 √ 6. √ From 2x2 + 6x + 3 = 0
7. 3 √ 8. 5 √ 9. - 4 3 √ x2 + x + = 0
i. + = - =-3
Two Unknowns from the Roots of an equation
Steps:
1. Identify the given equation as equation (1). ii. =
2. Form a new equation from the given roots as
equation (2) iii. 3 +3 =3( + )
3. Compare equations (1) and (2) to obtain the = 3( - 3 ) = - 9
value of the unknowns.
2 2
iv. + = ( + )2 - 2
Worked Examples = (-3)2 – 2. / = 9 – 3 = 6
1. If the root of the equation is x2 + ax + b = 0
are x = and x = 3, find the values of a and b. From 2x2 + px + q = 0
x2 + x + = 0

Baffour Ba Series , Further Mathematics for Schools Page 125


Roots are 2 + and 2 + From x2 – 3x + = 0;
Sum of roots = Sum of roots = 3
=2 + +2 + Products of roots =
=2 + +2 +
From the roots; ( )and ( )
=3 +3
Sum of roots
= 3( + )
=( )+( )
⇒ 3( + )= = +
3( - 3) = = + –4
-9=
⇒ + –4=3
- 18 = - p
+ =3+4
p = 18
+ =7
Products of roots =
b. From x2 – 3x + = 0;
(2 + ) (2 + )
= 2 (2 + ) + (2 + ) Products of roots =
=4 +2 2+2 2 + From the roots; ( )and ( )
= 4 + 2( 2 + 2) +
Product of roots
By substitution, =( )( )
= 4. / + 2(6) +. / = ( ) ( )
= –2 -2 +4
= + 12 +
= +4–2 -2
= = +4–2( + )
⇒ = ⇒ +4–2( + )=
q = 39
But + =7
3. The roots of a quadratic equation 2x2 – 6x +
+ 4 – 2 (7) =
5 = 0 are ( ) and ( )
i. Find the value of: + 4 – 14 =
a. α b. – 10 =
ii. Form a quadratic equation, with integer
= + 10
coefficient, with roots and .
=
Solution Substitute in:
i. a. 2x2 – 6x + 5 = 0 x2 – (sum of roots)x + (product of roots) = 0
x2 – 3x + = 0 x2 – 7x + =0

Baffour Ba Series , Further Mathematics for Schools Page 126


The equation is 2x2 – 14x + 25 = 0 Substitute in:
x2 – (sum of roots)x + (product of roots) = 0
3. If α and are the roots of the equation: x2 – x– =0
3x2 – x – 5 = 0, form the equation whose roots
The equation is 15x2 – 13x – 169 = 0
are 2α – , 2 – .
Exercises 4.32
Solution A. If α and are the roots of the equation:
3x2 – x – 5 = 0, 2
x – 4x – 3 = 0, form a quadratic equation,
x2 – – = 0, with integer coefficient, whose roots are:
i. 2 , 2 ii. α2, β2
+ = and =
iii. α + 3, β + 3 iv. 3α + 1, 3β + 1
v. α – 1, β – 1 vi. ,
From the given roots 2α – , 2 –
Sum of roots; vii. , viii. (1 – ), (1– )

2α – +2 –
Nature of Root of a Quadratic Equation
2 +2 – – Consider the quadratic equation ax2 + bx + c =
2 +2 –. / 0, a ≠ 0. The roots of the equation are given

2( + ) – . / by: x=

By substitution; The value of the expression (b2 – 4ac) will


⁄ determine the nature of the roots of the
2. / – ( *= – = equation and it‟s called the discriminant of

the equation. The three diagram below of the
Product of roots; curve y = ax2 + bx + c, (a > 0) shows the three
. – /. – / possible cases;

. – /– . – / y
4 – – +

4 –2–2 +
  x

By substitution‟ Two distinct real roots

= 4. /–2–2+ = –4– =- b2 – 4ac > 0


Baffour Ba Series , Further Mathematics for Schools Page 127


y (-8 k)2 – 4(5k + 1)(3k) ≥ 0
(64k 2 ) – 4(15k 2 + 3k) ≥ 0
64k 2 – 60k 2 – 12 k ≥ 0
4k 2 – 12k ≥ 0
k 2 – 3k ≥ 0
 x
Two equal roots
Let k 2 – 3k = 0 (replace ≥ with =)
b2 – 4ac = 0 k(k – 3) = 0
k = 0 or k -3 = 0
y k = 0 or k = 3
Test 1, between the roots in k2 - 3k ≥ 0

1 3
x
No real roots (1)2 – 3(1) ≥ 0
-2 ≥ 0 False
b2 – 4ac < 0
Therefore, the solution is not between 0 and 3
Solution is k ≤ 0 or k ≥ 3.
Note:
For real roots, we combine the first two
2. Show that the roots of the equation,
conditions:
(k – 2)x2 – (2 – 3k )x + 2k = 0 are always real,
k R
If b2 – 4ac ≥ 0, the roots are real.
Two equal roots are also called “repeated
Solution
roots”.
(k – 2)x2 – (2 – 3k )x + 2k = 0
a = (k – 2), b = (2 – 3k), c = 2k
You can also make use of the fact that (any real
b2 – 4ac = (2 – 3k)2 – 4(k – 2) (2k)
number)2 ≥ 0.
= (4 – 12k + 9k2 – 4(2k2 – 4k)
= 4 – 12k + 9k2 – 8k2 + 16k
Worked Examples
= k2 + 4k + 4
1. Find the value of k for which the equation
= (k + 2) (k + 2)
(5k + 1)x2 –8kx + 3k = 0 has real roots.
= (k + 2)2

Solution
(k + 2)2 cannot be negative, as it is a square, for
(5k + 1)x2 – 8 k x + 3k = 0
any value of k R.
a = (5 k + 1), b = -8, c = 3 k
Therefore, b2 – 4ac ≥ 0 and so the roots are
always real.
Condition for real roots is;
b2 – 4ac ≥ 0

Baffour Ba Series , Further Mathematics for Schools Page 128


Exercises 4.33 B. Find the value (s), or range of values, of k
A. In each cas e calculate b2 – 4ac and for which each of the following equations
describe the nature of the roots as either: has:
a. Real 2. Equal 3. No real roots. a. equal roots b. real roots c. no real roots
1. x2 – kx + 1 = 0
Nature 2. 8x2 – kx + 2 = 0
Equation b – 4ac
2
of roots 3. k x2 + (2k + 1)x + k = 0
1 x2 – 2x – 8 = 0 4. (k + 3)x2 + (6 – 2 k)x + (k – 1) = 0
2 x2 – 10x + 25 = 0 5. k x2 + kx + (k – 1) = 0
3 2x2 + 3x + 6 = 0 6. 4x2 + 2(k + 1)x + k2 = 0
4 (p -3) x2 + (2p – 1)
x + (p + 2) = 0 Challenge problems
5 (t + 1) x2 + (2t + 3) 1. Verify that the roots of the equation;
x + (t + 2) = 0 x2 – 3x = q2 – 2 are real for all q R.
6 x2 + 2px + p = 0
7 x2 + qx + q2 = 0 2. Verify that the roots of the equation:
8 x2 – 3x + (2 – k2) = 0 x2 – (a + b)x + (ab – c2 ) = 0 are real for all a,
9 x2 – 2(k – 1)x + (k – b, c R.
1)2 = 0
1 px2 + (p + 1)x + 1 3. Show that the roots of the equation:
0 =0 x2 – (2p + 5)x + 2 (2p + 3 ) = 0 are real for all
values of p R. Verify that one of the roots is
independent of p and the other is not

Baffour Ba Series , Further Mathematics for Schools Page 129


5 POLYNOMIALS Baffour Ba Series
Meaning coefficient, the first term is called leading
An algebraic expression which is the sum of coefficient.
terms involving only positive integral powers
of x is called a polynomial expression in x. Types of Polynomials
Thus for all polynomials, the power of the 1. A monomial is a polynomial that has one
variable is positive. Examples of polynomials term. Example 5x2, -12….
are 2x + 3, x4 – 3x, 4x3 – 15x2, 2x2 – 3x + 5… 2. A binomial is a polynomial that has two
terms. Example 5x2 – 12, x4 – x2…
Note: 3. A trinomial is a polynomial that has three
1. There cannot be a negative or fractional terms. Examples x4 – x2 + 12…
power of x.
2. A polynomial is often denoted as f(x). Worked Examples
Identify the following polynomials as
Degree of a Polynomial monomial, binomial or trinomial and state its
The degree of a polynomial is the highest degree.
power of the variable in the polynomial. Thus, 1. 5x2 – 10x3 + 6 2. x7 – x4 3. 8x 4. -15
2x + 3 has degree 1, x4 – 3x has degree 4, 2x2 –
3x + 5 has degree 2. Solutions
1. 5x2 – 10x3 + 6 is a trinomial with degree 3
The degree of a term is the power of the 2. x7 – x4 a binomial with degree 7
variable on the term. For example in 2x2 – 3x + 3. 8x is a monomial with degree 1
5, 2x2 is the first term with degree 2, -3x is the 4. -15 is a monomial with degree 0
second term with degree 1 and 5 is the third
term with degree 0. Evaluating Polynomials
Like other algebraic expressions involving
A number without a variable is called a variables, a polynomial has no specific value
constant term. Thus in 2x2 – 3x + 5, the unless the variables are replaced by numbers.
constant term is 5. The number preceding the
variable in each term is called the coefficient of A polynomial can be evaluated with or without
that term. In 2x2 – 3x + 5, 2 is the coefficient of the function notation, f(x).
x2, -3 is the coefficient of x.
Worked Examples
In general a polynomial is written in decreasing 1. Find the value of -3x4 – x3 + 20x + 3, when
order of exponent of the variable. Therefore, x = 1.
instead of - 4x2 + 1 + 5x + x3, we write:
x3 – 4x2 + 5x + 1. Solution
-3x4 – x3 + 20x + 3
When a polynomial is written with decreasing When x = 1,
Baffour Ba Series , Further Mathematics for Schools Page 130
= -3(1)4 – (1)3 + 20(1) + 3 Worked Examples
= -3 – 1 + 20 + 3 Perform the following:
= 19 1. (x2 – 6x + 5) + (-3x2 + 5x – 9)
2. (x2 – 5x – 3) – (4x2 + 8x – 9)
2. Find the value of x3 + x2 + x – 4 when x = -2. 3. (-5a3 + 3a – 7) + (4a2 – 3a + 7)
4. (4x3 – 3x + 2) – (5x2 – 7x – 6)
Solution
x3 + x2 + x – 4, Solutions
When x = -2, Method 1( Horizontal)
= (-2)3 + (-2)2 + (-2) – 4 1. (x2 – 6x + 5) + (-3x2 + 5x – 9)
= -8 + 4 – 2 – 4 = x2 – 6x + 5 – 3x2 + 5x – 9 (Remove brackets)
= -10 = x2 – 3x2 – 6x + 5x + 5 – 9 (Group like terms)
= -2x2 – x – 4 (simplify)
3. If f(x) = 3x2 – 4x + 2, find f(-3).
Method 2 (Vertical)
Solution
x2 – 6x + 5
f(x) = 3x2 – 4x + 2, +
-3x2 + 5x – 9
f(-3) = 3(-3)2 – 4(-3) + 2,
= 27 + 12 + 2 -2x2 – x – 4
= 41
2. (x2 – 5x – 3) – (4x2 + 8x – 9)
Exercises 5.1
Evaluate the following: Solution
1. f(x) = 3x4 – 2x3 + 7, f(-2) Method 1( Horizontal)
2. f(x) = -2x3 + 5x2 – 12 , f(5) 1. (x2 – 5x – 3) – (4x2 + 8x – 9)
3. 1.2x3 – 4.3x – 2.4, f(1.45) x2 – 5x – 3 – 4x2 – 8x + 9
4. -3.5x4 – 4.6x3 + 5.5, f(-2.36) = x2 – 4x2 – 5x – 8x – 3 + 9
5. 2x2 – 3x + 1, x = -1 = – 3x2 – 13x + 6
6. -3x3 – x2 + 3x – 4, x=3
3. (-5a3 + 3a – 7) + (4a2 – 3a + 7)
Addition and Subtraction of Polynomials
To add or subtract two or more polynomials: Solution
1. Identify the like terms and regroup them. Method 1
2. Create in missing terms. -5a3 + 3a – 7 + 4a2 – 3a + 7
3. Add or subtract the like terms accordingly. -5a3 + 0a2 + 3a – 7 + 0a3 + 4a2 – 3a + 7
Polynomials can be added or subtracted (create missing terms)

horizontally or vertically. In the vertical form, -5a3 + 0a3 + 0a2 + 4a2 + 3a – 3a – 7 + 7


(Group like terms)
line up the like terms.
-5a3 + 4a2 (Simplify)

Baffour Ba Series , Further Mathematics for Schools Page 131


Method 2 (Vertical) Multiplication of Polynomials
Reminders:
-5a3 + 0a2 + 3a – 7
1. x3 = x × x × x
0a3 + 4a2 – 3a + 7
2. x5 = x × x × x × x × x
-5a3 + 4a2 ⇒x3 × x5 = x8

4. (4x3 – 3x + 2) – (5x2 – 7x – 6) In general, if a is any real number and m and n


any positive integers, then am × an = am+ n. This
Method 1 is called the product rule.
(4x3 – 3x + 2) – (5x2 – 7x – 6)
= 4x3 – 3x + 2 – 5x2 + 7x + 6 Monomials
= 4x3 + 0x2 – 3x + 2 – 0x3 + 5x2 + 7x + 6 Steps:
= 4x3 – 0x3 + 5x2 + 0x2 – 3x + 7x + 2 + 6 1. Multiply integers and multiply variables
= 4x3 + 5x2 + 4x + 8 using the product rule.
2. Expand brackets if any.
Exercises 5.2
A. Perform the following: Worked Examples
1. (4x – 1) + (x3 + 5x – 6) 1. Find the product 3x2 (x3 – 4x)
2. (3x – 7) + (x2 – 4x + 6)
3. (x2 – 3x + 1) + (2x2 – 4x – 5) Solution
4. (x2 – 9x – 3) + (x – 4x + 8) 3x2 (x3 – 4x)
= (3x2 ) (x3 ) – (3x2 ) (4x)
5. (x3 – x2 – 5x) + (6 – x – 3x2)
= 3x5 – 12x3
6. (5.76x2 – 3.14x –7.09) + (3.9x2 + 1.21x + 5.6)
2. Find the product; (y3 – 3y + 4) (-2y)
B. Perform the following:
1. (2x2 – 3x) – (3x2 – 5x) Solution
2. (x5 – x3) – (-x4 + x4) (y3 – 3y + 4) (-2y)
3. (x2 – 3x + 4) – (x2 – 5x – 9) = (-2y) (y3 – 3y + 4)
4. (4 – 5x + x3) – (2 – 3x + x2) = (-2y) (y3) – (-2y) (3y ) + (-2y) (4)
5. (9 – 3x – x2) – (2 + 5x – x2) = -2y4 + 6y2 – 8y

C. Perform the indicated operator; 3. Multiply (x + 2) (x + 5)


1. (4m – 2) + (2m + 4) – (9m – 1)
2. (-5y – 1) – (8y – 4) – (y + 3) Solution
3. (-x3 – 5 x + 4) + (6x2 – 8x + 9) – (3x2 – 7x + 1) (x + 2) (x + 5)
4. (-8x2 + 5x – 12) + (-3x2 – 9x + 18) – (3x2 + = x (x + 5) + 2 (x + 5)
9x – 4) = x2 + 5x + 2x + 10
5. (-x3 – x2 – 1) – (x4 – x2 – x – 1) + (x3 – 3x2 + 6) = x2 + 7x + 10

Baffour Ba Series , Further Mathematics for Schools Page 132


4. Find the product ; (x + 3) (x2 + 2x – 7) 3. (x2 – 7x – 6 ) (7x2 – 3x – 7)
4. (5x + 2 ) (3x6 + 2x5 + 5)
Solution 5. (4x2 + x + 5) (3x6 + 2x5 + 5)
(x + 3) (x2 + 2x – 7)
= x (x2 + 2x – 7) + 3 (x2 + 2x – 7) Opposite of a Polynomial
= x3 + 2x2 – 7x + 3x2 + 6x – 21 The opposite or additive inverse of x is – x
= x3 + 2x2 + 3x2 – 7x + 6x – 21 because x + (-x) = 0.
= x3 + 5x2 – x – 21
To find the opposite of a polynomial, change
5. Find each product; the sign of every term of the polynomial. For
a. . /. / b. (x – 1) (x + 3) (x – 4) example, the opposite of x2 – 3x + 1 = - x2 + 3x – 1

Note;
Solution
The opposite of a + b ≠ b + a because, (a + b)
a. . /. / + (a – b) = 2a. Rather the opposite of (a + b) =
. /–2 . / - (a + b) = - a – b

x2 + – x–2 Worked Examples


2 Find the opposite of each polynomial
x – x–2
1. 9 – y2 2. a + 4 3. – x2 + 6x – 3
b. (x – 1) (x + 3) (x – 4)
Solution
= [(x – 1) (x + 3)] (x – 4)
1. 9 – y2
= x (x + 3) – 1 (x + 3)] (x – 4)
= -(9 – y2)
= (x2 + 3x – x – 3) (x – 4)
= - 9 + y2
= (x2 + 2x – 3) (x – 4)
= y2 – 9
= x2 (x – 4) + 2x(x – 4) – 3(x – 4)
= x3 – 4x2 + 2x2 – 8x- 3x + 12 2. a + 4
= x3 – 2x2 – 11x + 12 = -(a + 4)
= -9 – 4
Note:
Generally, to multiply polynomials, multiply
3. – x2 + 6x – 3
each term of the polynomial by every term of
= - (– x2 + 6x – 3)
the other polynomial, then combine like terms
= x2 – 6x + 3
if any.
Word Problems Involving Popynomials
Exercises 5.3 i. Decide if the problem involves addition,
Find the following product: subtraction or multiplication.
1. (7x2 – 6x – 6 ) (2x – 4) ii. If polynomials are given, add, subtract or
2. (x2 – 6x – 5 ) (7x2 + 6x – 5) multiply like normal.
Baffour Ba Series , Further Mathematics for Schools Page 133
iii. If polynomials are not given, use formula: Length of the new garden = x + 7
Area = L × B Width of the new garden = x – 2
Perimeter = Addition of all sides
Area of new garden;
Worked Examples = Length × Width
1. A parking space is 20 yards wide and 30 = (x + 7) (x – 2)
yards long. If the length and width is increased = x (x – 2) + 7 (x – 2)
by the same amount to handle an increasing = x2 – 2x + 7x – 14
number of cars, = x2 + 5x – 14
i. What polynomial represents the area of the
new space? Exercises 5.4
ii. What is the new area of the increase is 15 1. A triangle has threes sides with lengths 8x +
yards? 6, 5x – 5 and – 6x + 12. What is the perimeter
of the triangle?
Solution
i. Let the amount of increase be x. Then; 2. The dimensions of a window are 3x + 10 and
New length = x + 30 yards 2x + 6. What is the area of the window?
New width = x + 20 yards
3. Find the area of the figure below.
Area = Length × Width 15 x
= (x + 30) (x + 20) 4
= x (x + 20) + 30 (x + 20)
= x2 + 20x + 30x + 600 x
= x2 + 50x + 600
Division of Polynomials
2
ii. If x = 15, substitute in x + 50x + 600 There are two main methods of performing
⇒ (15)2 + 50(15) + 600 division of polynomials:
= 1575 square yards 1. The long division method.
2. The synthetic method.
2. Granny has a square garden with sides of
length x meters. If he increases the length by 1. The long Division Method
7m and decreases the width by 2m, then what If g(x) is a factor of f(x), then f(x) is divisible by
trinomial represents the area of the new garden? g(x). For example, if 2 is a factor of 6, then 6 is
divisible by 2. Likewise, x4 – 16 is divisible by
Solution x2 – 4, x2 + 4, x + 2, x – 2.
x
The polynomial x4 – 16 is not divisible by x2 +
x+7
3x + 1; however, the process of long division
x can be used to find a quotient and a remainder.

Baffour Ba Series , Further Mathematics for Schools Page 134


Steps
In the long division method, place the divisor in x2
front of a table and the dividend inside the x–2 x3 – 5x2 + 4x – 3
tables, creating in all missing terms as shown - x3 – 2x2
below: -3x2 + 4x – 3 Remainder
Divisor Dividend
Step 4: Divide the remainder by the divisor by
The long division process ends when we arrive performing -3x2 ÷ x = -3x, and record the
at a polynomial (the remainder) that is either 0 answer on top of the table as shown below.
or has a smaller degree than the divisor. The x2 – 3x
results of long division of f(x) by p(x) can be x–2 x3 – 5x2 + 4x – 3
expressed as: x3 – 2x2
f(x) = p(x) . q(x) + r(x) , where q(x) is the
-3x2 + 4x – 3 Remainder
quotient, and r(x) is the remainder.
Step 5. Multiply -3x by the divisor. That is;
Worked Examples -3x(x – 2) = -3x2 + 6x and record the answer
1. Divide x3 – 5x2 + 4x – 3 by x – 2 under the remainder and subtract to obtain
another remainder as shown below;
Solution x2 – 3x
(x3 – 5x2 + 4x – 3) ÷ x – 2 x–2 x-3 – 5x2 + 4x – 3
Step 1: Organize the work as shown below: 6 -3 x3 – 2x2
0 3
x – 2x x3 – 5x2 + 4x – 3 -3x2 + 4x – 3 R
6
-6 -3 -3x3 + 6x
0 Perform
Step 2: 3 x3 ÷ x = x2 and record the -2x – 3 R
6 on top of the table as shown below:
answer
Step 6. Divide the remainder by the divisor by
x2 performing -2x ÷ x = -2 and record the answer
x–2 x3 – 5x2 + 4x – 3 on top of the table as shown below.
x2 – 3x – 2
Step 3 : Multiply x2 (answer recorded) by the x–2 x-3 – 5x2 + 4x – 3
divided (x – 2). That is x2 (x – 2) = x3 – 2x2, 6 -3 x3 – 2x2
then record the answer at the bottom of the 0 3
dividend, and subtract from the dividend to -3x2 + 4x – 3 R
6
obtain the remainder as shown below. -3x3 + 6x
-2x – 3 R

Baffour Ba Series , Further Mathematics for Schools Page 135


Step 7. Multiply the answer recorded by the 2. The Synthetic Method
divisor. That is -2(x – 2) = -2x + 4 and record Strategy for Synthetic Division
the answer under the remainder, and subtract The strategy for getting the quotient Q(x) and
form the remainder. remainder R by synthetic division can be stated
as follows:
x2 – 3x – 2
1. List the coefficient of the polynomial
x–2 x3 – 5x2 + 4x – 3 (dividend).
x3 – 2x2 2. Be sure to include zeros for any missing
-3x2 + 4x – 3 Remainder terms in the dividend.
3x2 + 6x 3. For dividing x – c, place c to the left.
-2x – 3 Remainder 4. Bring the first coefficient down.
-2x + 4 5. Multiply by c and add for each column.
-7 Remainder 6. Read Q(x) and R from the bottom row.
⇒ – = (x2 – 3x – 2) + . /
– Note:
Multiply through by x – 2;
Synthetic division is used only for dividing a
x3 – 5x2 + 4x + 3 = (x2 – 3x – 2) (x – 2 ) + (-7) polynomial by a binomial x – c, where c is a
constant. For instance, if the binomial is x – 7,
2. Perform x4 – 16 ÷ (x2 + 3x + 1) then c = 7, and if the binomial is x + 7, then c = -7

Solution Worked Examples


1. Divide x3 – 5x2 + 4x – 3 by x – 2
x4 – 16 ÷ (x2 + 3x + 1) =
Set out the work as follows: Solution
x3 – 5x2 + 4x – 3 by x – 2
x2 – 3x + 8
x3 – 5x2 + 4x – 3 (dividend)
x – 2 x4 + 0x3 + 0x2 + 0x – 16
x–2 (divisor)
x4 + 3x3 + x2
-3x3 - x2 List all the coefficient of the dividend in order
-3x3 – 9x2 - 3x of ascending exponents.
8x2 + 3x – 16 1, -5, 4 and -3
8x2 + 24x + 8
- 21x2 – 24 From the divisor x – 2, use 2 and arrange as
follows:
= (x2 - 3x + 8) + . /
2 1 –5 4 -3
Multiplying through by x2 + 3x + 1
x4 – 16 = (x2 – 3x + 8 ) (x2 + 3x + 1 ) + (-21x – 24) Bring the first coefficient, 1, straight down.

Baffour Ba Series , Further Mathematics for Schools Page 136


Exercises 5.5
1 –5 4 -3 A. Find the quotient and remainder if f (x) is
2 ↓ Bring down divisible by p(x).
Then multiply
1 the 1 by the 2 from the divisor, 1. f(x) = 2x4 - x3 – x2 + 7x – 12; p(x) = x2 – 3
place the answer under -5 then add that column. 2. f(x) = 3x4 + 2x3 – x2 – x – 6; p(x) = x2 + 1
3. f(x) = 3x3 + 2x – 4; p(x) = 2x2 + 1
1 –5 4 -3 4. f(x) = 3x3 – 5x2 – 4x – 8; p(x) = 2x2 + x
2
2 Add 5. f(x) = 7x2 + 3x – 10; p(x) = x2 – x + 10
1 -3
Repeat the multiply – add – step for each of the B. Use synthetic division to find the quotient
remaining columns: and remainder if the first polynomial is
divided by the second.
1 –5 4 -3 1. f(x) = 2x3 – 3x2 + 4x – 5; p(x) = x – 2
2
2 -6 -4 2. f(x) = 3x3 – 4x2 - x + 8; p(x) = x + 4
3
Multiply 3. f(x) = x – 8x – 5; p(x) = x + 3
1 -3 -2 -7 ←Remainder 3 2
4. f(x) = 5x – 6x + 15; p(x) = x – 4
5 2
Quotient 5. f(x) = 3x + 6x + 7; p(x) = x + 2
4
From the bottom read the quotient and 6. f(x) = -2x + 10x – 3; p(x) = x – 3
remainder. Since the degree of the quotient is 7. f(x) = 4x4 – 5x2 + 1; p(x) = x –
one less than the degree of the dividend, the
8. f(x) = 9x3 – 6x2 + 3x – 4; p(x) = x –
quotient is 1x2 – 3x – 2. The remainder is – 7

The Remainder Theorem


2. Find the quotient and remainder when 2x4 –
Generally, when a polynomial is divided by a
5x2 + 6x – 9 is divided x + 2.
linear expression, there is a remainder.
Any polynomial can be written in the form:
Solution
Polynomial = (Divisor)(quotient) + Remainder
2x4 – 5x2 + 6x – 9 ÷ (x + 2)
⇒2x4 – 5x2 + 6x – 9 = 2x4 + 0x3 – 5x2 + 6x – 9
In particular, if the divisor is x – c and the
Coefficients = 2, 0, -5, 6, -9
polynomial is f(x), then;
f(x) = (x – c)(quotient) + Remainder.
-2 2 0 -5 6 -9
If x = c, then,
-4 8 -6 0
f(c) = (c – c)(quotient) + Remainder
2 -4 3 0 -9 f(c) = Remainder

Because the degree of the dividend is 4, the This provides an easy way of determining the
quotient has a degree 3. remainder when a polynomial is divided by
The quotient is 2x3 – 4x2 + 3x and the x–c
remainder is -9.

Baffour Ba Series , Further Mathematics for Schools Page 137


Worked Examples = 3. / + 5. / – . / + 1
1. Find the remainder when 3x3 + 4x2 – 5x – 2 is
divided by (x – 1) = + – +1=

Solution Exercises 5.6


Let f(x) 3x3 + 4x2 – 5x – 2 A. Find the remainder of the following:
From x – 1 = 0, x = 1 1. f(x) = x3 – 5x2 + 6x – 4 ÷ (x – 2)
f(1) = 3(1)3 + 4(1)2 – 5(1) – 2 2. f(x) = 4x3 + 3x2 + x + 2 ÷ (x – 1)
=3+4–5–2=0 3. f(x) = 2x4 – x3 + 3x2 – 1 ÷ (x + 1)
⇒The remainder is 0
4. f(x) = 2x3 – 6x – 5 ÷ (x + 3)
(This means that x – 1 is a factor of f(x))
5. f(x) = x3 – 4x2 – x ÷ (x – 4)
2. Find the remainder when f(x) = x4 + 4x2 + 3x
– 7 is divide by (x + 3) B. Use the remainder theorem to determine
the remainder when g(x) is divided by h(x).
Solution 1. g(x) = x3 + 4x2 + 11x – 5, h(x) = x – 1
f(x) = x4 + 4x2 + 3x – 7 2. g(x) = 2x3 – 5x2 + 8, h(x) = 2x – 1
From (x + 3) = 0, x = -3 3. g(x) = 4x3 + 5x2 + 6x – 1, h(x) = x + 2
f(-3) = (-3)4 + 4(-3)2 + 3(-3) – 7 4. g(x) = -5x3 – x2 – 10x + 9, h(x) = 5x + 1
= 81 + 36 – 9 – 7
5. g(x) = x4 + 5x2 + 2x – 8, h(x) = x + 1
= 101
6. g(x) = 3x5 – 8x4 + x2 + 2, h(x) = 2 – x
3 2
3. Find the remainder when f(x) = 2x – 5x + x
– 3 is divide by (x – 1) Using the Remainder Theorem to find One
Unknown Variable of a Polynomial
Solution If the given polynomial f(x), involves one
f(x) = 2x3 – 5x2 + x – 3 unknown variable, then one given divisor of
From (x –1) = 0, x = 1 f(x) can help to determine the value of the
f(1) = 2(1)3 – 5(1)2 + (1) – 3 unknown variable (constant).
=2–5+1–3=-5
If the polynomial f(x) divided by (x - c) gives a
3
4. Determine the remainder when f(x) = 3x + remainder, R, then form an equation involving
5x2 – x + 1 is divided by (2x – 1) f(c) = R and solve to obtain the value of the
unknown variable (constant).
Solution
f(x) = 3x3 + 5x2 – x + 1 Worked Examples
(2x – 1) = 0, x = 1. Determine the value of p if 18x3 + px2 – 8x +
9, is divided by 2x – 1 and gives a remainder of 6.
f. / = 3. / + 5. / – . / + 1

Baffour Ba Series , Further Mathematics for Schools Page 138


Solution 2b = 5 – 20
Let f(x) =18x3 + px2 - 8x + 9 2b = - 15
2x – 1 = 0, x = and R = 6 b=

f. / = 18. / + p. / – 8. / + 9
3. Calculate the value of m if 2x3 – 7x2 + mx – 26
= 18. / + p. / – 8. / + 9 is divided by x – 2, and gives a remainder of – 24.

= + – +9 Solution
= + –4+9 Let f(x) = 2x3 - 7x2 + mx – 26
x – 2 = 0, x = 2 and R = -24
= + +5
f(2) = 2(2)3 – 7(2)2 + 2m – 26
= 16 – 28 + 2m – 26
But f. / = 6
= 2m – 38
+ +5=6
But f(2) = -24
=6–5–
⇒2m – 38 = -24
=1– 2m = -24 + 38
= 2m = 14
m=7
p=
p = -5 4. If x5 – 2x3 – kx – 1 is divided by x – 1 and the
remainder is , find the value of k.
3 2
2. If x + x – x + b is divided by x – 2 and the
remainder is 2 , calculate the value of b. Solution
Let f(x) = x5 – 2x3 – kx – 1
Solution x – 1 = 0, x = 1 and R =
Let f(x) = x3 + x2 – x + b f(1) = (1)5 – 2(1)3 – k(1) – 1
x – 2 = 0, x = 2 and R = 2 =1–2–k–1
f(2) = (2)3 + (2)2 – 2 + b = -2 – k
=8+4–2+b
= 10 + b But f(1) =
⇒ -2 – k =
But f(2) = 2
-2(2) – (2)k = 1
10 + b = 2 -4 – 2k = 1
10 + b = -2k = 1 + 4
-2k = 5
2(10) + 2b = 5
20 + 2b = 5 k=

Baffour Ba Series , Further Mathematics for Schools Page 139


5. If x3 + 8x2 + mx – 5 is divided by x + 1 and Using the Remainder Theorem to find Two
the remainder is n, express m in terms of n. Unknown Variables of a Polynomial
If the given polynomial f(x), involve two
Solution unknown variables, then two given divisors of
Let f(x) = x3 + 8x2 + mx – 5 f(x) can help determine the value of the
x + 1 = 0, x = -1 and R = n unknown variables (constants).
f(-1) = (-1)3 + 8(-1)2 + m(-1) – 5
= -1+ 8 - m – 5 If the polynomial f(x) divided by (x - c) and
=2–m (x – k) gives the remainders, R1 and R2
respectively, then form an equation involving
But f(-1) = n f(c) = R1 and f(k) = R2 and solve to obtain the
n=2–m values of the unknown variables (constants).
m=2–n
Worked Examples
Exercises 5.7 1. When the polynomial 2x3 + px2 + qx + 1 is
1. Determine the value of p if 18x3 + px2 – 8x + divided by x + 1 or x – 4, the remainder is 5.
9 is divided by 2x – 1 and gives a remainder of 6. Determine the values of p and q.

2. Calculate the value of h if 3x5 + hx4 + 10x2 – Solution


21x + 12 is divided by x – 2 and gives a Let f(x) = 2x3 + px2 + qx + 1
remainder of 10. For x + 1 = 0, x = -1 and R = 5
f(-1) = 2(-1)3 + p(-1)2 + q(-1) + 1
3. When x3 + kx2 – 4x + 2 is divided by x + 2 = -2 + p – q + 1
the remainder is 26, find k. = -1 – p – q

4. When 2x3 – 3x2 + kx – 1 is divided by x – 1 But f(-1) = 5


the remainder is 2, find k. ⇒ 5 = -1 + p – q
5+1=p–q
5. The expression 4x2 – px + 7 leaves a 6=p–q
remainder of -2 when divided by x – 3. Find the p = 6 + q ………………..(1)
value of p.
For x – 4 = 0, x = 4 and R = 5
f(4) = 2(4)3 + p(4)2 + q(4) + 1
6. Let f(x) = (3x – 2) (x – k) – 8, where k is a
= 128 + 16p + 4q + 1
constant; = 129 + 16p +4q
a. Write down the value of f(k);
But f(4) = 5
b. When f(x) is divided by (x – 2), the
⇒ 5 = 129 + 16p +4q
remainder is 4, find the value of k. 5 – 129 = 16p + 4q

Baffour Ba Series , Further Mathematics for Schools Page 140


-124 = 16p + 4q ………………..(2) p – q = 1…………….(2)

Put eqn (1) into eqn (2); eqn (1) – eqn ( 2);
-124 = 16(6 + q) + 4q 2q = 8
-124 = 96 + 16q + 4q q=4
-124 – 24 = 16q + 4q
-220 = 20q Put q = 4 in eqn (1);
q = -11 p+4=9
p=9–4
Put q = -11 in eqn (1); p=5
p = 6 + (-11)
p = -5 Exercises 5.8
1. When the polynomial 3x3 + ax2 + bx – 9 is
2. If 3x3 + px2 + qx + 1 has a remainder 7x + 6 divided by x – 2 the remainder is -5. When it is
when divided by x2 – 1, find the values of p and q. divided by (x + 1) the remainder is -16. Find the
values of a and b.
Solution
Let f(x) = 3x3 + px2 + qx + 1 2. When the polynomial x4 – 4x3 + ax2 + bx + 1
x2 – 1 = (x – 1) ( x + 1 ) = 0 is divided by x – 1 the remainder is 7. When it
is divided by (x + 1) the remainder is 3. Find
When x = 1 the values of a and b.
f(1) = 3(1)3 + p (1)2 + q(1) + 1
=3+p+q+1 3. When the polynomial f (x) = x3 – px2 + qx + 2
=4+p+q where p and q are constants is divided by
(x – 2) the remainder is 2. When divided by
But f(1) = 7x + 6 (x + 1) the remainder is – 10.
= 7(1) + 6 = 13 a. Find the values of p and q.
⇒ 4 + p + q = 13 b. Hence, find f (-2)
p + q = 9……………(1)
4. Let f (x) = x3 – 2x2 + ax + b, where a and b
When x = -1 are constants. When f (x) is divided by (x – 2),
f(-1) = 3(-1)3 + p(-1)2 + q(-1) + 1 the remainder is 1 and when f (x) is divided by
= -3 – p – q + 1 (x + 1), the remainder is 28. Determine the
= -2 + p – q values of a and b.

But f(-1) = 7x + 6 5. Let f (x) = (x2 + p) (2x + 3) + 3, where p is a


= 7(-1) + 6 constant.
= -1 a. Write down the remainder when f(x) is
⇒ -2 + p – q = -1 divided by (2x + 3)

Baffour Ba Series , Further Mathematics for Schools Page 141


b. Given that the remainder when f(x) is divided (x – 3) and (x – 4). Find the remainder when
by (x – 2) is 24, prove that p = -1. f(x) divided (x – 3) ( x – 4).

Other Applications 3. When a polynomial P(x) is divided by x2 – 1,


Worked Example the remainder is 3x – 1. What is the remainder
1. When the polynomial f (x) is divided by (x – when P(x) is divided by x – 1?
1) and (x – 2), it leaves a remainder of 5 and 7
respectively. What is the remainder when Factor Theorem
divided by (x – 1) ( x – 2)? If an algebraic expression is divided by one of
its factors, then the remainder is zero. The
Solution expression (x – k) is a factor of the polynomial
f(x) = (Quotient) (Divisor) + Remainder f(x), if the remainder when f (x) is divided by
f(x) = Q(x) (x – 1) (x – 2) + R (x – k) is zero.
Since the divisor, (x – 1) (x – 2), has a degree of
2, it means the remainder has a degree 1, in the Generalizing this:
form ax + b 1. If f (k) = 0, then (x – k) is a factor of f (x).
2. If (x – k) is a factor of f(x), then f (k) = 0
f(x) = Q(x) (x – 1) (x – 2) + ax + b
f(1) = Q(1) (0) (x – 2) + a(1) + b = 5 The factor theorem can be extended to include
a + b = 5 ……….(1) the following:
f(2) = Q(1) (x – 1 ) (0) + a(2) + b = 7 1. If . / = 0, then (ax – b) is a factor of f(x).
2a + b = 7 ……….(2)
2. If (ax – b) is a factor of f(x), then f . / = 0.
Solving eqn (1) and (2); 3. If (x + a) and (x + b) are both factors of a
a = 2 and b = 3 polynomial f (x), then so is their product, (x +
a) and (x + b) = x2 + abx + ab, also a factor,
Substitute in R(x) = ax + b and vice versa.
R(x) = 2x + 3
The remainder is 2x + 3 The factor theorem can be used to factorize
polynomials or to find unknown coefficients in
Exercises a polynomial. Here are some examples:
1. When the polynomial f (x) is divided by (x –
1) and (x – 2), we get a remainder of 2 and 1 Factor Put factor = Factor
respectively. Find the remainder when f(x) 0 and solve theorem
divided by (x – 1) ( x – 2)? x+5 x = -5 f (-5)
x–2 x=2 f (2)
3x + 1 x=- f (- )
2. The remainder of a polynomial f(x) in x are
10 and 15 repectively when f(x) is divided by

Baffour Ba Series , Further Mathematics for Schools Page 142


Verifying the Factor Theorem 2. f(x) = 4x3 – 9x2 – 8x – 3, c=3
Worked Examples
3. f(x) = 4x3 – 6x2 + 8x – 3 , c=
1. Show that x – 2 is a factor of f(x) = x3 – 4x2 +
3x + 2 4. (x) = 27x4 – 9x3 + 3x2 + 6x + 1, c=-

Solution C. 1. Verify that (x – 1) is a factor of the


Method 1 polynomial x3 + 2x2 – x – 2 and find the other
f(x) = x3 – 4x2 + 3x + 2 two factors.
x–2=0 2. Verify that (x + 1) is a factor of the
x=2 polynomial x3 + 9x2 + 23x + 15 and find the
If x – 2 is a factor of f(x), the f(2) = 0 other two factors.
f(2) = (2)3 – 4(2)2 + 3(2) + 2
f(2) = 8 – 16 + 6 + 2 3. Verify that (2x – 1) is a factor of the
=0 polynomial 6x3 + 7x2 – 9x + 2 and find the
x – 2 is a factor of f(x) = x3 – 4x2 + 3x + 2 other two factors.
4. Verify that (x – 1) is a factor of 2x3 – 15x2 +
Method 2 (Synthetic Division) 34x – 24 and find the other two factors.
x3 – 4x2 + 3x + 2 ÷ (x – 2) = 0
Coefficients = 1, - 4, 3, 2 Using the Factor Theorem to find one
2 1 -4 3 2 Unknown Variable of a Polynomial
If the given polynomial f(x), involves one
2 -4 -2
unknown variable, then one factor of f(x) can
1 -2 -1 0 help to determine the value of the unknown
variable.
Since the remainder is 0, x – 2 is a factor of
f(x) = x3 – 4x2 + 3x + 2 If (x – c) are factors of the polynomial f(x), then
form an equation involving the f(c) = 0 and
Exercises 5.9 solve it to obtain the values of the constants.
A. Use the factor theorem to show that x – c
is a factor of f(x). Worked Examples
1. f(x) = x3 + x2 – 2x + 12, c = -3 1. If (2x – 1) is a factor of the polynomial
3 2
2. f(x) = x + x – 11x + 10, c=2 f(x) = 2x3 – 5x2 – kx + 3, find the value of k.
3. f(x) = x12 – 4096, c = -2 Hence, find the other two factors.
4. f(x) = x4 – 3x3 – 2x2 + 5x + 6, c = -2
Solution
f(x) = 2x3 – 5x2 – kx + 3
B. Use synthetic division to show that c is a
If (2x – 1) is a factor,
zero of f(x).
⇒ 2x – 1 = 0
1. f(x) = 3x4 + 8x3 – 2x2 – 10x + 4, c = -2
2x = 1

Baffour Ba Series , Further Mathematics for Schools Page 143


x= Since x + 2 = 0, x = -2
Since x + 2 is a factor of p(x),
⇒ f. / = 0 p(−2) = 0,
2 . / – 5 . / – k. / + 3 = 0 (−2)3 + k(−2)2 + (−2) − 6 = 0.
-8 + 4k – 2 – 6 = 0
2 . / – 5 . / – k. / + 3 = 0 -16 + 4k = 0
– – k+3=0 (Multiply through by 4) 4k = 16
k=4
1 – 5 – 2k + 12 = 0
p(x) = x3 + 4x2 + x − 6
-2k + 8 = 0
-2k = -8
ii. Using long division, we get
k=4

3 2
x2 + 2x – 3
Substitute k = 4 in f(x) = 2x – 5x – kx + 3
x+2 x3 + 4x2 + x − 6
⇒ f(x) = 2x3 – 5x2 – 4x + 3
x3 + 2x2
Now, divide 2x3 – 5x2 – 4x + 3 by (2x – 1)
2x2 + x
x2 – 2x – 3
2x2 + 4x
2x – 1 2x3 – 5x2 – 4x + 3
-3x – 6
2x3 - x2
-3x – 6
- 4x2 – 4x
0
- 4x2 + 2x
x3 + 4x2 + x − 6 = (x + 2)(x2 + 2x − 3).
-6x + 3
-6x + 3
Now, factorizing (x2 + 2x − 3);
0 (x2 + 2x − 3) = (x + 3) (x – 1)
Now factorize x2 – 2x – 3
Therefore p(x) = (x + 2)(x + 3)(x − 1).
= x2 – 2x – 3
= (x2 – 3x) + (x – 3) 3. If (x – k) is a factor of the polynomial f(x) =
= x(x – 3) + 1(x – 3) 4x3 – (3k + 2)x2 – (k2 – 2)x + 3, find the
= (x + 1) (x – 3) possible values of k.
Thus, the other two factors are: (x + 1) (x – 3)
Solution
2. Let p(x) = x3 + kx2 + x − 6. Suppose that f(x) = 4x3 – (3k + 2)x2 – (k2 – 2)x + 3,
(x + 2) is a factor of p(x). If x – k is a factor, then x = k
i) Find the value of k ⇒ f(k) = 4(k)3 – (3k + 2) (k)2 – (k2 – 2) (k) + 3,
ii. With the value of k found in (1), factorize = 4k3 – (3k + 2)k2 – (k2 – 2) k + 3
p(x). = 4k3 – 3k3 – 2k2 – k3 +2k + 3
= - 2k2 +2k + 3
Solution
Since f(k) = 0
p(x) = x3 + kx2 + x − 6

Baffour Ba Series , Further Mathematics for Schools Page 144


⇒ - 2k2 +2k + 3 = 0 Using the Factor Theorem to find Two
2k2 – 2k – 3 = 0 Unknown Variables of a Polynomial
a = 2, b = -2 and c = -3 If the given polynomial f(x), involves two
√ unknown variables, then two given factors of
k=
f(x) can help to determine the value of the two
( ) √( ) ( )( )
k= unknown variables (constants).
( )

If (x – c) and (x – k) are factors of the
k= polynomial f(x), then form two equations
√ √ involving the f(c) = 0 and f(k) = 0 and solve
k= or k =
them simultaneously to obtain the values of the
k = 1.82 or k = - 0.82
two constants.

Exercises 5.10
Worked Examples
1. If (x + 2) is a factor of the polynomial
1. Let f(x) = 2x3 + mx2 + nx + 2 where m and n
f(x) = 6x3 + kx2 + 11x – 6, find the value of k.
are constants. Given that x – 1 and x + 2 are
Hence, find the other two factors.
factors of f(x), find the value of m and the value
2. If (2x + 1) is a factor of the polynomial of n.
f(x) = 2x3 + 7x2 + kx + 2, find the value of k.
Hence, find the other two factors. Solution
f(x) = 2x3 + mx2 + nx + 2
3. If x + 2 is a factor of ax2 + 4x, find the value If (x – 1) is a factor, then f(1) = 0
of a. f(1) = 2(1)3 + m(1)2 + n(1) + 2
f(1) = 2 + m + n + 2
4. Consider the polynomial function P(x) = x4 – But f(1) = 0
3x3 + ax2 – 6x + 14, where a is an unknown real ⇒m+n+4=0
number. If x – 2 is a factor of P(x), what is the m + n = - 4 ……………(1)
value of a?
If (x + 2) is a factor, then f(-2) = 0
5. Find the values of a, b and c so that the given f(-2) = 2(-2)3 + m(-2)2 + n(-2) + 2
linear expressions are factors of the following f(-2) = 2(-8) + 4m – 2n + 2
polynomials: = -16 + 4m – 2n + 2
a. x3 –7x2 – x + a, x–1 = -14 + 4m – 2n
3 2
b. 4x + bx + 3x + 2, 2x – 1
3 2
c. 8x – 2x + cx + 6, 2x + 3 But f(-2) = 0
⇒ -14 + 4m – 2n = 0
Challenge Problems 4m – 2n = 14 ……………(2)
Find all the values of k such that f(x) is
divisible by the given linear polynomial. m + n = - 4 ……………….(1)
1. f(x) = k2 + x3 – 4kx + 3; x–1 4m – 2n = 14 …………….(2)

Baffour Ba Series , Further Mathematics for Schools Page 145


eqn (1) × 4; Substitute a = 1, b = -3 and c = 2 in
4m + 4n = -16 ……………(3) p(x) = =0

eqn (3) – eqn (2); p(x) = =0


6n = - 30
n = -5 ii. p(x) = =0

Put n = -5 in eqn (1) For p(0) = -


( )
m + (-5) = - 4 ……………….(1) ⇒
( )
=-
( )
m–5=-4
m=-4+5 =-
m=1 k=-4
f(x) = 2x3 + x2 – 5x + 2
For p(3) = -2
2. A polynomial p(x) is of the form: ( ) ( )
⇒ =-2
( )
p(x) = , where a, b, c , m and k are
constants. But k = - 4
i. If the truth set of p(x) = 0 is {1, 2}, find the ( ) ( )
( )
=-2
values of a, b and c.
ii. If p(0) = - and p(3) = -2, using your results =-2

in (i) find the values of m and k. =-2


iii. Write down the expression for p(x) and state 2 = -2(3m – 4)
the value of x for which it is defined. 2 = -6m + 8
6m = 8 – 2
Solution 6m = 6
i. p(x) = m=1
p(x) = 0
iii. p(x) =
⇒ =0 (Multiply both sides by mx + k)
For p(x) to undefined;
ax2 + bx + c = 0
x–4=0
x=4
But truth set of p(x) = 0 is {1, 2}
p(x) is defined for all values of x except 4.
ax2 + bx + c = 0 has factors (x – 1) or (x – 2)
ax2 + bx + c = (x – 1)( x – 2) = 0
Exercises 5.11
ax2 + bx + c = x(x – 2) – 1 (x – 2)
1. Let f(x) = px3 + 3x2 – 9x + q, where p and q
ax2 + bx + c = x2 – 3x + 2
are constants. Given that (x + 1) and (x – 2) are
a = 1, b = -3 and c = 2 (Comparing coefficients)
factors of f(x), find the value of p and the value
of q.
Baffour Ba Series , Further Mathematics for Schools Page 146
2. Let p(x) = 2x3 – ax2 – bx + 42, where a and b x3 + 2x2 + px + q, where p and q are constants
are constants. Given that (x – 2) and (x + 3) are is (x + 1). The polynomial leaves a remainder
factors of p(x), find the values of a and b. of 12 when divided by (x – 2).
a. Find the values of p and q.
3. Let f(x) = x3 – (h + 2)x + 2k and p (x) = 2x3 b. With these values of p and q, factorize the
+ hx2 – 4x – k. Given that (x + 3) is a common polynomial completely.
factor of f (x) and p(x), find the value of h and
the value of k. Solution
a. Let f(x) = x3 + 2x2 + px + q
4. Factorize x2 + x – 6. For x + 1 = 0, x = -1
Let f(x) = px3 + x2 – 20x + q, where p and q are f(-1) = (-1)3 + 2(-1)2 + p(-1) + q
constants. Given that x2 + x – 6 is a factor of = -1 + 2 – p + q
f(x), find the value of p and the value of q. =1–p+q

5. Given that px3 + 8x2 + qx + 6 is exactly But f(-1) = 0


divisible by x2 – 2x – 3, find the value of p and 1–p+q=0
the value of q. p – q = 1…………..(1)

Challenge Problems For x – 2 = 0, x = 2


1. Find the values of a, b and c if x + 2, x – 3 f(2) = (2)3 + 2(2)2 + p(2) + q
and 2x + 1 are factors of ax3 + bx2 + cx – 6 = 0. = 8 + 8 + 2p + q
= 16 + 2p + q
Using a Combination of the Factor and
Remainder Theorems to find Unknowns But f(2) = 12
If the given polynomial f(x), involves two ⇒16 + 2p + q = 12
unknown variables, then a given factor and a 2p + q = 12 – 16
given remainder of f(x) can help to determine 2p + q = - 4…………..(2)
the value of the two unknown variables
(constants). eqn (1) + eqn (2);
3p = -3
If (x – c) is a factor of the polynomial f(x) and p = -1
(x – k) divides polynomial f(x) to give a
Put p = -1 in eqn (1);
remainder, R, then two equations involving f(c)
= 0 and f(k) = R can be solved simultaneously -1 – q = 1
to obtain the values of the two unknowns -q=1+1
(constants). -q=2
q = -2
Worked Examples
Put p = -1 and q = -2 in x3 + 2x2 + px + q
1. One of the factors of the polynomial
= x3 + 2x2 – x – 2
Baffour Ba Series , Further Mathematics for Schools Page 147
b. x3 + 2x2 – x – 2 If the remainder when f(x) is divided by (x + 3)
is 25;
By long division method; ⇒ f(- 3) = 25……………….(iii)

x2 + x – 2
3 2
f(-3) = (-3)3 – 3a + b
x + 1 x + 2x – x – 2 f(-3) = - 27 – 3a + b……….(iv)
x3 + x2
x2 – x eqn (iii) = eqn (iv)
x2 + x - 27 – 3a + b = 25
-3a + b = 25 + 27
-2x - 2
-2x - 2 - 3a + b = 52……………….(2)

0 eqn (2) – eqn (1)


x3 + 2x2 – x – 2 = ( x + 1) (x2 + x – 2) -5a = 60
Now, factorize x2 + x – 2 a = - 12
x2 + x – 2 = ( x2 + 2x) – ( x – 2)
x(x + 2) – 1 (x + 2) Put a = -12 in eqn (1)
(x – 1) (x + 2) 2(-12) + b = - 8
⇒ x3 + 2x2 – x – 2 = ( x + 1) (x – 1) (x + 2) - 24 + b = -8
b = - 8 + 24
2. (x – 2) is a factor of the polynomial f(x) = x3 b = 16
+ ax + b, where a and b are constants. The
Put a = -12 and b = 16 in f(x) = x3 + ax + b
remainder when f(x) is divided by (x + 3) is 25.
f(x) = x3 – 12x + 16
Find;
a. the values of the constants a and b.
b. f(x) = x3 – 12x + 16
b. the remainder when f(x) is divided by (x – 3).
f(3) = 33 – 12(3) + 16
c. the truth set of the equation f(x) = 0
= 27 – 36 + 16 = 7
The remainder when f(x) is divided by
Solution
(x – 3) is 7.
a. f(x) = x3 + ax + b
x2 + 2x – 8
If (x – 2) is a factor of the polynomial f(x)
⇒f(2) = 0…………………..(i) x – 2 x3 + 0x2 – 12x +16
x3 - 2x2
But f(2) = 23 + 2a + b 2x2 – 12x
f(2) = 8 + 2a + b…………..(ii) 2x2 - 4x
-8x + 16
eqn (i) = eqn (ii); -8x + 16
8 + 2a + b = 0
0
2a + b = -8 …………………(1)

Baffour Ba Series , Further Mathematics for Schools Page 148


x3 – 12x + 16 = (x – 2) (x2 + 2x – 8) = 0 But f(2) = 0
Now, factorize x2 + 2x – 8 = 0 8p + 4q + 2r – 12
(x2 + 4x) – (2x – 8) = 0 8p + 4q + 2r = 12……………..(3)
x(x + 4) – 2 (x + 4) = 0
(x – 2) (x + 4) = 0 Reduce eqns (1), (2) and (3);
9p + 3q + r = 14……………..(1)
x3 – 12x + 16 = (x – 2) ( x – 2) (x + 4) = 0 -9p + 3q – r = 4……………...(2)
⇒ x = 2 or x = - 4 4p + 2q + r = 6…………...….(3)

3. f(x) = px3 + qx2 + rx – 12 has a remainder of eqn (1) + eqn (2);


30 when divided by x – 3. If x2 – x – 6 is a 6q = 18
factor of f(x), find the values of p, q and r and q = 3……………………….(4)
hence the zeroes of f(x).
eqn (2) + eqn (3);
Solution - 5p + 5q = 10………………(5)
f(x) = px3 + qx2 + rx – 12
For x – 3 = 0, x = 3 Put q = 3 in eqn (5);
f(3) = p(3)3 + q(3)2 + r(3) – 12 - 5p + 5(3) = 10
= 27p + 9q + 3r – 12 - 5p + 15 = 10
- 5p = 10 – 15
But f(3) = 30 -5p = -5
27p + 9q + 3r – 12 = 30 p=1
27p + 9q + 3r = 42……………(1)
Put p = 1 and q = 3 in eqn (1);
Now, by factorization; 9(1) + 3(3) + r = 14
If x2 + x – 6 = (x + 3) (x – 2) 9 + 9 + r = 14
⇒ x + 3 and x - 2 are factors of f(x) r = 14 – 9 – 9
For x + 3 = 0, x = -3 r=-4

f(-3) = p(-3)3 + q(-3)2 + r(-3) – 12 Exercises 5.12


= -27p + 9q – 3r – 12 1. The polynomial 2x3 + qx2 + rx + 2 has a
factor (x – 1) and leaves a remainder of 12
But f(-3) = 0 when divided by x – 2. Find the constants q and
-27p + 9q – 3r – 12 = 0 r and hence the other two factors of the
-27p + 9q – 3r = 12…………….(2) polynomial.

For x – 2 = 0, x = 2 2. Find q, p and r if x + 3 and x – 3 are factors


f(2) = p(2)3 + q(2)2 + r(2) – 12 of f(x) = px3 – qx2 + rx – 18 and leaves a
= 8p + 4q + 2r – 12 remainder of 8 if f(x) is divisible by x – 2.

Baffour Ba Series , Further Mathematics for Schools Page 149


3. If f(x) = 2x3 + ax2 + bx – 6, where a and b are By factor theorem, f(x) has factors x – 2, x + 1
constants. When f(x) is divided by (2x – 1) the and x – 3 . Thus,
remainder is – 5 and when f(x) is divided by (x f(x) = a(x – 2) ( x + 1) (x – 3), where a has a
+ 2) there is no remainder. nonzero value.
a. find the values of a and b; If a = 1, then by multiplication,
b. factorize f(x) completely. f(x) = x3 – 4x2 + x + 6

4. Let f(x) = x4 + 5x3 + ax + b, where a and b Exercises 5.13


are constants. The remaninder when f(x) is Find the polynomial f(x) with leading
divided by (x – 2) is equal to the remainder coefficient 1 and having the following
when f(x) is divided by (x + 1). degrees and zeros
a. Find the value of a; 1. degree 3; zeros -2, 0, 5
b. Given that (x + 3) is a factor of f(x), find the 2. degree 3; zeros ±2, 3
value of b. 3. degree 4; zeros -2, , ±1, 4
4. degree 4; zeros -3, 0, 1, 5
5. Given that f(x) = 2x3 – x2 + 2x – 16, use the
factor theorem to show that (x – 2) is a factor of Solving Cubic Equations
f(x). Any equation of the form ax3 + bx2 + cx + d =
b. Given that f(x) = (x – 2) (2x2 + bx + c), find 0, a ≠ 0 is called a cubic equation. The factor
the values of b and c. theorem is used to find one root, and hence one
c. Hence prove that f(x) = 0 has only one real factor.
solution.
Steps in Solving a Cubic Equation
3 2
6. Let f(x) = x + (p + 1)x – 18x + q, where p 1. Find the first root, k, by trial and error
and q are integers. Given that (x – 4) is a factor method. i.e try f(1), f(-1), f(2), f(-2) etc
of f(x); 2. If x = k is a root, then c – k is a factor.
a. Show that 16p + q + 8 = 0 3. Divide f(x) by (x – k) which always gives a
b. Given that (x + p) is also a factor of f(x), and quadratic expression.
p > 0; show that p2 + 18x + q = 0. 4. Let the quadratic equal to zero and solve by
c. Hence find the value of p and the factorization or formula.
corresponding value of q.
d. factorize f(x) completely. Worked Examples
1. Solve the equation 2x3 + x2 – 13x + 6 = 0
Finding a Polynomial with Prescribed Zeros
Worked Example Solution
1. Find the polynomial with degree 3 that has Let f(x) = 2x3 + x2- 13x + 6
zeros 2, -1 and 3. The first root will be a factor of the constant
term, 6 = 1, 2, 3, 6
Solution f(1) = 2(1)3 + (1)2- 13(1) + 6

Baffour Ba Series , Further Mathematics for Schools Page 150


= 2 + 1 – 13 + 6
=-4 1 1 -4 -17 60
1 -3 - 20
f(-1) = 2(-1)3 + (-1)2- 13(-1) + 6
1 -3 - 20 40
= -2 +1 + 13 + 6
= 18 1 is not a solution to the equation because the
remainder is not 0.
f(2) = 2(2)3 + (2)2- 13(2) + 6
= 16 + 4 – 26 + 6 Test x = 2;
=0 2 1 -4 -17 60
Therefore, x – 2 is a factor.
2 -4 - 42

Now, divide 2x3 + x2- 13x + 6 by (x – 2) 1 -2 - 21 18


2
2x + 5x – 3
2 is not a solution to the equation because the
x–2 2x3 + x2- 13x + 6
remainder is not 0.
2x3 - 4x2
Test x = 3;
5x2 - 13x
5x2 - 10x 3 1 -4 -17 60

-3x + 6 3 -3 - 60
-3x + 6
1 -1 - 20 0
0
3 is a solution to the equation because the
2
Let 2x + 5x – 3 = 0 remainder is 0.
Solving by factorization, ⇒ x – 3 is a factor of the polynomial.
(2x – 1) (x + 3) = 0 Now x3 – 4x2 – 17x + 60 ÷ ( x – 3)
⇒ 2x – 1 = 0 or x + 3 = 0
x= or x = -3 x2 – x – 20
x - 3 x3 – 4x2 – 17x + 60
The three roots of the equation are : -3, , 2.
x3 – 3x2
3 2 - x2 – 17x
2. The equation x – 4x – 17x + 60 = 0 is
known to have a solution that is between -3 and - x2 + 3x
3 inclusive. Find the solution set. - 20x + 60
- 20x + 60
Solution
{-3, -2, -1, 0, 1, 2, 3} contains one of the ⇒ (x – 3) (x2 – x – 20 ) = 0
solution set. By using synthetic division, Let x2 – x – 20 = 0
Test x = 1 (x2 – 5x ) + (4x – 20) = 0

Baffour Ba Series , Further Mathematics for Schools Page 151


x(x – 5) + 4(x – 5) = 0 B. Factorize the following:
(x – 5)(x + 4) = 0 1. x3 + 5x2 + 2x – 8 = 0.
2. 4x3 – 4x2 – 11x + 6 = 0.
⇒(x – 3) (x – 5)(x + 4) = 0
3. 6x3 + 13x2 – 4 = 0.
x = 3 or x = 5 or x = - 4 4. 3x3 + 7x2 + x – 6 = 0.

Exercises 5.14 C. Solve each of the following given that at


A. 1. Find the three linear factors of x3 – x2 – least one of the solutions is an integer
14x + 24. Hence, solve the equation x3 – x2 – between -5 and 5.
14x + 24 = 0 1. x3 – 13x + 12 = 0
2. x3 + 2x2 – 5x – 6 = 0
2. Factorize x3 – x2 – 2x + 1. Hence solve the 3. 2x3 – 9x2 + 7x + 6 = 0
equation 2x3 + 1 = x2 + 2x. 4. 6x3 + 13x2 – 4 = 0
5. 2x3 – 3x2 – 50x – 24 = 0
3. Show that x = is a root of the equation 6. x3 – 7x2 + 2x + 40 = 0
2x3 – 5x2 – 4x + 3 = 0 and find the other two
Challenge problems
roots.
1. Verify that – 4p is the root of the equation
4. Show that x = 2 is the root of the equation x3 x3 + 3px2 – 6p2x – 8p3 = 0. Hence or otherwise,
+ 4x2 – 11x – 2 = 0 and find the other two roots, find the three roots of x3 + 3px2 – 6p2x – 8p3 = 0
in terms of p. Ans;- 4p, -p, 2p
giving your answer in the form a b√ .
2. Factorize the following;
5. If x = - is a root of the equation 2x3 – 9x2 + a. a3 – b3 b. a3 + b3
kx + 6 = 0 , find the value of k . Find the other
two roots of the equation. 3. Solve each of the following given that at
least one of the solutions is an integer between
6. Let p(x) = ax3 – 5x2 - bx + 18 = 0. -5 and 5.
If -2 and 3 are the roots of the equation p(x) = a. x4 – 4x3 + 3x2 + 4x – 4 = 0.
0, find the values of a and b. b. x4 + x3 – 7x2 – x + 6 = 0
If p(k) = 0, k ≠ -2, -3, find the value of k.
Factorizing Cubic Expressions of the form:
7. If k is the root of the equation 3x3 – (k + 3)x2 x3 + cx + d
+ (7 – k – 4k2)x - 4 = 0, find the values of k. Given x3 + cx + d, where d is the constant of the
polynomial.
8. Find the values of the constants p , q and r 1. Assuming a is a factor of d, such that x - a or
for which (x – 4)(x – 2)(x + p) = x3 – 7x2 + qx + x = a is a factor of x3 + cx + d, then
r for all values of x R. Using these values of x3 + cx + d, factorizes as :
q and r, solve the equation x3 – 7x2 + qx + r = 0 = (linear factor) (quadratic factor)

Baffour Ba Series , Further Mathematics for Schools Page 152


= (x – a) (quadratic expression) 2. Factorize x3– 2x + 4
= (x – a) 0 ( ) . /1
Solution
x3– 2x + 4
2. Assuming - a is a factor of d, such that x + a
When x = -2
or x = - a is a factor of x3 + cx + d, then
x3– 2x + 4 = 0
x3 – cx + d, factorizes as:
= (linear factor) (quadratic factor) (x + 2) . /
= (x + a) (quadratic expression) (x + 2) ( )
= (x + a) 0 ( ) . /1
4. Factorize x4 + 27x
Worked Examples
Solution
1. Factorize x3 + 9x – 10
x4 + 27x = x(x3 + 27)
Solution (x3 + 27)
x3 + 9x – 10
When x = 1 When x = -3
x3 + 9x – 10 = 0 (x3 + 27) = 0

(x – 1) . ( ) / (x + 3) . /

(x – 1) ( ) (x + 3) ( )
4
x + 27x = x(x + 3) ( )

Baffour Ba Series , Further Mathematics for Schools Page 153


6 RATIONAL FUNCTIONS Baffour – Ba Series

Definition x=
( )
A function f is rational if f(x) = ( )
, where g(x) Domain = {x is real except x = }
and h(x) are polynomials. In other words, a
rational function is a ratio of two polynomials
2. f(x) =
in which the denominator is not zero.
Let 2x + 1 = 0
Rational Functions and their Domain 2x = - 1
The domain of f(x) consists of all real numbers x=
except the zeros of the denominator h(x). In
other words, the domain is the set of all real 3. f(x) =
numbers that can be used in place of the
Let 9x + 1 = 0
variable.
9x = -1

Steps; x=
1. Equate the denominator, h(x) to zero and Domain = {x is real except x ≠ }
solve for the value of x.
2. The value of x (or the involving variable)
4. f(x) =
obtained is not part of the domain if it makes
the denominator zero. Let x2 – 4 = 0
x2 – 22 = (x + 2) (x – 2) = 0
In a rational function, the denominator cannot ⇒x = 2 or x = -2
be equal to zero because that would be Domain = {x is real except x = 2 or x = -2)
undefined. Find which numbers that make the
fraction undefined by creating an equation The Range
The range of a rational function is the same as
whose denominator is not equal to zero.
the domain of the inverse function. Hence to
find the range:
Worked Examples
1. Find the inverse of the function.
Determine the domain of the following:
2. Find the domain of the inverse function.
1. f(x) = 3. f(x) =
2. f(x) = 4. f(x) = Worked Examples
Determine the range of the following:
Solution 1. f(x) = 2. f(x) =
1.
Let 3x – 8 = 0 Solutions
3x = 8 f(x) =

Baffour Ba Series , Further Mathematics for Schools Page 154


f(x) = y 3. f(x) = 6. f(x) =
y=
y (3x – 8) = 4x + 5 (change subject to x) Evaluating Rational Functions
3xy – 8y = 4x + 5 A rational function involving a variable has no
3xy – 4x = 8y + 5 value unless a value is assigned to the variable.
x(3y – 4) = 8y + 5 Once the variable is given a value, the rational
x= function can be evaluated.

Worked Examples
Now, let 3y – 4 = 0
3y = 4 1. Find the value of f(x) = for x = -3

y=
Solution
Range = { y : y R, y ≠ }
f(x) =
When x = -3
4. f(x) = ( )
f(x) = ( )
= = 13
Let y = (Solve for x)

y=( )( 2. If R(x) = , find R(4).


)

y= (Change subject to y)

y (x – 2 ) = 1 Solution
x y – 2y = 1 R(x) =
x y = 2y + 1 ( )
R(4) = ( )
=2
y=
Range = {y : y R, y ≠ 0} Exercises 6.2
A. Evaluate each of the following.
Exercises 6.1
1. f(x) = for x = -2
A. Find the domain of the following rational
functions. 2. f(x) = for x = 5
1. f(x) = ( )(
4. f(x) = +5
)
3. R(x) = , find R(3)
2. f(x) = 5. f(x) =
4. f(x) = , find R(-1)
3. f(x) = 6. f(x) = √
B. Find the range.
B. 1. If R(x) = , find R(3), R(5), R(2.05)
1. f(x) = 4. f(x) = +5
and R(1.999)
2. f(x) = 5. f(x) =

Baffour Ba Series , Further Mathematics for Schools Page 155


2. Given that f(x) = , find f(2), f(- 4), 2. Simplify f(mn) =
f(-3.02), f(-2.96)
Solution
Reducing Rational Functions ( )( )
f(mn) = = ( )
The steps involved in reducing rational
functions are;
1. Factor the numerator and denominator But = -1
completely. f(x) = (m + n) (-1)
2. Divide the numerator and denominator by f(x) = - m – n
the greatest common factor.
Exercises 6.3
Worked Examples Reduce each of the following.
1. Simplify f(x) = 1. f(x) = 4. f(x) =

Solution 2. f(x) = 5. f(x) =


( )( )
f(x) = = = = 3. f(x) = 6. f(xy) =
( ) ( )

2. Simplify f(w) = Identity


If two rational functions are equivalent, then
Solution they have the same numerical value for any
( ) replacement of the variable, conditioned not to
f(w) = =( =
)( ) give a zero denominator. For example, given
that f(x) = and g(x) = . Since f(x) =
Involving Rational Functions of Two
g(x), it implies that = . This is
Variables
Worked Examples satisfied by all real numbers except -3. The
1. Simplify f(xy) = equation is therefore an identity.

Multiplication of Rational Functions


Solution Rational functions are multiplied the same way
( )
f(xy) = = as rational numbers and rational expressions.
( )
With rational functions, we can factor, reduce
But =-1 and then multiply.
Worked Examples
f(xy) = (-1)
1. If f(x) = and g(x) = , find the
f(xy) = product of f(x) and g(x).

Baffour Ba Series , Further Mathematics for Schools Page 156


5. f(x) = and g(x) =
Solution
f(x) = and g(x) = ,
Division of Rational Functions
f(x) . g(x) = . , Division of rational functions is similar to that
of rational numbers. Invert the divisor and
( )( )
= . , multiply, not forgetting to factor and reduce
( )
where possible.
( )( )
= .( )(
,
)
( )
Worked Examples
= . ,
Given that f(x) = and g(x) =
= ( )
Find f(x) ÷ g(x).

= Solution
f(x) = and g(x) =
2. Given that f(xy) = and g(xy) = ,
find f(xy) . g(xy) f(x) ÷ g(x) = ÷

= ×
Solution
f(xy) = and g(xy) = , = ×
( )
f(xy) . g(xy) = . , ( )( ) ( )( )
= ×
( ) ( )
= .( )(
,
) ( )( ) ( – )
= ×
= . ,
( )( )
= =
( )
=
Exercises 6.4
In each of the following, find the product of =
f(x) and g(x)
1. f(x) = x2 – 6x + 9 and g(x) =
Exercise 6.5
2 ( )
2. f(x) = and g(x) = 4x + 20x + 25 In each of the following, find ( )
( )
3. f(x) = and g(x) = 1. f(x) = and g(x) =

4. f(x) = and g(x) = 2. f(x) = and g(x) =

Baffour Ba Series , Further Mathematics for Schools Page 157


3. f(x) = and g(x) = x2 – 9 =

4. f(x) = 2x2 – 3x – 5 and g(x) =


Type 2 (Different Denominators)
5. f(x) = 6x2 – x – 2 and g(x) = Add or subtract numerators whilst the lowest
common denominator is maintained. Simplify
Addition and Subtraction of Rational where possible.
Function
Type 1 (Identical Denominators) Worked Examples
Add or subtract numerators whilst a common 1. Given that f(x) = and g(x) = , find
denominator is maintained. Simplify where f(x) + g(x)
possible.
Solution
Worked Examples f(x) = and g(x) = ,
1. Given that f(x) = and g(x) = , find :
f(x) + g(x) = + ,
f(x) + g(x)
= + ,
( )
Solution
f(x) = and g(x) = , = ( )(
+ ,
) ( )
( )
f(x) + g(x) = + = = =2 =
( )
( )( )

2. If f(x) = ( and g(x) = ( , = ( )( )


)( ) )( )
find f(x) – g(x) = ( )( )

Solution
2. Given that f(x) = and g(x) = , find:
f(x) = ( and g(x) = (
)( ) )( )
f(x) – g(x)

f(x) – g(x) = ( –( Solution


)( ) )( )
( ) f(x) = and g(x) = ,
= ( )( )
f(x) – g(x) = – ,
= ( )
( )( )
= – ,
( )
=( )( ) = – ,
( )( )
= ( )( ) =

Baffour Ba Series , Further Mathematics for Schools Page 158


= 1. =
2. = +
( )
Exercise 6.6
3. ( )(
= + +
)
A. Given that f(x) = ,
4. ( )(
= + + + ,
)( )
g(x) = ,
5. ( )(
= + +
)( )
h(x) = .
where the numerators at the R.H.S. are
Simplify the following, leaving your answer in constants to be found.
factors;
1. f(x) + g(x) 4. g(x) – f(x) Worked Examples
2. f(x) + h(x) 5. h(x) – f(x) Find the partial fraction decomposition of the
3. g(x) + h(x) 6. h(x) – g(x) following:

B. Find the domain and zeros of the functions 1. ( 2.


)( )
defined in questions 1 to 6 of A 3. 4.
( )( )( )

Partial Fractions
Consider the identity below: Solutions
= + 1. ( = +
)( )

By adding the fractions and , we obtain


Multiply through by the least common
. The expression on the R.H.S. of the denominator, (x – 2) (x + 3).
equation is called the partial fraction . (x – 2) (x + 3) = . (x – 2) (x + 3)
( )( )
decomposition of .
+ . (x – 2) (x + 3)
( ) ⇒8x – 1 = A(x + 3) + B(x – 2)
For partial fraction decomposition of to be
( )
found, it is essential that f(x) have lower degree When x = -3
than g(x). If this is not the case, use long 8(-3) – 1 = A(-3 + 3) + B(-3 – 2)
division to obtain such expression. For -24 – 1 = 0 + (-5B)
example, given , we obtain -25 = -5 B
B=5
=x–6+
Type I: A Partial fraction decomposition in When x = 2
which each denominator is linear 8(2) – 1 = A(2 + 3) + B(2 – 2)
A rational expression with a linear denominator 16 – 1 = 5A + 0
is decomposed on the R.H.S. as shown below. 15 = 5A
Baffour Ba Series , Further Mathematics for Schools Page 159
A=3 B = -2

Now Put A = 3 and B = 5 in When x = 3,


= + 11(3) + 12 = C[2(3) + 3](3 + 2)
( )( )
33 + 12 = C(6 + 3) (5)
⇒( = +
)( ) 45 = C(9) (5)
45 = 45C
2. C=1

⇒ = +( (Multiply through by lcd) When x = ,


( ) )
2
3x – 16 = A(x – 4) + Bx
11. / + 12 = A. /. /
When x = 4, =A. /. /
3(4)2 – 16 = A(4 – 4) + 4B
=A. /
48 – 16 = 4B
32 = 4B =
B=8 18A = 36
A=2
Now, when x = 0 and B = 8 Substitute A = 2, B = -2 and C = 1
3(0)2 – 16 = A(0 – 4) + 8(0)
in = + +
-16 = - 4A ( )( )( )

A=4 = + +
( )( )( )

Substitute A = 4 and B = 8 in
= +( The partial fraction decomposition is;
( ) )

⇒ = +( = – –
( )( )( )
( ) )

3. = + + 4.
( )( )( )
( )( )( )
11x + 12 = +
Factorize the denominator to obtain a quadratic
( )( )( ) ( )( )( ) expression;
+
x3 – 2x2 – 3x = x (x2 – 2x – 3)
11x + 12 = A(x + 2)(x – 3) + B(2x + 3)(x – 3) +
C(2x + 3)(x + 2) By factorization,
When x = -2, x2 – 2x – 3 = (x + 1) (x – 3)
11(-2) + 12 = B[2(-2) + 3](-2 – 3) x3 – 2x2 – 3x = x (x + 1) (x – 3)
- 22 + 12 = B(- 4 + 3)(- 5) = + +
( )
-10 = 5B

Baffour Ba Series , Further Mathematics for Schools Page 160


( )( ) ( )( ) Solution
4x2 + 13x – 9 = + +
( )(
= +
( )( )
)
Multiplying throughout by (2x + 3)(x − 1),
7x + 8 = A(x − 1) + B(2x + 3).
4x2 + 13x – 9 = A(x + 1)(x – 3) + Bx(x – 3) +
Cx(x + 1) Substituting x = 1
7 + 8 = B(2 + 3).
15 = 5B
When x = 3, B = 3.
4(3)2 + 13(3) – 9 = 0 + 0 + C[3(3 + 1)]
36 + 39 – 9 = 12C Substituting x =
66 = 12C
7. / + 8 = A . /
C=
= A
A=1
When x = -1,
4(-1)2 + 13(-1) – 9 = 0 + B[-1(-1 – 3)] + 0 = +
( )( )
4 – 13 – 9 = 4B
- 18 = 4B
.Exercises 6.7
B=
A. Express in partial fractions:
1. ( 2. (
When x = 0, )( ) )( )

4(0)2 + 13(0) – 9 = A (0 + 1) (0 – 3) + 0 + 0 3. 4. ( )( )
- 9 = A (1) (-3)
-9 = - 3 A
A=3 B. Resolve into partial fractions:
1. 4.
Substitute A = 3, B = and C = in
2. 5.
= + + ( )( )
( )
3. ( 6.
)( )( ) ( )( )
⁄ ⁄
⇒ = + +
( )
The partial fraction decomposition is; Challenge Problems
Resolve into partial fractions :
= – +
( ) ( ) ( )
1. 3. ( )( )( )

5. Express the rational function ( in


)( ) 2. 4. ( )( )( )
partial fractions.

Baffour Ba Series , Further Mathematics for Schools Page 161


Type II: A Partial fraction decomposition
containing a repeated linear factor Alternatively;
A rational expression whose denominator From 2x + 3 = A(x – 1) + B
contains a repeated linear factor is decomposed 2x + 3 = Ax – A + B
on the R.H.S. as shown below. when x = 0, and B = 5
1. = + +( 2(0) + 3 = A(0) – A + 5
( ) )
3=-A+5
2. = + +( +( , A= 5 – 3
( ) ) )
A=2
3. = +( +( , The partial fraction is: ( = +(
( ) ) )
) )
where the numerators at the R.H.S. are
constants to be found. 2. = + +(
( ) )
Multiply through by lcd;
Worked Examples
Find the partial fraction decomposition of the x(x – 3)2 = x(x – 3)2 + x(x – 3)2
( )
following:
+( x(x – 3)2
)
1. ( 2.
) ( ) ⇒x2 + 10x – 36 = A(x – 3)2 + Bx(x – 3) + Cx
x2 + 10x – 36 = A(x2 – 6x+ 9) + B(x2 – 3x) + Cx
Solution x2 + 10x – 36 = Ax2 – 6Ax+9A + Bx2 – 3Bx + Cx
1. = +( x2 + 10x – 36 = Ax2 + Bx2 – 6Ax –3Bx + Cx+ 9A
( ) )
By regrouping;
x2 + 10x – 36 = (A + B)x2 + (–6A –3B + C)x+ 9A
(x – 1)2 = (x – 1)2 + (x – 1)2
( ) ( )
2x + 3 = A(x – 1) + B Equating coefficients, the following is obtained;
A + B = 1……………………(1)
When x = 1 - 6A – 3B + C = 10………….(2)
2(1) + 3 = A(1 – 1) + B 9A = - 36…………………….(3)
5=0+B
B=5 From equation 3,
9A = -36
Now, when x = 0, and B = 5 A=-4

Substitute in 2x + 3 = A(x – 1) + B Put A = - 4 in eqn (1);


2(0) + 3 = A(0 – 1) + 5 -4+B=1
3=-A+5 B=1+4
A=5–3 B=5
A=2 Put A = - 4, and B = 5 in eqn (2);
- 6(- 4) – 3(5) + C = 10

Baffour Ba Series , Further Mathematics for Schools Page 162


24 – 15 + C = 10
9 + C = 10 By expansion;
C = 10 – 9 = 1 3[x3 + 3x2 + 3x + 1] + B(x – 2) (x2 + 2x + 1) +
C(x2 + x – 2x – 2 ) + (x – 2)
The partial fraction is:
3[x3 + 3x2 + 3x + 1] + B (x3 + 2x2 + x –2x2 – 4x
= + +(
( ) ) – 2) + C(x2 + x – 2x – 2 ) + (x – 2)

3. Express ( in partial fractions 3[x3 + 3x2 + 3x + 1] + B (x3 – 3x – 2) + C(x2 – x


)( ) – 2) + (x – 2)
3x3 + 9x2 + 9x + 3 + Bx3 – 3Bx – 2B + Cx2 –
Solution Cx – 2C + x – 2
( )(
= + +( +(
) ) )
3x3 + Bx3 + 9x2 + Cx2 + 9x + x – 3Bx – Cx – 2B
– 2C + 3 – 2
( )(
= + +( +(
) ) )
3x + x + 1 = A(x + 1) + B (x – 2) (x + 1)2 + C
3 3
3x3 + Bx3 + 9x2 + Cx2 + 10x – 3Bx – Cx – 2B –
(x – 2) (x + 1) + D (x – 2) 2C + 1

When x = -1; (3 + B)x3 + (9 + C)x2 + (10 – 3B – C)x – 2(B +


3 (-1)3 + (-1) + 1 = A(-1 + 1)3 + B (-1 – 2) (-1 + C) + 1
1)2 + C (-1 – 2) (-1 + 1) + D (-1– 2)
-3 – 1 + 1 = -3 D Comparing L. H. S and R. H. S;
-3 = -3D That is comparing 3x3 + x + 1 to (3 + B)x3 + (9
D=1 + C)x2 + (10 – 3B – C)x – 2(B + C) + 1

When x = 2; Equating coefficients;


3(2)3 + (2) + 1 = A(2 + 1)3 + B (2 – 2) (2 + 1)2 3 + B = 3…………..…(1)
+ C (2 – 2) (2 + 1) + D (2 – 2) 9 + C = 0…………….(2)
24 + 2 + 1 = A (3)3 10 – 3B – C = 1……..(3)
27 = 9A
A=3 From eqn (1);
B = 0;
Considering the R.H.S.
A(x + 1)3 + B (x – 2) (x + 1)2 + C (x – 2) (x + 1) From eqn (2);
+ D (x – 2) C = -9

Substitute A = 3 and D = 1 When B = 0 and C = -9, eqn (3) is satisfied.


3(x + 1)3 + B (x – 2) (x + 1)2 + C (x – 2) (x + 1) = + +( +(
( )( ) ) )
+ 1 (x – 2)

Baffour Ba Series , Further Mathematics for Schools Page 163


= – +( Exercises 6.8
( )( ) ( ) )
Resolve into partial fractions.
4. Express the rational function ( ) (
in
) 1. ( 4.
) ( ) ( )
partial fractions.
2. 5.
Solution

( ) (
= +( + 3. ( 6.
) )
) ( )
2
9 = A(x + 1) (x – 2) + B(x – 2) + C(x + 1)
B. Express in partial fractions;
When x = -1
9 = A(-1 + 1) (-1 – 2) + B(-1 – 2) + C(-1 + 1)2 1. ( 2.
) ( )( )
9 = -3B 3. ( 4. (
)( ) )( )
B = -3

When x = -2 Challenge Problem


9 = A(2 + 1) (2 – 2) + B(2 – 2) + C(2 + 1)2 1. Find the values of A, B, C, D, if
9 = C (3)2 = + +( +(
( )( ) ) )
9 = 9C
C=1
Type III: A Partial fraction decomposition
containing an irreducible quadratic factor
When B = -3 and C = 1;
A rational expression whose denominator
9 = A(x + 1) (x – 2) + -3(x – 2) + 1(x + 1)2
contains an irreducible quadratic factor is
9 = A(x2 – 2x + x – 2) – 3x + 6 + (x2 + 2x + 2)
decomposed on the R.H.S. as shown below;
9 = A (x2 – x – 2) – 3x + 6 + x2 + 2x + 2
9 = Ax2 – Ax – 2A – 3x + 6 + x2 + 2x + 2 1. = +
( )
9 = Ax2 + x2 – Ax – 3x + 2x – 2A + 6 + 2
2. ( =( +
9 = Ax2 + x2 – Ax – x – 2A + 8 )( ) )
9 = x2 (A + 1) – x (A + 1) – 2A + 8
Worked Examples
9 = -2A + 8 1. Find the partial fraction decomposition of
2A = 9 – 8
2A = 1
A=
Solution
The partial fraction decomposition is:

= +( +
( ) ( ) ) x3+ 2x = x(x2 + 2) (Factorize the denominator)
( ) (
= –( + = + ( Decompose the expression)
) ( ) )

Baffour Ba Series , Further Mathematics for Schools Page 164


Multiply through by lcd, x(x2 + 2) =2+
⇒9x2 – 3x + 8 = A(x2 + 2) + (Bx + C)x
= x2(2x – 1) + 4(2x – 1)
= Ax2 + 2A + Bx2 + Cx
= (x2 + 4) (2x – 1)
= Ax2 + Bx2 + Cx + 2A
= (A + B)x2 + Cx + 2A
By decomposition,

Now, equating coefficients; ⇒ = +


9x2 – 3x + 8 = (A + B)x2 + Cx + 2A
A + B = 9 ……………(1) Multiply both sides by (x2 + 4) (2x – 1)
C = -3 ………………..(2) x2 – x – 21 = (Ax + B) (2x – 1) + C(x2 + 4)
2A = 8………………..(3) = 2x(Ax + B) – 1 (Ax + B) + C(x2+ 4C)
= 2Ax2 + 2Bx – Ax - B + Cx2 + 4C
From eqn (3), = 2Ax2 + Cx2 – Ax + 2Bx – B + 4C
A=4
x2 – x – 21= (2A + C) x2+ (-A + 2B)x – B + 4C
Put A = 4 into eqn (1);
4+B=9 Now, equating coefficients in
B=9–4 x2 – x – 21 = (2A + C)x2 + (-A + 2B)x – B + 4C
B=5 2A + C = 1 ……………(1)
-A + 2B = -1……………(2)
Substitute A = 4, B = 5 and C = -3 into - B + 4C = -21………….(3)
= +
From eqn (1);
⇒ = + C = 1 – 2A…………….(4)

Put eqn (4) into eqn (3);


2. Resolve into partial fractions. -B + 4(1 – 2A) = -21
- B + 4 – 8A = - 21……(5)
Solution
In ; From eqn (2);
- A = - 1 – 2B
Degree of numerator is equal to degree of
A = 1 + 2B…………..…(6)
denominator.

Put eqn (6) into eqn (5);


Therefore, long division is required.
- B + 4 – 8(1 + 2B) = -21
2 - B + 4 – 8 – 16B = -21
3 2
2x – x + 15x – 29 4x3 – x2 + 15x - 29 -17B – 4 = - 21
4x3 – 2x2 + 16x - 8 -17B = -17
x2 – x – 21 B=1

Baffour Ba Series , Further Mathematics for Schools Page 165


Put B = 1 in eqn (2); Equating coefficients;
- A + 2B = -1 3x2 + 9 = A(x2 + 2x + 7) + (Bx + C)(x − 5).
- A + 2(1) = -1
-A+2=-1 3x2 + 9 = Ax2 + 2Ax + 7A + Bx2 + Cx – 5Bx – 5C
- A = -1 – 2 3x2 + 9 = Ax2 + Bx2 + 2Ax – 5Bx + Cx + 7A – 5C
-A=-3 3x2 + 9 = (A + B)x2 + (2A – 5B + C)x + 7A – 5C
A=3 A + B = 3…………………….(1)
2A – 5B + C = 0……………….(2)
Put A = 3 into eqn (1); 7A – 5C = 9……………………(3)
2(3) + C = 1
6+C=1 From eqn (1);
C=1–6 A+B=3
C = -5 2 + B = 3 (But A = 2)
B=3–2
Substitute A = 3, B = 1 and C = - 5 into B=1

⇒ = + From equation (2);


Substitute A = 2 and B = 1
= = +
2(2) – 5(1) + C = 0
4–5+C=0
The partial fraction of the expression is; -1 + C = 0
C=1
=2+ -

Substitute A = 2, B = 1 and C = 1
3. Express the rational function ( )( )
( )(
= +
)
in partial fractions.
Exercises 6.9
Solution
A. Resolve into partial fractions:
= +
( )( ) 1. 5. ( )( )
( )

Multiplying throughout by: 2. ( 6. .


)( ) ( )
(x − 5)(x2 + 2x + 7),
3x2 + 9 = A(x2 + 2x + 7) + (Bx + C)(x − 5). 3. 7. ( )( )

Substitute x = 5 4. ( )( )
8. ( )( )
3 × 52 + 9 = A(52 + 2 × 5 + 7).
84 = 42A B. Express in partial fractions;
A = 2.
1. 2. ( )( )
3.( )( )
( )

Baffour Ba Series , Further Mathematics for Schools Page 166


4. ( 5. 6. ( = Ax3 + Bx2 + (A + C)x + B + D
)( ) ( )( ) )( )
Comparing coefficients,
5x3 – 3x2 + 7x – 3 = Ax3 + Bx2 + (A + C)x + B + D
Type IV: A Partial fraction decomposition A=5
containing a repeated quadratic factor B = -3
A rational expression whose denominator A + C = 7 ……………(1)
contains a repeated quadratic factor is B + D = -3…………..(2)
decomposed on the R.H.S. as shown below;
1. = +( Put A = 5 in eqn (1);
( ) )
5+C=7
Worked Examples C=7–5
Find the partial fraction decomposition of C=2

( ) Put B = -3 in eqn (2)


-3 + D = -3
Solution D = -3 + 3
The degree of the numerator is 3 and the degree D=0
Substitute A = 5, B = -3, C = 2 and D = 0 in
of the denominator is 4. Long division is not = +(
( ) )
required.
= +( = +(
) ( ) )
( )

Multiply both sides by lcd, (x2 + 1)2 Exercises 6.10


⇒ 5x3 – 3x2 + 7x – 3 Resolve into partial fractions.
= (Ax + B) (x2 + 1) + Cx + D 1. 2.
( ) ( )
= Ax(x2 + 1) + B(x2 + 1) + Cx + D
= Ax3 + Ax + Bx2 + B + Cx + D

Baffour Ba Series , Further Mathematics for Schools Page 167


7 BINOMIAL THEOREM Baffour Ba Series

Binomial Expansion
The Pascal‟s Triangle
Consider the following binomial expansions:

Binomial Expansion Coefficients


1. (x + y)0 1 1
2. (x + y)1 x+y 11
3. (x + y)2 x + 2xy + y2
2
121
4. (x + y)3 x3 + 3x2y + 3xy2 + y2 1331
5. (x + y)4 x4 + 4x3y + 6x2y2 + 4xy3 + y4 14641
6. (x + y)5 x5 + 5x4y + 10x3y2 + 10x2y3 + 5xy4 + y5 1 5 10 10 5 1

From the table, the coefficients of expansion form a pattern. This triangular array of coefficients for
the binomial expansion is called Pascal‟s triangle.

1
1 1
1 2 1
1 3 3 1
1 4 6 4 1
1 5 10 10 5 1
1 6 15 20 15 6 1
…………………………….

Notice that each line starts and ends with 1 and that each entry of a line is the sum of the two entries
above it in the previous line. For instance, 4 = 3 + 1, 10 = 6 + 4 and 20 = 10 + 10. Following this
pattern, the eighth rows of coefficients are :
1 6 15 20 15 6 1
2
x – 6x + 5
1 7 21 35 35 21 7
2
3x + 5x – 9

Pascal‟s triangle gives an easy way of obtaining the coefficients for the binomial expansion with
small powers, but it is impractical for larger powers. For large powers, use the formula involving
factorial notation or combinatorial notation.

Baffour Ba Series , Further Mathematics for Schools Page 168


Worked Examples
1. Use Pascal‟s triangle to expand (a + b)5

Solution
Number of terms = n +1 = 5 + 1 = 6 terms
Therefore, there will be 6 terms involving
a5, a4b1, a3b2, a2b3, a1b4, b5 each with degree 5.
From Pascal‟s triangle, the coefficients are respectively: 1 5 10 10 5 1
Therefore, the expansion of(a + b)5in descending powers of a is
(a + b)5= a5 + 5a4b1 + 10 a3b2 + 10a2b3 + 5a1b4 + b5

2. Expand (2x + 3y)3 in descending powers of x.

Solution
(2x + 3y)3 = (a + b)3
⇒a = 2x and b = 3y and the number of involving terms is 3 + 1 = 4 terms
(2x)3, (2x)2 (3y), (2x) (3y)2 (3y)3
From the Pascal‟s triangle, the coefficients obtained for row four are: 1 3 3 1
Therefore, the expansion of (2x + 3y)3 in descending powers of x is
(2x + 3y)3 = (2x)3 + 3(2x)2 (3y) + 3(2x) (3y)2 + (3y)3
= 8x3 + 36x2y + 54xy2 + 27y3

3. Obtain the expansion of . / , in descending powers of x

Solution
Let . / = ( )
⇒ a = 2x and b = -

= (2x)4,(2x)3. /, (2x)2. / , (2x)1. / ,. /

From Pascal‟s triangle, coefficients at row 5 are respectively: 1 4 6 4 1


. / = (2x)4+ 4(2x)3. / + 6(2x)2. / + 4(2x)1. / +. /
= 16x4– 16x3 + 6x2 – x +

4. Use Pascal‟s triangle to expand( ) . Simplify all terms.

Baffour Ba Series , Further Mathematics for Schools Page 169


Solution
( ) is expanded as:
(2x) , (2x)4 (-y)1, (2x)3 (-y2), (2x)2 (-y)3, (2x)(-y )4, (-y)5
5

Respective coefficients obtained from Pascal‟s triangle are: 1 5 10 10 5 1


( ) = (2x)5 + 5(2x)4 (-y)1 + 10(2x)3 (-y2) + 10(2x)2 (-y)3 + 5(2x)(-y)4 + (-y)5
= 32x5 – 80x4y + 80x3y2 – 40x2y3 + 10xy4 – y5

5. Expand . √ /

Solution
In . √ / number of terms is 5 + 1 = 6

From Pascal‟s triangle, the coefficients of the 6th row are : 1 5 10 10 5 1


. √ /

= . / + 5. / (-2√ )1 + 10 . / (-2√ )2 + 10. / (-2√ )3 + 5. / (-2√ )4 + (-2√ )5



= – ⁄ + – ⁄ + 80x – 32

Exercises 7.1
A. Use Pascal‟s triangle to expand:
1. (p + q)6 2. (x – y)7 3. (x + 2a)3
4. (a + 3b)4 5. (x2 – 2)4 6. (x2 + a2)5

B. Use Pascal‟s triangle to expand:


1. . / 2. . / 3. . / 4. . /

5. . / 6. . / 7. (√ √ ) 8. ( √ )

Factorials
Definition Consider 8! = 8.7.6.5.4.3.2.1 = 40,320, it can be
The product of all positive whole numbers from seen that:
n down to 1 is called “factorial n” and it is 8! = 8.7.6.5.4.3.2.1 = 8.7! = 8.7.6! = 8.7.6.5!
denoted by n!. (and so on)
Generally,
Thus n! = n(n – 1)(n – 2) …3.2.1 1. if 5! = 5.4! = 5.4.3!( and so on), then
By definition, 0! = 1 n! = (n(n – 1)! = n(n – 1) (n – 2)! and so on
2. If 7.6! = 7!, then (n +1) n! = (n + 1)!
Baffour Ba Series , Further Mathematics for Schools Page 170
Note: ( )
2. If = n2 + pn + q, find the value of p and
)
When simplifying factorials, it is a good
the value of q.
practice to start with the larger factorial and
work down to the smaller one.
3. If (n + 1)! + n2(n – 1)! = (an + b)n!, find the
value of a.
Worked Examples
1. If ( = k, find the value of k
) 4. If ( = k, find the value of k.
)

Solution ( )
5. If = an, find the value of a.
=k ( ) ( )
( )
( ) ( )
= = =8 C. Solve the following:
( ) ( ) ( )
k=8 ( )
1. 9(n – 4)! = (n – 3)! 4. =
( )

( ) ( ) ( )( )
2. Solve = 2. ( = 30 3. =
) ( )

Solution
( ) Combinatorial Numbers
=
( ) 1. By definition, ( ) =
= ( )
( )( )
( )( ) ( )
= 2. In practice,( ) =
( )
⇒n(n – 1 ) = 56
n2 – n – 56 = 0 Both give the same results; but the second is
easier to use in practical questions.
Solving by factorization
(n -8)(n + 7) = 0 For example:
n = 8 or n = -7 1. ( ) = = = = 15
( )
n = -7 is rejected as n! is defined for natural
,
number only. 2. ( ) = ⇒ = 15
,

Exercise 7.2
Memory Aid:
A. Evaluate the following:
( ) ( )= =
( ) ( )
1. 10! 2. 3. 4.
( )is pronounced as “n-c-r” or “n-choose-r”
( )
B. 1. If = n2 + an + b, find the value of a
( ) Other Results
and the value of b. 1. ( ) = 1 and ( ) = 1

Baffour Ba Series , Further Mathematics for Schools Page 171


2. ( ) = ( ) (n + 7) (n – 6) = 0 Solving by factorization,
n = - 7 or n = 6
Reject n = -7
Use results 2 if r is greater than . Thus n = 6
For example, ( ) =( ) =( )
( )
2. 4( )= ,n N
Worked Examples ( )( ) ( )( )( )
⇒ =
Calculate the following:
( )( )
1. ( ) 2. ( ) 3. ( ) 2(n – 1) =
12n – 12 = n2 + 3n + 2
Solution n2 – 9n + 14 = 0
1. ( ) = = 120 10 nCr 3 = 120 (n – 7) (n – 2) = 0 Solving by factorization,
n = 7 or n = 2
2. ( ) = 1
3. ( ) = ( )=( )= = 435 Exercises 7.3
A. Evaluate the expressions:
The following occur quite frequently when 1. 2 6 2. 3 4 3. 7 0 4. 5 0
solving equations involving combinatorial 5. 6. 7. 8. 9.
( )

numbers:
1. ( ) = n ( )
( ) B. 1. If ( = n2 + an + b, find the values of a
2. ( ) = )

( )( )
and b.
3. ( ) =
( )
2. If = n2 + pn + q, find the values of p
( )
Worked Examples
and q.
Solve:
1. ( ) = 21, n N
3. If (n + 1) + n2(n – 1 ) = (an + 1)n , find the
( )
2. 4( )= ,n N value of a
4.If ( = k, find the value of k.
)
Solution
1. ( ) = 21, n N ( )
( ) 5. If = an, find the values of a.
( ) ( )
= = 21
( )
= 21 C. Solve each of the following;
2 ( )
n + n = 2 × 21 1. 9(n – 4 ) = (n – 3) 2. ( = 30
)
n2 + n = 42 ( )( ) ( )
n2 + n – 42 = 0 3. = 4. =
( ) ( )

Baffour Ba Series , Further Mathematics for Schools Page 172


Binomial Theorem
An expression with two terms e.g. a + b, is called a binomial.
The Binomial theorem is used to write down the expansion of a binomial to any power,
e.g. (a + b)n

The Binomial Expansion of (a + b)n


The expansion of (a + b)n is found as follows;

Method I : Combinatorial Method


(a + b)n = ( )anb0 + ( ) an - 1 b1 + ( )an - 2 b2 +…+ ( )a1 bn-1 + ( ) a0 bn

Method II : Factorial method


( ) n–2 2 ( )( ) ( ) n–k k
(a + b)n = an + nan-1b + a b + …+ a b + …+nabn – 1 + bn

Notes
1. The expansion contains (n + 1) terms (one more than the power).
2. The power of a decreases by 1 in each successive terms.
3. The power of b increases by 1 in each successive term.
4. In each term, the sum of the indices of a and b is n.
5. The power of b is always the same as the lower number in the combination bracket
6. If the binomial is a difference, (a – b) the signs will be alternately +, - , +, -,…

Worked Examples
1. Write out all the terms in the expansion of (a + b)5

Solution
Method 1 (using Combinatorial method)
The power is 5, so there are six terms.
Step 1: Write down six pairs of the variable as shown below:
ab + ab + ab + ab + ab + ab

Step 2: Put in powers, starting with the highest power of a: sum of powers = 5 in each term)
a5b0 + a4b1 + a3b2 + a2b3 + a1b4 + a0b5
Step 3 : Put in combinatorial numbers
( )a5b0 + ( )a4b1 + ( )a3b2 + ( )a2b3 + ( )a1b4 + ( )a0b5

Step 4: a5 + 5a4b1 + 10a3b2 + 10a2b3 + 5 a1b4 + b5

Baffour Ba Series, Further Mathematics for Schools Page 173


In practice, the first three steps can be combined one step
Note: (any real number)0 = 1, thus a0 = 1, b0 = 1 etc

Method 2: (using Factorial method)


( ) n–2 2 ( )( ) ( ) n–k k
(a + b)n = an + nan- 1b + a b + …+ a b + …+ nabn – 1+ bn
( ) 5–2 2 ( )( ) 5-3 3 ( )( )( ) 5–4 4
(a + b)5 = a5+ 5a5-1b + a b + a b + a b +
( )( )( )( ) 5–5 5
a b
( ) 3 2 ( )( ) 2 3 ( )( )( ) ( )( )( )( ) 5
= a5+ 5a4b + ab + ab + ab4 + b
= a5 + 5a4b + a3b2 + a2b3 + ab4 + b5
= a5 + 5a4b + 10a3b2 + 10a2b3 + 5ab4 + b5

2. Expand fully the expression (2 − x)5.

Solution
First note that:
(A + B)5 = A5 + 5A4B + A3B2 + A2B3 + AB4 + B5.
That is,
(A + B)5 = A5 + 5A4B + 10A3B2 + 10A2B3 + 5AB4 + B5.

Now let A = 2 and B = - x to obtain


(2 − x)5 = 25 + 5(2)4(-x) + 10(2)3(- x)2 + 10(2)2(- x)3 + 5(2)(- x)4 + (- x)5.

That is,
(2 − x)5 = 32 − 80x + 80x2 − 40x3 + 10x4 − x5.

3. Expand fully the expression (1 + 2x)3.

Solution
(A + B)3 = A3 + 3A2B + AB2 + B3
= A3 + 3A2B + 3AB2 + B3.

Now let A = 1 and B = 2x,

(1 + 2x)3 = 1 + 3(2x) + 3(2x)2 + (2x)3


= 1 + 6x + 12x2 + 8x3.

4. Find the binomial expansion of (2x + 3y2)4

Baffour Ba Series, Further Mathematics for Schools Page 174


Solution
Let (2x + 3y2)4 = (a + b)n
⇒ a = 2, b = 3y2 and n = 4
(2x + 3y2)4 = ( )(2x)4(3y2)0 + ( ) (2x)3(3y2) + ( ) (2x)2(3y2)2 + ( )(2x)(3y2)3 +( )(2x)0(3y2)4
= 16x4 + 4(8x3)(3y2) + 6(4x2)(9y4) + 4(2x)(27y6) + 81y8
= 16x4 + 96x3y2 + 216x2y4 + 216xy6 + 81y8

5. Expand, using the binomial theorem, (x – 3y)4.

Solution
(x – 3y)4, the number of terms = 4 + 1 = 5 terms
(x – 3y)4= ( ) (x)4(-3y)0 +( ) (x)3(-3y)1 + ( )(x)2(-3y)2 + ( ) (x)1(-3y)3 + ( ) (x)0(-3y)4
= (1)(x4) (1) + (4)(x3)(-3y) + 6(x2) (9y2) + (4) (x) (-27y3) + (1)(1)(81y4)
= x4 – 12x3y + 54x2y2 –108xy3 + 81y4

6. Expand using binomial theorem . / . Show that one of the terms is independent of p.

Solution
In . / the power is 4 and the terms are 5.

. / = ( ) (p)4. / +( ) (p)3. / + ( )(p)2. / + ( ) (p)1. / + ( ) (p)0. /

= (1)(p4) (1) + (4)(p3) . / + 6(p2) . / + (4) (p) . /+ (1)(1). /

= p4 + 8p2 + 24 + +
The third term, 24 is independent of p (meaning the term does not contain p or the power of p is 0)

Exercises 7.4
A. Expand the following:
1. ( ) 2. ( ) 3. ( )
4. ( ) 5. ( ) 6. ( )

B. Expand the following:


1. . / 2. . / 3. . /

Baffour Ba Series, Further Mathematics for Schools Page 175


The Binomial Expansion of (1 + a)n
The binomial expansion of (1+ a)n is as follows:
(1 + a)n = ( ) + ( )a + ( ) a2 +…+ ( ) an - 1 + ( ) an

This form of binomial theorem is used to expand a binomial to any power when the first term of the
binomial is 1.

Type 1
Worked Examples
1. Expand, using the binomial theorem, (1 + 2x)5. Hence, expand (1 – 2x)5.

Solution
(1 + 2x)5 =( ) + ( ) (2x) + ( )(2x)2 + ( )(2x)3 + ( )(2x)4 + ( )(2x)5
= (1) + (5)(2x) + (10)(4x2) + (10)(8x3) + (5)(16x4) + (1)(32x5)
= 1 + 10x + 40x2 + 80x3 + 80x4 + 32x5

The only difference between the expansion of (1 + 2x)5 and (1 – 2x)5 is that the signs alternate :
+, –, + , – …
⇒(1 – 2x)5= 1 – 10x + 40x2 – 80x3 + 80x4 – 32x5

2. Expand and simplify (1 + x)3 + (1 – x)3

Solution
Method 1
(1 + x)3 + (1 – x)3
(1 + x)3 = ( ) + ( ) (x) + ( )(x)2 + ( )(x)3
=1 + 3x + 3x2 + x3
⇒(1 – x)3 = 1 – 3x + 3x2 – x3

(1 + x)3 + (1 – x)3 = [1 + 3x + 3x2 + x3] + [1 – 3x + 3x2 – x3]


= 6x2 + 2

Exercises 7.5
A. Expand:

1. (1 + 2x)4 2. (1 –3x)4 3. (1 – 2x)6 4. (1 – 5x)4( ) 5. . / 6. (√ *

Baffour Ba Series, Further Mathematics for Schools Page 176


2. Expand fully the following expressions:
(a) (2x − 1)5; (b) . / (c) . /

Evaluating Expansions of Sums and Difference


Sometimes, we may be asked to evaluate expansions such as (a +√ )n and write the answer in the
form c + d√

Learners can use the Pascal‟s triangle or the binomial theorem, unless stated.

Worked Examples
1. Expand (3 + √ )5, and write your answer in the form a + b√

Solution
Method 1:(Using Binomial Theorem)
In (3 + √ )5, there are 6 terms;
(3 +√ )5
=( )(3)5(√ )0 + ( )(3)4(√ )1 + ( )(3)3(√ )2 + ( )(3)2(√ )3 + ( )(3)1(√ )4 + ( )(3)0(√ )5

= (1)(243)(1) + (5)(81)(√ ) + (10)(27)(2) + (10)(9)(2√ ) + (5)(3)(4) + (1)(1)(4√ )


= 243 + 405√ + 540 + 180√ + 60 + 4√
= 843 + 589√

Method 2: (Using Pascal‟s Triangle)


Using Pascal‟s triangle, the respective coefficients are: 1, 5 , 10 , 10, 5, 1
(3 + √ )5=1 (3)5(√ )0 + 5(3)4(√ )1 + 10(3)3(√ )2 + 10(3)2(√ )3 + 5(3)1(√ )4 + 1(3)0(√ )5
= (1)(243)(1) + (5)(81)(√ ) + (10)(27)(2) + (10)(9)(2√ ) + (5)(3)(4) + (1)(1)(4√ )
= 243 + 405√ + 540 + 180√ + 60 + 4√
= 843 + 589√

2. Given that (1 + √ )4 + (1 – √ )4 = k, find the value of k.

Solution
(1 + √ )4 + (1 – √ )4 = k
Using Pascal’s triangle,
(1 + √ )4 = 1 + (4)(√ )1 + (6)(√ )2 + (4)(√ )3 + (1)(√ )4

Baffour Ba Series, Further Mathematics for Schools Page 177


= 1 + 4√ + (6)(3) + 12√ + 9
= 1 + 18 + 9 + 12√ + 4√
= 28 + 16√

(1 – √ )4 = 1 – 4√ + (6 )(3) – 12√ +9
= 1 + 18 + 9 – 16√
= 28 – 16√

⇒(1 + √ )4 + (1 – √ )4 = 28 + 16√ + 28 – 16√


= 28 + 28
= 56
So k = 56
Try the second method (Binomial theorem)

3. i. Use Pascal‟s triangle to simplify (1 + x)5 + (1 – x)5


ii. Use your results to simplify (1 + √ )5 + (1 – √ )5

Solution
i. (1 + x)5 + (1 – x)5
(1 + x)5 = 15 + 5x + 10x2 + 10x3 + 5x4 + x5
= 1 + 5x + 10x2 + 10x3 + 5x4 + x5

(1 – x)5 = 1 – 5x + 10x2 – 10x3 + 5x4 – x5 (signs alternate)

(1 + x)5 + (1 – x)5 = [1 + 5x + 10x2 + 10x3 + 5x4 + x5] + [1 – 5x + 10x2 – 10x3 + 5x4 – x5]
= 1 + 1 + 5x – 5x + 10x2 + 10x2 + 10x3 – 10x3 + 5x4 + 5x4 + x5 – x5
= 2 + 20x2 + 10x4

ii. (1 + x)5 + (1 – x)5 = (1 + √ )5 + (1 – √ )5


⇒ (1 + x)5 = (1 + √ )5
Therefore, x = √
(1 + x)5 = 1 + 5x + 10x2 + 10x3 + 5x4 + x5
(1 + √ )5 = 1 + 5(√ ) + 10(√ )2 + 10(√ )3 + 5(√ )4 + (√ )5
= 1 + 5(√ ) + 10(3) + 10(3)(√ ) + 5(9) + 9 (√ )
= 1 + 5(√ ) + 30 + 30√ + 45 + 9 (√ )
= 76 + 44√

Baffour Ba Series, Further Mathematics for Schools Page 178


(1 – √ )5 = 76 – 44√ (signs alternate)

⇒(1 + √ )5 + (1 – √ )5 = 76 + 44√ ) + ( 76 - 44√ )


= 76 + 76
= 152

Exercises 7.6
A. Evaluate the following leaving your answer in the form a + b√ ;
1. (1 + √ )4 2. (1 – √ )5 3. (1 + √ )6 4. (2 + √ )4
5. (1 + √ )4 6. (1 -2√ )6 7. (2 + √ )6 8. (√ – √ )5

B. 1. Given that(3 + √ )5 + (3 - √ )5 = k, find the value of k, using any suitable method

2. . i. Show that (1 + x)6 + (1 - x)6 = 2(1 + 15x2 + 15x4 + x6 )


ii. Hence evaluate (1 + √ )6 + (1 –√ )6.

3. i. Use Pascal‟s triangle to expand (a + b)5


ii. Hence, expand (1 + 2x)5 – (1– 2x)5
iii. Hence, write (1 + 2√ )5 – (1 – 2√ )5 in the form k√ , k N

Challenge Problems
1. i. Use Pascal‟s triangle or binomial theorem to expand (p +q)4 + (p – q)4
ii. Hence, or otherwise, write (x +√ )4 – (x – √ )4 as a polynomial in x
iii. Hence, evaluate (4 + √ )4 + (4 – √ )4

Substitution
Identify the number and its substitute. Perform the binomial expansions and substitute the values at
the final stage .

Worked Examples
Given that √ = 1.414, √ = 1.732, √ , evaluate without using tables;
3 4
(1 –√ ) (√ + √ ) correct to three decimal places.

Solution
(1 – √ )3 (√ + √ )4
(1 – √ )3 = 1 + 3(1)(-√ ) + 3(1) (-√ )2 + (-√ )3
= 1 – 3√ + 15 – 5√

Baffour Ba Series, Further Mathematics for Schools Page 179


= 16 – 8√
= 8 (2 – √ )
= 8 (2 – 2.236)
= 8 (- 0.236)
= -1.888……………………..(1)

(√ + √ )4 = (√ )4 + 4(√ )3(√ ) + 6(√ )2 (√ )2+ 4(√ ) (√ )3 + (√ )4


= 4 +8(√ ) (√ ) + 36 + 12(√ ) (√ ) + 9
= 49 + 20(√ ) (√ )
= 49 + 20(1.414) ( )
= 97.9810…………………..(2)

(1 –√ )3 (√ + √ )4 = (1) × (2)
= -1.888 × 97.9810
= -184.188 (3 decimal places)

Exercises 7.7
Given that √ = 1.414, √ = 1.732, √ , evaluate the following correct to three
decimal places:
1. (1 +√ )3 2. (1 – √ )3 3. (√ + √ )4 4. (√ – √ )4
5. (1 –√ )4– (1 + √ )3 6. (√ + 1)4 + (√ + √ )3 7. (√ + 1)4 (√ – √ )3

Related Application
This involves application of the binomial theorem to evaluate an exponential value. This is done as
follows:
I. Express the given value as a sum of two numbers, (usually a natural number plus a decimal
fraction, preceded by zero(s) with the same exponent). Consider the following instances:
a. As far as binomial theorem is concern, (2.01)3 is expressed as:

(2.01)3 = (2 + 0.01)3

Natural number + decimal fraction (preceded by zero(s)

Baffour Ba Series, Further Mathematics for Schools Page 180


b. As far as binomial theorem is concern, (1.997)3 (1+ 0.997)3. This is because the decimal
fraction is not preceded by zero(s). Therefore, if 2 – 1.997 = 0.003, then 1.997 = 2 – 0.003
⇒(1.997)3 (2 – 0.003)3

Incorrect
(1.997)3 (1 + 0.997)3

Natural number + decimal fraction (not preceded by zero(s)

Correct
(1.997)3 (2 – 0.003)3

Natural number + decimal fraction (preceded by zero(s)

c. As far as binomial expansion is concerned (0.9)4 (0 + 0.9)4. This is because the first part of the
summation is not a natural number. Therefore, if 1 – 0.9 = 0.1, then 0.9 = 1 – 0.1
⇒ (0.9)4 = (1 – 0.1)4

Incorrect
(0.9)4 (0 + 0.9)4

Not a natural number + decimal fraction preceded by zero(s)

Correct
(0.9)4 = (1 – 0.1)4 x

Natural number + decimal fraction (preceded by zero(s)3

II. Use any suitable method to expand the sum of the binomial.

Baffour Ba Series, Further Mathematics for Schools Page 181


Type 1
It involves evaluating a given exponential value to a given number of decimal places.

Worked Examples
1. Evaluate (2.01)5 to two decimal places:

Solution
Method 1:(Using Pascal‟s triangle)
(2.01)5 = (2 + 0.01)5
From the Pascal‟s triangle, the respective coefficients are: 1, 5 , 10, 10 , 5, 1
(2 + 0.01)5
(1)(2)5(0.01)0 + (5)(2)4(0.01)1 + (10) (2)3(0.01)2 + (10)(2)2(0.01)3 + (5)(2)1(0.01)4 + (1)(2)5 (0.01)0
32 + 0.08 + 0.008
32.808
32.81 (2 decimal places)

Method 2: (Combination method)


(2.01)5 = (2 + 0.01)5
(2 + 0.01)5
= ( )(2)5 (0.01)0 + ( )(2)4(0.01)1 + ( )(2)3(0.01)2+ ( )(2)2(0.01)3+ ( ))(2)1(0.01)4+ ( )(2)0(0.01)5
(1)(2)5 + (5)(16)(0.01)1 + (10) (8)(0.01)2 + (10)(4)(0.01)3 + (5)(2)1(0.01)4 + (1)(1) (0.01)5
32 + 0.08 + 0.008
32.808
32.81(2 decimal places)

2. Use Pascal‟s triangle to obtain the value of (1.002)5, correct to six places of decimals.

Solution
(1.002)5 = (1 + 0.002)5
From Pascal‟s triangle, the respective coefficients are: 1, 5, 10, 10, 5, 1
(1 + 0.002)5 (1)(0.002)0 + (5) (0.002)1 + (10) (0.002)2 + …
1+ 0.01 + 0.000040
1.010040
⇒ (1.002)5 = 1.010040 (6 decimal places)

Baffour Ba Series, Further Mathematics for Schools Page 182


3. Use Pascal‟s triangle to obtain the value of (1.997)4, correct to two places of decimals.

Solution
(1.997)4 = (2 – 0.003)4
From Pascal‟s triangle, the respective coefficients are: 1, 4, 6, 4, 1
(2 – 0.003)4
(1)(2)4(-0.003)0 + (4)(2)3(-0.003)1 + (6)(2)2(-0.003)2 + (4)(2)1(-0.003)3 + (1)(2)0(-0.003)4
(1)(16)(-0.003)0 + (4)(8)(-0.003)1 + (6)(4)(-0.003)2 + (4)(2)1(-0.003)3 + (1)(1)(-0.003)4
16 + 32(- 0.003) + 24(-0.003)2…
16 – 0.096 +… (Ignore the other terms since they involve more than 3 decimal places)
15.904
(1.997)4 = 15.90 (2 decimal places)

4. Obtain the value of (0.9)4 correct to three decimal places.

Solution
(0.9)4 = (1 – 0.1)4
From the Pascal‟s triangle, the respective coefficients are 1, 4, 6, 4, 1
(0.9)4 =14 (-0.1)0 + (4) (-0.1)1 + (6) (-0.1)2 + (4)(-0.1)3 + (-0.1)4
1 – 0.4 + 0.06 – 0.004 +….
0.656 (3 decimal places)

Exercises 7.8
Use any suitable method to evaluate the following correct to two decimal places.
1. (1 – 0.1)5 2. (2 – 0.001)4 3. (1 + 0.01)5 4. (1.03)4
5. (2.001)3 6. (1.999)6 7. (2.998) 8. (1.01)10 9. (0.998)6

Type 2
It involves expanding a given binomial and applying the results to evaluate a given exponential
value. This is usually refered to as appproxinations.

Worked Examples
1. Expand (2 + x)5 and hence, find the value of (1.9)5

Solution
(2 + x)5
Coefficient of expansion are 1, 5, 10, 10 5, 1
(2 + x)5 = 25 + (5) 24x + (10)23x2 + (10)22x3 + (5)2x4 + x5
(2 + x)5 = 32 + 80x + 80x2 + 40x3 + 10x4 + x5

Baffour Ba Series, Further Mathematics for Schools Page 183


(1.9)5 = (2 + (- 0.1))5 = (2 + x)5
⇒ x = - 0.1
(1.9)5 = (2 + (- 0.1))5
= 32 + 80 (- 0.1) + 80(- 0.1)2 + 40 (- 0.1)3 + 10 (- 0.1)4 + (- 0.1)5
= 32 – 0.8 – 0.04 + 0.001 – 0.00001
= 24.76099

2. a. Find the first four terms of the binomial expansion of (1 – 2x)6 in ascending powers of x.
b. Using the expansion in a. calculate to four decimal place the value of (0.98)6

Solution
(1 – 2x)6 = (1 + (– 2x))6

Coefficients of expansion are: 1 6 15 20 15 6 1


(1 + (– 2x))6 = 1 + (6)(-2x) + 15 ( ) + 20 ( )
2 3
= 1 – 12x + 60x v– 160x

(0.98)6 = (1 + ( - 0.02 ))6


Let (1 + ( - 0.02 ))6 = (1 + (– 2x))6
- 0.02 = - 2x
x = 0.01

Put x = 0.01 in 1 – 12x + 60x2 – 160x3


= 1 – 12(0.01) + 60(0.01)2 – 160(0.01)3
= 1 – 0.012 + 0.006 – 0.00016
= 0.9938 four decimal places

3. Expand (x + 2y)5 by the binomial theorem and apply your results to evaluate (1.02)5, correct to
four decimal places.

Solution
(x + 2y)5
= ( )(x)5(2y)0 + ( )(x)4(2y)1 + ( )(x)3(2y)2 + ( )(x)2(2y)3 + ( )(x)1(2y)4 + ( )(x)0(2y)5
= (1)(x)5 + (5)x4(2y)1+ (10) x34y2+ (10)x28y3+ (5)x116y4+ (1)32y5
= x5 + 10x4y+ 40x3y2+ 80x2y3+ 30xy4+ 32y5
Now, (1.02)5 = (1 + 0.02)5 = (x + 2y)5
⇒x = 1 and 2y = 0.02,
x = 1 and y = = 0.01 substitute in x5 + 10x4y + 40x3y2+ 80x2y3 + 30xy4 + 32y5

Baffour Ba Series, Further Mathematics for Schools Page 184


Hence,
(1.02)5 = 15 + 10 (1)4 (0.01) + 40(1)3 (0.01)2 + 80(1)2(0.01)3+ 30(1) (0.01)4 + 32(0.01)5
1 + 0.1 + 0.004 + 0.00008…
1.10408
1.1041(4 decimal places)

4. Write down the expansion of (1 + x)4. Taking the first three terms of the expansion, put x = 0.1,
and find the value of (1.025)4 correct to three places of decimals.

Solution
(1 + x)4
Using Pascal‟s triangle, the respective coefficients are 1, 4, 6, 4, 1
(1 + x)4 =14 ( x)0 + (4) ( x)1 + (6) ( x)2 + (4)( x)3 + ( x)4
= 1 + x + x2 + x3 + x4

When x = 0.1, substitute in 1 + x + x2 (the first three term of the expansion)


(1 + (0.1))4 = (1 + 0.025)4
= 1 + (0.1) + (0.1)2
= 1 + 0.1 + 0.00375
= 1.10375
(1 + 0.025)4 1.104 (three decimals places)

Exercises 7.9
1. Write down the expansion of (2 + x)5 in ascending powers of x. Taking the first three terms of the
expansion, put x = 0.001, and find the value of (2.001)5

2. Expand (2 – x)6 in ascending powers of x. Taking x = 0.002, and using the first three terms of the
expansion, find the value of (1.025)4, correct to three places of decimals.

3. i. Expand (1 + x)5 using pascals triangle.


ii. Use the results in (i) to evaluate (1.01)5 without using calculator Ans 1.0510 to 4 places of decimals.

Type 3
Worked Examples
1. By expressing (1 + x – 2x2)3 as a power of a binomial, obtain its full expansion.

Baffour Ba Series, Further Mathematics for Schools Page 185


Solution
(1 + x – 2x2)3
= [1 + (x – 2x2)]3
= 1 + 3(x – 2x2) + 3(x – 2x2)2 + (x – 2x2)3
= 1 + 3x – 6x2 + 3(x2 – 4x3 + 4x4) +x3 (1 – 2x)3
= 1 + 3x – 6x2 + 3x2 – 12x3 + 12x4 + x3[1 + 3(-2x) + 3(-2x)2 + (-2x2)3]
= 1 + 3x – 3x2 – 12x3 + 12x4 + x3 – 6x4 + 12x5 – 8x6
= 1 + 3x – 3x2 – 11x3 + 6x4 + 12x5 – 8x6

2. Obtain the expansion of (1 + x – 2x2)8, as far as the term in x3

Solution
(1 + x – 2x2)8 = [1 + (x – 2x2)]8
= 1 + 8 (x – 2x2) + (x – 2x2)2 + (x – 2x2)3 + …
= 1 + 8 (x – 2x2) + 28 (x2 – 4x3 + 4x4) + 56 (x3 + other terms)
= 1+ 8x – 16x2 + 28x2 – 112x3 + 56x3 + terms in x4
= 1 + 8x + 12x2 – 56x3 (as far as the term in x3)

Exercises 7.10
A. Using binomial theorem, expand the following:
1. (1 + x + 2x2)4 2. (1 – x – x2)3 3. (2 – x + x3)3 4. (1 + x – x3)4
5. . / 6. (3 – x + 2x2)5 7. (1 + 2x + x2)4 8. (1 – x + x2)5

Unknown Coefficients or an Unknown Index Worked Examples


In some cases the need arises to find unknown 1. The first three terms in the binomial
coefficients or an unknown index (power). expansion of (1 + ax)n are 1 + 2x + x2. Find the
When the index is unknown, make use of the
value of n and the value of a.
following;
( ) =1 ( ) =n Solution
( ) ( )( )
( )= ( )= The first three terms are;
Then equate the coefficients and solve the (1 + ax)n = ( ) + ( ) (ax) + ( ) (ax)2
resultant equations. ( ) 2 2
= 1 + n(ax) + ax
( ) 2 2
Likewise, the Pascal‟s triangle can also be used. = 1 + nax + ax
( ) 2 2
⇒1 + nax + a x = 1 + 2x + x2

Baffour Ba Series, Further Mathematics for Schools Page 186


Equate coefficients and solve the resultant = 14C4 (2x)10(-3)4
equations;
= 210 × 34
na = 2……………..(1)
= 1001 × 1024 × 81
( ) 2
a = ………(2) = 83,026,944

From eqn (1); Exercises 7.11


a = . /, put into eqn (2); 1. The first four terms of the expansion of
( ) (1 – x)n are 1 – 6x + ax2 + bx3. Find the value
. / = of n, the value of a and the value of b.
( )
=
2. The first three terms of the expansion of
( )
= (1 + ax)n are 1 + 20x + 150x2. Find the value of
n and the value of a.
6(n – 1) = 5n
6n – 6 = 5n 3. The first three terms of the expansion of
n=6 (1 + ax)n are 1 + 28x + 336x2. Find the value of
From eqn (1), a = = = n and the value of a.

Therefore, n = 6 and a = 4. The first three terms of the expansion of


(1 + kx)n are 1 – 18x + 135x2. Find the value of
2. Find the coefficient of x2 in the expansion of n and the value of k.
(3x + 5)6
5. The first four terms of the expansion of
Solution (1 + kx)n are 1 + 20x + 180x2 + ax3. Find the
To get the coefficient of x2 in (3x + 5)6 the value of n, the value of k and the value of a.
required term is (3x)2(5)4which has the
binomial coefficient 6C2 6. The first three terms of the expansion of
= 6C2 (3x)2(5)4 (1 + ax)n are 1 – 5x + x2. Find the value of n
2 4
= 3 ×5 and the value ofa.
= 15 × 9 × 625
= 84,375 7. The first three terms of the expansion of
(1 + ax)n are 1 + 8x + 30x2. Find the value of n
3. Find the coefficient of x10 in the expansion of and the value of a.
(2x – 3)14
8.The first four terms of the expansion of
Solution (1 – )n are 1 – 24x + ax2 + bx3. Find the value
Required term is (2x)10 (– 3)4
of n the value of a and the value of b.
By binomial theorem, the coefficient is;

Baffour Ba Series, Further Mathematics for Schools Page 187


9. The first three terms of the expansion of Note:
n
(1 + kx) are 1 + 2x + 2
x . Find the value of n The number at the bottom of the combination
bracket, r, is always one less than the term
and the value of k.
number, n + 1.
10.The first four terms of the expansion of Method 2
(1 + kx)6 are 1 + ax + 135x2 + bx3. Find the Similarly, for k ranging from 1 to n +1, the kth
value of n the value of a and the value of b. term of the expansion of (a + b)n is given by the
formula:
11.The first three terms of the expansion of
( )( )
an – k + 1 bk - 1
(a – )6, a > 0, in ascending powers of x, are 64
+ 16bx + bcx2, a, b, c R. Find the value of a, Worked Examples
1. Find the sixth term of the expansion
b and c.
(a2 – 2b)7
12. In the expansion of (1+ x)n + 1 the coefficient
of x4 is 6k and in the expansion of (1+ x)n - 1 , Solution
the coefficient of x2 is k. Find the value of n, Method 1
n > 2. General term: Ur + 1 = ( )(a2)n – r( )
There are 8 terms in the expansion. Thus,
Selecting a Particular Term U6 = Ur + 1⇒ r = 5
In some problems, only particular term is
U6 = ( )(a2)7 – 5( )
required. For example, the middle term, the
fifth term or the term independent of x (no x U6 = ( )(a2)2 ( )
term or power of x is zero).
U6 = (21) (-32) (a4) b5
Method 1 U6 = - 672a4b5
In these cases, we use the “general term”.
The general term in the binomial expansion of Method 2
(a + b)n is: Ur + 1 = ( ) k = 6 and n = 7
Substitute in:

( )( )
an – k + 1 bk - 1
(Sum of powers = n – r + r = m × n
= ( ) ( )6 – 1
( )( )
Ur + 1 = ( )
=( ( ) ( )5
)( )

(Same) = 21a4(-32b5)
+ = - 672a4b5

Baffour Ba Series, Further Mathematics for Schools Page 188


2. i. Find, and simplify the middle term in the Solution
binomial expansion of . / i. . /
General term:
Solution
Ur + 1 = ( ) (2x)9 – r. /
. / 9–r
= ( ) (2 )(x ) (-1)r
-2r

General term: Ur + 1 = ( )(6x)10 – r. / = (-1)r( )29 – rx9 – 3r


There are 11 terms in the expansion. Thus, the
ii. For the term independent of x, in the
middle term is U6
U6 = Ur + 1⇒ r = 5 binomial . / , the power of x = 0
U6 = ( )(6x)10 – 5. / Therefore, 9 – 3r = 0,
r = 3 (4th term)
U6 = ( ) (6x)5. /
Thus, the required term is;
U6 = (252) (7776x5) . /
= (-1)3( )29 –3x9 – 3(3)
U6 = 8064x5y5
= (-1)(84)(26) (x0)
= -5371
3. Find, and simplify the third term in the
expansion of . / 5. i. Write down the general term in the
binomial expansion of (2x + 5y2)n
ii. if k is a constant and kx3y4 is a term in the
Solution
binomial expansion of (2x + 5y2)n find the
. / values of n and k

General term : Ur + 1 = ( ) 6x8 – r. /


Solution
U3 = Ur + 1⇒r = 2 i. (2x + 5y2)n
U3 = ( ) (2x)8 – 2. / The general term
Ur + 1 = ( )( ) ( )
U3 = ( ) (6x)6. /
=( ) 5ry2r
U3 = (28) (6x6) . / =( ) 5r y2r
U3 = 1792x4
ii. If kx3y4 is a particular term then,
4. Find: 2r = 4 and n – r = 3
i. the general term, r = 2 and n – 2 = 3
ii. the term independent of x , in the binomial r = 2 and n = 5
expansion of . / k = ( ) 52
k = (10) (8) (25) = 2000

Baffour Ba Series, Further Mathematics for Schools Page 189


Exercises 7.12
1. . / 2. . /
A. Find the:
1. 4th term of (1 + 2x)6 3. . / 4. . /
2. 5th term of (a + 2b)9
3. 3rd term of (x – 2y)12 5. . / 6. . /
4. 7th term of (1 + )8
7. / 8. . /

5. 4th term of . – /
E. Find the indicated term of the binomial
6. 5th term of . /
expansion:
7. 6th term of . – / 1. (x + 2y)8, 4th term
2. (3a + b)7, 4th term
8. 4th term of . / 3. (2a – 2b)20, 7th term
4. (a2 + w2)9, 5th term
B. 1. Write down the first three terms in the
expansion of (1 + 3x)7, find their sum when Challenge Problems
x= 1. If k is a constant and kxy6 is a term in the
expansion of (2x+ 3y2)n, find the values of n
and k.
2. When . / is expanded in ascending
powers of x, find: 2. If h is a constant and hx4y6 is a term in the
i. the coefficient of x4, when y = 1 expansion of (3x – 4y2)n, find the values of n
ii. the sum of the first three terms when and h.
x = 0.1, y = 0.02
3. i. Write down the binomial expansion of
C. Find the middle term : (1 + 2x)n in ascending powers of x as far as the
1. (1 + 2x)10 2. (a + 2b)8 term containing x3.
3. (p – 2q)10 4. (a – 3b)6 ii. Given that the coefficient of x3 is twice the
coefficient of x2 and that both are positive, find
5. . / 6. . /
the value of n.
7. . / 8.. /
4. Write down the first three terms of the
binomial expansion, in ascending powers of x,
D. Find the term independent of x in the
of (1+ ax)n where a ≠ 0 and n N.
binomial expansion of each:

Baffour Ba Series, Further Mathematics for Schools Page 190


Sums of Binomial Coefficients
The binomial expansion (1 + x)n = ( ) + ( )x + ( ) x2 +…+ ( ) xn - 1 + ( ) xn
Can be used to find the sum of binomial coefficients by letting x = 1 or x = - 1 on both sides.

Other binomial series can be used.

Worked Examples
1. Evaluate the sum of the coefficients in the expansion of (x + 3)8

Solution
(x + 3)8= ( )x8+ ( )x7(3) + ( )x6(3)2 +…+ ( )(3)8
Let x = 1 on both sides
(x + 3)8= ( ) + ( )(3) + ( )(3)2 +…+ ( )(3)8
= 48
Thus, the sum of the coefficients is 48 = 65,536

2. Evaluate ( ) + ( ) + ( ) +…+ ( ) + ( )

Solution
( ) + ( ) + ( )+…+ ( ) + ( ) = 212 = 4,096

Exercises 7.13
A. Evaluate the sum of the coefficients in the expansions of the following:
1. (x + 2)7 2. (x + y)12 3. (3x + 1)12

B. Use the binomial expansion (1 + x)n to verify :


1. ( ) + ( ) + ( ) +…+ ( ) + ( ) = 28

2. ( ) + ( ) + ( ) +…+ ( ) + ( ) = 216

3. ( ) + ( ) + ( ) +…+ ( ) = ( ) + ( ) +…+ ( )

Negative and Rational Exponents

The exponent of a binomial may be negative or rational. For example, ( ) ,√ ,

( ) etc. In this case, the expansion is said to be infinite because it does not terminate. The
factorial method is applicable in this type of expansion. That is;

Baffour Ba Series, Further Mathematics for Schools Page 191


( ) n–2 2 ( )( ) ( ) n–k k
(a + b)n = an + nan - 1b + a b + …+ a b + …+ nabn – 1 + bn

It is always advisable to express the binomial in the form ( ) , where n is a rational number.
That is: ( ) = an . / , where a > 1. However, expansion is still possible without this
form.

Worked Examples
Type 1 (Negative exponents)
1. Use binomial theorem to expand ( ) in ascending powers of x, up to and including the
term in x3.

Solution
( ) ( )( )
(1 + x)n = 1 + nx + x2 + x3 + …
( ) ( )( )
( ) = 1 + (-3)x + x2 + x3 + …
( ) = 1 – 3x + 6x2 – 10x3 + …

2. Expand ( ) up to the term in x3

Solution
( ) =, ( )-
. /. / . /. /. /
= 1 + (- 6x) + (- 6x)2 + (- 6x)3
= 1 – 3x – x2 – x3

3. Expand ( ) up to the term in x3

Solution
( )
Now,
4 + 3x = 4 . /

( ) = 0 . /1

= . /

= . /

Baffour Ba Series, Further Mathematics for Schools Page 192


. /. / . /. /. /
= 8 . /. / . / . / 9

= 2 3
= – x +…

Conditions for Convergence of Binomial Series


When n is a positive integer, or a zero, the binomial series for (1 + x)n converges for all values of x,
since the expansion is finite. This is not the case however, when n is not a positive number. The
series expansion of (1 + x)n is convergent for -1 < x < 1 in general.

Worked Examples
State the values of x for which the following series expansion converge.
1. ( ) 2. ( ) 3.

Solution
1. ( ) = . / , which converges for -1 < < 1. Multiplying through by 2, the
series converges for -2 < x < 2

2. ( ) converges for -1 < 4x < 1. Dividing through by 4, the series converges for
<x<

3. =( ) = . / , converges for -1 < < 1.


Dividing through by , the series converges for <x<

4. Expand (2 − x)−3 as far as the term in x3 stating the values of x for which the series is valid.

Solution
We first convert the expression (2 − x)−3 to one in which the leading term in the bracket is 1. That
is,
(2 − x)−3 = 0 . /1 = 0 . /1

The required binomial expansion is thus:


( )( ) ( )( )( )
[ ( ). / . / . / ]

= 0 1
The expansion is valid for −1 < < 1. That is −2 < x < 2.

Baffour Ba Series, Further Mathematics for Schools Page 193


Type 2: Involving fractions
1. Change the given fraction to a negative exponent
2. Get the common factor out by factorization to obtain the form (a + b)n, where n is a negative
integer.
3. Expand by binomial theorem

Worked Examples
1. Obtain the first four terms of the expansion of ( )
in ascending powers of x.

Solution
Method 1

( )
=( ) = . /

= . /

( )( ) ( )( )( ) ( )( )( )( )
= [ . / . / . / ]

( ) ( )( ) ( )( )( )
= [ . / . / . / ]

= 0 1

= + x– x2 + x3

( )
= + x– x2 + x3

Method 2

( )
=( )
( )( ) ( )( )( )
= 2-3 + (-3)(2)-3 – 1(x) + (2-3 – 2)(-x)2 + (2-3 – 3)(-x)3
( )( ) ( )( )( )
= + (-3)(2)-4(x) + (2)-5 (-x)2 + (2)6 (-x)3

= + x– x2 + x3

( )
= + x– x2 + x3

Baffour Ba Series, Further Mathematics for Schools Page 194


2. Expand the expression ( )
as far as the term in x3

Solution

( )
=( )

( ) = . /
( )( ) ( )( )) )
( . /* = [ ( ). / . / . / ]

= 0 1
= + + +

3. Given that -1 < x < 1 , find the expansion of ( )(


in ascending powers of x, up to and
)
including the term in x3.

Solution

( )(
= (3 – 2x) ( ) ( )
)

= (3 – 2x) ( ) 0 . /1

= (3 – 2x) ( ) . /

= (3 – 2x) ( ) . /
= (3 – 2x) ( ). /
= (3 – 2x) . /. /
= – x+ x2 – x3…

Type 3

1. Expand √ in ascending powers of x, up to and including the term in x3

Solution

√ =( )( )
. /. / . /. /. /
( ) = 1 + (-x) + (-x)2 + (-x)3 + …
= 1 – x – x2 – x3 + …

Baffour Ba Series, Further Mathematics for Schools Page 195


. /. / . /. /. /
( ) =1+. / (-2x)2 + (-x)2 + (- x)3 + …
= 1 – x – x2 – x3 + …

Type 4
Express the number as a rational or negative exponent and write it as a product of 10 . Simplify
where possible, and expand the binomial using the factorial method.

1. Evaluate √ correct to five places of decimals.

Solution
Using 1.02 = 1 + 0.02,
√ =( )
. / . /. /
( ) = 1 + (0.02) + (0.02)2 + (0.02)3 + …..
= 1 + 0.01 − (0.0004) + (0.000008) - …..
= 1 + 0.01 − 0.00005 + 0.0000005 − . . .
1.010001 − 0.000050
= 1.009951
Hence √ = 1.00995

2. Calculate to 3 significant figures.

Solution
=( )
= 1 + 2(0.005) + 3(0.005)2 + 4 (0.005)3 + …
= 1 + 0.01 + 0.000075 + …
1.010075
= 1.01 (3 s.f)

3. Use binomial theorem to evaluate √ 8 correct to four decimal places

Solution
√ 8 = ( 8) = ( 8 )
= ( 8) ×( )
= 0( 8) 1 ( )

= ( ) ( )

Baffour Ba Series, Further Mathematics for Schools Page 196


= ( ) ( )

. /. / . /. /. /
But ( ) = 1 + . /( ) + ( ) + ( )
=1+ +. / +
= 1.1215

( ) ( ) = ( ) (1.1215)
= (1.25992) (2.15443) (1.1215)
= 3.0442

4. Use binomial theorem to evaluate √ correct to four significant figures

Solution
√ =( )

=. /

= . /
. /. / . /. /. /
= 24 . /. / . / . / 5

=2+ – +…
= 2. 0125 – 0.000004… + …
= 2.012 (4 s.f)

Type 5
Worked Examples
1. i. Find the first four terms of the expansion ( ) in ascending powers of x.
ii . use the expansion, to determine the value of √8 to five decimal places.

Solution
. /. / . /. /. /
( ) = 1 + . / (3x) + ( ) + ( )
( ) 2
=1+x+ x + x3
= 1 + x – x2 + x3

Baffour Ba Series, Further Mathematics for Schools Page 197


ii. √8 = (8 )
= (8 )

=8 . /

=8 ( )

But ( ) =( )
3x = 0.03
x = 0.01
Put x = 0.01 in 8 ( )
8 ( ) =8 . – /
=2. ( )– ( ) ( ) /
=2( ( )– )
= 2 (1.009601)
= 2.019202
= 2.01920 (5 decimal places)

2. Write down the expansion of ( ) up to and including the term in x3. By setting x = , use
this expansion to find ans approximation to √ to eight places of decimals.

Solution
. /. / . /. /. /
( ) =1+ ( )+ ( ) + ( ) +…
= 1 – x – x2 – x3 ……..
Setting x = ,

( . /* = ( )

= 1– – . / – . / ……..

=( 8) = √ 8 = 1 – 0.01 – 0.00005 – 0.0000005…


= 0.9899495

Now, 0.98 = =

Baffour Ba Series, Further Mathematics for Schools Page 198


⇒√ 8=√ = √

√ = 0.9899495
7√ = 10 0.9899495
√ =
√ = 1.41421357 (8d.p.)

Exercises 7.14 3. Using the first four terms of the expansion


for (1 + x)n, calculate an approximate value of
A. Use binomial theorem to evaluate each of
√ stating the result correct to five significant
the following:
figures.
1. √ correct to three decimal places.
2. √ 8 correct to four decimal places
3. correct to four decimal places. D. Use binomial theorem to obtain the find
√ four terms of the expansion of the
4. ( ) correct to four decimal places. expressions:
( )
B. Expand the following as far as the term in x3, 1. 2. 3.
stating the values of x for which the expansions ( ( )
are valid: 4. ( 5. √ ( 6. √ (
) ) )
(a) (3 + x)−1 (b) ( ) ⁄ (c) (2 + x)− 4.
Challenge Problem
C. 1. i. Expand ( ) in ascending powers 1. Obtain the expansion in ascending powers of
of x as far as the term in x2 and hence find the x, up to the term in x3, of for | | < . By
( )
approximation for √ 8
putting x = 0.004, deduce an appropriate value
ii. Deduce that √ = 3.464
of , giving your answer to three decimal

2. Determine the series expansion of ( places.
) up to and including the term in x3. By
setting x = , determine without calculator, the 2. Write down the binomial series for √ up
approximate value of √ , giving your answer to and including the term in x4.
to 7 decimal places.

Baffour Ba Series, Further Mathematics for Schools Page 199


Unknowns of Expansions with Rational Exponents
Worked Examples
1. When ( ) is expanded in ascending powers of x, the first four terms are :
A + Bx + x + Cx3
2

Find the possible values of k, A, B and C.

Solution
. /. / . /. / . /
( ) = 1 + . / (kx) + ( ) + ( )

= 1 + kx + ( ) +. /( )
= 1 + kx + ( ) ( ) – ( ) ( )
Now,
A + Bx + x2 + Cx3 comparaed to 1 + kx + ( ) ( ) – ( ) ( )
A = 1…………………....(1)
Bx = kx
B = k………………….(2)

= ………….…..(3)
2
3k = 27
k2 = 9
k=3

C= …………..(4)

Put k = 3 in eqn (2)


B = (3)
B=

Put k = 3 in eqn (4)


C= (3)3
C=
A = 1, B = , C = and k = 3

Baffour Ba Series, Further Mathematics for Schools Page 200


2. The first four terms in the expansion of ( ) are 1 – 6x + ax2 + bx3. Show that a = 15 and
obtain the value of b.

Solution
( )( ) ( )( )( )
( ) = 1 + n (- x) + (- x)2 + (- x)3
( ) ( )
= 1 – nx + (- x)2 + (- x)3
( ) )
= 1 – nx + (- x)2 + (- x)3

( ) )
Comparing 1 – nx + (- x)2 + (- x)3 to 1 – 6x + ax2 + bx3
nx = 6x
n=6
ax2 = . /

a= = = = 15
)
(-x)3 = bx3
( ) ( )
b= = 0 1 = -20

Exercises 7.15 b. (2 + x) √ for | | < 1


1. Write down the first four terms in the
binomial expansion of the following: 4. Expand in ascending powers of
( )( )
a. ( ) b. ( ) c. ( ) x, up to and including the term in x3. Stae the
range of the values of x for which the expansion
2. Write down the first four terms in the
is valid.
binomial expansion of the following:
a. b. √ c. √ 5. Obtain the expansion of ( ) in
ascending powers of x up to and including the
3. Find in simplified form , the first three non term in x2. Hence, show that if k3 and the higher
zero terms in ascending powers of x , in the powers of k are negelected, √ =
series expansion of the following :
4+ +
a. ( for | | <
)

Baffour Ba Series, Further Mathematics for Schools Page 201


8 LINEAR PROGRAMMING Baffour Ba Series

Inequalities 2. For two numbers a and b, we say that a is


We are sometimes concerned with two greater than b if and only if a is to the right of
algebraic expressions that are not equal, one b on the number line. For example, to compare
expression being greater than or less than the -3 and -7, locate each point on the number line
other. Statements that express the inequality of as shown above. Because -3 is to the right of -7
algebraic expressions are called Inequalities. on the number line, we say that -3 is greater
than -7, symbolically expressed as -3 > -7.
The symbols that are used to express
inequalities are given below: 3. The statement a b is true if a is less than b
or if a is equal to b.
Symbol Meaning
< Is less than 4. The statement a b is true if a is less than b
Is less than or equal to of if a is equal to b.
> Is greater than
Is greater than or equal Intervals
to The following nine types of subsets of R are
called intervals.
1. [a, b] = {x R : a ≤ x ≤ b}
In algebra, the size (greater or lesser) of a 2. (a, b) = {x R : a < x < b}
number is determined only by its position on 3. [a, b) = {x R : a ≤ x < b}
the number line. Thus, the following 4. (a, b] = {x R : a < x ≤ b}
conclusions can be made: 5. [a, ) = {x R : a ≤ x}
1. For two numbers a and b, we say that a is 6. (a, ) = {x R : a < x }
less than b if and only if a is to the left of b on 7. (- , b] = {x R : x ≤ b}
the number line. For example, to compare -3 8. (- , b) = {x R : x < b}
and -7 , locate each point on the number line as 9. (- , ) = R
shown below:
Meaning of - and
Intuitively, you may imagine that that is a point
-7 -3 0 , denoted by very far away on the right and
Because -7 is to the left of -3 on the number (- in the left). So (a, ) is the set whose
line we say that -7 is less than -3, symbolically elements are the points between a and , that is
written as -7 < -3. real numbers greater than a.

Baffour Ba Series, Further Mathematics for Schools Page 202


Basic Interval Notation (k any real number)

Inequality Solution Set with Graph


Interval Notation

x >k (k, )
(
k
[
x k [k, ) k

x <k [- , k) )
k
x k (- , k] ]
k
x
Note: Write the solution 3 set to each inequality in

A bracket is used next to k if k is in the interval interval notation and


– graph it
and a parenthesis when k is not in the interval. 1. x > -5 5 2. x 2
A parenthesis is always used on the infinite end x
of the interval because is not a number that Solution 2

might or might not be in the interval. The solution set to the inequality x > - 5 is
+
{x : x > -5}. The solution set is the set of all
1. Interval in the form (a, b), [a, b], (a, b] and numbers to the right of -5 on the number line.
4
[a, b) are called bounded intervals and those in The set is expressed in the interval notation as
x
the form (- , b), (- , b], (a, ), [a, ) and (- (-5, )

, ) are called unbounded intervals. ( 3
-7 -6 -5 -4 -3 -2 -1 0 1 2 3 4
2. Interval in the form (a, b), (- , b), (a, ) and
(- , ) are called open intervals. For each of 2. The solution set to the inequality x 2 is {x :
such intervals the end point(s), if there is any x 2}. The solution set includes 2 and all real
does not belong to the interval. numbers to the left of 2 on the number line. The
set is expressed in the interval notation as (- ,
3. Interval in the form [a, b], (- , b], [a, ) and 2]
(- , ) are called closed intervals. For each of
such intervals the end point(s), if there is any ]
belongs to the interval. -7 -6 -5 -4 -3 -2 -1 0 1 2 3 4

4. Interval in the form [a, b], are called closed Exercises 8.1 x
and bounded intervals. Write the solution set in interval notation
and graph it. y
Worked Examples 1. x 1 2. x -7 3. -5 > x
4. 3 5. x >20 6. x 4.
Baffour Ba Series, Further Mathematics for Schools Page 203
Solving Linear Inequalities state the solution set.
A linear inequality in one variable x, is any
inequality of the form ax + b < 0, where a and b Solution
are real numbers, with a 0. In place of < we 2x – 7 < -1 (Original equation)
may also use , > or . 2x < 6 (Add 7 to both sides)
x < 3 (Divide both sided by 2)
Properties of Inequality The solution set is {x : x < 3}, expressed in
1. Property of Addition interval notation as ( - , 3)
If the same number is added to both sides of an
inequality, the solution set to the inequality is )
-3 -2 -1 0 1 2 3 4
unchanged. The result is an equivalent
inequality. 2. Solve the inequality 5 – 3x < 11. Graph and
state the solution set.
2. Property of Multiplication
 If both sided of an inequality are Solution
multiplied by the same positive number, 5 – 3x < 11(Original equation)
the solution set to the inequality is -3x < 6 (Subtract 5from both sides)
unchanged. x > -2 (Divide both sided by-3, and sign reverses)

 If both sided of an inequality are The solution set is {x : x > -2}, expressed in
multiplied by the same negative interval notation as ( -2, )
number, and the inequality symbol
reversed, the solution set to the
-3 -2 -1 0 1 2 3 4
inequality is unchanged.

Note: 3. Solve - 4. State and graph the


1. Because subtraction is defined in terms of solution set.
addition, the addition property of inequality
also allows subtraction of the same number Solution
from both sides of an inequality. -4 (Original inequality)

2. Because division is defined in terms of -5 . / -5(- 4) (sign reverses)

multiplication, the multiplication property of 8 + 3x 20 (Simplify)


inequality also allows division of both sides by 3x 12 (Divide each side by 3)
the same nonzero real number as long as the x 4
inequality symbol is reversed. The solution set is (- , 4]

Worked Examples ]
-2 -1 0 1 2 3 4 5
1. Solve the inequality 2x – 7 < -1. Graph and

Baffour Ba Series, Further Mathematics for Schools Page 204


4. Solve x– x + . State and graph the Graphing Linear Inequality in one variable
If the inequality involves only x, then graph the
solution set.
vertical line x = k.
Solution  x > k is the region to the right of the
line.
x– x + . (Original inequality)
 x = k is the line itself.
 x < k is the region to the left of the line.
(Multiply through by 6, LCD)
(6) x – (6) (6) x + (6) .
Worked Examples
3x - 4 6x + 8 (Distributive property) Graph each inequality:
3x 6x + 12 (Add 4 to each side) a. y 4 b. x > 3
-3x 12 (subtract 6x from each side)
x -4 (Divide each side by -3, sign reverses) Solution
a. The line y = 4 is a horizontal line with y
The solution set is [- 4, ) intercept (0, 4).
Draw a solid line and shade below it as shown
[ below: y
-7 -6 -5 -4 -3 -2 -1 0 1 2
6

Exercises 8.2 4

A. Solve each of the following inequalities. 2


Express the solutions set in interval notation
and graph it. x
M
1. 2x – 3> 7 2. 3 – 5x 18 -2
u
3. 5 – 4x 19 4. 2x + 3 > 2(x – 4) l
t
5. -2(5x – 1) -5 (5 + 2x)
b. The line x = 3 is vertical line ithrough (3, 0).
6. -3(2x – 1) 2(5– 3x) Any point to the right of this linep
has an x –
l
coordinate larger than 3. y
B. Solve each inequality and graph the y
solution set; 3
x
1. 2 2. 3 – x 2 2

x
3. x – < x – 4. x – < x – 1
2
x2
5. > – 6. – >1 30 x
1 2 3 4
-2 x
7. – ( ) < x + 2 8. -3 . /> – – - x x x 2
3 2 3
3
5 x – – –
2
Baffour Ba Series, Further Mathematics for Schools x Page 205
2
+ 2 3 5
+ x x x
2 2
4 x
2
4 x +
x –
– 4
3
Linear Inequalities in Two variables The point (3, -2) does not satisfy the inequality.
If a, b and c are real numbers with a and b, both
not zero, then ax + by < c is called a linear Exercises 8.3
inequality in two variables. In place of <, the Determine which of the points following each
symbols >, or can be used. Examples of inequality satisfies that inequality:
linear inequalities in two variables are 3x – 4y 1. x – y > 5 (2, 3) (-3, -9) (8, 3)
8, y > 2x – 3, x – y – 10 < 0 2. 2x + y < 3 (-2, 6) (0, 3) (3, 0)
3. y - 2x + 5 (3, 0) (1, 3) (-2, 5)
Solving an Inequality in Two Variables 4. y - x + 6 (2, 0) (-3, 9) (4, 12)
An ordered pair is a solution of an inequality in 5. x > -3y + 4 (2, 3) (7, -1) (0, 5)
two variables if the ordered pairs satisfy the 6. x < -y – 3 (1, 2) (-3, -4) (0, -3)
inequality.
Graph of Linear Inequality in Two
Worked Examples Variables
Determine whether each point satisfies the The graph of linear inequalities in two variables
inequality 2x – 3y 6 consists of all points in the rectangular
a. (4, 1) b. (3, 0) c. (3, -2) coordinate systems that satisfy the inequality.

Solution Strategy for Graphing Linear Inequalities in


a. Substitute x = 4 and y = 1 in 2x – 3y 6 Two variables
2x – 3y 6 Solve the inequality for y, then graph
2(4) – 3(1) 6 y = mx + b
8–3 6  y > mx + b is the region above the line.
5 6 (False)  y = mx + b is the line itself.
The point (4, 1) does not satisfy the inequality.  y < mx + b is the region below the line.

b. Substitute x = 3 and y = 0 in 2x – 3y 6 Steps:


2x – 3y 6 I. Find the intercepts on x and y axes, bearing in
2(3) – 3(0) 6 mind that at the x – intercept, y = 0 and at the y
6–0 6 – intercept, x = 0.
6 6 (True) II. Prepare a table of values.
The point (3, 0) satisfies the inequality. III. Draw the boundary line according to the
table of values, using dashed line for the
c. Substitute x = 3 and y = -2 in 2x – 3y 6 symbols < and >, and solid lines for the
2x – 3y 6 symbols and .
2(3) – 3(-2) 6 IV. For boundaries with the symbol > or ,
6+6 6 shade the area above the line and for
12 6 (True) boundaries with symbol < or , shade the area
below the line.

Baffour Ba Series, Further Mathematics for Schools Page 206


Worked Examples y
Graph each inequality:
1. 2x – 3y < 6 2. y + 2x 3 3. y < +1 2

1
Solution x
0
1. In 2x – 3y < 6, solve for y -2 -1- 1 2 3 4
0
-1
-3y < 6 – 2x 3x
-3y < -2x + 6 3
-3
y> + (divide by -3 and reverse the inequality) –
y> –2 x2
2. In y + 2x 3, solve for y.
Now, in y = – 2; y -2x + 3
Intercept on x – axis,
Now in y = -2x + 3;
When y = 0, 0= –2 Intercept on x – axis;
3(0) = 3. / –3(2) When y = 0, 0 = -2x + 3
2x = 3
0 = 2x – 6
x=
-2x = -6
x = 1.5
x=3 Intercept on x axis is (1.5, 0)
Intercept on x – axis is(3, 0)
Intercept on y – axis;
Intercept on y – axis, When x = 0, y = - 2(0) + 3
When x = 0, y = -2 y=3
Intercept on y – axis is (0, -2) Intercept on y – axis is (0, 3)

Table of values Table of values

x 0 3 x 1.5 0
y -2 0 y 0 3

Draw the boundary line according to the table


Draw the boundary line according to the table
of values, using a dashed line for the boundary
of values. Because of the symbol , every point
because it is not included.
on or above the line satisfy the inequality.
Shade the region above the line because of the
symbol >.

Baffour Ba Series, Further Mathematics for Schools Page 207


To show that the line y = -2x + 3 is included,
y
draw a solid line and shade the region above.
3 -
y 92 4
-3 x–1 x2
2 2
x x
30
1 -3 -2 -1 –2 + 1 2 3 4 0
x 3-1x- -
0 -2 2
-2 -1 -1 1 2 3 4 x5 6
-x -3 x
3 - -x x
1x
-2 2
3 3
-3–
2
x –- +
–+ 2 x
4. Indicate thex solution set of the inequality
45
3x 2
2
+ 3x + 5y 30,4for x, y R.3
–xx
3. In y < + 122 -x +
–2 6 Solution x -
Now, in y = x
3++1 x In 3x + 5y 30,
3
+ solve for2 y,
– x 0
Intercept on x – axis; 5y 30 – 3x -
3x
4 x
When y = 0, 0–x= +1 5y -3x + 30 23
x
2
3(0) = (3) + (3)1 y + 03 - +
– 4
0 = +3 y +6 -
x
x = -3 42 6
3 is (-3, 0)
Intercept on x axis 0
Now, in y = x + + 6,
2

Intercept on y – axis; Intercept on x – axis;


-2 -
When x = 0, y= ( )+1 When y = 0, 1 0= + 6,
x 6
y=1 7 (5) 0= (5)x + (5) 6,
Intercept on y – axis is (0, 1) x 0 = -3x + 30
3x = 30 +
Table of values +x = 10 3
Intercept on x – axis is (10, 0)
x -3 0 Intercept on y6– axis;
y 0 1 0 ( )
When x = 0, y = + 6,
y=6
Draw the boundary line according to the table
Intercept on x – axis is (0, 6)
of values, using a dashed line because the
boundary is not included.

Baffour Ba Series, Further Mathematics for Schools Page 208


Table of values  If the tested area satisfies the inequality,
shade the tested area as the solution set.
x 10 0  If the tested area does not satisfies the
y 0 6 inequality, shade the opposite area as
the solution set.
Draw the boundary line according to the table
 Better still; the point (0, 0) can be used
of values, using a solid line because the
as the standard test point.
boundary is included. Shade the area below the
boundary because of the symbol, , as shown
Worked Examples
below:
Graph the inequality: 2x – 3y < 6

(0, 6) Solution
First find the intercept on the x – axis as (3, 0)
and the intercept on the y – axis as (0, -2). Draw
the boundary line and select any point on one
(10, 0) side of the line, say (0, 1), to test the inequality
as follows: y
2(0) – 3(1) < 6
Exercises 8.4
⇒ -3 < 6.
Graph each inequality: 2
Test point
1. y < x + 4 3. y < -2x + 5 1  (0, 1)
2. y < 2x + 2 4. y > - x + 3 x
0
-2 -1 1 2 3 4
0
-1
B. Graph each inequality:
-2
1. y > x – 3 3. y - x+3
-3
2. y < x + 1 4. y - x+2

The Test Point Method Since the statement is true, the whole of that
The test point method can be used to graph an region of the line satisfies the inequality so the
inequality. With this method, it is not region is shaded as the solution set of the
necessary to solve the inequality for y. inequality.
y

The graph of a linear equation in the form; 2


ax + by = c, separates the coordinate plane into 1
two regions. One region satisfies the inequality 0 x
-2 -1 1 2 3 4
ax + by > c, and the other region satisfies the -1 0
inequality ax + by < c. By testing a point in one -
region, the region which satisfies the inequality -3
2
can be identified. x

Baffour Ba Series, Further Mathematics for Schools Page 209
3
Exercises 8.5 y > x – 2 and y < - 2x + 3
Use the test point method to graph each;
1. x – 4y > 4 4. 3x – 4y < -12 Solution
2. 3x – 5y ≥15 5. y – x ≤ 7 y > x – 2 and y < - 2x + 3
Check (0, 0) in y > x – 2 and y < - 2x + 3
3. x – y < 5 6. x + 3y ≤ 12
0 > 0 – 2 (Correct) and 0 < - 2(0) + 3 (Correct )

Systems of Linear Inequalities: Check (0, -5) in y > x – 2 and y < - 2x + 3


A system of linear inequalities consists of two -5 > 0 – 2 (incorrect) and -5 < - 2(0) + 3 (Correct )
or more inequalities. A point is a solution to a
system of inequalities if it satisfies all of the Check (0, 5) in y > x – 2 and y < - 2x + 3
inequalities in the system. 5 > 0 – 2 (Correct) and 5 < - 2(0) + 3 (incorrect)

Graphical Solution Check (0, 0) in y > x – 2 and y < - 2x + 3


I. Graph the boundary line of each inequality 0 > 4 – 2 (incorrect) and 0 < - 2(4) + 3 (incorrect)
and shade the regions that satisfy each
The only point that satisfies both inequalities of
inequality.
the system is (0, 0). This means that every point
II. The region of intersection of all the
in the region containing (0, 0) also satisfies
inequalities is the region that satisfies all the
both inequalities.
inequalities.
For y = x – 2
Method 2
Intercept on x – axis is (2, 0) and
To determine the region which contains the
Intercept on the y – axis is (0, -2)
points that satisfies a system;
I. Draw the boundary line of each inequality to
x 2 0
divide the plane into four arts, using the
y 0 -2
intercept method.
II. Check one point in each of the region to see
For y= - 2x + 3
whether it satisfies both inequalities.
Intercept on x – axis is (1.5, 0) and
III. Shade the satisfactory region to indicate the
Intercept on the y – axis is (0, 3)
solution of the inequality.

x 1.5 0
Worked Examples
y 0 3
1. Graph all ordered pairs that satisfy the
following system of inequalities:

Baffour Ba Series, Further Mathematics for Schools Page 210


Intercept on x – axis
y When y = 0, 0 = -3x + 4
x Test Point 3x = 4
5 y (0,( 5 ) x = 1.3
4 0 Intercept on the x– axis is (1.3, 0)
3 ,
2 Intercept on y – axis
D1 5 When x = 0, y = -3(0) + 4
Test Point
o)
Test Point (0,0 (0, 4) y=4
m  (  ( x
-2 -1 E )
10 2 3 4 5 6 0
Intercept on y – axis is (0,4)
a-10 0
i
-2
, ,
Table of values
n-3
C 5 5
-4 x 1.3 0
o  Test( Point
-5 y 0 4
- (0,-5)0) )
d , For 2y – x = 2
o
y
m Solving for y;
5
5a 2y = x + 2
4i y= +
)
3n
y= +1
2

1
Intercept on x – axis
0
x When y = 0, 0 = + 1
-2 -1 1 2 3 4 5
-1
6 0
(2) 0 = (2) + (2) 1
-2
0=x+2
-3 x = -2
-4 Intercept on the x – axis is (-2, 0)
-5
Intercept on y – axis

2. Graph all ordered pairs that satisfy the When x = 0, y= +1


following system of inequalities: y=1
y > -3x + 4 and 2y – x > 2 Intercept on y – axis is (0, 1)

Solution Table of values


For y = -3x + 4 X -2 0
Y 0 1

Baffour Ba Series, Further Mathematics for Schools Page 211


Test inequalities are those that lie to the right of the
Check (0, 0) in y > -3x + 4 and 2y – x > 2 vertical line x = 4 and below the horizontal line
0 > -3(0) + 4 (incorrect) 2(0) – (0) > 2 (Incorrect) y=3
y
Check (2, 2) in y > -3x + 4 and 2y – x > 2
2 > -3(2) + 4 (Incorrect) 2(2) – (2) > 2 (Correct) 5
4
Check (1, 4) in y > -3x + 4 and 2y – x > 2 3
1 > -3(4) + 4 (correct) and 2(4) – (1) > 2 (correct) 2

1
Check (2, 1) in y > -3x + 4 and 2y – x > 2
1 > -3(2) + 4 (correct) and 2(1) – (2) > 2 (Incorrect) 0
x
-2 -1 1 2 3 4 5
-1
6 0
y x
The only point that satisfies both inequalities of
-2
the system is (1, 4). Shade all the region
-3
containing the point (1, 4) as shown below.
-4
-5

5 4. Graph the system of inequality;


4 y < x + 4 and y > x - 1
3
2 Solution
1 For y = x + 4
Intercept on x – axis
0
x
-2 -1 1 2 3 4 5 When y = 0, 0 = x + 4
-1
6 0
x = -4
-2 Intercept on the x – axis is (-4, 0)
-3

-4 Intercept on y – axis
-5 When x = 0, y = 0 + 4
y=4

3. Graph the system of inequality;


Intercept on y – axis is (0, 4)
x > 4 and y < 3.
Table of values
Solution
Graph the vertical line x = 4 and the horizontal
x - 4 0
line y = 3, the point that satisfy both
y 0 4

Baffour Ba Series, Further Mathematics for Schools Page 212


For y = x – 1 3. y > -2x + 1 and y < 3x + 5
Intercept on x – axis (-3, 2), (-1, 5), (3, 6),
When y = 0, 0 = x – 1
x=1 4. y < - x + 7 and y < -x + 9
Intercept on the x – axis is (1, 0)
Exercises 8.6
Intercept on y – axis A. Graph each system of inequalities:
When x = 0, y = (0) – 1 1. y > -x – 1 and y>x+1
y=-1 2. y < x + 3 and y < -2x + 4
Intercept on y – axis is (0, -1) 3. y < 2x – 3 and y > -x + 2
4. y > 2x – 1 and y < -x – 4
Table of values 5. 2x + y < 3 and x – 2y > 2
x 1 0
y 0 -1 B. Graph each system of inequalities:
1. 2x – 3y < 6 and x–y>3
y
2. 3x – 2y > 6 and x + y < 4
3. 3x – 5y < 15 and 3x + 2y < 12
5 4. x – 4y < 0 and x + y > 0
4 5. 3x + 2y < 2 and – x – 2y > 4
3
2 6. On a diagram, shade the area where the
following are all true:
1
x + y > 3, x + 2y < 8, x < 0
0 x
- 4 -3 -2 -1 1 2 3 4 5
-1 0 C. Graph each system of inequalities.
1. x > 5 and y>5
-2
2. x < 3 and y>2
-3
3. y < - 1 and x > -3
-4
4. y > -2 and x<1
-5
5. y < 2 and 2x + 3y < 6
6. x > 1 and y – 2x < 3

Try Linear Programming


Determine which of the points following each It is a method of solving certain problems in
system is a solution; which the mathematical model consists of a
1. x – y < 5 and 2x + y > 3 system of linear inequalities which admits
(4, 3), (8, 2), (-3, 10), many solutions. Of all the feasible solutions,
one (or more) may give the best solution called
2. x + y < 4 and 2x – y < 3 optimal solution.
(2, -3), (1, 1), (0, -1),

Baffour Ba Series, Further Mathematics for Schools Page 213


Graphing the constraints
y
In linear programming, there are two variables
that must satisfy several linear inequalities. 6
These inequalities are called the constraints, 3x +2y = 12
5
because they restrict the variables to only (0, 4)
certain values. 3
(2, 3)

2
A graph in the coordinate plane is used to x + 2y = 8
1
indicate the points that satisfy all of the
constraints. x
-1 0 1 2 3 (4, 0) 5 6 7 8 0
-1
Worked Examples -2
1. Graph the solution set to the system of -3
inequalities and identify each vertex of the
region:
x 0, y 0, 3x + 2y 12 and x + 2y 8 The vertices of the region are identified as (0,
0), (4, 0) and (0, 4). To find the fourth vertex,
Solution solve 3x + 2y = 12 and x + 2y = 8
Graph the line 3x + 2y = 12 using the intercepts simultaneously:
(0, 6) and (4, 0). The points on or below the 3x + 2y = 12 ………..(1)
line 3x + 2y = 12 satisfy 3x + 2y 12. x + 2y = 8 ……..……(2)

Graph the line x + 2y = 8, using the intercepts eqn (1) – eqn (2)
(0, 4) and (8, 0). The point on or below the line 2x = 4
x + 2y 8 x=2

The points that satisfy x 0, are on or to the


Put x = 2 in eqn (1)
right of the y – axis.
3(2) + 2y = 12
The point on or above the x – axis satisfy y 0, 2y = 6
y=3
Points that satisfy all the inequalities are in the The fourth vertex is (2, 3)
shaded region.

Baffour Ba Series, Further Mathematics for Schools Page 214


Exercises 8.7 Flour Fat
A. Indicate on Cartesian diagrams, the (g) (g)
solution set of each of the systems of First kind of cake (x) 200 25
Second kind of cake (y) 100 50
inequalities for x, y R:
1. x 0, y 0, x+ y 5
Suppose that x is the number of cakes of the
2. x 0, y 0, x + 2y 8
first kind and y is the number of cake of the
3. x 0, y 0, 2x + 3y 12
second kind.
4. x 8, y 0, y 5, x + y 8 Since we shall need (200x + 100y)g of flour and
we have 4000g,
B. Indicate on Cartesian diagrams, the 200x + 100y 4000
solution set of each of the systems of ⇒2x + y 40……………(1)
inequalities for x, y R:
1. x 0, y 0, x + y 4, x + y 7
Since we shall need (25x + 50y)g of fat and we
2. x 0, y 0, x + y 6, 3x + 8y 24 have 1200g,
3. x 0, x 0, y 0, y x+2 25x + 50y 1200
4. x 8, y 6, x + 4y 8, 2x + y > 8 ⇒x + 2y 48……………(2)

Writing the Constraints Since x and y cannot be negative integers,


(Mathematical model) x 0 …………………..(3)
1. Observe that there are two natural y 0 ………………….(4)
constraints: x 0 and y 0
2. Represent one same commodity by x and Simplifying the inequalities;
another same commodity by y. 2x + y 40……………(1)
x + 2y 48……………(2)
Worked Examples
x 0 …………………..(3)
1. One kind of cake requires 200 g of flour and
y 0 ………………….(4)
25g of fat, and another requires kind of cake
requires 100g of flour and 50g of fat. Suppose
Now, for the line 2x + y = 40
we want to make as many cake as possible but
have only 4000g of flour and 1200g of fat
Intercept on x – axis:
available, although there is no shortage of the
When y = 0, 2x + 0 = 40
various other ingredients. How many cakes of
2x = 40
each kind should we make?
x = 20
The intercept on the x – axis is (20, 0)
Solution
The data in the problem are set out concisely in
Intercept on y – axis:
the following table;
When x = 0, 2(0) + y = 40
y = 40

Baffour Ba Series, Further Mathematics for Schools Page 215


The intercept on the y – axis is (0, 40) (ft2) of plywood and 6ft2 of insulation. A large
dog house requires 16ft2 of plywood and 3ft2 of
x 20 0 insulation. Jude has available only 48ft2 of
y 0 40 plywood and 18ft2 of insulation. Write the
constraints on the number of small and large of
For the line x + 2y = 48, houses that he can build with the available
supplies and graph the solution set to the
Intercept on x – axis: system of constraints.
When y = 0, x + 2(0) = 48
x = 48 Solution
Let x represent the number of small dog houses
The intercept on the x – axis is (48, 0) and y represent the number of large dog houses

Intercept on y – axis: plywood Insulation


When x = 0, 0 + 2y = 48 (ft2) (ft2)
2y = 48 Small dog 8 6
y = 24 house (x)
The intercept on the y – axis is (0, 24) Large dog 16 3
house (y)
x 48 0
y 0 24 A small dog house requires 8sq feet (ft2) of
y
plywood and a large dog house requires 16ft2 of
44 plywood. Since only 48ft2of plywood is
40 available, we have;
36 8x + 16y 48……………(1)
32
28 A small dog house requires 6 (ft2) of insulation
24 and a large dog house requires 3ft2 of
20 (13, 16 .8) insulation. Since only 18ft2 of insulation is
16 available, we have;
12 6x + 3y 18……………(2)
8
4 Natural constraints
x
10 20 30 40 50 60 70 80 x 0 …………………..(3)
y 0 …………………..(4)
13g of first kind (x) and 16.8g of second kind (y)

2. Jude is in the business constructing dog Simplify the inequalities to get the following
houses. A small dog house requires 8sq feet constraints:
x + 2y 6……………(1)

Baffour Ba Series, Further Mathematics for Schools Page 216


2x + y 6……………(2)
y
x 0 ……………… ..(3)
y 0 …………………(4) 6
5
x + 2y = 6
Now, for the line x + 2y = 6 4
Intercept on x – axis: (0, 3)
When y = 0, x + 2(0) = 6 2 (2, 2)
x=6 1 2x + y = 6
The intercept on the x – axis is (6, 0)
x
0 1 2 (3, 0) 4 5 6 0
Intercept on y – axis:
When x = 0, 0 + 2y = 6
y=3 Exercises 8.8
The intercept on the y – axis is (0, 3) Graph the solution set to each system of
inequalities and identify each vertex of the
x 6 0 region.
y 0 3 1. x 0, y 0, x + y 5
2. x 0, y 0, 2x + y 4, x + y 3
For the line 2x + y = 6 3. x 0, y 0, 2x + y 3, x + y 2
Intercept on x – axis: 4. x 0, y 0, 3x + 2y 12, 2x + y 7
When y = 0, 2x + 0 = 6 5. x 0, y 0, x + 3y 15, 2x + y 10
x=3
The intercept on the x – axis is (3, 0) Maximum or Minimum Values
Consider the points belonging to the solution
Intercept on y – axis: set of the system of inequalities for which x
When x = 0, 2(0) + y = 6 W and y W. Each member of this set is a
y=6 called a feasible solution of the problem. For
The intercept on the y – axis is (0, 6) example in the diagram below any point in the
shaded region is a feasible solution:
x 3 0
y 0 6 Refer to page . …. example one

Baffour Ba Series, Further Mathematics for Schools Page 217


y
Worked Example
44
1. a. Show the solution set of the following
40
system of inequalities;
36
32
x + y 3, x + 2y 4, x 0, y 0
28
b. Mark with dots the points (0, 0), (1, 0), (2,
0), (3, 0), (0, 1), (1, 1), (2, 1), (0, 2)
24
(13, 16 .8) c. which points give the maximum value of
16 x + y on the solution set, for x, y W?
12
Solution
4 a. For the line x + y = 3
x Intercept on x – axis:
10 20 30 40 50 60 70 80
When y = 0, x + 0 = 3
The points (0, 0), (0, 24), (5, 21), (10, 19), (11, x=3
18), (12, 15), (15, 10), (20, 0) are all feasible The intercept on the x – axis is (3, 0)
solutions.
Intercept on y – axis:
To get a solution which is as good as possible, When x = 0, 0 + y = 3
called an optimal solution, the need arises to y=3
find points whose coordinates satisfy the given The intercept on the y – axis is (0, 3)
conditions and which gives a maximum value
of x + y. From the diagram, the values of x + y x 3 0
are investigated at points on, or near the y 0 3
boundary of the polygon formed, especially at
a vertex. The coordinates and the For the line x + 2y = 4
corresponding values of x and y are listed as
follows: Intercept on x – axis:
When y = 0, x + 2(0) = 4
x 0 0 5 10 11 12 15 20 x=4
y 0 24 21 19 18 15 10 0 The intercept on the x – axis is (4, 0)
x+y 0 24 26 29 29 27 25 20
Intercept on y – axis:
Optimal solutions are given by x = 11 and y = When x = 0, 0 + 2y = 4
18 and by x = 10 and y = 19. So the optimal y=2
value of x + y is 29. The intercept on the y – axis is (0, 2)

Hence the best use of ingredients is by baking x 4 0


11 cakes of the first kind and 18 of the second y 0 2
or 10 of the first and 19 of the second.

Baffour Ba Series, Further Mathematics for Schools Page 218


Natural constraints: x 0, and y 0 3. Graph the set determine by:
y x + 3y 30, 5x + y 50, 5x + 3y 90
6 a. Examine the values of:
5 i. 3x + y
4 ii. 2x + 3y at and near the points (0, 50), (5, 20),
3 (15, 5), (30, 0) and hence find the minimum
x+y=3
value on the solution set of each of these linear
(0, 2)
forms.
1 (1, 1)
x + 2y = 4
x Maximizing and Minimizing a Linear
0 1 2 (3, 0) 4 5 6 0 Function
The maximum or minimum value of a linear
x 0 1 2 3 0 1 2 0 function subject to linear constraints occur at a
y 0 0 0 0 1 1 1 2 vertex of the region determine by the
x+y 0 1 2 3 1 2 3 2
constraints.
c. From the table, the points that give the
maximum value of x + y is (3, 0) and (2, 1) A function of the form f(x, y)= Ax + By + C,
where A,B and C are real numbers, is called a
Exercises 8.9 linear function of two variables.
A. 1. Show the solution set of the system of
inequalities; Strategy for Linear programming
x + y 4, 2x + y 6, x 0, y 0 Use the following steps to find the maximum or
b. On the diagram mark with dots the points (3, minimum value of a linear function subject to
0), (2, 1), (1, 2), (2, 2), (0, 3), (1, 3), (0, 4) and linear constraints.
write the value of 2x + y at each point. 1. Graph the region that satisfies all the
c. which points give the maximum value of x + constraints.
y on the solution set , for x, y W? 2. Determine the coordinates of each vertex of
d. Find the maximum value of 2x + y on the the region.
solution set for x, y W? 3. Evaluate the function at each vertex of the
region.
2. Show the solution set of the system of 4. Identify which vertex gives the maximum or
inequalities; the minimum value of the function.
x + y 3, x + 2y 4, x 0, y 0
b. Mark the points (4, 0), (4, 1), (3, 1), (2, 1), Worked Examples
(2, 2), (1, 2), (1, 3) , (0, 3), (0, 4) and write the 1. A small dog house requires 8ft2 of plywood
and write the value of 2x + 3y at each point. and 6ft2 of insulation. A large dog house
c. Deduce the minimum value of 2x + 3y on the requires 16 ft2 of plywood and 3 ft2 of
solution set, for x, y W and the corresponding insulation. Only 48ft2 of ply wood an s18ft of
replacements for x and y. insulation are available. If a small dog house

Baffour Ba Series, Further Mathematics for Schools Page 219


sells for Ghȼ15 and a large dog house sell for Intercept on y – axis:
Ghȼ20, then how many dog houses of each type When x = 0, 0 + 2y = 6
should be built to maximize the revenue and to y=3
satisfy the constraints? The intercept on the y – axis is (0, 3)

Solution x 6 0
Let x represent the number of small dog houses y 0 3
and y represent the number of large dog houses.
For the line 2x + y = 6
plywood Insulation Intercept on x – axis:
(ft2) (ft2) When y = 0, 2x + 0 = 6
Small dog house (x) 8 6 x=3
Large dog house (y) 16 3 The intercept on the x – axis is (3, 0)

A small dog house requires 8sq feet (ft2) of Intercept on y – axis:


plywood and a large dog house requires 16ft2 of When x = 0, 2(0) + y = 6
plywood. Since only 48ft2 of plywood is y=6
available, we have; The intercept on the y – axis is (0, 6)
8x + 16y 48……………(1)
x 3 0
A small dog house requires 6 (ft2) of insulation y 0 6
and a large dog house requires 3ft2 of
insulation. Since only 18ft2 of insulation is y
available, we have; 6
6x + 3y 18……………(2) 5
x + 2y = 6
4
Simplify the inequalities to get the following (0, 3)
constraints: 2 (2, 2)
x + 2y 6……………(1)
1 2x + y = 6
2x + y 6……………(2)
x 0 ……………… ..(3) x
0 1 2 (3, 0) 4 5 6 0
y 0 …………………(4)

Now, for the line x + 2y = 6 From the graph, the vertices are (0,0), (0, 3), (3,
Intercept on x – axis: 0) and (2, 2)
When y = 0, x + 2(0) = 6 The revenue function is:
x=6 R(x, y) = 15x + 20y
The intercept on the x – axis is (6, 0)

Baffour Ba Series, Further Mathematics for Schools Page 220


The maximum value of the function occurs at carbohydrate. If the meal is to contain at least
the vertex. Hence evaluate the function at each 18 g of carbohydrate, then;
vertex as: 6x + 3y 18……………(2)
R(0, 0) = 15(0) + 20(0) = Gh¢0
R(0, 3) = 15(0) + 20(3) = Gh¢60 Simplify the two inequalities and use the two
R(3, 0) = 15(3) + 20(0) = Gh¢45 natural constraints to obtain the following
R(2, 2) = 15(2) + 20(2) = Gh¢70 inequalities:
x + 2y 6 …………….(1)
From the list, the maximum revenue is Gh¢70 2x + y 6……………..(2)
when two small and two large dog houses are x 0 ………………….(3)
built. The minimum revenue is Gh¢0 when no y 0…………………..(4)
dog houses of either type are built.
Now, for the line x + 2y = 6
2. One serving of food A contains 2g of protein Intercept on x – axis:
and 6g of carbohydrates. One serving of food B When y = 0, x + 2(0)= 6
contains 4g of protein and 3g of carbohydrates. x=6
A dietician wants a meal that contains at least The intercept on the x – axis is (6, 0)
12g of protein and at least 18g of carbohydrate.
If the cost of food A is Gh¢9.00 per serving and Intercept on y – axis:
the cost of food B is Gh¢20.00 per serving, then When x = 0, 0 + 2y = 6
how many serving of each food would y=3
minimize the cost and satisfy the constraints? The intercept on the y – axis is (0, 3)

Solution x 6 0
Let x represent the number of servings of food y 0 3
A and y represent the number of servings of
For the line 2x + y = 6
food B.
Intercept on x – axis:
When y = 0, 2x + 0 = 6
Protein (g) Carbohydrate(g) x=3
Food A (x) 2 6 The intercept on the x – axis is (3, 0)
Food B (y) 4 3
Intercept on y – axis:
Each serving of A contains 2g of protein and When x = 0, 2(0) + y = 6
each serving of B contains 4g of protein. If the y=6
meal is to contain at least 12 of protein, then
2x + 4y 12 ……………(1) The intercept on the y – axis is (0, 6)

Each serving of A contains 6g of carbohydrate x 3 0


and each serving of B contains 3g of y 0 6

Baffour Ba Series, Further Mathematics for Schools Page 221


each touring model and Gh¢12.00 on each
racing model, write down an expression in x
y
and y for his total profit. What is the maximum
6 x + 2y = 6 profit?
5
4 Solution
(0, 3) a. Let x represent touring models and y
2 represents racing models.
(2, 2)
1 2x + y = 6

x Touring model x 30x


0 1 2 (3, 0) 4 5 Racing model y 40y
6 0

From the graph the vertices are (0, 6), (6, 0), The dealer wishes to buy up to 25 cycles for
and (2, 2) stock s:
⇒x + y 25……………..(1)
The cost of x serving of A and y servings of B
is given by the function, He intends to purchase touring models at
C(x, y) = 9x + 20y Gh¢30.00 each and racing models at Gh¢40.00
each, and has planned an outlay of not more
The cost at each vertex is as follows; than Gh¢840.00
C(0, 6) = 9(0) + 20(6) = Gh¢120 ⇒30x + 40y 840……………(2)
C(6, 0) = 9(6) + 20(0) = Gh¢54
C(2, 2) = 9(2) + 20(2) = Gh¢58 The involving inequalities are:
The minimum cost of Gh¢54 is attained by x + y 25…………….(1)
using 6 servings of A and no servings of B. 3x + 4y 84………….(2)
x 0 ………………….(3)
3. A cycle dealer wishes to buy up to 25 cycles y 0…………………..(4)
for stock. He intends to purchase touring
models at Gh¢30.00 each and racing models at Now, for the line x + y = 25
Gh¢40.00 each, and has planned an outlay of Intercept on x – axis:
not more than Gh¢840.00 When y = 0, x + 0 = 25
a. Suppose he buys x touring models and y x = 25
racing models. Write down four inequalities The intercept on the x – axis is (25, 0)
that must be satisfied by x and y.
b. Show graphically the solution set of the Intercept on y – axis:
system of inequalities, taking replacements for When x = 0, 0 + y = 25
x and y from 0 to 30. y = 25
c. If he expects to make a profit of Gh¢10.00 on The intercept on the y – axis is (0, 25).

Baffour Ba Series, Further Mathematics for Schools Page 222


x 25 0 The profit at each vertex:
y 0 25 P(0, 0) = 10(0) +12(0) = Gh¢0
P(0, 2) = 10(0) +12(20) = Gh¢240
For the line 3x + 4y = 84 P(16, 9) = 10(16) +12(9) = Gh¢268
Intercept on x – axis: P(25, 0) = 10(25) +12(0) = Gh¢250
When y = 0, 3x + 4(0) = 84
3x = 84 The maximum profit of Gh¢268 is attained by
x = 28 buying 16 touring models and 9 racing models.
The intercept on the x – axis is (28, 0)
Exercises 8.10
Intercept on y – axis: 1. A factory turns out two articles A and B,
When x = 0, 3(0) + 4y = 84 each of which is processed by two machines M
4y = 84 and N, A requires two hours of M and 4 hours
y = 21 of N. B requires 4 hours of M and 2 hours of N.
The intercept on the y – axis is (0, 21) a. If x is the number of A and y is the number of
B produced daily, write down two inequalities
x 28 0 in x and y, noting that neither M nor N can work
y 0 21 more that 24 hour a day.
b. b. Assuming that all articles produced are sold,
y if each article A yields a profit of Ghȼ3.00 and
article B yields a profit of Ghȼ5.00, find a
relation giving the daily profit GhȼP;
c. From graphical consideration, find how
45
many of each article should be produced daily
40
35
for maximum profit. Calculate this profit.
30
25 2. An aircraft has seats for not more than 48
x + y = 25
20 passengers. Those willing to pay first – class
15 fares can take 60kg of baggage each, but tourist
10 (16, 9) class passengers are restricted to 20 kg each.
5 3x + 4y = 84 Only 1440kg of baggage can be carried
x together.
5 10 15 20 25 30 35 40 45
a. If x is the number of first class passengers
and y is the number of tourist passengers, write
c. The function is : down in simplest form, four inequalities
P(x, y) = 10x +12y satisfied by x and y.
From the graph, the vertices are (0,0), (0, 20), b. Illustrate on a squared paper the regions in
(16, 9) and (25,0) which feasible solution of these inequalities can
be found.

Baffour Ba Series, Further Mathematics for Schools Page 223


c. If first – class fares are Gh¢100.00 and a. suppose that x is the number of cars and y is
tourist fares Gh¢50.00 each, use the diagram to the number of buses to be accommodated, write
find the number of passengers of each kind that down a system of inequalities in x and y which
should be carried for maximum profit, and the models the given facts.
gross sum that will be obtained. b. Show on a graph the solution set of the
system of inequalities.
3. One cake requires 150g flour and 50g fat, c. If the parking charge for a car is Gh¢25 and
and another requires75 g flour and 75g fat. We for a bus Gh¢75, the income, I, is given by I =
want to make as many cakes as possible of the 25x + 75y. By maximizing I, find how many of
two kinds when 2.25 kg flour and 1.5 kg fat are each should be parked for maximum income,
available. and state this income. Ans: 20, 10, Gh12.50
a. Let x be the number of cakes of the first kind,
and y the number of cakes of the second kind. 6. A doctor advises a patient to take daily at
Write down two obvious inequalities for x and least 10 units of vitamin B1 and at least 15 units
y, and show that the two further inequalities of vitamin B2. That patient finds he can buy
reduce to 2x + y 30 and 2x + 3y 60. tablets containing 2 units of vitamin B1 and 1
b. Show on a Cartesian diagram the solution set unit of vitamin B2 or capsules containing 1 unit
of the above inequalities. of B1 and 3 units of B2.
c. Find replacements for x and y on the solution a. Suppose that the patient uses x tablets and y
set which will maximize x + y, and hence write capsules daily. Write down a system of four
down the number of each kind of cake which inequalities in x and y which must be satisfied.
gives as good a solution as possible. b. Taking a scale of 1 cm to 1 unit on each axis,
show the solution set of this system.
4. Sam and Doris manufacture rocking chairs c. Assuming that a tablet cost Gh¢0.5 and a
and porch swings. Each rocker requires 3 hour capsule Ghȼ1.00, write down the daily cost I
of work from Sam and 2 hours from Doris. terms of x, y and hence find how many of each
Each swing requires 2 hours of work from Sam the patient should take to make the cost as
and 2 hours from Doris. Sam cannot work more small as possible. State the daily cost.
than 48 hours per week, and Doris cannot work
more than 40 hours per week. If a rocker sells 7. Gh¢100 worth of product A needs 30 kg of
for Gh¢160.00 and a swing sells for raw material and 18 hours machine time.
Gh¢100.00, then how many of each should be Product B needs 20 kg of the same raw material
made per week to maximize the revenue? and 24 hours of machine time for Gh¢100
worth. Find, by graphical method, the
5. The parking area in a small car park is maximum value of products that can be made
360m2. The average area for a car is 6m2 and with 720 hours of machine time and 750kg of
for a bus 24m2. Not more than 30 vehicles can raw material. (Let x be the number Gh¢100
be accommodated. worth of A and y, the number of Gh¢100 worth
of B that are required.

Baffour Ba Series, Further Mathematics for Schools Page 224


8. A manufacturer makes two compounds X 10. An agency for famine relief has available
and Y which contains the following quantities 10 sacks of rice and 20 sacks of beans. Each
of vitamins A and B. sack weighs 50kg. A sack of rice occupies
0.1m3, and 750 meals can be served from it. A
X Y sack of beans occupies occupies a volume of
Vit A 5 units per g 5 units per g 0.25m3 and 1000 meals can be served from it.
Vit B 15 units per g 5 units per g The delivery van can carry 600kg in weight and
2 m3 in volume;
X cost Gh¢2.00 per g and Y cost Gh¢1.00 per g. i. If the van is loaded with x sacks of rice and y
The manufacturer wants a mixture of X and Y to sacks of of beans, write down six inequalities
contain at least 50 units of vitamin A and at which x and y must satisfy.
least 60 units of vitamin B. how many grams of ii. Illustrate these inequalities on a graph,
each does he use to make the mixture as showing the area in which x and y must lie.
cheaply as possible. iii. Write down an expression for the number of
meals which can be served from the van load.
9. A farmer who wishes to grow crop X and Y Hence, find how many sacks of each should be
has kept an area of about 70 hectares of land for taken in order to provide the grestest number of
this purpose. He has 240 man – days of labour meals.
available to work the land, and he spents up to
Gh1800. The requirement of the ccrops are as 11. A contractor hiring earth moving machine
follows: equipment has the choice of two machines.
Type A cost Ghȼ25 per day to hire, needs one
X Y man to operate it, and moves 30 tonnes of earth
Min number of hectares to be sown 10 20 per day. Type B cost Ghȼ10 per day, needs four
Man – days per hectares 2 4 men to operate it and moves 70 tonnes of earth
Cash cost per hectare in Ghȼ 30 20 per day. The contractor can spend up to
Ghȼ500 per day, has a labour force of 64 men
If x and y represent the number of hectares to available, and can use a maximum of 25
be used for the crop X and Y respectively, write machines on the site.
down in their simplest form the inequalities x i. If he hires x machines of type A and y
and y must satisfy. machines of type B, write down three
inequalities, other than x ≥ 0 and y ≥ 0, which
On the graph sheet, using a scale of 2cm to 10 must be satistfied. Show graphically the set of
units on both the x – axis and the y – axis, draw possible values of (x. y)
appropriate straight lines to find the region ii. Write down an expression in terms of x and y
within which the point (x, y) must lie if the for the number of tonnes of earth moved dialy,
inequalities are to be satisfied. Indicate the and hence find the maximum weight of earth
region by thickening its boundaries. the contractor can move in a day.

Baffour Ba Series, Further Mathematics for Schools Page 225


Challenge Problems so as to contain at least 6 tonnes of copper, 7.2
1. a. Graph the set determined by : tonnes of A and 6 tonnes of B.
x + 3y ≥ 30, 5x + y ≥50, 5x + 3y≥90 a. If x tonnes of X are mixed and y tons of Y,
b. Examine the values of: show that 5x + 2y 60, x 0, y 0 and write
i. 3x + y down in simplest form two other inequalities
ii. 2x + 3y at and near the points (0, 50), (5, 20), that must be satisfied.
(15, 5), (30, 0) and hence find the minimum b. show on a Cartesian diagram, with scale 2cm
value on the solution set of each of these linear to 5 units, the region of feasible solutions for
forms. the problem.
c. Given that X cost Gh¢400 a tonne and Y cost
2. The diest for some animals is to consist of Gh¢200 a tonne, write down an expression for
two foods A and B. The animals must have at the total cost. Hence, find the number of tonnes
least 20 units per day of a certain ingredients, of X and Y which must be mixed and the total
of which 3 units occur in each kilogram of A cost.
but only 1 unit in each kilogram of B. They also d. If the cost of Y is doubled, what would the
need 30 units per day of another ingredients, of minimum cost have been?
which 1 unit occurs in each kilogram of A and 2
units in each kilogram of B. 3. A factory employs unskilled workers who
a. Suppose that xkg of A and y kg of B are earn Ghȼ12.00 a week and skilled workers who
required. Write down three inequalities in earn Ghȼ24.00 a week . The weekly wage bill
addition to x ≥ 0 and y ≥ 0, that must be must not exceed Ghȼ2,160.00. The machine
satisfied by x and y. requires a minimum of 105 operators of whom
b. Illustrate the solution set by inequations by a at least 40 must be skilled. Union regulations
graph . require that the number of skilled workers
c. What is the least mass of A and of B to should be at least half the number of unskilled
provide the necessary diet? workers.
i. If x is the number of unskilled and y is the
3. The composition per tonne (1000kg) of two number of skilled workers, write down the
metal alloys x and y is shown in the following inequalities , other than x ≥ 0 and y ≥ 0 which
table; govern x and y.
ii. Illustrate these results on a diagram, and
Copper Metal A Metal B from your diagram,obtain the range within
kg kg kg which the number of unskilled workers must
Alloy X 500 300 200 lie.
Allow Y 200 300 500
Using the Search Line ax + by = k
Another metal alloy is to be produced as
Consider the shaded region below which shows
cheaply as possible from a mixture of x and y
the solution set of the system of inequalities:
3x + y 9, 2x + y 8, x 0, y 0 for x, y R

Baffour Ba Series, Further Mathematics for Schools Page 226


y point P at the extremity of the feasible region.
The required line is x + 2y = 7, and so the
maximum value of x + 2y in the region is 7.
That is at P(3, 2), x + 2y = 3 + 2(2) =7 .
4
Note:
3 The line parallel to the line ax + by = 0 can be
drawn using a ruler and set square. For x, y
2 P R, the optimal point or points, always come on
the edge of the feasible region and usually a
1 vertex.

x Worked Examples
1 2 3 4 5 6 7 8 9
Show the solution set of the following system
x + 2y = 0 x + 2y = 2 x + 2y = 4 x + 2y = 7 of inequalities:
x + y 4, x + 3y 6, x 0, y 0 for x, y R
To find the maximum value of the linear form a. on your diagram, draw the set of lines x + 2y
such as x + 2ysubject to the above conditions,
= k, k = 0, 1, 3, 5
consider the set of parallel lines corresponding
to the equation x + 2y = k, given by different b. Deduce the maximum value of x + 2y subject
replacements for k, where k R to the above restriction, and state the
corresponding replacements for x and y.
In the figure, the lines have equations given by (Use scale of 2cm : 1 unit)
k = 0, 2, 4, 7. Notice that the greater the value
of k the further the line is from the origin. Solution
Name the equations as:
Examining the properties of some of these x + y 4…………… (1)
lines: x + 3y 6………… (2)
1. k = 0, gives the line through the origin, x 0 ……………… (3)
x + 2y = 0, which gives minimum value zero of y 0…………………(4)
x + 2y in the region. That is, when x = 0 and y =
0, x + 2y = 0 + 2(0) = 0 For the line x + y = 4
Intercept on x – axis:
2. k = 2, gives the line which cuts off a When y = 0, x + 0 = 4
triangular portion of the feasible region in x=4
which x + 2y is less than or equal to 2. That is, The intercept on the x – axis is (4, 0)
when x = 0 and y = 1, x + 2y = 0 + 2(1) = 2
Intercept on y – axis:
3. To find an optimal value of x + 2y, draw a When x = 0, 0 + y = 4
line parallel to the line x + 2y = 0, through the y=4

Baffour Ba Series, Further Mathematics for Schools Page 227


The intercept on the y – axis is (0, 4) The maximum value of x + 2y is 5 and it occurs
at (3, 1).
x 4 0
y 0 4 Exercises 8.11
1. Two variable, x and satisfies the following
For the line x + 3y = 6 inequalities:
Intercept on x – axis: 2x + y 14, x + 2y 16, x – y 4
When y = 0, x + 3(0) = 6 a. Draw a graph of these inequalities and the
x=6 region in which (x, y) can lie.
The intercept on the x – axis is (6, 0) b. Find the maximum and the minimum values
of 3y + x.
Intercept on y – axis: c. Find also the maximum and minimum values
When x = 0, 0 + 3y = 6 of y + 3x. Ans : 28, 18
3y = 6
y=2 2. i. Show the solution set of the system of
The intercept on the y – axis is (0, 2) inequalities (x, y R):
x + y 6, 2x + y 3, x 1, x 4, y
x 6 0 ii. Using ruler and set square draw suitable
y 0 2 parallels to the line 4x + y = 0 to show that,
For x + 2y = 0, when x = 0, y=0
subject to the above conditions:
For x + 2y = 1, when x = 0, y = 0.5
ithe minimum value of 4x + y is 5
For x + 2y = 3, when x = 0, y = 1.5
ii. the maximum value of 4x + y is 18.
For x + 2y = 5, when x = 0, y = 2.5
y Quadratic Inequalities
A quadratic inequality is the one that can be
written in the following standard forms:
ax2 + bx + c < 0, ax2 + bx + c 0, ax2 + bx +
4 c > 0, ax2 + bx + c 0
x+y=4
3 To be precise, a quadratic inequality is said to
be in standard form when the inequality is set to
2 zero. For example, 2x2 - 6x + 4 = 0.
(3, 1)
1 Solving Quadratic Inequalities
x + 3y = 6 In general, quadratic inequalities are solved
x
1 2 3 4 5 6 7 40 45
with the following steps:
x + 2y = 5 1. Replace ≥, ≤, < > with = (make it an
x + 2y = 3 equation.
x + 2y = 1
2. Solve the equation to find the roots.

Baffour Ba Series, Further Mathematics for Schools Page 228


3. Test a number between the roots in the Note that product of two factors with the same
original inequality (usually 0) sign is positive and with opposite signs is
4. Two possibilities arise: negative.
a. If the inequality holds, then the  If the quadratic expression is less than
solution lies between the roots. or less than or equal to 0, then the
b. If the inequality does not hold, then values that cause the quadratic
the solution does not lie between the expression to negative are required.
roots.
 If the quadratic expression is greater
Note that you can also test a number outside the than or greater than or equal to 0, then
roots. The roots of the equation are also called the values that cause the quadratic
“critical values ”of the inequality. expression to negative are required.

Other methods such as the sign graph of factors 6. Write the solution set and graph it.
and the test point (similar to the generally
steps) can also be used. Worked Examples
1. Solve x2 + 2x – 15 < 0, using a sign graph of
Worked Examples factors. Write your answer in interval notation
Using a Sign Graph of Factors and graph it.
This method works if only the quadratic
inequality can be factorized. If not then the test Solution
point method is applicable. x2 + 2x – 15 < 0 (standard form)

Steps x2 + 2x – 15 = 0 (solve by factorization)


2
1. Write the inequality in standard form. (x – 3x) + (5x – 15) = 0
2. Solve the quadratic equation, ax2 + bx + c = x(x – 3) + 5(x – 3)
0, by factorization to obtain the roots or (x + 5)(x – 3) = 0
boundaries. ⇒x + 5 = 0 or x – 3 = 0
3. Use the boundaries obtained to mark off x= -5 or x = 3
intervals on the number line to create three -5 and 3 are boundary points.
regions and list all the factors found in step 2.
4. Find the sign of every factor in the interval The two boundaries create three regions (- , -
by choosing any value in the interval and 5) (-5, 3) and (3, ) on the graph or number
plugging into each factor. Make sure the sign line as shown below:
(positive or negative) of every factor in every
interval is found. x+5
5. Using the signs found, determine the sign of x -3
the overall quadratic function in each interval.
, -5 -5 (-5, 3) 3 ,

Baffour Ba Series, Further Mathematics for Schools Page 229


Test -6 in the first interval: Solution set = {x : -5 < x < 3}
When x = -6 Interval notation = (-5, 3)
(x + 5) and (x – 3)
- 6 + 5 = -1 and - 6 – 3 = -9 Graph
(–,–)
( )
-5 3
Test 0 in the second interval:
Alternatively;
When x = 0
(x + 5) and (x – 3) becomes
0 + 5 = 5 and 0 – 3 = -3
(+,–)
-5 3

Test 4 in the third interval:


When x = 4 2. Solve -x2 + 2x + 8 < 0 and represent the
(x + 5) and (x – 3) solution set graphically.
4 + 5 = 9 and 4 – 3 = 1
(+, +)
x+5
Solution
x -3 -x2 + 2x + 8 < 0 (original equation)
x2 – 2x – 8 > 0 (standard form)
-5 3
x2 – 2x – 8 = 0 ( Solve by factorization)
2
Product of signs: (x + 2x) – (4x – 8) = 0
Interval one; x ( x + 2) – 4(x + 2) = 0
= (–) (–) = + (x + 2) (x – 4) = 0 (factors)
x = -2 or x = 4 (boundaries)
Interval two; x+2
= (+) (–) = - x -4

Interval three; ( , -2) -2 ( ,4) 4 ( , )


= (+) (+) = +
Test -3 in the first interval:
2
Now, the original problem is x + 2x – 15 < 0.
For the quadratic expression to be less than 0, a When x = -3
negative sign is required. The only interval in (x + 2) and (x – 4)
which the quadratic is negative is the second -3 + 2 = -1 and -3 – 4 = -7
interval; (-5, 3) (–, –)

Baffour Ba Series, Further Mathematics for Schools Page 230


Test 0 in the second interval: Alternatively;

When x = 0
(x + 2) and (x – 4)
-2 4
0 + 2 = 2 and 0 – 4 = -4
(+, –)
3. Solve using a sign graph of factors. Write
Test 5 in the third interval: your answer in interval notation and graph the
When x = 5 solution set 17x + 5 - 6x2.
(x + 2) and (x – 4)
5 + 2 = 7 and 5 – 4 = 1 Solution
(+, +) 17x + 5 - 6x2 (original equation)
6x2 + 17x + 5 0 (standard form)
x+2
x -4
6x2 + 17x + 5 = 0 (solve by factorization)
2
(6x + 2x) + (15x + 5) = 0
-2 4
2x(3x + 1) + 5(3x + 1) = 0
Product of signs: (2x + 5) (3x + 1) = 0
Interval one; (2x + 5) = 0 or (3x + 1) = 0
= (–) (–) = + 2x = -5 or 3x = -1
x= or x =
Interval two;
= (+) (–) = – 2x + 5
3x + 1
Interval three;
= (+) (+) = + , ( , ) ,

For x2 – 2x – 8 > 0, the first and third intervals Test -3 in the first interval:
give positive values. 2(-3) + 5 = -1 and 3(-3) + 1 = -8
( , -2) and (4, ) respectively. (–,–)

Solution set = {x : 4 x < -2} Test -2 in the second interval:


2(-2) + 5 = 1 and 3(-2) + 1 = -5
Interval notation: ( ,-2] ∪ [4, ) (+, –)

Graph Test in the third interval:


2(0) + 5 = 5 and 3(0) + 1 = 1
] [ (+, +)
-2 4

Baffour Ba Series, Further Mathematics for Schools Page 231


The Test Point Method
2x + 5
This method works for any quadratic equation
3x +1
that has real number solution, whether it factors
or not.

Steps
Product of signs: 1. Write the inequality in standard form.
Interval one; 2. Solve the quadratic equation, ax2 + bx + c =
= (–) (–) = + 0, to get the boundary points.
3. Use the boundary points found to mark off
Interval two; test intervals on the number line.
= (+) (–) = – 4. Test a point of each test interval in ax2 + bx
+ c = 0, to see which interval is part of the
Interval three; solution set.
= (+) (+) = +  If the quadratic expression is less than
or less than or equal to (< or ) 0,
For 6x2 + 17x + 5 0, we are looking for the
then the interest is on values that make
expression to be greater than or equal to 0,
the quadratic expression negative.
which requires a positive sign (or 0).
 If the quadratic expression is greater
than or greater than or equal to (> or
The only interval in which the quadratic is
) 0, then the interest is on values that
positive is the first and third intervals:
make the quadratic expression positive.
( , - ] and [- , ) respectively.
Solution set = {x : - x - } 5. Write the solution set and graph it.

Worked Examples
Interval notation: ( , - ] ∪ [- , )
1. Solve using the test point method, write your
answer in interval notation and graph the
Graph:
solution set: 5x2 + 2x 0

] [ Solution
5x2 + 2x 0 (original equation)
5x2 + 2x 0 (standard form)
Alternatively;
5x2 + 2x = 0 (solving the equation)
x(5x + 2) = 0
x = 0 or 5x + 2 = 0
x = 0 or 5x = -2
x = 0 or x = -

Baffour Ba Series, Further Mathematics for Schools Page 232


Boundary points are 0 and - Test 1 in the third interval;
= 5(1)2 + 2(1)
The boundary points create three intervals:
=5+2=7
(- ,- )( , 3) and ( 0 , ) on the number
line as shown below: Since 7 is positive and we are interested in
values that cause the expression to be less than
or equal to (negative or 0), [0, ] would not be
part of the solution.

The original problem is 5x2 + 2x 0, so the Interval notation [ , 0]


point of interest is the value (s) less than or
equal to zero.
[ ]
3
-
Test -1 in the first interval;
2. Solve using the test point method, write your
5x2 + 2x = 0
answer in interval notation and graph the
solution set of x2 – 6x > -2
When x = -1,
= 5(-1)2 + 2(-1)
Solution
=5–2=3
x2 – 6x > -2
x2 – 6x + 2 > 0
Since 3 is positive and we are interested in
values that cause the expression to be less than x2 – 6x + 2 = 0
or equal to (negative or 0), (- , - ], would not a = 1, b = - 6 and c = 2
be part of the solution. √
Substitute in x =

Test in the second interval; ( ) √( ) ( )( )


x= ( )
5x2 + 2x = 0 √
x=
When x = x=3 √

⇒5. / + 2. / Boundary points are x = 3 + √ and x = 3 – √


= =
3–√ 3 √
Since is negative and the point of interest is
The boundary points create three intervals on
in the values that cause the expression to be less the graph.
than or equal to (negative or 0), [- , ] would (- , 3 – √ ), (3 – √ , 3 + √ ) and (3 + √ , )
be part of the solution.

Baffour Ba Series, Further Mathematics for Schools Page 233


Test 0 in the first interval; than 0 (positive), (3 – √ , ) would be part of
x2 – 6x + 2 = 0 the solution.
= (0)2 – 6(0) + 2 Interval notation: (- , 3 – √ ) U (3 +√ , )
=2
Graph:
Since 2 is positive and we are interested in
values that cause the expression to be greater ) (
3–√ 3 √
than 0, (- , 3 – √ ), would be part of the
solution. Exercises 8.12
A. Find the solution set to each of the
Test 1 in the second interval; following inequalities
x2 – 6x + 2 = 0 1. x2 + 2x – 15 > 0 4. x2 – 2x – 3 < 0
= (1)2 – 6(1) + 2 2. 2x2 – 3x > 4 5. 2x2 – 3x < - 4
= -3
Since -3 is negative and we are interested in 3. <1 6. ≥0
values that cause the expression to be greater 7. (2x + 7) (5 – 11x) ≤ 0
than 0 (positive), (3 – √ , 3 + √ ) would not be 8. 8(x + 1) – 2 < 5(x – 6) + 7
part of the solution.
B. Solve and graph your answer
Test 6 in the third interval; 1. x2 – 6 > -5x 4. 6x2 – 5x + 1 0
x2 – 6x + 2 = 0 2. – x2 + 4 < 0 5. x2 + 12x + 32 0
= (6)2 – 6(6) + 2 3. x2 – x – 6 < 0 6. x2 + 2x – 15 > 0
=2
C. Solve and represent your answer on a
Since 2 is positive and we are interested in number line
values that cause the expression to be greater 1. x2 + 10 + 21 ≥ 0 4. x2 – 6x + 5 > 0
2. x2 + 4x > 12 5. 22 + 9x – x2 > 0
3. x2 + 2x ≤ 8 6. 15 + 8x + x2 < 0

Baffour Ba Series, Further Mathematics for Schools Page 234


9 COORDINATE GEOMETRY Baffour Ba Series

A Straight Line 2. If P is the point (5, -2) and Q is the point


When two points are joined by a straight edge, (-9, -2), find /PQ/
a straight line is formed.
Solution
Distance between Two Points P(5, -2) and Q(-9, -2),
Given A(x1, y1) and B(x2, y2), the distance
between A and B written as /AB/ is calculated /PQ/ = √(( )– ) (( ) ( ))
as shown in the diagram below: /PQ/2 = √ ( )
B(x1, y1) /PQ/ = √
(y2) /PQ/ = 14 units

Exercises 9.1
(y1) A. Find /AB/ to the nearest hundredth
A(x1, y1)
1. A(2, 5) B(-3, 3) 3. A(0, -11) B(8, 2)
(x1) 2. A(-8, -6) B(1, 9) 4. A(6, - 4) B(0, 4)
(x2)
By Pythagoras theorem: Application to Triangles
/AB/2 = (x2 – x1 )2 + (y2 – y1 )2 Determining the Type of Triangle Using the
/AB/2 = √( – ) ( ) Distance Formula
Given the vertices of a triangle as A, B and C,
the type of triangle is verified by finding the
Worked Examples
magnitude of AB, the magnitude of BC and the
1. What is the distance between the points
magnitude of AC. Thereafter, observe the
(-2, 7) and (5, -3).
following:
I. If the magnitudes of the three sides are equal,
Solution
then ∆ ABC is an equilateral triangle.
A (-2, 7) and B (5, -3).
II. If the magnitudes of two sides are equal,
⇒ x1 = -2, y1 = 7, x2 = 5, y2 = -3
then ∆ABC is an isosceles triangle.
/AB/2 = √( – ) ( ) III. If the magnitudes of the three sides are
unequal, then ∆ ABC is a scalene triangle.
/AB/2 = √( – ( )) ( ) IV. If the magnitudes of the three sides form a
Pythagorean triples, the three points form the
/AB/2 = √ ( ) vertices of a right – angled triangle.

/AB/2 = √ Worked Examples


/AB/ = √ 1. A, B and C are the points (3, 5), (7, 2) and

Baffour Ba Series, Further Mathematics for Schools Page 235


(3, -1) respectively. What type of triangle is ∆ P(-3, 4), Q(3, 4 ) and C(0, 9).
ABC?
/PQ/ = √( – ) ( – )
Solution /PQ/ = √( ) ( )
A(3, 5) B(7, 2) and C(3, -1)
/PQ/ = √ ( )
/AB/ √( – ) ( – ) /PQ/ = √ units

/AB/ √( ) ( )
/QR/ = √( – ) ( – )
/AB/ = √ ( )
/QR/ = √( ) ( )
/AB/ =√
/QR/ = √ ( )
/AB/ = √
/QR/ = √
/AB/ = 5 units
/QR/ = √ units

/AC/ √( – ) ( – ) /PR/ = √( – ) ( – )
/AC/ = √( ) ( ) /PR/ = √( ) ( )
/AC/ = √ ( ) /PR/ = √ ( )
/AC/ = √ /PR/ = √
/AC/ = √ /PR/ = √ units
/AC/ = units | |=| | | | = 6 units
Therefore ∆ PQR is an equilateral triangle
/BC/ = √( – ) ( – )
3. What type of triangle is ∆ UVW with vertices
/BC/ = √( ) ( ) U(-2, 2), V(3, 2) and W(- 4, 5)?
/BC/= √( ) ( )
/BC/ = √ Solution
/BC/ = √ U(-2, 2), V(3, 2) and W(- 4, 5)
/BC/ = units
/UV/ = √( – ) ( – )
Since | | | | = 5, triangle ABC is an /UV/ = √( ) ( )
isosceles triangle. /UV/ = √ ( )
/UV/ = √ units
2. The triangle PQR has vertices at P(-3, 4),
Q(3, 4 ) and R(0, 9). Show that ∆ PQR is no
other triangle than an equilateral. /VW/ = √( – ) ( – )
/VW/ = √( ) ( )
Solution

Baffour Ba Series, Further Mathematics for Schools Page 236


/VW/ = √( ) ( ) √(
| | – ) ( – )
/VW/ = √
/VW/ = √ 8 units √( 8) ( )
√ 8 ( )

/UW/ =√( – ) ( – )

/UW/ = √( ) ( ) Check to see if the lengths, √ , √ and


/UW/ √( ) ( ) √ form Pythagoras triples:
/UW/ = √ | | √ , | |=√ , | |=√
/UW/ = √ units
| | | | | |. Therefore, ∆UVW is a By Pythagoras theorem
scalene triangle. (√ ) (√ ) = (√ )
100 + 225 = 325
Three Points Forming a Right Triangle 325 = 325
To determine whether three given points form The points form the vertices of a right triangle.
the vertices of a right triangle;
I. Find the distance between each pair of points Exercises 9.2
using the magnitude formula. Three points that form the vertices of a
II. If the distances between the points form triangle are given below. Show whether any
Pythagorean triples, the three points form the of them forms a right triangle;
vertices of a right triangle. 1. (5, 2), (0, -3), (4, - 4)
2. (7, 0), (-1, 0), (7, 4)
Worked Examples 3. (- 4, 3), (-7, -1), (3, -2)
Determine whether the points (– 8, 1), (-2, 9) 4. (-3, 2) , (-1, 5), (- 6, 4)
and (10, 0) form the vertices of a right triangle
B. 1. Show that the triangle whose vertices are
Solution (1, 1), (3, 2) and (2, -1) is isosceles.
Name the points as A(-8, 1), B (-2, 9) and C(10, 0)
| | √( – ) ( – ) 2. P, Q and R are the points (5, -3), (-6, 1) and
(1, 8) respectively. Show that P, Q, R is
√( 8) ( )
isosceles and find the coordinate of the mid –
√ 8 point of the base .

3. The triangle ABC has vertices at the points
| | √( – ) ( – ) A(1, 5), B(4, -1) and C(-2, -4).
√( ) ( ) i. Show that triangle ABC is right – angled.
√ ( ) ii. Find the area of triangle ABC


Baffour Ba Series, Further Mathematics for Schools Page 237
4. P, Q and R are the points (5, -3), (-6, 1) and Worked Examples
(1, 8) respectively. Show that the triangle PQR 1. Find the area of the following triangles
is an isosceles triangle and find the mid - point whose vertices are (0, 0), (5, 2) , (3, 4)
of the base.
Solution
5. The three points of a triangle are at A(1,1), (0, 0), (5, 2) and (3, 4)
B(4, 5) and C(5, -2). Find the lengths of the ⇒ x1 = 5, y1 = 2 , x2 = 3 and y2 = 4
side of the triangle and show that it is isosceles.
What else can you say about the triangle? A= | |
A = |( )( ) ( )( )|
6. The three points of a triangle are at A(1, 1),
B(4, 5) and C (1, 6). What type of triangle is ∆ A= | |
ABC? A= | |
A = 7 square units
Area of a Triangle
Case 1 2. Find the area of the triangle with vertices
The area of a triangle with vertices (0, 0), (x1, (-2, 1), (- 4, 9) and (3, -6)
y1) and (x2, y2) is given by the formula;
A= | | Solution
(-2, 1), (- 4, 9) and (3, -6)
Note: Now, translate the point (-2, 1) to (0, 0)
The modulus symbol, / / , is include to make (-2 + 2, 1 – 1) = (0, 0)
sure your answer is positive. Therefore, if the (- 4 + 2, 9 – 1) = (-2, 8)
formula gives a negative answer, simply ignore (3 + 2, - 6 – 1) = (5, -7)
the negative sign.
New points after translation;
Case 2 (0, 0), (-2, 8) and (5, -7)
If none of the vertices is at the origin, simply ⇒ x1 = -2, y1 = 8 , x2 = 5 and y2 = -7
select one of the vertices and move it to the
point (0, 0) by translation. Then apply the same A= | |
translation to the other two vertices to obtain
A = |( )( ) ( )(8)|
(x1, y1) and (x2, y2)
A= | |
Case 3 A= | |
To find the area of a quadrilateral (4 – sided
figure), divide it into two triangles. The A= | |
diagonal of a parallelogram bisects its area. A = 13 square units

Baffour Ba Series, Further Mathematics for Schools Page 238


3. Find the area of the parallelogram whose 5. (- 4, -8), (4, -5), (3, -2)
vertices are (0, 0), (1, 3) , (5, 5), (4, 2) A = 10 6. (-1, -4), (2, -1) , (-2, 3)

Solution B. Find the area of the parallelogram whose


Let the vertices of the parallelogram be A(0, 0), vertices are :
B(1, 3) , C(5, 5), and D(4, 2) 1. (0, 0), (1, 3) , (5, 5), (4, 2)
2. (-2, 4), (2, 4) , (2, 7), (-2, 7)
D(4, 2) C(5, 5)
3. (5, 1), (3, 1) , (5, 4), (7, 4)
4. (-1, 3), (0, 2) , (5, 4), (4, 5)

C. Find the area of the quadrilateral whose


A(0, 0) B(1, 3)
vertices are :
Area of ∆ACD : 1. (1, 1), (1, 2) , (9, 3), (6, 1)
A= | | 2. (2, - 4), (-1, - 4) , (-2, 2), (5, 5)
3. (5, -6), (5, - 4) , (0, 1), (-2, -9)
A = |( )( ) ( )( )| 4. (-2, 2), (-5, -6) , (8, - 4), (9, 0)
A= | |
Finding an Unknown Coordinate given the
A= | | = 5 square units
Area of a Triangle
1. Identify the three given points and translate
Area of ∆ABD : one of them to (0, 0) not forgetting the effects
A= | | on the other two points. Here, it is always good
A = |( )( ) ( )( )| to choose one of the known coordinates.

A= | | 2. Use the fact that Area of ∆ ;


A= | | A= | |
A = 5 square units
3. Substitute the known values and solve to
Area of ABCD : obtain the unknown value.
= Area of ∆ACD + Area of ∆ABD
= 5 + 5 = 10 square units Worked Examples
1. A(3k, 5), B(-2, 3) and C(-k, 4) are vertices of
Exercises 9.3 the triangle ABC. If the area of triangle ABC is
A. Find the area of the following triangles 4 square units, find the values of k.
whose vertices are :
1. (0, 0), (5, - 2) , (- 3, 4) Solution
2. (10, 8), (0, 0) , (3, 5) A(3k, 5), B(-2, 3) and C(-k, 4)
3. (1, 5), (-5, -3) , (4, 1)
4. (7, -1), (-5, 6), (3, -2) Translate B(-2, 3) to (0, 0)

Baffour Ba Series, Further Mathematics for Schools Page 239


B(-2 + 2, 3 – 3) = B(0, 0) Challenge Problems
A(3k + 2, 5 – 3) = A(3k + 2, 2) 1. P(h, k), Q(1, -2) and R (8, -3) are the vertices
C(-k + 2, 4 – 3) = B(- k + 2, 1) of the triangle PQR. Calculate the coordinates
⇒x1 = 3k + 2, y1 = 2, x2 = -k + 2, y2 = 1 of P if P is a point on the x – axis and area of
triangle PQR is 8 square units.
Area of ∆ ABC;
A= | | 2. A(2, -1) B(1, -3) , C(-1, 1) are three points.
Show that the area of triangle ABC is 4.
A = |( )( ) ( )( )|
K1( 1, 0) and K2(0, 2) are two points such
A= | | that : area of ∆K1AC = area of K2AC = 4.
Find 1 and 2.
A= | |
A= | | Mid-point of a Line
Given A(x1, y1) and B(x2, y2), the coordinates of
But area of ∆ABC = 4 the midpoint of the line joining A and B is
⇒ | |=4 calculated as:
/5k – 2/ = 8 M= . , /
5k – 2 = 8
5k – 2 = 8 or 5k – 2 = -8 Worked Examples
5k = 10 or 5k = - 6 1. Find the midpoint of the line joining the
k = 2 or k = points A(1, -3) and B(4, 5).

Solution
Exercises 9.4
A(1, -3) and B(4, 5)
1. A(-1, -3), B(4, 1) and C(3, k) are vertices of
⇒ x1 = 1, y1 = -3. x2 = 4, y2 = 5
triangle ABC. If the area of triangle ABC is 12
square units, find the two values of k. Substitute in M = . , /
M=. , /
2. The area of the triangle with vertices (0, 0),
(5t, 3t) and (t, 2t) is 14 square units. Find the M = . , / = (2.5, 1)
two possible values of t R.
2. Find the coordinate of the mid- point of the
3. The area of triangle with vertices P(-k, 1), line segment determine by the pints (-2, 4) and
Q(0, 3) and R(2k, -1) is 4 square units. Find the (6, -12)
two possible values of k.
Solution
4. A(1, 3) , B(-3, 1), C(5, -2), P(-1, 1), Q(9, 7), Let A(-2, 4) , B(6, -12) and M(x, y)
R(1, k), k > 0 are six points. If the area of ⇒ x1 = -2, y1 = 4 x2 = 6, y2 = -12
triangle ABC = area of triangle PQR, find the
Substitute in . , /
value of k.
Baffour Ba Series, Further Mathematics for Schools Page 240
( ) III. Form two separate equations and solve for
M=. , /
the values of x and y to obtain the coordinates
M=. , / of the unknown end point.

M = (2, - 4)
Worked Examples
1. M(4, 7) is the midpoint of the line joining
Exercises 9.5 A(x, y) and B (6, 11). Find the coordinates of
A. Find the coordinates of the mid-point of the point represented by x and y
the line segment ̅̅̅̅
1. A(3, 0) B(0, 0) 4. A(3, -1) B(-5, -7) Solution
2. A(4, 5) B(7, 9)
M(4, 7), A(x, y ) and B (6, 11)
3. A(-2, 6) B(8, - 4) 5. A(4, 5) B(3, - 4) Midpoint of AB = M
. , / = (4, 7)
B. 1. A diameter of a circle has extreme points
(7, 9) and (-1, 3). What would be the coordinate = 4……………(1)
of the center?
= 7…………..(2)

2. Find the midpoint of the line joining the


points (5, 8) and (-5, - 6). From eqn (1)
x+6=4×2
x+6=8
3. The endpoints of the line AB are . , / and
x=8–6
. , /. Determine the mid-point of line AB. x=2

4. Show that the line segment joining the points From eqn (2);
(8, 3) (-2, 7) and the line segment joining (11, y + 11 = 7 × 2
-2), (5, 12) bisect each other. y + 11 = 14
y = 14 – 11 = 3
x = 2 and y = 3
Finding an Unknown Endpoint given the
midpoint and an Endpoint Therefore, the coordinates of A is (2, 3)
I. Identify the given end point, the given
midpoint and the unknown end point 2. M(3, 8) is the midpoint of the line AB. A has
II. Represent the unknown end point by the the coordinate (-2, 3). Find the coordinates of
ordered pair (x, y). B.
II. Add the respective x and y components of
the end points divide each sum by 2 and equate Solution
to the respective x and y component of the Let the coordinates of B be (x, y)
midpoint. . , / = (3, 8)

Baffour Ba Series, Further Mathematics for Schools Page 241


⇒ = 3 and =8 Internal divisor
Now, P=. , /
=3
-2 + x = 3 × 2 External Case P(x, y)
n
-2 + x = 6 m
x=6+2 B(x2, y2)
x=8

Now, A(x1, y1)


=8
The coordinates of P which divides the line
3+y=8×2 segment A(x1, y1) and B(x2, y2) externally in the
3 + y = 16 ratio m : n is given by:
y = 16 – 3
y = 13 External divisor
Coordinates of B = (8, 13)
P=. , /

Exercises 9.6
Method 2
1. In the standard x – y coordinate plane, point The coordinates of P can also be found using
M with coordinates (5, 4) is the mid-point of similar triangles.
AB, and B has coordinates (7, 3). What are the
coordinates of A?
Worked Examples
1. Find the coordinates of the point that divides
2. For a line segment DE, one end point is E(6, the lines segment (- 4, 3) and (6, -12) in the
5) and the midpoint M(4, 2). Find the ratio 3: 2,
coordinates of the other point, D. i. Internally, ii. externally.

Division of a Line Segment in a given Ratio Solution


Internal case n Method 1
B(x2, y2) i. Internal divisor;
3
m 2
P(x, y)

(-4, 3) (x, y) (6, -12)


A(x1, y1)

The coordinates of P which divides the line


(x, y) =. , /
segment A(x1, y1) and B(x2, y2) internally in the
ratio m : n is given by: ( ) ( ) ( ) ( )
(x, y) =. , /

Baffour Ba Series, Further Mathematics for Schools Page 242


(x, y) = . , / (x, y) = (2, - 6)

(x, y) = (2, - 6) ii. External division;


3
ii. External divisor;
2
3
2 (-4, 3) (6, -12) (x, y)

(-4, 3) (6, -12) (x, y) =


⇒6 – x = 2(- 4 – 6)
(x, y) =. , / 6 – x = -8 – 12
6 – x = - 20
( ) ( ) ( ) ( )
(x, y) =. , / - x = -20 – 6
- x = -26
(x, y) =. , / x = 26
(x, y) = (26, - 42)
=
Method 2 ⇒-12 – y = 2(3 +12)
i. Internal division -12 – y = 2(15)
3 -12 – y = 30
2
- y = 30 + 12
- y = 42
(-4, 3) (x, y) (6, -12) y = - 42
(x, y) = (26, - 42)
=
2. A(-1, 3) and B(6, -11) are two points. P is a
⇒3(x – 6) = 2(- 4 – x)
point on ̅̅̅̅ such that /AP/ : /PB/ = 3 : 4. Find
3x – 18 = -8 – 2x
the coordinates of P.
3x + 2x = -8 + 18
5x = 10
Solution 4
x=2 3

=
A(-1, 3) P(x, y) B(6, -11)
⇒3(y + 12) = 2(3 – y)
3y + 36 = 6 – 2y Internal division in the ratio 3 : 4
3y + 2y = -8 + 18 ⇒m = 3 and n = 4
5y = -30
y = -6 P (x, y) = . , /

Baffour Ba Series, Further Mathematics for Schools Page 243


( ) ( ) ( ) ( )
P(x, y) = . , / find the point P which divides ⃗⃗⃗⃗⃗ internally in
the ratio 3 : 2.
P(x, y) = . , /
P(x, y) = (2, -3) 6. Find the coordinates of the points which
divides the line segment joining the points
3. P(-2, 5) and Q(1, 3) are two points. The (2, -5) and (-3, -2) externally in the ratio 4 : 3.
point R is on ̅̅̅̅ produced, such that /PQ/ :
7. P is a point on the line segment AB such that
/QR/ = 3 : 2. Find the coordinates of R.
⃗⃗⃗⃗⃗ = 3⃗⃗⃗⃗⃗ , if the coordinates of A and B are
(3, - 4) and (-5, 2) respectively, find the
Solution
coordinates of P.
External division in the ratio 3: 2
⇒m = 1 and n = 2
8. The line segment CD is produced to Q such
R (x, y) = . , /
that ⃗⃗⃗⃗⃗⃗⃗⃗ = 5⃗⃗⃗⃗⃗⃗ . If the coordinates of C and D
( ) ( ) ( ) ( )
R(x, y) = . , / are (4, 7) and (-2, 4) respectively, find the
coordinates of Q.
P(x, y) = . , /
R(x, y) = (7, -1) 9. i. If the point (6, 3) divides the segment of a
line from P(- 4, 10) and Q(x, y) in the ratio 2 :
Exercises 9.8 5, find the coordinates of (x, y) of Q.
1. Find the coordinates of the point that divides ii. What are the coordinates of the midpoint
the line segment A(-2, 5) and B(1, 3) in the ̅̅̅̅?
ratio 3 : 2,
i. internally, ii. externally. Challenge Problems
1. Find the coordinates of the point that divides
2. A(3, 2) and B(18, 12) are two points. AB is the segment (2h, 3k) to (12h, -17k) in the ratio
produced to C such that /AC/ : /BC/ = 7 : 2. 3 : 2;
Find the coordinates of C. i. internally, ii. externally.

3. A(4, 5) and B(7, -1) are two given points and 2. Copy and completed the following table
the point C divides the line segment AB by dividing line segment AB in the given
externally in the ratio 4 : 3. Find the ratio.
coordinates of C.
A B Ratio I E
4. A point divides internally the line segment 1. (3, 1) (3, -7) 2:1
joining the points (8, 9) and (-7, 4) in the ratio 2. (-5, 3) (10, -7) 3:2
2 : 3. Find the coordinate of the point. 3. (-4, 3) (12, -6) 1:3
4. (1, -2) (-20, 12) 4:3
5. (-3, -7) (9, -17) 5:1
5. If A and B are the points (1, 5) and (-4, 7), 6. (-2, 5) (14, -19) 5:3

Baffour Ba Series, Further Mathematics for Schools Page 244


3. Find the coordinates of the point which Solution
3
divides the line segment joining the points (x + 1
y, x – y) and (x – y, x + y) internally in the ratio
x : y.
A(5, -1) P(-1, 8) B(x, y)

4. Find the coordinates of the points which Internal division in the ratio 3 : 1
divides the line segment joining the point (a, b) ⇒ m = 3 and n = 1
and (b, a) externally in the ratio (a – b) : (a + b) P (-1, 8) = .
( ) ( )
,
( ) ( )
/

Finding an Unknown Endpoint P (-1, 8) = . , /


Given the point of division, the type of
⇒-1 = and 8 =
division, the ratio of division and an end point,
the other endpoint (unknown one) is calculated
as follows: Solving for x;
1. Identify the coordinates of the endpoints of -1 × 4 = 3x + 5
the line segment as A(x1, y1) and B(x2, y2) - 4 = 3x + 5
2. Observe the type of division, whether - 4 – 5 = 3x
internally or externally in the ratio m : n. -9 = 3x
3. Note the point of division as P(x, y) x = -3

4. Substitute P (x, y), A(x1, y1), B(x2, y2) and m Solving for y;
and n in : 8 × 4 = 3y – 1
32 = 3y – 1
 P (x, y) = . , / when
32 + 1 = 3y
division is internal. 33 = 3y
 P (x, y) = . , /, when y = 11
division is external. ⇒B(x, y) = (-3, 11)

5. Form two equations involving the x and y 2. A and B are two points. AB is produced to C
coordinates of the unknown end point. such that /AC/ : /BC/ = 7 : 2. Find the
coordinates of A given that the coordinates of B
6. Solve for the respective values of x and y. and C are (18, 12) and (24, 16) respectively.

Worked Examples Solution


1. The point P(-1, 8) divides the line segment 7
AB internally, such that /AP/ : /PB/ = 3 : 1. If 2
the coordinates of A are (5, -1), find the
coordinates of B.
A(x, y) B(18, 12) C(24, 16)

Baffour Ba Series, Further Mathematics for Schools Page 245


External division of AB in the ratio 7 : 2 that is nearer to (-2, 3).
⇒ m = 7 and n = 2
C (x, y) = . , /, 4. Find the length of the median of the triangle
( ) ( ) ( ) ( )
whose vertices are (2, - 4). (6, 2) and (- 4, 2).
C (24, 16) = . , /,
(24, 16) = . , / Finding the ratio
1. Identify the line segment AB and the point
of division , P(x, y) on the line.
⇒24 = and 16 =
II. Substitute in the formula ;
x=. , / OR y = . /
24 = ( Solving for x)
III. Workout to obtain the ratio m : n
5 × 24 = 126 – 2x
120 = 126 – 2x Note:
120 – 126 = -2x 1. A point divides a given line segment
- 6 = -2x internally or externally in a definite ratio
x=3 accordingly, as the value of m : n is positive or
negative respectively.
16 = (Solving for y)
5 × 16 = 84 – 2y 2. The same ratio is obtained using the
80 = 84 – 2y conditions of either x or y.
80 – 84 = -2y
- 4 = -2y Worked examples
y=2 1. Find the ratio in which the point (-11, 16)
A(x, y) = A(3, 2) divides the line segment joining the points
(-1, 2) and (4, -5).
Exercises 9.9
1. If the point (0, 4) divides the line segment Solution
joining the points (- 4, 10) and (2, 1) internally Let the points be A(-1, 2) and B(4, -5) and the
in a definite ratio, find the coordinates of the line segment AB is divided in the ratio m : n at
point which divides the segment externally in (-11, 16)
( )
the same ratio. -11 = ,
-11(m + n) = 4m – n
2. The straight line joining the points (2, - 2) -11m – 11n = 4m – n
and (4, 6) is extended each way a distance -11m – 4m = 11n – n
equal to half its own length. Determine the -15m = 10n
coordinates of the terminal point.
= =-
3. Find the coordinates of the point of trisection Therefore, the point (-11, 16) divides the line
of the line segment joining (-2, 3) and (3, -1) segment AB externally in the ratio 3: 2

Baffour Ba Series, Further Mathematics for Schools Page 246


2. Find the ratio in which the line segment that 2CQ = 5DQ; if the coordinates of C and D
joining the points (5, - 4) and (2, 3) is divided are (4, 7) and (-2, 4) respectively, find the
by the x - axis. coordinates of Q.

Solution Gradient of a Straight Line


Let the points be A(5, - 4) and B(2, 3) and the x The gradient of a straight line, also known as
– axis intersects the line segment AB at P such the slope of a straight line is a measure of the
that ̅̅̅̅ : ̅̅̅̅ = m : n steepness of the line. Steepness means, how an
( )
The coordinates of P = . , / angle is falling or rising. The gradient is
usually represented by the variable m.
Since the point P lies on the x – axis, the y –
coordinate of P is zero.
( )
The gradient of a line can be determined by
⇒ =0 from:
3m – 4n = 0 1. any two given points,
3m = 4n 2. any linear relation,
= 3. any linear graph.
Therefore, the axis divides the line segment
Gradient of a Line from Two Points
internally in the ratio 4 : 3
If two points are given as A(x1, y1) and B(x2,
Exercises 9.10 y2), then the gradient of the line AB, is
1. Find the ratio in which the point (1, 2) determine by the formula;
– –
divides the line segment joining the points m= or m = ,
– –
(-3, 8) and (7, -7) where m is the gradient
2. Find the ratio in which the point (5, -20) Worked Examples
divides the line segment joining the points 1. A is the point (4, -2) and B is the point (2, 1).
(4, 7) and (1, -2) Calculate the gradient of the straight line
joining the points A and B.
3. Determine the ratio in which the line 3x + 4y
– 9 = 0 divides the line segment joining the Solution
points A(1, 3) and B(2, 7) A(4, 2) and B (2, 1)
Let x1 = 4, y1 = -2, x2 = 2 and y2 = 1
Challenge Problems Gradient of AB;
1. In what ratio the segment joining the points – ( )
(3, 4) and (2, -3) is divided by the x – axis?. m= = =

Also find the ratio in which it is divided by the
y – axis. 2. The gradient of a line passing through the
points P(6, 7) and Q(x, 8) is . Find the value of x
2. The line segment ̅̅̅̅ is produced to Q such .

Baffour Ba Series, Further Mathematics for Schools Page 247


Solution ax + by + c = 0, then the gradient m = –
P(6, 7), Q(x, 8), m =
In words, Slope, m = –
Let x1 = 6, y1= 7, x2 = x and y2 = 8

Gradient of PQ; Note:


– 1. when using this method, make sure that
m= = = every term is on the left hand side of the

equation
= 2. The equation of a line through the origin,
with gradient m, is y = mx
=
Worked Examples
⇒x – 6 = 3 1. Find the gradient of line 4x + 2y – 5 = 0.
x=3+6
x=9 Solution
Method 1
Exercises 9.11 4x + 2y – 5 = 0 compared to ax + by + c = 0
1. Find x if the line through (-2, 4) and a = 4 and b = 2.
(x, 6) has a slope of . By substitution,
m = – = – = –2
2. Find y if the line through (5, 2) and (-3, y)
has a slope of . Method 2
Make y the subject of the equation
Gradient of a Line given its Equation 4x + 2y – 5 = 0
To find the gradient of a line given its equation: 2y = – 4x + 5
y=
Method 1
y= −
Get y on its own such that the equation will be
of the form, y = mx + c, where the gradient is y = –2x − ,
m or the coefficient of x. The gradient can also The co-efficient of x = −2, ⇒m = −2.
be identified simply as the number in front of x.
y = mx + c 2. Find the gradient of the line7x + 4y + 2 = 0,
and its intercepts on the x – axis and y – axis.
y = (slope)x + (where the line cuts the y – axis)
Solution
Express 7x + 4y + 2 = 0 in the form:
Method 2 y = mx + c by making y the subject,
If the equation of the line is in the form: 4y = – 7x – 2

Baffour Ba Series, Further Mathematics for Schools Page 248


y=– x– shown in the diagram above.
III. Calculate the gradient by,
y = – x – compared to y = mx + c –
m= = =
The gradient , m = – and the intercept on the y –

– axis, c = – Note:
If the line rises from left to right, the gradient is
To find the intercept on the x – axis, substitute positive.
y = 0 in 7x + 4y + 2 = 0 and solve for x.
7x + 4(0) + 2 = 0
Positive gradient
7x + 2 = 0
7x = – 2
x=–
If the line falls from left to right, the gradient is
The intercept on the x – axis is –
negative.

Gradient of a Line from a Linear Graph


Negative gradient
Study the diagram below carefully;
y Summary

B (x2,y2) zero gradient


y2
y2−y1 +ve gradient -ve gradient
A(x1,y1 )
y1 x2−x1

x
x1 x2
Worked Examples
1. Using a scale of 2cm to 1 unit on both axes,
To find the gradient of the line AB; plot the points A (0, −2) and B (3, 5) on a graph
I. Choose any two points on the line AB. sheet. Join the points with a ruler to form a
II. Draw a triangle to join the two points as straight line and find the gradient of the line.

Baffour Ba Series, Further Mathematics for Schools Page 249


Solution
y
6
Scale: 2cm : 1 unit on both axes
5 B (3, 5)

4
3
2

5 4 3 2 1 1 2 3 4 5x
-1

A(0, -2) - 2
-3
-4
-5
-6

Gradient (AB) = = = . Therefore, m = 1

Collinear Points Solution


Three or more points are said to be collinear Name the points as A(0, 0), B(3, 5) and C(21,
when they lie on the same straight line. Two or 35)
more points that lie on the same straight line Gradient of AB =

= =
have the same gradient. Therefore, to show –

whether or not, two or more points are Gradient of BC =



= = =
collinear: –

I. Find the gradient of the lines The gradient of AB = the gradient of BC


II. If the gradients are equal, conclude that they  A(0, 0), B(3, 5) and C(21, 35) are collinear.
are collinear, if not, conclude that they are not
collinear. 2. The points (2, -3), (3, -1) and (4, k) lie on a
straight line. Find k
Worked Examples
1. Show that the three points (0, 0), (3, 5) and Solution
(21, 35) are collinear. Name the points as A(2, -3), B(3, -1) and C(4,
k). If A, B and C are collinear, then gradient of

Baffour Ba Series, Further Mathematics for Schools Page 250


AB is equal to gradient of BC 1. (2, -3), (3, 1) (5, 9)
– ( ) ( ) 2. (- 3, 4), (1, 2) (7, - 1)
Gradient of AB = = =
– 3. (- 3, 1), (1, 2) (9, 4)
– ( )
Gradient of BC = = = 4. (1, 2), (0, - 1) (- 2, - 1)

5. (6, 1), (3, 3), (- 3, 7)
=
2=k+1 B. 1. The points (-1, m), (1, 2m) and (m, 9) are
2–1=k collinear. Find the possible values of m.
k=1
2. The points (-2, t), (t – 2 , t + 6) and (t + 2, 3t)
3. The point (2, a) , (-2, 2) and (3a, 9) are are collinear. Find the possible values of t.
collinear. Find the possible values of a.
3. The points (- 4, -5), . , / and (3, p + 2) lie
Solution
Let the points be A(2, a) , B(-2, 2) and C(3a, on a straight line. Find the value of p.
9). If the three points are collinear, then;
Equation of a Straight Line
Gradient of line AB = Gradient of line BC The equation of a straight line can be found
Gradient of line AB; when given any of the following:
1. two points on the line.
= =
2. the gradient of the line and a point on the
line.
Gradient of line BC; 3. the gradient of the line and the intercept on
= ( )
= the y – axis.

= The general form of the equation of a line is:


ax + bx + c = 0. However, the intercept form:
=
y = mx + c is equally acceptable.
(2 – a) (3a + 2) = (7) (- 4)
6a + 4 – 3a 2 – 2a = -28 A. Equation of a Line from two given Points
-3a2 – 2a + 6a + 4 + 28 = 0 Given A(x1, y1) and B(x2, y2) the equation of the
-3a2 + 4a + 32 = 0 line passing through A and B is determined as
3a2 – 4a – 32 = 0 (Solving by factorization) follows:
(a – 4) (3a + 8) = 0 1. Find the gradient of the line AB, using the
a = 4 or a = relation m =
2. Substitute the value of m and either (x1, y1) or
Exercises 9.7 (x2, y2) in the equations;
A. Show that the three given points in each y – y1 = m(x – x1) or y – y2 = m(x – x2)
case are collinear. respectively.

Baffour Ba Series, Further Mathematics for Schools Page 251


3. Simplify y – y1 = m(x – x1) or y – y2 = m(x – mid-point M of OC is joined to the point A(0,
x2) to obtain the equation of the line in the form 2 ). Find the equation of the line AM.
ax + by + c = o or y = mx + c.
Solution
Worked Examples 3x – 5y = 8………………….(1)
1. Find the equation of a line passing through x + 2y = 10………………….(2)
the points A(1, 5) and B(-3, 8).
eqn (2) × 3
Solution 3x + 6y = 30………………….(3)
Method 1
A(1, 5) and B(-3, 8). eqn (3) – eqn (1);
⇒x1 = 1, y1 = 5 and x2 = -3, y2 = 8 11x = 22
x=2
m= = =
Put x = 2 in eqn (1);
Put m = and x1 = 1, y1 = 5 in 3(2) – 5y = 8
y – y1 = m(x – x1) 6 – 5y = 8
5y = 6 – 8
⇒y – 5 = (x – 1)
5y = -2
4(y – 5) = -3(x – 1)
y=
4y – 20 = -3x + 3
4y + 3x – 20 – 3 = 0
3x + 4y – 23 = 0 C. , / and O( , )
Mid point (M) of OC;
Method 2 ⁄
A(1, 5) and B(-3, 8). M= ( , *= . , /
Let P(x, y) be a point on the line
⇒MAB = MBP
A(0, 2 ) and M . , /
=
A . , / and M . , /
4(y – 8) = -3(x + 3)
4y – 32 = -3x – 9
4y + 3x – 32 + 9 = 0 Gradient of AM;
⁄ ⁄
3x + 4y – 23 = 0 M=
The equation of the line is 3x + 4y – 23 = 0 ⁄ ⁄ ⁄
M= = =
2. Solve the simultaneous equations;
3x – 5y = 8, and x + 2y = 10. C is the point of Equation of AM;
intersection of the straight lines whose At C . , /;
equations these are, and O is the origin. The

Baffour Ba Series, Further Mathematics for Schools Page 252


y – = ( x – 2) 5(y + 1) = -2(x – 3)
5y + 5 = -2x + 6
y+ = (x – 2)
5y + 2x + 5 – 6 = 0
10 . / = -27(x – 2) 2x + 5y – 1 = 0
The equation of the line is 2x + 5y – 1 = 0
10 y + = - 27 x + 2
50 y + 20 = 5(-27x + 2) Note
50 y + 20 = - 135x + 10 You can change the subject of 2x + 5y – 1 = 0
135x + 50y + 20 – 10 = 0 to y to obtain the intercept form of the equation
135x + 50y + 10 = 0 if required.
27x + 10y + 2 = 0
Method 2
Exercises 9.12
m= , (x, y) = (3 , -1)
Find the equation of the line passing through
the following pair of points. Substitute in y = mx + c
1. (4, 6) and (8, 26) 3. (-2, 3) and (2, -5) ⇒-1 = (3) + c
2. (3, 4) and (5, 4) 4. (1, 1) and (4, -8) (-1)(5) = -2(3) + 5c
-5 = - 6 + 5c
B. Equation of a line given the gradient and a -5 + 6 = 5c
point on the line. 1 = 5c
Given the gradient and a point on the line, : c= ,
1. Identify the gradient of the line as m and the
point through which it passes as (x1, y1).
Substitute m = - and c = in y = mx + c
2. Substitute the value of m and (x1, y1) in
y – y1 = m(x – x1) . y= x+ ,
3. Simplify y – y1 = m(x – x1) to obtain the
The equation of the line is y = x+ .
equation of the line in the form:
ax + by + c = 0 or y = mx + c.
2. Find the equation of the straight line through
the point P(-1, 3) with a slope 3.
Worked Examples
1. Find the equation of a straight line with Solution
gradient , which passes through the point (3, -1) m = 3, (x, y) = (-1, 3)
Substitute in y = mx + c
Solution ⇒3 = 3 (-1) + c
m= , (x1, y1) = (3 , -1) 3 = -3 + c
Substitute in y – y1 = m(x – x1) 3+3=c
6=c
y – (-1) = (x – 3)
y+1= (x – 3) Substitute m = 3 and c = 6 in y = mx + c

Baffour Ba Series, Further Mathematics for Schools Page 253


y = 3 x + 6, The equation of the line is 2y – 5x + 8 = 0
The equation of the line is y = 3 x + 6
Exercises 9.14
Exercises 9.13 A. Find the equations of the lines:
Find the equation of the lines: 1. gradient 5, y – intercept 3
1. gradient 3, passing through (1, 4) 2. gradient -2, y – intercept -1
2. gradient -2, passing through (2, 0) 3. gradient 3, passing through the origin
3. gradient , passing through (5, -1) 4. gradient , y – intercept ,
4. gradient -1, passing through (1, -1) 5. gradient , y – intercept (0, 1)

C. The equation of a line given the gradient B. Determine the gradient and y – intercept
and the intercept on the y – axis for each of the staright lines in the table
Given the Gradient and the intercept on the y – below.
axis the equation of the line is found as follows:
1. Identify the gradient as m and the intercept Equation Gradient y - intercept
on the y – axis as c(when x = 0) y = 3x + 2
2. Substitute the values of m and c in y = mx + y = 5x – 2
c to obtain the equation of the line. y = -2x + 4
y= x-
Worked Examples 2 y – 10x = 8
Find the equation of a line with gradient , x+y+1=0
y = 2x
which passes cuts the y – axis at the point
y = - 4.
Parallel Lines
Two or more lines are said to be parallel if they
Solution cannot converge at a point.
m = and c = - 4
Substitute in y = mx + c Consider the diagram below:
⇒y = x + (- 4) y
y= x–4
The equation of the line is y = x – 4 θ θ x

Alternatively,
Express y = x – 4 in the form ax + by + c = 0 Line AB is said to be parallel to line CD
⇒ 2y = 5x – 2(4) because they make equal corresponding angles
2y = 5x – 8 with the x – axis. Therefore ⃗⃗⃗⃗⃗ and ⃗⃗⃗⃗⃗ are
2y – 5x + 8 = 0 said to have equal gradients.

Baffour Ba Series, Further Mathematics for Schools Page 254


In general, if two or more lines have the same = =
or equal gradients, they are said to be parallel.
Therefore, is parallel to
That is , if is parallel to then =

To prove whether or not two lines are parallel, 3. Show that the two lines 3x + 5y – 8 = 0 and
do the following: 5x – 3y – 11 = 0 are perpendicular.
1. Find the slope of each line
2. a. If the slopes are the same, the lines Solution
Let L1 = 3x + 5y – 8 = 0 and L2 = 5x – 3y – 11 = 0
are parallel
b. If the slopes are not the same, the lines
From L1 = 3x + 5y – 8 = 0,
are not parallel
m1 = =
Worked Examples
If A(0,3), B(7,2), P(6, -1) and Q(-1, -2) show From L2 = 5x – 3y – 11 = 0,
whether or not is parallel to m2 = = =

Solution
If L1 is perpendicular to L2, then m1 × m2 = - 1
A(0,3) and B(7,2), P(6, -1) and Q(-1, -2)
Let the gradient of AB be × =-1

= = =–

Perpendicular Lines
Two lines are said to be perpendicular when the
Let the gradient of PQ be
product of their gradient is – 1.That is if is
– ( )
=

= = = perpendicular to , then: × = – 1. In
other words, for any two lines and , if the
Therefore, is not parallel to gradient of one is m, the gradient of the other is

2. Given that Q(1, 4), R(6, 6), S(2, -1) and T(12,
3), show whether or not is parallel to . To prove whether or not two lines and are
perpendicular:
Solution 1. Find the slope of each line.
Q(1, 4), R(6, 6), S(2, -1) and T(12, 3) 2. Multiply both slopes.
Let the gradient of QR be 3. a. If the product in step 2 is -1, the
– lines are perpendicular.
= = =

b. If the product is not – 1, the lines
are not perpendicular.
Let the gradient of ST be
Note:
– ( )
= = = = Knowing the slope/gradient of a line, the

slope/gradient of a line perpendicular to it is

Baffour Ba Series, Further Mathematics for Schools Page 255


found by turning the known gradient upside show whether or not, is perpendicular to
down and changing its sign. For e.g. if a line
has a gradient of – , then the gradient of a Solution
Q(1, 4), R(6, 6), S(2, -1) and T(12, 3)
line perpendicular to it is
Let the gradient of QR be

Worked examples = = =

1. Write down the gradients of lines
perpendicular to the lines of gradients 3, , Let the gradient of ST be
– ( )
– 6 and respectively. = –
= = =

× = × =
Solution
 is not perpendicular to
Let = 3, = , = – 6 and =–
The gradient of the line perpendicular to: Exercises 9.15
= = A. For each of the following, determine
= = =–1× =–4 whether ̅̅̅̅ and ̅̅̅̅ are parallel,

perpendicular or neither;
= = = 1. A(2, 3) B(5, 6) P(-1, 4) Q(5, 10)
( )
2. A(1, 2) B(4, 5) P(-4, -1) Q(-10, 5)
= = ( ⁄ )
=–1× =
3. A(-2, 3) B(3, -3) P(4, 2) Q(-2, -3)
4. A(2, -3) B(-3, 3) P(2, 3) Q(4, 2)
2. Given A( – 1, – 1), B (0, 4), P(– 4, 3), Q(6,
5. A(3, 5) B(4, -2) P. , /, Q . , /
1), find whether or not AB is perpendicular to
PQ.
Equation of a parallel Line
Solution The Equation of a Line which Passes through
A( – 1, – 1), B (0, 4) P( – 4, 3), Q(6, 1), the Point ( , ) and Parallel to another
Let the gradient of AB be Line,
– –( ) Two or more lines are said to be parallel if they
= = = =5
– –( ) have the same gradient. That is if the gradient
of a line is m, then the gradient of
Let the gradient of PQ be the line parallel to it is m.
– –
= = = =–
– –( )
The equation of a straight line parallel to
=5×– =–1 another line of gradient m, passing through the
 AB is perpendicular to PQ point ( , ) is found by making y the subject
of the formula: y – = m( )
3. Given Q(1, 4), R(6, 6), S(2, -1) and T(12, 3),

Baffour Ba Series, Further Mathematics for Schools Page 256


Worked Examples 21 – 2 = 7c
1. What is the equation of the line parallel to 19 = 7c
the line 2x + 7y – 8 = 0, which passes through c=
the point (– 1, 3)?
Put m = – and c = in y = mx + c
Solution
Method 1 y=– x+
In 2x + 7y – 8 = 0 The equation is y = – x +
m=– = –
The line parallel to 2x + 7y – 8 = 0 has a The Equation of a Line which Passes through
gradient of – and passes through (–1 , 3) the Point ( , ) and Perpendicular to
⇒m = – , , another Line
Two lines are said to be perpendicular if the
Substitute in y – = m( ) product of their gradient is –1. That is, if the
gradient of a line is m, then the gradient of the
y – 3 = – (x – (–1)
line perpendicular to it is –
y – 3 = – (x + 1)
7(y – 3) = – 2(x + 1)
The equation of the straight line perpendicular
7y – 21 = – 2x – 2
to another line of gradient m, passing through
7y + 2x – 21 + 2 = 0
the point ( , ) is found by making y the
2x + 7y – 19 = 0
The equation is 2x + 7y – 19 = 0 subject of the formula: y – =– ( )
after substituting the values of m and ( , )
Method 2
Make y the subject of 2x + 7y – 8 = 0 Worked Examples
7y = – 2x + 8
1. Find the equation of the line perpendicular to
y = – x + compared to y = mx + c, the line 5x – 2y – 11 = 0, which passes through
gradient, m = – the point (2,–3).

Solution
The line parallel to 2x + 7y – 8 = 0 has a
In 5x – 2y – 11 = 0
gradient of – and passes through ( – 1 , 3)
m=
That is m = – , , The line perpendicular to the line 5x – 2y – 11 =
0, has a gradient of – and passes through (2,–3)
Substitute in y = mx + c
⇒m = – , ,
3=– (– 1) + c
Substitute in y – = m( )
3 × 7 = – 2 (– 1) + 7c
21 = 2 + 7c y – (– 3) = – (x – 2)

Baffour Ba Series, Further Mathematics for Schools Page 257


y + 3 = – (x – 2) Equation of Perpendicular Bisector of a line
I. Find the mid-point of the two given points
5(y + 3) = – 2(x – 2)
using the mid -point formula as (x1, y1).
5y + 15 = – 2x + 4
II. Find the gradient m of the two given points
5y + 2x + 15 – 4 = 0
and find its negative reciprocal to obtain the
2x + 5y – 11 = 0
gradient of the perpendicular line.
The equation is 2x + 5y – 11 = 0
III. Substitute the values of (x1, y1) and m in
y – y1 = m(x – x1) to obtain the equation of the
Method 2
perpendicular bisector of the two given points.
Make y the subject of 5x – 2y – 11 = 0
–2y = – 5x + 11
Worked Examples
y= x– compared to y = mx + c, 1. Find the equation of the perpendicular
gradient, m = bisector of AB, where A and B are the points
(- 4, 8) and (0, - 2).
The line perpendicular to the line
Solution
5x – 2y – 11 = 0, has a gradient of – and
Method 1
passes through (2,–3) A(– 4, 8) and B(0, – 2)
⇒m = – , x = 2 , y = -3 = – 4, = 8, = 0 and = – 2
Substitute in y = mx + c Perpendicular bisector of AB passes through the
–3 = – (2) + c mid – point of AB
( )
–3 × 5 = – 2 (2) + 5c = . , /=. , / = (–2, 3)
–15 = – 4 + 5c The perpendicular bisector passes through the
– 15 + 4 = 5c point (– 2, 3).
– 11 = 5c Gradient of AB
c=– – –
m= = = =–
– –( )

Gradient of the perpendicular bisector of AB =


Put m = – and c = – in y = mx + c
y=– x– Equation of the perpendicular bisector of AB
which passes through the midpoint (– 2, 3)
Exercises 9.16 y– ( )
1. Find the equation of the line perpendicular to
y = -2x + 3 through the point (-1, 2) But m = , = – 2, and =3
By substitution,
2. Find the equation of the line perpendicular to
7x – 2y = 3 through the point (1, 2) leavivg y – 3 = (x – (-2))
your answer rin the form ax + by + c = 0 5(y – 3) = ( )

Baffour Ba Series, Further Mathematics for Schools Page 258


5y – 15 = 2x + 4 3. Find the equation of the perpendicular
5y – 2x – 15 – 4 = 0 bisector of the points (1, 4) and (5, -2).
5y – 2x – 19 = 0 ( *
4. Find the equation of the right bisector of the
Method 2 line segment joining the points (3, 4) and (-1, 2)
A(– 4, 8) and B(0, – 2)
= – 4, = 8, = 0 and = – 2 The Angle a Line Makes with the x – axis
Perpendicular bisector of AB passes through the The angle a line makes with the positive x –
mid – point of AB axis is calculated by : θ = (m),
( ) where m is the gradient of the line.
= . , /=. , / = (–2, 3)
The perpendicular bisector passes through the Worked Examples
point (– 2, 3). 1. Find the angle between the x – axis and the
line joining the points (6, 10) and (5, 7).
Gradient of AB
– –
m= = = =– Solution
– –( )
(5, 10) and (4, 7)
Gradient of the perpendicular bisector of AB =
m= = =3
Substitute m = , x = – 2 and y = 3 in y = mx + c
θ= (3)
3 = (– 2) + c
θ = 720
5 × 3 = 2(–2) + 5c
15 = – 4 + 5c 2. Find the angle between the x – axis and the
15 + 4 = 5c line joining the points (3, -1) and (4, -2).
19 = 5c
c= Solution
(3, -1) and (4, -2)
( )
Put c = and m = in y = mx + c m= = =-1
y= x+ or y = ( *
θ= (-1),
0
Exercises 9.17 θ = - 45 ( Meaning 2nd quadrants)
1. Find the equation of the perpendicular θ = 1800 – 450
bisector of the points (2, 4) and (8, 0) in the θ = 1350
form ax + by = c
Application to Parallelograms
2. Find the equation of the perpendicular Given the vertices of a quadrilateral ABCD, it
bisector of the line segment joining points A(6, can be verified whether quadrilateral ABCD is a
-3) and (-2, 5) .

Baffour Ba Series, Further Mathematics for Schools Page 259


rectangle, a square, a rhombus or a Gradient of ̅̅̅̅ ;
parallelogram. –
mDC = = = =

First make a sketch of the figure as shown mAB = mDC


below
B C Also, gradient of ̅̅̅̅ = gradient of ̅̅̅̅

Gradient of ̅̅̅̅;
A D – ( )
mAD = = = =3

Take note of the fact that for all parallelograms
(rectangles, squares.. ) with vertices ABCD; Gradient of ̅̅̅̅;
I. ̅̅̅̅ is parallel to ̅̅̅̅ , meaning the gradient – ( )
mBC = = = =3
of ̅̅̅̅ is equal to that of ̅̅̅̅ and the magnitude – –( )

of ̅̅̅̅ is equal to the magnitude of ̅̅̅̅ . mAD = mBC


II. ̅̅̅̅ is parallel to ̅̅̅̅ , meaning the gradient
Therefore, ABCD is a parallelogram.
of ̅̅̅̅ is equal to that of ̅̅̅̅ and the magnitude
of ̅̅̅̅ is equal to the magnitude of ̅̅̅̅ . 2. The vertices of a parallelogram are at
III. The diagonals bisect each other, hence the P(2, 1) , Q(6, 3), R(5, 5), S(x, y).
mid-point of ̅̅̅̅ is equal to the mid-point of i. Find the coordinates of S.
̅̅̅̅ ii. What is the length of the diagonal QS?

Worked Examples Solution


1. Show that A(6, - 2), B(- 2, - 4), C(0, 2) and Method 1
D(8, 4) are vertices of a parallelogram. P(2, 1) , Q(6, 3), R(5, 5), S(x, y).
Q(6, 3) R(5, 5)
Solution
Method 1
A(6, - 2), B(- 2, - 4), C(0, 2) and D(8, 4) P(2, 1) S(x, y)
B(-2, -4) C(0, 2)
If PQRS is a parallelogram, then;
gradient of PQ = gradient of RS
A(6, -2) D(8, 4)
Gradient of PQ;
If ABCD is a parallelogram, then; –
mPQ = = = =
gradient of ̅̅̅̅ = gradient of ̅̅̅̅ –

Gradient of ̅̅̅̅ ; Gradient of SR;


– ( ) –
mAB = = = = mRS = =
– –

Baffour Ba Series, Further Mathematics for Schools Page 260


mPQ = mRS The coordinates of S(x, y) = S(1, 3)
=
Method 2
5 – x = 2(5 – y)
If PQRS is a parallelogram, then diagonals
5 – x = 10 – 2y
bisect each other;
5 – 10 = x – 2y
⇒Mid-point of QS = mid-point of PR
x – 2y = -5……………..(1)

Mid-point of QS;
Now, gradient of QR = gradient of PS
Gradient of QR; =. , /

mPQ = = =

Mid-point of PR;
S(x, y) R(5, 5)
Gradient of PS; =. , /
– =. , /
mRS = =

P(2, 10) Q(2, 10)
Now ,
mQR = mPS
. , / =. , /
=
2(x – 2) = -1(y – 1) ⇒ = …………….(1)
2x – 4 = - y + 1 = ………………..(2)
2x + y = 4 + 1
2x + y = 5………………...(2)
From eqn (1);
6+x=7
Solving equations (1) and (2) simultaneously;
x=7–6
x – 2y = -5………………..(1)
x=1
2x + y = 5……………….(2)
From eqn (2);
2 × eqn (1);
y+3 =6
2x – 4y = -10……………(3)
y=6–3
y=3
eqn (3) – eqn (2);
S(x, y) = S(1, 3)
- 5y = - 15
y=3
ii. Q(6, 3) and S(1, 3)
/QS/ = √( ) ( )
Put y = 3 in eqn (1);
x – 2(3) = - 5 /QS/ = √
x – 6 = -5 /QS/ = √
x=-5+6 /QS/ = 5 units
x=1

Baffour Ba Series, Further Mathematics for Schools Page 261


3. The points A(6, 6) B(2, -2) C(-8, -5) and D(- 6. Verify that A(- 4, -2), B(0, 0), C(2, 4) and D
4, 3) are vertices of a parallelogram. Verify that (-2, 2) are vertices of a rhombus.
the diagonals of this parallelogram bisect each
other. 7. A quadrilateral has vertices at A(0, 0) , B(7,
24), C(22, 44), D(15, 20). Find the length of the
Solution sides. What type of quadrilateral is ABCD?
B(2, -2) C(-8, -5)
A(6, 6) B(2, -2)
C(- 8, -5) D(- 4, 3) 8. Verify that the quadrilateral determined by
A(6, 6)
the points A(0, 7), B(-2, -1) C(2, -2) and D(4, 6)
D(-4, 3)
are vertices of a rectangle. What is the area of
Midpoint of ̅̅̅̅ = . , / = (-1, 0.5) the rectangle?

Midpoint of ̅̅̅̅ = . , / = (-1 , 0.5) Challenge Problem


ABCD is a quadrilateral where A, B, C and D
Midpoint of ̅̅̅̅ = Midpoint of ̅̅̅̅ = (-1 , 0.5)
are the points (3, -1), (6, 0), (7, 3) and (4, 2)
respectively. Prove that the diagonals bisect
Therefore, the diagonals ̅̅̅̅ and ̅̅̅̅ bisect
each other and hence, find the area of ABCD.
each other at the point (-1, 0.5)

Exercises 9.18 Point of Intersection of Two Lines


1. The points A(-3, 1), B(1, 2), C(0, -1) and D(x, To find the point of intersection of two lines,
y) are vertices of parallelogram ABCD. Find solve the equations of the lines simultaneously
the coordinates of vertex D. by any preferred method unless stated. The
solution set is the point of intersection of the
2. P(a, b), Q(7, 5), R(6, 2) and S(0, 4) are lines.
vertices of rectangle PQRS. Find the values of a
and b. Worked Example
1. Find the points of intersection of the lines
3. The points (1, 4) (6, 3) (10, 4) and (a, b) 2x – 3y = 6 and 4x + y = 19
form the vertices of a kite. Find the possible
values for a and b. Solution
2x – 3y = 6 ……………(1)
4. Verify that A(-5, 1) B(-2, 7), C(8, 3) and 4x + y = 19…………….(2)
D(5, -3) are vertices of a parallelogram.
eqn (1) 2;
5. i. The points A(4, 0) B(4, 4) C(0, 4) and D(0, 4x – 6y = 12……………(3)
0) are vertices of a square.
ii. Verify that the diagonals of this square are eqn (2) – eqn (3);
perpendicular. 7y = 7
y=1

Baffour Ba Series, Further Mathematics for Schools Page 262


Put y = 1 in eqn (1); 2y – 2 = 3x – 6
2x – 3(1) = 6 3x – 2y – 6 + 4 = 0
2x = 6 + 3 3x – 2y – 2 = 0
2x = 9
x= iii. The coordinates of Q and S.
x = 4.5 = =
 The point of intersection is (4.5, 1) ⇒ y – 1 = 1 ……………(1)
y=1+1
2. PQRS is a parallelogram in which the y=2
opposite vertices are P(2, 1) and R(4, 4). If the
slope of PQ is and the slope of PS = - 2, find; x – 2 = 3 …………….(2)
x=3+2
i. the equation of PQ;
x=5
ii. the equation of PR;
Q(5, 2)
iii. hence, or otherwise, find the coordinates of
Q and S.
From the parallelogram,
=
Solution
i. Q (x, y) R(4, 4)
= and =
=
4–b=1
P(2, 1) S(a, b ) b=4–1
b=3
The equation of PQ;
m = , passing through (2, 1) 4–a=3
y–1= (x – 2) a =4–3
a=1
3(y – 1) = (x – 2)
S(1, 3)
3y – 3 = x – 2
x – 3y – 2 + 3 = 0
Exercises 9.19
x – 3y + 1 = 0
A. Find the points of intersection of the
following pair of straight lines:
ii. the equation of PR;
1. y = 3x + 2 and 2x + 3y = 17
P(2, 1) and R(4, 4)
2. 5x + 3y = 2 and x–y=6
= = 3. y = 5x + 2 and y = 3x – 1
m = , passing through (2, 1)
B. 1. L : 2x – 5y – 9 = 0 and K : 3x – 2y – 8 = 0
y–1= (x – 2)
are two lines. L intersects K at the point q.
2(y – 1) = 3(x – 2) i. Find the coordinates of q.

Baffour Ba Series, Further Mathematics for Schools Page 263


ii. Find the equation of the line M such that L is | |
d= √ 3x + y -8= 0
perpendicular and q M. | ( ) ( ) ( )|
iii. Show that the point r (4, - 6) is on M. d= √
| | √
d=

2. P = {(x, y) : 4x + 3y = -7}, Q = {(x, y) : | |
5x – 9y = -13} and R = {(x, y) : 14x – 15y = d= = =√ units (-2, 4)
√ √
-33}, x, y R. Prove that P∩ Q⊂ R
2. Find the perpendicular distance from the
3. P = {(x, y) : 2x + 3y + 5 = 0} and Q = {(x, y) point (3, 5) to the line 3x + 4y – 17 = 0
: 5x – 4y + 1 = 0}, x, y R. Find P∩Q
Solution
4. The set of points L1 and L2 are defined by Point : (3, 5)
L1 = {(x, y) : x + y = 9 } and L2 = {(x, y) : Line : 3x + 4y – 17 = 0
x – y = 1} . Calculate the coordinates of the a = 3, b = 4, c = -17 , x1 = 3 and y1 = 5
| |
single member of the sets L1 ∩ L2. d= √
| ( ) ( ) ( )|
Perpendicular Distance from a Line d= √
The perpendicular distance, d, from the point | |
d= 3x + 4y -17 = 0
(x, y) to the line ax + by + c = 0 is given by: √
| |
| | d=
d= √

d=
ax + by + c = 0 d = 2.4 units
(3, 5)
Exercises 9.20
d
In each case, find the perpendicular distance
(x1, y1) from the point P(x, y) to the line L: ax + by +
The formula can also be written without the c=0
modulus bar. 1. P(1, 3) L: 3x + 4y + 10 = 0
2. P(8, 3) L: 5x – 12y + 9 = 0
Worked Examples 3. P(-1, 3) L: 15x – 8y + 5 = 0
Find the perpendicular distance from the point 4. P(3, 2) L: 24x – 7y – 8 = 0
(-2, 4) to the line 3x + y – 8 = 0
5. Show that the point (2, -1) is equidistant
Solution from the lines with equations 4x + 3y – 20 = 0
Point: (-2, 4) and 12x – 5y + 10 = 0.
Line: 3x + y – 8 = 0
a = 3, b = 1, c = -8 , x1 = -2 and y1 = 4 6. Find the shortes distance from the point (5, -
1) to the line y = 2x – 1

Baffour Ba Series, Further Mathematics for Schools Page 264


Distance between Two Parallel Lines | |
d= √
ax + by + k = 0
| ( ) ( )( ) | | |
d= = = = 1 unit
√( ) ( ) √

Exercises 9.21
x1, y1 ax + by + c = 0 Find the distance between each pair of
parallel lines, L and K.
To find the distance between two parallel lines,
do the following: 1. L : 5x + 12y – 5 = 0 and K : 5x + 12y + 10 = 0
1. Find one point on one of the lines. 2. L: 3x + 4y -12 = 0 and K: 6x + 8y – 9 = 0
2. Find the distance from this point to the other 3. L : 2x + y + 5 and K : 2x + 5 = 0
line. 4. Show that the point (4, - ) is on the line 3x
+ 4y – 10 = 0, and hence find the distance
Worked Examples between the parallel lines 3x + 4y – 10 = 0 and
Find the distance between the pair of parallel 3x + 4y – 15 = 0
lines L : 4x – 3y + 10 = 0 and K : 4x – 3y + 15 = 0
Slope of Two Lines Which Meet at a Point at
Solution a given distance from a given Point.
L : 4x – 3y + 10 = 0 The slope of any two lines which meet at a
K : 4x – 3y + 15 = 0 Point at a given distance from a given Point is
calculated as follows:
From L, when x = 0; 1. Identify the point through which the line
4(0) – 3y + 10 = 0 passes as (x1, y1).
– 3y + 10 = 0
– 3y = -10 2. Substitute the values of (x1, y1) in y – y1 =
y= m(x – x1) and simplify equating to zero on the
right hand side, to obtain the equation of the
The point (0, ) is a point on L
line.
4x - 3y + 15 = 0
K
3. Identify the given distance d, from the given
point as (x1, y1), differ from (x1, y1), in act 2.
4x -3y + 10= 0
L 4. From the equation of the line, find a, b and c
(0, )
and substitute in:
Now, distance between . , / and the line K : d=
| |

4x – 3y + 15 = 0
⇒ a = 4, b = -3, c = 15, x1 = 0 and y1 = Worked Examples
Substitute in Find the slope of the lines through the point

Baffour Ba Series, Further Mathematics for Schools Page 265


(-3, 2) which are at distance 2√ from the point 3. Find the equation of the two lines parallel to
(- 6, 1). 3x – 4y + 1 = 0 are two units from it.

Solution 4. L: 4x – 3y – 1 = 0 and K : 3x – 4y + 1 = 0 are


Given the point, (-3, 2), calculated the slopes; the equations of two lines. Find the locus of the
⇒ y – y1 = m(x – x1) point P(x, y) such that P is equidistance from L
y – 2 = m(x – (-3) 2√ and K.
y – 2 = m(x + 3) (-6, 1)
(-3, 2)
y – 2 = mx + 3m 2√ 5. A line with slope m, contains the point (-2,
y – 2 – mx – 3m = 0 1). Write its equation in the form ax + by + c =
-y + 2 + mx + 3m = 0 (Multiply through by –1 ) 0. Hence, find the equation of the two lines
mx – y + (3m + 2) = 0 (Re- arranging) through the point (-2, 1) and whose distance
from the origin is 1 unit.
But the distance from (-6, 1) to this line is 2√
From the equation of the line, a = m, b = -1 6. Find the equation of the two lines which
c = 3m + 2 and d = 2√ , substitute in : contain the point (4, 1) and are a distance 2√
| | units from (1, 2).
d= √
| ( ) ( ) | 7. Find the equation of the two lines which are
2√ =
√ ( ) perpendicular to the line 2x + 3y – 6 = 0 and
| |
2√ = √ which are a distance √ units from (3, 2)

| |
( √ ) = .√ / (Squaring both sides) Points on the same Side or Opposite sides of
a Line
8= To show whether two given points are on the
8( ) same or opposite sides of a given line,
8m + 8 = 1 – 6m + 9m2
2
1. Substitute the coordinates of the given points
0 = 9m2 – 8m2 – 6m + 1 – 8 into the equation of the line.
0 = m2 – 6m – 7 2. Two possibilities arise:
(m – 7) (m + 1) = 0 i. Same signs, then same side of the line.
m = 7 or m = -1 ii. Opposite signs, then opposite side of a line.

Exercises 9.22 Worked Examples


1. Find the values of a, if the point (4, 1) is 2 Show whether or not the points (101, 34) and
units from the line ax + 3y – 9 = 0 (58, 18) are on the same side of the line 5x –
13y – 60 = 0.
2. Find the values of k, if the distance from the
points (3, -1) to the line 3x + 4y – k = 0 is 6 Solution
units Put (x, y) = (101, 34) in 5x – 13y – 60 = 0
Baffour Ba Series, Further Mathematics for Schools Page 266
(101, 34) : 5(101) – 13(34) – 60 Worked Examples
= 505 – 442 – 60 = 3 1. Find the measure of the obtuse angle between
the lines 3x – 5y + 2 = 0 and x + 4y + 5 = 0
(58, 18) : 5(58) – 13(18) – 60
= 290 – 234 – 60 = - 4 Solution
Opposite signs and therefore, the points are on Let θ be the angle between the lines 3x – 5y + 2
opposite side of the line. = 0 and x + 4y + 5 = 0
From 3x – 5y + 2 = 0,` m1 =
Exercises 9.23
From x + 4y + 5 = 0, m2 =
1. i. Show that the points(100, 72) and (59, 27)
lie on opposite sides of the line x + y – 120 = 0.
tan θ = | |
2. Are the points (-20, 10) and (35, - 40) on the
. /
same side of the line 3x – 2y + 50 = 0? tan θ = | |=| |=| |=| |
. /. /
3. Investigate whether the points (30, 16) and
(-19, -11) are on the same side of the line tan θ = 1
10x – 20y – 1 = 0 as the origin (0, 0)
θ= (1)
0
θ = 45
Angles between Two Lines
The obtuse angle = 1800 – θ
If two lines L1 and L2 have slopes m1 and m2
= 1800 – 450
respectively, and θ is the angle between them,
= 1350
then:
tan θ = L1 2. L: tx – y – 3 = 0 and K : x – 2y – 1 = 0,
t R, are the equations of two lines. Given that
θ the angle between L and K is 450, find the two
possible values of t.
L2
In practice, the best approach is to find the Solution
acute angle θ by using: Let θ be the angle between the lines
tan θ = | | L: tx – y – 3 = 0 and K : x – 2y – 1 = 0

The obtuse angle is obtained by finding : From L: tx – y – 3 = 0 ,` m1 = =t


1800 – θ From K : x – 2y – 1 = 0, m2 = =
tan 450 = | |
Note
The formula has the variable, θ, m1 and m2. In . /
the question two of these variables‟ are given to tan 450 = | |
. /
work out for the third one.

Baffour Ba Series, Further Mathematics for Schools Page 267


1=| | -2 = 4p
p=
⇒1 = or = -1
OR
(Take “+” and “–” separately) = -1
1= p + 3 = -1 (1 – 3p)
2t – 1 = 2 + t p + 3 = -1 + 3p
2t – t = 2 + 1 3 + 1 = 3p – p
t=3 4 = 2p
p=2
Alternatively;
= -1 Congruencies of a Triangle
1. Centroid g
2t – 1 = -1 (2 + t)
A median of a triangle is a line segment from
2t – 1 = -2 – t
a vertex to the midpoint of the opposite side.
2t + t = -2 + 1
The three medians of a triangle meet at appoint
3t = -1
called the centroid, g.
t=
g divides each median in the ratio 2: 1
3. Find two possible values of p if the lines A(x1, y1)
px – y – 3 = 0 and 3x + y + 1 = 0 intersects at
450.

Solution
B(x2, y2) C(x3, y3)
From px – y – 3 = 0,` m1 = =p
From 3x + y + 1 = 0, m2 = = -3 Coordinate of g = . , /
( )
tan 450 = | |=| ( )
|
2. Circumcenter o
The circumcenter of a triangle is the point of
1=| |
intersection of the perpendicular bisectors of
⇒ = 1 or = -1 the sides.
A
(Take “+” and “– ” separately)
1=

1 – 3p = p + 3
B C
1– 3 = p + 3p

Baffour Ba Series, Further Mathematics for Schools Page 268


3. Ortho center h Excentre of a triangle is the point of
The ortho centre is the point of intersection of concurrency of bisectors of two exterior and
the perpendicular lines from the vertices to the third interior angle. Hence there are three
opposite sides. excentres I1, I2 and I3 opposite to three vertices
A of a triangle.

h Note:

 In a right angled triangle, orthocentre is the
B C
point where right angle is formed.
The ortho center can be located outside the  In an isosceles triangle, all of centroid,
triangle. orthocentre, incentre and circumcentre lie on
the same line.
The Incenter i  Orthocentre, centroid and circumcentre are
The incentre of a triangle is the point of always collinear and centroid divides the
intersection of the angle bisectors of angles of line joining orthocentre and circumcentre in
the triangle. An incentre is also the centre of the the ratio 2:1.
circle touching all the sides of the triangle.
Worked Examples
Note: Find the coordinates of the centroid of a
 Angle bisector divides the opposite sides in triangle with vertices A(5, -2) , B(1, 3) and
the ratio of remaining sides. That is: C(6, 2)
= =
Solution
 Incentre divides the angle bisectors in the A(5, -2) , B(1, 3) and C(6, 2)
ratio (b + c) : a, (c + a) : b and (a + b) : c. g=. , /
g=. , /=. , /=( , )
The Ex – center

Baffour Ba Series, Further Mathematics for Schools Page 269


10 PROBABILITY 1 Baffour Ba Series

Definition of Terms one side of a coin is represented by H and the


Experiment other side by T, then the sample space, for a
It is an activity in the form of demonstration coin is S = * , +. Similarly, the sample space
that is undertaken to obtain facts and draw for a Ludo die is S = {1, 2, 3, 4, 5, 6}
conclusions.
Event
Random Experiment An event is the occurrence with which a
It is an experiment whose results depend on particular interest is attached. It is always the
chance; meaning the result of the experiment subset of the sample space. For example, in a
cannot be predetermined. For instance, the toss die experiment, the sample space, S = {1, 2, 3,
of a coin is a random experiment because it 4, 5, 6}. If one is only interested in the
cannot be determine with surety and certainty, occurrence of an even number, then the event,
the face that will show up. E = {2, 4, 6}
Also, in the same experiment, if one is
Similarly, the game of football is a random interested in the occurrence of a number more
experiment because; one cannot predetermine than 4, then the event, E = {5, 6}
the outcome (whether win, draw or lose) with
certainty. Definition of Probability
Let S be the sample space of an experiment and
Outcome of an Experiment E, an event. The probability P(E) of E is given
The results we get from an experiment is called by;
Outcome of the experiment. In tossing a coin ( )
P(E) = ,
( )
for instance, either the front (also called head
Where n(E) and n(S) denote the number of
or back also called tail is bound to show up.
elements in the in the event and the sample
Hence there are only two expectations or
space respectively.
outcomes in the toss of a coin namely, head (H)
and tail (T). Likewise, in the game of football,
Since E is a subset of S, it is true that;
only three outcomes are expected namely;
0 ≤ n(E) ≤ n(S)
win , draw or lose ( ) ( )
⇒ ≤ ≤ (Divide through by n(s))
( ) ( ) ( )
The outcome of an experiment deals with 0 ≤ n(P) ≤ 1
numbers.
The first letter of an event is usually used to
Sample Space of an Experiment represent the probability of that event. For
It is the set of all possible outcomes of an instance, the probability of selecting a girl from
experiment. It is denoted by S; For instance, if a class can be written as P(G), and that of a boy

Baffour Ba Series, Further Mathematics for Schools Page 270


as P (B). Likewise, the probability of a number iii. Sample space S =* , , , , , +,
less than 5 can be written as P(<5). n(S) = 6 , E = {4}, n(E) = 1
( )
P(P) = =
( )
Note:
1. If P(E) = 0, then E has no element
2. In a class of 24 students, 8 of them are girls
2. If P(E) = 1, then E = S.
and the rest are boys. Find the probability of
3. A probability of zero (0) means an event will
picking at random, a boy at random from the
not occur.
class.
4. A probability of one means the event will
always occur.
Solution
Let G represent girls and B represent boys
Exercises 10.1
n(S) = 24, n(B) = 24 – 8 = 16,
List all the possible outcomes of the following; ( )
1. Days of the week on which a baby is born. P(B) = = =
( )
2. Months of the year in which workers are
paid. 3. A bag contains 12 mangoes of which 4 are
3. Time of the day for greetings are offered. not ripe. What is the chance of picking at
4. Observing the sex of a baby. random a ripe mango from the bag?
5. Playing the game of football.
6. Taking a penalty kick. Solution
7. Tossing a die? n(S) = 12, n(not ripe) = 4,
n(ripe) = n(S) – n(not ripe) = 12 – 4 = 8
( )
Worked Examples P(ripe) = = =
( )
1. A ludo die is thrown once, what is the
probability of scoring; Exercises 10.2
i. a prime number A. 1. What is the probability of an event which
ii. a composite number is certain to happen?
iii. a perfect square
2. What is the probability of an event which is
Solution impossible?
i. Sample space S = * , , , , , +
n(S) = 6, E = {2, 3, 5}, n(E) = 3 3. Given that the probability that an event will
( ) happen is a(0 ), what is the probability
P(E) = = =
( )
that the event will not happen?

ii. Sample space S =* , , , , , +, 4. Suppose that a committee of 2 is chosen at


n(S) = 6 C = {4, 6}, n(E) = 2 random from four people Andy, Ben, Cee and
( ) Dan. Find the probability that Andy is on the
P(C) = = =
( )
committee.

Baffour Ba Series, Further Mathematics for Schools Page 271


Equally Likely Events Worked Examples
Equally likely events are events of the same 1. There are 20 beads in a bag. The probability
experiment that have equal chance of that a red beads is taken at random from the
occurring. In other words, they are two or more bag is . Find the number of red beads in the
events of the same experiment that have the
bags.
same probability. For example, if a die is
tossed once: S = {1, 2, 3, 4, 5, 6} and n(S) = 6,
Solution
Each number on the die is equally likely to
n(S) = 20, n(E) = ? P(Red) =
show up in a single toss.
In a sample space, S, of equally likely But n(E) = P(R) × n(S).
outcomes, the probability of an event can be n(E) = × 20 = 5
expressed as ; The number of red beads in the bag is 5
P(E) =
( ) 2. A box contains 25 balls some of which are
P( E) = ( ) blue and others green. If the probability of
picking a green ball randomly from the box is .
Thus, in the throwing of a die, he probability of
( )
i. Determine the number of green balls in the
scoring the number 1, P(1) = = box.
( )
Likewise, the probability of scoring the number ii. How many blue balls are in the box?
( )
5, P(5) = =
( ) Solution
i. n(S) = 25, n(E) = ?, P(Green) =
P(1) and P(5), are said to have equally likely But n(E) = P(Green) × n(S).
outcomes because they have the same chance n(E) = 25 = 10
of occurrence or probability which is . Two or There are 10 green balls in the box
more of such events are called equally likely
events and they are said to have equally likely ii. n(Green) + n(Blue) = n(S)
outcomes. But n(G) = 10, n(S) = 25 and n(B) =?
n(B) = n(S) – n(G)
Finding n(E), given P(E), and n(S) n(B) = 25 – 10 = 15
To find the n(E), given P(E) and n(S), make Therefore, there are 15 blue balls in the box
( )
n(E), the subject of the formula, P(E) = to
( ) 5. There are 16 white, 20 blue and a number of
obtain, n(E) = P n(S) where, green identical tennis balls in a box. The
probability of picking a green tennis ball from
n(E) = number of event,
the box is .
n(S) = number of sample space
i. Find the total number of balls in the bag
P (E) = probability of obtaining that event.

Baffour Ba Series, Further Mathematics for Schools Page 272


ii. Find the probability of picking a blue tennis Method 2
ball from the same box. P(F) + ( ̅ ) = 1
( ̅ ) = 1 – P(F)
Solution But P(F) =
i. Let W represents white tennis ball,
B represents blue tennis ball, ( ̅) = 1 – = – = =
G represents green tennis ball, n(F) = × 120
x represents total number of balls in the bag. n(F) = 45
n(W) = 16, n(B) = 20, n (x) = ?
P(G) = , P(W) = , P(B) = 2. A bag contains some balls of which are
But P(W) + P(B) + P(G) = 1 red. Forty more balls of which five are red are
⇒ + + =1 added. If of all the balls are red, how many

+ =1– balls were there originally?

+ = – Solution
Let the number of balls be x
= n (R) =
18x = 36 × 25 If forty more balls are added, total number of
ball = x + 40
x= = 50
⇒The total number of balls in the bag is 50 Additional 5 red balls;
n (R) = +5
ii. n(S) = 50, n(B) = 20
( ) of all the balls are red;
P(B) = = =
( )

( )= +5
Some Solved Past Questions
20 × ( ) = 20 × + (20)5
1. The probability of picking a footballer from
a group of 120 sportsmen is How many 4( ) = 5x + 100
sportsmen were not footballers? 4x + 160 = 5x + 100
60 = 5x – 4x
Solution 60 = x
Method 1 Therefore, 60 balls were there originally
Let the number of footballers be n(F) and the
number of sportsmen who were not footballers 3. Three blue balls, five green balls and a
be n( ̅ ) number of red balls are put together in a sack.
n(F) = × 120 = 75 One ball is picked at random from the sack. If
( ̅ ) = 120 – 75 = 45 the probability of picking a blue ball is , find:

Baffour Ba Series, Further Mathematics for Schools Page 273


i. the number of red balls in the sack, 4. A bag contains a number of balls, 30 of
ii. the probability of picking a green ball. which are red and the remainder blue. If a ball
is chosen at random, the probability that it is
Solution red is 0.60. Find the number of blue balls in the
i.n(B) = 3, n(G) = 5 and n(R) = x bag.
n(S) = 3 + 5 + x
=8+x 5. A box contains 30 white, 25 blue and a
number of green identical balls. The probability
( )
P(B) = = , but P(B) = of picking a green ball from the box is . Find:
( )

= i. the total number of balls in the bag


ii. the number of green balls in the bag
3×6=8+x
iii. the probability of picking a blue ball from
18 = 8 + x
the box.
x = 18 – 8 = 10
But n(R) = x
6. A bag contains 60 balls, some of which are
n(R) = 10
red and some blue. The probability of a ball
( ) drawn at random, being red is . Find:
ii. P(G) = = , but x = 10
( )
i. the number of blue balls in the bag,
P(G) = = ii. the number of red balls that should be added
to the bag to change the probability to .
Exercises 10.3
1. Nissan and Benz cars numbered 48 are 7. A bag contains 12 balls of which x are red.
parked for sale. If the likelihood of Mr. White Write down the probability that a ball chosen at
buying a Nissan car is r te s , random from the bag at random is red. When
i. how many Nissan cars are on the park? 18 more red balls are added to the bag the
ii. find the number of cars that are Benz. probability is increased by . Write down an
equation in x and solve it.
2. A box contains 50 pens. The probability of
picking a blue pen at random from the box is . Complementary Events
i. How many blue pens are in the box? Complementary events are complementary sets
ii. If the rest of the pens are red, how many are for which the sample space, S, serve as the
they in the box? universal set.

3. There are 54 identical balls of white and Consider the table below;
green colors in a bag. The probability of
picking a green ball from the bag at random is Sample Event Complementary
. Find the number of white balls in the bag. Space Space Event Space

Baffour Ba Series, Further Mathematics for Schools Page 274


S = {1, 2, 3, =1– = – = =
1
4, 5, 6} E = {1, 2} E = {3, 4, 5, 6}
S = {H, T} E = {T} E1 = {H} 2. The probability that Paul will pass his
S = {1, 2, E = {3, 4,
examination is . What is the probability that
3…25} 5…25} E1 = {1, 2}
Paul will fail the same examination?
1
It is observed from the above table that the E
consist of all the elements of S that are not in Solution
E. Thus, E and E1 are called complementary Let the probability that Joe will pass be
events. P(P) = , then the probability that Joe will fail
is P(P1) ,
In general, if E is any event of a sample space P(P) + P(P1) = 1
S and E1 is the complementary event, then: P(P1) = 1 – P(P)
P(E) + P(E1) = 1……………….(1) P(P1) = 1 – = – =

It follows that;
Challenge Problems
P(E) = 1 – P(E1)……………….(2)
1. All possible two – digit numbers are formed
P(E1) = 1 – P(E)……………….(3)
from the digits 1, 2, 3, 4, 5, 6. Find the
probability that one of these numbers chosen at
For any two events A and B of the same
random will be divisible;
experiment,
i. by 5 ii.by 6
P (A) + P (B) = 1…… (1)

2. In an arithmetic test the lower – quartile,


From eqn (1);
median and upper quartile marks are 41, 55, 67
P (A) = 1 – P (B)…. .. (2)
respectively. What is the probability that a
P (B) = 1 – P (A)… …(3) pupil chosen at random scored:
i. less than 41; ii. between 41 and 67?
Worked Examples
1. The probability that it will rain on a Compound Events
particular day is . What is the probability that Events can be combined by the words “or” and
“and”. Events which are combined as such are
it will not rain on that day?
called combined events. The word “or”
corresponds to union (∪) and the word “and”
Solution
corresponds to intersection (∩). These are
Let the probability that it will rain be P(R) = ,
further explained below:
the probability that it will not rain , I. A∪B is the event that occurs if A or B (or
both) occur. In probability, “or” means
P( ̅ )= 1 – P(R)
addition. i.e P(A or B) = P(A) + P(B)

Baffour Ba Series, Further Mathematics for Schools Page 275


⇒P(A ∪ B) = P(A) + P(B) P(both will pass);
II. A∩B is the event that occurs if A and B = P(S and Y)
occur together. In probability, “and” means = × =
multiplication i.e. P(A and B) = P(A) P(B)
⇒P(A ∩ B) = P(A) + P(B) ii. P(both will not pass);
= P( ̅ and ̅ )
Worked Examples = × =
1. A fair die is thrown once, what is the
probability of obtaining a factor of 4 or iii. P(only one will pass);
= P(S and ̅ ) or P( ̅ and Y)
number greater than 4?
=. / +. /
Solution = + =
i. S = {1, 2, 3, 4, 5, 6}
n(S) = 6, iv. P(exactly one of them will pass);
Factors of 4 = {1,2,4} n(F4) = 3 P(only one will pass);
Numbers of greater than 4 = {5, 6}, n(G4) = 2 = P(S and ̅ ) or P( ̅ and Y)
P(F4) =
( )
= =. / +. /
( )
( ) = + =
P(G4) = =
( )
P(F4) or P(G4) = P(F4) + P(G4) v. P(at least one of them will pass);
⇒P(F4∪G4) = + = = P(S and ̅ ) or P( ̅ and Y) or P(S and Y)
=. /+. /+. /
2. The probability that Serwaa will pass a
= + + =
certain examination is and the probability that
her brother, Yaw will pass the same vi. P(at least one of them will not pass);
examination is . Find the probability that: = P(S and ̅ ) or P( ̅ and Y) or P( ̅ and ̅ )
i. both will pass the examination; =. /+. /+. /
ii. both will not pass the examination;
= + + =
iii. only one will pass the examination ;
iv. exactly one of them will pass the
examination. 3. The probablites that Sam, Ken and Tom will
v. at least one of them will pass; hit a target are , and respectively. If all the
vi. at least one of them will not pass. three men shoot at once, what is the probability
that:
Solution i. all of them hit the target;
i. P (S) = and P(Y) = ii. all of them miss the target;
P( ̅) = and P( ̅ ) = iii. only Tom hits the target;
iv. only Sam and Ken hit the target;
Baffour Ba Series, Further Mathematics for Schools Page 276
v. exactly one of them hit the target; = + + + + + + =
vi. at least one of them hit the target;
vii. at least two of them will hit the target;
vii. P(at least two of them hit the target);
viii. the target will be hit once.
P(SK ̅ ) or P(S ̅ T) or P( ̅KT) or P(SKT)

Solution = . / + . / + . / +
i. P (S) = , P(K) = and P(T) = . /
P( ̅) = , P( ̅ ) = and P( ̅ ) = = + + + =

P(all of them hit the target);


viii. P(the target will be hit once);
= P(S and K and T)
v. P(exactly one of them hits the target);
= × × =
= P(S ∩ ̅ ∩ ̅ ) or P( ̅ ∩ K∩ ̅ ) or P( ̅∩ ̅ ∩ T)
=. / +. /+. /
ii. P(all of them miss hit the target);
= P( ̅ and ̅ and ̅ ) = + + =
= × × =
4. Three men at a shooting range have
iii. P(only Tom hits the target); probabilities , and of hitting a bull‟s eye
= P( ̅ and ̅ and T)
with gun shot. Each man fires one shot. Find,
= × × =
correct to three decimal places, the probability
that only one man hits the bulls eye..
iv. P(only Sam and K hits the target);
= P(S and K and ̅ )
Solution
= × × =
Let the men be A, B and C respectively;

v. P(exactly one of them hits the target); P(A) = , P(B) = and P(C) =
= P(S ∩ ̅ ∩ ̅ ) or P( ̅ ∩ K∩ ̅ ) or P( ̅∩ ̅ ∩ T) P( ̅) = , P( ̅ ) = and P( ̅ ) =
=. / +. /+. /
= + + = P(only one man hits the bulls eye);
P(A ∩ ̅ ∩ ̅ ) or P( ̅ ∩ B∩ ̅ ) or P( ̅∩ ̅ ∩ C)
vi. P(at least one of them hit the target); =. / +. /+. /
P(S ̅ ̅ ) or P( ̅K ̅ ) or P( ̅ ̅ T) or P(SK ̅ ) or = + + =
P(S ̅ T) or P( ̅KT) or P(SKT
=. /+. /+. /+ Exercises 10.4
1. Romeo has four exercise books on his desk.
. / + . / + . / +
They are for Mathematics, Chemistry, Physics
. / theory and Physics Practicals. Juliet also has
four exercise books on his desk; they are for

Baffour Ba Series, Further Mathematics for Schools Page 277


Mathematics, Biology, Chemistry and Physics b. B will win both games;
Theory. A thief comes and steals one book at c. B will win the match?
random from each students desk. What are the
probabilities that they are : , , 7. Bag A contains 10 balls, of which 3 are red
and 7 are blue. Bag B contains 10 balls of
a. both mathematics books;
which 4 are red and 6 blue. A ball is drawn at
b. both pysics books;
random from each bag. Find the probability
c. both sceience books.
that :
2. Five people are in a room together. Find the i. both are red ;
probability that all five were born on a different ii. at least one is blue.
day of the year.
8. Two boxes each contain five tickets
3. Three people are in a an elevator together. numbered from 1 to 5. Two tickets are drawn at
Find the probability that all three were born on random, one from each box. Calculate the
the same day of the week. probability that:
i. the sum of the numbers on the two tickets is
4. The probability that a player can get a hit is 8 or more;
. Find the probability that he will get a hit ii. the product of these numbers is 8 or more.
three times in a row.
B. 1. The set A = {x : x is an integer such that 1
≤ x ≤ 200}?
5. The probability that a drug will cure a
a. Find how many elements of A contain the
certain disese is . However, if the drug is used,
digit 1 at least twice.
the probability that it will cause side effect is . b. If an element of A is selected at random,
Find the probability that a patient who uses the write down the probability that it contains the
drug will be cured and will suffer no side digit 1 either once or not at all.
effects.
2. A = {1, 2, 3}, B = {4, 5, 6, 7}; C = {(a, b): a
6. Two chess players, A and B, play a match in
A, b B}
which the players have the first move
P = {8, 9}; Q = {10, 11, 12, 13, 14}; R = {(p,
alternatively in successive games. When A has
q): p P, q Q}. Find :
the first move, the probability that A will win is
i. n(C)
and the probability of a draw is . When B
ii. the probability that a member of C∪R,
has the first move, the probability that B will chosen at random, is a member of C.
win is and the probability of a draw is also .
In a match of only two games, what are the 3. i. µ = {1, 2, 3, …100}, A = {multiples of 5},
probabilities that: B = {multiples of 3} and C = {multiples of
a. B will win when he has the first move and 2}. If a member x, of the universal set is
draw when he has the second move? chosen at random, find the probability that:

Baffour Ba Series, Further Mathematics for Schools Page 278


a. x A∩B b. x A∪B c. x A∩C No. of times a No. of possible
coin is tossed outcomes
4. µ = {(x, y) : x and y are positive integers} 1 2=
A = {(x, y) : 1 ≤ x ≤ 4 and 1 ≤ y ≤ 3}, B = {(x, 2 4=
3 8=
y) : x + y = 4} and C = {(x, y) : x = 1}. Find
n
the value of n(A) and the probability that a
member of A chosen at random will also From the table, if a coin is tossed n times, the
belong to : number of possible outcomes =
a. the set B b. the set B∪C
Worked Examples
5. A two figure number is written down at A fair coin is tossed twice.
random. Find the probability that; i. Write down the set of possible outcomes.
i. the number is greater than 67; ii. What is the probability of obtaining?
ii. the number contains at least one figure 5. a. exactly two heads, b. no head,
c. at least one head, d. a head and a tail.
The Experiment of Tossing a Coin
1. If a fair coin is tossed once (same as two Solution
coins tossed together), the possible outcomes i. S = {HH, HT, TH, TT}, n(S) = 4
are, S = {H, T}, and the number of possible
outcomes, n(S) = 2 ii. a. S = {HH, HT, TH, TT}, n(S) = 4
Let A denote the event of obtaining two heads
H T A = {HH}, n(A) = 1
H HH HT ( )
T TH TT P(A) = =
( )

The probability of obtaining two heads =


2. If a fair coin is tossed three times (same as
three coins tossed together), the possible
b. Let B be the event of obtaining no head;
outcomes, S = {HHH, HHT, HTH, HTT, THH,
B = {TT}, n(B) = 1
THT, TTH, TTT}, the number of possible ( )
outcomes, n(S) = 8. This is illustrated as : P(B) = =
( )

The probability of obtaining no head =


HH HT TH TT
H HHH HHT HTH HTT c. Let C be the event of obtaining at least one
T THH THT TTH TTT
head;
C = {HH, HT, TH}, n(C) = 3
The number of possible outcomes of the above
( )
experiments is summarized in the table P(C) = =
( )
below; The probability of obtaining at least one head =

Baffour Ba Series, Further Mathematics for Schools Page 279


d. Let D denote the event of obtaining a head ( )
P(C) = = =
( )
and a tail
D = {HT, TH}, n(D) = 2 The Experiment of Throwing a Die
( )
P(B) = = = 1. If a fair die is thrown once, the sample space,
( )
S = {1, 2, 3, 4, 5, 6} and the number of sample
The probability of obtaining a head and a tail =
space, n(S) = 6

2. Three fair coins are tossed once. 2. If a fair die is thrown twice or two dice are
a. Write down the set of all possible outcomes thrown together, the sample space, S, is
b. What is the probability of obtaining? obtained in a tabular form shown below:
i. exactly two heads? Die 1
ii. an even number of heads 1 2 3 4 5 6
1 1,1 1,2 1,3 1,4 1,5 1,6
iii. not more than one head?
2 2,1 2,2 2,3 2,4 2,5 2,6

Die 2
3 3,1 3,2 3,3 3,4 3,5 3,6
Solution 4 4,1 4,2 4,3 4,4 4,5 4,6
2. a. The set of possible outcomes, 5 5,1 5,2 5,3 5,4 5,5 5,6
S = {HHH, HHT, HTH, HTT, THH, THT, 6 6,1 6,2 6,3 6,4 6,5 6,6
TTH, TTT},
Number of possible outcomes, n(S) = 36
b. i. Let A represent the event of obtaining
exactly two heads; Note:
1. The pair of numbers through which the
S = {HHH, HHT, HTH, HTT, THH, THT, diagonal drawn from n to n passes sum up ton
TTH, TTT},n(S) = 8, 2. The pair of numbers at the left (up) of a
A = {HHT, HTH, THH}, n(A) = 3 diagonal drawn from n to n are less than n
( )
P(A) = = = 3. The pair of numbers at the right (down) of a
( )
diagonal drawn from n to n are greater or
ii. Let B represent the event of obtaining no more than n
tail;
Worked Examples
S = {HHH, HHT, HTH, HTT, THH, THT,
1. Two fair dice, A and B, each with faces
TTH, TTT}, n(S) = 8,
numbered 1 to 6 are thrown together.
B = {HHT, HTH, THH, TTT}, n(B) = 4
( ) i. Construct a table showing all the equally
P(B) = = =
( ) likely outcomes.
ii. From your table, list the pair of numbers on
iii. Let C represent the event of obtaining not the two dice for which the sum is;
more than one head; a. 5 b. 10
S = {HHH, HHT, HTH, HTT, THH, THT, c. more than 10 d. at least 10
TTH, TTT},n(S) = 8, iii. Find the probability that the two dice show:
C = {HTT, THT, TTH, TTT}, n(C) = 4

Baffour Ba Series, Further Mathematics for Schools Page 280


a. Different scores b. the same scores ( )
P(B) = = =
( )
iv. Find the probability that the sum of the
iv. a. Let C represent the pair of numbers that
numbers on the two dice is:
sum up to 5 =(1, 4), (2, 3), (3, 2), (4, 1)
a. 5 b. 10
n(C) = 4 and n(S) = 36
c. more than 10 d. at least 10
( )
Solution P(C) = = =
( )
i. Die 1
1 2 3 4 5 6 b. Let D represent the pair of numbers that sum
1 1,1 1,2 1,3 1,4 1,5 1,6 up to 10 = (4, 6), (5, 5), (6, 4)
2 2,1 2,2 2,3 2,4 2,5 2,6 n(D) = 3 and n(S) = 36
Die 2

3 3,1 3,2 3,3 3,4 3,5 3,6 ( )


4 4,1 4,2 4,3 4,4 4,5 4,6 P(D) = = =
( )
5 5,1 5,2 5,3 5,4 5,5 5,6
6 6,1 6,2 6,3 6,4 6,5 6,6 c. Let E represent the pair of numbers that sum
up more than 10 = (5, 6), (6, 5), (6, 6)
From the table, the total number of possible n(E) = 6 and n(S) = 36
outcomes, n(S) = 36 ( )
P(E) = = =
( )
ii. a. The pair of numbers for which the sum is
5 = (1, 4), (2, 3), (3, 2), (4, 1) d. Let F represent the pair of numbers that sum
up to at least 10
= (4, 6), (5, 5), (6, 4), (5, 6), (6, 5) and (6, 6)
b. The pair of numbers for which the sum is 10
n(F) = 6 and n(S) = 36
= (4, 6), (5, 5), (6, 4) ( )
P(F) = = =
( )
c. The pair of numbers for which the sum is
more than 10 = (5, 6), (6, 5) and (6, 6) 2. Two fair dice are thrown at the same time.
d. The pair of numbers for which the sum is at a. Draw the sample space for the possible
least 10 = (4, 6), (5, 5), (6, 4), (5, 6), (6, 5) and outcomes.
(6, 6)
b. Find the probability of obtaining:
iii. a. Let A represent the pair of numbers that i. a total score of 6 or 8,
show different scores ; ii. the same number on the two dice,
n(A) = 30 and n(S) = 36 iii. a total not less than 5.
( )
P(A) = = = 1 2 3 4 5 6
( )
1 1,1 1,2 1,3 1,4 1,5 1,6
2 2,1 2,2 2,3 2,4 2,5 2,6
b. Let B represent the pair of numbers that 3 3,1 3,2 3,3 3,4 3,5 3,6
show the same number; 4 4,1 4,2 4,3 4,4 4,5 4,6
= (1, 1), (2, 2), (3, 3), (4, 4), (5, 5) and (6, 6) 5 5,1 5,2 5,3 5,4 5,5 5,6
n(B) = 6 and n(S) = 36 6 6,1 6,2 6,3 6,4 6,5 6,6

Baffour Ba Series, Further Mathematics for Schools Page 281


b. E (6) = [(5, 1) (4, 2), (3, 3), (2, 4), (1, 5)] n(E) = 9
n(E6) = 5 P(E) = =
E (8) = [(5, 3) (4, 4), (3, 5), (2, 6)]
n(E8) = 4
ii. a pair of prime numbers, P
From the table, the total number of possible
= (2, 3), (2, 5), (3, 2), (3, 3), (3, 5), (5, 2), (5,
outcomes, n(S) = 36
3), (5, 5)
n(P) = 8
i. The probability of a total score of 6 or 8
( ) ( ) P(P) = =
P(6 or 8) = +
( ) ( )

= + = iii. A total score that is atmost 4,


E = (1,1), (1, 2), (1, 3), (2, 1), (2, 2), (2, 3)
ii. The probability of the same number on the n(E) = 6
two dice P(E) = =
E = [(1, 1) (2, 2) (3, 3) (4, 4) (5, 5) (6, 6)]
n (E) = 6 n(s) = 36 4. Two dice are thrown together and the scores
P(same number) = = are added. Copy and complete the following
table of total scores;
iii. a total of not less than 5
n (E) = 30 n(s) = 36 1 2 3 4 5 6
1 2
P(not less than 5) = =
2
3 7
3. In the throw of two fair dice, what is 4
the probability of throwing? 5
6 12
i. a pair of even numbers;
ii. a pair of prime numbers; i. What is the probability of scoring exactly 9
iii. a total score that is at most 4. ii. What is the probability of scoring an even
iii. What is the probability of scoring either 7
1 2 3 4 5 6
1 1,1 1,2 1,3 1,4 1,5 1,6 or 11
2 2,1 2,2 2,3 2,4 2,5 2,6
3 3,1 3,2 3,3 3,4 3,5 3,6 Solution
4 4,1 4,2 4,3 4,4 4,5 4,6
5 5,1 5,2 5,3 5,4 5,5 5,6 1 2 3 4 5 6
6 6,1 6,2 6,3 6,4 6,5 6,6 1 2 3 4 5 6 7
2 3 4 5 6 7 8
3 4 5 6 7 8 9
n(S) = 36
4 5 6 7 8 9 10
A pair of even numbers, E; 5 6 7 8 9 10 11
(2, 2), (2, 4), (2, 6), (4,2), (4, 4), (4, 6), (6, 2), 6 7 8 9 10 11 12
(6, 4), (6, 6)
Baffour Ba Series, Further Mathematics for Schools Page 282
n(S) = 36 C. Throwing a Coin and a Die Together
i. Probability of scoring exactly 9 If a coin and a die are thrown together or one
ii. Probability of scoring an even number after the other, the outcomes are obtained as
iii. Probability of scoring either 7 or 11 shown in the table below;

5. Two fair dice are thrown once. Find the Dice


probability of obtaining a difference of 2 or a 1 2 3 4 5 6
product greater than 8. Coin H
Solution T

1 2 3 4 5 6
1 1,1 1,2 1,3 1,4 1,5 1,6 Worked Examples
2 2,1 2,2 2,3 2,4 2,5 2,6 1. In a game, a player throws a die and tosses a
3 3,1 3,2 3,3 3,4 3,5 3,6 coin. If the coin lands heads, he scores twice
4 4,1 4,2 4,3 4,4 4,5 4,6 the number on the die. If the coin lands tails,
5 5,1 5,2 5,3 5,4 5,5 5,6
he scores three times the number on the die.
6 6,1 6,2 6,3 6,4 6,5 6,6
What is the probability of scoring more than 6?
n(S) = 36
Let A be pair with difference of 2 Solution
A = (1, 3), (2, 4), (3, 5), (4, 6),
(3, 1), (4, 2), (5, 3), (6, 4), Dice
n(A ) = 8 1 2 3 4 5 6
Coin H 2 4 6 8 10 12
P(difference of two) =
T 3 6 9 12 15 18
Let B be pairs with product greater than 8;
B = (2, 5), (2, 6), (3, 3), (3, 4), (3, 5) n(S) = 12,
(3, 6), (4, 3), (4, 4), (4, 5), (4, 6) More than 12 = { 8, 9, 10, 12, 12, 15, 18}
(5, 2), (5, 3), (5, 4), (5, 5), (5, 6) n( More than 12) = 7
(6, 2), (6, 3), (6, 4), (6, 5), (6, 6) P( More than 12) =
n(B) = 20
2. A coin is toss twice and a die is rolled once.
P(product greater than 8) = Find the probability of the following events:
A ∩ B = (3, 5) (4, 6), (5, 3), (6, 4)
a. two heads and a 5;
n(A ∩ B) = 4
P(A∩B) = b. exactly one head and a 1 or 2;
P(A or B) = P(A) + P(B) – P(A∩B) c. at least one head and not a 2.
= + – = =

Baffour Ba Series, Further Mathematics for Schools Page 283


Solution Worked Examples
a. 1. A number is selected from each of the sets
1 2 3 4 5 6 {2, 3, 4} and {1, 3, 5}. Find the probability
HH HH1 HH2 HH3 HH4 HH5 HH6 that:
HT HT2 HT2 HT3 HT4 HT5 HT6 i. the sum of the two numbers is less than 7 and
TH TH1 TH2 TH3 TH4 TH5 TH6 greater than 3
TT TT1 TT2 TT3 TT4 TT5 TT6 ii. the sum of the numbers is a prime number.

n(S) = 24 Solution
Two heads and a 5;
E = {HH5}, n(E) = 1 2 3 4
P(two heads and a 5) = 1 (1, 2) (1, 3) (1, 4)
3 (3, 2) (3, 3) (3, 4)
5 (5, 2) (5, 3) (5, 4)
b. exactly one head and a 1 or 2;
E = {HT1, HT2, TH1, TH2}, n(E) = 4 n(S) = 9
P(exactly one head and a 1 or 2) = = Sum of the two numbers less than 7 and greater
than 3 = (1, 3), (1, 4), (3, 2), (3, 3)
c. at least one head and not a 2; n(E) = 4
( )
E = HH1, HH3, HH4, HH5, HH6, P= =
( )
HT1, HT3, HT4, HT5, HT6,
TH1, TH3, TH4, TH5, TH6, ii.sum of the numbers that is a prime number
N(E) = 15 E = (1, 2), (1, 4), (3, 2), (3, 4), (5, 2), n(E) = 5
P(P) =
P(at least one head and not a 2) = =
2. A two digit numeral (base ten) is formed by
Other Related Experiments choosing both digits at random from the set {6,
In some related instances, two setsof elements 7, 8, 9}. The same digit may be chosen twice.
may be provided for an experiment. For Find the probability that the number is;
example, given A = {1, 2, 3} and B = {t, h, e}, a. even, b. divisible by 4, c. prime.
if A and B occur at the same time, then the set
of possible outcomes or sample space is Solution
represented as shown below: i.
6 7 8 9
1 2 3 6 66 67 68 69
t 1t 2t 3t 7 76 77 78 79
h 1h 2h 3h 8 86 87 88 89
e 1e 2e 3e 9 96 97 98 99

Number of sample space, n(S) = 9 n(S) = 16


Baffour Ba Series, Further Mathematics for Schools Page 284
Even numbers, E = {66, 68, 76, 78, 86, 88, 96, a. two even numbers;
98} b. two prime number;
n(E) = 8 c. either two prine or two even numbers.
P(E) = =
4. A die is rolled three times. Find the
probablity that the outcome is 6 on the first
ii. Numbers divisible by 4,
roll, an even number on the second roll and an
= {68, 76, 88, 96}
odd prime number on the third roll.
n( numbers divisible by 4) = 4
P( numbers divisible by 4) = =
5. Two unbiased dice, one red and one white
each number 1, 2, 3, 4, 5, 6 are thrown at
iii. Prime numbers = {67, 79, 89, 97} random on to a table.
n(prime) = 4 i. if the number shown on the red die is x and
P(Prime) = = on the red die is y, list all the possible pairs (x,
y) which can appear. Call this set of pairs
Exercises 10.5 ii. List the members of the set P, a subset of ,
1. The following is an incomplete table of such tha x + y = 8 ⇔ (x, y) P.
possible outcomes when a die is thrown twice iii. What is the probability that a total score of
8 will occur.
1 2 3 4 5 6 iv. List the members of the set R, another
1 1, 1 1, 2 1, 3 subset of , such that x and y are both even ⇔
2
3 (x , y) R.
4
5 B. 1. A two digit number is formed as follows.
6
The tens digit is chosen at random from {3, 4,
i. Copy and complete the table. 5} and the ones digit is chosen at random from
ii. Use your table to find the probability of {0, 1, 2}. Find the probability that the number
throwing: will be:
a. no six, a. even; b. prime. c. divisible by 7;
b. at least one five,
c. two sixes. 2. A two digit numeral in base ten is fromed by
choosing the tens digit from the set {4, 5, 6, 7,
2. Two symmetrical dice are thrown together. 8} and the units digits from the set {1, 2, 3, 9}.
what is the probability that one of the dice will Find the probability that the resulting number
show a a perfect square and the other will sjow is:
a prime number? a. even;
b. greater than 50;
3. Two symmetrical dice are thrown together. c. both even and greater than 50;
What is the probability of throwing : d. either even or greater than 50 (or both)

Baffour Ba Series, Further Mathematics for Schools Page 285


3. A three digit numeral in base ten is formed of the other. In other words, when two events
by arranging the digits 5, 6, 7 in random order. cannot occur at the same time, they are
Find the probability that the number will be; considered mutually exclusive. For a mutually
i. odd, exclusive event, P(A∩B) = 0.
ii. divisible by 3;
iii. between 660 and 770. Worked Examlpes
1. What is the probability of getting a 2 or a 5
4. A committee of three students is to be when a die is rolled?
chosen from {Abel, Benjamin, Charles,
Delilah, Elizabeth}. If this committee is chosen Solution
at random, find the probability that : S = {1, 2, 3, 4, 5, 6}
a. they will all be boys; n (S) = 6
b. Abel will be on the committee; P(2) = and P(5) =
c. there will be one girl and two boys on the
committee . Probability of getting a 2 or a 5,
P(2 or 5) = P(2) + P(5) – P(2 and 5)
5. Suppose that the probability of a child being
= + –0= =
a boy is . Find the probability that a family od
three children will have; 2. Given that X and Y are mutually exclusive
a. at least two boys; events such that P(X) = 0.5, P(Y) = n and
b. exactly two boys; P(X or Y) = 0.8,
c. all girls. i. find the value of n;
ii. what is the probability of X or Y occurring in
6. A two digit numeral (base ten) is formed by
terms of X and Y?
choosing both digits at random from {6, 7, 8,
9}. However, the same digit may not be chosen Solution
twice. Find the probability that the number P(X or Y) = P(X) + P(Y)
will be: But P(X or Y) = 0.8, P(X) = 0.5, P(Y) = n
a. odd, b. divisible by 6; c. a perfect square By substitution,
P(X or Y) = P(X) + P(Y)
7. A boy tosess a coin and throws a die. If the 0.8 = 0.5 + n
coin lands head, he scores the number shown 0.8 – 0.5 = n
on the die. If the coin lands tails, he scores 0.3 = n
double the die number. What is the probability n = 0.3
that he will score an odd number.
ii. P(X or Y) = P(X) + P(Y)
Mutually Exclusive Events:
Mutually exclusive events are those where the 3. Two mutually exclusive events A and B are
occurrence of one indicates the non-occurrence

Baffour Ba Series, Further Mathematics for Schools Page 286


such that P(A) = and P(B) = . What is the Find the probability that a pupil selected at
random is either underweight or over weight.
probability of either A or B occurring?

3. A counsellor tells a student that his


Solution
probability of earning a grade of D in
P(A or B) = P(A) + P(B)
mathematics is and the probability of earning
But P( ) = and P( ) = an F grade is . Find the probability that the
⇒P(A∪ ) = + = student earns a C or better grade.

The probability of A or B occurring is Independent and Dependent Events


Independent Event
4. Consider the example of finding the When multiple events occur, if the outcome of
probability of selecting a black card or a 6 from one event does not affect the outcome of the
a deck of 52 cards. other events, they are called independent
events. For example if, a die is rolled twice, the
Solution outcome of the first roll doesn‟t affect the
We need to find out P (B or 6) second outcome. These two are independent
Probability of selecting a black card = events.
Probability of selecting a 6 =
Worked Examples
Probability of selecting both a black card and a 1. A coin is tossed twice. What is the
6= probability of getting two consecutive tails ?
P(B or 6) = P(B) + P(6) – P(B and 6)
= + – = = Solution
Method 1
When a coin is tossed twice, the sample space
Exercises 10.6
S = {(H, H), (H,T), (T, H), (T,T)}.
1. If A and B are mutually exclusive events and
n(S) = 4
P(A) = and P(B) = , find the probability E = (T,T) n(E) = 1
of A or B occurring.
( )
P(T, T) = =
( )
2. In a certain study, pupils were classified as
being underweight or overweight or having
normal weight. The probability that a pupil is Method 2
underweight is 0.3 and the probability that a Probability of getting a tail in first toss
pupil has a normal weight is 0.5 and the P(T) =
probability that a pupil is overweight is 0.2. Probability of getting a tail in second toss

Baffour Ba Series, Further Mathematics for Schools Page 287


P(T) = By substitution;
= + P(B) – P(B)
P(T ∩ T) = × =
– = P(B) – P(B)
2. Events A and B are sucht that P(A) = and – = P(B) . /
P(B) = . Find P(A∪B), if A and B are : = P(B)
a. mutually exclusive; ⁄
b. independent. P(B) = =

Solution Dependent Events


a. If events A and B are mutually exclusive; Two events that are not independent are said to
P(A ∪ B) = P(A) + P(B) be dependent. In other words, when two events
= + = occur and the outcome of one event affects the
outcome of the other, they are called dependent
b. a. If events A and B are independent ; events.
P(A ∪ B) = P(A) + P(B) – P(A ∩ B)
P(A ∪ B) = P(A) + P(B) – P(A) × P (B) Worked Example
= + –. / 1. What is the probability of the occurrence of
a number that is odd or less than 5 when a fair
= – = die is rolled.

3. Two events A and B are such that P(A∪B) = Solution


and P(A) = . Find P(B) if the events A and B S = (1, 2, 3, 4, 5, 6)
n(S) = 6
are :
Let the event of the occurrence of a number
a. mutualy exclusive;
that is odd be „A‟ and the event of the
b. indepenedent.
occurrence of a number that is less than 5 be
Solution „B‟. We need to find P(A or B).
a. If events A and B are mutually exclusive;
A = (odd numbers ) = {1, 3 and 5} n(A) = 3
P(A ∪ B) = P(A) + P(B)
P(A) =
= + P(B)

B = (numbers less than 5 = 1, 2, 3 and 4) n(B) = 4


P(B) = – =
P(B) =

b. If events A and B are independent ; P(A ∩ B) = P(A) × P(B)


P(A ∪ B) = P(A) + P(B) – P(A ∩ B)
× = =
P(A ∪ B) = P(A) + P(B) – P(A) × P (B)
(numbers that are both odd and less than 5 = 1 and 3)

Baffour Ba Series, Further Mathematics for Schools Page 288


P(A or B) = P(A) + P(B) – P(A or B) b. P(divisible by 20 or 49);
P(A ∪ B) = P(A) + P(B) – P(A ∩ B) P (A ∪ C) = P(A) + P(C) – P(A ∩ C)
= + – = + –0
P(A ∪ B) = .
P(A ∪ B) = .
2. The probabilities that a student will pass two
different tests are 0.8 and 0.7. If the probability Exercises 10.7
of passing at least one test is 0.9, find the 1. If X and Y are two independent events such
probability of pasing both test. that P(X) = and P(X∩Y) = , find:
i. P(Y) ii. P(X∪Y)
Solution
P ( ) = 0.8, P ( ) = 0.7, P ( ∪ ) = 0.9 2. From the given probabilities, determine
P ( ∪ ) = P( ) + P ( ) – P( ∩ ) whether or not events A and B are independent
By substitution, events:
0.9 = 0.8 + 0.7 – P( ∩ ) i. P(A) = , P(B) = and P(A ∩ B) =
P( ∩ ) = 0.8 + 0.7 – 0.9
P( ∩ ) = 0.6 ii. P(A) = , P(B) = and P(A∩B) =

3. The pages of a book are numbered from 1 to 3. The probability that Tom will solve a
200. If a page of this book is opened at random, problem is and the probability that Jerry will
what is the probability that its number will be; solve the same problem is . Find the
a. divisible by 20 or 50;
probability that at least one of them will solve
b. divisible by 20 or 49.
it.
Solution
4. Three children leave their village to start a
a. n(S) = 200
new life in either Accra or Kumasi. The
A = {numbers divisible by 20}
B = {numbers divisible by 50} probability that May will go to Kumasi is ,
C = {numbers divisible by 49} that Martha will go to Kumasi is and that
n(A) = 10, n(B) = 4 and n(C) = 4
Mary will go to Accra is . Find the probability
P(A) = , P(B) = and P(C) =
that exactly two of them will end up in Accra.
P(divisible by 20 or 50);
5. The administrators of a program to prepare
P (A ∪ B) = P(A) + P(B) – P(A ∩ B)
people for a senior high school exams have
= + –. / founf that 80% of the students require tutoring
= + – = . in mathematics, 60% need help in English, and
45% need help in both maths and English. Find

Baffour Ba Series, Further Mathematics for Schools Page 289


the probability that a student selected at P(M) = 0.60
random needs help in either maths or English. ( )
P(S|M) = = = = 0.67
( )

6. The insurance underwriters have determined


2. In an exam, two reasoning problems, A and
that in any one year, the probability that
B, were asked. 35% students solved problem A
George will have a car accident is 0.05, and
and 15% students solved both the problems.
that if he has an accident, the probability that
How many students who solved the first
he will be hospitalized is 0.40. Find the
problem will also solve the second one?
probability that George will have an accident
but will not be hospitalized.
Solution
Probability of student solving problem A,
7. A woman visits her cabin in Canada. The
P(A) = 0.35
probability that her landmower will start is ,
and the probability that her gas powered saw Probability of student solving both problem,
will start is and that her outboard motor will P(A and B) = 0.15
Probability of solving B if A is solved,
start is . Find the probability that: ( )
P(A|B) = = = = 0.42
a. all the three will start; ( )
b. none will start;
c. exactly one will start; 3. When two dice are rolled, find the
d. exactly two will start. probability of getting a greater number on the
first die than the one on the second, given that
Conditional Probability the sum should equal 8.
Conditional probability is calculating the
probability of an event given that another event Solution
has already occured . Let the event of getting a greater number on the
first die be G.
The formula for conditional probability P(A|B), There are 5 ways to get a sum of 8 when two
read as P(A given B) is dice are rolled
P(A|B) = P (A and B) / P(B) = {(2, 6), (3, 5), (4,4), (5, 3), (6, 2)}.

Worked Examples And there are two ways where the number on
1. In a class, 40% of the students study maths the first die is greater than the one on the
and science. 60% of the students study maths. second given that the sum should equal 8,
What is the probability of a student studying G = {(5, 3), (6, 2)}.
science given he/she is already studying maths? Therefore,

Solution P(Sum equals 8) = and P(G) =


P(M and S) = 0.40 Now, P(G|sum equals 8);
Baffour Ba Series, Further Mathematics for Schools Page 290
( ) ⁄ 3. A mathematics teacher gave her class two
= =( *=
( ) ⁄ tests. 30% of the class passed both tests and
45% of the class passed the first test. What
4. A math teacher gave her class two tests. percent of those who passed the first test also
25% of the class passed both tests and 42% of passed the second test?
the class passed the first test. What percent of
those who passed the first test also passed the 4. Given that the toss of a die is even, what is
second test? the probability that it is divisible by three?

Solution Selection with Replacement


( )
P(Second/First); = “Selection with replacement” means selecting
( )
an item from a lot and putting it back from
P(S/F) = = 0.60
where it was taken. The addition and
multiplication rules of probability stated above
5. A jar contains black and white marbles. Two are used to determine the probability of A or B
marbles are chosen without replacement. The (two or more items of different or the same
probability of selecting a black marble and then kind) from a lot with replacement.
a white marble is 0.34, and the probability of
selecting a black marble on the first draw is Generally, if a selection is done with
0.47. What is the probability of selecting a replacement;
white marble on the second draw, given that 1. The number of items in the various groups
the first marble drawn was black? does not change.
2. The total number of items in the lot (Sample
Solution space) does not change.
( )
P(White/Black) = ( )
Worked Examples
P (W/B) = = 0.72 1. A pack contains 4 blue, 2 red and 3 green
pens. If a pen is drawn at random from the
Exercises 10.8 pack, replaced and the process repeated 2 more
1. The probability that it is Friday and that a times, What is the probability of drawing 2
student is absent is 0.03. Since there are 5 blue pens and 1 green pen?
school days in a week, the probability that it is
Friday is 0.2. What is the probability that a Solution
student is absent given that today is Friday? n(B) = 4, n (R) = 2 and n (G) = 3
n(S) = 4 + 2 + 3 = 9
2. At Prempeh College, the probability that a
P(First B) =
student takes Twi and Science is 0.087. The
probability that a student takes Twi is 0.68. P(Second B) =
What is the probability that a student takes P(one G) =
Science given that the student is taking Twi?
Baffour Ba Series, Further Mathematics for Schools Page 291
P(2 blue pens and 1 green pen); ii. P(same colour);
= × × = = P(B and B)
= × =
2. A bag contains 4 red balls and 6 black balls.
A ball is selected from the bag after which it is 4. A bag contains 5 red and 4 white identical
replaced and a second ball selected and balls and a second bag contains 3 red and 6
replaced. What is the probability that both balls white identical balls. If one ball is selected at
are red? randon from each of the bags, find the
probability that the two balls selected will be of
Solution a. different collours;
n(R) = 4 and n (B) = 6, n(S) = 10 balls b. same colour.
P(both ball are red);
= P(R) × P(R) Solution
= × = = i. Let the first bag be bag A and the second be
bag B
3. A box A contains 3 white, and 2 blue balls.
Another box B contains 4 green and 5 blue For bag A;
balls. A ball is picked at random from each n(R) = 5, n(W) = 4, n(S) = 9
box, find the probability that:
i. one is green and the other is white; For bag B;
ii. they are of the same colour. n(R) = 3, n(W) = 6, n(S) = 9

Solution P(different colours);


i. Let W represents white balls, G represents = P( and ) or P( and )
green balls and B represents blue balls. =. / +. /
= + =
For box A;
n(W) = 3 n(B) = 2, n(S) = 5
ii. P(same colous);
P(W) = and P(B) =
= P( and ) or P( and )
For box B, =. / +. /
n(G) = 4, n(B) = 5, n (S) = 9 = + =
P(G) = and P(B) =
5. Three bags P, Q and R contains red, blue and
P(one green and the other white) ; white balls respectively of equal sizes. The raio
= P(G and W) of the ball in the bag are P : Q = 2 : 3, and Q :
= × = R = 4 : 5. All the balls are removed into a big
bag and properly mixed together.

Baffour Ba Series, Further Mathematics for Schools Page 292


a. Find the probability of picking a: from the bag. Calculate the probability that the
i. red ball ii. blue ball iii. white ball balls were:
b. If two balls are picked at random one after a. both white;
the other with replacement, find the probability b. both red;
of picking: c. different colours.
i. a white ball and a blue ball
ii. blue ball first, then a red ball. 3. A bag containing 3 white balls and 5 black
balls has 4 balls drawn at a time, in such a way
Solution that the first ball is replaced before the next one
a. = and = is drawn. Find the probability of:
a. selecting 3 white balls;
= × = b. selecting at most two white balls;
= × = c. selecting a white ball, two black balls and a
white ball in that order;
⇒ P = 8, Q = 12 and R = 15
d. selecting two white balls and two black
n(R) = 8, n(B) = 12 and n(W) = 15
balls.
n(S) = 8 + 12 + 15 = 35
i. P(R) = , P (B) = and P (W) = 4. A box contains n balls of which 3 are red, 4
blue and the rest are white balls which are
b. i. P(white and blue) ; identical except for colour. A ball is selected at
= P(W and B) random from the bag. Its color is noted and
= × = = replaced. A second ball is selected from the
bag. If the probability of selecting 2 blue balls
ii. P(Blue and Red) ; is , find:
= P(B and R) i. the value of n;
= × = ii. the probability that one is red and the other
is white.
Exercises 10.9
1. A bag contains 3 green balls and 6 yellow Selection Without Replacement
balls. A person draws a ball at random from the “Selection without replacement” means
bag, notes its colour and puts it back. This is selecting an item from a lot without putting it
done three times. What is the probability of back from where it was taken.
drawing two green balls and one yellow ball
( in any order) Generally, if a selection is done without
replacement,
2. Two white balls and six red balls were 1. The number of balls or items in the various
placed in a bag. A ball was taken at random groups reduces.
from the bag, its colour was noted and the ball 2. The total number of items in the lot (Sample
was replaced. A second ball was then drawn space) reduces.

Baffour Ba Series, Further Mathematics for Schools Page 293


3. The probability of subsequent draws P(2 blue pens and 1 green pen);
depends on the previous selection. = × × =

Worked Examples
3. A box contains 10 red bulbs and 15 green
1. A box contains 4 choco bars and 4 ice
bulbs of the same type and size. A bulb is
creams. Tom eats 3 of them, by randomly
selected at random and is not replaced, a
choosing. What is the probability of choosing 2
second ball is then selected at random, find the
choco bars and 1 ice cream?
probability that:
i. both bulbs are red,
Solution
ii. both bulbs are green,
n (C) = 4, n(I) = 4, n(S) = 4 + 4 = 8
iii. they are of the same colour;
P( 1 chocobar) = = iv. they are of different colours;
v. the first ball is green and the second is red.
Now, n (C ) = 3 , n (I) = 4 and n(S) = 7
P (2nd chocobar ) = Solution
i. n(R) = 10, n (G) = 15 and n(S) = 25
Now, n (C ) = 2 , n (I) = 4 and n(S) = 6 P (First R) =
P (1 icecream) = =
Now, n(R) = 9, n (G) = 15 and n(S) = 24
P(2 chocobars and 1 ice cream); P (Second R) =
= × × = P(both balls are red);
= P( First R and Second R)
2. A pack contains 4 blue, 2 red and 3 green = × =
pens. If 2 pens are drawn at random from the
pack, not replaced and then another pen is ii. Probability that both balls are green;
drawn. What is the probability of drawing 2 n(R) = 10, n (G) = 15 and n(S) = 25
blue pens and 1 green pen?
P (First G) =

Solution: Now, n(R) = 10, n (G) = 14 and n(S) = 24


n(B) = 4, n(R) = 2, n(G) = 3 and n(S) = 9 P (Second R) =
P(first blue pen) =
P(both balls are green);
Now, n(B) = 3, n(R) = 2, n(G) = 3 and n(S) = 8 = P( First G and Second G)
P(second blue pen) = = × =
Now, n(B) = 2, n(R) = 2, n(G) = 3 and n(S) = 7
iii. Probability that both balls are of the same
P(1 green pen) =
colour;

Baffour Ba Series, Further Mathematics for Schools Page 294


= P( 1st R and 2nd R) or P( 1st G and 2nd G) By substitution,
=. /+. / P( ∩ ) = P( ) ( )

= + = = × =

iv. P(different colours); ii. Let P( ) be the probability of the first


= P(R and G) or P(G and R) black marble and P( ) e the probability of
=. / +. / the second black marble;
P( ∩ ) = P( ) ( )
= + =
But P( )= and P( )= =

v. P(first green and the second red); P( ∩ )= P( ) ( )


n(R) = 10, n (G) = 15 and n(S) = 25 = × =
P (First G) =
iii. Two marbles of the same color;
Now, n(R) = 10, n (G) = 14 and n(S) = 24 = P( ∩ ) or P( ∩ )
P (Second R) = = [P( ) ( )- + [P( ) ( )]

P(first green and the second red); =. /+. /


= P( First G and Second R) = + =
= × =
5. A box contains 3 blue and 2 red balls and
4. A box contains 10 marbles, 7 of which are another contains 2 blue and 1 red ball. If a ball
black and the rest red. Two marbles are drawn, is chosen at random from each box, without
one after the other without replacement. Find replacement, find the probability that:
the probability of getting: i. both balls are blue;
i. a red and a black marble, ii. one ball is blue and the other is red.
ii. two black marbles,
iii. two marbles of the same color. Solution
i. Let the boxes be A and B respectively.
Solution For Box A;
i. n(S) = 10 n(B) = 3, n (R) = 2 and n(S) = 5
Number of black marbles = 7
Number of red marbles = 10 – 7 = 3 For Box B;
Let P( ) be the probability of the first red n(B) = 2, n(R) = 1 and n(S) = 3
marble and P( ) the probability of the second
black marble; P(Both balls are red);
P( ∩ ) = P( ) ( ) = P( and )
But P( )= and P( )= = = P( ) × P( )

Baffour Ba Series, Further Mathematics for Schools Page 295


= × = i. the value of x;
ii. the probability the one is white and one is
red.
ii. P(one blue and the other is red)
= P( and ) or P( and )
Solution
= P( ∩ ) + P( ∩ )
n(S) = x, n(W) = 4, n (B) = 5
= . /+. / n (R) = x – 4 – 5 = x – 9
= × = If the probability of selecting two white balls is
,
6. A box A contains 3 whute and 2 blue balls. ⇒ P(W ∩ W) × =
Another box B contains 4 green and 5 blue
=
balls. A ball is picked at random from each ( )
box, find the probability that: 12 × 55 = 6x(x – 1)
i. one is green and the other is white; 660 = 6x2 – 6x
ii. they are of the same colour. 0 = 6x2 – 6x – 660
x2 – x – 110 = 0
Solution (x – 11) (x + 10) = 0
i. For Box A; x = 11 or x = - 10
n(W) = 3, n (B) = 2 and n (S) = 5 x = 11

For Box B; ii. n(S) = 11, n(W) = 4, n (B) = 5


n(G) = 4, n (B) = 5 and n (S) = 9 n (R) = 11 – 4 – 5 = 2
P (one is white and one is red);
P(one is green and the other is white)
P (W ∩ R) = × = =
= P(G ∩ W)
= × = Some Solved Past Questions
1. A bag contains 6 red, 8 black and 10 yellow
ii. P(they are of the same color); identical beads. Two beads are picked at
= P( and ) random, one after the other without
= P( ) × P( ) replacement. Find the probability that:
= × = a. both are red,
b. one is black and the other yellow.
7. A box contains x balls of which 4 are white,
Solution
5 blue and the rest are red. Two balls are
a. n (R) = 6 , n(B) = 8 and n(Y) =10
selected at random one after the other without
n(S) = 6 + 8 + 10 = 24
replacement. If the probability of selecting two
P(R1) =
white balls is , find:

Baffour Ba Series, Further Mathematics for Schools Page 296


Without replacement, Exercises 10.10
n( R) = 5 and n (S) = 23 A. 1. A bag contains 5 red apples and 3 yellow
P(R2) = apples. What is the probability of picking two
red apples without the first being replaced?
P(R1 and R2) = P(R1 ∩ R2)
2. A bag contains 8 white 3 blue and 6 green
= P(R1) × P(R2)
counters. Two counters are darwn randomly
= × = = from the bag. Find each probailty if the counter
is not replaced before the second counter is
b. P(R1 and Y2) = P(R1∩Y2) drawn.
= P(R1) × P(Y2) i. P (2 green) ii. P (blue, white)
= × = =
3. A variety of box of drinks contains 10 cans
of coke, 6 cans of fanta amd 4 cans of lemon.
2. A box P contains 3 white and 4 green balls.
Tom pulls out one can and drinks it. His friend,
Another box Q contains 5 white and 2 green
Jerry also pulls out one can and drinks it. What
balls. A ball is drawn at random from box P
is the probability that the first was lemon and
and dropped into box Q. A ball is then drawn at
the second was coke?
random from box Q. Find the probability that
the ball drawn from box Q is white.
4. Two chips are selected from a box
Solution containing 6 blue chips, 4 red chips and 3 green
i. For Box P; chips. The first chip selected is not replaced
n(W) = 3, n (G) = 4 and n (S) = 7 before the second is drawn. Find P (red, green).

For Box Q; B. 1 A box contains x balls of which 4 are


n(W) = 5, n (G) = 2 and n (S) = 7 white, 5 blue and the rest are red. Two balls are
selected at random one after the other without
A ball is drawn at random from box P and replacement. If the probability of selecting two
dropped into box Q. white balls is , find:
Box P; n (S) = 6 i. the value of x;
Box Q; n (S) = 8 ii. the probability the one is white and one is
red.
P(ball drawn from box Q is white)
= P( ∩ ) + P( ∩ ) 2. A box contains 8 balls, of which x are red
=. /+. / and 5 are white. Two balls are selected at
random one after the other without
= ×
replacement. If the probability of selecting two
= red balls is , find:

Baffour Ba Series, Further Mathematics for Schools Page 297


i. the value of x; stage in the diagram. For example, for two
ii. the probability the one is red and the other is actions, there will be two actions as shown
white. below:

3. Three bags P, Q and R contains red, blue 2


1
and white balls respectively of equal sizes. The
raio of the ball in the bag are P : Q = 2 : 3, and
Q : R = 4 : 5. All the balls are removed into a
big bag and properly mixed together.
III. At the end of each route along the branches
of the tree, the final outcome is written.
a. Find the probability of picking a:
i. red ball ii. blue ball iii. white ball
IV. Work out the probability of each outcome
b. If two balls are picked at random one after
shown by a branch on the diagram. Write each
the other without replacement, find the
probability on it branch of the tree, not
probability of picking:
forgetting the fact that probabilities on adjacent
i. a white ball and a blue ball
branches add up to 1.
ii. blue ball first, then a red ball.
iii. both red or both blue or both white.
V. Determine the probability of the final
outcome by finding the route that leads to the
4. A box contains 6 red, 8 black and x yellow
final outcome. Multiply together the
identical balls. Two balls are picked at random
probabilities from any branch as you go along.
one after the other without replacement. If the
probability of selecting one black and the other
Worked Examples
yellow is , find: 1. A fair coin is tossed twice. List all the
i. the value of x; possible outcomes using a tree diagram.
ii. the probability that both are yellow;
iii. the probability that both are black or both Solution
nd
are yellow or both red. 2 Coin
st
1 Coin H
Probability and Tree Diagrams H T
A tree diagram called probability tree diagram, H
can be used to solve problems involving T
combined or compound events. T
Steps: Possible outcomes = {HH, HT, TH, TT}
I. For each action in the problem, show the
possible outcomes (or events) at the end of a 2. With the aid of a tree diagram, list all the
branch of the tree possible outcomes when a coin is tossed three
II. Each action in the problem is shown by a times

Baffour Ba Series, Further Mathematics for Schools Page 298


Solution A pen is taken at random from the bag with
rd
3 Coin replacement. A second pen is then taken from
nd
2 Coin H the bag. What is the probability that:
st
H i. both pens are red?
1 Coin T
H ii. both pens are of the same color?
H
T
T Solution
H
H i. Number of red pens (R) = 5
T
T Number of blue pens (B) = 2
T H Total number of pens = 5 + 2 = 7
T P(R) = and P(B) =
nd
2 action
Possible outcomes = {HHH, HHT, HTH, HTT, R
st
THH, THT, TTH, TTT} 1 action

R
B
3. One element is drawn at random from each
of the three sets, A = {a, b}, B = {c, d, e}, C =
B
{f, g}. Use tree diagram to list all possible
outcomes B
R
Solution
f The probability that both pens are red;
c g P(R∩R) = P(R) × P(R)
f = =
d g
a f
e ii. The probability that both pens are of the
g
same color;
f
c g P(R∩R) or P(B∩B)
f = P(R) × P(R) + P(B) × P(B)
b d
g =. /+. /
f = + =
e

g 5. A bag contains 4 red, 5 blue and 7 green


Possible outcomes = {acf, acg, adf, adg, aef, marbles, which are all identical except for
aeg, bcf, bcg, bdf, bdg, bef, beg} color. A marble is selected at random, the color
is noted and it is not replaced in the bag. A
4. There are 5 red pens and 2 red pens in a bag. second marble is selected at random from those

Baffour Ba Series, Further Mathematics for Schools Page 299


remaining in the bag. Find the probability that 2. In two independent events, neither of the two
the marbles selected are: has an influence on the other. A game is played
i. both red, by rolling a die and then tossing a coin. You
ii. of the same color ? win if the die shows 3 or a 4 and the coins
shows heads.
Solution i. Use tree diagram to find the sample space.
i. R ii. What is your probability of winning?
B
R 3. Two balls are drawn without replacement
G
from a bag containing 12 similar balls, 4 red,
R 2 yellow and 6 blue.
B i. Represent the possible outcomes on a tree
B
diagram.
G ii. From the tree diagram, find the probability
R
of selecting two blue balls.
G
B iii. From the tree diagram, find the probability
of selecting one red and one yellow ball.
G
i. Probability that both marbles are red; 4. There are 5 red pens and 3 blue pens in a
P(R∩R) box. A pen is chosen at random from the box.
= × = Use tree diagram to find the probability that:
i. both pens are blue,
ii. both pens are of the same color.
ii. Probability that both marbles are of the same
color;
5. A coin is tossed three times in succession.
P(R∩R) or P(B∩B) or P(G∩G)
Draw the probability tree diagram for the
=. /+. /+. / various outcomes and hence write down the
probability of obtaining ;
= + +
i. two heads only,
= ii. one head only,
iii. no heads.

Exercises 10.11 6. A bag contains 5 white balls and 3 black


balls. One ball is drawn from the bag and a
1. Three coins are tossed once. Use the tree
second is then drawn. Find the probability of
diagram to list all the possible outcomes.
drawing one ball of each color.

Baffour Ba Series, Further Mathematics for Schools Page 300


11 VECTORS IN A PLANE Baffour Ba Series

Meaning of Vectors 2. Axial vectors : The vectors which act along


A vector is a quantity that has magnitude and the axis of rotation are called axial vectors.
direction. It is usually represented by a directed Exaplmes are angular velocity, angular
line segment. By magnitude, we mean the acceleration, angular momentum
quantity has distance coverage and by direction, 3. Inertial or Pseudo vectors
we mean the quantity can move in either
direction. Examples of vectors are force, Inertial or Pseudo vectors
velocity, displacement, acclearation etc. A Unit Vector
It is a vector whose magnitude is 1, and has any
Quantities such as area, volume, length given direction only. A unit vector is obtained
temperature, and time have magnitude only and by dividing he vector by its magnitude.
can be completely characterized by a single real
number (with appropriate unit). Such quantities Free Vector
are called scalar quantities. It is a vector which can be displaced parallel to
itself and applied at any other point.
Vector Representation
Consider the figures below; Position Vector
A vectoer that represents the position of a point
y Q with reference to a fixed point is called a
position vector.

P Negative Vector
x A vector with the same magnitude as another
A vector with initial point P and a terminal(end vector but opposite directions are called
negative vectors.
point) Q is represented as ⃗⃗⃗⃗⃗ or ⃖⃗⃗⃗⃗⃗. The
u Q
direction of the vector is indicated by placing Q
an arrow head at Q. P -v

P
A vector with no specific end points is also
represented by boldface letters such as u and v. The Null or Zero Vector
In hand written work, notations such as ⃗ and It is a vector whose magnitude is zero and has
is equally accepted. no direction. It is usually obtained by adding or
subtractinh two or more vectors.
Types of Vectors
In general, vectors may divided in to 3 types Like or Parallel Vectors
1. Proper vectors : Examples displacement, They are two vectors with the same magnitude
force, momentum etc but different directions.

Baffour Ba Series, Further Mathematics for Schools Page 301


Unlike Vectors Column Vectors
They are two vectors with unequal magnitudes It is a vector that is written in the form . /
and different directions.
Magnitude of a Vector
Co – Initial Vectors If x and y are the components of a vector a (i.e.
If two or more vectors have a common initial
a = . /, then the magnitude of a, written as:
point, then these types of vectors are called co –
initial vectors | |=√

Collinear Vectors Worked examples


Two or mor vector are collinear if they have the 1. i. Given that a = ( ), b = ( ) and c =
fall on the same line or are parallel. ⁄
( *, find the magnitude of each vector.

Co – Planer Vectors
They are two or more vectors lying in the same Solution
plane. ⁄
a = ( ), b = ( ) and c = ( *,

Equal Vectors | | = √( ) =√ =√ units
Vectors that have the same magnitude and
direction are said to be equal. Equal vectors are
| | = √( ) ( ) =√ = 2 units
also said to be equivalent.

u Q | | = √. / . / =√ =√ = 1 unit
Q
P v

P Addition of Vectors
Given that a = . / and b = . /, then
In mathematics, a vector is determined only by
its magnitude and direction, not by its location. a + b =. /
Thus, equivalent vectors are regarded as equal a
so the figures above can be written as u = ⃗⃗⃗⃗⃗ , v a

= ⃗⃗⃗⃗⃗ and u = v. In short, a vector can be


translated from one location to another, a+b
provided neither the magnitude nor direction is
changed. Worked Examples
Given that a = ( ), b = ( ), c = ( ) and
Some of these types of vectors involving d = ( ), find:
calculations are further treated in this course 1. a + b 2. c + d

Baffour Ba Series, Further Mathematics for Schools Page 302


Solution 8. 1a = a
1. a + b = ( ) + ( ) = ( ) 9. 0a = 0
2. c + d = ( ) + ( ) = ( )
Scalar Multiplication
To find the scalar multiplication of a vector,
Subtraction of Vectors
multiply each component of the vector by the
Given that a = . / and b = . /, then
scalar. That is, if a = . / and k is a scalar, then
1. a – b = . /
ka = k . / = . /.
2. b – a = . /
Worked Examples
Illustration Given that a = ( ) and b = ( ), find:
y R 1. 5a 2. 3b 3. 5a + 3b
a–b 4. 3b – 5a 5. ( )
b
Q
a Solution
P
x 1. 5a = 5 ( ) = ( )=( )
O

Worked Examples 2. 3b = 3 ( ) = ( )=( )


Given that a = ( ) and b = ( ) 3. 5a + 3b = 5 ( ) + 3 ( )
Find: = ( )+ ( ) =( )
1. a – b 2. b – a
4. 3b – 5a = 3 ( ) – 5 ( )
Solution
=( )–( )=( )
a = ( ) and b = ( )
1. a – b = ( ) – ( ) = ( )
5. ( )= [ ( ) ( )]

2. b – a = ( ) – ( ) = ( ) = [( ) ( )] = ( )=4 5=( )

Properties of Addition and Subtraction


Relating Free Vectors and Position Vectors
1. a + b = b + a
2. a + (b + c) = ( a + b) + c
B ⃗⃗⃗⃗⃗ = ⃗⃗⃗⃗⃗ − ⃗⃗⃗⃗⃗
3. a + 0 = a
4. a + (- a) = 0 ⃗⃗⃗⃗⃗ = b – a
5. m(a + b) = ma + mb A
6. (m + n) a = ma + na
7. (mn)a = m(na) = n(ma) O

Baffour Ba Series, Further Mathematics for Schools Page 303


For all free vectors such as ⃗⃗⃗⃗⃗ , ⃗⃗⃗⃗⃗ , ⃗⃗⃗⃗⃗ , ⃗⃗⃗⃗⃗ ⃗⃗⃗⃗⃗ = ⃗⃗⃗⃗⃗ – ⃗⃗⃗⃗⃗
etc, the following generalizations can be made: ⃗⃗⃗⃗⃗ = ( ) – ( ) = ( )
(i) ⃗⃗⃗⃗⃗ = ⃗⃗⃗⃗⃗ − ⃗⃗⃗⃗⃗ (ii) ⃗⃗⃗⃗⃗ = ⃗⃗⃗⃗⃗ − ⃗⃗⃗⃗⃗
(iii) ⃗⃗⃗⃗⃗ = ⃗⃗⃗⃗⃗ − ⃗⃗⃗⃗⃗ (iv) ⃗⃗⃗⃗⃗ = ⃗⃗⃗⃗⃗ − ⃗⃗⃗⃗⃗ b. ⃗⃗⃗⃗⃗ = ⃗⃗⃗⃗⃗ – ⃗⃗⃗⃗⃗
⃗⃗⃗⃗⃗ = ( ) – ( ) = ( )
Worked Examples
1. Given that a = (3, 6), b = (-7, 2), c = (2, 5) / ⃗⃗⃗⃗⃗ / = √ =√ = 10.05 units
and d = ( ). Find:
(a) ⃗⃗⃗⃗⃗ (b) ⃗⃗⃗⃗⃗ (c) ⃗⃗⃗⃗⃗⃗ + ⃗⃗⃗⃗⃗ Exercises 11.1
1. Find ⃗⃗⃗⃗⃗ for the points P(-2, 3), Q(4, 7)
Solution
1. (a) ⃗⃗⃗⃗⃗ = ⃗⃗⃗⃗⃗ − ⃗⃗⃗⃗⃗ 2. A is 2i – 5j, B is 6i + 12j, C is -3i – 10j and
D is 8i. Convert in component forms the
=( )–( )=( )=( )
vectors:⃗⃗⃗⃗⃗⃗ , ⃗⃗⃗⃗⃗⃗ , ⃗⃗⃗⃗⃗ n ⃗⃗⃗⃗⃗

(b) ⃗⃗⃗⃗⃗ = ⃗⃗⃗⃗⃗ ⃗⃗⃗⃗⃗


The Zero Vector
=( )−( )=( )=( ) It is a vector with both components zero. For
example a = ( )
(c) ⃗⃗⃗⃗⃗⃗ + ⃗⃗⃗⃗⃗ = ⃗⃗⃗⃗⃗⃗ − ⃗⃗⃗⃗⃗ + ⃗⃗⃗⃗⃗ − ⃗⃗⃗⃗⃗
= ( ) −( ) + ( ) − ( ) Worked Examples
=( )+( )= ( ) Given that a = ( ) and b = ( ), what type of
vector is a + b?
2. The position vectors of three points A, B and
C are 2i + 4j, 3i – 4j and 4i + 6j respectively. Solution
Find; a = ( ) and b = ( ),
a. ⃗⃗⃗⃗⃗ and ⃗⃗⃗⃗⃗ b. / ⃗⃗⃗⃗⃗ / a+b=( )+( )
a+b=( )
Solution
a + b is a zero vector.
2i + 4 j, 3i – 4j and 4i + 6j
⃗⃗⃗⃗⃗ = 2i + 4 j = ( )
The Negative of a Vector
⃗⃗⃗⃗⃗ = 3i – 4j = ( )
Given that vector a = . / , then the negative of
⃗⃗⃗⃗⃗ = 4i + 6j = ( )
a written as; – a = - . / = . /
⃗⃗⃗⃗⃗ = ⃗⃗⃗⃗⃗ – ⃗⃗⃗⃗⃗ The negative of a vector is also called the
inverse or opposite vector.
⃗⃗⃗⃗⃗ = ( ) – ( ) = ( )

Baffour Ba Series, Further Mathematics for Schools Page 304


Worked Example Now, let i = ( ) and j = ( ) and put in eqn (1)
1. If a = ( ) , find the negative of a. ⇒ a = xi + yj
(x, y) (x, y)
Solution
a=( ) a a
yj
-a=-( )
j
-a=( ) x x
0 i 0 xi
2. Given that u = ( ) and v = ( ) and that The vector sum xi + yj is a linear
m = 4u – v, find: combination of i and j.
i. the negative of m ii. | |
Worked Examples
Express the following in i and j form.
Solution 1. (5, 2) 2. (4, -3) 3. (0, - 4)
u = ( ) and v = ( )
m = 4u – v Solution
1. (5, 2) = 5i + 2j
m = 4( ) – ( ) = ( ) – ( ) = ( )
2. (4, -3) = 4i + (- 3)j = 4i – 3j
3. (0, - 4) = 0i + (- 4)j = - 4j
m= ( )=( )
- m=–( )= ( ) Exercises 11.2
Express the following in i and j form.
1. (7, 3) 2. (-10, -3) 3. (1, - 1)
ii. | | = √( ) ( ) 4. (0, -5) 5. ( 9, 0) 6. (0, 0)
=√
=√ Linear Combination of i and j Vectors
Below are the rules for addition, subtraction
The i and j Form of Vectors and multiplication by scalar k, given that a =
A unit vector is a vector of magnitude 1. The (x1 i + y1 j) and b = (x2 i + y2 j):
vectors i and j are unit vectors as in the vector c 1. (a + b) = ( x1 i + y1j) + (x2 i + y2 j):
= . , /, since it has a magnitude of 1. = (x1 + x2) i + (y1 + y2) j :

2. (a - b) = ( x1i + y1j) - (x2 i + y2j):


The vectors i and j can be used to obtain an
= (x1 - x2) i + (y1 - y2) j
alternative way of denoting vectors. Thus, if
a = . /, then a = ( ) + . / 3. ma = m(x1 i + y1j)
a = x ( ) + y ( )………….(1) = (mx1) i + (my1) j

Baffour Ba Series, Further Mathematics for Schools Page 305


Worked Examples 2. If a = -5i + 2j and b = i – 3j, find | ( )|
1. If p = 3i + j and q = -5i + j, find p + q.
Finding the Magnitude of i and j Vectors
Solution Given the vector a = (x, y) = (xi + yj),
p = 3i + j and q = -5i + j, 1. make a sketch to know the quadrant in which
p + q = 3i + j + -5i + j the vector falls.
p + q = 3i + j - 5i + j 2. the magnitude of a is calculated as:
p + q = 3i – 5i + j + j
/a/ = √
p + q = -2i + 2 j

Finding the Direction of a given Vector


2. If a = 5i + j and b = 4i – 7j, express 3a – 2b
1. The direction of a is calculated as ;
as a linear combination of i and j.
Solution θ = . /, where θ is the angle at the
3a – 2b = 3(5 i + j) – 2( 4 i – 7j) horizontal.
= (15 i + 3j) – (8 i – 14j) 4. Avoid negative signs in the calculation of the
= 15 i + 3j – 8 i + 14j direction because they indicate the quadrant in
= 15 i – 8 i + 3j + 14j which the vector falls.
= 7 i + 17j 5. Take measurement from the north pole in the
clockwise direction. The angle turned through
3. Two vectors are given by a = ( ) and to locate the vector gives the direction of the
b = ( ) . Find | | vector.

Worked Examples
Solution
1. If p = 3i + j and q = -5i + j, find the
a = ( ) and b = ( ) .
magnitude and direction of (p + q).

a – 2b = ( ) – 2 ( ) Solution
a – 2b = ( ) – ( ) p = 3i + j and q = -5i + j,
a – 2b = ( ) p + q = 3i + j + -5i + j
p + q = 3i + j - 5i + j
| | = √8 ( ) p + q = 3i – 5i + j + j
| |=√ p + q = -2i + 2 j
| |=√
| | = 10.63 (2 d.p) Magnitude of p + q;
/p + q/ = √( )
Exercises 11.3 /p + q/ = √
1. Given that a = i + 2j and b = 3i – 5j, find the
/p + q/ = √8
following;
/p + q/ = 2.8284 units
i. 4a – 5b ii. /a – b/

Baffour Ba Series, Further Mathematics for Schools Page 306


Direction of (p + q) = 900 + 900 + 900+ θ; Substitute in tan θ =
θ= . / tan θ =
θ= . / p+q tan θ =
0
θ = 45
tan θ =
θ
Direction of (p + q) ; θ= . /
= 900 + 900 + 900+ θ; θ = 590
= 900 + 900 + 900 + 45 Vector ⃗⃗⃗⃗⃗ has a direction of about 0590
= 3150;
p + q = (3 units, 3150)
4. Find the magnitude and direction of the
vectors - 10j.
2. Find the magnitude and direction of the
vectors -5i + 12j.
Solution
Let v = - 10j = (0, -10)
Solution
/v/ = √ ( )
Let v = - 5i + 12j
/v/ = √
/v/ = √( )
/v/ = 10 units
/v/ = √
/v/ = √ v Direction of v;
/v/ = 13 units From the diagram,
θ Direction of v = 900 + θ
From the diagram,
But θ = . /
Direction of v ,
0
= 900 + 900 + 900 + θ θ=0

But θ = . /
Therefore, direction of v;
0
θ = 67 v = 900 + 00
v = 900
Therefore, direction of v ; v = (10 units, 900)
v = 900 + 900 + 900 + 670
v = 3370 Exercises 11.4
v = (13 units, 3370) Find the magnitude and direction of the
following vectors.
3. Find the direction of vector PQ whose initial 1. a = - 4 i + 5j 4. a = - 2 i – 2√ j
point P is at (2, 3) and end point Q is at (5, 8)
2. a = 10 i – 10 j 5. a = 3 i – 3 j
3. a = - 18 j 6. a = - 5i
Solution 4. i – j 8. a = - 12j
P (2, 3) and Q (5, 8)

Baffour Ba Series, Further Mathematics for Schools Page 307


The Values of Scalars in Equal Vectors 22 = 6m + 46n……………(4)
Given a vector equation with two different
eqn (4) – eqn (3);
scalars, such as m. / + n . / = ( ), where m
7 = 28n
and n are scalars, the values of the scalars (m
n=
and n) are found as follows:
1. Expand the given equation n=
2. Equate the x component of the L.H.S of the
equation to the x – component of R.H.S. and Put n = - n = in eqn (1);
name it as eqn (1)
3. Similarly, equate the y component of the 5 = 2m + 6. /
L.H.S of the equation to the y – component of 5 = 2m +
R.H.S. and name it as eqn (2) 20 = 8m + 6
4. Solve the pair of equations obtained 14 = 8m
simultaneously to obtain the values of the m=
scalars
m=

Worked Examples (m, n) = . , /


1. The position vectors of the points P, Q, R
and S relative to a fixed point O are p = 2i + 3j, Exercises 11.5
q = 6i + 23j, r = 2i – 7j, s = 7i + 4j. Find the 1. If p = ( ), q = ( ) and r = ( ), find the
scalars m and n such that s = mp + nq + r values of the constants x and y such that
2p = 5xy – 3yr
Solution
s = mp + nq + r 2. If a = 2i + 3j, b = 4i + 7j and c = 8i + 11j ,
7i + 4j = m (2i + 3j) + n (6i + 23j) + (2i – 7j) find:
( ) = m ( ) + n ( ) + ( ), i. m and n such that c = ma + nb, where m
( ) – ( ), = m ( ) + n ( ) and n are scalars.
( )= m( )+n( ) ii. if d = c – 2a
( )= m( )+n( )
3. The vectors p = 2i + 3j, q = 2i + 5j and
( )= ( )+( )
r = ( q- p)

⇒ 5 = 2m + 6n………...…….(1) i. find the vector r.


11 = 3m + 23n……………(2) ii. if mp + nq = ( ), find m and n, where m and
n are scalars.
eqn (1) × 3;
15 = 6m + 18n……………...(3) 4. If p = ( ), q = ( ) and r = ( ), find:

eqn (2) × 2;
Baffour Ba Series, Further Mathematics for Schools Page 308
a. m and n, such that r = mp + nq, where m and a = /a/ (cos θ i + sin θ j)
n are scalars.
b. find│g│, if g =3q + r Method 2
Consider the diagram below:
Horizontal and Vertical Components of a
y
Vector a = . /
(x, y)
Method 1
Consider the diagram below: a
y
y θ
(x, y) x x
,
a Let θ be an angle in standard position measured
θ
from the positive y – axis in the clockwise
x direction to the vector;
a = (x, y) = x i + y j
Let θ be an angle in standard position measured From the diagram, (when the right triangle is
from the positive x – axis in the anticlockwise obtained at the right), we consider θ as the
direction to the vector; acute angle the vector makes with the x – axis
a = (x, y) = x i + y j in the very quadrant that the vector falls.
Hence,
From the diagram,
cos θ = …………………...(1)
cos θ = …………………..(1)
sin θ = ……………………(2)
sin θ = ………………….(2)
But from the diagram; a = / a /
But from the diagram; a = / a / eqn (1) and eqn (2) becomes;
eqn (1) and eqn (2) becomes; cos θ = ………………….(1)
cos θ = ………………….(1)
sin θ = ………………….(2)
sin θ = ………………….(2)
From eqn (1) and eqn (2);
From eqn (1) and eqn (2); x = /a/ cos θ
x = /a/ cos θ y = /a/ sin θ
y = /a/ sin θ
Using these formulas,
Using these formulas, 1. a = (x, y) in the first quadrant;
a = (x, y) a = (/a/ cos θ, /a/ sin θ)
a = (/a/ cos θ, /a/ sin θ) a = (/a/ cos θ i + /a/ sin θ j)
a = (/a/ cos θ i + /a/ sin θ j) a = /a/ (cos θ i + sin θ j)

Baffour Ba Series, Further Mathematics for Schools Page 309


2. a = (- x, y) in the second quadrant; a = (- 25 cos 600) i + ( - 25 sin 600) j
a = ( – /a/ cos θ, /a/ sin θ) a = 12.5 i – 21.6506 j
a = (– /a/ cos θ i + /a/ sin θ j) a 13i – 22 j
a = /a/ (– cos θ i + sin θ j) x
600 0
30
3. a = (- x,- y) in the third quadrant; 25

a = (– /a/ cos θ, – /a/ sin θ) a


a = (– /a/ cos θ i – /a/ sin θ j) y
a = – /a/ (cos θ i + sin θ j) Method 2
a = (25 sin 2100) i + (25 cos 2100) j
4. a = (x, - y) in the fourth quadrant; a = 12.5 i - 21.6506 j
a = (/a/ cos θ, – /a/ sin θ) a 13i – 22 j
a = (/a/ cos θ i – /a/ sin θ j)
a = /a/ (cos θ i – sin θ j) Velocity Vector
A velocity vector represents the rate of change
Method 3 of the position of an object. The magnitude of a
Worked Examples velocity vector gives the speed of an object
1. Express v = (12km, 0630) as a standard basis while the vector direction gives its direction.
vector.
Worked Examples
Solution If the wind is blowing at 12m/h in the direction
Method 1 S500E, express its velocity as a vector v.
v = (12km, 0630)
v = (12 cos 270) i + (12 sin 270) j Solution
v = 10.6921 i + 5.4479 j θ = 900 + 400 = 1300
v 11i + 5j
Using the formula for horizontal and vertical
Method 2 component,
v = (12 sin 630) i + (12 cos 630) j v = (x, y) = (x i + y j)
v = 10.6921 i + 5.4479 j But x = /v/ cos θ
v 11i + 5j x = 12 cos 1300
0
40
2. Find the x and y components of a 25m y = /v/ sin θ 500
displacement at an angle of 2100. Express x and y = 12 sin 1300
y in standard basis form. v
⇒v = (x, y)
Solution v = (x i + y j)
Method 1 v = (12 cos 1300) i + (12 sin 1300) j
Let the vector be a = (25m, 2100) v = -7.7 i + 9.2 j

Baffour Ba Series, Further Mathematics for Schools Page 310


Method 2 component is 57.34 feet per second and the
vertical component is 40.15 feet per second.

0
Method 2
40
i. Let v represent the velocity
500
Express v in unit vector form;
v = (70 sin 550) i + (70 cos 550) j
From the diagram, the vector falls in the second v = 57.34i + 40.15j
quadrant. Therefore;
v = (-x , y) = (-xi + yj) 4. The vector ⃗⃗⃗⃗⃗ has magnitude 5 units and its
inclined at 1500 to the x – axis. Express ⃗⃗⃗⃗⃗ in
Angle the vector makes with x – axis, the form ai + bj, where a , b R.
θ = 500
Solution
But x = – /v/ cos θ ⃗⃗⃗⃗⃗ = (5 sin 1500) i + (5 cos 1500) j
x = – 12 cos 500
⃗⃗⃗⃗⃗ = 2.5i – 4.3j
y = /v/ sin θ
y = 12 sin 500 600
5
⇒v = (- x, y)
v = (- x i + y j)
v = ( -12 cos 500) i + (12 sin 500) j
The Force Vector
v = ( -12 × 0.6428) i + (12 × 0.76600) j
A force has both magnitude and direction.
v = -7.7 i + 9.2 j
Therefore, a force is a vector quantity. Its unit
is Newtons (N).
5. A ball is thrown with initial velocity of 70
feet per second, at an angle 350 with the
Worked Examples
horizontal. Find the vertical and horizontal
A force of 100 N is acting at a point making an
components of the velocity.
angle of 300 with the horizontal. Determine the
components of this force along X and Y
Solution
directions.
Method 1
i. Let v represent the velocity
Solution
Express v in unit vector form;
Force, F = 100 N
V = 70 ( cos 350) i + 70 (sin 350) j
Angle made by F with horizontal, θ = 30°
V = 57.34i + 40.15j
Let Fx = Component along x-axis
Since the scalars are the horizontal and vertical
Fy = Component along y-axis
components of v, it implies that the horizontal

Baffour Ba Series, Further Mathematics for Schools Page 311


Unit Vector
y A unit vector is a vector that has a magnitude of
1. They are usually labeled with a cup “˄” on
top of the given vector. Thus, the unit vector of
is wirten as ̂.
F = 100N

To find the unit vector of a given vector;


300 = (x, y);
x 1. Find the magnitude of the vector as;
Then: /⃗ / = √
Fx = F cos θ
F = 100 cos 30°
2. The unit vector =
F = 100 × 0.866

F = 86.6 N. Mathematically, ̂ = ⃗

Fy = F sin θ
Worked examples
Fy = 100 sin 30°
1. Given the vector = 12i - 5j, find the unit
Fy = 100 × 0.5
vector of a in component form.
Fy = 50 N.
Solution
Exercise 11.6
= 12i – 5j ,
1. Consider two forces of magnitudes 7N and
= (12, - 5)
8N acting on a particle as shown in the diagram
below; / /=√ ( )
/ /=√
7 / /=√
/ / = 13
8
⃗ ( , )
̂= ⃗
= =. , /
What is the magnitude and direction of the
resultant force? Ans 11N, 2210
2. Given the vector ⃗ = (-2, - 4), find the unit
7 vector of a in component form.
2. Consider two forces 9
of magnitudes 7N and 9 N
Solution
acting on a particle as 600 300
⃗ = (-2, - 4)
shown in the diagram above;
What is the magnitude and direction of the / ⃗ / = √( ) ( )
resultant force? /⃗ / = √
/⃗ / = √

Baffour Ba Series, Further Mathematics for Schools Page 312


Unit vector, ̂=–. , /
⃗ ( , )
̂= = =. , / ̂=–( , )
⃗ √ √ √
̂ = (1, 0)
Exercises 11.7
If a = i + 3j, b = 2i – 5j, c = -2i + 4j, find: 2. Find the unit vector b in the opposite
a. the component form of the vectors: direction of = (5, -12), that has magnitude 6.
i. a + b iv. a + b – c
ii. b + c v. 3a + 2b Solution
iii. a – b = (5, - 12)
b. the magnitude of the vectors in (a); / / = √( ) ( )
c. unit vectors in the direction of a + b and / /=√
b+c / /=√
/ / = 13
Unit Vector in the Opposite Direction to a
given Vector
Let the unit vector be ̂ ,
Given the vector = (x, y), the vector, ̂ in the
The unit vector in the opposite direction is;
opposite direction to vector is calculated as:
̂=– ⃗ = ( , )
=. , /
1. Find the magnitude of the vector as ⃗

/⃗ / = √
Now, multiply ̂ by 6 to obtain a vector of
2. The unit vector ̂ in the opposite direction to magnitude of 6 in the opposite direction of a.
, calculated as; ⇒ 6̂

̂= = 6̂ = 6 . , /

̂=. , /
Worked Examples
1. Find the unit vector in the opposite direction 3. Find the unit vector of magnitude 3 in the
to vector given by its components = (-1, 0) opposite direction of A(4, 0) and B(2, 1).

Solution
Solution
= (-1, 0)
A(4, 0) and B(2, 1).
/ / = √( ) ⃗⃗⃗⃗⃗ = ⃗⃗⃗⃗⃗ – ⃗⃗⃗⃗⃗
/ /=√ ⃗⃗⃗⃗⃗ = ( ) – ( )
/ /=1
⃗⃗⃗⃗⃗ = ( )
⃗ ⃗⃗⃗⃗⃗ = (-2, - 1)
̂=– ⃗
( , )
̂= – /⃗⃗⃗⃗⃗ / = √( )

Baffour Ba Series, Further Mathematics for Schools Page 313


/⃗⃗⃗⃗⃗ / = √ = 2i + ( i – j) + ( i + j)
√ √
/⃗⃗⃗⃗⃗ / = √
= . /i+. /j
√ √ √ √
Let the unit vector be ̂,
=. /i +. / j
The unit vector in the opposite direction is; √ √
⃗⃗⃗⃗⃗
̂ =– = 7.6567i – 1.4142j
⃗⃗⃗⃗⃗
( , )
̂ =– Magnitude of the forces;

̂= . , /
√ √
= √( ) (– )
̂ =. , /
√ √
=√
= 7.8N
The unit vector with a magnitude of 3 in the
opposite direction implies 3 ̂;
Exercises 11.8
3̂ = . , / Find the unit vector that has;
√ √

3 ̂ = 3. , /=. , / i. the same direction as the vector a.


√ √ √ √
ii. opposite direction of the vector a.
1. -8i + 15j 4. a = (0, 6)
Solved Past Question 2. 5i – 3j 5. a = (-2, 3)
Three forces of magnitude 2N, 5N and 3N act 3. a = (2, -5) 6. a = (- 4, 4)
along the vectors i, i – j and i + j respectively. 4. a = (- 4, -2) 8. a = (3, -3)
Calculate, correct to one decimal place, the
magnitude of the resultant force. B. 1. Find the vector of magnitude 6 that has
the opposite direction of a = 4i – 7j
Solution
Let the force 2N along i be ; 2. Find the vector of magnitude 4 that has the
= 2 . / = 2 . / = 2i opposite direction of a = (2, -5)

3. Find a vector that has the same direction as


Let the force 5N along i – j be ;
(-6, 3) and;
=5( *= 5. /= ( i – j) i. twice the magnitude
√ ( ) √ √
ii. one – half the magnitude
Let the force 3N along i be
4. Find a vector that has the opposite direction
= 3 .√ /= 3. /= ( i + j)
√ √ of 8i – 5j and;
i. three times the magnitude;
Resultant force F; ii. one – third the magnitude.
= + +

Baffour Ba Series, Further Mathematics for Schools Page 314


The Dot (scalar) Product = (-5) (2) + (3) (6)
Let a = (x1, y1) = x i + y j and b = (x2, y2) = x2 i = -10 + 18
+ y2 j. The dot product of a and b denoted as =8
a . b is
a.b=( , ).( , )=( , ) +( , ) ii. a = 4i + 6j , b = 3i – 7j
a . b = (4i + 6j) . ( 3i – 7j)
The symbol a . b is read as “a dot b”. Dot = (4)(3) + (6) (-7)
product is also referred to a scalar product or = 12 – 42
inner product. Note that a . b is a real number = - 30
but not a vector.
Properties of Dot Product
Worked Examples If a, b and c are vectors and k is a real number ,
then;
Find a . b given;
1. a . a = ‖ ‖2
i. a = (-5, 3) and b = (2, 6)
ii. a = 4i + 6j , b = 3i – 7j 2. a . b = b . a
3. a . (b + c) = a . b + a . c
Solution 4. (ka) . b = k(a . b)
i. a = (-5, 3) and b = (2, 6) 5. 0 . a = 0
a . b = (-5, 3) . (2, 6)

Dot product or scalar product Cross product or vector product


If the product of two vectors is a scalar quantity,the If the product of two vectors is a vector quantity
product is called scalar product or dot product. then the product is called vector product or cross
product.
The dot product is defined by the relation: The cross product is defined by the relation:
A . B = AB cos θ A × B = AB sin θ
The scalar product obeys commutative law as The vector or cross product does not obey
A.B =B.A commutative law
A×B≠B×A
If two vectors are perpendicular to each other If two vectors are parallel to each other,their
then their scalar product is zero. vector product is zero.
A.B = 0 A×B=0

Baffour Ba Series, Further Mathematics for Schools Page 315


Theorem on Dot Product q = b + 3a,
If θ is the angle between two nonzero vectors a
q = ( ) + 3( )
and b , then

a . b = | | | | cos θ………………(1) q= . /+( )
From eqn (1), q=( )
cos θ = | || |
The angle between vectors p and q
Worked Examples p . q = | | | | cos θ
1. Find the angle between a = (4, -3) and b = (1, cos θ = | | | |
2)
( )( ) ( )( )
cos θ =
√ √
Solution
a = (4, -3) and b = (1, 2) cos θ =
√ √

cos θ = | || | cos θ =
√ √

( )( ) ( )( )
cos θ = cos θ = 0.0535
√ √
θ= (0.0535)
cos θ =
√ √ θ= (0.0535)
cos θ = θ = 87 0


cos θ =
Exercises 11.9

θ= . / A. Find the dot product of the two vectors
θ = 100.30 and the angle between the two vectors.
1. 4i – j and -3i + 2j
2. Find correct to the nearest whole number, the 2. 6j and - 4i
angle between vectors p and q if p = 2a – b and 3. 8i – 3j, and 2i – 7j
4. 9i and 5i + 4j
q = b + 3a, given that a = i + j and b = 3i – 4j
5. 4i – 7j and - 2i + 3j

Solution B. Given that a = (2, -3), b = (3, 4) and c =


a = i + j and b = 3i – 4j (-1, 5), find the following:
a = ( ) and b = ( ) 1. a . (b + c)
p = 2a – b 2. b . (a – c)
p = 2( ) – ( ) 3. (2a + b) . (3c)
p=( ) –( )
C. 1. The position vectors of the vertices A, B,
p=( )
C of a triangle are i + j, 2i – j and – i + j

Baffour Ba Series, Further Mathematics for Schools Page 316


respectively. Find < BAC correct to the nearest cos θ = | || |
degree.
= √
2. Find correct to the nearest degree, the angle - 4 (5√ ) = 5(-8i + 3m)
between the vectors a = ( ) and b = ( ) - 20√ ) = 15m – 40

3. Two vectors are defined as p = 6i + 8j and q Reduced to


= 8i + j, find: - 4 (√ ) = (-8 + 3m)
i. the magnitude and direction of p and q. - 4 (√ ) = 3m – 8
ii. the angle between p and q.
Squaring both sides;
4. Given that a = 3i – 4j and b = 6i + 4j, find ( √ ) =( – 8)
2 2
the angle between the two vectors, correct to 16 (4 + m ) = 9m – 48m + 64
the nearest degree. 64 + 16m2 = 9m2 – 48m + 64
16m2 – 9m2 – 48m + 64 – 64 = 0
Finding the Constants of a Vector given the 7m2 – 48m = 0
Angle Between two Vectors. m (7m – 48) = 0
1. Given that the angle between p = 4i + 3j and m = 0 and 7m – 48 = 0
q = -2i + mj is . /, find the constant m. m = 0 and 7m = 48
m = 0 and m =
Solution
Let θ be the angle between p and q 2. The cosine of the angle between the vectors
⇒θ= . /, x = 3i – j and y = 2i + mj is

. Find the values
cos θ = of m.
cos θ = | || |
………………..(1)
Solution
But p . q = ( 4i + 3j) (-2i + mj)
Let θ be the angle between x and y
p . q = -8i + 3m

cos θ =
| |=√ cos θ = | ………………..(1)
|| |
| |=√ But x . y = ( 3i – 3j) (2i + mj)
| |=5 x.y=6–m

| | = √( ) | |=√ ( )
| |=√ | |=√

From eqn (1); | |=√

Baffour Ba Series, Further Mathematics for Schools Page 317


| |=√ The vectors a and b are perpendicular if and
only if they lie on mutually perpendicular lines
From eqn (1); that passes through the origin.
cos θ = | | | |
Theorem on Parallel Vectors

= The vectors a and b are parallel if and only if
(√ )(√ )
the angle θ between them is either 0 or .
√ (√ √ ) = 5(6 – m)
√ = 30 – 5m Worked Examples
1. Given a = i – 3j and b = -2 i + 12 j
Squaring both sides;
i. Show that a and b are parallel.
(√ ) =( – )
ii. Find the scalar k such that b = ka
200 + 50m = 900 – 300m + 25m2
2

50m2 – 25 m2 + 300m + 200 – 900 = 0 Solution


25m2 + 300m – 700 = 0
i. a = i – 3j and b = -2i + 12j

Reduce to: a = ( , - 3) and b = (-2, 12)


m2 + 12m – 28 = 0
(m – 2 ) ( m + 14) = 0 . /( ) ( )( )
cos θ =
m = 2 or m = - 14 √ √

cos θ =
Parallel and Orthogonal Vectors √ √
y
cos θ =
A( , )
B( , ) cos θ = - 1
a θ= (-1)
0
θ b θ = 180 ( )
O x
ii. Since a and b are parallel, there is a scalar k
Let θ be the angle between two non-zero such that b = ka
vectors a and b. That is; - 2i + 12j = m. /
1. a and b are parallel if θ = 0 or θ =
= m i - 3mj
2. a and b are perpendicular if θ =

Equating coefficient of i and j;


The vectors a and b are parallel if and only if
they lie on the same line that passes through the ⇒ - 2i = k i and 12j = - 3kj
origin. In this case, , b = ma for some real - 2 = k and 12 = - 3k
number m. k = - 4 and k = - 4
b=-4a
Baffour Ba Series, Further Mathematics for Schools Page 318
Note that a and b have opposite directions, 2. If the vector xi +yj and 3i – j are
because k < 0. perpendicular, find y when x = 2.

Exercises 11.10 Solution


A. Show that the pair of vectors are parallel xi + yj and 3i – j
and determine whether they have the same If xi + yj and 3i – j are perpendicular,
or opposite directions. (xi +yj ) (3i – j) = 0
1. a = 6i + 18j and b = - 4i – 12j 3x – y = 0
2. a = 3i – 5j and b= i+ j
When x = 2
3. a = i + 6j and b = - 10i + 24 j 3(2) – y = 0
4. a = i + j and b=8i+6j 6–y=0
y=6
5. If ai – 4j is parallel to 2i – 6j, find the value
of a. 3. Given that a = 3i + xj and b = xi + 6j, find the
6. Given that a = 4i + 3j and b = 5i + xj, find value of x if a is perpendicular to b.
the value of x if a and b are parallel.
Solution
a = 3i + xj and b = xi + 6j
Theorem on Orthogonal Vectors
The vectors a and b are orthogonal a = ( ) and b = ( )

(perpendicular) if and only if a . b = 0


If a is perpendicular to b then a . b = 0
(3i + xj) ( xi + 6j ) = 0
Worked Examples
( ).( )=0
1. Show that the pair of vectors are orthogonal
i. i , j ii. 2i + 3j , 6i – 4j 3x + 6x = 0
10x = 0
Solution x=0
i. i , j
4. P(-2, 3), Q(10, 4), R(3, 12), S(4, 0) are four
i . j = (1, 0) . (0, 1)
points in a plane.
= (1) (0) + (0) (1)
i. Use the dot product of vecors to show that
=0+0
⃗⃗⃗⃗⃗ is perpendicular to RS.
=0
ii. Find , correct to the nearest degree the
measure of < PSQ.
ii. 2i + 3j , 6i – 4j
= (2i + 3j) . (6i – 4j) Solution
= (2) (6) + (3) (-4) P(-2, 3), Q(10, 4), R(3, 12), S(4, 0)
= 12 – 12
⃗⃗⃗⃗⃗ = ⃗⃗⃗⃗⃗⃗ – ⃗⃗⃗⃗⃗
=0

Baffour Ba Series, Further Mathematics for Schools Page 319


⃗⃗⃗⃗⃗ = ( ) – ( ) = ( ) cos θ =
(√ ) (√ )

⃗⃗⃗⃗⃗ = ⃗⃗⃗⃗⃗ – ⃗⃗⃗⃗⃗ θ = ( *


(√ ) (√ )
⃗⃗⃗⃗⃗ = ( ) – ( ) = ( ) θ = 119. 74
θ = 1100 (nearest degree)
If ⃗⃗⃗⃗⃗ is perdicular to ⃗⃗⃗⃗⃗ , then their dot
prodeuct is 0 5. The vertices of a parallelogram are at
P(2, 1) , Q(6, 3), R(5, 5), S(x, y).
⃗⃗⃗⃗⃗ . ⃗⃗⃗⃗⃗ = ( ) . ( ) = 12 + (-12) = 0 i. Find the coordinates of S.
Therefore, If ⃗⃗⃗⃗⃗ is perdicular to ⃗⃗⃗⃗⃗ ii. Find |⃗⃗⃗⃗⃗⃗⃗⃗|
iii. Draw the diagonals ⃗⃗⃗⃗⃗⃗⃗⃗
| | and ⃗⃗⃗⃗⃗⃗⃗⃗
| | to meet
b. at M and use dot product to determine the value
Q(10, 4), R(3, 12),
of angle PMS.

Solution
Method 1
P(-2, 3), θ P(2, 1) , Q(6, 3), R(5, 5), S(x, y).
S(4, 0)
Q(6, 3) R(5, 5)
From the diagram ,
⃗⃗⃗⃗ . ⃗⃗⃗⃗⃗ = / ⃗⃗⃗⃗ /. /⃗⃗⃗⃗⃗ / cos θ
But ⃗⃗⃗⃗ = ⃗⃗⃗⃗⃗ – ⃗⃗⃗⃗⃗ P(2, 1) S(x, y)
⃗⃗⃗⃗ = ( ) – ( ) = ( )
If PQRS is a parallelogram, then diagonals
⃗⃗⃗⃗⃗ = ⃗⃗⃗⃗⃗⃗ – ⃗⃗⃗⃗⃗ bisect each other;
⇒Mid-point of QS = mid-point of PR
⃗⃗⃗⃗⃗ = ( ) – ( ) = ( )
Mid-point of QS;
/ ⃗⃗⃗⃗ / = √( ) =√ =. , /
/⃗⃗⃗⃗⃗ / = √
/⃗⃗⃗⃗⃗ / = √ Mid-point of PR;
=. , /= . , /
⃗⃗⃗⃗⃗ ⃗⃗⃗⃗⃗⃗
cos θ =
⃗⃗⃗⃗⃗ ⃗⃗⃗⃗⃗ Now ,
( )( )
cos θ = . , / =. , /
(√ ) (√ )
⇒ = …………….(1)
cos θ =
(√ ) (√ )
= ………………..(2)

Baffour Ba Series, Further Mathematics for Schools Page 320


From eqn (1); ⇒ ⃗⃗⃗⃗⃗⃗ = s – m
6+x=7 =( )
x=7–6 =1
=( )
From eqn (2);
y+3 =6 |⃗⃗⃗⃗⃗⃗⃗⃗⃗| = √( )
y=6–3=3 |⃗⃗⃗⃗⃗⃗⃗⃗⃗| = 2.5 units
S(x, y) = S(1, 3) ⃗⃗⃗⃗⃗⃗ ⃗⃗⃗⃗⃗⃗⃗
cos θ = ⃗⃗⃗⃗⃗⃗ ⃗⃗⃗⃗⃗⃗

ii. Q(6, 3) and S(1, 3) ( ) ( )


cos θ = ( )(
/QS/ = √( ) ( ) )
( )( ) ( )( )
/QS/ = √ cos θ = ( )( )
/QS/ = √
/QS/ = 5 units cos θ =

iii. Q(6, 3) R(5, 5) θ= . /

M θ = 126.8699
θ = 1270 (nearest degree)
P(2, 1) S(1, 3)

Angle PMS is between PM and MS Exercises 11.11


Let < PMS = θ A. Show whether the following pair of
Using dot product; vectors are orthogonal or not.
⃗⃗⃗⃗⃗⃗ . ⃗⃗⃗⃗⃗⃗ = |⃗⃗⃗⃗⃗⃗⃗⃗⃗| |⃗⃗⃗⃗⃗⃗⃗⃗⃗| cos θ 1. 4i – j and 2i + 8j
⃗⃗⃗⃗⃗⃗ ⃗⃗⃗⃗⃗⃗⃗ 2. - 3i + 6j and – i + 2j
cos θ = 3. 8i – 4j and - 6i – 12j
⃗⃗⃗⃗⃗⃗ ⃗⃗⃗⃗⃗⃗
4. - 2i + 5j and 3i + 6j
5. - 4j and -7i
Mid-point of PR= m;
=. , / B. Determine k such that the two vectors are
=. , / orthogonal.
1. 3i – 2j , 4i + 5kj
2. 9i – 16kj, i + 4kj
⇒ ⃗⃗⃗⃗⃗⃗ = – p
3. 4ki + j , 9ki – 25j
=( )
4. 5ki + 3j , 2i + 7j
=( )
C. 1. For the vector x = 3i + 2j, y = i + mj,
⃗⃗⃗⃗⃗⃗⃗⃗⃗
| | = √( ) determine the values of m for which:
⃗⃗⃗⃗⃗⃗⃗⃗⃗
| | = 2.5 units i. x is perpendicular to y;

Baffour Ba Series, Further Mathematics for Schools Page 321


ii. x is parallel to y; = | | + | | - 2(q.p)
iii. the angle between x and y is 300. = b2 + c2 – 2bc cos ̃

2. Given the vector u = 3i + 2j, and v = 2i + mj, But ⃗⃗⃗⃗⃗ . ⃗⃗⃗⃗⃗ = a2


determine the values of m so that: a2 = b2 + c2 – 2bc cos ̃
i. u and v are at right This results is called the cosine rule.
ii. u and v are parallel.
iii. the angle between x and y is 450. Proof of sine Rule
The proof shows tha any 2 of the 3 vectors
Sine and Cosine Rules comprising the triangle have the same cross
Notation product as any other two vectors.
̃ is the angle at A and a is the length of the a C

side opposite ̃ ; By the triangular law b


̃ is the angle at B and b is the length of the B
of vectors, A
side opposite ̃ c=a +b c
̃ is the angle at C and c is the length of the a=c–b
side opposite ̃ b=c–a
C
̃
a Now,
b b × a = ⃗⃗⃗⃗⃗
| | ⃗⃗⃗⃗⃗
| | sin C
̃
B c × a = ⃗⃗⃗⃗
| | ⃗⃗⃗⃗⃗
| | sin B
̃ c c × b = ⃗⃗⃗⃗
| | ⃗⃗⃗⃗⃗
| | sin A
A
But
Proof of Cosine Rule b × a = (c – a) × a
Consider the figure below. =c×a–a×a
C
=c×a

c × b = c × (c – a)
q
=c×c–a×c
B =-c×c+a×c
̃ p = a×c
A =c×a
Let ⃗⃗⃗⃗⃗ = p and ⃗⃗⃗⃗⃗ = q
b×a=c×a=c×b
Then BC = q – p
⃗⃗⃗⃗⃗ . ⃗⃗⃗⃗⃗ = (q – p) (q – p) ⃗⃗⃗⃗⃗
| | ⃗⃗⃗⃗⃗
| | sin C = ⃗⃗⃗⃗
| | ⃗⃗⃗⃗⃗
| | sin B = ⃗⃗⃗⃗
| | ⃗⃗⃗⃗⃗
| | sin A
= q.q. – p.q – q.p + p.p ⇒ = =

Baffour Ba Series, Further Mathematics for Schools Page 322


Solved Past Question a.b = /a/ /b/ cos (1800 – θ)
ABC is a triangle with ⃗⃗⃗⃗⃗ = a, ⃗⃗⃗⃗⃗ = b, ⃗⃗⃗⃗⃗ = c
and < ABC = θ,. But cos (1800 – θ ) = - cos θ
i. If one of the vectors can be expressed in a.b = – /a/ /b/ cos θ
terms of the other two, write down the
expression. Put a.b = – /a/ /b/ cos θ in eqn (1)
ii. using dot product, show that: /c/2 = /a/2 + /b/2 – 2 /a/ /b/ cos θ
c2 = a2 + b2 – 2ab cos θ, where a = /a/, b = /b/ c2 = a2 + b2 – 2 ab cos θ
and c = /c/.
iii. For what value of θ will the result in (ii) iii. If θ = 900 then;
represent the Pythagoras theorem. c2 = a2 + b2 – 2 ab cos 900
c2 = a2 + b2 – 2 ab (0)
Solution c2 = a2 + b2 (Pythagoras theorem )
i. Given ⃗⃗⃗⃗⃗ = a, ⃗⃗⃗⃗⃗ = b, ⃗⃗⃗⃗⃗ = c and < ABC = θ
The Triangular Law of Vectors
B
Consider the diagram below:
C
c a 5
4
θ α 3
A b C 1 A B
From the diagram; 2
1 2 3 4 5
⃗⃗⃗⃗⃗ = ⃗⃗⃗⃗⃗ + ⃗⃗⃗⃗⃗
From the diagram, ⃗⃗⃗⃗⃗ = ⃗⃗⃗⃗⃗ + ⃗⃗⃗⃗⃗ . ⃗⃗⃗⃗⃗ is
c=a+b
said to be the resultant vector of ⃗⃗⃗⃗⃗⃗ and ⃗⃗⃗⃗⃗⃗ .
ii. c = a + b ⇒⃗⃗⃗⃗⃗ = ⃗⃗⃗⃗⃗ + ⃗⃗⃗⃗⃗
c.c = (a + b) (a + b)
c.c = a.a + a.b + b.a + b.b Similarly, if ⃗⃗⃗⃗⃗ + ⃗⃗⃗⃗⃗ = ⃗⃗⃗⃗⃗⃗ then, ⃗⃗⃗⃗⃗ is the
c.c = a.a + b.b + 2a.b resultant vector of ⃗⃗⃗⃗⃗ and ⃗⃗⃗⃗⃗ .

But a = /a/ ⇒ a2 = /a/2 Worked Examples


⇒ b2 = /b/2 1. Given that ⃗⃗⃗⃗⃗ = ( ) and ⃗⃗⃗⃗⃗ = ( ), find the
⇒ c2 = /c/2
resultant vector.
/c/2 = /a/2 + /b/2+ 2(a.b)…………(1)
Solution
But a.b = /a/ /b/ cos α ⃗⃗⃗⃗⃗ = ⃗⃗⃗⃗⃗ + ⃗⃗⃗⃗⃗
Where α is the angle beween a and b = ( ) + ( )=( )
α = 1800 – θ
2. If ⃗⃗⃗⃗⃗ = ( ) and ⃗⃗⃗⃗⃗ = ( ), find ⃗⃗⃗⃗⃗

Baffour Ba Series, Further Mathematics for Schools Page 323


Solution 2. ⃗⃗⃗⃗⃗ = ⃗⃗⃗⃗⃗ ⃗⃗⃗⃗⃗
⃗⃗⃗⃗⃗ = ⃗⃗⃗⃗⃗ + ⃗⃗⃗⃗⃗ =( )−( )=( )=( )
⃗⃗⃗⃗⃗ = ⃗⃗⃗⃗⃗⃗ – ⃗⃗⃗⃗⃗
=( )–( )=. –
/=( ) 3. ⃗⃗⃗⃗⃗ = ⃗⃗⃗⃗⃗⃗ − ⃗⃗⃗⃗⃗
=( )–( ) =( )=( )
Relating Free Vectors and Position Vectors
4. ⃗⃗⃗⃗⃗ = ⃗⃗⃗⃗⃗⃗ − ⃗⃗⃗⃗⃗
=( )–( )=( )=( )
B
A 5. ⃗⃗⃗⃗⃗⃗ + ⃗⃗⃗⃗⃗ = ⃗⃗⃗⃗⃗⃗ − ⃗⃗⃗⃗⃗ + ⃗⃗⃗⃗⃗ − ⃗⃗⃗⃗⃗
O
=( )−( )+( )−( )
=( )+( )= ( )
From the diagram above;
⃗⃗⃗⃗⃗ = ⃗⃗⃗⃗⃗ + ⃗⃗⃗⃗⃗
Applications
⃗⃗⃗⃗⃗ – ⃗⃗⃗⃗⃗ = ⃗⃗⃗⃗⃗
Worked Examples
⃗⃗⃗⃗⃗⃗ = ⃗⃗⃗⃗⃗⃗ – ⃗⃗⃗⃗⃗⃗ 1. P, Q, R and T are points in the Cartesian
⃗⃗⃗⃗⃗⃗ = b – a plane. The coordinates of P and Q are (4, 1) and
(-3, 2) respectively. ⃗⃗⃗⃗⃗ = ( ), ⃗⃗⃗⃗⃗ = ( ).
For all free vectors such as ⃗⃗⃗⃗⃗ , ⃗⃗⃗⃗⃗ etc, the
Find ⃗⃗⃗⃗⃗ .
following generalizations can be made:
(i) ⃗⃗⃗⃗⃗ = ⃗⃗⃗⃗⃗ − ⃗⃗⃗⃗⃗
Solution
⃗⃗⃗⃗⃗⃗ = b – a P(4, 1), Q(-3, 2), ⃗⃗⃗⃗⃗ = ( ) and ⃗⃗⃗⃗⃗ = ( )
(ii) ⃗⃗⃗⃗⃗ = ⃗⃗⃗⃗⃗⃗ − ⃗⃗⃗⃗⃗ ⃗⃗⃗⃗⃗ = r – t
But ⃗⃗⃗⃗⃗ = r – q,
⃗⃗⃗⃗⃗⃗ = q – p
( )=r–( )
( )+( )=r
Worked Examples
Given that a = (3, 6), b = (-7, 2), c = (2, 5) ( )=r
and d = ( ). Find: ( ) =r
(1) ⃗⃗⃗⃗⃗ (2) ⃗⃗⃗⃗⃗ (3) ⃗⃗⃗⃗⃗
⃗⃗⃗⃗⃗ = t – p, by substitution
(4) ⃗⃗⃗⃗⃗ (5) ⃗⃗⃗⃗⃗⃗ + ⃗⃗⃗⃗⃗
( )=t–( )
Solution ( )+( )=t
(1) ⃗⃗⃗⃗⃗ = ⃗⃗⃗⃗⃗ − ⃗⃗⃗⃗⃗
( )=t
=( )–( )=( )=( )
( ) =t

Baffour Ba Series, Further Mathematics for Schools Page 324


⃗⃗⃗⃗⃗ = r – t, but r = ( ) and t = ( ) i. the co-ordinates of Q and S,
⃗⃗⃗⃗⃗ = ( ) – ( ) ii. the magnitude of PR.

⃗⃗⃗⃗⃗ = ( ) 6. a. A, B, C and D are four points such that


A (-3, 2), C (6, 3), ⃗⃗⃗⃗⃗ ( ) and ⃗⃗⃗⃗⃗ ( ).
Exercises 11.12 Calculate:
1. Given that ⃗⃗⃗⃗⃗ = ( ) and ⃗⃗⃗⃗⃗ = ( ) i. the coordinates of B and D,
Find: (i) ⃗⃗⃗⃗⃗ (ii) ⃗⃗⃗⃗⃗ (iii) | ⃗⃗⃗⃗⃗ | ii. the vectors ⃗⃗⃗⃗⃗ and ⃗⃗⃗⃗⃗ ,
b. what is the relationship between ⃗⃗⃗⃗⃗ and ⃗⃗⃗⃗⃗
2. O is the origin and A and B are the points (4,
5) and (–1, 3) respectively. 7. The vertices of a triangle are P (1,-3), Q( 7,
i. Express 2a – 3b in component form. 5) R (-3, 5).
ii. What are the components of ⃗⃗⃗⃗⃗ ? i. Express ⃗⃗⃗⃗⃗ , ⃗⃗⃗⃗⃗ and ⃗⃗⃗⃗⃗ as column vectors.
iii. What are the components of ⃗⃗⃗⃗⃗ , where P is ii. Show that triangle PQR is isosceles.
the midpoint of ⃗⃗⃗⃗⃗ ?
8. PQRS is quadrilateral with P (2, 2), S (4, 4)
iv. Calculate the length of AB, leaving your
answer in surd form. and R (6, 4). If ⃗⃗⃗⃗⃗ ⃗⃗⃗⃗⃗ , find the coordinates
of Q.
3. A and B are the points (–1, 1) and (3, 4)
respectively, Parallelogram Law of Vectors
i. write down the components of the position Consider the parallelogram ABCD below;
vectors a and b . D C
ii. write down the components of the vector
represented by ⃗⃗⃗⃗⃗ .
iii. calculate the magnitude of b – a
A
B
4. A (4, 7) is the vertex of triangle ABC. /AD/ = /BC/ and /AB/ = /DC/
⃗⃗⃗⃗⃗ = ( ) and ⃗⃗⃗⃗⃗ = ( ). ⃗⃗⃗⃗⃗ = ⃗⃗⃗⃗⃗ + ⃗⃗⃗⃗⃗
⃗⃗⃗⃗⃗ = ⃗⃗⃗⃗⃗ + ⃗⃗⃗⃗⃗
a. find the co-ordinates of B and C.
b. if M is the midpoint of the line ⃗⃗⃗⃗⃗ , find ⃗⃗⃗⃗⃗⃗ . Where the diagonal ⃗⃗⃗⃗⃗ is the resultant vector.

5. The points P, Q, R, and S are vertices of Worked Examples


parallelogram in the Cartesian plane. The co- 1. The vertices of a parallelogram ABCD are
ordinates of P and R are (-8, 2) and (5, -2) A(2, 7), B(-3, 6), C(- 4, -2) and D(x, y). Write
⃗⃗⃗⃗⃗ and ⃗⃗⃗⃗⃗ as column vectors and deduce the
respectively and ⃗⃗⃗⃗⃗ ( ). Find:
values of x and y.

Baffour Ba Series, Further Mathematics for Schools Page 325


Solution From triangle ABC;
Method 1 ⃗⃗⃗⃗⃗ = ⃗⃗⃗⃗⃗ + ⃗⃗⃗⃗⃗
A(2, 7), A(2, 7), B(-3, 6),
⃗⃗⃗⃗⃗ = ⃗⃗⃗⃗⃗ – ⃗⃗⃗⃗⃗
B(-3, 6), ⃗⃗⃗⃗⃗ = (5i – 3j) – (3i + j )
C(- 4, -2)
⃗⃗⃗⃗⃗ = 5i – 3j – 3i – j
D(x, y)
D(x, y), C(- 4, -2) ⃗⃗⃗⃗⃗ = 5i – 3i – 3j – j

⃗⃗⃗⃗⃗ = ⃗⃗⃗⃗⃗ – ⃗⃗⃗⃗⃗ ⃗⃗⃗⃗⃗ = 2i – 4j


A B
⃗⃗⃗⃗⃗ = ( ) – ( ) = ( )
ii. From ∆ BCD:
⃗⃗⃗⃗⃗⃗ = ⃗⃗⃗⃗⃗ + ⃗⃗⃗⃗⃗
⃗⃗⃗⃗⃗ = ⃗⃗⃗⃗⃗ – ⃗⃗⃗⃗⃗⃗
But ⃗⃗⃗⃗⃗ = ⃗⃗⃗⃗⃗ D C
⃗⃗⃗⃗⃗ = ( ) – . / = . /
⃗⃗⃗⃗⃗⃗ = ⃗⃗⃗⃗⃗ + ⃗⃗⃗⃗⃗
⃗⃗⃗⃗⃗⃗ = ⃗⃗⃗⃗⃗ + ⃗⃗⃗⃗⃗⃗⃗⃗⃗
Since ABCD is a parallelogram,
⃗⃗⃗⃗⃗⃗ = (2i – 4 j) + (- 3i – j)
⃗⃗⃗⃗⃗ = ⃗⃗⃗⃗⃗
⃗⃗⃗⃗⃗⃗ = 2i – 4 j – 3i – j
( )=. / ⃗⃗⃗⃗⃗⃗ = 2i – 3i – 4j – j
⇒ -5 = - 4 – x ⃗⃗⃗⃗⃗⃗ = - i – 5j
x=-4+5
x=1
b. Unit vector in the direction of ⃗⃗⃗⃗⃗
-1 = -2 – y = × ⃗⃗⃗⃗⃗
|⃗⃗⃗⃗⃗ |
y = -2 + 1 = × (2i – 4j )
y = -1 √ ( )

= × (2i – 4j)

2. ABCD is a parallelogram such that ⃗⃗⃗⃗⃗ = 3i √
= × (2i – 4j)
+ j and ⃗⃗⃗⃗⃗ = 5i – 3j. Find:

a. i. ⃗⃗⃗⃗⃗ ii. ⃗⃗⃗⃗⃗⃗ = × (2i – 4j)

b. the unit vector in the direction of ⃗⃗⃗⃗⃗ = × (2i – 4j)

Solution 2. ABCD is a parallelogram, with vertices


a. i. A B A(x, y), B(5, 7), C (4, 3) and D (1, 2).
i. Find ⃗⃗⃗⃗⃗⃗ and ⃗⃗⃗⃗⃗ and hence find the values of
x and y.
ii. Calculate the magnitude of ⃗⃗⃗⃗⃗ .
D C

Baffour Ba Series, Further Mathematics for Schools Page 326


Solution parallel to RS. Find two pairs of coordinates of
i. ⃗⃗⃗⃗⃗⃗ = b – a the point S.
⃗⃗⃗⃗⃗⃗ = ( ) – . / = . /
4. A rhombus ABCD has A at (0, 2) and B at (5,
3). If the diagonals intersect at (0, 3), find the
⃗⃗⃗⃗⃗⃗⃗ = c – d co-ordinates of C and D.
⃗⃗⃗⃗⃗⃗⃗ = ( ) – ( ) = ( )=( )
5. In ∆ABC, ⃗⃗⃗⃗⃗ = ( ) and ⃗⃗⃗⃗⃗ = ( ). If P is
For parallelogram ABCD, ⃗⃗⃗⃗⃗⃗ = ⃗⃗⃗⃗⃗ the mid point of ⃗⃗⃗⃗⃗ , express ⃗⃗⃗⃗⃗ as a column
. / =( ) vector.
⇒ 5 – x = 3 and 7 – y = 1
5 – 3 = x and 7 – 1 = y 6. OAB is traiangle where O is the origin. P and
x = 2 and y = 6 Q are mid – points ⃗⃗⃗⃗⃗ and ⃗⃗⃗⃗⃗ respectively.

ii. A (2, 6) and C (4, 3) The position vectrors of ⃗⃗⃗⃗⃗ and ⃗⃗⃗⃗⃗ are a and
⃗⃗⃗⃗⃗ = c – a b respectivey. Write down the position vectors
=( )–( )=( ) of P and Q in terms of a and b

/⃗⃗⃗⃗⃗ / = √ Position Vector of a Point Dividing a


= √ Straight Line in a given Ratio
A
=√ m
= 3.6 units P
a
r n
Exercises 11.13
1. A (-2, 3), B (2, -1), C (5, 0) and D (x, y) are O b B
the vertices of the parallelogram ABCD.
From the diagram, the line AB is divided in the
a. Find ⃗⃗⃗⃗⃗ and ⃗⃗⃗⃗⃗⃗ Hence find the coordinates ratio, AP : PB = m : n
D.
=
b. Calculate, correct to one decimal place,
⃗⃗⃗⃗⃗⃗ . ⇒nAP = mPB

In terms of position vectors,


2. P(-1, -2), Q(5, k), R(8, 2) and S(h, 1) are the
n(OP – OA) = m(OB – OP)
four vertices of the parallelogram PQRS. Find
the values of h and k.
By substitution,
3. P(-1, 5), Q(-2, 1), R(3, -2) and S(a, b) are the n(r – a) = m(b – r)
four vertices of a parallelogram, with PQ nr – na = mb – mr
nr + mr = mb + na

Baffour Ba Series, Further Mathematics for Schools Page 327


r(n + m) = mb + na Solution
r= a A
O
Y 2
r is the position vector of the point diving the
line AB in the ratio m : n b X
1
If the mid point of AB is at P, then m = n
⃗⃗⃗⃗⃗ = 2⃗⃗⃗⃗⃗ B
⇒r=
⃗⃗⃗⃗⃗⃗
⃗⃗⃗⃗⃗⃗
= ⇒ ⃗⃗⃗⃗⃗ : ⃗⃗⃗⃗⃗ = 2 : 1
r=
( )
r= If X divides the line ⃗⃗⃗⃗⃗ in the ratio 2 : 1;
( ) ( )
⇒ ⃗⃗⃗⃗⃗ = = =
r = (a + b)

Y is the mid – point of ⃗⃗⃗⃗⃗ ;


Worked Examples
⃗⃗⃗⃗⃗ = ⃗⃗⃗⃗⃗
Given A (1, 6) and B(1, 3), find the position
vector of the point which divides AB in the ratio
2:1 ⃗⃗⃗⃗⃗ = ⃗⃗⃗⃗⃗ – ⃗⃗⃗⃗⃗
⃗⃗⃗⃗⃗ = ⃗⃗⃗⃗⃗ – ⃗⃗⃗⃗⃗
Solution
⃗⃗⃗⃗⃗ = . /–b
A (1, 6) and B(1, 3), A 2
m:n=2:1 P ⃗⃗⃗⃗⃗ = ( )–b
a
Let OP = r r 1 ⃗⃗⃗⃗⃗ = a + b

O ⃗⃗⃗⃗⃗ = a + b
r= b B

By substitution, 3. In triangle ABC, X lies on AB such that AX :


( ) ( ) ( ) ( ) ( ) ⁄
XB = 1 : 2 and Y lies on AC such that AY : YC
r= = = =( *=( ) = 1 : 2. Show that XY is parallel to BC and its

one – third its length.
2. i. OAB is a triangle, where O is the origin.
Let a, b be the position vectors of A and B Solution
From the diagram
respectively. If X is a point on ⃗⃗⃗⃗⃗ such that
⃗⃗⃗⃗⃗ = ⃗⃗⃗⃗⃗ + ⃗⃗⃗⃗⃗ = 3 + 3 = 6
⃗⃗⃗⃗⃗ = 2⃗⃗⃗⃗⃗ abnd Y is the mid – point of ⃗⃗⃗⃗⃗ ,
⃗⃗⃗⃗⃗ = ⃗⃗⃗⃗⃗ + ⃗⃗⃗⃗⃗ = 1 + 1 = 2
show that ⃗⃗⃗⃗⃗ = a – b ⃗⃗⃗⃗⃗
⃗⃗⃗⃗⃗
=

Baffour Ba Series, Further Mathematics for Schools Page 328


⃗⃗⃗⃗⃗ such that BP : PC = 3 : 1; the point Q is on CA
⃗⃗⃗⃗⃗
=
such that CQ : QA = 2 : 3; the point R is on BA
⃗⃗⃗⃗⃗ = ⃗⃗⃗⃗⃗ C
produced such that BR : AR = 2 : 1. The
3⃗⃗⃗⃗⃗ = ⃗⃗⃗⃗⃗ 2 position vectors of P, Q and R are p, q and r
⃗⃗⃗⃗⃗ is parallel to ⃗⃗⃗⃗⃗ Y respectively. Show that q can be expressed in
1 terms of p and r and hence, or otherwise, show
that P, Q and R are collinear. State the ratio of
A X B
1 2 the lengths of the line segment PQ and QR.
Exercises 11.14
1. In the diagram beleow, AT = 2BT, 4. The points A, B and C have position vectors
⃗⃗⃗⃗⃗ = a, ⃗⃗⃗⃗⃗ = b, ⃗⃗⃗⃗⃗ = 2b, a, b and c respectively reffered to an origin O.
i. Given that the point X lie on AB produced so
C
that AB : BX = 2 : 1, find x , the position vector
B
of X in terms of b and c.
2b ii. If Y lies on BC, between B and C so that BY
T : YC = 1 : 3, find y , the positin vector of Y in
b
terms of b and c.
O a A iii. Given that Z is the mid point of AC, show
that X, Y and Z are collinear.
Find in terms of a and b;
a. i. ⃗⃗⃗⃗⃗ ii. ⃗⃗⃗⃗⃗ iii. ⃗⃗⃗⃗⃗ iv. ⃗⃗⃗⃗⃗ v. ⃗⃗⃗⃗⃗
5. The position vectors of three points A, B and
b. From (iv) and (v), what can you deduce
C relative to the origin O are p, 3q – p and 9q –
about O, T and C?
5p respectively. Show that the points A, B and
a
C lie on the same straight line, and state the
Challenge Problems ratio AB : BC.
From an origin O, the points A, B and C have
Given that OBCD is a parallelogram and that E
position vectors a, b and 2b respectively. The
is the point such that ⃗⃗⃗⃗⃗⃗ = ⃗⃗⃗⃗⃗ , find the
points O, A and B are not collinear. The mid =
point of AB is M, and the point of trisection of position vectors of D and E relative to O.
AC nearer to A is T. Darw a diagram to show O,
A, B, C, M, T. Application
Find in terms of a and b, the position vectors M Using Vector Approach to Solve Bearings
and T. Use your resukts to prove that O, M and I. Make a sketch or accurate drawing of the
T are collinear and fins the ratio in which M diagram representing the problem.
divides OT.
2. Apply the triangular law of vectors which
3. The vertices A, B and C of a triangle have states that ⃗⃗⃗⃗⃗ = ⃗⃗⃗⃗⃗ + ⃗⃗⃗⃗⃗ , for a right triangle
position vectrprs a, b and c respectively, with vertices, A, B and C.
relative to an origin O. The point P is on BC

Baffour Ba Series, Further Mathematics for Schools Page 329


3. Express the bearing of A from B as a vector ⃗⃗⃗⃗⃗ = ( )+( )
and the bearing of C from B as a vector and ⃗⃗⃗⃗⃗ = ( )
substitute in ⃗⃗⃗⃗⃗ = ⃗⃗⃗⃗⃗ + ⃗⃗⃗⃗⃗ to get the
component vector ⃗⃗⃗⃗⃗ = . /. / ⃗⃗⃗⃗⃗ / = √( 8 8 ) ( )
| | = √ ,8
4. To find the distance between A and C is | | = 144km (to the nearest kilometer)
to find the magnitude of ⃗⃗⃗⃗⃗ , such that:
/ ⃗⃗⃗⃗⃗ / = √ ii. . bearing of A from C;
⃗⃗⃗⃗⃗ = - ⃗⃗⃗⃗⃗
5. To find the bearing of A from C is to find the =( )
angle θ, ⃗⃗⃗⃗⃗ makes with the x – axis, using the
Angle ⃗⃗⃗⃗⃗ makes with the x – axis,
relation, θ = tan-1. /
θ = tan-1. /
0
6. To obtain the bearing of A from C, take θ = 16 (nearest degree) at the x – axis
measurement from the north pole of C to the
line A (direction of A). ⃗⃗⃗⃗⃗ = ( ), falls in the third quadrant

Worked Examples Bearing of A from C;


C
1. An aircraft flew from port A on a bearing of = 900 + 900 + θ0 0
16
to another port B 80 km apart. It then flew = 900 + 900 + 740 74
0

from port B on a bearing of to port C, a = 2540 A


distance of 120 km. Calculate: ⃗⃗⃗⃗⃗ = (144km, 2540)
i. the distance from port A to port C, to the
nearest kilometer; Method 2
ii. the bearing of port A from port C, to the From triangular law of vectors;
nearest degree. ⃗⃗⃗⃗⃗ = ⃗⃗⃗⃗⃗ + ⃗⃗⃗⃗⃗
Resolving along the x – axis,
Solution ⃗⃗⃗⃗⃗ = . /+. /
Method 1 : Resolving along the y - axis
i. ⃗⃗⃗⃗⃗ = (80km, 1300) ⃗⃗⃗⃗⃗ = ( )
⃗⃗⃗⃗⃗ = (120km, 0400)
⃗⃗⃗⃗⃗ = ……… / ⃗⃗⃗⃗⃗ / = √( 8 8 ) ( )
| | = √ ,8
From the triangular law of vectors;
| | = 144km (to the nearest kilometer)
⃗⃗⃗⃗⃗ = ⃗⃗⃗⃗⃗ + ⃗⃗⃗⃗⃗

Resolving along the y – axis, ii. bearing of A from C;


⃗⃗⃗⃗⃗ = . ⃗⃗⃗⃗⃗ = - ⃗⃗⃗⃗⃗
/+. /

Baffour Ba Series, Further Mathematics for Schools Page 330


⃗⃗⃗⃗⃗ = ( ) / ⃗⃗⃗⃗⃗ / = √( ) ( )
/ ⃗⃗⃗⃗⃗ / = √
Angle ⃗⃗⃗⃗⃗ makes with the x – axis, / ⃗⃗⃗⃗⃗ / = 12km (to the nearest kilometer)
θ=t n . /

θ = 160 (nearest degree) at the x – axis ii. Bearing of Q from R;


⃗⃗⃗⃗⃗ = ⃗⃗⃗⃗⃗
⃗⃗⃗⃗⃗ = ( ), falls in the third quadrant ⃗⃗⃗⃗⃗ = ( )
⃗⃗⃗⃗⃗ = ( ); third quadrant;
Bearing of A from C;
C
= 900 + 900 + ( 900 – θ0 ) 0
= 900 + 900 + ( 900 – 160 )
16 0
74 Angle ⃗⃗⃗⃗⃗ makes with the x – axis,
= 2540 A θ=t n . /
0
θ = 61 (nearest degree) at the x – axis
⃗⃗⃗⃗⃗ = (144km, 2540)
Bearing of Q from R; R
2. Three schools, P, Q and R are situated as = 900 + 900 + ( 900 – θ0 ) 61
0
0
follows: Q is on a bearing of 5km, 1430 from P 29
= 900 + 900 + ( 900 – 610 )
and R is on the bearing 11 km, 0530 from P. = 2090 Q
i. Find the distance between school Q and ⃗⃗⃗⃗⃗ = (12km, 2090)
School R, to the nearest kilometer,
ii. Find the bearing of school Q from school R 3. The bearing of P from X, 10km away is 0250.
to the nearest degree. Another point Q is 6km from X and on a
bearing of 1620. Calculate:
Solution
i. the distance PQ;
i. ⃗⃗⃗⃗⃗ = (5km, 1430) ii. the bearing of P from Q.
⃗⃗⃗⃗⃗ = (11km, 0530)
⃗⃗⃗⃗⃗ = ……… Solution
i. ⃗⃗⃗⃗⃗ = (10km, 0250)
From the triangular law of vectors; ⃗⃗⃗⃗⃗ = (6km, 1620)
⃗⃗⃗⃗⃗ = ⃗⃗⃗⃗⃗ + ⃗⃗⃗⃗⃗
⃗⃗⃗⃗⃗ = ………
⃗⃗⃗⃗⃗ = ⃗⃗⃗⃗⃗⃗⃗⃗⃗ + ⃗⃗⃗⃗⃗

Resolving along the y – axis, From the triangular law of vectors;


⃗⃗⃗⃗⃗ = . /+. / ⃗⃗⃗⃗⃗ = ⃗⃗⃗⃗⃗ + ⃗⃗⃗⃗⃗
⃗⃗⃗⃗⃗ = ⃗⃗⃗⃗⃗⃗⃗⃗⃗ + ⃗⃗⃗⃗⃗
⃗⃗⃗⃗⃗ = ( )+( ) ⃗⃗⃗⃗⃗ = – . / +. /
⃗⃗⃗⃗⃗ = ( ) ⃗⃗⃗⃗⃗ = ( )

Baffour Ba Series, Further Mathematics for Schools Page 331


/ ⃗⃗⃗⃗⃗ / = √( ) ( ) The distance from the finishing point is 114km
(to the nearest kilometer)
/ ⃗⃗⃗⃗⃗ / = √
/ ⃗⃗⃗⃗⃗ / = 15km (to the nearest kilometer)
ii. Bearing from finishing point;
⃗⃗⃗⃗⃗ = - ⃗⃗⃗⃗⃗
ii. Bearing of P from Q;
⃗⃗⃗⃗⃗ = ( )
⃗⃗⃗⃗⃗ = ⃗⃗⃗⃗⃗⃗⃗⃗⃗
⃗⃗⃗⃗⃗ = ( ) ; first quadrant
⃗⃗⃗⃗⃗ = ( )
⃗⃗⃗⃗⃗ = ( ) first quadrant
The angle ⃗⃗⃗⃗⃗ makes with the x - axis

Angle ⃗⃗⃗⃗⃗ makes with the x – axis, θ = tan-1. /

θ = tan-1. / θ = 690 (nearest degree) at the x – axis


K
0
θ = 81 (nearest degree) at the x – axis

114km
Bearing of K from F;
P = 900 – 690 0
Bearing of P from Q; 68
= 210
15km

= 900 – θ0 0
9
F
⃗⃗⃗⃗⃗ = (114km, 0210) 5
= 900 – 810 Q 81
0

= 0090
Solved Past Questions
⃗⃗⃗⃗⃗ = (15km, 0090)
1. Two ships A and B leave a port
simularaneously. A steam at 10 on the
4. From Kwadaso, I travelled 100km on a 0
bearing of 1580, and then 80km on a bearing of bearing 160 while B steams on the bearing of
2600. Find the distance and bearing from my 2150. After one hour, the bearing of B from A is
finishing point. 2600. Find the speed of B, correct to the nearest
whole number.
Solution
O
i. ⃗⃗⃗⃗⃗⃗ = (100km, 1580) Solution
700
⃗⃗⃗⃗⃗ = (80km, 2600) ⃗⃗⃗⃗⃗ = (10 , 1600) 0
35 200
⃗⃗⃗⃗⃗ = ……… ⃗⃗⃗⃗⃗ = (x, 2600)
10

⃗⃗⃗⃗⃗ = ⃗⃗⃗⃗⃗⃗ + ⃗⃗⃗⃗⃗


After one hour; A
800
⃗⃗⃗⃗⃗ =. /+. / ⃗⃗⃗⃗⃗ = (y, 2150)
450
⃗⃗⃗⃗⃗ = ( ) B
=
/ ⃗⃗⃗⃗⃗ / = √( ) ( )
OB =
/ ⃗⃗⃗⃗⃗ / = √
/ ⃗⃗⃗⃗⃗ / = 114.3390 km OB = 14

Baffour Ba Series, Further Mathematics for Schools Page 332


2. Two ships S and T leave the port O at the Exercises 11.15
same time. Ship S moves at 22 on the Use the vector method in each case.
bearing of 125 and T on a bearing of 2000.
0
1. Two boats A and B leave a port at the same
After 1 hours, the bearing of ship T from from time. A travels 15km on a bearing of 0200 while
S is 2400. Calculate, correct to one decimal B travels 14km on a bearing of 2900. Calculate,
place; correct to two decimal places, the;
i. the initial speed of the ship T. a. distance between A and B,
ii. the distance between the positions of the b. bearing of A from B.
ship at that instance.
2. From Bamako, Enugu is 1800 km away on a
Solution bearing of 1660 and Monrovia is 900 km away
i. on a bearing of 2060. Calculate :
O i. the distance between Enugu and Monrovia,
350
ii. the bearing of Enugu from Monrovia.
200
550 22
3. The bearing of Q from P is 1500 and the
bearing of P from R is 0150. If Q and R are
650 A
24km and 32 km respectively from P:
i. calculate the distance between Q and R,
400
correct to twodecimal places;
B
ii. find the bearing of R from Q, correct to the
Using the sine rule; nearest degree.
=
4. An aircraft leaves an airport and flies 50 km
OT = = 31 on a bearing of 1300. It then flies 80 km on a
bearing of 0850.
ii. Using the sine rule; i. How far is it from the airport?
= ii. Find the bearing of the position of the
aircraft from the airport.
AB =
AB = 33 5. Two ships leave a harbor at the same time.
The speed form A to B = 33 One sails at 18 on a bearing of 0100.
The other sails at 15 on a bearing of
0
Distance from A to B; 065 . How far apart are the ships after one
= speed × time hour?
= 33 ×1
= 33 × 6. A and B are two survey points; A is 2.3 km
due north of B. A third point C lies on a bearing
= 49.5km

Baffour Ba Series, Further Mathematics for Schools Page 333


of 3180 from B and 2460 from A. How far is C projection of a on b and its denoted by projb a.
from A and from B? The fomula compb a = shows how to
| |
compute this projection without knowing the
7. An aircraft pilot set a course of 1560 and flies
angle θ.
with an airspeed of 250 . There is a
Proof:
wind of velocity 50 blowing from the
0 If θ is the angle between a and b , then, from
direction with bearing 085 . In what direction
the theorem on dot product, a . b = /a//b/ cos θ
does the aircraft travel, and with what ground
speed? Thus, | |
= /a/ cos θ = compb a

Definition of Compb a Worked Examples


Let θ be the angle between two vectors a and b. Given that a = 10i + 4j and b = 3i – 2j, find the
The component of a along b , denoted by compb a and compa b
compb a, is given by :
compb a = /a/ cos θ Solution
A a = 10i + 4j and b = 3i – 2j,

B a ( )( ) ( )( )
a compb a = | |
= = = 6.10
b √ ( √
( )( ) ( )( )
θ Q compa b = | |
= = = 2.04
√ √
l O
Exercises
Referring to the diagrams above, the Given that a = (2, -3), b = (3, 4) and c = (-1,
component of a along b is found by projecting 5), find the following:
the endpoint of a onto the line l containing b. 1. compc b 2. Compb c
For this reason, /a/ cos θ is often called the 3. compc c 4. compa (a + c)

Baffour Ba Series, Further Mathematics for Schools Page 334


12 STATICS Baffour Ba Series

Scalar and Vector Quantities

Scalar Vector
Physical quantities having magnitude Physical quantities having magnitude and
only but no direction direction
These quantities are completely described
These quantities are completely described
by: a number and a suitable unit by numbers, a suitable unit and by certain
direction.
These quantities are added, subtracted, These quantities cannot be added,
multiplied and divided by simple subtracted, multiplied and divide by simple
arithmetic rules. arithmetical rules.

List of scalar and vector quantities ⃗⃗⃗⃗⃗ + ⃗⃗⃗⃗⃗ = ⃗⃗⃗⃗⃗ ……………(1)


Mass, weight, speed, work, power Energy, ⃗⃗⃗⃗⃗⃗ + ⃗⃗⃗⃗⃗ = ⃗⃗⃗⃗⃗ ……………(2)
force, torque, displacement distance
But /AC/ = /OB/, put in eqn (2)
The Resultant Vector, Parallelogram Law and ⃗⃗⃗⃗⃗ + ⃗⃗⃗⃗⃗ = ⃗⃗⃗⃗⃗ ……………(3)
Triangular Law of Vectors
Consider the figure below;
eqn (1) and eqn (2) are called the triangular
y law of vectors.
C
A (x1, y1)
eqn (3) is called the parallelogram law of
vectors.
B(x2, y2)
θ In both laws, ⃗⃗⃗⃗⃗ is called the resultant vector.
O x

The Magnitude of the Resultant Vector


If the vector OA and vector OB correspond to CASE I
a = (x1, y1) and b = (x2, y2) respectively, then by 1. If the angle between ⃗⃗⃗⃗⃗ and ⃗⃗⃗⃗⃗ is θ, then
forming a parallelogram from the figure, (since the magnitude of the resultant vector ⃗⃗⃗⃗⃗ , is
opposite sides are equal in length), the found by applying the cosine rule.
parallelogram OACB is obtained where; |⃗⃗⃗⃗⃗ | = |⃗⃗⃗⃗⃗ | + |⃗⃗⃗⃗⃗ | – 2|⃗⃗⃗⃗⃗⃗⃗⃗⃗| ⃗⃗⃗⃗⃗⃗⃗⃗⃗
| | cos (1800 – θ)
/OA/ = /BC/ and /OB/ = /AC/
2. If the angle between ⃗⃗⃗⃗⃗ and ⃗⃗⃗⃗⃗ is θ, where
It follows that ∆OBC and ∆OAC are congruent. θ > 1800, then the magnitude of the resultant
Thus;
Baffour Ba Series, Further Mathematics for Schools Page 335
vector ⃗⃗⃗⃗⃗ , is found by applying the cosine rule; V1 = (10 sin 700) i + (10 cos 700) j
V2 = (15 sin 2600) i + (15 cos 2600) j
|⃗⃗⃗⃗⃗ | = |⃗⃗⃗⃗⃗ | + |⃗⃗⃗⃗⃗ | – 2|⃗⃗⃗⃗⃗⃗⃗⃗⃗| ⃗⃗⃗⃗⃗⃗⃗⃗⃗
| | cos (θ - 1800 )

V1 = + 9.3969 i + 3.4202 j
CASE II
V2 = (-14. 7721) i + (- 2 . 6047) j
If the angle between ⃗⃗⃗⃗⃗ and ⃗⃗⃗⃗⃗ is 900, then the
cosine rule is reduced to the Pythagoras
R = V1 + V2
theorem. Hence, the magnitude of the resultant
R = (9.3969 – 14.7721)i + (3.4202 – 2.6047) j
vector ⃗⃗⃗⃗⃗ , |⃗⃗⃗⃗⃗ | = |⃗⃗⃗⃗⃗ | + |⃗⃗⃗⃗⃗ | R = – 5.3752 i + 0.8155 j

Worked Examples /R/=√


1. Find the magnitude of the resultant of the
/R/ = √( ) ( 8 )
vectors (10m, 0700) and (15m, 2600)
/R/ = √ 8 8
Solution /R/ = √
Method 1 /R/ = 5.4367 N
Let ⃗⃗⃗⃗⃗ = (10m, 0700) and ⃗⃗⃗⃗⃗ = (15m, 2600)
Method 3
From the diagram,
A V1 = (10 cos 200) i + (10 sin 200) j
10 V2 = (-15 cos 100) i + (-15 sin 100) j
O 200
15 800 V1 = 9.3969 i + 3.4202 j
B
V2 = (-14. 7721) i + (- 2 . 6047) j

From the diagram, R = V1 + V2


< between OA and OB = 200 + 900 + 800 R = (9.3969 – 14.7721) i - (3.4202 – 2.6047) j
= 1900 R = – 5.3752 i + 0.8155 j

= |⃗⃗⃗⃗⃗ | + |⃗⃗⃗⃗⃗ | – 2|⃗⃗⃗⃗⃗⃗⃗⃗⃗| ⃗⃗⃗⃗⃗⃗⃗⃗⃗


| | cos (θ - 1800 ) /R/=√
R = 10 + 15 – 2 (10) (15) cos (1900 – 1800)
2 2 2 / R / = √( ) ( 8 )
2 0
R = 325 – 300 cos 10 /R/=√ 88
R2 = 325 – 295.4423 /R/=√
R2 = 29.5577 / R / = 5.4367 N
R=√
R = 5.4367 N Force
Force is an interaction between two objects that
Method 2 causes them to change motion. Alternatively, a
From the diagram,
Baffour Ba Series, Further Mathematics for Schools Page 336
force can keep an object at rest. Force is a= = 0.3
measured in newtons (N).
4. A man pushes a 50.0 kg block of ice across a
The formula for force states that force is equal
frozen pond. He applies a force of 25.0N,
to mass multiplied by acceleration. That is:
pushing the block away from him. What is the
F = ma;
accelartion of the block?
where the units for acceleration is meter per
second squared ( ) and the units for mass
Solution
is kilograms (kg)
F = 25 N, m = 50 kg and a = ?
F = ma
Worked Examples
a= = = 0.5
1. Find the force that is required accelerate a
1,000 kg car at 5 .
5. A mass of 50 kg is accelerated with 2 .
Solution Calculate the force required.
F = ma
F = 1000 × 5 Solution
F = 5,000N F = ?, m = 50 kg and a = 2
F = ma
2. An object has a mass of 19.53 kg and an F = 50 × 2 = 100 N
acceleration of 12.2 . What is the force on
The Resultant Force
the object?
The resultant force is the single force which
represents the vector sum of two or more
Solution
forces. For example, two forces of magnitude
F = ?, m = 19.53 kg and a = 12.2
and acting upon a particle have a
F = ma
resultant as shown below.
F = 19.53 × 12.29
F = 240.02 N

3. A force of 5 kilonewtons kN acts on a mass


of 15 tonnes. Calculate the acceleration.
(1 tonne = 1000 kg)
The method of combining all the forces into a
single force is called composition of forces.
Solution
F = ma
If a body is subjected to a number of forces of
a= different magnitude and direction in a single
F = 5kN = 5 × 1000N = 5,000N point of a single plane, then such system of
m = 15 tonnes = 15 × 1000 kg = 15,000 kg forces is known as concurrent coplanar forces.

Baffour Ba Series, Further Mathematics for Schools Page 337


Different Laws of Forces C
1. Parallelogram law of forces
A
2. Triangular law of forces
5
3. Polygon law of forces 0 1200
20
0 B 1200
65
Law of Polygon of Forces 8
The law of polygon of forces states that if a o
number of forces acting at a single point be
represented by the sides of a polygon taken in < ⃗⃗⃗⃗⃗⃗⃗ makes with the x – axis
order, then the resultant of these forces is θ = 900 – 650
obtained by the closing side of the polygon θ = 250
taken in opposite order. The polygon law of
forces is an extension of the triangular law of Using the formula for horizontal and vertical
forces. The resultant force of the system of components;
coplanar forces can be determined either by ⃗⃗⃗⃗⃗⃗ = (5 cos 700) i + (5 sin 700) j
analytical method or by graphical method. ⃗⃗⃗⃗⃗⃗⃗ = (8 cos 250) i + (8 sin 250) j

Resolution of Forces
But ⃗⃗⃗⃗⃗⃗ = ⃗⃗⃗⃗⃗⃗ + ⃗⃗⃗⃗⃗⃗⃗
Two forces acting at a point
⃗⃗⃗⃗⃗⃗ = (5 cos700) i + (5 sin700) j + (8 cos 250) i
Force is also represented by a vector. If two
forces are acting on an object, the resultant + (8 sin 250) j
force experienced by the object is the vector ⃗⃗⃗⃗⃗⃗ = (5 cos 700 + 8 cos 250) i + (5 sin 700 + 8
sum of the two forces. sin 250) j
⃗⃗⃗⃗⃗⃗ = 8.9606 i + 8.0794 j
Worked Example ⃗⃗⃗⃗⃗⃗ = 9 i + 8 j
1. Two forces ⃗⃗⃗⃗⃗ and ⃗⃗⃗⃗⃗ of magnitude 5 kg
and 8 kg respectively act at a point O. The Magnitude of ⃗⃗⃗⃗⃗⃗ ;
direction of ⃗⃗⃗⃗⃗ is N200E, and the direction of |⃗⃗⃗⃗⃗ | = √ 8
⃗⃗⃗⃗⃗ is N650E. Approximate the magnitude and |⃗⃗⃗⃗⃗ | = √8
direction of the resultant ⃗⃗⃗⃗⃗⃗ . |⃗⃗⃗⃗⃗ | = √
|⃗⃗⃗⃗⃗ | = 12kg
Solution 135
0
C
8
Method 1 B
From the diagram, Direction of |⃗⃗⃗⃗⃗ |; 0
20 12 5

< ⃗⃗⃗⃗⃗⃗ makes with the x – axis; Using sine rule; θ


A
0 0 = 8
θ = 90 – 20 O
θ = 700 sin θ =
sin θ = 0.4714
Baffour Ba Series, Further Mathematics for Schools Page 338
θ= (0.4714) 82 = 52 + 122 – 2 (5)(12) cos θ
θ = 280 64 – 25 – 144 = - 2 (5)(12) cos θ
- 105 = - 120 cos θ
⇒ The direction of ⃗⃗⃗⃗⃗ ; cos θ = 135
0
C
= N(28 + θ) E B
8
θ= . /
= N 480 E 20
0
12 5
θ = 28 θ
A
Method 2: (Resolving along the y - axis) 8
⃗⃗⃗⃗⃗⃗ = ⃗⃗⃗⃗⃗⃗ + ⃗⃗⃗⃗⃗⃗⃗ Direction of OC O
⃗⃗⃗⃗⃗⃗ = ( )+( ) = 200 + θ
= 200 + 280
⃗⃗⃗⃗⃗⃗ = ( ) = 480
/⃗⃗⃗⃗⃗⃗ / = √(8 ) (8 )
⃗⃗⃗⃗⃗⃗ / = 12 kg 135
0 2. Two forces, F1 and F2 with magnitude 20N
8
C
B and 30N, respectively act on an object at a
Direction of OC; 0
20 12 5 point P as shown below.
θ y
θ= . / 8
A F
θ = 420 (at x – axis) O
F2
⇒ the direction of ⃗⃗⃗⃗⃗ F1
0
=N E 150
450
x
But = 900 – 420 = 480 P
P
Hence the direction of ⃗⃗⃗⃗⃗ = N 480 E
Magnitude and direction of ⃗⃗⃗⃗⃗ = (12kg, 0480)
Find the resultant force acting at P.
Method 3: (using the cosine rule)
Magnitude of ⃗⃗⃗⃗⃗ ; Solution
From the diagram, Method 1
< AOB = 650 – 200 = 450 Let F1 = 20 and F2 = 40
< OBC = 1350 Resolving along the x – axis,
i.e. < AOB + < OBC = 1800 F1 = . /=( )
F2 = . /=( )
Considering ∆OBC,
/OB/ = 8, /BC/ = 5 and < OBC = 1350
/OC/2 = 82 + 52 – 2(8) (5) cos 1350 F = F1 + F2
/OC/2 = 145.6 F=( ) +( )
/OC/ = √ F=( )
/OC/ = 12.1 kg F=( )
Direction of |⃗⃗⃗⃗⃗ |, F = -12i + 29j

Baffour Ba Series, Further Mathematics for Schools Page 339


Method 2 Solution
y
Express F1 and F2 in component form.
Simplify the scalars; C

F1 = 20 (cos 450) i + 20 (sin 450)j A


√ √ 3N
= 20 . / i + 20 . / j
0 B
= 10 √ i + 10 √ j 60 4N
x
O
F2 = 30 (cos 1500) i + 30 (sin 1500) j

= 30 . / i + 30 . / j
From the diagram,
= - 15 √ i + 15 j < AOB + < OBC = 1800 (co – interior angles)
But < AOB = 600
The resultant force; 600 + <OBC = 1800
F = F1 + F2 < OBC = 1800 – 600
F = (10 √ i + 10 √ j) + (- 15 √ i + 15 j) < OBC = 1200
F = (10 √ - 15 √ ) i + (10 √ + 15) j
F = - 12 i + 29 j Considering ∆OBC,
/OB/ = 4, /BC/ = 3 and < OBC = 1200
3. Consider two forces of magnutudes 5N and 7 /OC/2 = 42 + 32 – 2(4) (3) cos 1200
N acting on a particle with an angle of 900 /OC/2 = 16 + 9 – 2(4) (3) (0.5)
between them. What is the magnitude and /OC/ = √
direction of the resultsnt force. /OC/ = 3.6056 N

Solution Exercises 12.1


Py Pythagoras theorem, A. The vectors (a) and (b) represent two
=√ forces acting at the same point and θ is the
=√ 5 smallest positive angle between a and b.
Approximate the magnitude of the resultant
=√
force.
= 8.6
1. /a/ = 40N, /b/ = 70N, θ = 450
2. /a/ = 5.5N, /b/ = 6.2N, θ = 600
Direction of the resultant vector;
3. /a/ = 2N, /b/ = 8N, θ = 1200
θ= . / 4. /a/ = 30N, /b/ = 50N, θ = 1500
0
θ = 54.4 (at y – axis)
B. The magnitudes and directions of two
4. Two forces of 3N and 4N are acting at a forces acting at a point O are given in (a)
point such that the angle between them is 600. and (b). Approximate the magnitude and
Find the resultant force. directions of the resultant force.

Baffour Ba Series, Further Mathematics for Schools Page 340


1. (a) 90N, N750W (b) 65N, S50E Resultant of Collinear Coplanar Forces.
2. (a) 20N, S170W (b) 50N, N820W Collinear coplanar forces are those forces
3. (a) 6N, 1100 (b) 2N, 2150 which act in the same plane and have a
4. (a) 70N, 3200 (b) 40N, 300 common line of action. The resultant of these
forces are obtained by analytical method or
Law of Polygon of Forces graphical method.
The law of polygon of forces states that if a
number of forces acting at a single point be 1. Analytical method.
represented by the sides of a polygon taken in The resultant is obtained by adding all the
order then the resultant of these forces is
forces if they are acting in the same direction.
obtained by the closing side of the polygon
taken in opposite order. The polygon law of
forces is an extension of the triangular law of The figure below shows three collinear
forces. The resultant force of the system of coplanar forces F1, F2 and F3 acting on a rigid
coplanar forces can be determined either by body in the same direction.
analytical method or by graphical method.

Coplaner Forces Acting at a Point


Resultant of Coplanar Forces F1 F2 F3
When a number of coplanar forces are acting
on a rigid body, then these forces can be
Their resultant R is the sum of these forces
replaced by a single force which has the same
given by:
effect on the rigid body as that of all the forces
R = F 1 + F2 + F3
acting together. This single force is known as
If any one of these forces (say force F2 ) is
the resultant of several forces. Hence a single
acting in the opposite direction, as
force which can replace a number of forces
shown below.
acting on a rigid body, without causing any
change in the external effects on the body, is
known as the resultant force.
F1 F2 F3
The resultant of coplanar forces may be
determined by the following two methods :
Then, their resultant will be given by:
1. Graphical method
R = F1 - F2 + F3
2. Analytical method.

The resultant of the following coplanar forces Graphical method.


will be determined by the above two methods: In this method, a suitable scale is chosen and
(i) Resultant of collinear coplanar forces vectors are drawn to the chosen scale. These
(ii) Resultant of concurrent coplanar forces. vectors are added/ a or subtracted to find the
resultant.

Baffour Ba Series, Further Mathematics for Schools Page 341


The resultant of the three collinear forces F1, F2 Worked Examples
and F3 acting in the same direction will be 1. Three collinear horizontal forces of
obtained by adding all the vectors. magnitude 200 N, 100 N and 300 N are acting
on a rigid body. Determine the resultant of the
The force F1 = ab to some scale, force F2 = bc forces analytically and graphically when;
and force F3= cd. Then the length ad represents i. all the forces are acting in the same direction,
the magnitude of the resultant on the scale ii. the force 100 N acts in the opposite
chosen. Thus, R = /ad/ direction.
F1 F2 F3
a Solution
b c d
Method 1 : Analytical method
R = F1 + F2 + F3
F1 = 200 N, F2 = 100 N and F3 = 300 N
i. When all the forces are acting in the same
Consider the body below. direction, then resultant is given by equation;
R = F 1 + F2 + F3
R = 200 + 100 + 300
F1 F2 F3 R = 600 N

ii. When the force 100 N acts in the opposite


The resultant of the forces F1, F2 and F3 acting direction, then resultant is given by equation:
on the body will be obtained by subtracting the R = F1 – F2 + F3
vector F2. This resultant is shown in the R = 200 – 100 + 300
diagram below. R = 400N

F1 F2 F3 Method 2: Graphical method


a c b d F1 = 200 N, F2 = 100 N and F3 = 300 N
R = F1 - F2 + F3 Select a suitable scale, say 100 N = 1 cm. Then
to this scale, we have
The force F1 = ab as drawn to some suitable F1 = = 2 cm,
scale. This force is acting from a to b. The
F2 = = 1 cm ,
force , F2 is taken equal to be on the same scale
in opposite direction. This force is acting from F3 = = 3 cm
b to c. The force F3 is taken equal to cd. This i. When all the forces act in the same direction.
force is acting from c to d. The resultant force Draw vector ab = 2 cm to represent F1,
is represented in magnitude by ad on the vector bc = 1cm to represent F2 and
chosen scale. That is R = ad vector cd = 3 cm to represent F3 as shown
below:

Baffour Ba Series, Further Mathematics for Schools Page 342


F1 F2 F3 Solution
1
Let F1 = 470N and F2 = 538N
a 2 b c 3 d
F = F2 – F1 = 538 – 470 = 68N in the direction
Resultant = /ad/ of the 538 N force.
By measurement length ad = 2 + 1 + 3 = 6 cm
Resultant = Length ad × chosen scale Exercises
= 6 × 100 (Chosen scale is 1 cm = 100 N) 1. Find the resultant of the three forces on the
object.
= 600 N.
15N
10N
ii. When force 100 N = F2, acts in the opposite
10N
direction.
Draw length ab = 2cm to represent force F1 2. Find the net force on the system of the two
From b, draw bc = 1 cm in the opposite objects below:
direction to represent F2 . From c draw cd = 3
cm to represent F3 as shown below: 25N 10N
F1 F2 F3
a 2 c 1 b 3 3. In the diagram below, resolve the forces into
horizontal and vertical components to find the
Resultant = /ad/ resultant force on the object.
By measurement, /ad/ = 2 – 1 + 3 = 4 cm
0
:. Resultant = /ad/ × chosen scale 30
10N
= 4 × 100
= 400 N 3N

2. Determine the magnitude and direction of the Resultant of Concurrent Coplanar Forces.
resultant of the forces 1.3N and 2.7 N having Concurrent coplanar forces are those forces
the same line of action and acting in the same which act in the same plane and they intersect
direction. or meet at a common point. We will consider
the following two cases:
i. When two forces act at a point.
Solution
ii. When more than two forces act at a point.
Let F1 = 1.3N and F2 = 2.7N
F = F1 + F2 1. When two forces act at a point
F = 1.3 + 2.7 = 4N in the direction of the (a) Analytical method
original forces When two forces act at a point, their resultant
is found by the law of parallelogram of forces.
3. Determine the magnitude and direction of the
The magnitude of resultant is obtained from the
resultant of the forces 470N and 538 N having
the same line of action but acting in the equation /R/ = √ and the direction of
opposite direction.

Baffour Ba Series, Further Mathematics for Schools Page 343


resultant with one of the forces is obtained from Then resultant R = /OC/ × chosen scale.
the equation, θ = B C
P R
Suppose two forces P and Q act at point O as θ α
shown below: O Q A
B C
vi. Also the measure of angle θ, will give the
R direction of resultant.
P θ
α
The resultant can also be determine graphically
O Q A by drawing a triangle OAC as shown and
Let θ is the angle between them and α be the explained below;
angle made by the resultant R with the direction i. Draw a line OA parallel to P and equal to P.
of force P. ii. From A, draw a vector AC at an angle a with
the horizontal and cut AC equal to Q.
Forces P and Q form two sides of a
iii. Join OC. Then OC represents the magnitude
parallelogram and according to the law, the
and direction of resultant R.
diagonal through the point O gives the resultant C
R, determined as:
R
R = ⃗⃗⃗⃗⃗ = ⃗⃗⃗⃗⃗ + ⃗⃗⃗⃗⃗ Q
R=. /+ ( ) α θ
O P A
The magnitude of resultant is given by: Magnitude of resultant,
R=√ cos R = Length OC × chosen scale.
The above method of determining the resultant The direction of resultant is given by angle α.
is also known as the cosine law method. Hence measure the angle α.

The direction of the resultant with the force P is Type 1: (known angle at the y – axi)s
Worked Examples
given by θ = . / 1. Forces of 7.6N at 320 and 11.8N at 1430 act
at a point. Calculate the magnitude and
(b) Graphical method direction of their resultant.
i. Choose a convenient scale to represent the
forces P and Q. F1 = 7.6N 0
Solution 69
ii. From point O , draw a vector OA = P. Let F1 = 7.6N 320 11.8
iii. Now from point 0, draw another vector OB F2 = 11.8N 580
O 0 θ
53 N
F
= Q and at an angle of θ as shown below.
R
iv. Complete the parallelogram by drawing
lines AC parallel to ⃗⃗⃗⃗⃗ and BC parallel to ⃗⃗⃗⃗⃗ . F2 = 11.8N
7.6N
690
v. Measure the length OC.

Baffour Ba Series, Further Mathematics for Schools Page 344


Resolving along the vertical axis; Using the cosine rule,
F1 = . /=( ) FR2 = F12 + F22 – 2(F1) (F2) cos 690
FR2 = (7.6)2 + (11.8)2 – 2(7.6) (11.8) cos 690
F2 = . /=( )
FR = √ cos
Let the resultant force be FR;
FR = √
FR = F1 + F2
FR = 11.52 N (2 d.p)
FR = ( )+( )
FR = ( ) To find the direction of the resultant force, use
sine rule;
/FR/ = √ Let < F1OFR = θ
/FR/ = √( 88) ( 8 ) =
/FR/ = √ 8 ⇒ =
/FR/ = 11.52N (2 d.p.)
θ sin θ =
FR sin θ = 0.9527
Direction of the resultant;
From the diagram, the direction of the resultant θ= (0.9527)
0
force = 900 + θ θ = 73
⃗⃗⃗⃗⃗⃗⃗ = (11.52N, 1050)
But θ = . /
From the diagram, the direction of the resultant
θ = 150 (nearest degree)
force;
= 320 + θ
Direction of the resultant force
= 320 + 730
= 900 + 150
= 1050
= 1050
⃗⃗⃗⃗⃗⃗⃗ = (11.52N, 1050)
2. Two forces with magnitude 150N and 100 N
act on an object at angles of 400 and 1700
Method 2
respectively. Find the magnitude and direction
Conesider one part of the parallelogram which
of the resultant force, (round to 2 decimal
gives the triangle beow
palces in all intermediate steps and the final
F1 answer)
690
0
32 Solution
O Let F1 = 150 N and F2 = 100 N
θ
F2
FR

Baffour Ba Series, Further Mathematics for Schools Page 345


Method 2 : Using cosine rule
Magnitude of the resultant force;
0
50 R=√ ( )( )
F1 100 N
R = 114.76N (2d.p)
0
FR
50
0
3. Determine the magnitude and direction of the
80
resultant of the force in the diagram below;
0 150 N
50 150 N
F2
300
Resolving along the vertical axis; 200 N
F1 = . /=( ) Solution
Method 1: Resolving along the vertical axis;
F2 = . /=( ) 150 N

Let the resultant force be FR; 300


FR = F1 + F2 200 N

FR = ( )+ ( ) Resolving along the vertical axis;


FR = ( ) F1 = . /=( )
F2 = . /=( )
Magnitude of the resultant force;

/FR/ = √ Let the resultant force be FR;


/FR/ = √( 8 ) ( ) FR = F1 + F2
FR = ( ) +( )
/FR/ = √ 8 FR
/FR/ = 114.96N (2 d.p.) FR = ( )

θ Magnitude of the resultant force;


From the diagram , the direction of the resultant
/FR/ = √( ) ( )
force = 900 – θ
/FR/ = √ 8 8
But θ = . /
/FR/ = 102.66N
θ = 80 (nearest degree)
Direction of resultant force;
Direction of the resultant force;
θ = . /
= 900 – 80
0
= 820 θ = 47
⃗⃗⃗⃗⃗⃗⃗ = (114.96N, 0820)
Method 2: Using the cosine rule
F2 = (150)2 + (200)2 – 2(150)(200) cos (2100 – 1800)
Baffour Ba Series, Further Mathematics for Schools Page 346
F2 = (150)2 + (200)2 – 2(150)(200) cos 300 | | = 6.3N
F2 = 10538.4757
F=√ 8 Direction of the resultant force;
F = 102.66N = 900 + θ
150 N 200N F=( ) (lies in the fourth quadrant)
F 150N θ= . /
300 150
0 0
30
200 N θ = 71.70 θ
θ = 720
Direction of the resultant force; 6.3N
Using the sine rule; F
Direction of resultant;
= = 900 + 720
F
⇒ =
150 = 1620
θ N0
30 The resultant force is (6.3N, 1620)
sin θ = 200
N
θ= 0.7306 Type 2: (Two forces, and an included angle)
0
θ = 47 If the two forces are P and Q, and the angle
F = (102.66N, 0470) between them is θ, then the resultant force;
R = ∑Fx + ∑Fy
4. Force A (8 N, 0300) and B (10 N, 1500) act Where ∑Fx = P + Q cos θ
on a particle. Find the magnitude and direction ∑Fy = Q sin θ
of the resultant force.
Worked Examples
Solution 1. Two forces of 3N and 4N are acting at a
A (8 N, 0300) and B (10 N, 1500) point such that the angle between them is 600.
Let the resultant force be F Find the :
F= + i. resultant force,
=. /=( ) ii. magnitude of the resultant force,
ii. direction of the resultant with the force 4N
=. /=( )
Solution
F=( )+( ) i. Let P = 3N, and Q = 4N, θ = 600
F=( ) ∑Fx = 3 + 4 cos 600 y
C
=5 A
∑Fy = 4 sin 600
Magnitude of the force; 3N
= 3.4641 0 B
| | = √( ) ( ) 60 4N
x
| |=√ O

| |=√ The resultant force R;

Baffour Ba Series, Further Mathematics for Schools Page 347


= ∑Fx + ∑Fy
= 5 + 3.4641 D C
= 8.5 (1 d.p)
50
0 0
ii. The magnitude of the resultant force R; 60 120
A 80
| |= √ ) B
Let F1 = 80N, and F2 = 50N
| | = √( ) ( )
∑Fx = 80 + 50 cos 600
| |=√
= 105 kN
| | = 6.08 N
∑Fy = 50 sin 600
= 43.3 kN
Alternatively;
Magnitude of the resultant force; The resultant force R;
R=√ ( )( ) = ∑Fx + ∑Fy
R=√ = 105 + 43.3
R = 6.08N = 148.3N

ii. Direction of the resultant with force 4N; The magnitude of resultant R;
| |= √ )
θ= . /
| |= √ )
θ = ( 8) | | = 113.58 kN
θ = 350 (with force P that is 3N)
Alternatively;
Direction of resultant with force Q or 4N; Let ⃗⃗⃗⃗⃗ = R (Magnitude of the resultant force).
= 600 – 350 R2 = 502 + 802 – 2(50) (80) cos 1200
= 250 R R2 = 2500 + 6400 – 8000 cos 1200
3N
0 R2 = 12900
35 0
25 R=√
4N
R = 113.58
R = 114kN
2. Find the resultant of two forces 80 N and 50
N acting at a point with an included angle of Exercises 12.2
60° by analytical method. Also draw the free 1. Use the cosine and sine rules to determine
body diagram. the magnitude and direction of the resultant of
the force 13N at 00 and 25N at 300.
Solution
Method 1: Resolving the forces 2. Use the parallelogram law to determine the
magnitude and direction of the resultant of the
force 9N at 1260 and 14N at 2230.

Baffour Ba Series, Further Mathematics for Schools Page 348


3. Two forces F1 and F2 with magnitude 10N 3. Two forces with magnitudes of 20N and 14N
and 12 N act on an object at a point P as shown and an angle of 550 between them are applied
below. F on an object. Find the magnitude and direction
of the resultant force to the nearest whole
F1 number.
F2

0 θ 4. Two forces with magnitudes of 48N and 65N


45 300
P and an angle of 650 between them are applied
Find the resultant force F acting at P as well as on an object. Find the magnitude and direction
its magnitude and direction. of the resultant force to the nearest whole
number.
4. Forces F1= (12N, 0300) and F2 = (8 N, 1000)
act on a particle at O. Find the magnitude and 5. Determine the of the resultant of the forces
direction of the resultant 47N at 790 and 58 N at 2470. Ans 15.5N

5. Two force with magnitudes of 70N and 40N Angle Between the Resultant Force and a
and an angle of 1300 between them are applied Force Vector
B Q C
on an object. Find the magnitude of the
resultant force to the nearest whole number.
R
P β P
6. Two force with magnitudes of 77N and 45N θ α
and an angle of 450 between them are applied O Q A
on an object. Find the magnitude and direction
of the resultant force to the nearest whole 1. Consider triangle OAC,
number. Then by cosine rule;
P2 = Q2 + R2 – 2(Q)(R) cos α
B. 1. Determine the magnitude and direction It follows that:
of the resultant force. α= . /
( )( )
a. 0 b.
10 ⃖⃗
20
0 2. Consider triangle OBC,
3N 20N Then by cosine rule;
2N
Q2 = P2 + R2 – 2(P)(R) cos β
15N

2. Two forces with magnitudes of 62N and 62N It follows that:


and an angle of 1450 between them are applied β= . /
( )( )
on an object. Find the magnitude and direction
of the resultant force to the nearest whole
2. Note that α + β = θ
number.

Baffour Ba Series, Further Mathematics for Schools Page 349


Worked Example 3. Two forces with magnitudes of 45N and 62N
Two forces with magnitudes of 40N and 42 N are applied to an object. The magnitude of the
are applied to an object. The magnitude of the resultant is 100N. Find the measurement of the
resultant is 31N. Find the measurement of the angle between the resultant vector and the
angle between the resultant vector and the vector of the 45 N force to the nearest whole
vector of the 42 N force to the nearest whole number.
number.
4. Two forces with magnitudes of 200N and
Solution 340N are applied to an object. The magnitude
Let P = 40N , Q = 42N and R = 31N of the resultant is 150 N. Find the measurement
of the angle between the resultant vector and
40
R = 31 the vector of the 200 N force to the nearest
α whole number.
42
Finding an Unknown Force
2 2 2
R = P – Q + 2RQ cos α. 1. Make a sketch of the problem
312 = 402 – 422 + 2(31)(42) cos α. 2. Resolve the forces (usually along the vertical
961 = 1600 – 1764 + 2604 cos α axis), to obtain the sum of the forces in terms of
961 – 1600 + 1764 = 2604 cos α the uknown.
1125 = 2604 cos α 3. Equate the sum obtained in (2), to the
cos α = magnitude of the resultant force to obtain the
original force. i.e. F2 = F12 + F22
α= . /
4. Evaluate F2 and find the sum of F12 and F22,
α = 640 using binomial expansion ( where necessary)
5. Solve to obtain the value of the unknown
Exercises 12.2C force.
1. Two forces with magnitudes of 15N and 10N
are applied to an object. The magnitude of the Worked Examples
resultant is 24 N. Find the measurement of the 1. A body at rest is acted upon by two forces
angle between the resultant vector and the
(40N, 0450) and (QN, 1350).
vectore of the 10 N force to the nearest whole
number. i. Express each of the forces as a column
vector.
2. Two forces with magnitudes of 6N and 18N
ii. If the magnitude of the resultant force is
are applied to an object. The magnitude of the
resultant is 13 N. Find the measurement of the 50N, find the force Q and the direction of the
angle between the resultant vector and the resultant force to the nearest degree.
vector of the 18 N force to the nearest whole
number.

Baffour Ba Series, Further Mathematics for Schools Page 350


Solution F=( )
Resolving along the vertical axis;
F1 = . / F1 = 40N Direction of the resultant force;
F1 = ( ) 0
45
= 900 – θ
0
45
θ= . / F
F2 = . /
θ
θ = 80
F2 = . /
F2 = QN
Direction of F;
Let /F/ be the magnitude of the resultant force = 900 – 80
/F/ = 50N = 820
F = F1 + F2 ⃗⃗⃗⃗⃗ = (50N, 0820)
F2 = F12 + F22
F=( ) + . / 2. When more than two forces act at a point
(a) Analytical method.
F=. /……….(1) The resultant of three or more forces acting at a
point is found analytically by a method which
F12 = (28.2843 + 0.7071Q)2 is known as rectangular components methods.
= 28.28432 + 2(28.2843)(0.7071Q) + In this method, all the forces acting at a point
0.7071Q)2 are resolved into horizontal and vertical
= 800 + 1131.3623Q + 0.4999Q2 components and then algebraic summation of
horizontal and vertical components is done
F22 = (28.2843 – 0.7071Q)2 separately. The summation of horizontal
= 28.28432 – 2(28.2843)(0.7071Q) + component is written as ΣFx and that of vertical
0.7071Q)2 as ΣFy. Then resultant R is given by:
= 800 – 1131.3623Q + 0.4999Q2 R= +
F2 = F12 + F22
502 = (800 + 1131.3623Q + 0.4999Q2 ) + (800 The magnitude of the resultant force;
– 1131.3623Q + 0.4999Q2)
| |=√
2500 = 1600 + 0.9999Q2
2500 – 1600 = 0.9999Q2
900 = 0.9999Q2 The angle made by the resultant with horizontal
Q2 = is given by: tan θ =
2
Q = 900
Q = 30N Illustration
Let four forces F1, F2, F3 and F4 act at a point 0
Put Q = 30 N in equation 1 as shown below.
( )
F=. ( )
/……….(1)

Baffour Ba Series, Further Mathematics for Schools Page 351


The resultant is obtained graphically by
y
drawing polygon of forces as shown and
F2 F1 explained below.
θ2 θ1 d F3 c
θ3 x
θ4
F4
F3 F2
F4 e
R
F1 b
The forces F1, F2, F3 and F4 are resolved into a
horizontal and vertical components as follows;
Resultant
F1 = . /, F2 = . /
F3 = . /, F4 = . / Steps
I. Choose a suitable scale to represent the given
forces.
Resultant force, F
II. Take any point a. From a, draw vector ab
F = F1 + F2 + F3 + F4
parallel to force OF1. Cut ab = force F1 to the
The angle (θ) made by resultant with x-axis is scale.
III. From point b, draw be parallel to OF2. Cut
given by tan θ =
be = force F2:
IV. From point c, draw cd parallel to OF3. Cut
b. Graphical method. cd = force F3.
The resultant of several forces acting at a point V. From point d, draw de parallel to OF4 . Cut
is found graphically with the help of the de = force F4 .
polygon law of forces, which may be stated as VI. Join point a to e. This is the closing side of
“If a number of coplanar forces are acting at a the polygon. Hence ae represents the resultant
point such that they can be represented in in magnitude and direction.
magnitude and direction by the sides of a
polygon taken in the same order, then their Magnitude of resultant R = Length ae × scale.
resultant is represented in magnitude and The resultant is acting from a to e.
direction by the closing side of the polygon
taken in the opposite order. Worked Examples
1. Three forces A, B and C of magnitude 30 N,
Let the four forces F1, F2, F3 and F4 act at a 25 N and 45 N respectively act at a point. The
point O as shown below. angle between the forces A and B is 35o and the
F1
force C makes an angle of 130° with the force A
F2
in the anticlockwise direction. Find the
O magnitude of the resultant force and its
direction.
F3 F4

Baffour Ba Series, Further Mathematics for Schools Page 352


Solution Method 2: Graphical Method
Method 1: Analytical Method 1. Take some suitable scale for drawing the
Let FA = 30 N, A force polygon.
B C
FB = 25 N 1 mm = 1 N
FC = 45 N 30N C B
0 25N
35 R
500 350
A
400 O
B
45N 500 A
C
2. Take an arbitrary point and draw a line
Considering the equilibrium and resolving the parallel to the force vector A equal to the
forces, along the horizontal axis; magnitude of the force A (30 N) in the same
direction from as per the suitable scale as
FA = . /, FB = . /, FC = . / shown in figure.

3. Draw a line parallel to the force B equal to


F = FA + FB + FC
the value of 25 N in the same direction as that
F=. / +. /+. / of B as shown in figure.
F=( )+( )+( )
4. Similarly, draw the line parallel to the force
F=( )
vector C. Now, we have obtained three sides of
a polygon and the polygon is in open state.
The resultant force, R,

R=√ 5. Now close the polygon. Measure this closing


side of the polygon and convert in to the scale.
R = √( ) ( ) This value gives the resultant force and the
R = √8 8 direction is as shown in the figure of the force
R = 29.4936 kN polygon.

The direction of the resultant, 2. The four coplanar forces are acting at a point
900 + 900 + 900 + θ as shown in the diagram below.
F2 156N y

θ= . /
0 0 F1 104
θ = 47 (with the horizontal) 24
N
0
10
3
0 x
O
The direction of the resultant,
= 900 + 900 + 900 + 470 F3 252 9
0

= 3170 N
F4 288
N

Baffour Ba Series, Further Mathematics for Schools Page 353


Determine the resultant in magnitude and F4 = . /=( )
direction:
i. analytically, ii. graphically.
Resultant force = Sum of vector forces;
R = F1 + F2 + F3 + F4
Solution
= ( ) + ( ) + ( ) +
Analytical method.
F1 = 104 N, F2 = 156 N, F3 = 252 N, and ( )
F4 = 228N. =( )
I. Consider force F1 = 104 N as shown below.
The magnitude of resultant,
F1 = 104
80
0 | | = √( 8 ) ( 8 ) .
0
10
| |=√
| | = 260.24 N
| | = 260 N
F1 = . /=( )
The direction of resultant;
II. Consider force F2 = 156 N. θ= . / 0
17
F2
730
156N 0
θ = 170 (at x – axis)
24
R 260N
0
66 Direction of the resultant force;
= 900 + 900 + (90 – θ)0
= 900 + 900 + (90 – 17)0
F2 = . /=( ) = 900 + 900 + 730
= 2530
III. Consider force F3 = 252 N. R = (260 N, 2530)

3
0 Method 2 : Graphical method.
F3
F2 156N y
252 N

F3 = . /=( ) 0 F1 104
24
0 N
10
3
0 x
IV. Consider force F4 = 228 N. O
F3 252 9
0

81
0 N
0 F4 288
9
288N N
The figure above shows the point at which four
F4
forces 104 N, 156 N, 252 N and 228 N are

Baffour Ba Series, Further Mathematics for Schools Page 354


acting. The resultant force is obtained Then vector de represents the force 228 N in
graphically by drawing polygon of forces as magnitude and direction.
explained below .
VI. Join point a to e. The line ae is the closing
y
252N c side of the polygon. Hence the side ae
d
represents the resultant in magnitude and
156N direction. Measure the length of ae.
228N
104N b By measurement, length ae = 10.4 cm
a x
. . Resultant, R = Length ae × Scale
Resultant R = 10.4 × 25 (·: 1 cm = 25 N)
e
R = 260N.
I. Choose a suitable scale to represent the given
forces. Let the scale be 25 N = 1 cm. Hence, the 3. Forces = (2i + 3j) N, = (-5i + j) N and
force 104 N will be represented by = 4.16 = 6i N act on a particle. Find the magnitude
and direction of the resultant force F.
cm. Force 156 N will be represented by =
6.24 cm, force 252 N will be represented by = Solution
= 10 . 08 cm and the force 228 N will be = (2i + 3j) N, = (-5i + j) N and = 6i N
Resultant force F;
represented by = 9.12 cm.
F= + +
F = (2i + 3j) + (-5i + j) + 6i
II. Take any point a. From point a, draw vector
F = (2i – 5i + 6i) + (3j + j )
ab parallel to line of action of force 104 N. Cut
F = 3i + 4j
ab = 4.16 cm. Then ab represents the force 104
N in magnitude and direction.
Magnitude of the resultant force;
| |=√
III. From point b, draw vector be parallel to
force 156 N and cut be = 6.24 cm. Then vector | |=√
be represents the force 156 N in magnitude and | |=√
direction. | |=5N
The magnitude of the force is 5 N
IV. From point c, draw a vector cd parallel 252
N force and cut cd = 10.08 cm. Then vector cd Direction of the force;
represents the force 252 N in magnitude and F = 3i + 4j (first quadrant) F
direction. θ=
θ
θ = 370
V. Now from point d, draw the vector de
parallel to 228 N force and cut de = 9.12 cm. 4. A body is acted on by three forces = (10N,

Baffour Ba Series, Further Mathematics for Schools Page 355


0900), = (15N, 1800) and = (5N, 2700). From the diagram, the magnitude and direction
Find to three significant figures, the magnitude of R is (8.1N, 1420)
of the resultant force.
Exercises 12.4
Solution 1. Determine the magnitude and direction of the
= (10N, 0900), = (15N, 1800) and following coplanar forces which are acting at a
F3 = (5N, 2700). point by resolution of forces:
A 12N horizontally to the right, B 20N acting
Resultant force F; 1400 to force A, force C 16N acting 2900 to
F= + + force A.
F=. /+. /+. /
2. Determine the magnitude and direction of the
F= ( )+ ( ) + ( ) resultant of the force 10N acting at 320 to the
F=( ) horizontal, 15N acting 1700 to the horizontal
and 20N acting at 2400 to the horizontal.
Magnitude of the resultant force;
| |=√ ( ) 3. Force 23N acts at 800 to the horizontal, force
| |=√ 30N acts at 370 to the force one, and force 15N
act at 700 to the force two. Find the magnitude
| |=√
and direction of the resultant force.
| | = 15.8N (3 s.f)

Solved Past Questions 4. The following coplanar forces act at a point;


1. A body is acted on by three forces (10N, Force A acts horizontally to the right, force B
0300), (15N, 1200) and (15N, 2400). 23N at 810 to the horizontal, force C 7N, 2100
Determine the magnitude of the resultant force to the horizontal, force D 9N at 2650 to the
by drawing, using a scale of 4 cm to 5N. horizontal, force E 28N at 3240 to the
horizontal. Determine the magnitude and
Solution direction of the resultant of the forces .
(10N, 0300), (15N, 1200) (15N, 2400)
Scale = 5N : 4cm 5. Four horizontal wires are attached to a
10N = 8cm, 15N = 12cm telephone pole and exert tensions of 30N to the
F1 (8cm, 300), (12cm, 1200), (12cm, 2400) south, 20N to the east, 50N to the north- east
and 40N to the north – west. Determine the
0
30 12cm
resultant force of the pole and its direction.
8cm 0
30
0
0 30 Challenge Problem
30 0 0
60 0 60 Four coplanar forces are acting on a body such
30 0 12cm
0 60
30 that they are in equilibrium. The vector diagram
for the force is such that the 60N force acts

Baffour Ba Series, Further Mathematics for Schools Page 356


vertically upward, the 40N acts at 650 to the cos θ = (√ ) (√ )
60N force, the 100N force acts from the nose of
the 40N and the 90N force acts from the nose cos θ =

of the 100N force. Determine the direction of θ= . /

the 100N and 90N forces relative to the 60N 0
θ = 27
force.

2. Three forces – 63j , 32.14i + 38.3j and 14i –


Finding the Acceleration
24.25j act on a body of mass 5 kg. Find correct
1. Three forces = (2i - j) N, = (-5i + 3j) N
to one decimal place,
and = 6i – j act on a particle of mass
i. the magnitude and direction of the resultant
0.001kg. Find:
force.
i. the acceleration of the particle ;
ii. the acceleration of the body.
ii. the acute angle between the resultant force
on the particle and the vector i + j , correct to
Solution
one decimal place.
– 63j , 32.14i + 38.3j and 14i – 24.25j
Solution
Resultant force F;
i. = (2i - j) N, = (-5i + 3j) N and = 6i – j
F = (– 63j) + (32.14i + 38.3j) + (14i – 24.25j)
Resultat force F;
F = (32.14i +14i) + (38.3j – 24.25j – 63j)
F= + +
F = 46.14i – 48.95j
F = (2i – j) + (-5i + 3j) + 6i – j
F = (2i – 5i + 6i) + (-j + 3j – j)
Magnitude of the force;
F = 3i + j
| | = √( ) ( 8 )
F = ma; | |=√ 88
a= = = (3i + j) = 1000 (3i + j) | |=√
| | = 67.3N
ii. the acute angle between the resultant force
Direction of the resultant force;
on the particle and the vector i + j
= 900 + θ
Let V = i + j = ( ) and F = 3i + j = ( )
F = 46.14i – 48.95j
FV = ( ) ( ) (dot product)
F=( ) (lies in the fourth quadrant)
FV = 3 + 1 = 4
θ= . /
0
Angle between F and V; θ = 47
cos θ =
Direction of the resultant in the 4th quadrant
cos θ = = 900 + 470
(√ ) (√ )
= 1370

Baffour Ba Series, Further Mathematics for Schools Page 357


The resultant force is (67.3N, 1370) a = (2 , 0300)
a=. /
ii. F = ma
a = .√ /
a= = = 13.5

3. A body of mass 6 kg, initially at rest, acted /a/ = √ (√ )


upon by two forces = (12N, 900) and
= (3N, 3300). Find the magnitude of its /a/ = √
acceleration. /a/ = 2

Introduction to Law of Polygon


Solution
If a body is subjected to many forces on its
= (12N, 900) and = (3N, 3300).
plane at a single point then they are called
=. /=( ) as coplanar concurrent forces. The effect of
=. /=( ) the forces acting on the body is unknown. It is
necessary to determine the resultant force of the
coplanar forces to know this effect. There are
Resultant force, F;
many methods to determine the resultant force.
F= +
F= ( )+ ( )
Equilibrium of Forces
F=( ) In general, an object can be acted on by several
forces at the same time. If the size and direction
Magnitude of the resultant force; of the forces acting on an object are exactly
| |=√ ( ) balanced, then there is no net force acting on
| |=√ the object and the object is said to be
| | = 12N in equilibrium. In other words, for forces in
equilibrium, the sum of the component of the
Direction of resultant force; forces in any direction must be zero. That is:
F = F1 + F2 + F3……….+ Fn = 0
F=( ) (lies in the first quadrant)
Because there is no net force acting on an
θ= . / object in equilibrium, then from Newton's first
0
θ = 30 law of motion, an object at rest will stay at rest,
The resultant force is (12N, 0300) and an object in motion will stay in motion.

F = ma Polygon of Forces
a= = =2 For equilibrium, forces are represented in
Since acceleration acts in the same direction of magnitude and directin to form a polygon
the force shape.

Baffour Ba Series, Further Mathematics for Schools Page 358


If a number of forces are acting at a point , then ⇒ F1 + F2 + F3 = F
the missing side in the polygon represents the ( ) +( )+( ) =F
resultant force. ( ) =F

At equilibrium,
No equilibrium
Equilibrium F1 + F2 + F3 + G = 0
R But F1 + F2 + F3 = F = ( )

Note; the arrow direction on this force is in the ⇒F+G =( )


opposite direction to the rest. ( ) +G= ( )
G= ( )
Worked Examples G = -7i – 2j
1. Two forces 3i – 4j and -14i – 3j act on a
particle. Find the third force that will keep the 3. A particle at rest has forces (2i + 3j), (mi +
body in equilibrium . 6j) and (- 4i + nj) acting on it. What are the
values of m and n?

Solution Solution
Let F1 = 3i – 4j , F2 = -14i – 3j and F3 be the At equilibrium (at rest), all the forces add up to
third force that will keep the body in zero.
equilibrium. (2i + 3j) + (mi + 6j) + (- 4i + nj) = 0
⇒ F1 + F2 + F3 = 0 (2 + m – 4) i + (3 + 6 + n) j = 0
( ) + ( ) + F3 = ( )
The scalar multipliers of each unit vector equals
( ) + F3 = ( ) zero
F3 = ( ) + ( ) 2+m–4=0
m=4–2
F3 = ( ) m=2
F3 = 11i + 7j
3+6+n=0
2. Three forces 4i + 3j, -2i – 3j and 5i + 2j act n=-3–6
at a point O. Find the net force F and the n=-9
additional force G, such that equilibrium m = 2 and n = -9
occurs.
4. A particle at rest has an unknown force F and
Solution two other forces (2i – 5j), (-3i + j), acting on it.
Let F1 = 4i + 3j, F2 = -2i – 3j, F3 = 5i + 2j Find the magnitude of F to two decimal places.
F1 = ( ), F2 = ( ) , F3 = ( )

Baffour Ba Series, Further Mathematics for Schools Page 359


Solution F1 = . /= ( )
Method 1
Let F1 = 2i – 5j, F2 = -3i + j and F be the F2 = . /=( )
unknown force.
⇒ Let F1 = ( ), F2 = ( ), F = ? At rest,
F1 + F2 + F3 = 0
At equilibrium (at rest) , ( ) +( ) + F3 = ( )
F1 + F2 + F = 0
( ) + F3 = ( )
( ) +( ) + F = ( )
F3 = ( )
( )+F=( )
F=( )
Magnitude of F3;
Magnitude of F; /F3/ = √( 8 ) ( 8 )
/F/ = √ /F3/ = √ 8
/F/ = 4.12 N /F3/ = 4.14 N

Method 2 Direction of F3 = ( ),
Let F1 = 2i – 5j, F2 = -3i + j and F be the F3 lies in the second quadrant;
unknown force.
θ= . /
F3
θ = 450 4.14
At equilibrium (at rest) ,
0
45
F1 + F2 + F = 0
(2i – 5j) + (-3i + j) + F = 0 Direction of F3;
= 900 + 900 + 900 + 450
2i – 3i – 5j + j + F = 0
= 3150
-i – 4j + F = 0
F3 = (4.14N, 3150)
F = i + 4j
F=( ) 3. In the diagram below, find the force F, that
will keep the forces F1, F2 and F3 acting at point
/F/ = √ =√ = 4.12 N O in equilibrium.
y
F2 F1
5. A body of mass 2kg is at rest under the
7N
action of three coplaner forces F1, F2 and F3, 8N
where F1 = (8N, 0600) , F2 = (8N, 2100). Find 800 0
70 0 50
the magnitude and direction of the force F3. x

5N
Solution F3
m = 2 kg, F1 = (8N, 0600) , F2 = (8N, 2100)

Baffour Ba Series, Further Mathematics for Schools Page 360


Solution Exercises 12.5
Redraw the diagram as shown below; 1. Forces F1 = 12N, F2 = 30 N and F3 = 18N act
y in the directions of i, i – j and i + j respectively.
F2 F1
Calculate to two decimal places , the magnitude
7N
8N
and direction of the resultant force
0
800
500 500 x 2. In the diagram below, find the force F, that
20
will keep the forces F1 , F2 and F3 acting at
5N
point O in equilibrium.
y
From the diagram, resolving along the F1
horizontal axis;
6N
F1 = . /, F2 = . /, F3 = . / 1300
0 x
4N 120
At equilibrium,
F1 + F2 + F3 + F = 0 F2

. /+. /+. /+F=( )

( )+F=( ) Lami‟s Theorem


If three coplanar forces simultaneously acting
F=( )
at a point are in equilibrium, then each force is
F=( ) proportional to the sine of the angle of the other
two forces.
Solved Past Questions
1. Two forces F1 = 3i – 4j and F2 = -14i – 3j act Consider the forces A, B and C acting at a point
on a particle. Find the third force F3 that will O as shown below;
keep the particle in equilibrium. A

Solution
F1 = 3i – 4j, F2 = -14i – 3j, F3 = ?
⇒ F1 = ( ), F2 = ( ), F = ? α
B
At equilibrium (at rest) , C
F1 + F2 + F = 0 By Lami‟s theorem,
( ) +( ) + F = ( ) = =
( )+F=( )
F=( ) This is further illustrated as shown below
F = 11i + 8j

Baffour Ba Series, Further Mathematics for Schools Page 361


Method 2: Using triangle of forces
m n
α β T1
600
750
T1 T2
m αβ n 20N
450 T2

mg
Now let α + β = θ Using sine rule;
=
= = T1 =
T1 = 14.64 N
Worked Examples
1. A 2 kg mass is suspended by two light Using the sine rule;
inextensible strings inclined at 600 and 450 to =
the vertical. What are the tensions in the
strings? (g = 10m/s2) T2 =
T2 = 17.93 N
Solution The tensions in the strings are 14.6N and 17.9N
F = ma
F = 2 × 10 = 20N Method 3 : Using the vector method

600 450 0
45
600
T1 T2 T1 600 0 T2
1050 45
300 450

20N
Method 1
Using the sine rule; 20N
At rest,
=
T1 + T2 + W = 0
T1 =
T1 = 14.46 N Resolving along the horizontal axis;
. / +. /+ ( ) =( )
Using the sine rule;
. / +. / =( )
=

T2 = = 17.73 N - T1 cos 300 + T2 cos 450 = 0……….(1)


The tensions in the strings are 14.6N and 17.9N T1 sin 300 + T2 sin 450 = 20……….(2)

Baffour Ba Series, Further Mathematics for Schools Page 362


From eqn (1), =
T1 cos 300 = T2 cos 450
T1 =
T2 =
T1 = 50 N
T2 = 1.2247(T1)……………………..(3)
Using the sine rule;
Put T2 = 1.2247(T1) in eqn (2);
=
T1 sin 300 + 1.2247(T1 ) sin 450 = 20
0.5 (T1) + (1.2247)(0.7071) (T1) = 20 T2 =
1.3660 (T1) = 20 T2 = 86.6 N
T1 = The tensions are 50N and 86.6N
T1 = 14.64N
Method 2: Using triangle of forces
Put T1 = 14.64 N in eqn (3); T1
600
T2 = (1.2247)(14.64)
T2 = 17.93 100N
The tensions in the strings are 14.6N and 17.9N 300 T2

2. A body of mass 10 kg is suspended by means


of two light inextensible strings ̅̅̅̅ and ̅̅̅̅ Using sine rule;
which are inclined at 600 and 300 respectively =
to the downward vertical. Let T1 and T2 be the
T1 =
tensions in ̅̅̅̅ and ̅̅̅̅ respectively. Calculate
the values of T1 and T2. T1 = 50 N

Solution Using the sine rule;


F = ma =
F = 10 × 10 = 100N
T2 =

A B T2 = 86.6 N
0
30 The tensions in the strings are 50N and 86.6N
600
T1 T2
3. An object of mass 2kg is suspended by
P means of an inextensible string. The object is
then drawn aside by means of a horizontal force
of magnitude F newtons until the string makes
100N an angle of 300 with the downward vertical. If
Method 1 T is the tension in the string,
Using sine rule; i. express as vectors in component form all the

Baffour Ba Series, Further Mathematics for Schools Page 363


forces acting on the object. Method 2: Using triangle of forces
ii. write down a vector equation connecting all T1
300
the forces in (i)
iii. Use your vector equation to obtain the 200N
values of F and /T/, each to one decimal place. 600 T2
(Take g = 8m/s2)

Solved Past Question Using sine rule;


1. A body of mass 20 kg is suspended by two =
light inextensible strings ̅̅̅̅ and ̅̅̅̅ which are
inclined at 300 and 600 respectively to the T1 =
downward vertical. Find the tension in both T1 = 173.2 N
strings (Take g = )
Using the sine rule;
Solution =
F = ma
T2 =
F = 20 × 10 = 200N
T2 = 100 N
X Y
0
60 The tensions are 173.2N and 100N
300
T1 T2
2. A bucket of water of mass 20kg is being
P pulled out of a well. Find the magnitude of the
tension in the rope when:
i. it is moving with a constant speed;
200N ii. it has an acceleration of 0.3 ( g = 9.8 )

Solution T
Method 1: Using the sine rule
Using sine rule; i. Let T be the tension in the
= rope
For the bucket to move up,
T1 = a
T > mg
T1 = 173.2 N
Net force on the bucket of
Using the sine rule; water;
mg
= = T – mg

T2 = But F = ma
T2 = 100 N ⇒ ma = T – mg
The tensions are 173.20N and 100.00N At constant velocity, a = 0

Baffour Ba Series, Further Mathematics for Schools Page 364


⇒20 (0) = T – 20 (9.8) 4. a. A bucket of water is suspeneded by three
0 = T – 196 chords of negligible mass. The tensions in the
T = 196N chord are T1, T2 and T3. Dwar a vector diagram
to show te addition of the three forces acting at
ii. ma = T – mg point P.
T = ma + mg
300 400
At a = 0.3 T1 T2
T = 20 (0.3) + 20 (9.8)
T3 P
T = 6 + 196
M
T = 202 N
b. Given T3 = 20N, find T1 and T2.
Exercises 12.5B
1. A load of 12.5N is lifted by two strings 5. In each of the figure below, find the tensions
connected to the same point on the load, in each chord for the system.
making angles 220 and 310 on opposite sided of a.
the vertical. Determine the tensions in the 400 500
T1 T2
strings.
T3 P
2. A two legged string and hoist chain used for 5kg M
lifting machine parts is shown below.
Determine the forces in each leg of the sling if 600
b.
T1
parts exerting a downwad force is 15kN are
lifted. T2

T3
3. Three chords are knotted at point P, with two
10kg
of thses chords fastened to the ceiling making
angles of θ1 and θ2 and a block of mass m hangs
from the third one as shown below: 6. A mass of 100 g is hanging from two
M N massless ropes attached to the ceiling. One rope
θ1 θ2 makes an angle of 600 with the ceiling , while
the other make an angle of 290. Find the tension
P in the two ropes.
M
a. Find the magnitude of the tension in each 7. There are two strings, one with an angle of
chord in terms of θ1, θ2 and m, so that the 250, and the other with an angle of 650, and a
system is at rest. mass 5 kg. Label the tension from the strings as
b. Find the numerical values of the three T1 and T2 respectively and find their values.
tensions found above for θ1 = 450 and θ2 = 300 (Ans 20.7N; 44.4N)
and m = 1 kg

Baffour Ba Series, Further Mathematics for Schools Page 365


Moments of Forces Principles of Moments
Moments or torque of a force about a turning When an object is in equilibrium, the sum of
point is the force multiplied by the the anti – clockwise moment about a turning
perpendicular distance to the force from the point must be equal to the sum of the clockwise
turning point (reference point). In other words, moments. This is known as the principles of
it is the rotational effect of a force calculated by moment.
multiplying the force by the perpendicular
distance between a pivot and the force. 0 1=0 1

Moments are measured in Newton meter (Nm)


Illustrations
Mathematically;
Mo = Fd , where: ∆
F is the force in newtons; F1 F2
d is the perpendicular distance in meters.
Sum of anti - clockwise moments;
= ×
Note:
Sum of clockwise moments;
1. The weight of a body is concerntrated at the
= ×
center.
2. The resultant force on the body must to zero.
3. The resultant moment of the forces on the 0 1=0 1
body about all points must to zero. × = ×

Worked Examples Also


1. A 20N force acts at a perpendicular distance
of 0.50 m from the turning point. What is the
moment of the force? ∆

Solution
F = 20N, d = 0.50m Sum of anti - clockwise moments;
0.5 = ×
m
Sum of clockwise moments;
Turning
point =( )+( )
20N

Mo = Fd 0 1=0 1
Mo = 20 × 0.5
Mo = 10 Nm × =( )+( )

Baffour Ba Series, Further Mathematics for Schools Page 366


Worked Examples Solution
A uniform beam PQ of length 100 cm weight /AD/ = 30m
35 N . It is placed on a support at a point 40 cm If C is the mid point of AD, then
from P. Weights of 54N and W are attached to /AC/ = /CD/ = 15m
P and Q respectively, to keep the beam in C
A B D
equilibrium. Find, correct to the nearest whole
number, the value of W. 5m 10m 15m

Solution W N 5N 8N 20N
Taking moments about C;
Sum of anticlockwise moments;
∆ = (W × 15) + (5 × 10)
54 N
Sum of anti clockwise moments;
Sum of clockwise moments; = 20 × 15
=( )+( )
0 1=0 1
Sum of anti - clockwise moments; (w × 15) + (5 × 10) = 20 × 15
= 54 × 40 15W + 50 = 300
15W = 300 – 50
0 1=0 1 15W = 250
( )+( ) = 54 × 40 W = 16.7N
350 + 60W = 2160
60 W = 2160 – 350 3. A uniform bar PR of length 100 cm and
60W = 1810 weight 25 N rest on a support at its midpoint Q.
A force of 10N acts vertically downwards at R
W=
and another force of 10N acts vertically at S. If
W = 30.17N the bar is in equilibrium, and the force x acts at
W = 30N (Nearest whole number) P, find x.
100 cm
2. In the figure below, find the value of the P S Q R
weight W if AD is a uniform rod of length 30m
25cm
and C is the mid-point of AD. x 10N 10N
C Solution
A B D
100 cm
5m P S Q R

25cm 25cm 50cm


WN 5N 8N 20N
x 10N 10N

Baffour Ba Series, Further Mathematics for Schools Page 367


Taking moments about Q; 5. Taking moments about R1;
Sum of anticlockwise moments; Anti clockwise moments;
= (x × 50) + (10 × 25) =( )+ ( )

Sum of anti clockwise moments; Clockwise moments ;


= 10 × 50 =( ( )) + ( )

0 1=0 1 Worked Examples


(x × 50) + (10 × 25) = 10 × 50 1. A uniform beam MN of weight 20 N and
50x + 250 = 500 length 60cm is supported at two points X and
50x = 500 – 250 Y, such that /MX/ = 10 cm and /YN/ = 15 cm.
50x = 250 Two forces 10N and 15N are placed at M and
x=5N N respectively. If the system remains in
equilibrium under the action of these forces,
Involving Upward Forces / Reactions calculate the reactions at X and Y ( g = 10 )
Illustration
Consider the diagram below and observe the Solution
following:
M N
10 cm ∆ 20 cm 15 cm ∆ 15 cm
X Y
10N 20N 15N

Taking moments about Y;


Anti clockwise moments = (10 × 45) + (20 × 15)
Clockwise moments = (15 × 15) + ( × 35)
1. and are the upward forces, also called
the forces of reaction at and
0 1=0 1
respectively.
2. , and W are the downward forces.
3. For the system to be in equilibrium, total (10 × 45) + (20 × 15) = (15 × 15) + ( × 35)
upward force should be equal to total 450 + 300 = 35 + 225
downward force. That is : 450 + 300 – 225 = 35
+ = + +W 525 = 35
4. Taking moments about R2; = 15N
Anti clockwise moments;
( )) + ( )
Taking moments about X;
=(
Anti clockwise moments;
Clockwise moments ; = (10 × 10) + ( × 35)Clockwise moments;
=( )+ ( ) = (15 × 50) + (20 × 20)

Baffour Ba Series, Further Mathematics for Schools Page 368


0 1=0 1 Solution
i. Weight W = mg = m × 10 = 10 m N
(10 × 10) + ( × 35) = (15 × 50) + (20 × 20) 100N
100 + 35 = 750 + 400
C
35 = 750 + 400 – 100 A
∆ ∆
B
3.5m 1.5m 5m
35 = 1050
= 30 80 10m N
The reactions at X and Y are 15N and 20 N
Taking moments about A;
respectively.
(80 × 3.5) + (10 m × 5) = (100 × 10)
280 + 50m = 1000
Alternatively;
Taking moments about Y; 50m = 1000 – 280
Anti clockwise moments = (10 × 45) + (20 × 15) 50m = 720
Clockwise moments = (15 × 15) + ( × 35) m = 14.4 kg

ii. Total upward force = Total downward force


0 1=0 1
+ 100 = 80 + (10 × 14.4)
+ 100 = 80 + 144
(10 × 45) + (20 × 15) = (15 × 15) + ( × 35)
= 80 + 144 – 100
450 + 300 = 35 + 225
= 124 N
450 + 300 – 225 = 35
525 = 35
3. A uniform beam has length 8m and mass 60
= 15N
kg. It is suspended by two ropes as shown in
the diagram below:
Total upward force = Total downward force
T1 T2
+ = 10 + 20 + 15
1m 3m 2 2m
15 + = 10 + 20 + 15
= 10 + 20 + 15 – 15
W
= 30 N Find the tension in each rope.
The reactions at X and Y are 15N and 20 N
respectively. Solution
W = mg = 60 × 9.8 = 588 N
2. A uniforn plank AB, 10 m long and of mass
m kg, rests on two supports A and B. A load of
5m
8 kg is placed on the plank at point C, such that
/AC/ = 3.5 cm. The reaction at B is 100 N. If the
1m 3m 2m 2m
system remains in equilibrium under the action
of these forces, calculate: 4m 4m
i. the value of m;
588 N
ii. the reaction at A. (g = 10 )
Baffour Ba Series, Further Mathematics for Schools Page 369
Taking moments about : Taking moments about :
5 × = 3 × 588 Clockwise moments
5 = 3 × 588 2× = 1.5 × 490
= = 352.8N 2 = 1.5 × 490
= 353N ( 3 s.f. ) = = 367.5N
= 368N ( Nearest whole number )
Taking moments about :
5 × = 2 × 588 Taking moments about :
5 = 2 × 588 2× = 0.5 × 490
= = 235.2N 2 = 0.5 × 490

= 235N ( 3 s.f. ) = = 122.5N


= 123N ( Nearest whole number )
Checks
+ = vertical equation Checks
+ = 352.8 + 235.2 + = 490 = vertical equilibrium
+ = 588N + = 122.5 + 367.5
+ = 490 N
3. A beam of mass 50 kg and length 5 m rest on
two supports as shown in the diagram below . ii. Let consider the greatest mass that can be
placed at the left end of the beam .
1m 2m
∆ 2m
1m 1.5m 0.5m 2m
i. Find the magnitude of the reaction force
exerted by each support. 2.5m 2.5m
ii. Find the maximum mass that could be placed mg 490N
at either end of the beam if it is to remain in
equilibrium.
Taking moments about :
Solution 1 × mg = 1.5 × 490
W = mg = 50 × 9.8 = 490 N 1 × m × 9.8 = 1.5 × 490
m= = 75kg
2m
1m 1.5m 0.5m 2m Consider the greatest mass that can be placed at
the right end of the beam.
2.5m 2.5m
Taking moments about :
490N 2 × mg = 0.5 × 490
2 × m × 9.8 = 0.5 × 490

Baffour Ba Series, Further Mathematics for Schools Page 370


200x + 600 = 3520
200x = 3520 – 600
2m 200x = 2920
1m 1.5m 0.5m 2m x = 14.6 m

2.5m 2.5m Exercises 12.6


490N mg 1. A non uniform beam PQ of length 4m and
weight 100 N, rest horizontally on supports at
m= P and Q. If the center of gravity of the beam is
1.5m from P, find in newtons, the reaction at P.
m = 12.5 kg
Hence, the greatest mass that can be placed at
2. If the see – saw below is in equilibrium, what
either end is 12.5 kg.
is the weight of W. Ans 312N
4. A uniform pole, PQ, 30m long and of mass 4 2.5m 1.2m
kg is carried by a boy at P and a man 8m away
from Q. Find the distance from P where a mass W 650 N
of 20 kg should be attached so that the man‟s
support is twice that of the boy, if the system is 3. The diagram below shows a metre stick
in equilibrium (g = 10 ) pivoted at the centre.
0 10 m 50 m 80 m 100 m

Solution
R 2R 90 g M

P∆ ∆ What weight at M will keep the meter stick in


x 15 – x 7m 8m Q
equilibrium? Ans: 120g
200N
40N
4. If a uniform meter stick with its centre of
Total upward forces = Total downward forces gravity at C is supported on a pivot with
2R + R = 200N + 40N weights attached on either side as shown below
3R = 240N in the diagam.
R = 80N
30cm 10cm
2R = 2 (80) = 160N
C m
4N 12 N
Taking moments about P;
Clockwise moment = 200x + (40 × 15) What is the mass of the metric stick? Ans: 0.2 kg
Anti clockwise = 2R × (15 + 7)
Clockwise moment = Anti clockwise moment 5. A force of 20N acts at a distance of 20cm
200x + 600 = 2R × 22 from a pivot. What is the moment of this force
200x + 600 = 160 × 22 about the pivot in Ncm?

Baffour Ba Series, Further Mathematics for Schools Page 371


6. A ruler is placed on a pivot. Three forces act Challenge Problem
on it as shown below: The figure below shows a beam which has a
pinned support at A and rest on a roller support
20N at B.

30cm 200 cm 100 cm 300 cm


70cm
m B

A
14N 6N 60N 50N 20N
a. Which force has no moment about the pivot? 375 cm
Expalin your answer.
ii. Calculate the moments of the other two Determine the reactive forces at the two
forces about the pivot and explain whether the supports due to the three concerntrated load
ruler is turning or not. shown if the beam is in equilibrium.

Baffour Ba Series, Further Mathematics for Schools Page 372


13 DYNAMICS Baffour Ba Series

Concept of Motion Acceleration is also a vector quantity. The S.I.


An object is said to be in motion if it changes unit of acceleration is meter per second squares
its position with time. A body which does not ( ).
move is said to be at rest, motionless, or
stationary. An objects state of rest or motion Equations of Motion
cannot be be changed unless it is acted on by a The equations of motion apply to any object
force. Motion is described in terms moving with constant acceleration. The
acceleration, velocity and displacement as equations are also known as SUVAT equations,
described below. because they each contains four of the letters s,
u, v, a, t. Here s represents displacement, then u
Displacement initial velocity, v final velocity, a acceleration
Displacement is the shortest distance form the and t time.
initial to the final position of an object. A
displacement vector represents the length and Given any three of these values, the other two
direction of the straight path. can be found using these equations.

Displacement is a vector quantity as it has both These equations are:


magnitude and direction, whereas distance is a v = u + at (1)
scaler quanty measuring only the length. The SI s = ut + at2 (2)
unit of displacement is meters (m).
v2 = u2 + 2as (3)
2
s = vt − at (4)
Velocity
Velocity is speed in a given direction. Speed s = (u + v)t (5)
describes only how fast an object is moving,
whereas velocity gives both the speed and Equation (1) comes intuitively: if an object has
direction of the objects motion being a vector a constant acceleration for a period of time, the
quantity. The S.I. unit of velocity is meter per velocity gained plus the initial velocity will be
second ( ). the final velocity. From this and equation (2),
which is derived using the knowledge from
Acceleration calculus that displacement is the integral of
Acceleration is defined as the rate of change of velocity, the other equations can be derived.
velocity with time. It is the rate at which an For example, v = u + at can be re – arranged to
object speeds up or slows down. If the object a= .
speeds up, it is said to have positive
acceleration and if it slows down, it is said to Put a = into equation (2) to obtain:
have negative acceleration called retardation. s = ut + . t2

Baffour Ba Series, Further Mathematics for Schools Page 373


s = ut + (v − u)t Using the third equation of motion:
v² = u² ± 2gh
s = [ u + (v − u) ] t
v² = (9)² + 2 × 10 × 95
s = (u + v)t …………….. equation (5). v² = 81 + 2 × 950
v² = 81 + 1900
Exercises 13.1 v² = 1981
Derive equations (3) and (4) from equations (1) v=√ 8
and (2). v = 44.508
v = 44.51
Worked Examples
1. A ball is kicked from rest along the ground at 3. A stone is released vertically downward from
a constant acceleration, after 3s the ball is the top of a roof 20m high. Assuming
moving with a velocity of 20m/s . How far has negligible air resistance, calculate:
the ball travelled in this time? i. the velocity of the stone when it is 10m above
the ground.
Solution ii. the velocity with which it hits the ground.
Initial velocity (u) = 0 m/s (body at rest)
final velocities (v) = 20m/s Solution
displacement (s) = ? i. s = 20 – 10 = 10m, u = 0, v = ?, g = 10 2

time (t) = 3 s v2 = u2 + 2gs


v2 = (0)2 + 2(10) (10)
s = (u + v) t. v2 = 200
v=√
s = (0 + 20) × 3
v = 14.14
s = 30m.
ii. h = 20, u = 0, g = 10
2. A stone is thrown horizontally with a v2 = u2 + 2gs
velocity of 9 from a height of 95m. v2 = (0)2 + 2(10) (20)
Calculate the speed at which the stone hits the v2 = 400
ground. (Take g = 10 )
v=√
v = 20
Solution
Final velocity of the stone = v
4. A skydiver falling freely under the force of
Height of projection, h = 95m
gravity has an initial velocity of 18 . How
Horizontal velocity of projection, u = 9
long will it take him to fall 50m? Neglect air
v = ? and g = 10
resistance and take the acceleration due to
2
gravity to be 9.8 .
Note : the stone is thrown horizontally, hence, g
= + positive

Baffour Ba Series, Further Mathematics for Schools Page 374


Solution 160 = 10t2
s = 50 m, u = 18 , a = 9.8 t=? t2 = 16
Substitute in eqn (2) t = 4s
50 = 18t + (9.8) t2,
The Range h;

Now, make t the subject in order to obtain; h = gt²

(2)50 = (2)18t + (2) · 9.8t2, h = gt²


100 = 36t + 9.8t2, h = × 10 × 4²
9.8t2 + 36t – 100 = 0 (divide through by 2)
h = × 10 × 16
4.9t 2 + 18t − 50 = 0.
h = × 160
Using the quadratic formula; h = 80m
√ Hence, the range = 80m
t=
√( ) ( )( )
t= Exercises 13.2
( )

1. A car is moving along a straight horizontal
t= road with a velocity of u. It accelerates
t=

or t =
√ uniformly at a rate of 15 for 25m giving a
velocity of 30 . What is the value of u?
t = 1.8481 or t = -5.5215 (ignore –ve answer)
t = 1.85s ( 3 signicant figures) 2. A bullet is fired horizontally with an initial
velocity of 550 m/s at a target located 100m
5. A bullet is fired horizontally with a velocity from the riffle. How much time is required for
of 60 from the top of a building 80m the bullet to reach the target?
high. Calculate, how far from the foot of the
building will the bullet be assumed to touch the 3. A bullet is fired horizontally at a velocity of
ground. (Assume g = 10 ) 300 m/s from a tower 20 m high. At what
distance did the bullet travel? A = 2.02s, 606m
Solution
u = 60 , h = 80m Motion Under Gravity
h = gt², Gravity always act downward, so when an
object is moving upward, that is, in upward
direction, the acceleration of the upward
substitute h = 80m, g = 10 and t =?
moving object is moving in opposite direction
calculate t, {time taken to reach the ground}
of acceleration due to gravity. In other words,
and substitute t in h = gt² to obtain the range.
acceleration due to gravity is coming down and
h = gt², acceleration of the moving upward object is
moving upward, hence, a negative sign comes
80 = (10)t²,
in. That is, accceleration of the moving upward
Baffour Ba Series, Further Mathematics for Schools Page 375
object = − acceleration due to gravity (because T = 2t
of the retarding effect of gravity as the object T = 2 × 7.5
rises). T = 15s

Worked Examples 2. A ball is thrown vertically with an initial


1. A tennis ball is thrown vertically upward velocity of 20 . Find :
from the ground with a velocity of 75 . a. the maximum height reached by the ball;
Calculate: b. the time it reached the maximum height.
a. the maximum height reached; c. the velocity and time as it returns to the
b. the time to reach the maximum height; ground. (Take g = 10 )
c. the time of flight of the ball.
Solution
Solution Initial velocity u = 20
a. u = 75 and g = −10 At the maximum height v = 0
s=?
Note :
The negative sign will not reflect in the solution Using v2 = u2 + 2as
but only if you are using the three equations (0)2 = (20)2 + 2(-10)s
of motion under the influence of gravity . 0 = 400 – 20s
20s = 400
At maximum height reached: s = 20
Final velocity, v = 0. The ball will attain a mximum height of 20m.
v2 = u2 + 2as
0 = u2 + 2gh b. Time taken to reach the maximum height;
-u2 = 2gh using v = u + at
( ) v = 20 + (-10)t
h= = = = 281.25 281.3m
( )
- 20 = - 10t
t = 2s
b. Time to reach the maximum height t:
From v = u + at c. Total time for the journeys
= 2s up + 2s down
v = u + gt = 4s
t= = = 7.5s
The velocity at the time of reaching the ground
c. Time of flight is two times the time to reach is:
maximum height. v = ?, u = 0, t = 2s
v = 0 + (-10) (2)
Time of flight, v = - 20
T = 2 × time to reach maximum height, t. v = 20 ( in the downward direction)

Baffour Ba Series, Further Mathematics for Schools Page 376


3. A ball is thrown vertically upwards with a Using the quadratic method;
velocity of 15 . Find its height when it is a = 1, b = -8 and c = 6
moving with a velocity of 25 (g = 10 ) √
t=
( ) √( ) ( )( )
Solution t= ( )
u = 15 , v = 25 ,s=? √
Thrown vertically upwards, means motion t=
against gravity. Thus a = -g = - 10 √ √
t= or t =
Using v2 = u2 + 2as
t = 0.8377 or t = 7.1673
(25)2 = (15)2 + 2(-10)s
t = 1s or t = 7s
625 = 225 – 20s
20s = 225 – 625
Solved Past Questions
20s = - 400
1. A particle is projected vertically upwards
s = - 20
with speed of 25 from a point on the
s = 20m
ground. Find:
i. the position of the particle after 4s.
4. A particle is projected vertically upwards
ii. the maximum height reached;
from a point O with speed of 40 . Find:
iii. the time taken to reach the maximum height.
i. the maximum height reached;
iv. the time when the particle is 30m above the
ii. the time when the particle is 30m above the
ground. (g = 10 )
point O, correct to the nearest whole number.
Solution
Solution
i. s = ? u = 25, a = - g = - 10, t = 4
i. v = 0 , u = 40 , a = -g = -10
2 2
using v = u + 2as Using s = ut + at2
02 = 402 + 2(-10) s s = 20 (4) + (-10) (4)2
0 = 1600 – 20s
s = 100 – 80
20s = 1600
s = 20
s= = 80 The position of the particle after 4s is 20m
The maximum height reached is 80m
ii. the maximum height reached;
ii. s = 30m, u = 40, a = -10 u = 25, a = - 10, s = ?, v = 0 (at the max height)
Using s = ut + at2 v2 = u2 + 2as
02 = (25)2 + 2(-10) s
30 = 40t + (-10) t2
0 = 625 – 20s
30 = 40t – 5t2 20s = 625
6 = 8t – t2
s= = 31.25m
t2 – 8t + 6 = 0

Baffour Ba Series, Further Mathematics for Schools Page 377


iii. The time taken to reach the maximum c. How long does the ball takes to hit the
height; ground after it reaches it highest point. A = 2.45
v = 0, u = 25, a = - 10 and t = ? d. What is its velocity when it returns to the
Using v = u + at level from which it started.
0 = 25 + (-10) t
10t = 25 5. A ball is thrown vertically upwards with a
t = 2.5 s velocity of 50 . Find the:
i. velocity of the body at the end of 4s.
iv. The time when the particle is 30m above the ii. maximum height reached by the body.
ground; iii. time it takes to reach a height of 120m.
s = 30m, u = 25, a = -10, t = ?
s = ut + at2 Motion along Inclined Planes
The inclined plane is a problem setting in
30 = 25t + (-10) t2
which a massive object is on a slope, and only
30 = 25t – 5t2 subject to motion in the direction down the
6 = 5t – t2 incline. Although gravity pulls an object
t2 – 5t + 6 = 0 straight down, the presence of the slope
(t – 2) ( t – 3) = 0 ( By factorization) prevents this. Because objects can't move
t = 2s or t = 3s through the solid surface of the incline, the
object is limited to movement along the surface
Exercises 13.3 of the incline.
1. A ball is thrown vertically upwards with an
initial velocity of 30 . What is the Inclined plane problems involving
maximum height that the ball will reach? gravity, forces of friction , moving objects etc.
require vector representations of these
2. A ball is thrown upwards with a velocity of
quantities. Components are better in
100 . When will the ball reach the
representing forces using rectangular system of
ground?
axes since they make calculations such as
the addition of forces easier. Free body
3. A ball is thrown vertically upwards with a
diagrams are also used as well as Newton's
velocity of 20 . Find the:
second law to write vector equations.
a. Find the time it reaches the maximum height.
b. Calculate the maximum height reached.

4. A ball is thrown vertically upwards with a θ


Parallel force
velocity of 24 .
Perpendicular force
a. How high does it rise? θ
= 15
b. How long does it take to reache its highest P
= W = mg
point?
2
0
Baffour Ba Series, Further Mathematics for Schools Page 378
5. Normal Force , also known as normal
R reaction (R) is always perpendicular to the
F surface, R = mg cos θ
R
θ
6. Frictional force acts against the parallel force
θ
and the normal force acts against the
perpendicular force.
θ
W Worked Examples
From the diagram, mg θ
1. A 2 kg box is put on the surface of an
W inclined plane at 27° with the horizontal. The
cos θ = ……….(1) R
surface of the inclined plane is assumed to be
R = W cos θ frictionless.
R = mg cos θ F
i. Draw a free body diagram of the box on the
inclined plane and label all forces acting on the
sin θ = ……….(2) box.
F = W sin θ ii. Determine the acceleration a of the box
F = mg sin θ down the plane.
iii. Determine the magnitude of the force
By substitution; exerted by the inclined plane on the box.
R
F Solution R
i. R F
θ
mg sin θ
mg cos θ θ
θ W mg sin θ
mg cos θ
For all inclined planes: 270 W
1. Weight always acts vertically (straight) mg
downward. Weight (W) = mg = where is Two forces act on the box: the weight W of the
the force of gravity. box and R, the force normal to and exerted by
the inclined plane on the box.
2. Objects move in the direction of net force,
ma ii. Down the plane,
F = mg sin θ
3. Objects on an angle have a force parallel to F = 2(10) sin 270
the surface, F = mg sin θ. The parallel force
F = 9.098N
acts down the plane.

4. Objects on an angle have a force But F = ma


perpendicular to the surface, = mg cos θ. The a= = = 4.549
perpendicular force acts perpendicular to the a = 4.5N (1 decimal place)
surface.
Baffour Ba Series, Further Mathematics for Schools Page 379
iii. Magnitude of the force exerted by the (Take g = )
inclined plane on the box.
|R| = |W| cos (27°) Solution
|R| = mg cos (27°) i. Acceleration of the particle,
|R| = 2 × 10 cos (27°)
R
|R| ≈ 17.8N R F
θ
2. A metal sphere of mass 2kg is placed on a mg sin θ
smooth inclined plane at angle of 300 to the mg cos θ
300 W
horizontal. When the sphere is released from mg
rest, it moves down the plane. Calculate: F = mg sin θ
i. the force that moves the sphere down the F = 8 (10) sin 300
plane. F = 40 N
ii. acceleration of the sphere down the plane.
(Take g = ) But F = ma
a= = =
Solution
i. R ii. Reaction between the particle and the plane;
R F R = mg cos θ
θ R = 8 (10) cos 300
mg sin θ R = 69.28N
mg cos θ
270 W
mg 4. A body of mass 3kg is placed on a smooth
Force that moves the sphere down the plane; inclined plane at angle of 400 to the horizontal.
F = mg sin θ Find, correct to one decimal place, the
F = 2 (10) sin 300 magnitude of the force required to keep the
F = 10 N body in equilibrium. (take g = 10 )

ii. Acceleration of the sphere down the plane; Solution


m = 2 kg, F = 10 N, a = ?
R
F = ma F
R
a= = θ
a= mg sin θ
mg cos θ
400 W
3. A body of mass 8 kg slides down a smooth mg
inclined at an angle of 300 to the horizontal. The force required to keep the body in
Find the: equilibrium;
i. acceleration of the particle, F = mg sin θ
ii. reaction between the particle and the plane. F = 3 (10) sin 400

Baffour Ba Series, Further Mathematics for Schools Page 380


F = 19.29 N Solution
F = 19.3 N ( one decimal place) i. Initial velocity, u = 10m/s, angle of projection
= 20°.
5. A book of mass 0.8 kg is placed on a plane Time of flight,
inclined at 300 to the horizontal. Find the t= =
( )
=
( )
= 1s
frictional force if the book:
i. does not slide; .
ii. The maximum height, h attained.
ii. slides with an acceleration of (Take
g= ) h= = = = 1.25 m
( )

Solution iii. The range,


i. R
F R= = = = 8.66 m
R
θ
mg sin θ Trial Test
mg cos θ
30 0
W A bullet is fired at an angle of 45° to the
mg horizontal with a velocity of 490m/s.
If the book does not slides, Calculate,
F = mg sin θ 1. The time taken to reach maximum height.
F = 0.8 (10) sin 300 a) 0.345s b) 48s c) 4.8s
F=4N d) 34.6s e) 3.54s

ii. If the book slides, then mg sin θ > F 2. The maximum height attained;
Net force acting on the book a) 610000 b) 61000 c) 600
= mg sin θ – F d) 61200 e) 6000

Net force = ma 3. The horizontal distance (range) from the


⇒ mg sin θ – F = ma point of projection.
0.8 (10) sin 300 – F = 0.8 × 12 a) 0.0243 b) 0.00242 c) 24000
4 – F = 0.96 d) 243 e) 2400
F = 4 – 0.96
F = 3.04 Exercises 13.4
1. A small block of weight 100 N is placed on
5. A ball is projected at an angle of elevation of an inclined plane which makes an angle of 30°
30° with an initial velocity of 10 . with the horizontal. Find:
Calculate: i. correct to one decimal place, the magnitude
a. the time of flight; of the force required to keep the body in
b. maximum height attained; equilibrium.
c. the range. ii. acceleration of the particle (Take g = 10 )

Baffour Ba Series, Further Mathematics for Schools Page 381


2. A cart weighing 420 N rests on a 23º incline. Worked Examples
Calculate the component of its weight that 1. A body of mass 15 kg is placed on a plane
presses the cart to the hill. inclined at angle of 400 to the horizontal.
Calculate :
3. Calculate the parallel force component of the i. the frictional force that will keep the body in
weight (6.1 × 105 N) of a car resting on a hill equilibrium.
which is 35º above horizontal. ii. the coefficient of friction. (Take g = )

4. A crate having a mass of 114 kg rests on a Solution


R
ramp with an angle of incline of 15.2º. What F
R
force does the crate exert perpendicular to the
θ
ramp? mg sin θ
mg cos θ
400 W
5. What is the weight of a box that exerts a
mg
parallel force component of 330 N and a The frictional force required to keep the body
perpendicular component of 390 N? What is the in equilibrium ;
angle of inclination of the hill? F = mg sin θ
F = 15 (10) sin 400
6. A block slides down an inclined plane with F = 96.418 N
increasing speed. The force on the block are
shown in the diagram below, and the net force ii. The coefficient of friction;
is 5N. F=µR

µ=
θ

0
F = mg sin θ
45
F = 15 (10) sin 400
20N F = 96.418
a. Draw a vector diagram to show the addition
R = mg cos θ
of the two forces to give the net force.
F = 15 (10) cos 400
b. Find the magnitude and direction of ⃗ F = 114.907

Friction and Coefficient of Friction Coefficient of friction,


The force of friction is directly related to the µ= = = 0.839
normal reaction. That is:
F R
3. A body of mass 5kg is placed on a plane
F=µR
inclined at an angle of 450 to the horizontal. If
Where µ is the constant of proportionality
called coefficient of friction. the coefficient of friction is , calculate correct

Baffour Ba Series, Further Mathematics for Schools Page 382


to one decimal place, the minimum force ii. Find the force of friction acting on the
required to move the body down the plane. particle.
iii. Find the normal force exerted by the
Solution inclined on the particle.
R F
P 2. A box of mass 10 kg rest on a 350 inclined
θ plane with a horizontal. A string is used to keep
mg sin 450 the box in equilibrium. The string makes an
0
mg cos 450 angle of 250 with the inclined plane. The
45 mg
coefficient of friction between the box and the
If the body is about to slide down the plane, inclined plane is 0.33.
frictional force act up the plane, thus opposing i. Draw a free body diagram including all forces
and limiting the motion. acting on the particle with their labels.
F=µR ii. Find the magnitude of the tension T in the
R = mg cos 450 string.
R = 5 (10) cos 450 iii. Find the magnitude of the force of friction
R = 50 cos 450 acting on the particle.
µ=
3. A 100 kg box is to be lowered at constant
speed down an inclined plane 4 meters long
F = (50 cos 450)
from the back of a lorry 2 meters above the
F = 17.677N ground. The coefficient of kinetic friction is
equal to 0.45. What is the magnitude of the
Let P be the minimum force required; force Fa to be applied parallel to the inclined
P + F = mg sin 450 plane to hold back the box so that it is lowered
P = mg sin 450 – F at constant speed?
P = 5 (10) sin 450 – 17.677
P = 35.355 – 17.677 Box
P = 17.7N 4m
2m Lorry
Exercises 13.5
1. A particle of mass 5 kg rests on a 30°
inclined plane with the horizontal. A force Fa of 4. i. If the coefficient of kinetic friction
magnitude 30 N acts on the particle in the between a 35-kg crate and the floor is 0.30,
direction parallel and up the inclined plane. what horizontal force is required to move the
i. Draw a free body diagram including the crate to the right at a constant speed across the
particle, the inclined plane and all forces acting floor?
on the particle with their labels. ii. Suppose the same 35 kg crate was not
moving at a constant speed, but rather

Baffour Ba Series, Further Mathematics for Schools Page 383


accelerating at 0.70 m . Calculate the applied measured in kilogram meters per second. If the
force. The coefficient of kinetic friction is still objects move in opposite directions before the
0.30. collision, the opposing velocities will partially
cancel one another out. After the collision,
5. A 100kg wooden crate rests on a wooden when the objects remain joined, they'll move
ramp with an adjustable angle of inclination. together with their combined momentum.
a. Draw a free body diagram of the crate.
b. if the angle of the ramp is set to 100, Steps
determine; I. Multiply the first objects mass by its velocity.
i. the component of the crate‟s weight that is For example, if it weighs 500 kg and travels at
perpendicular to the ramp. 20 , then its momemtum = MV
ii. the component of the crate‟s weight that is
parallel to the ramp Momemtum = 500 × 20 = 10,000 kg
iii. the normal force between the crate and the
ramp. II. Describe the second objects velocity in
iv. the static friction force between the crate terms of the first objects direction. For
and the ramp. examples, if the first object travels at 30
c. At what angle will the crate just begin to slip in the direction oppsite to the first object‟s
direction, multiply the velocity by -1 , giving
Momentum and Impulse the second object a velocity of – 30
Momentum
If the mass of an object is m and it has a III. Multiply the second object‟s nmass by its
velocity v, then the momentum of the object is velocity. For example, if it weighs 1,000 and
defined to be its mass multiplied by its velocity. has a velocity of – 30 , then its
That is momentum = mv. momentum will be 30,000 kgm/s.

Momentum has both magnitude and direction IV. Add the two velocities together to
and thus, it is a vector quantity. The units of determine which way the object will move after
momentum are kg m or newton seconds, collision. For example, a collision between an
(Ns). This is often referred to as linear object with a momentum of 10,000 kg
momentum in order to distinguish it from and an object with momentum of -30,000 kg
angular momentum. gives a result of – 20,000 kg . A
negative results means the object will move in
When two objects collide, their total the second object‟s original direction after the
momentum does not change. The total collision.
momentum, before and after the collision,
equals the sum of the objects' individual Worked Examples
momenta. For each object, this momentum is 1. A cyclist and his bike have a combined mass
the product of its mass and its velocity, of 100 kg and are travelling along a straight

Baffour Ba Series, Further Mathematics for Schools Page 384


horizontal road at 9 m . A motorcyclist and Impulse
her bike, which have a combined mass of 300 If a force F acts for a short time, t, on a body,
kg, travel in the opposite direction at 21 m . the impulse of F on the body is the quantity Ft.
What is the momentum of the cyclist and what It is a vector quantity and like momentum, has
is the momentum of the motorcyclist? units kg m or N s. That is ;
Impulse = Ft
Solution
Momemtum = MV Often, impulse is found indirectly by
If the direction of the cyclist is defined as considering the change in momentum of a
positive, then: body, which is especially useful when the force
momentum of cyclist; and time are unknown, but both the momentum
= 100 × 9 before and after can be found, as for example in
= 900 kg m a collision.

momentum of motorcyclist; Worked Example


= 300 × (−21) 1. A cyclist and his bike have a combined mass
= − 6300 kg m of 100 kg and are travelling along a straight
horizontal road at 9 . A motorcyclist and
2. Three forces are given by F1 = (5N, 0900), her bike, which have a combined mass of 300
F2 = 8N, 1200) and F3 = (4N, 2100). kg, travel in the opposite direction at 21 .
i. Write down the three forces as column What is the momentum of the cyclist and what
vectors. is the momentum of the motorcyclist?
ii. Find the magnitude and direction of the
resultant force, F, for the three forces , correct Solution
to the nearest degree. If the direction of the cyclist is defined as
iii. If F acts on a particle of 2 kg which is positive, then:
initially at rest, find the momentum acquired by Momentum of cyclist;
the particle after moving through a distance of = 100 × 9
3 meters. = 900 kg

3. A particle of mass 2kg lying on a smooth Momentum of motorcyclist;


horizontal table is acted on by two horizontal = 300 × (−21)
forces 60N, 0300) and 20N, 3150). Find: = − 6300 kg m
i. the magnitude and direction of the single
force F which is equivalent to the two given 2. A child kicks a stationary football, of mass
force; 0.5 kg, which then moves with a velocity of 15
ii. the acceleration of the particle under the two m . What impulse does the child impart on
given forces. the football?

Baffour Ba Series, Further Mathematics for Schools Page 385


Solution Momentum before blow;
Impulse = Change in momentum = mu
I = final momentum − initial momentum =3×2
I = mv – mu = 6 kgm/s
I = (0.5 × 15) – (0.5 × 0 )
I = 7.5 kg (2 s.f.) Momentum after blow;
= mv
3. A particle of mass 3kg is moving in a fixed =3×-4
direction with a speed of 2 . The particle is = - 12 kg
given a blow which causes it to change its
speed. Find the change in momentum if, after Change in ;
the blow, the particle: = final momentum − initial momentum
i. moves with a speed of 4 in the same = mv – mu
direction; = - 12 – 6
ii. moves with a speed of 4 in the opposite = - 18 kg
direction,
4. A particle of mass 2kg is moving in a straight
line with a velocity 2i – 3j. It is given a push
Solution
which causes it to move with a velocity 5i + j.
a.
3kg 3kg Find the change in momentum.
u=2 v=4
Solution
Momentum before blow;
Momentum before push;
= mu
= mu
=3×2
= 2 (2i – 3j)
= 6 kg
Momentum after push;
Momentum after blow;
= mv
= mv
= 2 (5i + j)
=3×4
= 12 kg Change in momentum;
= mv – mu
Change in momentum; = 2(5i + j) – 2 (2i – 3j)
= final momentum − initial momentum = 2[(5i + j) – (2i – 3j)]
= mv – mu = 2[(5i – 2i) + (j + 3j)]
= 12 – 6 = 2[(5 – 2)i + (1 + 3)j]
= 6 kgm/s = 2[3i + 4j]
b. = 6i + 8j
3kg 3kg
u=2 v = -4 Magnitude of change in momentum;
Baffour Ba Series, Further Mathematics for Schools Page 386
=√ 8 v = 12i + 3j
=√
= 10 kg in the direction of 3i + 4j Change in momentum;
= final momentum − initial momentum;
Some Solved Past Questions = mv – mu
1. A body of mass 0.5kg is moving in a straight = m(v – u)
line with a velocity - 2i + 6j. It is hit by an = 0.8 [( ) ( – )]
object which causes it to move with velocity = 0.8 ,( )-
4i – 3j. Find the: = 0.8 ,( )-
i. change in momentum; = 0.8 ( )
ii. magnitude of the change in momentum. = 6.4i + 4.8j kg

Solution ii. Magnitude of change in momentum;


i. m = 0.5kg, u = -2i + 6j and v = 4i – 2j = √( ) ( 8)
Change in momentum; =√
= final momentum − initial momentum; = 8 kg
= mv – mu
= m(v – u)
Exercises 13.6
= 0.5 [( ) ( – )] A. Calculate the momenta of the following in
= 0.5 ,( )- newton – seconds:
= 0.5 ,( )- 1. A table tenis ball of mass 4g moving at 20 m/s.
= 0.5 ( 8)
= -3i + 4j kg 2. A stone of mass 15kg moving at 5m/s

ii. Magnitude of change in momentum; B. 1. A rhino, of mass 1800 kg, is running at 9


= √( ) ( ) m . What is the rhino‟s momentum?
=√
2. The momentum of a bullet, of mass 19
= 5 kg
grams, is 8.55 kg m . What is the velocity of
the bullet?
2. A particle of mass 800g is moving in a
straight line with a velocity of 4i – 3j. It is
3. A badmington player hits a shuttlecock, that
given a blow which causes it to move with a
came over the net at 32 m , back in the same
velocity of 12i + 3j. Find:
direction that it came from, with a velocity of
i. the change in momentum;
28 m . If the shuttlecock has a mass of 40
ii. the magnitude of the change in momentum.
grams, what is the impulse that the player
(through the racket) imparts on the shuttlecock?
Solution
i. m = 800g = 0.8 kg, u = 4i - 3j and

Baffour Ba Series, Further Mathematics for Schools Page 387


4. A bus, of mass 4000 kg, reduces its speed with and as respective final velocities
from 25 to 12 in 8 seconds. What is after collision, the law can be written as:
the average braking force? + = +

5. Three forces (-10i + 4j), (16i – 10j ) and 3i N


in the x – y plane act on a body of mass 3kg. Note:
Find the magnitude and direction of the 1. If the particles move together after impact,
acceleration of the body. then:
+ = +
Conservation of (Linear) Momentum:
The law of conservation of momentum can be 2. If the particles move together after impact, in
explained from the second law of the same direction, then:
motion. Newton‟s second law of motion says + = +
that rate of change of linear momentum of a
body is equal to the net external force applied 3. If the particles move together after impact, in
on it. Mathematically, it is expressed as: the opposite direction, then:
– = +
= (mv)dt = m = ma = Fnet
Observe the following in the calculation of the
If the net external force acting on a body is common velocity, v, after impact;
zero, then the rate of change of momentum is 1. If the particles move together after impact,
also zero, which means that there is no change then:
in momentum. + =( + )v

Direct Impact 2. If the particles move together after impact, in


Two bodies of mass M and m are moving in the same direction, then:
opposite directions with the velocities v. If they + =( + )v
collide and move together after collision, there
is the need to find the velocity of the system. 3. If the particles move together after impact, in
Since there is no external force acting on the the opposite direction, then:
system of two bodies, momentum will be – =( + )v
conserved. Thus;
Initial momentum = Final momentum Worked Examples
(Mv – mv) = (M + m)VFinal 1. A lead ball A of mass 2 kg moving with
velocity ( ) collides with another ball B
From this equation, the final velocity of the
system can easily be found. of mass 1 kg which is at rest. If ball A now
moves with velocity ( ) , find the
For two objects with initial masses of and velocity with which ball B moves.
and initial velocities and respectively,

Baffour Ba Series, Further Mathematics for Schools Page 388


Solution 5 × 103 ( ) + 3 × 103 ( )
= 2 kg, =( ) , = 1 kg =( )+( )…………….(1)
=( ) , = ( ) and =?
By law of conservation of momentum, R.H.S.
momentum before impact = momentum after 5 × 103( ) + 3 × 103
impact; =( ) + 3000 ……………..(2)
+ = +
2( ) + 1( ) = 2( ) + 1 By law of conservation of momentum,
( )–( ) =( )+1 ( )+( )=( ) + 3000
( )–( ) = ( )+( )–( ) = 3000
= ( ) ( ) = 3000
= ( )
2. A car of mass 5 × 103 kg travelling with a ⁄
velocity (10 , 0900) collides with another =( *

car of mass 3 × 103 kg travelling with a velocity =( ) (express as distance bearing)
of (5 , 2700). If the velocity of the first car
= (5 m/s, 0900)
immediately after the collision is (4 ,
0
090 ), find the velocity of the second car
3. Two masses 20 kg and 15 kg moving with
immediately after collision.
velocities 25 and 20 respectively
Solution collide. They move together after collision.
Before collision Find their common velocities if they were
m1 = 5 × 103 kg , u1 = (10 , 0900) moving in;
m2 = 3 × 103 kg , u2 = (5 , 2700) i. the same direction;
ii. opposite direction.
After collision
m1 = 5 × 103 kg v1 = (4 , 0900) Solution
m2 = 3 × 103 kg v2 = ? i. In the same direction,
= 20 kg, = 25 , = 15 kg
By law of conservation of momentum, = 20 and v = ?
m1 u1 + m2 u2 = m1 v1 + m2 v2 Before impact
Now,
20
u1 = . /=( ) 20kg 25 15kg

u2 = . /=( ) After impact


35kg
v1 = . /=( )
v

L.H.S.
Baffour Ba Series, Further Mathematics for Schools Page 389
Let v be the common speed after collision. Solution
Momentum before impact; i. In the same direction,
+ =( )v Before impact
20(25) + 15(20) = (20 + 15)v (same direction)
8 5
500 + 300 = 35v 3kg 5kg
800 = 35v
v= After impact

v = 22.86 8kg
v
ii. In the opposite direction,
Before impact Let v be the common speed after collision.

20kg 25 20 15kg Momentum before impact;


+ =( )v
3(8) + 5(5) = (3 + 5)v (same direction)
After impact
24 + 25 = 8v
35kg 49 = 8v
v
v=
v = 6.125
Let v be the common speed after collision.
Momentum after impact; ii. In the opposite direction,
Before impact
– =( )v
20(25) – 15(20) = (20 + 15)v (opp direction) 5
kg 8
320kg 5kg
500 – 300 = 35v
200 = 35v After impact

v= 8kg

v = 5.71 v

4. A particle of mass 3 kg is moving along a


track at 8 towards another particle of Let v be the common speed after collision.
mass 5kg travelling at 5 on the same – =( )v
track. If the particles move together after 3(8) – 5(5) = (3 + 5)v (same direction)
impact, find the common velocity if before 24 – 25 = 8v
impact, they were travelling: -1 = 8v
i. in the same direction;
v=
ii. in opposite direction.
v = - 0 .125

Baffour Ba Series, Further Mathematics for Schools Page 390


5. A particle of mass 4 kg moving with velocity tan θ =
( ) collides with another particle of mass
θ= . /
6 kg moving with a velocity ( ) . They
θ = 90
move together after collision. If they were
θ
moving in opposite directions. Find: v
Direction of v ;
i. the magnitude of their common velocity.
= 900 + 900 + θ
ii. the direction, correct to their nearest degree.
= 900 + 900 + 90
= 1890
Solution
i. = 4 kg, =( ) , = 6 kg Some Solved Past Questions
=( ) v=? 1. A particle of mass 8 kg moves with a
Before impact velocity (12i + 8j) and collides with
another particle of mass 5 kg , moving with a
4kg 6kg velocity of (3i + 18j) . If after the impact,
they move together, find correct to two decimal
After impact
places, the magnitude of their common
10kg velocity.
v
Solution
= 8 kg, = (12 i + 8j) , = 5 kg
If they move in opposite direction; = (3i + 18j) and v = ?
– =( )v
4( ) – 6( ) = (6 + 4)v By law of conservation of momentum;
+ =( + )v
( ) – ( ) = 10v
– ( ) = 10v
By substitution,
v= – ( ) 8( ) + 5( ) = (8 + 5)v
v= ( ) ( ) + ( ) = 13v
v= ( ) ( ) = 13v
v= ( )
Magnitude of the common velocity;
v= ( )
/ v / = √( ) ( )
/v/=√
Magnitude of the common velocity;
/ v / = 6.08
/ v / = √(8 ) ( 8 )
/v/=6 (Nearest whole number)
/v/=√
ii. Direction of the velocity; / v / = 14.6066
/ v / = 14.61 (2 d. p)
Baffour Ba Series, Further Mathematics for Schools Page 391
2. A body of mass 1.5kg moving with a Exercises 13.7
velocity of 20 , collides with another 1. A truck of mass 3 × 104 kg travelling at 15
body of mass 3.5kg moving with the velocity 6 collides with another truck of mass 2 ×
4
, along the same line but in opposite 10 travelling at 10 . They moved together
direction. after collision. Find their common velocity, if
i. if they moved together after collision, find the trucks were travelling;
their common velocity. i. in the same direction;
ii. If the velocity of the 1.5 kg mass changes to ii. in the opposite direction.
3.4 , but moves in the opposite direction,
find the velocity of the 3.5 kg mass correct to 2. A truck of mass 3 × 104 travelling at 15
the neasres whole number. . collided with another truck of mass 2 ×
4
10 travelling at 10 . Assuming that they
Solution move on together, find their commom velocity
i. By law of conservation of momentum, if the if the truckd were travelling:
particles move together after impact, in the i. the same direction
opposite direction, then: ii. opposite directions
– =( + )v
= 1.5kg, = 20 , = 3.5 kg 3. A car of mass 750 kg was coating along a
=6 v=? horizontal road at 8 when it ran into a
By substitution; sheep of mass 50 kg standing on the road. Fter
1.5(20) – 3.5(6) = (1.5 + 3.6)v the collision, the sheep, the sheep stuck to the
30 – 21 = 5v front of the car. Find the speed of the car
9 = 5v immediately after collision. Ans 7.5m/s
v=
4. A car of mass 1000 kg travelling at (20
v = 1 .8 , 0090) collides with a lorry of mass 5000
Their common velocity is 1 .8 kg travelling at (15 , 0000) . Find the
magnitude and direction, to the nearest degree,
ii. By law of conservation of momentum; of the common velocity with which the vehicles
If the smaller mass moves in the opposite begin to move on together immediately after
direction after collision, then: the collision.
– = – +
1.5(20) – 3.5(6) = – 1.5 (3.4) + 3.5 5. A spaceship of mass 200t travelling at
30 – 21 = – 5.1 + 3.5 8005m/s was guided into the back of another
9 + 5.1 = 3.5 spaceship of mass 100t travelling in the same
14.1 = 3.5 direction at 7990 . The two space ships
= = 4.03 locked together after impact. Find their
common velocity immediately after locking
=4 (Nearest whole number)
together.

Baffour Ba Series, Further Mathematics for Schools Page 392


6. A bullet of mass 0.096 kg is fired the collision, the speed of the first ball is
horizontally into 12 kg block of wood on a reduced to 6 and the second ball travels at
smooth horizontal floor. If the velocity of the v m/s.
block and the bullet after impact is 0.48 , a. What is the total momentum of the two balls:
calculate to the nearest whole number, the i. before collision
initial velocity of the bullet. ii. after collision.
b. Calculate the velocity of the second ball after
Challenge Problems collision.
1. A machine gun of mass 8 kg fires a bullet of
mass 0.04 kg and recoils horizontally. If the 5. A lorry of mass 2 × 104 kg moving with a
barrel of the gun is horizontal and the bullet velocity (5 , 0000) collides with another of
travels along it with speed 500 , find the mass 1.5 × 104 kg moving with a velocity
speed with which the gun begins to recoils. i. Write down the two velocities as a column
vector.
2. A particle of mass 8 kg moving at 12m/s ii. If the lorries move on with a common
strikes another particle of mass 12 kg moving at velocity, find the magnitude and direction (to
6m/s in the same direction. If after impact, the the nearest degree), the common velocity.
particles stick together and eventually come to iii. Find the magnitude in newtons and the
rest after travelling 1.8m, magnitude of the constant force of resistance
i. find their common velocity after impact and which would bring the combined lorries to rest
their subsequent retardation. in a distance of 100m.
ii. What would have been the magnitude ot the
common velocity of the particles after impact , 6. A bullet of mass 0.02kg travelling
if they had been moving at right – angles to horizontally at 200 hits a stationary block
each other instaed of the same straight line of wood of mass 5kg, passes through it and
emerges horizontally with a speed of 50 .
3. A ball of mass 50 g has a velocity of ( ) If the block is free to move on a smooth
m/s. It collides with a ball of mas 40 g moving horizontal plane, find the speed with which it is
with a velocity of ( ) moving after the bullet passes through it .
i. If the two balls moved with the same velocity
after the collision, find that velocity. Velocity-Time graph
ii. What are the velocities of the two balls after It is a graph plotted with time on x – axis and
the coliision if the 50 g ball is then moving velocity on y – axis.
parallel to the y – axis and then the 40 g ball is
moving parallel to the x – axis. In case of motion with uniform acceleration,
the velocity time graph consist of staright lines.
4. A ball of mass 40 g rolling at a speed of 9 This graph can be used to find uknowns like
collides with a ball of mass 30 g rolling distance travelled, acceleration, time and
in the same directionat 4 . As a results of velocity.

Baffour Ba Series, Further Mathematics for Schools Page 393


Note:
V (m/s)
1. Area under a velocity – time diagram gives
the total distance travelled or displacement.
2. The slope of a velocity – time diagram gives
12
the acceleration.
6

X Y Z
O 6 12 18
v2 21 t(s)

i. What is the mopeds acceleration in each of


v1 the time intervals OX, XY and YZ;
ii. What was the total distance between the two
time
t1 t2 sets of traffic lights.

From the velocity – time graph above, Solution


Acceleration = = i. Acceleration =
Acceleration in the interval OX ;
Worked Examples A= =2
1. A bus traveled along astaright road for 600m.
Acceleration in the interval XY ;
It travels at a constant velocity for the whole
journey, which takes 90 seconds. A= =0
i. Sketch the displacement time graph;
ii. What was the velocity of the bus? Acceleration in the interval XZ ;
A= =4
Solution A=-4 since the moped decelerates
S (m)
ii. Total distance travelled;
600
= Area of ∆ + Area of □ + Area of ∆
400 = bh + LB + bh
200 = (0.5 × 6 × 12) + (12 × 12) + (0.5 × 3 × 12)
t (s) = 36 + 144 + 18
3 6 9 = 198m
(s)
0 0 0
ii. Velocity = = 6.7m/s
Exercises 13.8
1. A toy car is placed on the floor of a sports
2. The figure below shows the velocity time
hall. It moves in a starght line starting from
graph for a moped, which travelled between
rest. It travels with constant acceleration for 4
two sets of traffic lights on a starght road.
seconds reaching a velocity of 5m/s. It then

Baffour Ba Series, Further Mathematics for Schools Page 394


slows down with constant deceleration of 1 ii. the distance covered by the sprinter in the
for 2 seconds. It then hits a wall and first 5 seconds of the race.
stops. V
a. Make a sketch of the velocity time graph for
the toy car.
b. Work out the total distance travelled by the
toy car.

2. Here is the velocity time graph of a car for


50 seconds. 20.32

4. Below is a sketch of the speed time graph of


V a cyclist moving on a starht road over a 7
second period.
30
V

5
50 T(s)
Work out the average acceleration during the
50 seconds. 2

3. A sprinter runs a race of 200 m. His total o 3 7 t(s)


time for running the race is 20.32seconds.
Below is the sketch of the speed – time graph Calculate:
for the motion of the sprinter. i. the acceleration for the first 3 seconds.
Calculate : ii. the distance covered by the cyclist over the
i. the maximum speed of the sprinter during the first 7 second period.
race.

Baffour Ba Series, Further Mathematics for Schools Page 395


14 THE CIRCLE Baffour Ba Series

Defining a Circle Equation of a Circle Centered at the Origin


A circle is a set of all points in a plane that lie a Consider the figure below;
fixed distance from a given point in the plane. y
The fixed distance is called the radius and the P(x, y)
given point is called the center. (y – 0)
O(0, 0)

(x – 0) Q
Equation of a Circle with Center (a, b)
Consider the diagram below;
y
P(x, y) x
If P(x, y) is a point on the circle, then its
O(a, b) (y – b)
 distance from the center is calculated by
(x – a)Q
Pythagoras theorem:
OP2 = OQ2 + QP2
But OP = r, OQ = x – 0 and QP = y – 0
x Substitute in /OP/2 = /OQ/2 + /QP/2
The distance formula or the Pythagoras /OP/2 = /OQ/2 + /QP/2
theorem can be used to find the equation of a r2 = (x – 0 )2 + (y – 0)2
circle with center (a, b) and radius r. r2 = x2 + y2

If P(x, y) is a point on the circle, then its This is the standard equation of a circle radius,
distance from the center is: r, with its center at the origin.
OP2 = OQ2 + QP2
But OP = r, OQ = x – a and QP = y – b Note:
Substitute in /OP/2 = /OQ/2 + /QP/2 1. That the graph of the equation r2 = (x – a )2
/OP/2 = /OQ/2 + /QP/2 + (y – b)2 with r > 0, is a circle with center (a,
r2 = (x – a )2 + (y – b)2 b) and radius r.
↓ 2. That the circle centered at the origin with
This is called the standard form for the radius, r , (r > 0) has the standard equation;
equation of a circle, with center (a, b) . r2 = x2 + y2

The equation of a circle can be found from the A. Finding the equation of a circle, given the
following: center and the radius
1. a given cenrtre and radius Steps
2. a given center and a point on the circle 1. Identify the given center as (a, b) and the
3. a given end points of the diameter radius as r.
4. any given three points on the circle. 2. Substitute (a, b) and r in;

Baffour Ba Series, Further Mathematics for Schools Page 396


r2 = (x – a )2 + (y – b)2 B. Use the given information to write a
3. Expand and simplify to obtain the equation standard equation of a circle.
of the circle in the general form:
x2 + y2 + 2gx + 2fy + c = 0 Center Diameter
1. (2, -6) 12
Worked Examples 2. (-3, 0) 24
1. Write the equation for the circle with the 3. (4, - 4) 15
given center and radius; 4. (-1, 2) 11
a. center = (0, 0), radius = 2
b. center = (-1, 2), radius = 4 C. 1. A circle has a radius of units and it is
centered at (-2.4, - 4.4). Write the equation of
Solution
the circle.
a. center = (0, 0), radius = 2
center = (a, b) = (0, 0) and r = 2
2. State the equation of a circle in standard
Substitute in r2 = (x – a )2 + (y – b)2
form which has a center at (14, 10) and a radius
22 = (x – 0 )2 + (y – 0)2
of 10.
4 = x2 + y2 standard equation
2 2
x +y –4=0
3. State the equation of a circle in standard
form which has a center at (-5, 4) and a radius
b. center = (-1, 2), radius = 4
of 9.
center = (a, b) = (-1, 2) and r = 2
Substitute in r2 = (x – a )2 + (y – b)2
4. State the equation of a circle in standard
42 = (x – (-1))2 + (y – 2)2
form which has a center at (-16, 9) and a radius
42 = (x + 1)2 + (y – 2)2 standard equation
2 2 of 13.
16 = x + 2x + 1 + y – 4y + 4
x2 + y 2 + 2x – 4y + 1 + 4 – 16 = 0
5. State the equation of a circle in standard
x2 + y 2 + 2x – 4y – 11 = 0 General equation
form which has a center at (-7, 5) and a radius
of 18.
Exercises 14.1
A. Use the given information to write a
B. Finding the equation of a circle, given the
standard equation of a circle.
center and a point on the circle
1. Identify the given center as C(a, b) and the
Center Radius
given point on the circle as P(a1, b1).
1. (2, 2) 5
2. Find the length of CP, which is the radius of
2. (-3, -1) 4
the circle, by the relation:
3. (0, - 4) 1
/CP/ = r = √( ) ( )
4. (-6, 2) 10

Baffour Ba Series, Further Mathematics for Schools Page 397


3. Substitute (a, b) and r in r2 = (x – a )2 + 3. Write the center – radius equation of a circle
(y – b)2 to obtain the standard equation of the with a center at (17, 8) and passes through the
circle. point (11, -12).

Worked Examples 4. Write the center – radius equation of a circle


1. The point (1, 2) is on a circle whose center is with a center at (9, 12) and passes through the
(5, -1). Write a standard equation of the circle. point (15, 12).

Solution The General Equation of a Circle


Center = C(5, -1) and P(1, 2) Consider the standard equation of a circle with
CP = radius (r) center (a, b)
r = √( ) ( ) (x − a) 2 + (y − b) 2 = r 2

r = √( Expanding the brackets gives


r = √(
C(5, x 2 − 2ax + a 2 + y 2 − 2by + b 2 = r 2
 1)
r=√ x2 − 2ax + y 2 − 2by + a 2 + b 2 − r 2 = 0 .

r = 5 units P(1, 2)
At this point, replace − a by g and − b by f.
2 2 2
Put (5, -1) and r in r = (x – a ) + (y – b) This gives:
⇒ 52 = (x – 5)2 + (y – (-1))2 x 2 + 2gx + y 2 + 2fy + g 2 + f 2 − r 2 = 0 .
52 = (x – 5)2 + (y + 1)2
(x – 5)2 + (y + 1)2 = 52 Now look at the last three terms on the left-
hand side, are g 2 + f 2 − r 2 . These do not
Exercises 14.2 involve x or y at all, so together they just
A. Write a standard equation of the circle represent a single number called c. Substituting
the respective centers and a point on the this into the equation finally gives x2 + 2gx +
circle. y2 + 2fy + c = 0 . This is the general equation of
Center Point on the Circle a circle.
1. (0, 0) (0, 3)
2. (1, 2) (4, 6) Properties of the General Equation of a
3. (3, 2) (5, 2) Circle: x2 + 2gx + y2 + 2fy + c = 0
4. (-5, 3) (1, 4) 1. It is quadratic in both x and y .
2. There is no term in xy.
B. 1.Write the center – radius equation of a 3. The coefficient of x2 is the same as the
circle with a center at (4, 11) and passes coefficient of y2. (In fact, if the equation has
through the point (5, 3). both coefficients equal to 1, it can always be
multiplied by a non-zero constant to obtain
2. Write the center – radius equation of a circle another valid equation.
with a center at (20, 10) and passes through the
point (9, -8). The centre of the circle is at (a, b) = (- g, - f)

Baffour Ba Series, Further Mathematics for Schools Page 398


and, since c = g 2 + f 2 − r 2, r 2 = g 2 + f 2 − c , 4. (x – )2 + (y – )2 =
so that the radius of the circle is given by :
5. x2 + y2 = 4
r = g 2 + f 2 − c.
Type 2: (The form; x2 + y2 + 2gx + 2fy + c = 0
Exercises 14.3
Method 1
A. Find the equation of the circle with given
If the equation of the circle is of the form:
centre and radius:
x2 + y2 + 2gx + 2fy + c = 0, then ;
1. centre (3, 5), radius 3;
1. Equate 2g to the coefficient of x and 2f to the
2. centre (−2, 3), radius 1;
coefficient of y, and solve for the pair (g, f).
3. centre (−1, −3), radius 2;
2. Obtain (-g, -f) as the center of the circle
4. centre (2, −2), radius 5;
3. Observe that the constant term;
5. centre (0, 5) radius 4.
c = (-g)2 + (-f)2 – r2. Hence, the radius
r2 = c – (-g)2 – (-f)2
Finding the Center and Radius, given the
4. Solve for r to obtain the radius of the given
equation
circle.
Type 1: (The form r2 = (x – a )2 + (y – b)2
5. When - g = a and - f = b, then;
If the equation of the circle is of the form;
r2 = c – a2 + b2
r2 = (x – a )2 + (y – b)2, then (a, b) is the center
and r is the radius. This is called the center –
Note;
radius form of the equation of a circle.
Ensure that the coefficients of x2 and y2 of the
given equation of the circle is 1.
Worked Examples
1. Give the center and radius of the circle,
Method 2
(x – 5)2 + (y – 1)2 = 25
Given the general format x2 + y2 + 2gx + 2fy + c
= 0, the center and radius of the circle is found
Solution
by completing the square for each variable and
(x – 5)2 + (y – 1)2 = 25
express in the center – radius form (refer to
Express in standard form
type 1).
(x – a )2 + (y – b)2 = r2
⇒(x – 5 )2 + (y – 1)2 = 52
Worked Examples
1. i. Express the equation r2 = (x – a )2 + (y –
Compared to the standard equation,
b)2 in the form x2 + y2 + 2gx + 2fy + c = 0
Center (a, b) = (5, 1) and radius r = 5
ii. Write down g, f, c in terms of a, b, r.
Exercises 14.4
A. Give the center and radius of the circle; Solution
1. (x – 5)2 + (y – 1)2 = 25 r2 = (x – a )2 + (y – b)2
2. (x + 2)2 + (y – 3)2 = 36 By expansion,
3. (x + 5)2 + (y + 3)2 = 1 x2 – 2ax + a2 + y2 – 2by + b2 = r2

Baffour Ba Series, Further Mathematics for Schools Page 399


x2 + y2 – 2ax – 2by + a2 + b2 – r2 = 0 13 = (4)2 + (1)2 – r2
13 = 16 + 1 – r2
Now let g = - a and f = - b, substitute in 13 – 16 – 1 = - r2
x2 + y2 – 2ax – 2by + a2 + b2 – r2 = 0…(1) - 4 = - r2
⇒ x2 + y2 + 2gx + 2fy + a2 + b2 – r2 = 0 r2 = 4
r2 = 22
Now, let a2 + b2 – r2 = c r=2
⇒ x2 + y2 + 2gx + 2fy + c = 0 ………...(2)
Method 2 : (Completing the square)
ii. Comparing eqn (1) and eqn (2), x2 + y2 – 8x + 2y + 13 = 0
-2ax = 2gx
-a=g Complete the square as follows;
a=-g x2 – 8x + y2 + 2y = - 13
x2 – 8x + . / + y2 + 2y + . / = -13 +
-2by = 2fy
-b=f . / +. /
b=-f x2 – 8x + ( ) + y2 + 2y + 12 = - 13 +
Therefore, the coordinates of the center (- 4)2 + 12
(a, b) = (- g, - f )

2 2 2
(x – 4)2 + (y + 1)2 = -13 + 16 + 1
a +b –r =c (x – 4)2 + (y + 1)2 = 4
but a = -g and b = -f (x – 4)2 + (y + 1)2 = 22
(-g)2 + (-f)2 – r2 = c Center = (a, b) = (4, -1)
Radius = 2
2. Determine the coordinates of the center and
radius of the circle x2 + y2 – 8x + 2y + 13 = 0 3. State the center and radius of the circle
x2 – 8x + y2 – 8y – 12 = 0
Solution
x2 + y2 – 8x + 2y + 13 = 0 Solution
2g = - 8 x2 – 8x + y2 – 8y – 12 = 0
g=-4 x2 + y2 – 8x – 8y – 12 = 0
2f = 2 2g = - 8
f=1 g=-
Center = (-g, -f) = (4, -1) g=-4

2f = - 8
From x2 + y2 – 8x + 2y + 13 = 0
c = 13 and (-g, -f) = (4, -1) f=
c = (-g)2 + (-f)2 – r2 f=-4

Baffour Ba Series, Further Mathematics for Schools Page 400


Center = (-g, -f) = (4, 4) - = - r2
r2 =
From x2 + y2 – 8x – 8y – 12 = 0,
c = - 12 and (-g , -f) = (4, 4) r= √ units
But c = (-g)2 + (-f)2 – r2
By substitution;
- 12 = (4)2 + (4)2 – r2 5. Determine the coordinates of the center and
radius of the circle 2x2 + 2y2 – 8x + 5y + 10 = 0
- 12 = 16 + 16 – r2
- 12 = 32 – r2
Solution
r2 = 32 + 12
2x2 + 2y2 – 8x + 5y + 10 = 0
r2 = 44
x2 + y2 – x + y + =0
r = √ units
x2 + y2 – 4x + y + 5 = 0
4. The equation of a circle is given by 2g = - 4
x2 – 7x + y2 + 16y + 43 = 0. Find the center and g = -2
radius of the circle.
2f =
Solution
x2 – 7x + y2 + 16y + 43 = 0 f=
x2 + y2 – 7x + 16y + 43 = 0 Center = (-g , -f) = . , /
2g = - 7
g= From x2 + y2 – 4x + y + 5 = 0
c = 5 and (-g , -f) = . , /
2f = 16
But c = (-g)2 + (-f)2 – r2
f=
By substitution;
f=8
5=( ) +. / – r2
Center = (-g, -f) = ( , -8)
5=4+ – r2
From x2 + y2 – 7x + 16y + 43 = 0 5–4– = – r2
c = 43 and (-g , -f) = . , 8/
- = - r2
2 2 2
But c = (-g) + (-f) – r
By substitution; r2 = =√ = units
43 = . / + (-8)2 – r2
Exercises 14.5
43 = + 64 – r2
1. A certain circle is represented by the
43 – – 64 = – r2 equation x2 + y2 + 16x – 14y + 49 = 0. Find the
center and radius of the circle .

Baffour Ba Series, Further Mathematics for Schools Page 401


2. State the coordinates of the center and radius
of the circle x2 – 8x + y2 – 12y – 20 = 0 P(x, y)

3. The equation of a circle is given by x2 – 6x +


A(x1,y1)  B(x2,y2)
y2 – 6y – 8 = 0. Find the center and radius of the
circle.

4. Find the coordinates of the center and radius Thus, ̅̅̅̅ is perpendicular to ̅̅̅̅. This implies
of the circle represented by x2 – 8x + y2 − 5y – that the product of the gradient of AP and BP is
13 = 0. – 1. That is; × = -1

5. Determine the co-ordinates of the centre and Simplify × = -1 to obtain the general
the value of the radius of the circle whose equation of the circle.
equation is x2 + y2 − 2x + 4y − 11 = 0.
Method 2
6. Determine the co-ordinates of the centre and Given the end points of the diameter as A(x1,
the value of the radius of the circle whose y1) and B (x2, y2),
equation is 36x2 + 36y2 − 36x − 24y − 131 = 0. 1. Find the mid-point of AB as O, where O is
the center of the circle. That is:
B. State the center and radius of each circle
O=. , /
1. x2 + 24x + y2 + 10y + 160 = 0
2. x2 + 26x + y2 + 28y + 364 = 0 2. Now use the fact that /OA/ = /OB/ = r, to
3. x2 – 6x + y2 – 32y = - 264 obtain the radius of the circle.
4. – 6x + x2 = 97 + 10y – y2 3. Using the coordinates of the center , O, and
5. x2 + 24x + y2 + 10y + 160 = 0 the radius, r, obtain the standard equation and
expand to obtain the general equation of the
C. Identify the centre and radius of each. circle.
1. x2 + y 2 − 2x − 4y − 20 = 0,
2. x2 + y 2 − 4x + 6y + 4 = 0, Method 3
3. x2 + y 2 + 2x − 3 = 0, 1. If A(x1, y1) and B (x2, y2) is the diameter of
4. x2 + y 2 + 6x + 7y – 14 = 0, the circle, then the equation of the circle is
2 2 (x – x1)(x – x2) + (y – y1)(y – y2) = 0.
5. 3x + 3y − 6x + 9y + 5 = 0.

2. Expand and simplify to obtain the general


The Equation of a Circle given the End Points
of the Diameter equation of the circle .
Method 1
Given the diameter of a circle as AB, and any Worked Examples
point say P(x, y) on the circle, AB subtends an 1. The points (8, 4) and (2, 2) are the ends of
angle of 900 at the point P. the diameter of a circle.

Baffour Ba Series, Further Mathematics for Schools Page 402


i. Find the coordinates of the center and the ⇒ 24 = 52 + 32 – r2
radius. 24 = 25 + 9 – r2
ii. Deduce the equation of the circle. r2 = 25 + 9 – 24
r2 = 10
Solution r=√
Method 1
A(8, 4) and B(2, 2) i. The coordinates of the center is (5, 3) and the
Let P(x, y) be a point of a circle. radius is √
P(x, y)
Gradient of AP; ii. The equation of the circle is;
= x2 + y2 – 10x – 6y + 24 = 0
A(8, 4)
 B(2, 2)
Gradient of BP; Method 2
Let A(8, 4) and B(2, 2) be the end points of the
=
diameter.
Center of AB = mid-point of AB
Since AP is perpendicular to BP;
Center of AB = . , /
⇒ × = -1
= (5, 3)
. /. / = -1
( )( )
( )(
= -1 ⇒ center, C = (a, b) = (-g, -f) = (5, 3)
)
(y – 4) (y − 2) = -1[ (x − 8) (x – 2)]
y (y – 2) – 4(y – 2) = - 1[x(x – 2) – 8(x – 2)] Let AC be the radius, r, of the circle;
y2 – 2y – 4y + 8 = -[x2 – 2x − 8x + 16] /AC/ = √( 8) ( )
y2 – 6y + 8 = - (x2 – 10x + 16) /AC/ = √( ) ( )
y2 – 6y + 8 = - x2 + 10x – 16 /AC/ = √
x2 + y2 – 10x – 6y + 8 + 16 = 0
/AC/ = √
x2 + y2 – 10x – 6y + 24 = 0
⇒r=√
Hence, the equation of the circle is
x2 + y2 – 10x – 6y + 24 = 0
Alternatively;
From x2 + y 2 – 10x – 6y + 24 = 0 (x – a )2 + (y – b)2 = r2
2gx = -10x Let x = 8 , y = 4, a = 5 and b = 4
g = -5 (8 – 5)2 + (4 – 3)2 = r2 P(x, y)
2fy = - 6y 32 + 12 = r2
y = -3 r2 = 10
r=√ A(8, 4)  B(2, 2)
Center = (a, b) = (-g, -f) = (5, 3) O (5, 4)
c = a2 + b2 – r2
But c = 24

Baffour Ba Series, Further Mathematics for Schools Page 403


Equation of a circle with center, (a, b) = (5, 3) ( )( )
( )(
= -1
)
and r = √
(y – 6) (y + 10) = -1 (x + 2) (x – 10)
(x – a )2 + (y – b)2 = r2
y(y + 10) – 6(y + 10) = - 1[x(x–10) + 2(x – 10)]
(x – 5)2 + (y – 3)2 = 10
y2 + 10y – 6y – 60 = -[x2 – 10x + 2x – 20]
x2 – 10x + 25 + y2 – 6y + 9 = 10
y2 – 4y – 60 = -(x2 – 8x – 20)
x2 + y2 – 10x – 6y + 25 + 9 – 10 = 0
y2 – 4y – 60 = -x2 + 8x + 20
x2 + y2 – 10x – 6y + 24 = 0
x2 + y2 – 8x – 4y – 60 – 20 = 0
x2 + y2 – 8x – 4y – 80 = 0
Method 3
If A(x1, y1) and B (x2, y2) is the diameter of the
Method 2
circle, then the equation of the circle is
If A(x1, y1) and B (x2, y2) is the diameter of the
(x – x1)(x – x2) + (y – y1)(y – y2) = 0.
circle, then the equation of the circle is
But the end points are A(8, 4) and B(2, 2) (x – x1)(x – x2) + (y – y1)(y – y2) = 0.
Put in (x – x1)(x – x2) + (y – y1)(y – y2) = 0.
But the end points are A(-2, 6) and B(-10, 10)
⇒ (x – 8)(x – 2) + (y – 4)(y – 2) = 0.
Put in (x – x1)(x – x2) + (y – y1)(y – y2) = 0.
x2 – 2x – 8x + 16 + y2 – 2y – 4y + 8 = 0
⇒ (x + 2)(x –10) + (y – 6)(y + 10) = 0.
x2 – 10x + 16 + y2 – 6y + 8 = 0
x2 – 10x + 2x – 20 + y2 + 10y – 6y – 60 = 0
x2 + y2 – 10x – 6y + 16 + 8 = 0
x2 – 8x – 20 + y2 + 4y – 60 = 0
x2 + y2 – 10x – 6y + 24 = 0
x2 + y2 – 8x + 4y – 20 – 60 = 0
2. AB is the diameter of a circle. Find the x2 + y2 – 8x + 4y – 80 = 0
equation of the circle through the points A(-2,
6) and B(10, -10) 2. If A(-8, 1) and B(2, 1) is the diameter of a
circle, find the equation of the circle.
Solution
A(-2, 6) and B(10, -10) Solution
Let P(x, y) be a point of a circle. A(-8, 1) and B(2, 1)
Let P(x, y) be a point of a circle.
P(x, y)
Gradient of AP; P(x, y)
=
A(-2, 6)
 B(10, -10) A(-8, 1)
 B(2, 1)
Gradient of BP;
= Gradient of AP;
= =
( )
Since AP is perpendicular to BP;
⇒ × = -1 Gradient of BP;
. /. / = -1 =

Baffour Ba Series, Further Mathematics for Schools Page 404


Since AP is perpendicular to BP; B. 1. A is the point (- 4, 2) and B is the point (6,
⇒ × = -1 - 4). Find the equation of the circle which has
. /. / = -1 AB as diameter.
( )( )
( )(
= -1 2. P is the point (- 5, 3) and Q is the point (5, -
)
(y – 1) (y – 1) = -1[ (x + 8) (x – 2)] 21). Find the equation of the circle which has
y(y – 1) – 1(y – 1) = - 1[x(x – 2) + 8(x – 2)] PQ as diameter.
y2 – y – y + 1 = -[x2 – 2x + 8x – 16]
y2 – 2y + 1 = -(x2 + 6x – 16) 3. Write the equation of a circle that has end
y2 – 2y + 1 = -x2 – 6x + 16 points of its diameter (-17, -9) and (-19, -9)
x2 + y2 + 6x – 2y + 1 – 16 = 0
x2 + y2 + 6x – 2y – 15 = 0 Challenge Problem
Write the equation of a circle whose center is
3. Write the equation of a circle whose center is (-1, 3) and whose circumference is 12
(-10, 3) and whose circumference is 8
Equation of a Circle through 3 given Points
Solution Method 1
Center = (-10, 3) = (a, b) 1. Identify the three given points (x, y)
C= d=8 2. Substitute each point (x, y) in x2 + y2 + 2gx +
2fy + c = 0 to obtain three equations.
Diameter (d) = 2r = 8 3. Solve the three equations simultaneously to
radius r = = 4 obtain the values of g, f and c. Note that c is
obtained by substituting (g, f) in either eqn (1)
r2 = (x – a)2 + (y – b)2 or eqn (2) or eqn (3).
42 = (x + 10)2 + (y – 3)2 4. Substitute the values of g, f and c in x2 + y2 +
16 = x2 + 20x + 100 + y2 – 6y + 9 2gx + 2fy + c = 0 to obtain the equation of the
x2 + y2 + 20x – 6y + 100 + 9 – 16 = 0 circle.
x2 + y2 + 20x – 6y + 93 = 0
Method 2
Exercises 14.6 1. Using the center – radius formula;
A. ̅̅̅̅ is the diameter of a circle. Find the (x – a )2 + (y – b)2 = r2, substitute each given
equation of the circle through the following point (x, y) and simplify to obtain three
points. equations.
1. A(-5, 3) and B(13, -7) 2. Take note of the fact that the center of the
2. A(3, 8) and B(1, 4) circle to any of the given point on the circle
3. A(8, -2) and B(2, 8) forms the radius of the circle. Therefore, equate
4. A(- 4, 3) and B(16, -7) equations (1) and (2), (2) and (3) to obtain
5. A(-7, 7) and B(9, -5) equation (4) and equation (5) respectively.

Baffour Ba Series, Further Mathematics for Schools Page 405


3. Solve equations (4) and (5) to obtain the Method 4
center of the circle (a, b). Suppose the points A(x1, y1), B(x2, y2) and C(x1,
4. Substitute (a, b) in either eqn (1) or eqn (2) y1) lie on the same circle as shown below.
to obtain the length of the radius (r) of the
circle. L2
5. Substitute (a, b) and r in (x – a )2 + (y – b)2 = L1
r2 and simplify to obtain the equation of the
circle in the form x2 + y2 + 2gx + 2fy + c = 0 A(x1, y1)
C (x3, y3)
Method 3 B(x2, y2)
Suppose the points A(x1, y1), B(x2, y2) and C(x1, Then;
y1) lie on the same circle. 1. Find the equation of the perpendicular
1. Find a point P(x, y) on the circle which lies bisector of AB and that of BC and name them
between the points B and C. L1 and L2 respectively. Note that the
2. Draw a line from B to P and another line perpendicular bisectors of AB and BC pass
from C to P and name them L1 and L2 through the mid-points of AB and BC
respectively as shown below. respectively. Thus, the midpoints must be
A calculated first.
2. Find the point of intersection of L1 and L2 to
obtain the coordinates of the center O (x, y) =
(a, b) of the circle.

3. Find the distance OA = OB = OC to obtain
B
B  C the radius of the circle.
4. With the center and radius of the circle,
P(x, y)
L2 L1 obtain the standard or general equation of the
3. A Line from A to P passes through the center circle.
of the circle, thus AP is the diameter, such that
A and P are the endpoints of the diameter not Worked Examples
forgetting that AP subtends an angle of 900 at 1. A circle passes through the points A(-5, 2),
< ABP and < ACP. B(-3, - 4) and C(1, 8). What is the equation of
4. Find the equations of L1 and L2 using the fact the circle?
that AB is perpendicular to P(x, y) is the point
of intersection of L1 and L2 obtained by solving Solution
equations of L1 and L2 simultaneously. A(-5, 2), B(-3, - 4) and C(1, 8)
5. With A(x1, y1) and P(x, y) = (x2, y2), obtain Substitute each point (x, y) in x2 + y2 + 2gx +
the general equation of the circle by 2fy + c = 0
substituting and simplifying;
(x – x1)(x – x2) + (y – y1)(y – y2) = 0. When (x, y) = (-5, 2)
(-5)2 + 22 + 2g(-5) + 2f(2) + c = 0

Baffour Ba Series, Further Mathematics for Schools Page 406


25 + 4 – 10g + 4f + c = 0 Now, put g = -2, f = -1 and c = - 45 in
29 – 10g + 4f + c = 0 ……….(1) x2 + y2 + 2gx + 2fy + c = 0
x2 + y2 + 2 (-2)x + 2 (-1)y + (- 45) = 0
When (x, y) = (-3, - 4) x2 + y2 – 4x – 2y – 45 = 0
(-3)2 + (- 4)2 + 2g(-3) + 2f(- 4) + c = 0
9 + 16 – 6g – 8f + c = 0 Method 2
25 – 6g – 8f + c = 0 ………...(2) A(-5, 2), B(-3, - 4) and C(1, 8)
Using the center – radius formula;
When (x, y) = (1, 8) (x – a )2 + (y – b)2 = r2
12 + 82 + 2g(1) + 2f(8) + c = 0
1 + 64 + 2g + 16f + c = 0 When (x, y) = (-5, 2);
65 + 2g + 16f + c = 0 ……….(3) (-5 – a )2 + (2 – b)2 = r2
25 + 10a + a2 + 4 – 4b + b2 = r2
eqn (2) – eqn (1); a2 + b2 + 10a – 4b + 29 = r2………..(1)
- 4 + 4g – 12f = 0……………(4)
When (x, y) = (-3, - 4);
eqn (3) – eqn (2); (-3 – a)2 + (- 4 – b)2 = r2
40 + 8g + 24f = 0……………(5) 9 + 6a + a2 + 16 + 8b + b2 = r2
a2 + b2 + 6a + 8b + 25 = r2 …………(2)
Reduce eqn (4) and eqn (5);
-1 + g – 3f = 0 ……………..(4) When (x, y) = (1, 8);
5 + g + 3f = 0 ……………….(5) (1 – a )2 + (8 – b)2 = r2
1 – 2a + a2 + 64 – 16b + b2 = r2
eqn (4) + eqn (5); a2 + b2 – 2a – 16b + 65 = r2 …………(3)
4 + 2g = 0
2g = -4 Since the radius of the circle is the same;
g = -2 equate eqn (1) and eqn (2);
a2 + b2 + 10a – 4b + 29 = a2 + b2 + 6a + 8b + 25
Put g = -2 in eqn (4); 10a – 4b + 29 = 6a + 8b + 25
-1 – 2 – 3f = 0 0 = 10a - 6a – 4b – 8b + 29 – 25
-3 – 3f = 0
10a – 6a – 4b – 8b + 29 – 25 = 0
-3f = 3
4a – 12b + 4 = 0…………(4)
f = -1

Equate eqn (2) and (3);


Put g = -2 and f = -1 in eqn (3);
a2 + b2 + 6a + 8b + 25 = a2 + b2 – 2a –16b + 65
65 + 2(-2) + 16(-1) + c = 0
6a + 8b + 25 = - 2a – 16b + 65
65 – 4 – 16 + c = 0
6a + 2a + 8b + 16b + 25 – 65 = 0
45 + c = 0
8a + 24b – 40 = 0……….(5)
c = - 45

Baffour Ba Series, Further Mathematics for Schools Page 407


Reducing eqn (4) and eqn (5); AP is the diameter of the circle.
a – 3b + 1 = 0…………(4) L1 is perpendicular to AB
a + 3b – 5 = 0………..(5) ⇒ =. /= = -3

eqn (4) + eqn (5);


2a – 4 = 0 Gradient of L1 = and passes through B(-3, -4)
2a = 4
a=2 Equation of L1;
y + 4 = (x + 3)
Put a = 2 in eqn (4);
3(y + 4) = x + 3
2 – 3b + 1 = 0
3y + 12 = x + 3
3 – 3b = 0
x – 3y + 3 – 12 = 0
3b = 3
x – 3y – 9 = 0
b=1
The coordinates of the center (a, b) = (2, 1) L2 is perpendicular to AC

Substitute (a, b) = (2, 1) in eqn (1); ⇒ =. /= =1


a2 + b2 + 10a – 4b + 29 = r2………..(1)
22 + 12 + 10(2) – 4(1) + 29 = r2 Gradient of L2 = 1 and passes through C(1, 8)
4 + 1 + 20 – 4 + 29 = r2
50 = r2 Equation of L2;
y - 8 = - 1(x – 1)
From any point P(x, y) on the circle, to the y – 8 = -x + 1
y+x–8–1=0
center (a, b) = (2, 1) and r = √
The equation of the circle is:
P(x, y) is the point of intersection of L1 and L2
(x – 2)2 + (y – 1)2 = √ x – 3y – 9 = 0 ……………(1)
x2 – 4x + 4 + y2 – 2y + 1 = 50 x + y – 9 = 0 …………… .(2)
x2+ y2 – 4x – 2y + 4 + 1 – 50 = 0
x2 + y2 – 4x – 2y – 45 = 0 eqn (2) – eqn (1);
- 4y = 0
Method 3 A(-5, 2) y=0

A(-5, 2),
B(-3, - 4) Put y = 0 in equation (1)
C(1, 8) x=9

⇒P(x, y) = (9, 0)
B(-3, -4)
C(1, 8)
A(-5, 2) and P(9, 0) are the ends of the

L2 P(x, y) diameter;
L1

Baffour Ba Series, Further Mathematics for Schools Page 408


⇒ Equation of the circle; perpendicular bisector of BC.
(x – x1) (x – x2) + (y – y1) (y – y2) = 0
(x + 5) (x – 9) + (y – 2) (y – 0) = 0 Mid-point of BC;
x2 – 9x + 5x – 45 + y2 – 0y – 2y – 0 = 0 =. , / = (-1, 2)
x2 – 4x – 2y + y2 – 45 = 0
x2 + y2 – 4x – 2y – 45 = 0
Gradient of BC;

Method 4 = = =3
A(-5, 2), B(-3, - 4) and C(1, 8) Gradient of L2 =

L2
Equation of L2 passing through (-1, 2);
L1
y–2= (x + 1)
A(-5, 2) 3(y – 2) = - (x + 1)
C(1, 8) 3y – 6 = - x – 1
B(-3, -4) x + 3y – 6 + 1 = 0
x + 3y – 5 = 0
L1 is perpendicular to AB and Passes through
x + 3y = 5 …………………(2)
the mid-point of AB. L1 is therefore, the
perpendicular bisector of AB.
Point of intersection of L1 and L2 is the center
O(x, y)
Mid-point of AB;
=. , / = (- 4, -1) Solving equations (1) and (2);
x – 3y = -1 ……………(1)
Gradient of AB; x + 3y = 5..……………(2)
= = = -3
eqn (2) – eqn (1);
Gradient of L1 = 6y = y
y=1
Equation of L1 passing through (- 4, -1);
y + 1 = (x + 4) Put y = 1in equation (1);
x – 3 = -1
3(y + 1) = x + 4
x = -1 + 3
3y + 3 = x + 4
x=2
x – 3y + 4 – 3 = 0
Center O(x, y) = (a, b) = (2, 1)
x – 3y + 1 = 0
x – 3y = - 1 …………………(1)
But radius r = /OA/ = /OB/ = /OC/
If r = /OA/;
L2 is perpendicular to BC and Passes through
the mid-point of BC. L2 is therefore, the r = √( ) ( )

Baffour Ba Series, Further Mathematics for Schools Page 409


r = √( ) ( ) 122 + (-13)2 + 2g(12) + 2f(-13) + c = 0
r=√ 144 + 169 + 24g – 26f + c = 0
313 + 24g – 26f + c = 0 ……….(3)
Now, with center (2, 1) and radius √ ;
Now, from eqn (2),
Equation of the circle;
c = 2g – 1
(x – 2)2 + (y – 1)2 = (√ )
x2 – 4x + 4 + y2 – 2y + 1 = 50 Put c = 2g – 1 in eqn (1)
x2 + y2 – 4x – 2y + 5 – 50 = 0 13 + 4g + 6g + (2g – 1) = 0
x2 + y2 – 4x – 2y – 45 = 0 13 + 4g + 6f + 2g – 1 = 0
12 + 6g + 6f = 0……….(4)
Note
When one of the given points contains a zero Put c = 2g – 1 in eqn (3)
coordinate, make c the subject of the equation 313 + 24g – 26g + (2g – 1) = 0
formed from that point and substitute in the 313 + 24g – 26f + 2g – 1 = 0
other two equations to generate equations (4) 312 + 26g – 26f = 0……….(5)
and (5)
Reduce eqn (4) and eqn (5);
Worked Examples 2 + g + f = 0……….(4)
1. Find the equation of the circle which passes 12 + g – f = 0……….(5)
through the points P(2, 3), Q(-1, 0) and R(12,
-13) eqn (5) – eqn (4);
10 – 2f = 0
Solution -2f = - 10
P(2, 3), Q(-1, 0) and R(12, -13) f=5
Substitute each point (x, y) in x2 + y2 + 2gx +
2fy + c = 0 Put f = 5 in eqn (5);
2+g+5=0
When (x, y) = (2, 3) 7+g=0
22 + 32 + 2g(2) + 2f(3) + c = 0 g = -7
4 + 9 + 4g + 6f + c = 0
13 + 4g + 6f + c = 0 ……….(1) Put f = 5, g = -7 in eqn (1);
13 + 4(-7) + 6 (5) + c = 0
When (x, y) = (-1, 0) 13 – 28 + 30 + c = 0
(-1)2 + 02 + 2g(-1) + 2f(0) + c = 0 15 + c = 0
1 + 0 – 2g + c = 0 c = - 15
1 – 2g + c = 0 ……….(2)
Now, substitute g = -7, f = 5 and c = -15 in
When (x, y) = (12, -13) x2 + y2 + 2gx + 2fy + c = 0

Baffour Ba Series, Further Mathematics for Schools Page 410


x2 + y2 + 2(-7)x + 2(5)y + (-15) = 0 a – b – 12 = 0………..(5)
x2 + y2 – 14x + 10y – 15 = 0
The equation is x2 + y2 – 14x + 10y – 15 = 0 eqn (4) + eqn (5);
2a – 14 = 0
Method 2 2a = 14
Using the center – radius formula; a=7
(x – a )2 + (x – b)2 = r2
Put a = 7 in eqn (4)
When (x, y) = (2, 3); 7+b–2=0
(2 – a )2 + (3 – b)2 = r2 5+b=0
4 – 4a + a2 + 9 – 6b + b2 = r2 b = -5
a2 + b2 – 4a – 6b +13 = r2………..(1) The coordinates of the center of the circle;
(a, b) = (7, -5)
When (x, y) = (-1, 0);
(-1 – a)2 + (0 – b)2 = r2 Substitute (a, b) = (7, -5) in eqn (1);
1 + 2a + a2 + b2 = r2 a2 + b2 – 4a – 6b +13 = r2………..(1)
a2 + b2 + 2a +1 = r2 …………(2) 72 + (-5)2 – 4(7) – 6(-5) +13 = r2
49 + 25 – 28 + 30 + 13 = r2
When (x, y) = (12, -13); 89 = r2
(12 – a )2 + (-13 – b)2 = r2
144 – 24a + a2 + 169 + 26b + b2 = r2 From any point P(x, y) on the circle, to the
a2 + b2 – 24a + 26b +313 = r2 …………(3) center (a, b) = (7, -5) and r = √8
The equation of the circle is:
Since the radius of the circle is the same;
(x – 7)2 + (y – (-5))2 = √8
equate eqn (1) and eqn (2);
a2 + b2 – 4a – 6b +13 = a2 + b2 + 2a +1 (x – 7)2 + (y +5)2 = √8
- 4a – 6b + 13 = 2a + 1 x2 – 14x + 49 + y2 + 10y + 25 = 89
0 = 2a + 4a + 6b + 1 – 13 x2+ y2 – 14x + 10y + 49 + 25 – 89 = 0
6a + 6b – 13 + 1 = 0 x2 + y2 – 14x + 10y – 15 = 0
6a + 6b – 12 = 0…………(4)
Exercises 14.7
equate eqn (2) and (3); A. Find the equation of a circle which
a2 + b2 + 2a +1 = a2 + b2 – 24a + 26b +313 contains:
2a + 1 = - 24a + 26b + 313 1. (2, 2), (6, 4) and (4, 8)
2a + 24a – 26b + 1 – 313 = 0 2. (-3, -4) (-5, 2) and (1, 8)
26a – 26b – 312 = 0……….(5) 3. (0, 0) (4, 0) and (6, -2)
4. (4, 1) (-2, 1) and (2, 3)
Reducing eqn (4) and eqn (5); 5. (-2, -1) (0, -5) and (1, -2)
a + b – 2 = 0…………(4) 6. (3, 1) (8, 2) and (2, 6)

Baffour Ba Series, Further Mathematics for Schools Page 411


Area of a Circle 3. x 2 + y 2 + 2x − 3 = 0,
The area of a given circle is calculated by the 4. x 2 + y 2 + 6x + 7y – 14 = 0,
formula A = r2. Therefore, from the equation
of a given circle, the radius can be obtained and Tangent to a Circle
substituted in A = r2 A tangent to a circle is a straight line which
touches the circumference of a circle at one
Worked Examples point only.
1. What is the area of the circle x2 + y2 = 40?
A tangent is always perpendicular to the radius
Solution at the point of tangency (point of contact)
x2 + y2 = 40
centre = (0, 0) and r2 = √ B

A = r2 O
A= √
C
A=√
A

2. What is the area of the circle 9x2 + 9y2 = 25? In the diagram above, ̅̅̅̅ is the tangent to the
circle center O, ̅̅̅̅ is the radius of the circle
Solution and C is the point of tangency. ̅̅̅̅ is
9x2 + 9y2 = 25 perpendicular to ̅̅̅̅, meaning they meet an
x2 + y2 = angle of 900.

Verifying that a Point Lies on a Circle


centre = (0, 0) and r2 = Given a circle of the form x2 + y2 + 2gx + 2fy +
A = r2 c = 0, the point (x, y) lies on the circle if the
A= values of (x, y) satisfy the equation of the circle.

A= Steps:
1. Identify the equation of the circle and the
Exercises 14.8 given point (x, y) assumed to be on the circle.
Find the area of the following circles, leaving 2. Substitute the values of (x, y) into the
your answer in terms of equation of the circle.
1. x2 + y2 = 16? 2. x2 + y2 = 100 3. If the L.H.S. of the equation is equal to the
3. x2 + y2 = 13 4. 4x2 + 4y2 = 9 R.H.S. of the equation, then the point lies on
5. 16x2 + 16y2 = 1 the circle, if not the point does not lie on the
circle.
B. What is the area of the following circles?
1. x2 + y 2 − 2x − 4y − 20 = 0, Worked Examples
2. x 2 + y 2 − 4x + 6y + 4 = 0, 1. Verify that the point (3, 2) lies on the circle

Baffour Ba Series, Further Mathematics for Schools Page 412


x2 + y2 – 8x + 2y + 7 = 0 product of the gradient of the radius and that of
the tangent is – 1.
Solution
x2 + y2 – 8x + 2y + 7 = 0 Steps:
L.H.S. = ? R.H.S. = 0 1. Obtain the center of the circle as C(x1, y1)
Put (x, y) = (3, 2) in x2 + y2 – 8x + 2y + 7 = 0 2. Identify the point of tangency as P(x2, y2)
32 + 22 – 8(3) + 2(2) + 7 = 0 Find the gradient of the radius CP, which
9 + 4 – 24 + 4 + 7 = 0 passes through (x1, y1) and (x2, y2), as;
L.H.S. = R.H.S. m=
Therefore, the point (3, 2) lies on the circle.
3. The gradient of the tangent is the negative
2. i. Find the coordinates of the center c, of the
reciprocal of the gradient of the radius.
circle whose equation is x2 – 4x + y2 – 6y = 4
ii. Show that the point T(6, 4) lies on the circle.
⇒gradient of tangent = –
Solution
i. x2 – 4x + y2 – 6y = 4 Worked examples
x2 + y2 – 4x – 6y – 4 = 0 1. i. Find the coordinates of the center of the
Compared to x2 + y2 + 2gx + 2fy + c = 0 circle whose equation is x2 – 4x + y2 – 6y = 4.
2gx = - 4x ii. Show that the point T(6, 4) lies on the circle.
g = -2 iii. Find the gradient of CT and hence deduce
the gradient of the tangent at T.
2fy = - 6y
f = -3 Solution
Center = (a, b) = (-g, -f) = (2, 3) i. x2 – 4x + y2 – 6y = 4
x2 + y2 – 4x – 6y – 4 = 0
ii. x2 – 4x + y2 – 6y = 4 Compared to x2 + y2 + 2gx + 2fy + c = 0
LHS = ?, R. H. S = 4 2gx = - 4x
g = -2
Substitute T(x, y) = T(6, 4)
62 – 4(6) + 42 – 6(4) = 4 2fy = - 6y
L.H.S. = R. H. S f = -3
Center = (a, b) = (- g, - f) = (2, 3)
Finding the Gradient of the Tangent
The gradient of the tangent can be obtained ii. x2 – 4x + y2 – 6y = 4
once the gradient of the radius is known. This is L.H.S. = ?, R. H. S = 4
because, the radius is perpendicular to the Substitute T(x, y) = T(6, 4)
tangent at the point of tangency. As a result, 62 – 4(6) + 42 – 6(4) = 4

Baffour Ba Series, Further Mathematics for Schools Page 413


L.H.S. = R. H. S Worked Examples
Therefore the point T(6, 4) lies on the circle. 1. i. Find the coordinates of the center c, of the
circle whose equation is x2 – 4x + y2 – 6y = 4
iii. Diagram ii. Show that the point T(6, 4) lies on the circle.
iii. Find the gradient of CT and hence deduce
C(2, 3)
the gradient of the tangent at T.
iv. Find the equation of the tangent to the circle
T(6, 4) at T.
C(2, 3) and T(6, 4)
⇒ x1 = 2, y1 = 3, x2 = 6, y2 = 4 Solution
= = = i. x2 – 4x + y2 – 6y = 4
x2 + y2 – 4x – 6y – 4 = 0
Hence, the gradient of the tangent at T = - 4
Compared to x2 + y2 + 2gx + 2fy + c = 0
2gx = - 4x
Equation of a Tangent
g = -2
The equation of the tangent is stated in the
general form: ax + by + c = 0, because the
2fy = - 6y
tangent is a straight line.
f = -3
Center = (a, b) = (-g, -f) = (2, 3)
Steps:
1. Identify the equation of the circle in the
ii. x2 – 4x + y2 – 6y = 4
form: x2 + y2 + 2gx + 2fy + c = 0, the center of
LHS = ?, R. H. S = 4
the circle (-g, -f) = (x1, y1) and a point on the
Substitute T(x, y) = T(6, 4)
circle as (x2, y2).
62 – 4(6) + 42 – 6(4) = 4
2. Find the gradient of the radius through the
L.H.S. = R. H. S
points (x1, y1) and (x2, y2) using = Therefore the point T(6, 4) lies on the circle.
3. Obtain the gradient of the tangent as:
=– iii. Diagram
3. Substitute the coordinates of the point of C(2, 3)

tangency (x2, y2), and the gradient of the


C(2, 3) and T(6, 4) T(6, 4)
tangent, m, into the formula y – y2 = m(x – x2).
⇒ x1 = 2, y1 = 3, x2 = 6, y2 = 4
4. Simplify to obtain the equation of the tangent
in the form ax + by + c = 0 = = =
Hence, the gradient of the tangent at T = - 4
Note:
Do not substitute the coordinates of the center iv. Put the tangential point x2 = 6, y2 = 4, and
in y – y2 = m(x – x2), because the tangent does the gradient of the tangent, m = - 4 in
not pass through the center. y – y2 = m (x – x2).

Baffour Ba Series, Further Mathematics for Schools Page 414


y – 4 = - 4 (x – 6). Solution
y – 4 = - 4x + 24 x2 + y2 + 4x + 10y - 71 = 0
y + 4x – 4 – 24 = 0
4x + y – 28 = 0 2gx = 4x
The equation of the tangent to the circle at the g=2
C(-2, -5)
point (6, 4) is 4x + y – 28 = 0
2fy = 10y P(4, 3)
2. Show that the point (3, 2) lie on the circle f=5
x2 + y2 – 8x + 2y + 7 = 0, and find the equation Center = (a, b) = (- g, - f) = (-2, -5)
of the tangent at this point.
Gradient of radius CP;
Solution C(-2, -5) and P(4, 3)
x2 + y2 – 8x + 2y + 7 = 0 ⇒ x1 = -2, y1 = -5, x2 = 4, y2 = 3
Compared to x2 + y2 + 2gx + 2fy + c = 0 = =
( )
= =
( )
2gx = - 8x
Hence, the gradient of the tangent at P,
g=-4
C(4, -1) m=–
2fy = 2y Now, substitute x2 = 4, y2 = 3 and m = –
f=1 P(3, 2)
y – y2 = m (x – x2)
Center = (a, b) = (-g, -f) = (4, -1)
y–3= (x – 4)
C(4, -1) and P(3, 2) 4(y – 3) = - 3(x – 4)
⇒ x1 = 4, y1 = 1, x2 = 3, y2 = 2 4y – 12 = - 3x + 12
( ) 3x + 4y – 12 – 12 = 0
= = = -3
3x + 4y – 24 = 0
Hence, the gradient of the tangent at P, m = The equation of the tangent is 3x + 4y – 24 = 0
Now, substitute x2 = 3, y2 = 2 and m =
4. Find the equation of the tangent to the circle
y – y2 = m (x – x2)
x2 + y2 – 6x + 4y + 5 = 0 at the points where it
y – 2 = (x – 3) meets the x – axis.
3(y – 2) = (x – 3)
3y – 6 = x – 3 Solution y
x – 3y – 3 + 6 = 0 x2 + y2 – 6x + 4y + 5 = 0
x – 3y + 3 = 0 2g = - 6
Therefore, the equation of the tangent to the g=-3

circle at (3, 2) is x – 3y + 3 = 0
2f = 4
x
3. Find the equation of the tangent to the circle f=2
x2 + y2 + 4x + 10y – 71 = 0 at the point P(4, 3).

Baffour Ba Series, Further Mathematics for Schools Page 415


Center = (a, b) = (-g, -f) 4. Find the equation of the tangent to the circle
= (3, -2) x2 + y2 − 2x − 2y − 23 = 0 at the point (5, 4).

At the x – axis; 5. Find the equation of the tangent to the circle


(3, 0) x2 + y2 − 2x + 5y = 0 at the point (2, 0).

Gradient of radius ; 6. Find the points of intersection of the line


m=
( )
= y = 2x + 1 and the circle x 2 + y 2 − 2y + 4 = 0.
Show that the line y = 2x + 1 is a diameter of
the circle. Find the equation of the tangent to
Gradient of tangent;
the circle at one of the points of intersection.
m= =0
7. Find the points of intersection of the line
Substitute x2 = x, y2 = 0 and m = 0 in y = x − 3 and the circle x 2 + y 2 − 2x + 2y + 1 =
y – y2 = m (x – x2) 0. What are the tangents at the points of
y – 0 = 0 (x – x2) intersection? Where do they intersect?
y=0
8. Find the points where the circle x2 + y2 − 10x
Exercises 14.9 − 10y + 40 = 0 and the line y + 2x = 10
A. Verify that the given points lie on the intersect. Find the equation of the tangent to the
respective circles and find the equation of circle at each of the points of intersection. Find
the tangent to the circles at the given points. the point of intersection of these two tangents.
1. x2 + y2 + 2x – 2y – 8 = 0 (2, 2)
2 2
2. x + y + 2x + 4y – 12 = 0 (3, -1) Challenge Problems
2 2
3. x + y – 8x – 2y = 0 (3, 5) 1. Find the equation of a circle with center
2 2
4. x + y + 6x – 2y = 0 (0, 0) (- 4, 7) which has the x – axis as a tangent
2 2
5. 2x + 2y – 8x – 5y – 1 = 0 (1, -1)
2 2
6. x + y − 6x − 8y − 49 = 0 P(-2, -3) 2. Write down the equation of a circle that has
x = - 4 , x = 8, y = - 2 and y = 10 as tangents.
B. 1. A circle has the equation x2 + y2 − 4x − 8y
− 5 = 0. Write down the equation of the tangent 3. Show that the points (7, 12), (-3, -12), and
to this circle at the point A(-3, 4). (14, -5) lie on a circle with a centre (2, 0)?

2. A circle has the equation x2 + y2 + 6x + 4 = 0. 4. The circle x2 + y2 + 2gx + 2fy + c = 0 passes


Find the equation of the tangent to this circle at through the points A(-1, -2) , B (1, 2) and C (2,
the point P(-5, -1). 3). Write down three equations which must be
satisfied by g, f, c. Solve these equations and
3. Find the equation of the tangent to the circle write the equation of the circle ABC.
x2 + y2 − 8x + 2y − 3 = 0 at the point A(2, 3).

Baffour Ba Series, Further Mathematics for Schools Page 416


Length of a Tangent from an External Point Centre = (a, b) = (-g, -f) = (2, 4)
to a Circle But c = -5
Steps c = a2 + b2 – r2
1. From the given equation of the circle, -5 = (2)2 + (4)2 – r2
identify the center of the circle (a, b) and the - 5 = 4 + 16 – r2 (8, 2)

radius, r, of the circle. r2 = 4 + 16 + 5 d


2. From the given external point (x, y), draw a 2
r = 25 (2, 4)
l
tangent to the circle and from the point to the r=√ 5
center of the circle as shown below; r=5
(x, y)
d By Pythagoras theorem;
(a, b)
l d2 = l2 + r2
r l2 = d2 – r2

By Pythagoras theorem; Now, r = 5, a = 2, b = 4, x = 8, y = 2


d2 = l2 + r2 But d2 = ,( ) ( ) -
l2 = d2 – r2 2
Put in l = ,( ) ( ) - – r2
l2 = ,(8 ) ( ) - – 52
But d2 = (x – a)2 + (y – b)2
l2 = ,( ) ( ) - – 52
l2 = ,( ) ( ) - - r2
l2 =36 + 4 – 25
Substitute the values of (x, y), (a, b) and r to
l2 = 40 – 25
obtain l, the length of the tangent.
l2 = 15
By simple algebraic manipulation, the l=√
length, L, of the tangent is calculated by : l = 3.87 units
L=√
2. Find the length of the tangents to the circle
x2 + y2 – 8x + 10y + 5 = 0 from the point (5, 7).
Worked Examples
1. Find the length of the tangents from the point
Solution
(8, 2) to the circle x2 + y2 – 4x – 8y – 5 = 0
x2 + y2 – 8x + 10y + 5 = 0
Compared to x2 + y2 + 2gx + 2fy + c = 0
Solution
2gx = - 8x
x2 + y2 – 4x – 8y – 5 = 0
g=-4
Compared to x2 + y2 + 2gx + 2fy + c = 0
2fy = 10y
2gx = - 4x
f=5
g = -2
2fy = - 8y Centre = (a, b) = (- g, - f) = (4, -5)
f=-4 But c = 5
c = a2 + b2 – r2

Baffour Ba Series, Further Mathematics for Schools Page 417


5 = (4)2 + (-5)2 – r2 Redraw
5 = 16 + 25 – r2 12 + 32 – r2 = 5 M
r2 = 16 + 25 – 5 (5, 7) 1 + 9 – r2 = 5
d
r2 = 36 r2 = 1 + 9 – 5 (1, 3)
d
r=√ (4, -5)
r2 = 5 l
r=6 l r=√ √
6 (3, 4)

Now, from the diagram,


By Pythagoras theorem; l2 + r2 = d2……………(1)
d2 = l2 + r2 But r2 = 5
l2 = d2 – r2 l 2 = (x – 3)2 + (0 – 4)2
l 2 = x 2 – 6x + 9 + 16
Now, r = 6, a = 4, b = -5, x = 5, y = 7 l 2 = x 2 – 6x + 25
But d2 = ,( ) ( ) - d2 = (x – 1)2 + (0 – 3)2
2
Put in l = ,( ) ( ) - – r2 d 2 = x 2 – 2x + 1 + 9
l2 = ,( ) ( ( )) - – 62 d 2 = x 2 – 2x + 10
l2 = ,( ) ( ) - – 36
2
l =1 + 144 - 36 eqn (1) now becomes;
l2 = 145 – 36 x2 – 6x + 25 + 5 = x 2 – 2x + 10
l2 = 109 x2 – 6x + 30 = x2 – 2x + 10
l=√ - 6x + 30 = - 2x + 10
l = 10.44 units -6x + 2x = 10 – 30
- 4x = -20
3. The tangent to the circle x2 + y2 – 2x – 6y + 5 x=5
= 0 at the point (3, 4) meets the x – axis at M. But distance of M from C;
Find the distance of M from the center of the d2 = x2 – 2x + 10
circle.
Put in x = 5
Solution d2 = 52 – 2(5) + 10
From x2 + y2 – 2x - 6y + 5 = 0 d2 = 25 – 10 + 10
2gx = -2x d2 = 25
g = -1 d=√
2fy = - 6y d=5
y = -3
Center = (a, b) = (-g, -f) = (1, 3) Exercises 14.10
Find the length of the tangent from the given
But from the equation of the circle; points to the following circles;
c = 5, a = 1, b = 3 1. x2 + y2 + 4x – 6y + 10 = 0 (0, 0)
⇒a2 + b2 – r2 = c 2 2
2. x + y + 6x + 10y – 2 = 0 (-2, 3)

Baffour Ba Series, Further Mathematics for Schools Page 418


3. x2 + y2 − 10x + 8y + 5 = 0 (5, 4) 3. Substitute the coordinates of the point of
tangency (x2, y2), and the gradient of the
Challenge Problem radius/normal, m, into the formula:
1. Calculate the length of the tangents drawn to y – y2 = m(x – x2).
the circle 3x2 + 3y2 − 5x + 11y − 54 = 0 from 4. Simplify to obtain the equation of the normal
the points (- 12, - 3) in the form ax + by + c = 0

Normal to a Circle Worked Examples


A normal line is a line that is perpendicular to 1. Find the equation of the normal to the circle
the tangent line at the point of tangency. The x2 + y2 = 40 at the point (6, 2).
normal being perpendicular to the tangent must
be parallel to the radius. Hence, the gradient of Solution
the normal is equal to the gradient of the radius. x2 + y2 = 40
The normal of a circle always passes through Centre = (a, b) = (-g, -f) = (0, 0)
the center of the circle. Gradient of radius = Gradient of normal
Tangent Gradient of radius,
m= = =

⇒ Gradient of normal =

Normal Substitute x2 = 6 and y2 = 2 and m = in


y – y2 = m(x – x2)
y – 2 = (x – 6)
Equation of a Normal to a Circle
The equation of the normal is stated in the 3(y – 2) = x – 6
general form: ax + by + c = 0, because the 3y – 6 = x – 6
normal is a straight line. x – 3y – 6 + 6 = 0
x – 3y = 0
Steps: The equation of the normal is x – 3y = 0
1. Identify the equation of the circle in the
form: x2 + y2 + 2gx + 2fy + c = 0, the center of 2. Find the equation of the normal to the circle
the circle (-g, -f) = (x1, y1) and a point on the x2 + y2 – 2x – 4y + 3 = 0 at the point (2, 3)
circle as (x2, y2).
2. find the gradient of the radius through the Solution
x2 + y2 – 2x – 4y + 3 = 0
points (x1, y1) and (x2, y2) using =
2gx = -2x
3. Obtain the gradient of the tangent as the g = -1
same as the gradient of the radius: . That is: 2f = - 4
gradient of radius = gradient of normal f = -2

Baffour Ba Series, Further Mathematics for Schools Page 419


Centre = (a, b) = (-g, -f) = (1, 2) 4y + 12 = x – 4
x – 4y – 4 - 12 = 0
Gradient of radius = Gradient of normal x – 4y – 16 = 0
C(1, 2) and P(2, 3) The equation of the tangent is x – 4y – 16 = 0
Gradient of radius, CP,
m= = = =1 Gradient of radius = gradient of normal
⇒Gradient of normal, m = - 4
⇒ gradient of normal = 1
Substitute x2 = 3 and y2 = 1 and m = - 4 in
Substitute x2 = 2 and y2 = 3 and m = 1 in y – y1 = m(x – x1)
y – y2 = m(x – x2) y – 1 = - 4 (x – 3)
y – 3 = (x – 2) y – 1 = - 4x + 12
y–3=x–2 y + 4x – 1 – 12 = 0
x–y–2+3 =0 y + 4x – 13 = 0
x–y+1=0 4x + y – 13 = 0
The equation of the normal is: x – y + 1 = 0 Thhe equation of the normal is 4x + y – 13 = 0

3. Find the equation of the tangent and normal Exercises 14.11


1. Find the equation of the tangent and normal
to the circle x2 + y2 – 8x + 6y + 8 = 0 at the
to the circle x2 + y2 – 4x – 10y – 71 = 0 at the
point (3, 1)
point (10, 11).
Solution
2. Find the tangent and normal line to the circle
x2 + y2 – 8x + 6y + 8 = 0
x2 + y2 = 5 at the point (-2, 1).
2gx = -8x
g=-4
3. Find the tangent and normal line to the circle
2fy = 6y
x2 + y2 – 6x – 10y – 82 = 0 at the point k (-1, -5).
f=3
center = (a, b) = (-g, -f) = (4, -3)
Condition of Tangency
This simply refers to the condition for a line to
Gradient of radius through C(4, -3) and P(3, 1)
be a tangent to a circle.
x1 = 4 and y1 = -3, x2 = 3 and y2 = 1
( )
m= = = =-4 A. Circle centered at the origin O (0, 0)
A line touches a circle if the distance, d of the
⇒ gradient of tangent =
center of the circle to the line is equal to the
radius, r of the circle. That is d = r.
Substitute x1 = 4 and y1 = -3 and m = in The distance of the center (0. 0) of the circle
y – y1 = m(x – x1) x2 + y2 = r2 from the line Ax + By + C = 0 or
y – (-3) = (x – 4) y = mx + c is given by;
4(y + 3) = x – 4 d=r= √
……….(1)

Baffour Ba Series, Further Mathematics for Schools Page 420


For center (x, y) = (0, 0) d=r= ……….(1)
( ) ( ) √
r= √
r=√ With center (a, b)
( ) ( )
d=r=
r(√ )=C (Square both sides) √
( ) ( )
, (√ )- = C2 d=r=
√( )
r2(A + B ) = C2
r= √
Similarly, from y = mx + c; r(√ ) = b – ma – c (Square both sides)
y – mx – c = 0 , (√ )- = (b – ma – c)2
-mx + y – c = 0 compared to Ax + By + C r2(m2 + 1 ) = (b – ma – c )2
A = -m, B = 1 and C = - C substitute in equation (1); This is the condition for the line y = mx + c to
D=r= ……….(1) be a tangent to the circle (x – a)2 + (x – b)2 = r2.

With center (0, 0) Worked Examples


( ) ( ) 1. Show that the line 3y = 2x – 8 is a tangent to
D=r= √ the circle x2 + y2 – 4x – 6y = 0
( ) ( )
D=r=
√( )
Solution
r=√
3y = 2x – 8 (Straight line)
r(√ )=c x2 + y2 – 4x – 6y = 0 (circle)
From the 3y = 2x – 8
Squaring both sides; y= -
, (√ )- = c2
⇒m= and c =
r2(m2 + 1 ) = c2
This is the condition for the line y = mx + c to
be a tangent to the circle x2 + y2 = r2 From x2 + y2 – 4x – 6y = 0
c=0
B. Circle centered at (a, b) 2g = - 4
Circle centered at (a, b) has the general g = -2
equation (x – a)2 + (x – b)2 = r2.
Here, the distance of the circle center (a, b) to 2f = - 6
the line – mx + y – c = 0 must be equal the f = -3
radius of the circle. (-g, -f) = (a, b) = (2, 3)
Similarly, from y = mx + c;
y – mx – c = 0 But a2 + b2 – r 2 = c
- mx + y – c = 0 compared to Ax + By + C 22 + 32 – r 2 = 0
A = - m, B = 1 and C = - C 4 + 9 = r2
Substitute in equation (1) r2 = 13

Baffour Ba Series, Further Mathematics for Schools Page 421


Tangency condition;
10 [. / ]=( ( ) )
r2(m2 + 1 ) = (b – ma – c )2
10 . / = (3 – c)2
13(. / *=( . /( ) . /*
= (3 – c)2
13. /= . /
3–c= √
13 . / = . /
18. 778 = 18.778 3–c=
Since the L. H. S = R. H. S, ⇒3–c= or 3 – c = -
3y = 2x – 8 is a tangent to x2 + y2 – 4x – 6y = 0
c=3– or c = 3 +
2. Given the line -3x + y + 1 = 0 and a circle x2 c= or c =
+ y2 – 6x – 4y + 3 = 0, find the equation of
tangents to the circle which are perpendicular to The equation of tangents are;
the line. T1 = x+. /

Solution T1 = x – and T2 = x+
-3x + y + 1 = 0
y = 3x – 1 Points Inside , On or Outside a Circle
Gradient of the line , m = 3 To find whether a point is inside, on or outside
Gradient of tangent = a circle:
From x2 + y2 – 6x – 4y + 3 = 0
Method 1
2gx = - 6x
Calculate the distance from the center to the
g = -3
point and compare this distance to the radius.
2fy = - 4y
Three cases arise;
f = -2
1. Distance from the center, to the point is less
than the radius.
Center of circle = (a, b) = (-g, -f) = (3, 2)
c = a2 + b2 – r2


But c = 3, (a, b) = (3, 2)
⇒ 3 = 32 + 22 – r2
r2 = 9 + 4 – 3 2. Distance from the center , to the point is
r2 = 10 equal the radius.


Tangency condition;

r2(m2 + 1 ) = (b – ma – c )2
Substitute r2 = 10, m = - , (a, b) = (3, 2)

Baffour Ba Series, Further Mathematics for Schools Page 422


3. Distance from the center, to the point is Substitute (-2, 1)
greater than the radius. (-2)2 + (1)2 – 2(-2) + 8(1) – 8
=4+1+4+8–8

=9>0
Therefore, (-2, 1) is outside the circle.

Exercises 14.12
A. In each of the following determine
Method 2 whether the given point is inside, on or
If the coordinates of a point satisfies the outside the given circle.
equation of a circle x2 + y2 + 2gx + 2fy + c = 0, 1. (3, -2), x2 + y2 = 13
then the point is on the circle. Otherwise the 2. (5, 3), (x – 3)2 + (y – 2)2 = 20
point is either inside or outside the circle. By 3. (4, -1), x2 + y2 + 6x – 4y – 3 = 0
substituting the coordinates into the equation of 4. (-1, 5), x2 + y2 + 4x – 6y – 25 = 0
a circle, one of the following equations may 5. (4, 3), x2 + y2 – 4x + 2y – 15 = 0
arise: 6. (-1, 4), x2 + y2 + 10x – 6y + 21 = 0
1. L.H.S < RHS, the point is inside the circle.
2. LHS = RHS, the point is on the circle. B. 1. The equation of a circle with radius length
3. LHS > RHS, the point is outside the circle. 4 is x2 + y2 – 6x + 2y + k = 0. Find the value of
k, k Z
Worked Examples
Determine whether the points (-3, -2) (5, -1) 2. The equation of a circle with radius length 6
and (-2, 1) are inside, on or outside the circle is x2 + y2 – 2kx + 4y – 7 = 0, k Z.
x2 + y2 – 2x + 8y – 8 = 0 i. Find the centre of the circle and the radius
length in terms of k.
Solution ii. Find the values of k.
Method 2
x2 + y2 – 2x + 8y – 8 = 0 3. The equation of a circle with radius length 5
is x2 + y2 + 2x – 4ky + 12 = 0, k Z. Find the
Substitute (-3, -2) values of k
(-3)2 + (-2)2 – 2(-3) + 8(-2) – 8
= 9 + 4 + 6 – 16 – 8 4. A(k, 1) and B(-7, -k) are the end points of the
= -5 < 0 dianmeter of circle C. If the centre of C is (2,
Therefore, (-3, -2) is inside the circle -5) , find the value of k and the radius length of C.
Substitute (5, -1) 5. A(-2, 0) and B(6, 2) are points of a circle of
(5)2 + (-1)2 – 2(5) + 8(-1) – 8 center C(2k, k).
= 25 + 1 –10 – 8 – 8 i. Express in term of k:
=0 a. | | b. | |
Therefore, (5, -1) is on the circle.

Baffour Ba Series, Further Mathematics for Schools Page 423


ii. Find the value of k and the equation of the 3. Solve the quadratic equation either by
circle. factorization or quadratic method to obtain the
value(s) of x.
Intersection of a Circle and a Line 4. Substitute the value (s) of x in the linear
There are three ways a line and a circle can be equation and obtain the value of y.
associated. 5. Check your solutions by substituting your
1. The line cuts the circle at two distinct points. point(s) in both equations.

Note:
If the equation of the line is in the form x = a,
substitute the value of a in the circle equation.

In this case, there are two points of intersection. Worked Examples


2. The line is a tangent to the circle. 1. Find the point of intersection of the circle
x2 + y2 + 18x + 20y + 81 = 0 and the line
y=x+1

Solution
x2 + y2 + 18x + 20y + 81 = 0……(1)
y = x + 1………………………..(2)
In this case, there is only one point of
intersection Put eqn (2) into equation (1)
x2 + (x + 1)2 + 18x + 20 (x + 1) + 81 = 0
3. The line misses the circle. x2 + x2 + 2x + 1 + 18x + 20x + 20 + 81 = 0
2x2 + 40x + 102 = 0
x2 + 20x + 51 = 0
(x + 3) (x + 17) = 0 (Factorization)
x + 3 = 0 or x + 17 = 0
In this case, there is no point of intersection. x = -3 or x = -17

Regardless of the case, the three situations can Put x = -3 in eqn (2)
be verified algebraically using the substitution y = -3 + 1= -2
method. (x, y) = (-3, -2)

Steps Put x = -17 in eqn (2)


1. Solve the linear equation for y. y = -17 + 1 = - 16
2. Substitute the value of y in the equation of (x, y) = (-17, -16)
the circle and simplify to obtain a quadratic Therefore, the line intersects the circle at the
equation involving x. points (-3, -2) and (-17, -16)

Baffour Ba Series, Further Mathematics for Schools Page 424


2. Find the point of intersection of the line x – 1 = - 284
= 0 and the circle x2 + y2 + 18x + 20y + 81 = 0. b2 – 4ac < 0, so there are no real roots.
Therefore, the line does not meet the circle.
Solution
x2 + y2 + 18x + 20y + 81 = 0……(1) 4. Show that the line y = 2x + 1 intersects the
From x – 1 = 0, circle x2 + y2 – 6x – 7y + 9 = 0 and determine
x = 1……………………………..(2) the point of intersection.

Put eqn (2) into equation (1); Solution


12 + y2 + 18(1) + 20y + 81 = 0 x2 + y2 – 6x – 7y + 9 = 0………….(1)
y2 + 20y + 100 = 0 y = 2x + 1………………………..(2)
(y + 10) (y + 10) = 0 (Factorization)
y + 10 = 0 or y + 10 = 0 Put eqn (2) into eqn (1);
y = -10 or y = -10 x2 + (2x + 1)2 – 6x – 7(2x + 1) + 9 = 0
x2 + 4x2 + 4x + 1– 6x – 14x – 7 + 9 = 0
Put y = -10 in eqn (1) 5x2 – 16x + 3 = 0
x2 + (-10)2 + 18x + 20(-10) + 81 = 0 (5x – 1) (x – 3) = 0 (Factorization)
x2 + 100 + 18x – 200 + 81 = 0 ⇒x = or x = 3
x2 + 18x – 19 = 0 (Factorization)
x=1 Put x = in eqn (2);
The point of intersection is (x, y) = (1, -10)
y = 2( ) + 1 =
3. Show whether or not the line y = -x + 3 (x, y) = ( , )
intersects with the circle x2 + y2 + 18x + 20y +
81 = 0
Put x = 3 in eqn (2);
y = 2(3) + 1 = 7
Solution
(x, y) = (3, 7)
x2 + y2 + 18x + 20y + 81 = 0……(1)
Therefore the line intersects the circle at the
y = - x + 3………………………..(2)
points ( , ) and (3, 7)
Put eqn (2) into eqn (1);
x2 + (-x + 3)2 + 18x + 20 (- x + 3) + 81 = 0 Exercises 14.13
x2 + x2 – 6x + 9 + 18x – 20x + 60 + 81 = 0 A. Find the coordinates of the point, or
2x2 – 8x + 150 = 0 points of intersection of the given line and
x2 – 4x + 75 = 0 circle. State wheteher or not the line is a
Since the equation cannot factorize, find the tangent to the circle.
discriminant; 1. x – 3y = 0, x2 + y2 = 10
b2 – 4ac 2. x + 3y – 5 = 0, x2 + y2 = 5
= (- 4)2 – 4(1) (75) 3. x + 2y – 5 = 0, x2 + y2 = 10

Baffour Ba Series, Further Mathematics for Schools Page 425


4. 4x – y – 17 = 0, x2 + y2 = 17
5. x – y – 1 = 0, x2 + y2 – 2x – 2y + 1= 0
6. x – 2y – 1 = 0, x2 + y2 + 2x – 8y – 8 = 0

B. 1. Show that the line 3y = 2x – 8 is a tangent It is also possible that one circle may be inside
to the circle x2 + y2 – 4x – 6y = 0 and determine another as shown;
the point of contact.

2. Show that the line y = 2x – 8 does not


intersect the circle x2 + y2 – 2x – 2y – 3 = 0.

3. Find the point of intersection of the circle Unlike the point of intersection of a straight
with the line given their equations (x – 2)2 + (y line and a circle, it is not possible to substitute
+ 3)2 = 4 and 2x + 2y = -1. one equation into the other. Instead, length is
investigated to see whether the two circle meet
C. 1. Show that the line y = -3x – 10 is a
or not.
tangent to the circle equation circle x2 + y2 – 8x
+ 4y – 20 = 0 and find the point of contact. If one circle has radius R and a second has
radius r and the distance between their centers
2. Show that the line y = 2x + 1 does not
is d, then;
intercept the circle with equation x2 + y2 – 2x +
1. If d > R + r , then the two circles do not
4y + 1 = 0
intersect.
3. i. A circle has center (a, 0), a > 0 and radius
4 units. Write down the equation of the circle.
ii. Show that if y = x is a tangent to the circle,
then a = 4√ .
2. If d = R + r, the two circles intersect in one
point.
4. A circle with center (5, 6) touches the line
2y + 3x. Find the equation of the circle.

D. The equation of a circle is (x – 2)2 + (y – 3)


= 4. Show whether each point is on the circle,
in the circle or outside the circle; 3. If d < R + r, then the two circles have two
1. (0, 0) 2. (2, - 4) 3. (0, - 3) 4. (1, - 4) points of intersection.
5. (3, -1) 6. (2, -5) 7. (2.5, -3) 8. (2, 0)

Point of Intersection of Two Circles


Two circles can intersect in two points, one
point or not at all as shown below.

Baffour Ba Series, Further Mathematics for Schools Page 426


Worked Examples (x – 1)2 + (y + 1)2 = 16
1. Find the point of intersection of the circles
given by the equations x2 + y2 – x + y = 4 and Expand brackets in both equations
x2 + y2 = 5 (x – 2)2 + (y – 3)2 = 9
x2 – 4x + 4 + y2 – 6y + 9 = 9……….(1)
Solution (x – 1)2 + (y + 1)2 = 16
x2 + y2 – x + y = 4 ……………(1) x2 – 2x + 1 + y2 + 2y + 1 = 16…..…(2)
x2 + y2 = 5…………………….(2)
Multiply all terms of eqn (1) by -1
-1 × eqn (1) - x2 + 4x – 4 – y2 + 6y – 9 = - 9……..(3)
- x2 – y2 + x – y = - 4 …………(3)
Add eqn (2) and eqn (3) to obtain a linear
eqn (2) + eqn (3); equation
x–y=1 x2 – 2x + 1 + y2 + 2y + 1 = 16……..(2)
x=1+y - x2 + 4x – 4 – y2 + 6y – 9 = - 9……..(3)
⇒ 2x – 3 + 8y – 8 = 7
Put x = 1 – y in eqn (1); 2x + 8 y = 7 + 3 + 8
(1 + y)2 + y2 – (1 + y) + y = 4 2x + 8 y = 18
1 + 2y + y2 + y2 – 1 – y + y = 4
2y2 + 2y = 4 x + 4y = 9 (Reduced )
2y2 + 2y – 4 = 0 Make x the subject to obtain;
y2 + y – 2 = 0 x = 9 – 4y
(y + 2) (y – 1) = 0 Put x = 9 – 4y in eqn (1)
y = - 2 or y = 1 (9 – 4y)2 – 4(9 – 4y) + 4 + y2 – 6y + 9 = 9
81 – 72y + 16y2 – 36 + 16y + 4 + y2 – 6y = 0
Put y = - 2 or y = 1 in x = 1 + y 17y2 – 62y + 49 = 0
When y = - 2,
x = 1 – 2 = -1 Solving by quadratic formula;
√ √
y= = 2.49 or y = = 1.16
When y = 1,
x=1+1=2 Substitute y = 2.49 and y = 1.16 in x = 9 – 4y
The points of intersection are (- 1, 2) and (2, 1)
When y = 2.49, x = 9 – 4(2.49) = - 0.96
2. Find the point of intersection of the circles
given by the equations (x – 2)2 + (y – 3)2 = 9 When y = 1.16, x = 9 – 4(1.16) = 4.37
and (x – 1)2 + (y + 1)2 = 16
Therefore, the two points of intersection of the
Solution two circles are (- 0.96, 2.49) and (4.37, 1.16)
(x – 2)2 + (y – 3)2 = 9

Baffour Ba Series, Further Mathematics for Schools Page 427


Orthogonal Circles 2(- 4) (0) + 2 (-3) (-1) = 21 + (-15)
Two circles are said to be orthogonal if the 6=6
tangent at their point of intersection are at right L.H.S = R. H. S.
angles. This implies that orthogonal circles are The given circles are orthogonal.
perpendicular to each other.
Exercises
1. Show that the circles x2 + y2 – 8x + 6y – 23
= 0 and x2 + y2 – 2x – 5y + 16 = 0 are
orthogonal.

Loci
If a point moves on a plane satisfying some
geometrical conditions, then the path traced out
by the point is called its locus, plural loci.
Conditions Locus is determined when some geometrical
1. If two circles are cut orthogonally, then it conditions are given. Likely, the conditions of
must satisfy the following condition: all points on the locus will satisfy the given
2 +2 = + geometrical conditions.

2. If the two circles have radii r, R and their Equation of Locus


centrs are located at distance d, from each The algebraic form of a given geometrical
other, then the circles are orthogonal if and only condition which is satisfied by the coordinates
if : d2 = R2 + r2 of all points on the locus is called the equation
In this case, it is required that the points of of the locus to the moving point. Thus, the
intersection of the two circles are calculated. coordinates of all points on the locus satisfy its
equation of locus but the coordinates of the
Worked Examples point which does not lie on the locus, do not
Show that the circles x2 + y2 – 8x – 6y + 21 = 0
satisfy the equation of locus. On the other hand,
and x2 + y2 – 2y – 15 = 0 are orthogonal.
the points whose coordinates satisfy the
equation of locus lie on the locus of the moving
Solution
point. Examples of moving points are the hands
x2 + y2 – 8x – 6y + 21 = 0 ……….(1)
of a wall clock, standing and ceiling fans, etc.
x2 + y2 – 2y – 15 = 0…………….(2)
Practical Explanation
From eqn (1);
= - 4, = - 3, and = 21
A h P
From eqn (2);
=0 = - 1 and = - 15 B
Substitute in 2 +2 = +

Baffour Ba Series, Further Mathematics for Schools Page 428


If a goat at P is tied to a taut rope of length, h The equation of the locus of a moving point
pegged at A as shown above, then the P(x, y) which is always at a constant distance
movement of the goat can be described as (r) from a fixed point
follows: Worked Example
1. Moving in a circular/elliptical path, either 1. Find the equation of the locus of a moving
clockwise or anticlockwise. point P(x, y) which is always at a distance of 5
2. Moving towards A, giving PA; units from a fixed point Q(2, 4)
3. Moving towards B, giving PB.
Solution
The points in the path is simply described by (x – x1)2 + (y – y1)2 = r2
the equation AP + BP = h (x – x1)2 + (y – y1)2 = r2
x2 + 4x + 4 + y2 – 8y + 16 = 25
Forms of Equation of Locus x2 + y2 + 4x – 8y + 4 + 16 – 25 = 0
1. The equation of the locus of a moving point x2 + y2 + 4x – 8y – 5 = 0
P(x, y) which is always at a constant distance
(r) from a fixed point (x1, y1) is: Exercises 14.15
(x – x1)2 + (y – y1)2 = r2 1. Show that the equation of the locus of a point
that moves in such a way that its distance from
2. The equation of the locus of a moving point a fixed point (3, -1) is 6 units, is given by x2 +
P(x, y) which is always at a constant distance y2 – 6x + 2y – 26 = 0
(r) from a two fixed point (x1, y1) and (x2, y2)
with a ratio m : n respectively is: 2. Find the equation of the locus of a moving
( ) )
= point P(x, y) which is always at a distance of 5
( ) ( )
units from a fixed point Q(2, 4)
3. The equation of the locus of a moving point
P(x, y) which is always equidistance from two Locus of a Point Equidistance from Two fixed
fixed point A(x1, y1) and B(x2, y2) is the Points
perpendicular bisector of the straight line AB, Worked Examples
such that AP = PB or PA = PB 1. Find the equation of the locus of a moving
point which is always equidistance from the
Steps in finding plane loci: points (2, -1) and (3, 2). What curve does the
1. Choose a coordinate system that will make locus represent?
computations and equations very simple.
2. Identify the given conditions and write it in Solution
mathematical form involving the coordinates Let the point be P(x, y), A(2, -1) and B(3, 2).
(x, y). PA = PB
3. Simplify the resulting equations. Since we need /AP/ and /PB/ in terms of x and y;
4. Identify the shape of the equation if the need
be. PA2 = PB2

Baffour Ba Series, Further Mathematics for Schools Page 429


PA2 = (x – 2)2 + (y – (- 1) )2 PB2 = x2 – 10x + 25 + y2 + 2y + 1
PA2 = (x – 2)2 + (y + 1)2
PA2 = x2 – 4x + 4 + y2 + 2y + 1 PB2 = x2 + y2 – 10x + 2y + 25 + 1
PA2 = x2 + y2 – 4x + 2y + 4 + 1 PB2 = x2 + y2 – 10x + 2y + 26
PA2 = x2 + y2 – 4x + 2y + 5
But PA2 = PB2
PB2 = (x – 3)2 + (y – 2)2 ⇒x2 + y2– 6x – 4y + 13 = x2 + y2 – 10x + 2y + 26
PB2 = x2 – 6x + 9 + y2 – 4y + 4 – 6x - 4y + 13 = – 10x + 2y + 26
PB2 = x2 + y2 – 6x – 4y + 4 + 9 – 6x + 10x – 4y – 2y + 13 – 26 = 0
PB2 = x2 + y2 – 6x – 4y + 13 4x – 6y – 13 = 0
The equation of locus of P is 4x – 6y – 13 = 0
PA2 = PB2
x2 + y2 – 4x + 2y + 5 = x2 + y2 – 6x – 4y + 13 3. What is the locus of the point which moves
– 4x + 2y + 5 = – 6x – 4y + 13 so that its distance from the point (-2, 1) is
– 4x + 6x + 2y + 4y + 5 – 13 = 0 equal to its distance from (3, -2)?
2x + 6y – 8 = 0 (Divide through by 2;)
Solution
Let the point be P(x, y), A(-2, 1) and B(3, -2)?
x + 3y – 4 = 0
PA = PB
The equation of locus of P is x + 3y – 4 = 0
Express /AP/ and /PB/ in terms of x and y;
This represents a straight line.
PA2 = PB2
2. Find the equation of locus of point P which
PA2 = (x + 2)2 + (y – 1)2
moves so that it is equidistance from two fixed
PA2 = x2 + 4x + 4 + y2 – 2y + 1
points A and B whose coordinates are (3, 2) and
PA2 = x2 + y2 + 4x – 2y + 4 + 1
(5, -1).
PA2 = x2 + y2 + 4x – 2y + 5

Solution PB2 = (x – 3)2 + (y + 2)2


Let the point be P(x, y), A(3, 2) and B(5, -1). PB2 = (x – 3)2 + (y + 2)2
PA = PB PB2 = x2 – 6x + 9 + y2 + 4y + 4
Express /AP/ and /PB/ in terms of x and y; PB2 = x2 + y2 – 6x + 4y + 9 + 4
PA2 = PB2 PB2 = x2 + y2 – 6x + 4y + 13

PA2 = (x – 3)2 + (y – 2)2 But PA2 = PB2


PA2 = x2 – 6x + 9 + y2 – 4y + 4 ⇒ x2 + y2 + 4x – 2y + 5 = x2 + y2 – 6x + 4y + 13
PA2 = x2 + y2 – 6x – 4y + 4 + 9 4x - 2y + 5 = – 6x + 4y + 13
PA2 = x2 + y2 – 6x – 4y + 13 4x + 6x – 2y – 4y + 5 – 13 = 0
10x – 6y – 8 = 0
PB2 = (x – 5)2 + (y – (-1)2 5x – 3y – 4 = 0
PB2 = (x – 5)2 + (y + 1)2 The equation of locus of P is 5x - 3y – 4 = 0
Baffour Ba Series, Further Mathematics for Schools Page 430
Exercises 14.16 (r) from a two fixed point (x1, y1) and (x2, y2)
A. 1. Write the equation of the locus of points with a ratio m : n respectively is:
equidistance from points A(1, 4) and B(7, 8) ( ) )
=
( ) ( )

2. N is a locus which moves in such a way that


NP = NQ. Given that P and Q are coordinates Worked Examples
(-3, 6) and (6, - 4) respectively, find the 1. Find the equation of the locus of a moving
equation of locus N point P, such that its distance from the point
A(4, 3) and the point B (5, 1) is in the ratio 3 : 1
3. What is the equation that represents a locus
of points equidistance from (-3, -2) and (-3, 4) Solution
A(4, 3) and B (5, 1), P (x, y) is in the ratio 3 : 1
4. Find the locus of the point P(x, y) such that By the given condition;
the distance from A(-5, 2) is equal to its =
distance from B(4, -3) 3PB = PA
(3PB)2 = PA2
5. Find the equation of the locus of a moving 9PB2 = PA2
point P(x, y) which is always equidistance from
points A(-2, 3) and B(4, -1). 9PB2 = 9[(x – 5)2 + (y – 1)2]
= 9[x2 – 10x + 25 + y2 – 2y + 1]
6. A is the point (2, -5) and B is the point (4, = 9[x2 + y2 – 10x – 2y + 26]
-9). Find the mid-point of AB and the locus of = 9x2 + 9y2 – 90x – 18y + 234
the point equidistant from A and B. = 9x2 + 9y2 – 90x – 18y + 234

B. 1. What is the locus of a point which is


PA2 = (x – 4)2 + (y – 3)2
equidistance from the origin and the point (-2, 5)
= x2 – 8x + 16 + y2 – 6y + 9
2. What is the locus of a point which is = x2 + y2 – 8x – 6y + 25
equidistance from the points (6, 3) and (-5, 2)
3. Describe the locus of the point equidistance But 9PB2 = PA2
from the points (4, -8) and (4, - 4). ⇒ 9x2 + 9y2 – 90x – 18y + 234 = x2 + y2 – 8x –
6y + 25
4. Describe the locus of the point equidistance 9x2 – x2 + 9y2 – y2 – 90x + 8x – 18y + 6y + 234
from the points (2, - 4) and (6, - 4). – 25 = 0
8x2 + 8y2 – 82x – 12y + 209 = 0
Involving a Constant Distance (r) from Two
Fixed Points with a Ratio, m : n 2. A(2, 0) and B (0, -2) are two fixed points.
The equation of the locus of a moving point Point P moves with a ratio so that AP : PB = 1 :
P(x, y) which is always at a constant distance 2. Find the equation of the locus of point P

Baffour Ba Series, Further Mathematics for Schools Page 431


Solution 4. The ratio of the distance of a moving point
A(2, 0) , B (0, -2), P (x, y) ratio 1 : 2 from the point (3, 4) and (1, -2) is 2 : 3. Find
By the given condition; the locus of the moving point.
AP : PB = 1 : 2
= 5. A (-1,5) and B (2, -1) are two given points in
a plane. . P(x, y) is a variable point in the plane
2PA = PB
such that /AP/ = 2/BP/.
(2PA)2 = PB2
i. show that the equation of the locus of P is
4PA2 = PB2
x2 + y2 – 6x + 6y – 2 = 0.

4PA2 = 4[(x – 2)2 + (y – 0)2] ii. Find a function of the form . / →. /


= 4[x2 – 4x + 4 + y2 ] such that the locus of the image in (i) has an
= 4[x2 + y2 – 4x + 4] equation of the form x2 + y2 = r2 where a, b r
= 4x2 + 4y2 – 16x + 16 are constants. Describe the locus of P.

PB2 = (x – 0)2 + (y + 2)2 Locus of a Point P(x, y) such that PA is


= x2 + y2 + 4y + 4 Perpendicular to PB, where A and B are
= x2 + y2 + 4y + 4 constants
Worked Examples
But 4PA2 = PB2 1. A point P(x, y) moves so that AP and BP are
⇒ 4x2 + 4y2 – 16x + 16 = x2 + y2 + 4y + 4 perpendicular. Given A(3, 2) and B(- 4, 1) Find
4x2 – x2 + 4y2 – y2 – 16x – 4y + 16 – 4 = 0 the equation of the locus of P.
3x2 + 3y2 – 16x – 4y + 12 = 0
Solution
Since the product of slopes of perpendicular
Exercises 14.17
lines is -1,
1. P is a moving point such that its distances
from the point A(2, 5) and B (0, 3) is in the ratio . /. / = -1
2 : 1. Find the equation of locus P. ( )( )
( )(
= -1
)
(y – 2)(y – 1) = - (x – 3)(x + 4)
2. Given A(5, -2) and B(2, 1) are two fixed
y(y – 1) – 2(y – 1) = - [x(x + 4) – 3 (x + 4)]
points Point Q moves such that the ratio of AQ
: QB = 2 : 1. Show that the equation of locus of y2 – y – 2y + 2 = - [x2 + 4x – 3x – 12]
the point Q is x2 + y2 – 2x – 4y – 3 = 0 y2 – 3y + 2 = - [x2 + x – 12]
y2 – 3y + 2 = - x2 – x + 12
x2 + y2 + x – 3y + 2 – 12 = 0
3. A and B are two given points whose
coordinates are (-5, 3) and (2, 4) respectively. A x2 + y2 + x – 3y – 10 = 0
point moves in such a manner that PA : PB = 3
: 2. Find the equation of the locus traced out by Locus of a Point p(x, y) that is always at a
P. What curve does it represents? Constant Distance Units from a given Line

Baffour Ba Series, Further Mathematics for Schools Page 432


Worked Examples Exercises 14.19
1. What is the locus of the point P(x, y) that is A. 1. Find the locus of a point which moves so
always 3 units from the line x = 5. that its distance from the point (8, 0) is twice its
distance from the line x = 2 .
Solution
A(5, y) P(x, y) and d = 3 2. Find the locus of a point which moves so that
By the given condition; its distance from the point (2, 0) is half its
̅̅̅̅ = 3 distance from the line x = 8 .

But ̅̅̅̅̅ = √( ) ( ) Involving Sums and Differences


2 2
⇒ (x – 5) + (y – y) = 3 2 Worked Examples
x2 – 10x + 25 + 0 = 9 1. A is a point (1, 0) and B(-1, 0). Find the locus
x2 – 10x + 25 – 9 = 0 of the point P which moves so that PA – PB = 2
x2 – 10x + 16 = 0
(x – 2) (x – 8) = 0 (factorisation) Solution
x – 2 = 0 or x – 8 = 0 Let the point be P(x, y), A(1, 0) and B(-1, 0)
x = 2 or x = 8 (Two vertical lines) By the given condition;
PA – PB = 2
2. Find the locus of a point which is PA = 2 + PB
equidistance from the origin and the line x = -1.
PA2 = (2 + PB)2 (Square both sides)
Solution PA2 = 4 + 4PB + PB2
Let the point be P(x, y), A (0, 0) and B(-1, y) PA2 – PB2 – 4 = 4PB……..(1)
By the given condition,
PA = PB PA2 = (x – 1)2 + (y – 0)2
PA2 = PB2 PA2 = x2 – 2x + 1 + y2
PA2 = x2 + y2 – 2x + 1
PA2 = (x – 0)2 + (y – 0)2
PA2 = x2 + y2 PB2 = (x – 1)2 + (y – 0)2
PB2 = x2 + 2x + 1 + y2
PB2 = (x + 1)2 + (y – y)2 PB2 = x2 + y2 + 2x + 1
PB2 = x2 + 2x + 1 + 0
PB2 = x2 + 2x + 1 PA2 – PB2
PA2 = x2 + y2 – 2x + 1 – [x2 + y2 + 2x + 1]
But PA2 = PB2 PA2 = x2 + y2 – 2x + 1 – x2 – y2 – 2x – 1
⇒x2 + y2 = x2 + 2x + 1 PA2 = - 4x
x2 – x2 + y2 = 2x + 1
y2 = 2x + 1 Now, substitute PA2 – PB2 in eqn (1)
Therefore, the locus of the points is y2 = 2x + 1 ⇒- 4x – 4 = 4PB

Baffour Ba Series, Further Mathematics for Schools Page 433


⇒(- 4x – 4)2 = (4PB)2 (Square both sides) Divide through by 16;
2
16x + 32x + 16 = 16PB 2
x2 + 8x + 16 = 4x2 + 4y2 + 8x + 4
16x2 + 32x + 16 = 16(x2 + y2 + 2x + 1)
16x2 + 32x + 16 = 16x2 + 16y2 + 32x + 16) Regroup and equate to zero;
16y2 = 0 4x2 – x2 + 4y2 + 8x – 8x + 4 – 16 = 0
y=0 3x2 + 4y2 – 12 = 0
The locus of the point P is y = 0 (a straight line) The locus of the point is 3x2 + 4y2 – 12 = 0
This represents an ellipse.
2. A is the point (1, 0) and B(-1, 0) is the point
P which moves so that PA + PB = 4. 3. Find the locus of point which moves so that
the sum of the squares of its distances from the
Solution points (-2, 0) and (2, 0) is 26.
Let the point be P(x, y), A(1, 0) and B(-1, 0)
By the given condition; Solution
PA + PB = 4 Let the point be P(x, y), A(-2, 0) and B(2, 0)
PA = 4 – PB By the given condition,
PA2 + PB2 = 26
PA2 = (4 – PB)2 (square both sides)
2 2
PA = 16 – 8PB + PB PA2 = (x + 2)2 + (y – 0)2
PA2 – PB2 – 16 = -8PB……..(1) PA2 = x2 + 4x + 4 + y2
PA2 = x2 + y2 + 4x + 4
PA2 = (x – 1)2 + (y – 0)2
PA2 = x2 – 2x + 1 + y2
PB2 = (x – 2)2 + (y – 0)2
PA2 = x2 + y2 – 2x + 1
PB2 = x2 – 4x + 4 + y2
PB2 = (x – 1)2 + (y – 0)2 PB2 = x2 + y2 – 4x + 4
PB2 = x2 + 2x + 1 + y2
PB2 = x2 + y2 + 2x + 1 But PA2 + PB2 = 26
⇒ x2 + y2 + 4x + 4 + x2 + y2 – 4x + 4 = 26
PA2 – PB2 2x2 + 2y2 + 8 = 26
PA2 = x2 + y2 – 2x + 1 – [x2 + y2 + 2x + 1] 2x2 + 2y2 = 26 – 8
PA2 = x2 + y2 – 2x + 1 – x2 – y2 – 2x – 1 2x2 + 2y2 = 18
PA2 = - 4x x2 + y2 = 9
The locus of the point is x2 + y2 = 9
Now, substitute PA2 – PB2 in eqn (1)
⇒- 4x – 16 = -8PB Exercises 14.20
⇒(- 4x – 16)2 = (-8PB)2 (Squaring both sides) 1. Find the locus of a point which moves so that
16x2 + 128x + 256 = 64PB2 the sum of the squares of its distances from the
16x2 + 128x + 256 = 64(x2 + y2 + 2x + 1) point (3, 0) and (-3, 0) is always equal to 50.
16x2 + 128x + 256 = 64x2 + 64y2 + 128x + 64

Baffour Ba Series, Further Mathematics for Schools Page 434


2. A is the point (3, -1) and B is the point (5, 3). PA = 2PB
Show that the locus of the point P, which PA2 = (2PB)2
moves so that PA2 + PB2 = 28, is a circle. Find PA2 = 4PB2
its center and radius. PA2 = (x + 1)2 + (y – 2)2
PA2 = x2 + 2x + 1 + y2 – 4y + 4
3. Find the locus of a point which moves so that PA2 = x2 + y2 + 2x – 4y + 5
the sum of the squares of its distances from the
lines x + y = 0 and x – y = 0 is 4. 4PB2 = 4[(x – 0)2 + (y – 0)2]
4PB2 = 4(x2 + y2)
Involving Other Conditions 4PB2 = 4x2 + 4y2
1. Choose a coordinate system that will make
computations and equations very simple. But PA2 = 4PB2
2. Identify the given conditions and write it in ⇒ x2 + y2 + 2x – 4y + 5 = 4x2 + 4y2
mathematical form involving the coordinates 4x2 – x2 + 4y2 – y2 – 2x + 4y – 5 = 0
(x, y). 3x2 + 3y2 – 2x + 4y – 5 = 0
3. Simplify the resulting equations. Therefore, the locus of the point P is;
4. Identify the shape of the equation if the need 3x2 + 3y2 – 2x + 4y – 5 = 0 (circle)
be.
3. Find the equation of the locus of the point
Worked Examples which moves so that its distance from (4, -3) is
1. A point moves in such a manner that three always one – half its distance from (-1, -1).
times of its distance from the x – axis is greater
by 7 than 4 times of its distance from the y – Solution
axis. Find the equation of its locus. Let the point be P(x, y), A(4, - 3) and B(-1, -1)
By the given condition;
Solution PA = PB
Let the point be P(x, y),
2PA = PB
Then by the given conditions, the distance of P
(2PA)2 = PB2
from the x – axis is:
4PA2 = PB2
3x = 4y + 7
3x – 4y – 7 = 0 (circle)
4PA2 = 4[(x – 4)2 + (y + 3)2]
= 4(x2 – 8x + 16 + y2 + 6y + 9)
2. Find the locus of a point P, whose distance
= 4(x2 + y2 – 8x + 6y + 25)
from A(-1, 2) is twice its distance from the
= 4x2 + 4y2 – 32x + 24y + 100
origin.
PB2 = (x + 1)2 + (y + 1)2
Solution
= x2 + 2x + 1 + y2 + 2y + 1
Let the point be P(x, y), A(-1, 2) and B(0, 0)
= x2 + y2 + 2x + 2y + 2
By the given condition;

Baffour Ba Series, Further Mathematics for Schools Page 435


4PA2 = PB2 times its distance from the origin.
4x2 + 4y2 – 32x + 24y + 100 = x2 + y2 + 2x + 2y + 2
4x2 – x2 + 4y2 – y2 – 32x – 2x + 24y – 2y + 4. A point moves so that it distance from the
100 – 2 = 0 origin is twice its distance from the point (3, 0).
3x2 + 3y2 – 34x + 22y + 98 = 0 (Circle) Show that the locus is a circle, and find its
center and radius.
Exercises 14.21
A. 1. Given A(0, 3) and point B(1, 4) find the B. Find the equation for the set of points
equation of the locus of a moving point Q, such (x, y) satisfying the given conditions;
that AQ = 2QB 1. It is equidistance from (5, 8) and (-2, 4)
2. The sum of the distance from (0, 4) and
2. A point P moves so that its distance from (0, - 4) is 10.
A(2, 1) is twice its distance from B(- 4, 5). 3. The sum of the squares of its distance from
What is the locus of P? (3, 0) and (-3, 0) is 50.
4. It is on the line having slope 2 and containing
3. Find the locus of a point which move so that the point (3, -2).
its distance from the point A(-2, 0) is three 5. The difference of its distances from (3, 0)
and (-3, 0) is 2.

Baffour Ba Series, Further Mathematics for Schools Page 436


15 PARABOLA Baffour Ba Series

Meaning of a Parabola √( ) = x + a.
It is the set of all points (x, y) in a plane that are
the same distance from a fixed line, called Squaring both sides gives;
directrix and a fixed point or focus not on the (x − a)2 + y2 = x2 + 2ax + a2,
directrix. or
x2 − 2ax + a2 + y2 = x2 + 2ax + a2.
The Standard Equation of a Parabola This reduces to:
A parabola is the path traced out by (or “locus” y2 = 4ax
of) a point, P, whose distance, SP, from a fixed This is called the standard equation of a
point, S, called the “focus”, is equal to its parabola with “vertex” at the origin, focus at
perpendicular distance, PM, from a fixed line, l, (a, 0) and axis of symmetry along the x-axis.
called the “directrix”.
All other versions of the equation of a parabola
M P which shall be considered is based on this
version.
S
l Note
(i) If a is negative, the bowl of the parabola
For convenience, take the directrix, l, to be a faces in the opposite direction towards negative
vertical line - with the perpendicular onto it x values.
from the focus, S, being the x-axis. The y – axis
can be taken to be the directrix itself, but the (ii) Any equation of the form y2 = kx, where k is
equation of the parabola turns out to be simpler a constant, represents a parabola with vertex at
using a different line; namely the line parallel the origin and axis of symmetry along the x-
to the directrix passing through the mid-point of
axis. It‟s focus will lie at the point . , /; it is
the perpendicular from the focus onto the
directrix. This point is one of the points on the worth noting this observation for future
parabola so that the curve can pass through the reference.
origin. y
P Other Forms of the Equation of a Parabola
M
(a) Vertex at (0, 0) with focus at (0, a)
x y
S(a, 0)
x = -a
l (0, a)

Letting the focus be the point (a, 0) (since it lies x


on the x-axis) the definition of the parabola
implies that SP = PM. That is,
Baffour Ba Series, Further Mathematics for Schools Page 437
This parabola is effectively the same as the point (h, k) with X-axis and Y -axis, the
standard one except that the roles of x and y parabola would have equation : X2 = 4aY.
have been reversed. We may assume, therefore
that the curve has equation x2 = 4ay; With reference to the original axes, therefore,
the parabola has equation:
(b) Vertex at (h, k) with focus at (h + a, k) (x − h)2 = 4a (y − k).
y
Notes:
i. Such a parabola will often be encountered in
(h + a, k)
(h, k)  the expanded form of this equation, containing
k x quadratic terms in x and linear terms in y.
) Conversion to the stated form by completing
the square in the x terms will make it possible
Consider a temporary change of origin to the to identify the vertex and focus.
point (h, k), with X-axis and Y -axis, the
parabola would have equation : Y2 = 4aX. Worked Examples
1. Find the equation of a parabola with focus at
With reference to the original axes, therefore, (0, 4) and vertex at (0, 0)
the parabola has equation:
(y − k)2 = 4a(x − h). Solution
Vertex at (0, 0) and focus at (0, 4)
Notes: Distance between the vertex and focus
(i) Such a parabola will often be encountered in | | = √( ) ( )
the expanded form of this equation, containing | |=√
quadratic terms in y and linear terms in x. | |=4
Conversion to the stated form by completing
the square in the y terms will make it possible Vertex at (0, 0) and focus at (0, 4) implies that
to identify the vertex and focus. the parabola opens up on the y – axis. The
equation of the parabola is given by:
(c) Vertex at (h, k) with focus at (h, k + a) x2 = 4ay
y x2 = 4(4)y
x2 = 16y
(h, k + a)
k 2. Find the equation of a parabola with vertex at
(h, k) ) (0, 2) and focus at (0, 6)
x
Solution
Vertex at (0, 2) and focus at (0, 6)
Considering a temporary change of origin to the
Distance between the vertex and focus

Baffour Ba Series, Further Mathematics for Schools Page 438


| | = √( ) ( ) Exercises 15.1
| |=√ 1. Find the equation of a parabola with vertex
| |=4 at (3, 1) and focus at (3, 5).

Vertex at (0, 2) and focus at (0, 6) means that 2. Find an equation for the parabola with focus
the parabola opens up on the y – axis at (-8, - 4) and vertex at (6, - 4)

The equation of a parabola with vertex at (h, k) 3. If a parabolas vertex is at (4, -2) and its
is given by : (x – h)2 = 4a(y – k) focus at (4, 4), write the equation of this
parabola.
Vertex at (h, k) = (0, 2) ⇒ h = 0, k = 2
By substitution, the equation of the parabola is 4. Find the equation of the parabola whose
(x – 0)2 = 4(4)(y – 2) coordinates of vertex and focus are (-2, 3) and
(1, 3) respectively.
x2 = 16(y – 2)
x2 = 16y – 32 Equation of a Parabola given Focus and
Directrix
3. Find the equation of a parabola with vertex at 1. The equation of a parabola with focus (a, b)
(-2, -2) and focus at (-2, -8). and directrix y = c is
(x – a)2 + b2 – c2 = 2 (b – c) y
Solution
Vertex at (-2, -2) and focus at (-2, -8). 2. The equation of a parabola with focus (a, b)
and directrix x = c is
Distance between the vertex and focus
(y – b)2 = (x – c)2 – (x – a)2
| | = √( ) ( 8 )
| |=√ Worked Examples
| |=6 1. If the focus of a parabola is (2, 5) and the
directrix is y = 3, find the equation of the
The vertex and the focus are on the same parabola.
vertical line x = -2, with focus below the vertex.
Therefore, the parabola opens downward, so Solution
a = - 6. Its equation is given by: Focus = (2, 5) = (a, b) , ⇒ a = 2, b = 5
(x – h)2 = 4a(y – k) Directrix y = 3, ⇒ c = 3
The equation of a parabola with focus (a, b)
Vertex at (h, k) = (-2, -2) ⇒ h = -2, k = -2 and directrix y = c is given by :
By substitution, the equation of the parabola is (x – a)2 + b2 – c2 = 2 (b – c) y
(x – (-2)2 = 4 (-6) (y + 2)
By substitution;
(x + 2)2 = -24 (y + 2)
(x – 2)2 + 52 – 32 = 2 (5 – 3) y

Baffour Ba Series, Further Mathematics for Schools Page 439


x2 – 4x + 4 + 25 – 9 = 2(2) y at (- 4, - 2) and directrix given by the equation x
x2 – 4x + 20 = 2(2) y = - 8.
x2 – 4x + 20 = 4y
5. Write an equation for the parabola with focus
y= – +
at (0, -2) and directrix at x = 2
y= –x+5
6. Write the equation for a parabola with a
2. Find the equation of the parabola with focus focus at (6, - 4) and directrix at y = -7
at (8, 0) and directrix given by the equation x =
2. Determining the Focus and Directrix from a
the Equation of a Prabola
Solution Recall the standard form of the equation of a
Focus = (a, b) = (8, 0) ⇒ a = 8 and b = 0 parabola:
Directrix ( x = 2) ⇒ c = 2 1. y2 = 4ax with “vertex” at the origin, focus at
(a, 0) and axis of symmetry along the x-axis.
The equation of a parabola with focus (a, b)
and directrix x = c is 2. x2 = 4ay; vertex at (0, 0) with focus at (0, a)
(y – b)2 = (x – c)2 – (x – a)2
3. (y − k)2 = 4a(x − h), vertex at (h, k) with
By substitution, focus at (h + a, k)
The equation of a parabola with focus (a, b)  when a > 0, the parabola opens to the
and directrix x = c is right
(y – 0)2 = (x – 2)2 – (x – 8)2  when a < 0, the parabola opens to the
y2 = x2 – 4x + 4 – [x2 – 16x + 64] left.
y2 = x2 – 4x + 4 – x2 + 16x – 64
y2 = 12x – 60 4. (x − h)2 = 4a (y − k), vertex at (h, k) with
focus at (h, k + a)
Exercises 15.2  when a > 0, the parabola opens up.
1. Write an equation for a parabola with focus  when a < 0, the parabola opens down
(5, -2) and directrix y = - 5.
Worked Examples
2. Write an equation for a parabola with focus 1. Find the focus and the directrix of the
(-2, 5) and directrix y = 3. parabola y2 = 16x

3. Find the equation of the parabola with given Solution


focus (10, 1) and directrix x = 5. Let y2 = 16x………………..(1)
Compared to the general eqn of a parabola
4. Find the equation of the parabola with focus y2 = 4ax……………………(2)

Baffour Ba Series, Further Mathematics for Schools Page 440


4a = 16 Since a < 0, the parabola opens to the left.
a =4 Hence, the directrix is located 1 unit to the right
of the vertex. That is x = 0 + 1.
Focus = (a, 0) = (4, 0) Therefore, the directrix is x = 1

The directrix is represented as x = - p 4. Find the directrix of the parabola with the
Directrix, x = - 4 equation x2 – 4x + 16y – 76 = 0

2. Find the directrix of the parabola with the


Solution
equation (x – 5)2 = 20y
x2 – 4x + 16y – 76 = 0………..(1)
(x − h)2 = 4a (y − k)…………(2)
Solution
(x – 5)2 = 20y…………………(1)
Completing the square in eqn (1);
(x − h)2 = 4a (y − k)…………(2)
x2 – 4x = - 16y + 76
x2 – 4x + 4 = - 16y + 76 + 4
By comparison
x2 – 4x + 4 = - 16y + 80
Vertex (h, k) = (5, 0)
(x – 2)2 = - 16y + 80
20y = 4ay (x – 2)2 = - 16 (y – 5) ……….(1)
20 = 4a
(x − h)2 = 4a (y − k)…………(2)
a=5
By comparison;
Since a > 0, the parabola upens up.
Vertex = (2, 5)

The directrix will be located 5 units down from


4ay = - 16y
the vertex. That is y = 0 – 5.
4a = -16a
Therefore, the directrix is y = - 5
a=-4
3. Find the directrix of the parabola with the
Since a < 0, the parabola opens down. Hence,
equation (y – 3)2 = - 4x
the directrix will be located 4 units above the
vertex. That is y = 5 + 4. Therefore, the
Solution
equation for the directrix is y = 9
(y – 3)2 = - 4x……………..…..(1)
(y − k)2 = 4a (x − h)…………(2)
5. Find the focus of the parabola with the
By comparison, equation x2 + 10x – 4y + 17 = 0
Vertex = (0, 3)
Solution
4ax = - 4x x2 + 10x – 4y + 17 = 0………..(1)
a = -1 (x − h)2 = 4a (y − k)…………(2)

Baffour Ba Series, Further Mathematics for Schools Page 441


Completing the square in eqn (1); B. For each parabola, determine the co-
x2 + 10x = 4y – 17 ordinates of the vertex, the focus
x2 + 10x + 25 = 4y – 17 + 25 1. (y − 1)2 = 4(x − 2) 3. 2x = y2 + 4y + 6;
x2 + 10x + 25 = 4y + 8 2. (x + 1)2 = 8(y − 3); 4. x2 + 4x − 4y + 6 = 0
(x + 5)2 = 4 (y + 2)……………(1)
Equation of Tangent to a Parabola
2
(x − h) = 4a (y − k)…………(2) The equation of the tangent to a parabola
y2 = 4ax at the point ( , ) is given by:
By comparison; y = 2a( )
Vertex = (-5, -2)
Equation of Normal to a Parabola
4ay = 4y
The equation of the normal to a parabola
a=1
y2 = 4ax at the point ( , ) is given by:
Since a > 0, the parabola opens up. Hence, the x – 2a – + 2ay = 0
focus will be located 1 unit up from the vertex.
That is (-5, -2 + 1). Worked Examples
The focus is then located at (-5, -1) 1. A Parabola y2 = 12x passes through the point
P(3, 6). Find:
Exercises 15.3 i. equation of the tangent;
1. Find the directrix of the parabola with the ii. equation of the normal to the parabola at the
equation (x – 5)2 = 4(y + 1) point P.

2. Find the focus of the parabola with the Solution


equation (x – 9)2 = - 8 (y – 5) y2 = 12x P(3, 6)
Now; y2 = 12x compared to y2 = 4ax
3. What is the focus of the parabola with the 4a = 12
equation (x + 2)2 = - 12y a=3

4. Find the directrix of the parabola with the The general equation of a tangent is :
equation (y – 4)2 = - 8(x + 1) yy1 = 2a (x + x1)

5. Find the directrix of the parabola with the P(3, 6) =( )


equation y2 + 2y – 12x + 13 = 0 6y = 2(3) (x + 3)
6y = 6 (x + 3)
6. Find the vertex, the focus and the directrix of 6y = 6x + 18
the parabola defined by the equations. 6y – 6x – 18 = 0
i. 2y2 + 8y + x + 1 = 0 ii. x2 – 8x – y + 2 = 0 y–x–3=0

Baffour Ba Series, Further Mathematics for Schools Page 442


ii. Equation of the normal is given by; 3y2 = 36 – 12y
y– = ( ) 3y2 + 12y – 36 = 0
( , ) = (3, 6) and a = 3 y2 + 4y – 12 = 0
(y – 2) ( y + 6) = 0
y–6= ( )
( ) y = 2 or y = - 6
y–6= ( )
y–6= ( ) When y = 2 , x = = =
y–6=-x+3
K=. , /
y+x–6–3=0
x+y–9=0
( )
When y = - 6, x = = =6
2. The line 2y + 3x – 6 = 0 intersects the M=( , )
parabola y2 = 6x at K and M respectively. Find:
i. Find the coordinates of K and M; ii. equation of the normal to the parabola at the
ii. the equation of the normal to the parabola at
point . , /
the point . , /
y2 = 6x compared with y2 = 4ax
Solution 6 = 4a
i. Line 2y + 3x – 6 = 0 a= =
parabola y2 = 6x
Equation of the normal is given by;
Point of intersection = (K, M)
2y + 3x – 6 = 0………………..(1) y- = ( )
y2 = 6x………………………..…..(2) ( , ) = . , / and a =

y–2= . /
From eqn (1); . /
3x = 6 – 2y
y–2= . /
x= ……………………(3)
3(y – 2) = . /
From eqn (2); 3y – 6 = - 2x +
x= ………………………..(4) (3) 3y – (3) 6 = - (3) 2x + (3)
9y – 18 = - 6x + 4
eqn (3) = eqn (4); 9y + 6x – 18 – 4 = 0
= 6x + 9y – 22 = 0
3y2 = 6(6 – 2y )

Baffour Ba Series, Further Mathematics for Schools Page 443


16 SEQUENCE AND SERIES Baffour Ba Series

Sequence un = a + (n − 1)d
A sequence is a collection of numbers arranged un = 15 + (n − 1)(-3)
in a definite order, connected by a simple rule. un = 18 − 3n.
For example, perfect squares are listed as 1, 4, The n-th term is un = 18 − 3n.
9, 16, 25, 36…Other examples of sequences are
even numbers: 2, 4, 6,.. odd numbers: 1, 3, 5… 2. Determine the n-th term of the arithmetic
progression 8, 8.125, 8.25, 8.375, 8.5, ...
Series
When the numbers or terms of a sequence are Solution
added or considered as a sum, it is called a 8, 8.125, 8.25, 8.375, 8.5, ...
series. Thus we have a series of even number as a = 8 , n = ? d = 8.125 – 8 = 0.125
2 + 4 + 6 + 8… and the series of odd numbers un = a + (n − 1)d
as 1 + 3 + 5 + 7 … un = 8 + (n − 1)(0.125)
un = 7.875 + 0.125n.
A series may end after a definite number of The n-th term is un = 7.875 + 0.125n.
terms. Such a series is called a finite series. e.g.
2 + 4 + 6 + 8. On the other hand, a series may 3. The 13th term of an arithmetic progression is
not end and it is then called an infinite series. 10 and the 25th term is 20; calculate:
e.g. 2 + 4 + 6 + 8… a. the common difference;
b. the first term;
Arithmetic Progressions c. the 17th term.
The “sequence” of numbers, a, a + d, a + 2d,
a + 3d, ... is said to form an “arithmetic Solution
progression”. a. Letting a be the first term and d be the
common difference,
The symbol a represents the “first term”, the a + 12d = 10……………..(1)
symbol d represents the “common difference” a + 24d = 20……………..(2)
and the “n-th term” is given by the expression
eqn (2) – eqn (1);
Un = a + (n − 1)d
12d = 10,
Worked Examples d= =
1. Determine the n-th term of the arithmetic
progression 15, 12, 9, 6,... b. Put d = in eqn (1);

Solution a + 12. / = 10…………..(1)


15, 12, 9, 6,... a + 10 = 10
a = 15 , n = ? d = 12 – 15 = -3 a=0

Baffour Ba Series, Further Mathematics for Schools Page 444


c. un = a + (n − 1)d 4. Find the 17th term of the arithmetic
a = 0, d = and n = 17 progression with first term 5 and common
difference 2.
= 0 + (17 − 1) . /
= 16 . / 5. Write down the 10th and 19th terms of each
of the following A.P.
=
i. 8, 11, 14, . . ., ii. 8, 5, 2 . . ….,

Solved Past Question Finite Arithmetic Sequence


Three consecutive terms of a linear sequence It is an A.P. that comes to an end. In other
(A. P) are ,1 and , where m ≠ 0 and words, it an A.P., that has a given last term (L).
m ≠ -2. Find the value of m. Eg. 2, 4, 6, ..50

Solution For a finite A.P;


= L
, 1 and ,
⇒ a + (n – 1) d = L
Common difference, d;
By changing the subject to n, the number of
1– = -1 terms of an A.P, given the last term can be
( ) calculated.
=
= Worked Examples
= Determine the number of terms in the sequence;
5, 11, 17….77 and hence find the 30 the term.
(m – 1) (m + 2) = m (4 – m)
m2 + 2m – m – 2 = 4m – m2
Solution
m2 + m – 2 = 4m – m2
5, 11, 17….77
m2 + m2 + m – 4m – 2 = 0
a = 5, d = 6 , L = 77
2m2 – 3m – 2 = 0
=5+6(n–1)
(m – 2) (2m + 1) = 0
=5+6n–6
m = 2 or m = = 6n–1
⇒ 6 n – 1 = 77
Exercises 16.1
1. Write down the first five terms of the A.P. But =L
with first term 8 and common difference 7. 6n = 77 + 1
6n = 78
2. Write down the first five terms of the A.P.
n = 13 terms
with first term 2 and common difference - 5.
= 6 (30) – 1
3. What is the common difference of the A.P.
= 179
11, - 1, - 13, - 25, . . . ?

Baffour Ba Series, Further Mathematics for Schools Page 445


Finding the nth Term from Two given Terms 13d = 65
Method 1 d=5
1. Make use of the formula un = a + (n − 1)d
2. Substitute the respective values of the two Put d = 5 in eqn(1)
given terms separetly in un = a + (n − 1)d to a + 6(5) = 17
obtain two given equations. a = 17 – 30
3. Solve simultaneously to obtain the values of a = −13
a and d.
4. Substitute the values of a and d obtained in Put a = −13 and d = 5 in = a + (n − 1)d
(3) in un = a + (n − 1)d to arrive at the formula = -13 + (n − 1)(5)
for the nth term. = − 13 + 5n – 5
= 5n −18
Method 2
1. Most A.Ps follow the rule y = mx + c Method 2
2. Set the two given terms and their values as =17 and = 82.
(x1 , y1) and (x2, y2) , put in m = For the nth term of an A.P of two given tems,
Set y = mx + c
3. Substitute the values of m and (x1 , y1) in
(x1 = 7, y1 = 17) and (x2 = 20, y2 = 82) , put in
equation y = mx + c to obtain the value of the
intercept on y – axis. m=
4. Substitute the values of m and c in y = mx + c. m= –
5. Now, let = y and n = x. Then by
substitution, = mn + c, where m and c are m=
constants. m=5

Worked Examples Use the point (x1 = 7, y1 = 17) to find the y-


1. Find the common difference, first term intercept:
and nth term rule for the arithmetic sequence in 17 = 7 (5) + c
which =17 and = 82. 17 = 35 + c
17 – 35 = c
Solution c = - 18
= 17 and = 82.
= 17, ⇒ a + (7 −1) d =17 Put m = 5 and c = - 18 in y = mx + c
a + 6d = 17………………..(1) y = 5x – 18
= 82, ⇒ a + ( 20 − 1) d = 82 = 5x – 18
a + 19 d = 82……………..(2) When n = x, = 5n – 18

Solve the resulting system: 2. Find the common difference, first term
eqn (2) – eqn (1) and nth term rule for the arithmetic sequence in
19d – 6d = 82 – 17

Baffour Ba Series, Further Mathematics for Schools Page 446


which = −13 and = − 71. Hence find B. Use the two given terms to find
the value of an nth term rule for the sequence.
1. = - 31 and = - 46
Solution 2. = 25 and = 73
= - 13 and = -71. 3. =-2 and = - 14
=13, ⇒ a + (11 −1) d = -13 4. = 14 and = 23
a + 10d = -13…………….(1) 5. = 4n + 3 and = 4n + 65
= -71, ⇒ a + ( 40 − 1) d = - 71
a + 39 d = -71……………..(2) C. 1. Find the nth term rule for the arithmetic
sequence with = 13 and = 77..
eqn (2) – eqn (1);
39d – 10d = -71 – (-13) 2. Find the nth term rule for the arithmetic
29d = -58 sequence with = - 75 and = - 273.
d = -2
3.i. What is the nth term rule for the linear
Put d = -2 in eqn(1) sequence whose thirteenth term is - 20 and
a + 10(-2) = -13 thirtieth term is - 54?
a = -13 + 20 ii. Use the above information to find the value
a=7 of . /

Put a = 7 and d = -2 in = a + (n − 1)d 4. Find the nth term rule for the arithmetic
= 7 + (n − 1)(-2) sequence with = 23 and = 65. Hence
= 7 – 2n + 2 determine the value of the sum of and .
= – 2n + 7 + 2
= – 2n + 9 6. Find a fifth of the sum of and of the
arithmetic sequence with = - 3n + 7 and
ii. = – 2n + 9 = - 3n −16
= – 2(26) + 9
= – 52 + 9 7. Find the nth term and the nineth term for the
= – 43 arithmetic sequence whose sixth term is
−13 and fifteenth term is - 40.
Exercises 16.2
A. Use the two given terms to find Inserting Arithmetic Means
an nth term rule for the sequence. The arithmetic mean of two numbers a and b is
1. = 23 and = 85 defined as . This is also called the average
2. =-6 and =-3
of a and b.
3. = 24 and =9
4. = - 27 and = - 47
The three numbers a, , b form a
5. = 37 and = 85

Baffour Ba Series, Further Mathematics for Schools Page 447


finite arithmetic sequence, with a common Sum of Arithmetic Series
difference d = (b – a) If the terms of an arithmetic progression are
added together, an “arithmetic series” is
In general, if , … are real numbers obtained .
between a and b, such that a, , ... , b is a The total sum of the first n terms of such a
finite arithmetic sequence, then , … are n series can be denoted by Sn such that
arithmetic means beteween a and b. Sn = a + [a + d] + [a + 2d] + ... + [a + (n − 2)d]
+ [a + (n − 1)d]…………..(1)
The process of of determing the numbers ,
But this is not the most practical way of
….. is called inserting n arithmetic means
evaluating the sum of the n terms, especially
between a and b.
when n is a very large number.

Worked Example A more convenient formula for Sn is to write


1.Insert three arithmetic means between 2 and 9.
down the existing formula backwards. That is,
Sn = [a + (n − 1)d] + [a + (n − 2)d] + ... + [a +
Solution
2d] + [a + d] + a……………….(2)
Let the three arithmetic means between 2 and 9
be , and such that the arithmetic (finite)
Adding the two equations gives
sequence is: 2, , , , 9
2Sn = [2a + (n − 1)d] + [2a + (n − 1)d] + ... +
[2a + (n − 1)d] + [2a + (n − 1)d],
From the sequence, first term a = 2 and =9
where, the R.H.S, are n repetitions of the same
= a + (n – 1) d
expression. Hence,
9 = 2 + (5 – 1) d
2Sn = n[2a + (n − 1)d]
9 = 2 + 4d
Sn = [2a + (n − 1)d]..……………(a)
9 – 2 = 4d
7 = 4d
This version of the formula is suitable if the
d=
values of a, n and d are known;
An alternative version can also be used if only
=a+d=2+ = the first term, the last term and the number of
= +d= + = terms are known. In this case,
Sn = [FIRST + LAST]
= +d= + =
Sn = [a + l ]…………………….(b)
Exercises 16.3 which is simply n times the average of the first
1.Insert three arithmetic means between 2 and 10. and last terms.

2. Insert 3 arithmetic means between 3 and -5. Comparing equations (a) and (b);
l = a + (n − 1)d

Baffour Ba Series, Further Mathematics for Schools Page 448


Worked Examples ( )
=
1. Determine the sum of the natural numbers
40 = n − 1
from 1 to 100.
n = 41.

Solution
Sn = (a + ℓ)
a = 1 and l = 100 (last term), n = 100
The sum is given by : S41 = (1 + 101)
Sn = [FIRST + LAST] S41 = (102)
Sn = [1 + 100] S41 = 41 × 51
Sn = 5050. S41 = 2091 .

2. Find the sum of all odd numbers between 4. Find the sum of the first 50 terms of the
100 and 200? sequence 1, 3, 5, 7, 9, . . . .

Solution
Solution
1, 3, 5, 7, 9, . . . .
101,103,105...........199
a = 1 , d = 2 , n = 50 .
a = 101 d = 2 = l = 199
= a + (n – 1)d Sn = [2a + (n − 1)d]
199 = 101 + (n – 1) 2 S50 = [2(1) + (50 − 1)2]
199 – 101 = (n – 1) 2 S50 = 25 × (2 + 49 × 2)
=n–1 S50 = 25 × (2 + 98)
49 + 1 = n S50 = 2500 .
n = 50
5. Find the sum of the series + 3 + + …+
= (a + l)
Solution
= (101 + 199)
+ 3 + + …+
= 25 × 300
= 7,500 a= ,d=3- = ,l= .

3. Find the sum of the series But l = a + (n – 1) d


1 + 3.5 + 6 + 8.5 + . . . + 101 . = + (n – 1)
27 = 3 + 3(n – 1)
Solution
27 = 3 + 3n – 3
d = 2.5, a = 1, l = 101 and n = ?
27 = 3n
ℓ = a + (n − 1)d
n=9
101 = 1 + (n − 1) × 2.5 (By substitution)
100 = (n − 1) × 2.5 The sum of the nth term of the A.P :

Baffour Ba Series, Further Mathematics for Schools Page 449


Sn = (a + l ) a + 2(2) = 7
a+4=7
Substitute a = , d = , l = and n = 9.
a=7–4
S9 = . / a=3
i. the commom difference is 2
S9 = . /
ii. the first term is 3
S9 =
S9 = 67.5 7. An arithmetic series has the property that the
sum of the frist ten terms is half the sum of the
6. The sum of the first twelve terms of an A.P. next ten terms. Also, its 100th term is 95. Find
is 168. If the third term is 7, find the value of : the first term and the common difference.
i. the common difference;
ii. the first term. Solution
= [first + last]
Solution Sum of first ten terms;
Sn = [2a + (n − 1)d]
= [a + (a + 9d)]
= [2a + (12 − 1)d] = 5 [a + a + 9d]
= 6[2a + 11d] = 5 [2a + 9d]

But = 168 Sum of next ten terms;


6[2a + 11d] = 168 = [(a + 10d) + (a +19d)]
12a + 66d = 168
= 5 [a + 10d + a +19d]
2a + 11d = 28……………………(1)
= 5 [2a + 29d]

= (a + (3 − 1)d
Sum of first ten terms = (Sum of next ten terms;)
= a + 2d
But =7 5 [2a + 9d] = ( , - )
a + 2d = 7 ………………………..(2) (2a + 9d) = ( )
2(2a + 9d) = ( )
eqn (2) × 2;
4a + 18d = 2a + 29d
2a + 4d = 14……………………..(3)
4a – 2a = 29d – 18d
2a = 11d
eqn (2) – eqn (3);
a= d………………………………..(1)
11d – 4d = 28 – 14
8d = 16
d=2 Also, 100th term is 95
= a + 99d
Put d = 2 in eqn (2) But = 95

Baffour Ba Series, Further Mathematics for Schools Page 450


⇒ 95 = a + 99d ……………………..(2) i. calculate the value of x;
ii. find the first term and the common
Put a = d in eqn (2) difference;
iii. find the sum of the first 18 terms of the
95 = d + 99d
sequence.
2 × 95 = 2 × d + 2 × 99d
190 = 11d + 198d 2. The following is an arithmetic sequence:
190 = 209d 1 – p , 2p – 3, p + 5…
i. Calculate the value of p
d=
ii. Write down the value of the first tern and the
d= common difference.
iii. Determine the sum of the first 32 terms of
Put d = in eqn (1); the sequence.

a= × 3. The following arithmetic sequence is given:


a=5 20, 23, 26, 29, …, 101
First term is 5 and the common difference is i. How many terms are there in the sequence?
ii. If the even numbers are removed from the
Exercises 16.4 sequence, calculate the sum of the terms of the
1. Find the sum of the first 23 terms of the A.P. remaining sequence.
4, - 3, - 10, . . ..
4. Consider the arithmetic sequence:
2. An arithmetic series has first term 4 and - 8, - 2, 4, 10…
common difference 1. Find: i. Write down the next four terms of the
i. the sum of the first 20 terms, sequence
ii. the sum of the first 100 terms. ii. If the nth term of the sequence is 148,
determine the value of n.
3. Find the sum of the arithmetic series with iii. Calculate the smallest value of n for which
first term 1, common difference 3, and last the sum of the first n terms of the sequence will
term 100. be greater than 10140.

4. The sum of the first 20 terms of an arithmetic 5. Given the arithmetic sequence:
series is identical to the sum of the first 22 y – 3, 2y – 4 and 23 – y
terms. If the common difference is - 2, find the i. Determine the value of y.
first term. ii. Find the sum of the first 26 terms of the
sequence.
Exercises
1. 3x + 1, 2x and 3x – 7 are the first three terms 6. The first three terms of an arithmetic
of an arithmetic sequence. sequence are 2k – 7, k + 8 and 2k – 1.

Baffour Ba Series, Further Mathematics for Schools Page 451


i. Calculate the value of the 15th term of the Multiply through by 2;
sequence. 504 = 16n + 5n2 – 5n
ii. Calculate the sum of the first 30 even terms 5n2 + 11n = 504
of the sequence 5n2 +11n – 504 = 0
(n – 9) ( 5n + 56) = 0
Finding the Number of Terms or the Common n = 9 or n = (Ignore negative answer)
Difference Given the Sum of Terms
n=9
1. How many terms of the arithmetic series
10 + 12 + 14 + ... must be taken so that the sum
3. An arithmetic progression has 3 as its first
of the series is 252 ?
term. Also, the sum of the first 8 terms is twice
Solution the sum of the first 5 terms. Find the common
a = 10 and d = 2, Sn = 252, n = ? difference.
Sn = [2a + (n − 1) d].
Solution
252 = [20 + (n − 1) × 2]. a = 3, S8 = 2S5, d = ?
504 = n [20 + 2n − 2] Sn = [2a + (n − 1)d]
504 = n [18 + 2n ]
S8 = [2(3) + (8 − 1)d]
504 = 18n + 2n2
n2 + 18n − 504 = 0 S8 = 4 (6 + 7d)
n2 + 9n − 252 = 0 S8 = 24 + 28d
(n – 12) ( n + 21) = 0
n = 12 or n = - 21 (Ignore negative answer) S5 = [2(3) + (5 − 1)d]
n = 12 S5 = (6 + 4d)
S5 = 15 + 10d
2. Find the number of terms of the arithmetic
series 4 + 6 + 9 + 11 ... needed to make a
Given the fact that S8 = 2S5,
total of 126. 24 + 28d = 2(15 + 10d )
24 + 28d = 30 + 20d
Solution 28d – 20d = 30 – 24
a = 4 and d = 6 - 4 = , Sn = 126, n = ? 8d = 6
Sn = [2a + (n − 1) d]. d= =

126 = [2(4) + (n − 1) ]. The common difference is

126 = [8 + (n − 1)].
4. The first and third terms of a linear sequence
252 = n [ 8 + n − ]. (A. P.) are 25 and 19 respectively. If the sum of
252 = 8n + n2 − n ] all the terms in the sequence is 82, calculate the
number of terms in the sequence.

Baffour Ba Series, Further Mathematics for Schools Page 452


Solution Solution
un = a + (n − 1) d. i. un = a + (n − 1) d.
= a + (1 − 1)d = (a + (1 − 1)d
=a =a
But = 25
⇒ a = 25………………………(1) + = 42
⇒ a + a + (n − 1) d = 42
= (a + (3 − 1)d 2a + (n − 1) d = 42……………….(1)
= a + 2d
But = 19 = 420
⇒ a + 2d = 19………………(2) , ( ) - = 420…………(2)

Put a = 25 in eqn (2) Put eqn (1) into eqn (2);


25 + 2d = 19 , - = 420
2d = 19 – 25 42n = 2 × 420
2d = - 6 42n = 840
d = -3 n = 20
The number of terms is 20.
a = 25 and d = -3, Sn = 82, n = ?
Sn = [2a + (n − 1) d]. ii. The common difference;
82 = [50 + (n − 1) × (-3)] Put n = 20 in eqn (1);
164 = n [50 - 3n + 3] 2a + (20 − 1) d = 42
164 = n (53 - 3n) 2a + 19 d = 42…………………(iii)
164 = 53n – 3n2
3n2 – 53n + 164 = 0 The second term is 4;
(n – 4) (3n – 41) = 0 =4
n = 4 or 3n – 41 a + d = 4……………………….(iv)
n = 4 or 3n = 41
n = 4 or n = (Ignore n = ) Solving eqn (iii) and (iv);
2 × eqn (iv)
n=4
2a + 2d = 8……………………..(v)
5. The sum of the first and last terms of a linear eqn (iii) – eqn (v);
sequence (A. P.) is 42. If the sum of all the
17d = 34
terms is 420 and the second term is 4, find: d = 2.
i. number of terms; The common difference is 2
ii. common difference;
iii. last term of the sequenece. iii. The last term of the sequence;

Baffour Ba Series, Further Mathematics for Schools Page 453


Put d = 2 in eqn (iv); a = 17 – 20
a+2=4 a = -3
a=4–2
a=2 = , ( ) -
= , ( ) ( ) -
Put a = 2 , d = 2 and n = 20
in un = a + (n − 1) d = 10 , ( )-
= 10, -
= 2 + (20 − 1) 2
= 700
= 2 + 19 (2)
= 2 + 38 2. The thirteenth term of an A.P. is 27 and the
seventh term is three times the second. Find:
= 40
i. the first term;
The last term is 40.
ii. the common difference;
iii. the sum of the first ten terms.
Sum of A.P. from Two Given Terms
1. Form two equation from the two given terms
Solution
using the equation = a + (n − 1)d
= a + (n − 1)d
2. Solve the two equations simultaneously to
= a + (13 − 1)d
obtain the values of a and d.
3. Substitute the values of a and d obtained in
But = 27
Sn = [a + (n − 1) d]
⇒a + (13 − 1)d = 27
a + 12d = 27…………………..(1)
Worked Examples
1. The 6th and 12th terms of a linear sequence The seventh term is three times the second
(A.P.) are 17 and 42 respectively find the sum =3
of the first 20 terms. a + (7 − 1)d = 3[ a + (2 − 1)d ]
a + 6d = 3( a + d )
Solution
a + 6d = 3a + 3d
= 17
6d – 3d = 3a – a
a + 5d = 17………………………..(1)
3d = 2a
= 41
a + 11d = 41……………………….(2) a= ………………………..(2)

eqn (2) – eqn (1); Put a = in eqn (1);


6d = 24
+ 12d = 27
d=4
3d + 24d = 54
Put d = 4 in eqn (1) 27d = 54
a + 5(4) = 17 d=2

Baffour Ba Series, Further Mathematics for Schools Page 454


Put d = 2 in eqn (2); ii. If the sum of the first n terms of the sequence
a=
( ) is – 560. Calculate n.
a = 3,
Word Problems Involving A.P.
i. The first term, a = 3, i. Carefully read the problem and identify the
given values and the unknown.
ii. the common difference , d = 2 ii. Use the appropriate formula whether
un = a + (n − 1)d or Sn = [2a + (n − 1)d]
iii. Put a = 3, d = 2 and n = 10 in
Sn = [2a + (n − 1)d] Worked Examples
1. Assuming a student is paying off a student
S10 = [2(3) + (10 − 1)2]
loan in monthly installments. After the fifth
S10 = 5 (6 + 18)
payment the remaining balance on the loan is
S10 = 5 (24)
Ghȼ17,500.00. After the 16th payment, the
S10 = 120
remaining balance is Ghȼ12,000.00
i. What is the nth term rule for the sequence
Exercises 16.5
represented by this situation?
A. 1. The third term of an arithmetic series is
ii. What is the outstanding balance after the 20th
34 and the 17th term is - 8. Find the sum of the
payment?
first 20 terms.
Solution
2. Find the common differencse of an A.P.
= 17,500 and = 12,000 =?
whose first term is 5 and the sum of the first
four terms is half the sum of the next four For the nth term of an A.P of two given tems,
terms. Set y = mx + c
(x1 = 5, y1 = 17,500) and (x2 = 16, y2 = 12,000)
3. The sum of the first 13 terms of an A.P. is 21
, put in
and the sum of the first 21 terms is 13. Show
m=
that the sum of the first 34 terms is -34
, ,
m= –
= = − 500
4. An A.P. is given by k, , , 0, . . ..
i. Find the sixth term. Use the point (x1 = 5, y1 = 17,500) to find
ii. Find the n th term. the y-intercept:
iii. If the 20th term is equal to 15, find k. 17,500 = - 500 (5) + c
17,500 = − 2,500 + c
5. The first and second terms of an arithmetic 17,500 + 2,500 = c
sequence are 10 and 6 respectively. c = 20,000
i. Calculate the 11th term of the sequence;
Put m = − 500 and c = 20,000 in y = mx + c

Baffour Ba Series, Further Mathematics for Schools Page 455


y = − 500 x +20,000 l = [a + (n − 1) d].
= − 500 n + 20,000. 1760 = [1160 + (n − 1) 40].
1760 = [1160 + 40n − 40].
ii. outstanding balance after the 20th payment
1760 = [1120 + 40n ].
= − 500 n + 20,000.
= − 500 (20) + 20,000. 1760 – 1120 = 40n
= − 10,000 + 20,000. 640 = 40n
= 10,000 n= = 16 terms

2. A contractor agrees to sink a well 250 metres


Sn = [a + l ]
deep at a cost of Ghȼ2.70 for the first metre,
Ghȼ2.85 for the second metre and an extra 15p S16 = ( )
for each additional metre. Find the cost of the S16 = 8(2920)
last metre and the total cost. S16 = 23,360
The total amount invested is Ghȼ23,360.00
Solution
a = 2.70 , d = 0.15, n = 250 Exercises 16.6
The cost of the last metre is the 250-th term of 1. A contest will have 5 cash prizes totalling
the series and therefore Ghȼ5,000.00 and there will be a Ghȼ100.00
un = [a + (n − 1) d]. difference between successive prizes. Find the
U250 = [2.70 + (250 – 1) × 0.15] first prize.

U250 = [2.70 + (249) × 0.15 2. A company is to distribute Ghȼ46,000.00 as


= Ghȼ40.05 bonuses to its top ten sales people. The tenth
sales person on the list will receive
Now, a = 2.70, n = 250, l = 40.05, Sn = ? Ghȼ1,000.00, and the difference in bonus
The total cost ; money between successfully ranked sales
Sn = [a + l ] people is to be constant. Find the bonus for
Sn = ( ) each sales person.

Sn = Ghȼ5,343.75
3. A body falls 8m in the first second of its
motion, 24 meters in the second, 40 m in the
3. A man started a project with Ghȼ1,160.00. If
third and so on. Howlong will it take to fall
he increases this value with Ghȼ450.00 every
2048 meters?
month until he last invested Ghȼ1,7760.00,
calculate the total amount he invested in the
4. A factory produced 1,100 cars in the third
project at the time he invested Ghȼ1,760.00
year and 2,700 cars in the eleventh year.
Assuming production increases by a fixed
Solution
number every year, find the production of cars
a = 1,160, d = 40 , l = 1,740

Baffour Ba Series, Further Mathematics for Schools Page 456


in the 20th year and the total cars produced in to 49. Sammy has a house on the same street.
the first 20 years? His house is situated such that the sum of the
numbers of the houses preceeding his house is
5. In a flower bed, there are 31 rose plants in equal to the sum of the numbers of the houses
the first row, 28 in the second, 25 in the third following it. What is sammy‟s house number?
and so on. There are 7 rose plants in the last
row. How many rows are there in the flower Geometric Progressions
bed? The sequence of numbers
a, ar, ar2, ar3, ... is said to form a “geometric
6. A manufacturer of radio sets produced 600 progression”.
units in the third year and 700 units in the 7th
year. Assuming that the production uniformly The symbol a represents the “first term”, the
increases by a fixed number every year. Find : symbol r represents the “common ratio” and the
i. the production in the first year; “n-th term” is given by the expression:
ii. the total production in 7 years; = arn−1.
c. the production in the 10th year.
Worked Examples
7. Ataa Afua repays her total loan of
1. Determine the n-th term of the geometric
Ghȼ118,000.00 by paying every month starting
progression 3, - 12, 48, - 192, ...
with the first instalment of Ghȼ1,000.00 If she
increased the instalment by Ghȼ100.00 every
Solution
month,
a = 3, r = = - 4, n = 7
i. what amount will be paid by her in the 30th
n−1
instalment? = ar
ii. what amount of loan does she have to pay = 3(- 4)n−1
after the 30th instalment? The progression has n-th term, = 3(- 4)n−1

8. A sum of Ghȼ700.00 is tob used to give 7 2. Determine the seventh term of the geometric
cash prizes to students of a school for their progression 3, 6, 12, 24, ...
overall academic performance. If each price is
Ghȼ20.00 less than its preceeding prize, fins the Solution
value of each of the prizes. a = 3, r = = 2, n = 7
= arn−1
9. A woman takes up a job of Ghȼ8,000.00 per
= 3( )
month with an annual increment of Ghȼ100.00 6
= 3(2 )
What will she earn over a period of 10 years?
= 192.
10. The houses on Bantama high street are The seventh term is 192.
made in such a way that they all lie in one row.
The houses are numbered consecutively from 1 3. The third term of a geometric progression is

Baffour Ba Series, Further Mathematics for Schools Page 457


4.5 and the nineth term is 16.2. Determine the II. Write two (or more) equations, each for the
common ratio. given terms.
III. Solve the equations simultaneously to
Solution determine the respective values of a and r.
ar2 = 4.5 and ar8 = 16.2
Dividing the second of these by the first gives Worked Examples
1. The fifth and ninth terms of an exponential
=
sequence are 16 and 256 respectively. Find:
i. the first term and the common ratio ,
Therefore, ii. the seventh term.
r6 = 3.6
r = 1.238 Solution
i. U5 =
Exercises 16.7 U5 =
1 Write down the first five terms of the But U5 = 16
geometric progression which has first term 1 ⇒ 16 = ………………..(1)
and common ratio 1.
U9 =
2. Find the 10th and 20th terms of the G.P. with U9 =
first term 3 and common ratio 2. But U9 = 256
3. Find the 7th term of the G.P. 2, - 6, 18, . . ., ⇒ 256 = ………………..(2)

4. Find the 12th term of the geometric sequence eqn (2) ÷ eqn (1);
-1, 2, - 4, 8… =
5. Find the 7th term of the geometric sequence 16 =
2, , , … r= √
r=2
6. The first term of a G. P. is 2 and the
commom ratio is 3. Find the 5th term. Put r = 2 in eqn (1);
7. The first term of a G. P. is 16 and the 16 = ( )
commom ratio is . Find the 4th term. 16 = 16a
a=
Finding a and r of a G.P. from Two given a=1
Terms The first term is 1 and the common ratio is 2
Given the values of two terms of a G. P. the
first term, a, and the common ratio, r, can be b. The seventh term,
calculated as follows: U7 =
I. Identify the values of the given terms. U7 =

Baffour Ba Series, Further Mathematics for Schools Page 458


But a = 1 and r = 2 ii. The eighth term U8 =
U7 = ( ) ( ) But a = 1 and r =
U7 =
U7 = U8 = . /

U8 = . /
2. The third and sixth terms of an exponential
sequence are and respectively. Find: U8 =

i. the first term and the common ratio,


ii. the 8th term of the sequence. 3. The second and fourth terms of an
exponential sequence of positive terms are 9
Solution and 4 respectively. Find:
Un = i. the common ratio,
U3 = ii. the first term.
U3 = ar2
Solution
But U3 =
i. Un =
⇒ = ar2……….(1) U2 =
U2 = ar
U6 = But U2 = 9
U6 = ar5 ⇒ 9 = ar ……….(1)
But U6 =
U4 =
⇒ = ar5……….(2)
U4 = ar3
But U4 = 4
eqn (2) ÷ eqn (1); ⇒4= ……….(2)
÷ =
eqn (1) ÷ eqn (2);
=
=
. / =
=
⇒r =
r=√ =
Put r = into eqn (1);
ii. Put r = into eqn (1);
= a. /
9= (a)
= a
4a = 4 9 × 3 = 2a
a=1 a= = 13.5
The first term is 1 and the common ratio is Therefore the first term is 13.5

Baffour Ba Series, Further Mathematics for Schools Page 459


Exercises 16.7B A convenient formula for Sn is to write down
1. The seventh and tenth terms of an both Sn and rSn, the latter giving
exponential sequence of positive terms are 64 rSn = ar + ar2 + ar3 + ar4 + ... + arn−1 + arn….(2)
and 512 respectively. Find:
i. the common ratio, eqn (1) – eqn (2);
ii. the first term; Sn − rSn = a − arn, so that
iii. Write the first 5 terms of the sequence. ( )
Sn =

2. The sixth term of a G.P is 25 and the eighth This version of the formula is most commonly
term is 6.25. Write the first five terms of the used when r < 1 .
geometric sequence.
The alternative version of this formula is :
3. Find the 8th term of a geometric sequence ( )
Sn = and it is most commonly used
whose 3rd term is and whose 6th term is
when r > 1.

4. Consider the geometric sequence: 4, -2, 1 … Note:


i. Determine the next three terms of the Either version of the formula may be used with
sequence. disregard to the value of r.
ii. Determine n, if the nth term is
Worked Examples
5. A geometric sequence has = 20 and = 1. Find the sum of the geometric series
40. Determine: 2 + 6 + 18 + 54 + . . . where there are 6 terms in
i. the common ratio and the first term. the series.
ii. a formula for
Solution
a = 2, r = 3 and n = 6.
Geometric Series ( )
If the terms of a geometric progression are Sn =
added together, a “geometric series” is ( )
Sn =
obtained. ( )
Sn =
The total sum of a geometric series with n Sn = − (-728)
terms may be denoted by Sn such that Sn = 728 .
Sn = a + ar + ar2 + ar3 + ... + arn−1…..(1)
2. Find the sum of the geometric series
Like the arithmetic series, this is not the most 8 − 4 + 2 − 1 + . . . where there are 5 terms in
practical formula for evaluating Sn. the series.

Baffour Ba Series, Further Mathematics for Schools Page 460


Solution The common ratio is 3
a = 8, r = and n = 5.
( )
ii. Put r = 3 in eqn (1)
Sn = 18 = ( )
( . / * ( . /* . / 18 = 9a
S5 = = ⁄
= ⁄
= a=2
. /
Sum of first n terms is given by;
( )
3. Determine the sum of the geometric series Sn =
4+2+1+ + + +…
But r = 3 , a = 2 and n = 10
( )
Solution S10 =
The sum is given by ( ) ( )
S10 = = = 59,048
( . / * ( )
S6 = = = 7.875
Exercises 16.8
1. Find the sum of the first four terms of the
4. The 3rd term of a geometric series with a
geometric sequence that begins with 1 and a
positive common ratio is 18 and the fifth term
common ratio of 3.
is 162. Find:
i. the common ratio;
ii. the sum of the first ten terms. Geometric Means
If A and C are positive real numbers, then a
positive number B is called the geometric mean
Solution
of A and C if ABC is a geometric sequence.
= 18 and = 162
=
If the commom ratio is r, then :
=
= r= =
18 = ……………………..(1) ⇒ B2 = AC
Taking the square root of both sides, the
= geometric mean of the positive number A and C
= is √ .
162 = ……………………..(2)
In general, k real numbers B1, B2 …Bk are k
eqn (2) divided by eqn (1); geometric means between A and C, if A, B1, B2
= …Bk, C is a geometric series.

9 = r2
Worked Examples
r=√ 1. If 20, A and 45 form a geometric sequence,
r=3 find the value of A.

Baffour Ba Series, Further Mathematics for Schools Page 461


Solution needed so that the sum of the first n terms is
20, A and 45 greater than 20?
r= =
Solution
A2 = 20 × 45
The sequence is a geometric progression with
A=√
a = 1 and r = 1.1.
A = 30
To find the smallest value of n such that
The geometric sequence is 20, 30 and 45.
Sn > 20. Now;
( )
Exercises 16.9 Sn =
1. Find the geometric mean of 12 and 48. ( )
> 20
2. Find the geometric mean of 20 and 25. ( )
> 20
3. Insert two geometric means between 4 and 1 – 1.1n < -2
500. – 1.1n < - 3
4. Insert three geometric means between 2 and 1.1n > 3
lg 1.1n > lg3 (take logarithms of both sides)
512.
n lg 1.1 = lg 3
n=
Finding the Number of Terms of G. P.
For all finite geometric progressions; n = 11.5267 . . .
Un = arn−1 = L The smallest whole number value of n is 12.
Where L is the last term of the sequence.
3. What is the smallers number of terms of the
Worked Examples geometrical progression, 8 + 24 + 72 + …, that
1. How many terms are there in the geometric will give a total greater vthan 6,000,000?
progression 2, 4, 8, . . . , 128 ?
Solution
Solution 8 + 24 + 72 + …,
a = 2 and r = 2, Un = 128. a = 8 and r = 24 ÷ 8 = 3
Un = arn−1
128 = 2 × 2n −1 To find the smallest value of n such that
64 = 2n −1 Sn > 6,000,000.
26 = 2n −1 Sn =
( )

6=n−1 ( )
n=7. > 6,000,000
( )
There are 7 terms in this geometric progression. > 6,000,000
4( ) > 6,000,000
2. How many terms in the geometric , ,
progression 1, 1.1, 1.21, 1.331, . . . will be 3n – 1 >

Baffour Ba Series, Further Mathematics for Schools Page 462


3n – 1 > 1,500,000 Solution
3n > 1,500,000 + 1 In an AP;
3n > 1,500,001 = a, =a+d, = a + 4d
But a = 7
lg 3n = lg 1,500,001 (take log of both sides)
= 7, = 7 + d, = 7 + 4d
n lg 3 = lg 1,500,000
, , The terms of the G. P are:
n= = 12.94 (4 significant figures)
7, 7 + d , 7 + 4d
The smallest number of terms required is 13. ⇒ = (for common ratio)
(7 + d) (7 + d) = 7 (7 + 4d)
Exercise 16.10
49 + 7d + 7d + d2 = 49 + 28d
1. Find the sum of the first five terms of the
49 + 14d +d2 = 49 + 28d
G.P. with first term 3 and common ratio 2.
d2 + 14d – 28d = 0
d2 – 14d = 0
2. Find the sum of the first 20 terms of the G.P.
d(d – 14) = 0
with first term 3 and common ratio 1.5.
d = 0 or d – 14 = 0
3. How many terms in the G. P; 4, 3.6, 3.24,... d = 0 or d = 14 (ignore d = 0)
are needed so that the sum exceeds 35? d = 14

4. The sum of the first 3 terms of a geometric 2. The 2nd , 4th and 8th terms of an arithmetric
sequence (A. P.) form three consecutive terms
series is . The sum of the first six terms is
of a geometric sequence.The sum of the third
. Find the first term and common ratio. and fifth terms of the A.P. is 20. Find:
i. the first four terms;
5. The first term of a G. P is 3 and the eight ii. the sum of the first ten terms of the
term is 384. Find the common ratio, the sum arithmetic sequence.
and the product of the first 8 terms.
Solution
6. Calculate the product of the first five terms = a + (n – 1) d
of the sequence 3, 6, 12, … =a+d, = a + 2d, = a + 4d and

A.P. combined with G.P. = a + 7d


Worked Examples
1. The first, second and fifth terms of a linear But + = 20
sequence (A.P.) are three consecutive terms of ⇒ a + 2d + a + 4d = 20
an exponential sequence (G.P.). If the first term 2a + 6d = 20
of the linear sequence is 7, find its common a + 3d = 10 …………………(1)
difference.

Baffour Ba Series, Further Mathematics for Schools Page 463


Common ratio of G.P. ; =5. /
r= =
=
r= =
(a + 3d) (a + 3d) = (a + 7d) ( a + d) 3. The 1st, 5th and 10th terms of an linear
(a + 3d)2 = (a + 7d) ( a + d) sequence are in geometric sequence. If the sum
a2 + 6ad + 9d2 = a2 + ad + 7ad + 7d2 of the 2nd and 8th terms of the linear sequence
a2 + 6ad + 9d2 = a2 + 8ad + 7d2 is 30. Find the:
9d2 – 7d2 = 8ad – 6ad i. first term;
2d2 = 2ad ii. non – zero common difference of the linear
d2 = ad sequence.
d = a …………………………..(2)
Solution
Put d = a in eqn (1) = a + (n – 1) d
d + 3d = 10 =a, = a + 4d, = a + 9d
4d = 10
d= For a G.P, common ratio;
r= =
d=
r= =
⇒a=
= a, = a + d, = a + 2d and
(a + 4d) (a + 4d) = a ( a + 9d)
= a + 3d
(a + 4d)2 = a ( a + 9d)
a2 + 8ad + 16d2 = a2 + 9ad
= , 16d2 = 9ad – 8ad
= + =5 16d2 = ad
16d = a…………………………..(1)
= + 2. / =
= + 3. / = 10 Sum of 2nd and 8th term trems of the AP is 30
+ = 30
The first four terms of the A.P. are: , 5, , 10
⇒ (a + d) + (a + 7d ) = 30
a + d + a + 7d = 30
ii. = , ( ) - 2a + 8d = 30
n = 10, a = , and d = , a + 4d = 15 …………………(2)

= 0 . / ( ) 1 Put 16d = a in eqn (2)


=50 ( ) 1 2(16d) + 8d = 30
32d + 8d = 30
=5. / 40d = 30

Baffour Ba Series, Further Mathematics for Schools Page 464


d= 2. Mr. Oppong makes monthly payments into
his savings account. He saved Ghȼ800.00 in the
d=
second month, Ghȼ3,200.00 in the third month
and he continued in tha same proportion in
Put d = in eqn (2) subsequent months. In what month would his
savings first exceed Ghȼ20,000.00?
16. / = a
4×3=a Solution
a = 12 = 200, = 800 and = 3,200
The first term, a = 12 and the common
r= = =4
difference d =
The sequence is a G.P;
( ) ( ) ( )
Sn = = =
Word Problems Involving G. P
I. Carefully read the problem and identify the
given values and the unknown. For the savings to exceed Ghȼ20,000.00;
II. Use the appropriate formula whether ( )
> 20,000
( )
= or Sn = 200 (1 – 4n) > - 60,000
,
(1 – 4n) > -
Worked Examples
1 – 4n > - 300
1. The expenses of a company are
1 + 00 = 4n
Ghȼ200,000.00 a year. It is decided that each
301 = 4n
year they shall be reduced by 5% of those for
lg 301 > lg 4n
the preceding year. What will be the expenses
lg 301 > n lg 4
during the fourth year, the first reduction taking
place at the end of the first year ? n>
n > 4.1168
Solution n=5
Geometric progression with a = 200,000 and His saving would first exceed Ghȼ20,000.00 in
r = 0.95. the fifth month.
=
Exercises 16.11
The expenses during the fourth year will thus 1. Every year, a gardner takes 4 runners from
be the fourth term of the progression; each of his one year old strawberry plants in
= 200,000 ×(0.95)4 – 1 order to form 4 additional plants. If he starts
= 200,000 ×(0.95)3 with 5 plants, how many new plants will he
= Ghȼ171,475.00 take at the end of the 6th year and what will
then be his total number of plants ?

Baffour Ba Series, Further Mathematics for Schools Page 465


2. A parent places in a savings bank Ghȼ25.00 The Sum to Infinity of a Geometric Series.
on his son‟s first birthday, Ghȼ50.00 on his In a geometric series with n terms, suppose that
second, Ghȼ75.00 on his third and so on, the value of the common ratio, r, is numerically
increasing the amount by Ghȼ25 on each less than 1. Then the higher the value of n, the
birthday. How much will be saved up (apart smaller the numerical value of rn, to the extent
from any accrued interest) when the boy that, as n approaches infinity, rn approaches
reaches his 16th birthday if the final amount is zero.
added on this day ? Ans Ghȼ3400
To conclude, although it not possible to reach
3. The yearly depreciation of a machine is 25% the end of a geometric series which has an
of its value at the ebgining of the year. If the infinite number of terms, its sum to infinity
original cost of thee machine is Ghȼ20,000.00, may be given by: =
what is its value after 6 years?
Worked Examples
4. A truck transports 600kg of vegetables to the
1. Determine the sum to infinity of the
market on monday. From Monday to Sunday,
the amount of vegatables the truck transport geometric series 5 − 1 + – …
increased 15% per day. What is the total
amount of vegetables the truck transport every Solution
week, to the nearest whole number. 5 − 1 + –…
a = 5, r =
5. In 2013, the number of student in a small
school was 284. It is estimated that the student The sum to infinity is
population will increased by 4% every year. = = = = = = 4.17
. / ⁄
i. Write a formula for the student population.
ii. Estimate the student population in 2020.
2. Find the sum to infinity of the geometric
6. Jack has purchased 20 books. The first book progression 1, , , …
cost Ghȼ 1.00, the second cost Ghȼ2.00, the
third Ghȼ4.00, the fourth Ghȼ8.00 and so on. Solution
How much did Jack pay for the 20 books? 1, , , …
a = 1 and r = for −1 < r < 1
Challenge Problems
1. A superball is dropped from a height of 10m. = = = =
⁄ ⁄
At each rebound, it rises to a height which is
90% of the height from which it has just fallen. 3. The sum to infinity of an exponential
What is the total distance through which the sequence is 9. If the second term is 2, find the
ball will have moved before it finally comes to values of :
rest ? i. the first term

Baffour Ba Series, Further Mathematics for Schools Page 466


ii. the common ratio. The first term is 3 or 6 and the respective
common ratio is or r =
Solution
= 9 and =2
4. The sum to infinity of an exponential
=
sequence (G.P) with a positive common ratio is
25 and the sum of the first two terms is 16.
But =9 Find:
=9 i. the fifth term;
a = 9(1 – r) ii. the sum of the first four terms.
a = 9 – 9r ………………(1)
Solution
= arn−1 = 25
= ar2−1 =
ar2−1 = 2 (But U2 = 2)
25 = (But = 25)
ar = 2
25(1 – r) = a
a = ………………….(2)
a = 25 – 25r ………………(1)

eqn (1) = eqn(2); The sum of the first two terms is 16;
9 – 9r = + = 16
9r – 9r2 = 2
9r2 – 9r + 2 = 0 But = arn−1
(3r – 1) (3r – 2) = 0 =
r= or r = =a

=
Put r = in eqn (1) =
a = 9 – 9. / = ar
a=6
+ = 16
Therefore, when a = 6, r = ⇒ a + ar = 16……………….(2)

Put r = in eqn (1); From eqn (2)


a(1 + r) = 16
a = 9 – 9. /
a= …………………………(3)
a=9–6
a=3
eqn (3) = eqn (2);
Therefore, when a = 3, r =
= 25 – 25r

Baffour Ba Series, Further Mathematics for Schools Page 467


16 = (1 + r) (25 – 25r) 0 = 2x2 – x2 – 3x – 2x + 1 – 1
16 = 25 – 25r + 25r – 25r2 x2 – 5x = 0
16 = 25 – 25r2 x (x – 5) = 0
16 = 25(1 – r2) x = 0 or x = 5
= 1 – r2 Since x > 0, x = 5

r2 = 1 – b. The constant ratio, r ;


2
r = r=

r=√ = Put x = 5 in r =
r= = =
( )
Put r = in eqn (1);
c. For an infinite exponential sequence;
a = 25 – 25. / = 25 – 15 = 10
=

Now a = 10 and r = , = arn−1


Given a = (2x – 1) = 2(5) – 1 = 9 and r =
i. = . / = = =
⁄ ⁄
= . / = . /= = 27

( ) Exercises 16.12
ii. =
1. Find the sum to infinity of the G.P. with first
( . / *
term 3 and common ratio .
= = 189.3990

2. The sum to infinity of a G.P. is four times the


5. The first three terms of an infinite first term. Find the common ratio.
exponential sequence (G.P) are (2x – 1), (x + 1)
and (x – 1), where x > 0, calculate the; 3. The sum to infinity of a G.P. is twice the sum
a. value of x of the first two terms. Find possible values of
b. constant ratio; the common ratio.
c. sum of all the terms of the sequence.
Reccurence Relation
Solution It is an equation that defines a sequence based
a. For a G.P, r = = on a rule which actually helps in finding the
= next term dependent upon the previous term.
This means, if the previous term is defined,
(x + 1) (x + 1) = (x – 1) (2x – 1)
then from the equation, the next term or series
x2 + x + x + 1 = 2x2 – x – 2x + 1
of the term can easily be determined. The set of
x2 + 2x + 1 = 2x2 – 3x + 1

Baffour Ba Series, Further Mathematics for Schools Page 468


new terms could be found out if there is a = 3 (3) – 4
standard pattern developed. =5

If the nth term of a sequence is denoted by , = =3 –4


then the recurrence relation is of the form: =3 –4
= f( ) for some for function f. = 3 (5) – 4
For example in =4– = 11
When = 2, then is expressed as:
3. A sequence is defined by =3+ , =
=4–
1 where n ≥ 1. Find the sum of the first ten
=4– terms of the sequence.
=4–1
=3 Solution
=3+
Worked Examples =1
1. The nth term of a sequence is . If =3
and = + 5, find . = =3+
=3+
Solution
= +5 =3+1
=3 =4

= = +5 Common difference, d = –
= +5 d=4–1=3
=3+5
= , ( ) )-
=8
= , ( ) ( ) )-
= = +5
=5, ( )-
= +5
=8+5 =5, -
= 13 = 145

2. A sequence is defined by =3 – 4, 4. The nth term of a sequence is 2n2 – n – 1. If


n ≥ 1. If = 3, find . its (n – 1) th term is 119, find the value of n.

Solution Solution
=3 –4 = 2n2 – n – 1
=3 = 2(n – 1)2 – (n – 1) – 1
= 2(n2 – 2n + 1)2 – n + 1 – 1
= =3 –4
= 2n2 – 4n + 2 – n + 1 – 1
=3 –4
= 2n2 – 4n – n + 2 + 1 – 1

Baffour Ba Series, Further Mathematics for Schools Page 469


= 2n2 – 5n + 2 2. Find the first term of the sequence a, and the
commom ratio r, such that | |< 1.
2n2 – 5n + 2 = 119 (But = 119) 3. Substitute a and r in =
2n2 – 5n + 2 – 119 = 0
2n2 – 5n – 117 = 0 Worked Examples
(n – 9) ( 2n + 13) = 0 1. Write 0.9999… as an exponential series and
n = 9 or 2n = - 13 show that in the limit, the sum of series equals
n = 9 or n = one.

Exercises 16.13 Solution


1. For all integers k 2, find the terms , 0.9999… = 0.9 + 0.09 + 0.009 + 0.0009 + …
and of the sequence defined by = + This is a G.P. with first term, a = 0.9 and
given that = 1 and = 3. common ratio, r, = = 0.1

2. Let { } be a sequence that satisfies the


The series is an infinite series with sum;
recurrence relation = + for n =
2, 3, 4, …and suppose that = 3 and = 5. = = = =1
What are the values of and ?
2. Express 0.1 ̇ as an infinite geometric series
3. Let { } be a sequence that satisfies the and find the sum of the geometric series.
recurrence relation = + 3 for n = 1, 2,
3, 4, …and suppose that = 2 , what rae the Solution
values of , and ? 0.1 ̇ = 0.1666666…
= 0.1 + 0.06 + 0.006 + 0.0006 + …
4. Let { } be a sequence that satisfies the = + + + ….
recurrence relation = – for n =
2, 3, 4, … and suppose that = 3 and = 5. From the series, the term after form a
What are the values of and ?
geometric series; with a = and
Recuring Decimals as Geometric Series r= ÷ =
A geometric series converges to
if | |< 1. This means that the common ratio of Sum of the geometric series ;
the geometric series should be less than one, ⁄ ⁄
= = =
else the series will not converge. ⁄ ⁄

0.1 ̇ = + =
Steps:
1. Express the recurring decimal as a sum in the 3. Convert 0. ̇ ̇ to an infinite converging
decimal or fractional form. geometric series.

Baffour Ba Series, Further Mathematics for Schools Page 470


Solution Word Problems Involving Infinite G. Series
0. ̇ ̇ = 0.47474747… The yearly output of a silver mine is found to
= 0.47 + 0.0047 + 0.000047 + … be decreasing by 25% of its previous year‟s
= + + + …. output. If in a certain year, its output was
Ghȼ25,000.00, what could be reckoned as its
From the series, form a geometric series, with a total future output?

= and r = ÷ = Solution
The total output, in cedis, for subsequent years
Sum of the geometric series ; = 25000 × 0.75 + 25000 × (0.75)2 + 25000 ×
⁄ ⁄
= = = = (0.75)3 + ...
⁄ ⁄

4. Express 0.45 + 0.0045 + 0.000045 + … in = = 75,000.


the form , b ≠ 0, where a and b are integers.
The Sigma Notation (Σ)
Solution If the general term of a series with n terms is
0.45 + 0.0045 + 0.000045 + known, then the complete series can be written
= + + +… down in short notation as indicated by the
, , ,
following illustrations:
1. a + (a + d) + (a + 2d) + ... + (a + [n − 1]d)
The series is an infinite series with a = and
= ( , - )
r= ÷ =
,
2. a + ar + ar2 + ...ar n−1 =

Substitute a and r in = 3. 12 + 22 + 32 +...n2 =


⁄ ⁄ 4. −13 + 23 − 33 + 43+...(−1)nn3 = ( )
= = = = =
⁄ ⁄

Notes:
Exercises 16.14
I. It is sometimes more convenient to count the
A. Find the sum of the following infinite
terms of a series from zero rather than 1.
geometric series, if it exist.
For example:
1. 1.5 + 0.015 + 0.00015 + … 3. , …
a + (a + d) + (a + 2d) + ......a + [n − 1]d
2. 8 + 4 + 2 + 1 … 4. 0. 3 ̇ ̇
= ( ) and
5. 1+ + +… 6. 0. ̇ ̇
a + ar + ar2 + ar3 + ......ar n−1 =
B. 1. Calculate the fraction that is equivalent to
0.181818…. In general, for a series with n terms starting at u0,
2. What fraction that is equivalent to 3. 2777… + + + +…+ =

Baffour Ba Series, Further Mathematics for Schools Page 471


II. We may also use the sigma notation for ⁄ ⁄
= = =1
“infinite series” such as those already ⁄ ⁄

encountered in the sum to infinity of a


3. Find the sum ( )
geometric series. For e.g.1 + + + + ..=
or Solution
( )
Worked Examples = 3(1) + 2 = 5
1. Evaluate ( ) = 3(50) + 2 = 152
( )
Solution =
( )
( ) = = 3925
= 2(0 – 3) + 2(1 – 3) + 2(2 – 3) + 2(3 – 3) + 2(4
– 3) + 2(5 – 3) + 2(6 – 3)
= 2(– 3) + 2(– 2) + 2( – 1) + 2(0) + 2(1) + 2(2) Exercises 16.16
+ 2(3) Write down the next two terms and also the n-
= – 6 + (– 4) + (– 2) + 0 + 2 + 4 + 6 th term of the following sequences of numbers
=0 which are either arithmetic progressions or
geometric progressions:
2. Evaluate . / 1. 40, 29, 18, 7, ...; 2. , , 7, ...;
3. 5, 15, 45, ...; 4. 10, 9.2, 8.4, ...;
Solution
5. 81, −54, 36, ...; ; 6. , , …..
. /

When r = 1, . / = 7. For the geometric series 1 + 1.2 + 1.44 + ...,


find the 6th term and the sum of the first 10
When r = 2, . / =
terms.
When r = 3, . / =
Sum to obtain the infinite series 8. Express the series + + + + ….n terms
= + + +… in both of the forms and
Hint: Find the pattern in the numerators and
Sum to infinity is given by: denominators separately
=
9. Determine the sum to n terms of the series
But a = and r = ÷ =
1·3+2·5+3·7+4·9+...

Baffour Ba Series, Further Mathematics for Schools Page 472


17 INDICES AND LOGARITHMS Baffour Ba Series

Positive Integral Indices Note:


The task of multiplying a number by itself, a Not all natural numbers exhibit the property of
certain number of times gives rise to the idea of exponents of one base. In as much as some can
powers of numbers. For example, be expressed as exponents of one base, mostly
can be explained as 2 multiplied by itself 4 prime base, others cannot be expressed as such.
times. This can be written in short form Few numbers can also take a composite base.
as: 4
For example 36 = 6 × 6 = 62

In the expression, , 2 is called the base and 4 Worked Examples


is called the index or exponent and it is read as Express the following as exponents
“two exponent four”. 1. 32 2. 81 3. 125

Generally, in an expression of the form m,a is Solution


called the base and m is called the index or 1. Least prime factors of 32 = {2}
exponent or power and it is read as “a 32 = 2 × 2 × 2 × 2 × 2 = 25
exponent m” or “a to the power of m”.
2. Least prime factors of 81 = {3}
Laws of indices 81 = 3 × 3 × 3 × 3 = 34

2. Least prime factors of 125 = {5}


1. × = 5. . / = 125 = 5 × 5 × 5 = 34

Application of the Laws of Indices


2. = 6. =1
1. To simplify an exponential expression
means, leave the answer in exponent form
3. ( ) = 7. = 2. To evaluate an exponential expression means
to find the value of the expression.
) 8. ⁄ ⁄ In dealing with complex exponential
4. ( = =( )
expressions, apply the laws of indices one after
the other. Also:
Breaking a Number as an Exponent I. Arrange the numbers (base) in ascending
1. Find the prime factors of the number. order.
2. Identify the least prime number. II. Align the base when the problem involves
3. Find the number of times the least prime division.
number multiplies it self to produce the give
number.

Baffour Ba Series, Further Mathematics for Schools Page 473


Worked Examples 6.
1. Simplify
Solution
Solution = = = ×8
Re-arrange the terms to have
5–2 7–2
= 33 × 55 Exercises 17.1
A. Simplify the following;
2. Evaluate 1. 2. 3.
( )( )
4. 5. 6.
Solution
5–2 7–2
= 33 × 55
= 27 × 3125 = 84,375 B. Evaluate the following;
1. 2.
3. Simplify, 6 4
3. , 4. ( )
6 4 4 2
Solution 5. ( ) ( )
=
= 8–0 10 – 9 Zero Power of a Natural Number
Study the pattern below carefully with
= 28 × 9
reference to the division rule:
am ÷ an = am – n
4. Evaluate 24

Solution
=
= 8–0 10 – 9

= 28 × 9
= 256 × 9
= 2,304
Since the value of = 1, it follows
that , so (a≠0)
5. Simplify
Worked Examples
Solution Simplify the following;
= = × = × 1.

Baffour Ba Series, Further Mathematics for Schools Page 474


Solution For e.g.√ ⁄
=( ) ⁄
=
9–9 0

Worked Examples
2. 0
1. Evaluate ⁄

Solution Solution
4 × 690 = 4 × 1 = 4 ⁄
=( ) ⁄
= = 3 × 3 × 3 = 27

Exercises 17.2 ⁄
Evaluate each of the following: 2. Evaluate . /
1. 2.
Solution
11 0 ( ) ⁄ ( ) ⁄
3. 4. (8 8 8) 8 ( ⁄ ) ⁄
=( =( = =
) ⁄ ) ⁄
6 4 5 5
5. Show that; (13 ) ( )=1
3. Simplify ( )
Rational Indices
By the law of indices, Solution
⁄ ⁄ ⁄ ⁄
………….(1) ( ) =
2 × 2 = 4…………………..(2) = ( ) ⁄
×( ) ⁄

=2×7
Comparing eqn (1) and eqn (2)

=2
4. Evaluate ( )

But √ = 2. Therefore, we define as the
square root of 4 written as√ = ⁄ Solution
⁄ ⁄

( ) =
In general,√ = ⁄ ⁄
⁄ = ( ) × ( )
For e.g.√ = ⁄ ( ) =3
=3×5
⁄ ⁄ ⁄
= 15
Similarly, = =a

We define as the cube root of a written ⁄ ⁄

⁄ 5. Evaluate
as; √ =

⁄ Solution
In general, √ = Method 1
⁄ ⁄
For e.g. √ =( ) =3 ⁄ ⁄ ( ) ⁄ ⁄ ( )

= = = = =

Generally, if is taken to be the nth root

Method II
of , then =√

Baffour Ba Series, Further Mathematics for Schools Page 475


Express each as an exponent with base 2 and 1. , (a ≠ 0) and is the reciprocal
apply the laws of indices.
of

Exercises 17.3 2. = (a ≠ 0) and is the


A. i. Express with signs of the form √ m reciprocal of

1. 2. 3.
Worked Examples
1. Express as a negative index.
ii. Write the following in exponent form;
1. √ 4 2. √ 5 3. √ 6
Solution
B. Find the values of the following: 1. =

1.8 4. ( 8) 7.
2. 5. 8 8. 2. Write in the form
⁄ ⁄
3. 6. 9.
Solution
C. Evaluate the following; = = =
⁄ ⁄ ⁄
1. . / 2. . / 3. . /
⁄ ⁄ ⁄ 3. Simplify
4. . / 5. . / 6. . /

7. 8. (8 ) Solution
= = = =
Find x if
1. (x + 1)5 = 243 2. (x – 1)5 = ⁄8) ⁄
4. Find the value of (

Negative Indices Solution


From the law of indices, = , but ( ⁄8) ⁄

and = 2 ⁄ ⁄
⁄ ( )
(8⁄ ) = ⁄ = = =
( ) ⁄
By substitution,
1 = = and therefore, is taken to ⁄
5. Find the value of . /
be equal to
Similarly, = = , where is the Solution

reciprocal of . / = (8 ) ⁄
=( ) ⁄
=3

In general,

Baffour Ba Series, Further Mathematics for Schools Page 476



6. Find the value of 8
4.
Solution ( ) ( )
= – = – = –
Method 1
( ) ( ) ( )
⁄ ⁄ = –
8 =( ) = =
= –
= –
Method II
⁄ ⁄ = - = + = =

8 =. / = ⁄ =( ) ⁄
=

Exercises 17.4
More Examples A. Express as negative exponents;
Simplify the following without calculator: 1. 2. 3.
1. + +8 2. 4. = 3x 5. 6.

B. Find the values of the following:


3. ( ) 4. 1. . / 2. . / 3. . /
⁄ ⁄ ⁄
4. . / 5. . / 6. . /
Solution
1. + +8 C. Find the values of the following;

=( ) +( ) +( ) 1. 2. 8 ⁄ 3. ⁄

= 52 + 23 + 2 – 2 4. ⁄
5. ⁄
6. ⁄

= 25 + 8 +
D. Simplify the following;
= 25 + 8 + ⁄ ⁄ ⁄ ⁄
1. 5.
= 33 + 2. ⁄
8 ⁄
6. ⁄ ⁄

= 33 ⁄ ⁄ ⁄ ⁄
3. ⁄ 7. ⁄

⁄ ⁄ ⁄ ⁄

2. 4. ⁄ 8. ⁄ ⁄

( ) ( )
= = = + = 1 + = Exponential Equations 1
( ) An equation involving powers of numbers, is
called an exponential equation. For example,
3. ( ) 8, are exponential equations.
=. / =. / =. / = =
The steps in solving exponential equations are;

Baffour Ba Series, Further Mathematics for Schools Page 477


1. Express all the numbers as exponents of the Solution
same base (usually a prime number). ×
( )
2. Express both sides as a power of the same = 20
number, using the laws of indices. x + 2 + 3x = 0 (Equating exponents)

3. Equate the exponents and solve the equation. 4x + 2 = 0


4x = –2
Worked Examples x=

1. Find the value of x in = x=


Solution 5. Solve . / – . / =0
=
Solution
= (Equating exponents)

3x – 1 = -1 . / – . / =0
3x = - 1 + 1 – ⁄
=0
3x = 0 ( )
– =0
x=0
( )
– =0
2. Solve for x given that = 49 ( )
=
-3x = - (Equating exponents)
Solution
-3x× 2 = - × 2 (Multiply through by 2)

2 -6x = -3
(Equating exponents)
x=
3 x=

( )
3. If = 256, determine the value of x 6. Solve =

Solution Solution
= 256 = ( )
2x + 3 = x2 (Equating exponents)
= x2 – 2x – 3 = 0
= (Equating exponents) (x 2 – 3 x) + (x – 3) = 0 (By quadratics formula)
– 5x = 8 x (x – 3) + 1 (x – 3) = 0
x=– (x + 1) (x – 3) = 0
x + 1 = 0 or x – 3 = 0
4. If ×8 , find the value of x x = -1 or x = 3

Baffour Ba Series, Further Mathematics for Schools Page 478


Exercises 17.5 3. Solve the quadratic equation by any
A. Solve the following; preferred method to obtain two answers,
1. = 5. =8 ignoring a negative answer.
4. Equate the answer obtained in (3) to the
2. = 6. . / =2
common exponent in (1)
( )
3. = 7. =( ) 5. Obtain equal bases and equate the exponent
4. = to obtain the value(s) of the variable as the
truthset or solution set of the variable
B. Solve the following equations:
1. = 27 6. = 0.125 Woked Examples
2. 1. Solve 22x - 5(2x) + 4 = 0
3. 4
8 8x – 5
4. = 1000 9. =√ Solution
5. = 81 10. = 22x - 5(2x) + 4 = 0
Let a = 2x
C. Find the value of the variables: ⇒ a2 – 5a + 4 = 0
1. . /x = 5. = a2 – 4a – a + 4 = 0
a(a – 4) – 1 (a – 4) = 0
2. . / = 6. . / = . /
(a – 1) (a – 4) = 0
3. 4 × =. / 7. . / = a = 1 or a = 4

4. . / =8 8. . / =. /
But a = 2x
⇒ 2x = 1 or 2x = 2
D. Find the ruth set of the following: 2x = 20 or 2x = 2
1. 82x. . / = .. / x = 0 or x = 1

2. 4x. . / =8.( ) 2. Find the truth set of 2(2 + 2x) + 3(2x) – 1 = 0

3. 92x. . / = 27 . ( )
Solution
4. 27 × =8 2(2 + 2x) + 3(2x) – 1 = 0
( )
5. = 22. 22x + 3(2x) – 1 = 0
6. = 0.1× (4) 22x + 3(2x) – 1 = 0

Exponential Equations 2 Let 2x = a


1. Identify the common exponent in the given 4a 2 + 3a – 1 = 0 (Factorization)
exponential equation and equate it to any
a = or a = -1 (Ignore negative answer)
preffered variable.
2. Substitute the variable in the equation to
obtain a quadratic equation. ⇒ 2x =

Baffour Ba Series, Further Mathematics for Schools Page 479


2x = (a – 3) . /=0 (By factorisation)
= a = 3 or a =
x=-2
But a = 3x
3. Find the value of x in 4x – 5(2x ) + 4 = 0
⇒ 3x = 3 or 3x =
Solution 3x = 3 or 3x =
22x – 5(2x ) + 4 = 0 3x = 3 or 3x = 3 – 2
x = 1 or x = -2
Let 2x = a
a2 – 5 a + 4 = 0 5. Solve 52x + 1 – 126 (5x – 1) + 1 = 0
(a2 – 4 a) – (a + 4) = 0
a (a – 4) – 1 (a – 4) = 0 Solution
(a – 1) (a – 4) = 0 52x + 1 – 126 (5x – 1) + 1 = 0
⇒ a = 1 or a = 4 52x . 51 – 126(5x . )+1=0
52x . 31 – 126 . /+1=0
Now,
2x = 1 or 2x = 4
Let a = 5x
When 2x = 1, 5a2 – a+5=0 (Multiply through by 5)
2x = 20, 2
25a – 126a + 5 = 0
x=0
(a – 5) . / = 0 (By factorization)
When 2x = 4, a = 5 or a =
2x = 22,
x=2 But a = 5x
Therefore, x = 0 or x = 2
⇒ 5x = 5 or 5x =
3. Solve 32x + 1 – 28(3x- 1) + 1 = 0 5x = 5 or 5x =
5x = 5 or 5x = 5 – 2
Solution x = 1 or x = -2
32x + 1 – 28(3x- 1) + 1 = 0
32x . 31 – 28(3x . )+1=0 Exercises 17.6
2x 1
3 . 3 – 28 . /+1=0 A. Solve the following equations :
1. 32x – 30 (3x – 2) + 1 = 0
Let a = 3x 2. 32(1+ x) – 3x = 3x + 3 – 3
3a2 – a+1=0 (Multiply through by 3) 3. 22x – 3 – 2x – 2 – 1 = 0
9a2 – 28 + 3 = 0 4. 22x + 2 – 9 (2x) + 4 = 0

Baffour Ba Series, Further Mathematics for Schools Page 480


5. 52x – 3 – 126 (5x – 3) = - 1 Any statement in index form has an equivalent
6. 5 2x x
– 30 . 5 + 125 = 0 log form, for example,
8= (Index form)

B. Find the truth set of the following: ⇒log 8 = 3 (Log form)

1. 32x + 3 – 3x + 2 – 3x + 1 + 1 = 0
2. 22x + 6 – 65(2x + 3) + 64 = 0 Worked Examples
3. 5x – 24 = (52 – x) 1. Express = 16 in logarithmic notation
4. 3x . 9x – 1 = 27
5. 4x + 2x = 23 + x Solution
6. 3x + 1 – 3x – 1 = 24√ In a = ,log b a = c
If = 16, then log2 16 = 4
C. Solve:

1. 22x – (2x) + = 0 2. Write 8 = in logarithmic form

2. 22x – (2x) + = 0
Solution
x–2 –x
3. 2 – (2 )= If 8 ⁄
= , then log8 = ⁄
4. 4x + 1 + 2x + 1 + . / =
3. Express in index notation log2 32 = 5
5. = 2 – 3x – 1

6. . / . = Solution
If log2 32 = 5, then 32 =
Logarithms
Exercises 17.7
From the calculator, it is seen that the log of
A. Write the following in logarithm form:
100 to the base 10 is 2. This is stated
1. 125 = 4. 0.01 =
mathematically as log10 100 = 2. This means
⁄ ⁄
that 100 = , where 10 is the base and 2 is 2. =7 5. =5

the index (logarithm). Note that log10 is written 3. = 64 6. =
simply as log.
B. Express in exponential form:
Relating Indices and Logarithms 1. log 4 2 = 3. log 27 3 =
In general, if a = then the logarithm of a is
2. log 5 625 = 4 4. log (0.1 × 108) = 7
expressed as: logb a = cread as“the logarithm of
a to the base b is c” Laws of Logarithms
Thus: Logarithms and their operations are govern by
some basic properties and laws as shown below
If a = , then =c for a, b and x belonging to the set of real
numbers;

Baffour Ba Series, Further Mathematics for Schools Page 481


Laws of Lagarithm Examples loga (xy) = (b + c) loga a
1. loga (xy) = logax + logay e.g. log2 (4 × 2) loga (xy) = b + c…………..(1)
= log2 4 + log2 2
= log2 22 + log22 From x = a b and y = a c
=2+1 loga x = b and loga y = c
=3
Substitute in eqn (1)
2. loga. / = logax – logay log 2. / loga (xy) = loga x + loga y
= log 2 4 – log 2 2
= log 2. / 1. Let x = a b and y = a c
= log 22 = 1 Then =
y log2 4
3. loga x = y loga x
= log 2 22 =
= 2 log 2 2
=2×1 = 2
loga . / = log a
4. loga a =1 e.g. log 3 3 = 1
loga . / = (b – c) log a a
5. loga1 =0 log 10 1 = 0
loga . / = b – c……………(2)

6. loga. / = logax + logay loga 2 + loga 6 –


loga 4 From x = a b and y = a c
– logaz = loga. / loga x = b and loga y = c
= loga. /
Substitute in eqn (2)
= loga 3
loga . / = loga x – loga y
log 1000 10
7. loga x =
=
3. Let x = ab and loga y = c
= =
Then =( )
= = loga = loga ( )
8. loga b= log 4 64 loga = bc loga a
=
(change of base)
= loga = bc ……………(3)
From x = a b, loga x = b
=
=3
Substitute in eqn (3);
9. =b =4
loga = c loga x

Proofs of law 1 , 2 and 3:


Application of the Laws of Logarithms
1. Let x = a b and y = a c
Worked Examples
Then xy = ab ac
1. Write log3 x + log3(5y) as a single logarithm.
xy = ab + c
loga (xy) = loga ( )
Baffour Ba Series, Further Mathematics for Schools Page 482

Solution = loga x3 + loga – loga z2
log3 x + log3(5y) = 3 loga x + loga y – 2 loga z
= log 3 5xy

2. Express log 3 (2z) – log3 x as a single 6. Express lga√ in terms of log of x, y and z
logarithm.
Solution
Solution
log 3 (2z) – log3 x . lga √
= log3 ⁄
= lga . /

3. Express loga (x2 – 1) – loga y – 4 loga z as a = lga


single logarithm. = lga x2 y3 – lg z4
= (lga x2 + lga y 3 – lga z4)
Solution
loga(x2 – 1) – loga y – 4 loga z = (2 lga x + 3 lga y – 4 lga z)

) – loga y – loga z4 = lg x + lg y – 2 lg z
= loga(
= loga √ – (loga y + loga z4)
Exercises 17.8
= loga √ – loga (yz4)
A. Express as a single logarithm:

= loga 1. log 2 + log 12 – log 6
2. log 3 + log 16 – log 4 – log 6
4. Write loga 6xy3 – 2 loga 3xy2z as a single 3. (log6 4 + log6 9)2
logarithm. 4. (log525 + log5 15 – log5 75)4
5. 4 lg 5 – 2 log 5 + 2
Solution
loga 6xy3 – 2 loga 3xy2z B. Write each as a single logarithm.
= loga 6xy3 – ( loga 3xy2z)2 1. 5 log 4 – 3 log 4 + 1
2. 4 log 2 + log 2 – 1
= log ( ) 3. 3 log 4 + log 4 – 2
= log 4. 3 log 4 – log 4 – 2


C. Write each as a single logarithm.
5. Express loga in terms of logarithms of x,
1. lga x – 2 log ay + loga z
y and z.
2. lga 18x2 + 3 log a z – loga 6y
Solution 3. lg a x2y + 2 log a 5xy3 – loga 10 x2 y2
√ ⁄ 4. lga x3 y4 – 2 loga 4 y2z + loga 8 x2yz
loga = loga ( ) – loga z2

Baffour Ba Series, Further Mathematics for Schools Page 483


5. lga 8x2z3 + loga 3 + loga 27x4y6 z9 that log . / = (log a + log b)
6. lga 8 x2 y4 + loga 5 + loga 8 x6 z4
Solution
D. Express each in terms of log of x, y and z a2 + b2 = 7ab
√ √ (a + b)2 = a2 + 2ab + b2
1. log a 2. log a (a + b)2 = a2 + b2 + 2ab

3. log a √ 4. log a √
(a + b)2 = 7ab + 2ab
(a + b)2 = 9ab
A common Logarithm a+b =√
A common logarithm is a logarithm of base 10.
This is expressed as log10 but usually written as a+b=( )
log. a+b=3( )
=( )
Proof of Logarithmic Identities
Equations that are true for all values of the
variable are called identities. Equations with Take log on bothe sides;
lofarithms are logarithmic identities. log . / = log ( )

To proof an identity is to show the log . / = log (ab)


mathematical steps involved in making one side log . / = (log a + log b)
the same as the other side.
3. Let a represent any positive number other
Worked Examples
1. Show that loga (x + y ) – loga (x – y ) = loga than 1. If loga (x + 1) – loga 2 = logax, then
(x2 – y2) prove that (x – 1)2 = 0

Solution Solution
loga (x + y ) – loga (x – y ) = loga (x2 – y2) loga (x + 1) – loga 2 = logax
L. H. S. R. H. S
2[log ( ) – log ] = logax
2 2
Consider the L.H.S; loga (x + 1) – loga 2 = logax
loga (x + y ) – loga (x – y ) loga (x + 1)2 – loga 4 = logax
= loga (x + y ) (x – y ) loga (x + 1)2 – loga 4 – logax = 0
( )
= loga (x2 – xy + xy – y2) log =0
= loga (x2 – y2) ( )
= a0
L. H. S = R. H. S
( )
=1
2 2
2. If log a + b = 7ab where a > 0, b > 0. Show (x + 1)2 = 4x

Baffour Ba Series, Further Mathematics for Schools Page 484


x2 + 2x + 1 = 4x Simplifying Logarithms Expressions
x2 + 2x – 4x + 1 = 0 Type I: Form log a b
x2 – 2x + 1 = 0 I. Express b as a product of two factors to
x2 – 2x + 12 = 0 obtain log a (c × d)
(x – 1)2 = 0 II. Apply the laws to obtain log ac + log ad
III. Express c and d as exponents, if possible
Exercises 17. 8B and apply the exponential law of logarithm
A. Prove the following identities. IV. Apply other laws if necessary
1. logb a =
Worked Examples
2. loga ( √ ) = loga c 1. Simplify log2 72
3. loga . / = - loga x
Solution
4. + = log 2 72 = log2 (8 × 9)
5. log( x – 1) + log (x – 2) = loga (x2 – 3x + 2) = log2 8 + log2 9
= log2 23 + log2 32
B. 1. Let p = loga x and q = loga y. Express logx = 3 + 2 log2 3
a and logy a in terms of p and q respectively.
Hence, show that logxy a = 2. Simplify log 540

Solution
2. If x = loga (bc), y = loga (ca) and z = loga
log 540 = log5 (5 × 8)
(ab), prove that x + y + z = xyz – 2
= log5 5 + log5 8
= 1 + log5 23
Logarithmic Expressions
= 1 + 3 log5 2
A logarithmic expression is a logarithm
statement that is not equated or does not
Exercises 17.9
contain an equal sign.
Simplify;
1. log372 2. log 363 3. log 5150
To evaluate a logarithmic expression is to find
the value of the expression by applying the 4. log 8120 5. log 2√ 8 6. log √ 8
laws of indices. Hence, answers should not
Type II: Involving Addition and Subtraction
contain the word “log”.
1. Ensure that all the logs are of the same base
2. Express the given expression as a single
To simplify a logarithmic expression is to apply
logarithm of the form log ab.
the laws to the extend that no law is applicable. 3. Go throught the processes of type I to
Hence, a simplified answer should contain the complete simplification, only if a is a factor of b.
4. If a is a not a factor of b, then leave your
word“log”
answer in the form: log ab

Baffour Ba Series, Further Mathematics for Schools Page 485


Worked Examples = log2 (x3) + log2(y) – log2(x + 3)4
1. Simplify log 49 + log 4 21 – log 47 = log2(x3y) – log2 (x + 3)4
= log2(x3y) − log2(x + 3)4
Solution = log2(x + 3) (4x3y)
log 4 9 + log 4 21– log 47
= log 4. / Exercises 17.10
= log 4 27 A. Simplify the following:
= log 4 33 1. log ₃ 15 – log ₃ 5 + 2 log ₃ 5
= 3 log 4 3 2. 2 + 2 log₂ 5 – 3 log₂ 3
3. 2 log₂ 5 – 3 log₂ 3
2. Simplify log2 3 – log2 15 + log2 50 4. ½ log ₉ 36 + 2 log ₉4 – 3log ₉ 4
5. log ₅ 8 + log ₅ (1/1000)
Solution 6. ½ log ₅ 36 – ⅓ log ₅ 8 + 3 log ₈ 2
log2 3 – log2 15 +log2 50
= log2 . / B. Simplify
= log210 1. 3 + 5 log ₃ 5
= log2 (2 × 5) 2. log ₃ . / + log ₃ . /
= log2 2 + log 2 5 3. log ₅ 5 + log ₅ 20 – log ₅24 + log ₅ 25 – 3
= 1 + log 2 5 4. 2 log ₄2 + 3 log ₂ 5 – log ₂ 15 + log ₂ 7

( )
3. Simplify log 2 Evaluating Logarithmic Expressions
Type I: Form loga b
Solution Given a logarithmic expression of the form
( ) loga b to determine its value:
log 2 I. Equate the expression to any preferred
= lg 2 (x + y)3 – lg 2 (x2 y2) variable, say x. i.e. loga b = x
= 3 lg 2 (x + y) – [ lg 2 (x2) + lg 2 (y2)] II. Write the logarithmic expression as an
= 3 lg 2 (x + y) – lg 2 (x2) – lg 2 (y2) exponential equation. That is: b =
= 3 lg 2 (x + y) – 2 lg 2 x – 2 lg 2 y III. Express the exponential equation as
equation with a common base, if possible.
4. Simplify 3log2(x) – 4log2(x + 3) + log2(y). IV. If not possible, take the log on both sides of
the equation. log b = log
Solution V. Simplify the variable side of the equation.
3log2(x) – 4 log2(x + 3) + log2(y) i.e.log b = x log a
= log2(x3) – log2 (x + 3)4 + log2(y) VI. Make the variable the subject and work out
= log2(x3) – log2(x + 3)4 + log2(y) to obtain the value of the variable. i.e. x=
= log2 (x3) + log2(y) – log2 (x + 3)4

Baffour Ba Series, Further Mathematics for Schools Page 486


Worked Examples Let log3 9 = x
1. Use tables or calculator to find an 9=
approximate value of log2 7 ⇒ = (equating exponents)
x=2
Solution 4. Evaluate log 0.1 10
Let log27 = x
⇒7= Solution
log 7 = log 2x (Taking log on both sides) Let log0.1 10 = x
log 7 = xlog 2 10 =
x= 10 =. /
x = 2.807 (3 d. p.) 10 = ( )
10 =
2. Evaluate log2 64 1= - x
x = -1 (Try other methods)
Solution
Let log2 64 = x 5. Evaluate log3 27√
64 =
= (equating exponents) Solution
x=6 Let log3 27√ = x
27√ =
3. Evaluate log381 × =
=
Solution
Method 1 =
log3 81 x=

= log334
6. Find the value of log 5
= 4 × × log3 3, (log3 3 = 1)

=4× ×1 Solution
=2 log 5
= log 5
Method 2 = log 5 ( )
log381 = log 5
= log38 ⁄ = - 2 log 5 5
⁄ =-2×1
= log3 ( )
=-2 (Try an alternative method)
= log3 9

Baffour Ba Series, Further Mathematics for Schools Page 487


7. Evaluate log 9 2. log √ 4. log 2 2 √

Solution E. Evaluate :
log 9 1. 3. = 10x
2. 4. = 50 –
Let log 9 = x

⇒9=. / Type II: Involving Addition and Subtraction


1. Ensure that all the logs are of the same base
9=( )
3 2. Express the given expression as a single
3=
logarithm of the form: loga b.
3=-x
3. Equate the single log expression to any
x = -3
preferred variable to obtain a logarithmic
equation.
8. Evaluate without calculators
4. Write the logarithmic equation as an
exponential equation.
Solution
5. Solve for the value of the variable in the
=( ) = 23 = 8 exponential equation as in type I.

Exercises 17.11 Worked Examples


A. Evaluate: 1. Evaluate log 5 12 + log 5 10 to four decimal
1. log 3 243 2. log 27 3 3. log 2 32 places.
4. log 8 2 5. log 0.01 10 6. . /
Solution
log 5 12 + log 5 10
B. Evaluate: = log 5 (12 × 10)
1. lg5 2. log 81 3. log = log 5 120

4. lg16 5. lg5 6. log 4 Let log 5 120 = y


7. log 27 8. log 120 =
log 120 = log
log 120 = y log 5
C. Find the values of the following:
y=
1. log 3 81 4. log 2 64 7. – log 2
y = 2.9746 (4 d.p)
2. log 2 8 5. log 49 7 8. log 2 4
3. log 27 . / 6. log 7 9. log 16 8 2. Evalutae (log 6 4 + log 6 9)2.

Solution
D. Evaluate to 2 significant figures: Method 1
1. log √ 3. log √ log 6 (4 × 9)2
Baffour Ba Series, Further Mathematics for Schools Page 488
log 6 (36)2
= log . / – log . / +log . /
log 6 1296
= log . / – log . / + log . /
Let log 6 1296 = y
1296 = = log . /
log1296 = log = log . /
log1296 = y log ,
= log . /
y=
= log 1010
y=4 =1

Method 2
5. Without using Mathematical tables or
2 log 6 (4 × 9)
calculators , simplify;
= 2 log 6 36
= 2 log 6 62 log10 . / – 2 log10 . / + log10. /
= 2 × 2 log
=2×2×1 Solution
=4 log10 . / – 2 log10 . / + log10. /

3. Evaluate log 4 9 + log 4 21 – log 4 7 log10 . / – log10. / + log10. /
log10 . / – log10 . / + log10. /
Solution
log 4 9 + log 4 21 – log 4 7 lg . /
= log 4 . / lg . /
= log 4 27 lg ( )
lg 10
Let log 4 27 = x

Exercises 17.12
log 27 = log
Find the values of the following:
log 27 = x log 4
1. log√ + log√ – log√
x=
2. log 6 24 + log 6 15 – log 6 10
x = 2.3775 3. log 7 98 + log 7 30 – log 715
4. log 3 – log 15 + log 50
4. Evaluate without using tables or calculators: 5. log 5 + log 16 – log 4 – log 10
log 10. /– 2 log 10. / + log 10. /
Division of Logarithms
Solution To simplify expressions involving division of
log 10. / – 2 log 10. / + log 10. / logarithms

Baffour Ba Series, Further Mathematics for Schools Page 489


I. Express each number as an exponent with a log p = x and log q = y
common base p= and q =
II. Apply the law: loga xy = y loga x pq =
III. Simplify by crossing out common factors =
⇒log
Worked Examples
= log
Simplify :
= log
Solution = log 10 – log10 10 2
= x + y log 10 10 – 2 log10 10
=x+y–2
Express 125 and 25 as a power of 5
= = Exercises 17.13
Evaluate:
1. (a) log 27 ÷ log 3; 2.
2. Simplify –
3. (log 16 − log 2) † log 2 4.
Solution

5. 6.


Make log 6 – log 2 a single log
⇒log . / Solving Exponential Equations
= log 3 In an exponential equation, when the exponent
Substitute in place of log 6 – log 2 of a number at a side of the equation is a
variable, as in 3x = 2

I. Take the log on both sides of the equation.
= = II. Simplify the variable part of the equation.
III. Make the variable the subject and work out
(evaluate) to obtain the value of the variable to
3. Simplify the given approximation.

Solution Worked Examples


⁄ 1. Solve the equation = 5, using tables or
= = = =2 calculators to 5 significant figures.

4. Given that log p = x and log q = y, express Solution


log in terms of x and y In = 5,
log = log 5 (Take log on both sides)
Solution x log 2 = log 5

Baffour Ba Series, Further Mathematics for Schools Page 490


= 2. = 6. =
3. 7. ×
x = 2.3219 (5 s.f) 4. ( ⁄ ) = 6 8. ( ⁄ ) = ⁄

2. Solve the exponential equation – 1 = 8, Exponential Equations 2


approximate your answer to 4 decimal places. It involves an exponential equation that has
been compressed or simplified already, such as
Solution in ( ) . In this case, expand or split up such
–1=8 exponents and represent a common exponential
=8+1 factor by any preffered variable. This usually
=9 leads to a quadratic equation that can be
log = log 9 solved by factorization or quadratic method.
3x log 4 = log 9
Do not end by solving the quadratic equation
x=
because, the answer obtained is just a
x = 0.5283 (4 d.p) representation.

3. Solve for x in the indicial equation Reminder : = .


43x −2 = 26x + 1.
Worked Examples
Solution 1. Find correct to 2 decimal places , the value
43x −2 = 26x + 1 (Taking log on both sides) of x for which 52x – 5 (x + 1) + 4 = 0
(3x − 2) log 4 = (x + 1) log 26;
(3x − 2) 0.6021 = (x + 1) 1.4150; Solution
1.8063x − 1.2042 = 1.4150x + 1.4150; 52x – 5 (x + 1) + 4 = 0
(1.8603 − 1.4150) x = 1.4150 + 1.2042; 52x – 5 x . 5 + 4 = 0 (split up the combined exponent)
0.3913 x = 2.6192;
x= Let 5x = a
x = 6.6936 a2 – 5a + 4 = 0
(a2 – 4a) – (a + 4) = 0
Exercises 17.14 a (a – 4) – 1 (a – 4) = 0
A. Solve for n, to 3 significant figures; (a – 1) (a – 4) = 0
1. =5 2. = 2500 ⇒ a = 1 or a = 4
3. = 680 4. = 31
5. = 3460 6. = 4800 Now,
7. = 8. = 5x = 1 or 5x = 4

B. Solve the equations: When 5x = 1,


1. = 21 – 3x 5. = 5x – 2 lg 5x = lg 1, (Take log on both sides)

Baffour Ba Series, Further Mathematics for Schools Page 491


x lg 5 = lg 1, a (a2 – 4) + 3 = 0
x= a (a2 – 4) = - 3
a = -3 or a 2 – 4 = - 3
x=0
a = -3 or a 2 = - 3 + 4
a = - 3 or a = 1 (ignore a = - 3 )
When 5x = 4,
a=1
lg 5x = lg 4,
x lg 5 = lg 4,
Now 3x = 1
x= lg 3x = lg 1
x = 0.86 (2 d.p ) x lg 3 = lg 1 ,
Therefore, x = 0 or x = 0.86 x=
x=0
2. Find the truth set of 6(9x) + 3x – 1 = 0,
Therefore, x = 0
correct ot 2 decimal places
4. Find to 3 signficant figures the truth set of
Solution
2x – 27(2 – x) = 6
6(9x) + 3x – 1 = 0
6(32x) + 3x – 2 = 0
Solution
Let 3x = a
2x – 27(2–x) = 6
6a2 + a – 2 = 0 (Factorization)
2x – 27. =6
a = or a = (Ignore a = )

Let 2x = a
x
When 3 =
a– = 6a
x
lg 3 = lg a2 – 6a – 27 = 0
x lg 3 = lg , (a – 9) (a + 3) = 0 ( Factorization)
a = 9 or a = -3 (Ignore a = - 3)
x=
x = - 0.63 (2 d.p) Now,
Therefore, x = - 0.63 2x = 9
x lg 2 = lg 9 (Take log on both sides)
3. Solve for x, given 33x – 3(x + 1) – 3x + 3 = 0 x=
x = 3.12 (2.d.p)
Solution
33x – 3x.3 – 3x + 3 = 0
5. Solve the equation = 3 and
Let a = 3x
a3 – 3 a – a + 3 = 0 approximate the answer to three significant
a3 – 4 a + 3 = 0 figures.

Baffour Ba Series, Further Mathematics for Schools Page 492


Solution √

b. √
= d. =8
=3
x
5 – =3×2 Involving Cubic Equations
5x – =6 When the equation become a cubic equation
5x – –6=0 after substitution, three answers are expected
5x – –6=0 but not negative answers. Therefore, ignore
negative answers.
Let 5x = a To the positive answer, use indices when the
the bases are the same and use logarithm (on
a– –6=0
both sides) when the bases are not the same, to
a2 – 1 – 6a = 0 find the value of the involving variable.
a2 – 6a – 1 = 0 (Quadratic formula)
a = 3 + √ or a = 3 – √ (Ignore negative answer) Worked Examples
a=3+ √ 1. Find the truthset of 23x – 6(22x) + 11(2x) – 6 = 0

⇒ 5x = 3 + √ Solution
log 5x = log (3 + √ ) 23x – 6(22x) + 11(2x) – 6 = 0
x log 5 = log (3 + √ ) Let 2x = a
( √ ) a3 – 6a2 + 11a – 6 = 0
x= (a – 1) (a – 3) (a – 2) = 0
x = 1.1299 a = 1 or a = 3 or a = 2
x = 1.23 (3 s.f) 2x = 1 or 2x = 3 or 2x = 2

Exercises 17.15 2x = 1
A. Find the solution to 2 decimal places 2x = 20
where appropriate x =0 or
1. 5x + 125( ) = 30
2. 3( ) + 9( ) = 28 2x = 3 (Different bases)
x
3. 4 – 3( )=8 lg 2x = lg 3 (Take log on both sides)
x
4. 2 – 6( )=6 x lg 2 = lg 3
2x x
5. 2 + 4(2 ) – 32 = 0 x= 2 or
6. – 12( ) + 27 = 0
2x = 2
Challenge Problems
x=1
1. Find the truth set of 4x + 2 = 3(2x) where x R
Truth set is x = 0 or x = 2 or x = 1
2. Solve :
2. Solve 9(33x) – 91 (32x) + 91 (3x) – 9 = 0
a. = 125 c. = 0.125

Baffour Ba Series, Further Mathematics for Schools Page 493


Solution B. Solve :
9(32x) – 91 (32x) + 91 (3x) – 9 = 0 1. 53x – 52x + 1 – 13 (5x) – 7 = 0
Let a = 3x 2. 33x + 4 (32x) + 5 (3x) + 2 = 0
9a3 – 91a2 + 91a – 9 = 0
3. 53x – 9(52x) + 15 (5x) + 25 = 0
. /(a – 9) (a – 3) = 0
4. 83x + 4(82x) – 11 (8x) + 6 = 0
a = or a = 9 or a = 3
3x = or 3x = 9 or 3x = 3 Substitution
Sometimes the estimated value of the log of a
3x = or 3x = 32 or 3x = 3 number may be given to simplify or evaluate
3x = 3 – 2 or 3x = 32 or 3x = 3 other logarithmic expressions. In such cases:
x = -2 or x = 2 or x = 1 I. Express the given log or simplified as an
The truth set is x = -2 or x = 2 or x = 1 exponent or a product of the number whose
estimated value has been given
3. Solve 23n + 2 – 7 (22n + 2 ) – 31 × 2n – 8 = 0 II. Apply the law: loga y = loga x, if necessary.
III. Substitute the estimated value into the
Solution expression and simplify
23n + 2 - 7 (22n + 2 ) – 31 × 2n – 8 = 0
23n . 22 – 7(22n . 22) – 31 × 2n – 8 = 0 Worked Examples
4(23n) – 28 (22n) – 31 × 2n – 8 = 0 Given that log 2 = 0.3010, find without using
tables or calculators the values of;
Let a = 2n i. log 8 ii. log 8
4a3 – 28a2 – 31a – 8 = 0
. / (a – 8) = 0 Solution
i. log 8 = log
a= or a = 8
= 3 log 2,
a=8 (Ignore negative answer)
Substitute log 2 = 0.3010
n
2 =8 = 3 × 0.3010
2n = 23 = 0.9030
n=3
The truthset is n = 3 ii. log 80 = log ( 8 × 10)
= log 8 + log 10
Exercises 17.16 = log 23+ log 10
A. Find the truthset of the following: = 3 log 2 + log 10
1. 33x – 7 (32x) + 15 (3x) – 9 = 0 = 3 × 0.3010 + 1
2. 43x – 7 (42x) + 8 (4x) + 6 = 0 = 1.9030

3. 4(23x ) – 13 (22x) + 11 (2x) – 2 = 0 2. Given thatlog 3 5 = 1.465, evaluate without


3x 2x x
4. 7 + 2 (7 ) – 7 – 2 = 0 using tableslog 3 25 + log 3 15

Baffour Ba Series, Further Mathematics for Schools Page 494


Solution ⇒ × 0.9358
log 3 25 + log 3 15
= 0.4679
= log 3 52 + log 3 (3 × 5)
= 2log 3 5 + log 3 3 + log 3 5
2. If log x = 0.5 and log y = 1.5, find x + y

Substitutlog 3 5 = 1.465
Solution
⇒2 × 1.465 + 1 + 1.465
log10 x = 0.5 and log10 y = 1.5
= 5.395
x= and y =

x+y= +
3. If log 5 = 0.6990, find log = 34.8

Solution 3. Given that log 6 = 0.778, find without using



log = log( ) tables or calculators the value of log 600.
= log( )
Solution
= log 5, log 600 = log (6 × 100)
= log 6 + log 100
Substitute log 5 = 0.6990 = log 6 + log
= 0.6990 = log 6 + 2 log 10
= 0.4660 = log 6 + 2(1)
= log 6 + 2
Some Solved Past Questions Substitute log 6 = 0.778
1. If log x 3 = 0.5283 and log x7 = 0.9358, find 0.778 + 2 = 2.778
logarithm to base x of ;
i. 27 ii. √ 4. Given that log 10 2 = 0.3010 and log 10 5 =
0.6990, evaluate log 10 50 – log 10 40
Solution
log x3 = 0.5283 and logx7 = 0.9358,
Solution
i. log x 27
log 10 50 – log 10 40
= 3 log x33
= log10 (5 × 10) – log10 (4 × 10)
= 3log x3
= log10 (5 × 10) – log10 (22 × 10)
But log x3 = 0.5283
= log10 5 + log 10 10 – 2 log10 2 + log10 10
⇒ 3 × 0.5283= 1.5849
But log10 2 = 0.3010 and log10 5 = 0.6990
ii. log x√ ⇒ (log5 + log10 ) – (2 log 2 + log 10)

= log x = (0.6990 + 1) – (2 × 0.3010 + 1)
= log x7 = 1.6990 – 1.6020
= 0.097
But log x7 = 0.9358

Baffour Ba Series, Further Mathematics for Schools Page 495


5. Given that log 9 = 0.9542, find the value of 8. Given that log 10 2 = 0.3010 and log 10 3 =
log 0.009 0.4771, find without using tables or calculator
log 10 √ 8
Solution
log 0.009 C. If log5 2 = 0.431 and log5 3 = 0.682, find
= log ( 9 × ) the values of the following;
= log 9 + log 10 1. log5 6 2. log5 27 3. log5 8
= log 9 + (-3) log 10 10
4. log5 12 5. log √ 6. log5. /
= (-3) log 10 10 + log 9
= – 3 + 0.9542 7. log5. / 8. log5 100 9. log5 1.5
= ̅ . 9524
Involving Change of Base
6. Given that log 10 2 = 0.3010 and log 10 3 = Given log to simplify using change of base:
0.4771, evaluate log 10 0.24 I. Find the common factor of the base (a) and
the number (b) to obtain c
Solution
II. Express the logarithm of b to base c to the
log 10 0.24
logarithm of a to base c as a common ratio to
= log 10 . /
complete the process. i.e. loga b =
= log 10 24 – log 10 100
III. Simplify or evaluate (as stated) where
= log 10 (23 × 3) – log 10 (102)
possible.
= log 10 23 + log 10 3 – log 10 102
= 3 log 10 2 + log 10 3 – 2 log 10 10
Proof
= 3 (0.3010) + (0.4771) – 2 (1)
For y = logb M,
= - 0.6199
⇒by = M in exponential form .
(Take base – a logarithm of both sides)
Exercises 17.17
It follows that loga (b y) = logaM
A. Given that log 2 = 0.3010 and log3 =
( Apply power property of logarithms)
0.4771, find the values (correct to 4 s.f)
y logab = loga M,
1. log √ 3. log 18
2. log 1.5 4. 2 log 21 – log 98 y= (Divide by )

5. Given that log 5 = 0.6989, find correct to But logb M = y


four places of decimals, the value of log 40
logb M =

6. Given loga3 0.48 and loga5 1.72, evaluate


Worked Examples
loga 45 without the use of a calculator
1. Evaluate log9 27

7. If log52 = 0.431 and log53 = 0.682, find the Solution


value of log5. / + 2 log 5. / – log 5 . / log9 27= = =

Baffour Ba Series, Further Mathematics for Schools Page 496


2. Simplify log8 92 1. lga 64 = 2. lg4 x =
3. lg3 34x = 18 4. log 10x = – 3
Solution
5. =5
log 892 = = =

Solution
Exercises 17.18
1. lga 64 =
A. If f(x) = 2 log 5 x , evaluate:

64 = ( )
1) f(5) 2) f(25) 3) f. / 4) f(√ )
B. Use the change of base method to simplify 642 =
the following; ( ) =
1. log 6 45 2. log2 30 3. log4 128 ( ) =
4. log 1272 5. log8 2 6. log30 100 ( ) =
2
4 =a
C. From the change of base formula, a = 16
evaluate the following:
1. log 2 7; 2. log3 0.04;
2. lg4 x =
3. log5 3; 4. 3 log3 2 − log3 4 + log3
x=
Logarithmic Equations x=( )
A logarithmic equation is a logarithm statement x=
that is equated or contains an equal sign. In x=
other words, it is an equation involving
x=
logarithms.

To solve logarithmic equation is to find the 3. lg3 34x = 18


value or values of the variable that make the 34x = 318
statement or equation true. 4x = 18
x=
Simple Logarithmic Equations
It is an equation that contains a single logarithm
with an unknown variable. The purpose is to 4. log 10x = – 3
find the value of the unkown variable that 10x =
makes the equation true. Express the given x = -3
logarithmic equation as an exponential
equation, and equate exponenents where 5. =5
necessary.
( ) =5
2
Worked Examples x =5
Solve the following equations: x= ⁄

Baffour Ba Series, Further Mathematics for Schools Page 497


6. Solve log8 (x – 5) = without using C. Solve the following:
1. log9 x9 = 9 3. logx 21 = 7
calculators.
2. logx 2 = 10 4. log81 x = -1
Solution
Complex Logarithmic Equations 1
log8 (x – 5) =
A complex logarithmic equation is the one that

x–5=8 contains more than one logarithm with
( ) =8 variable(s), usually containing addition and
( ) =8 subtraction.
( ) = 64
( ) = There are two methods for solving equations
x–5=4 involving logs:
x=4+5 Method 1
x=9 1. Get a single log on both sides of the
equation.
7. Find x when 6 log (10x) = 30 II. Equate the expressions and solve for the
value of the variable i.e. write the equation in
Solution the form: logb x = logb y ⇒x = y and solve .
log (10x) = 30
Method 2
log (10x) = I. Get a single log in the equation,
log (10x) = 5 II. Change from logarithmic form to
10x = 105 exponential form. i.e. write the equation in the
x= form: log b a = c as a =
x = 10000
Note:
1. Make sure that all logs have the same base.
Exercises 17.19
If necessary, use the change of base law to
A. Find the value of the variable:
1. 2 = lg3 x 5. lga 125 = -3 write logs to the same base
2. 4 = lga 625 6. lg 3 x = - 4
2. Logs are defined for only positive numbers.
3. 3 = lg2 x 7. lg 9 x =
Therefore reject any solution that give rise to
4. lg 4 x = 8. lg x2 = - 4 log (negative number) in the original equation

B. Solve the following without calculators Worked Examples


1. log2 (x – 5) = 4 4. = 13 1. Solve log 2 x = 3 – log 2 (x – 2)
2. log3 (x – 4) = 2 5. 10 = 5(210).
x

Solution
3. log √ = 6. log6 (4x + 8) = 2
Method 1

Baffour Ba Series, Further Mathematics for Schools Page 498


log 2 x = 3 – log 2 (x – 2) –
= 16
log 2 x + log 2 (x – 2) = 3
3x – 1 = 16(3x + 1)
log 2 x (x – 2 ) = 3
3x – 1 = 48x – 16
log 2 (x2 – 2x ) = 3
48x – 3x = 16 – 1
=
45x = 15
8
8 x=
( )( )
or Some Solved Past Questions
or (Reject x = – 2) 1. Solve for x in 3 log x + log 3 = log 81
Therefore, x = 4
Solution
3 log x + log 3 = log 81
Method 2
log x3 + log 3 = log 81
log 2 x = 3 – log 2 (x – 2)
log 3x3 = log 81
log 2 x + log 2 (x – 2) = 3
⇒ = 81
log 2 x (x – 2) = log 2 8
log 2 ( ) = log 28 = 27
x = 33
3
8
x=3
8
( )( ) 2. Solve log (8x + 1) – log (2x + 1) = log (x + 2)
or
or Solution
Reject x = – 2. Therefore, x = 4 log (8x + 1) – log (2x + 1) = log (x + 2)

2. Solve log10 x2 + log10 2 = log10 50 log10. / = log10 (x + 2)


=x+2
Solution
8 = (x + 2) ( )
log10 x2 + log10 2 = log10 50
log10 (2x2) = log10 50 8x + 1 = x(2x + 1) +2(2x + 1)
8x + 1 = 2x2 + x + 4x + 2
= 50
8x + 1 = 2x2 + 5x + 2
= 25
0 = 2x2 + 5x – 8x + 2 – 1
=
2x2 – 3x + 1 = 0
x=5
a = 2, b = - 3 and c = 1
3. Solve log (3x – 1) – log (3x + 1) = log 16 √
Substitute in x =
Solution ( ) √( ) ( )( )
x= ( )
log (3x – 1) – log (3x + 1) = log 16

– x=
log . / = log 16

Baffour Ba Series, Further Mathematics for Schools Page 499


x=
√ 3. log ( x – 7) – log 3 = 2
√ √ 4. log 2 9 + log 2(x + 3 ) = 3
x= or x =
5. log x 81 – 0.5 = log x 27
x = 1 or x =

3. Find the value of x if log4 16 = logx 36 C. Find the value of x


1. log x – 1 = log (x – 9)
Solution 2. log (2x + 1) = 1 – log x
log416 = logx 36 3. log2 (x + 2) + log2 (x – 2) = 5
log442 = logx 36 4. log2 (2x – 5) + log2 (x - 2) = 4
2 log44 = logx 36
2 = log D. Sove the following :
⇒ = 36 1. log5 3(x + 5) + log5 (x – 1) = log5 8
x= 6 2. log a (x – 6) + log a (x – 4) = loga x
x=6 3. log ( ) – log (3 + x) = log (3x – 2)
4. =1
( )
4. Solve lg3 (1 – x) = lg3 (x + 16 – x2)
5. =5
Solution
lg3 (1 – x) = lg3 (x + 16 – x2) Challenge Problems
(1 – x) = x + 16 – x2 1. Solve the equations:
(1 – x) – (x + 16 – x2) = 0 a. log3 (x – 2) = log3 27 – log3(x – 4) –
1 – x – x – 16 + x2 = 0
b. log2 (x + 3) = log2 (x – 3) – log3 9 –
-2x – 15 + x2 = 0
x2 – 2x – 15 = 0 2. Evaluate without using tables
(x + 3) (x – 5) = 0 (a) log10 27 ÷ log10 3;
x = -3 or x = 5 (b) (log10 16 − log10 2) † log10 2.

Exercises 17.20 3. Using properties of logarithms where


A. Solve the following equations: possible, solve for x the following equations:
1. lg x + lg (3x + 1) = 1 1. log + 2 log – log = 1 + log x;
2. lg8 (x + 5) + lg8 (3x – 1) = 2
3. lg(2x + 3) + lg (3x – 1) = 1 2. 2 log10 6 − (log10 4 + log10 9) = log10 x.
4. lg2 x – lg2 (x + 1) = 3 3. 3 log 3 + log 16 – log 3 = log x
5. log3 x – log3 4 = log3 2

B. Solve the following equations: Complex Logarithmic Equations 2


1. log2 (x + 2) – log2 x = 3 This type of logarithmic equations contains one
2. log (2x – 1) – log 5 = 1 of the following:

Baffour Ba Series, Further Mathematics for Schools Page 500


1. Exponent of a variable 2. Solve log √ = √log
2. Root signs
Solution
Apply the laws of logarithms where necessary log √ = √log
to obtain find the truth set of the equation. ⁄
log = √log

Worked Examples log x = √log


1. Solve the equation log (x2) = (log x)2 without
. log / = (√log )
tables or calculator.
(log ) = log x
Solution (log ) = 4 log x
Method 1 (log ) – 4 log x = 0
log (x2) = (log x)2 log x ( log x – 4 ) = 0
log (x × x ) = (log x)2 log x = 0 or log x – 4 = 0
log x + log x = (log x)2 log x = 0 or log x = 4
2 log x – (log x)2 = 0 x = 100 or x = 104
log x (2 – log x ) = 0 x = 1 or x = 10000
log x = 0 or 2 – log x = 0
log x = 0 or log x = 2 ( )
3. Solve =1
x = 100 or x = 102
x = 1 or x = 100 Solution
( )
=1
Method 2
log (x2) = (log x)2 lg2 (9 – 2x ) = 3 – x
log (x × x ) = (log x)2 9 – 2x = 23 – x
log x + log x = (log x)2 9 – 2x =
9 – 2x =
Let log x = a
⇒ a + a = a2
Let y = 2x
2a = a2
a2 – 2a = 0 9–y=
a(a – 2) = 0 9y – y2 = 8
a = 0 or a – 2 = 0 y2 – 9y + 8 = 0
a = 0 or a = 2 (y – 8) (y – 1) = 0
y = 8 or y = 1
⇒log x = 0 or log x = 2
x = 100 or x = 102 ⇒ 2x = 8 or 2x = 1
x = 1 or x = 100 2x =23 or 2x = 20
x = 3 or x = 0

Baffour Ba Series, Further Mathematics for Schools Page 501


4. Solve log , log ( )- = 1 = 2 – 6 (24)
= 2 – 144
Solution
x2 – 40x = - 144
log , log ( )- = 1
x2 – 40x + 144 = 0
, log ( )- = 3
(x – 36) (x – 4) = 0
log ( )=3–1 x = 36 or x = 4
log ( )=2
x 2
2 –7=3 Exercises 17.21
2x – 7 = 9 A. Solve the following equations for x:
2x = 9 + 7
1. log √ = √log 2. log (log x) = 2
2x = 16
2x = 24 3. log (x3) = (log x)2 4. log √ = √log
x=4 5. log √ =2 6. √ = 108

5. Solve log ( )=2 B. Solve:


1. 4 log . / + log . / = 2 log x
Solution
2. 2 log x – 2 log (x + 1) = 0
log ( )=2
2
x – 2x – 65 = (5 – x )2 3. log x =
x2 – 2x – 65 = 25 – 10x + x2 4. log ( ) – 3 log (4 – x) = 0
10x – 2x = 65 + 25 5.
( )
=3
8x = 90 ( )

x=
C. Solve:
1. log ( ) + log ( )=1
6. Solve log . √ /=0 2. log x + =2
3. log7 x + log8 7 = 2.5
Solution 4. log ( ) – log ( )=1
log . √ /=0 5. √log + 2 log √ = 2
. √ / = 100 6. log7 2 + log 49 x = log √

. √ /=1
D. Solve:
√ = 1. log16 x + log4 x + log 2 x = 7
⁄ 2. log4(x + 2) . logx 2 = 1
( ) =
3. log x2 – 3 log x = log x2 – 4
( ) = . / 4. (log3 x)2 – log3 x3 + 2 = 0
5. log(log x) = log (log x3 – 2)
= ( )

Baffour Ba Series, Further Mathematics for Schools Page 502


E. Solve each, using change of base: 2. Solve the pair of simultaneous equations:
1. log2 x = log4 (x + 6) log(y – x) = 0 and 2 log y = log (21 + x)
2. log2 (x – 1) = log4 (4x – 7)
3. log3 x + 3 logx 3 = 4 Solution
4. log4 x + 2 logx 4 = 3 log(y – x) = 0 and 2 log y = log (21 + x)
5. log5 x – 1 = 6 logx 5 log(y – x) = 0 …………………(i)
6. 4 logx 2 = 2 log2 x + 3 2 log y = log (21 + x)…………(ii)

Involving Simultaneous Equations From eqn (i);


Two or more logarithmic equations involving y – x = 100
two different variables are solved y – x = 1 ………………………(1)
simultaneously either by method of substitution
or elimination. From eqn (ii);
log y2 = log (21 + x)
Worked Examples y2 = 21 + x ……………………(2)
1. Solve the simultaneous equations:
log (5x – y ) = log 9 and log (3x + 2y) = log 8 From eqn ( 1);
x=y–1
Solution
log (5x – y ) = log 9………………..(1) Put x = y – 1 in eqn (2);
log (3x + 2y) = log 8 ………………(2) y2 = 21 + (y – 1)
y2 = 21 + y – 1
Divide both sides by log y2 – y – 20 = 0
5x – y = 9……………(1) (x – 5) (x + 4) = 0
3x + 2y = 8………….(2) x = 5 or x = - 4

eqn (1) × 2
Put x = 5 in eqn (1);
10x – 2y = 18………(3)
y–5=1
eqn (2) + eqn (3) y=6
(3x + 10x) + (2y – 2y) = 8 +18
13x = 26 Put x = - 4 in eqn (1);
x=2 y – (- 4) = 1
y +4=1
Put x = 2 in eqn (1) y = -3
5(2) – y = 9 The truth sets are (5, 6) and (- 4, -3)
10 – y = 9
y = 10 – 9 3. Solve the simultaneous equation:
y=1 ab = 52 and log4 a – log4 b = 3
Therefore, x = 2, y = 1

Baffour Ba Series, Further Mathematics for Schools Page 503


Solution From equation (ii);
ab = 52 ……………………....(1) log2 y – log2 x = 4
log4 a – log4 b = 3 …………..(2) log2 . / = 4
= 24
From equation (2);
log4 . / = 3 = 16
y = 16x………………….(2)
= 43
= 64 eqn (1) = eqn (2);
a = 64b……………………(3) 2x + 3 = 16x
16x – 2x = 3
Put a = 64b in eqn (1); 14x = 3
64b2 = 52 x=
2
b =
b2 = Put x = in eqn (1);

b=. / y = 2. / + 3
√ y= +3
b=
7y = 3 + 21

7y = 24
Put b = in eqn (3);
y=

a = 64 . / (x, y) = . , /
a = 16 (√ )
√ Exercises 17.22
(a, b ) = . (√ ), /
Solve the pair of equations;
1. log 2(2x + y) = 3and log 2 (3x – 4y) = 0
4. Solve these equations simultaneosly
2. log3 (3x – y) = log3 (y + 1) and log3 2 + log 3
8y = 42x + 3 and log2 y = log2 x + 4
(x + y) = 2
Solution 3. log x2 = log y and log (2x + y) – log 3 = 0
8y = 42x + 3…………………(i)
4. log x + log y = 1 and y = 2x + 1
log2 y = log2 x + 4………….(ii)

B. Solve the pair simultaneously:


From eqn (i);
1. log x + log y = 2
22y = 22(2x + 3)
x – y = 20
2y = 2(2x + 3)
2. log x + log y = l
2y = 4x + 6
log (10x ) – log y = 2
y = 2x + 3…………….….(1)

Baffour Ba Series, Further Mathematics for Schools Page 504


3. log x + log y = 3 Let y = f(x)
2 log x – 2 log y = -1 f(x) = lg5 3x – 2
4. logx (y – 18) = 2
logy (x + 3) = Exercises 17.23
Find the logarithmic function from each of
Logarithmic Functions the relation;
Let y = log a x, where a > 0, then by definition, 1. x = 5y 2. x = 4y 3. x = 10y
x = ay. This can be expressed in function 4. 81x = 3y 5. x = 32 (2y) 6.375x = 125(5y)
notation as f(x) = ax, called exponential function
of x and it is the inverse of the logarithm Word Problems Involving Logarithms
function f(x) = log a x, To solve an exponential or logarithmic word
problems, convert the narrative to an equation
Worked Examples and solve the equation.
1. Find the logarithmic function from the
relation, 2x = 6y Worked Examples
1. The population of a certain country in a
Solution certain year was ,,000 and increases at a rate of
2x = 6y 4 percent per anum. If the population after t
lg 2x = lg 6y years is p (1 + r)t, where p is the initial
lg 2x = y lg 6 population and r is the rate of increase. Find;
lg6 2x = y lg6 6 (change of base) i. the population after 10 years.
lg6 2x = y ii. when the population will double

Let f(x) = y Solution


f(x) = lg6 2x i. Population after 10 years = p (1 + r)t
p = 12890000, r = and t = 10 years
2. Find the logarithmic function from the
Population after 10 years;
relation, 3x = 25(5y)
= 12890000 (1 + )10
Solution = 12890000 (1.04)10
3x = 25(5y) = 19, 060, 000
lg 3x = lg 25(5y)
lg 3x = lg 52 (5y) Method 2 (Using logarithm)
lg 3x = lg 52 + log 5y
lg 3x = 2 lg 5 + y log 5 Number Logarithm
lg5 3x = 2 lg5 5 + y log5 5 (change to base five) 12890000 7.1103
(1.04)10 10 × 0.0170
lg5 3x = 2 + y 12890000 × (1.04)10 7.1103 +
y = lg5 3x – 2 0.1700
1.906 × 107 7.2803

Baffour Ba Series, Further Mathematics for Schools Page 505


ii. Let t years be the time the population will To find the rate for 1 year:
double. Then Step 1: Substitute 1 for t in the equation
2p = p (1.04)t
2,500 = 1,000 . / to derive r:
2 = (1.04)t
( )
log 2 = log (1.04)t 2,500 = 1,000 . /
log 2 = t log (1.04) ,
=. /
t= ,

t= 2.5 = . /
t = 17.71 log 2.5 = log . /
t 18 years
Therefore, the population will double in 18 log 2.5 = 360 log . /
years = log . /

2. If you invested Ghȼ1,000 in an account 0.0011 = log . /


paying an annual percentage rate (quoted rate) =1+
compound daily (based on a bank year of 360 –1=
days) and you wanted to have Ghȼ2,500 in your
360 ( – 1) = r
account at the end of your investment time,
r = 0.9129
what interest rate would you need if the
r 91.29%
investment time were 1 year, 10 years?
To find the rate for 10 year:
Solution
Substitute 10 for t in the equation
Use the formula
2,500 = 1,000 . / to derive r:
2,500 = 1,000 . /
( )
where Ghȼ2,500 is the balance at the end of a 2,500 = 1,000 . /
certain time period, Ghȼ1,000 is the beginning ,
investment, t is the number of years, and r is =. /
,
the annual percentage rate. The annual rate of 2.5 = . /
r% is converted to a daily interest rate since the
compounding is daily (360 times per year). log 2.5 = log . /
log 2.5 = 3600 log . /
Take the annual interest rate of 4% and divide
by 360 to obtain the daily interest rate. The = log . /
exponent is 360t because there are 360
compounding periods in every year. Therefore, 0.00011 = log . /
360t represents the number of compounding
periods during t years. =1+

Baffour Ba Series, Further Mathematics for Schools Page 506


–1= Year Population
(billions)
360 ( – 1) = r
1930 2.070
r = 0.0912 1940 2.295
r 9.12% 1950 2.500
1960 3.050
Exeercises 17.24 1970 3.700
1. Use the compound interest formula: 1980 4.454
1990 5.279
A=p. / for t by using logarithms. 2000 6.080
A sum of GhȼP investedar r% compound 2010 6.215
interest payable annually, accumulates to an
Many experts believe that the world can
amount P . / at the end of n years. If a
support about 10 billion people. Predict when
sum of Ghȼ60 is invested at compound interest the population will reach 10 billion.
at 5% per anum , calaculate the least number of
complte years it takes for the sum to amount to 6. The illicit use of methamphetamine has
more than Ghȼ60.00 Ans 23 yrs skyrocketed in rural areas. The chemical by-
products of creating the drug are a big
2. Use the compound interest formula to environmental hazard, and toxic spill cleanup
determine how long it will take for a sum of crews are often called in to clean up abandoned
money to double if it is invested at a rate of 6% meth labs. The table below shows the number
per year compounded monthly. of meth labs cleaned by toxic spill crews in
Washington State alone.
3. If Ghȼ1,000 is invested at a rate of 12% per
year compounded quarterly, what is the years Population (billions)
principal after one year? 1995 42
1996 146
4. A person invests Ghȼ 4000 in an account at 1997 208
5% interest compounded annually. Let V = f(t) 1998 350
represent the value (in cedi) of the account after 1999 792
"t" years or any fraction thereof. 2000 1458
a) The equation for "f" is f(t) = 2000 (1.05t).
b) The V-intercept is (0, 2000). a) Find an equation for "f."
When will the balance be Ghȼ3000? b) Use "f" to predict in which year the number
of meth labs cleaned by toxic spill crews will
5. The equation f(t) =1.197(1.0163t) models the equal the number of households in
world population (in billions) at "t" years since Washington. Assume there are 2.2 million
1900. households in Washington.

Baffour Ba Series, Further Mathematics for Schools Page 507


7. A person drinks a cup of coffee. We assume Type 1: y = ax2 + b.
that the caffeine enters his bloodstream Method
immediately, and that there was no caffeine in Let X = x2, so that y = aX + b and hence obtain
his bloodstream prior to drinking the cofee. The a straight line by plotting y against X.
half-life of caffeine in a person's bloodstream is
about 6 hours. A cup of coffee contains about Type 2 : y = ax2 + bx
100 milligrams of caffeine. Method
a. Let f(t) represent the number of milligrams of Here, consider the equation in the equivalent
caffeine in the person's bloodstream at "t" hours form: = ax + b so that, by letting Y = , gives
after drinking the cup of coffee. Y = ax + b, a straight line will be obtained by
Find an equation for "f." plotting Y against x.
b. The person drinks the cup of coffee at 8am
and goes to bed at 11 pm. Use "f" to predict the Note:
amount of caffeine in his bloodstream when he If one of the sets of readings taken in the
goes to bed. experiment happens to be (x, y) = (0, 0, ignore
it in this example.
Experimental Laws
Expressions Reducible to Linear Form Type 3: xy = ax + b.
Experimental laws which are not linear can Method
sometimes be reduced to linear form. To do this Two alternatives are available here as follows:
1. Rewrite the equation in terms of new (a) Letting xy = Y , giving Y = ax + b, we
variables so that there is a straight line could plot a graph of Y against x.
relationship between the new variables.
(b) Writing the equation as y = a + , we could
2. From this straight line graph, take the usual
two sets of readings (x1, y1) and (x2, y2) let = X, giving y = a + bX, and plot a graph
3. Then substituted them into the form of the of y against X.
experimental equation which occurs
immediately after taking logarithms of both Type 4 : y = axb
sides. Method
This kind of law brings in the properties of
Note logarithms since, if we take logarithms of both
It will not matter which base of logarithms is sides (base 10 will do here), we obtain
being used since logarithms to two different log10 y = log10 a + b log10 x.
bases are proportional to each other anyway.
The logarithmic graph paper does not, Letting log10 y = Y and log10 x = X, we have
therefore, specify a base. Y = log10 a + bX,
so that a straight line will be obtained by
plotting Y against X.

Baffour Ba Series, Further Mathematics for Schools Page 508


Type 5: y = abx v. Solve for a and b (the slope of the line) in the
Method simultaneous equations:
Here again, logarithms may be used to give log10 y1 = log10 a + b log10 x1,
log10 y = log10 a + x log10 b log10 y2 = log10 a + b log10 x2.
Letting log10 y = Y , we have
Y = log10 a + x log10 b, If it is possible to choose readings which are
which will give a straight line if we plot Y powers of 10, so much the better, but this is not
against x. essential.

Note: Type 2
In all 5 of the above examples, it is even more 2. y = abx.
important not to try to read off the gradient and Method
the intercept from the graph drawn. As before, i. Taking logarithms (base 10),
take two sets of readings for x (or X) and y (or log y = log (abx)
Y ), substitute them in the straight-line form of log y = log a + log bx
the equation and solve two simultaneous linear log10 y = log10 a + x log10 b.
equations for the constants required. ii. Plot a graph of y against x with y on a
logarithmic scale and x on a linear scale.
Ploting the Logarithm Graph iii. Estimate the position of the best straight
Type 1 line.
1. y = axb iv. Read off from the graph two sets of co-
Method ordinates, (x1, y1) and (x2, y2), as far apart as
i. Taking logarithms (base 10), possible.
log10 y = log10 a + b log10 x. v. Solve for a and b the simultaneous equations:
ii. Plot a graph of y against x, both on log10 y1 = log10 a + x1 log10 b,
logarithmic scales. log10 y2 = log10 a + x2 log10 b.
iii. Estimate the position of the “best straight If it is possible to choose zero for the x1 value,
line”. so much the better, but this is not essential.
iv. Read off from the graph two sets of co-
ordinates, (x1, y1) and (x2, y2), as far apart as
possible.

Worked Examples
1. Below is a table of values for y = 3x2

x 5 10 15 20 25 30 35
y 75 300 675 1200 1875 2700 3675

Draw a straight line graph from the table and use your graph to obtain the slope of the line.

Baffour Ba Series, Further Mathematics for Schools Page 509


Solution
y = 3x2
x 5 10 15 20 25 30 35
y 75 300 675 1200 1875 2700 3675

lg x 0.699 1.0 1.176 1.301 1.398 1.477 1.544


lx y 1.875 2.477 2.829 3.079 3.273 3.431 3.565

Plot log x on x- axis agsinst log y on y – axis


log y

0 0.5 1.0 1.5 2.0 log x


From the graph, the gradient of the line is:
m= = = =2

Baffour Ba Series, Further Mathematics for Schools Page 510


2. The relation between the variables x and y is of the form y = axb. Fom an experiment, the
approximate values of y for the various value of x are given below.

x 2 4 6 8 10 12 14 16 18 20
y 14 56 126 224 350 504 686 895 1132 1400

From the graph of log x, find the values of the constants a and b to two significant figures.

Solution

x 2 4 6 8 10 12 14 16 18 20
y 14 56 126 224 350 504 686 895 1132 1400

log x 0.301 0.602 0.778 0.903 1.000 1.079 1.146 1.200 1.255 1.300
log y 1.146 1.748 2.100 2.350 2.540 2.70 2.836 2.952 3.054 3.146
log y

0.2 0.4 0.6 0.8 1.0 1.2 1.4


log x

From the graph, the gradient of the line is:


m= = = = 2.01

From y = axb

Baffour Ba Series, Further Mathematics for Schools Page 511


log y = log (axb)
log y = log a + log
log y = log a + b log x
log y = b log x + log a
log y = b log x + log a
Compared to Y = mX + c,
log y = Y, m = b, log x = X and log a = c, where m is the gradient and c is the intercept on y – axis.

From the graph, m = b = 2.01

From the graph, the intercept on y axis is c = 0.6


But c = log a = 0.6
⇒ log a = 0.6
a=
a = 3.98

3. Find correct to one significant figure, the values of k and a in the relation, y = kax given by the
following table.

x 1 2 3 4 4.5 5
y 2 7 22 65 112 194

Solution
y = kax
x 1 2 3 4 4.5 5
y 2 7 22 65 112 194

x 1 2 3 4 4.5 5
log y 0.301 0.845 1.342 1.813 2.049 2.288

Baffour Ba Series, Further Mathematics for Schools Page 512


log y
2.5

2.0

1.5

1.0

0.5

1 2 3 4 5 6 x

From the experimental equation 2.3 = 5m + c………………..(2)


y = kax eqn (2) – eqn (1);
lg y = lg ( kax) 2 = 4m
log y = log k + log m = 0.5
log y = log k + x log a
log y = x log a + log k Put m = 0.5 in eqn (1) ;
Y = mX + c 0.3 = 0.5 + c
c = 0.3 – 0.5
⇒ m = log a and c = log k c = - 0. 2

From the graph, For the values of k and a;


when x = 1, y = 0.3 Now, m = log a = 0.5
Substitute in Y = mX + c log a = 0.5
0.3 = m (1) + c a=
0.3 = m + c ……………………..(1) a = 3.162

when x = 5, y = 2.3 Also, c = log k = - 0. 2


Substitute in Y = mX + c log k = - 0.2
2.3 = m (5) + c k=
k = 0.631

Baffour Ba Series, Further Mathematics for Schools Page 513


Exercises 17.25
In these exercises, use logarithmic graph x 0.2 0.4 0.6 0.8 1.4 1.8
paper where possible. y 0.508 0.645 0.819 1.040 2.130 3.420
1. Two variables x and y are related by y = axb.
The following table gives the approximate Use a straight line graph to find approximately
values of y for corresponding values of x. the values of a and b.

x 2 10 20 30 40 45 50 5. The table below gives the pressure, P, and


y 7 16 22 27 32 34 35 the volume, V , of a certain quantity of steam
at maximum density:
i. By drawing the appropriate straight line
graph, determine the values of a and b. P 12.27 17.62 24.92 34.77 47.87 65.06
ii. Use your graph to estimate the values of y V 3390 2406 1732 1264 934.6 699.0
for x = 5 and x = 37.
Assuming that PVn = C, use a straight line graph
to find approximately the values of n and C.
2. Two variables x and y are related by y = kxn.
The following table gives the approximate
6. The coefficient of self induction, L, of a coil,
values of y for corresponding values of x.
and the number of turns, N, of wire are related
by the formula L = aNb, where a and b are
x 5 10 15 20 25 30 40
constants. For the following pairs of observed
y 17 47 87 134 188 246 379 values, use a straight line graph to calculate
approximate values of a and b:
i. By drawing the appropriate straight line
graph, determine the values of k and n. N 25 35 50 75 150 200 250
ii. Use your graph to estimate the values of y L 1.09 2.21 5.72 9.60 44.3 76.0 156.0
for x = 60 and x = 200.
7. Measurements taken, when a certain gas
3. The table below gives the approximate undergoes compression, give the following
values of y for given values of x in an
values of pressure, p, and temperature, T:
experiment.
P 10 15 20 25 35 50
x 1 2 3 4 5 6
T 270 289 303 315 333 353
y 3 6 11 22 45 90

x Assuming a law of the form T = apn, use a


If x and y are related by y = ab , find the values
straight line graph to calculate approximately
of the constant a and b.
the values of a and n. Hence estimate the value
4. The following values of x and y are assumed of T when p = 32
to follow the law y = abx:

Baffour Ba Series, Further Mathematics for Schools Page 514


18 TRIGONOMETRY Baffour Ba Series

Radian Measure ⇒ R = θ × , where θ is angle in


In calculus and advanced trigonometry, angles
degrees and R is the radian
are usually measured in radians.
Summary
One radian is the measure of the angle at the
center of the circle subtended by an arc equal in
To change Multiply by
length to the radius of the circle, Degrees to radians
Radians to degrees
r
 θ r
Worked Examples
r
Convert 2250 to radians

Solution
Radians are often called „circular measure‟ radians = 1800
and are denoted by rads. ⇒R=θ× ,
Let θ = 2250
The number of radians in one complete
revolution is given by the ratio:
radians = 1800
= = 2 radians
⇒ R = 2250 ×
R= radians
Relationship Between Radians and Degrees
One complete revolution = 2 radians = 3600
Some Common angles
0
Thus, 2 radians = 360
radians = 1800 Degree Radians
00 0
300
Note:
You can write = 1800, omitting the word 450
„radian‟ , where means „ radians.‟ 600
0
Mathematically, ≠ 180
900
Changing from Degrees to Radians 1200
Using the relation; 1500
radians = 1800
1800

Baffour Ba Series, Further Mathematics for Schools Page 515


2700 θ = 1200
3600 2
Exercises 18.2
Exercises 18.1 Express in radians, leaving in your
Express each of the following number of answers;
radians in degrees; 1. 720 2. 22.50 3. 1500 4. 2100 5. 2400
6. 1350 7. 2880 8. 1050 9. 4500 10. 3900
1. 2. 3. 4. 5.
6. 7. 8. 9. 10. Changing Radians to Degrees, Minutes and
Seconds
Changing from Radians to Degrees
Using the relation; Worked Example
radians = 1800 If θ = 3, approximate θ in terms of degrees,
⇒θ=R× , where R is angle in radians minutes and seconds.

Solution
Worked Examples
3 radians = 3 × multiply by
1. Convert radians to degrees
171.88730 approximate

Solution 1710 + 0.88730(60‟ ) 10 = 60‟‟

radians = 1800 = 1710 + 53.238 multiply


= 1710 + 53‟ + 0.238 (60‟‟) 10 = 60‟‟
⇒θ=R× , = 1710 53‟ + 14.28‟‟ multiply
1710 53‟ + 14‟‟ approximate
Let R = 1710 53‟14‟‟ approximate

⇒θ= × ,
Changing Minutes and Seconds as Decimal
θ= Degrees
θ = 3000 = . / , and =. / =. /

2. Convert radians to degrees


Worked Examples
Express 19047‟23”as a decimal to the nearest
Solution ten – thousandth of a degree.
Let R =
Solution
⇒θ= × ,
19047‟23” = 190 + . / + . /
⇒θ= × ,
190 + 0.78330 + 0.00640
θ= = 19.78970

Baffour Ba Series, Further Mathematics for Schools Page 516


Exercises 18.3 Worked Examples
A. Express the following in terms of degrees, 1. In the diagram below, a central angle θ is
minutes and seconds, to the nearest second. subtended by an arc 10cm long on a circle of
1. 81.72380 3. 310.62150 radius 4cm.
2. 12.8640 4. 63.1690 i. approximate the measure of θ in degrees.
ii. find the area of the circular sector
B. Express θ in terms of degrees, minutes determined by θ.
and seconds, to the nearest second.
1. θ = 2 2. θ = 4 3. θ = 5 4. θ = 1.5 Solution
i. l = 10cm r = 4cm and θ = ?
C. Find the angle that is complementary to θ. l = rθ
1. θ = 5017‟34‟‟ 3. θ = 32.50 θ= = 2.5
2. θ = 630 4‟ 15‟‟ 4. θ = 82.730
Change θ = 2.5 to degrees
D. Find the angle that is complementary to θ.
1. θ = 480 51‟37‟‟ 3. θ = 136.420 θ = 2.5 ×
2. θ = 1520 12‟ 4‟‟ 4. θ = 15.90 θ = 143.240 4cm
θ 10cm
Length of a Circular Arc ii. A = r 2 θ
If an arc of length l on a circle of radius r
A = (4)2 (2.5)
subtends a central angle of radian measure θ,
then: l = rθ A = 20cm2

From the diagram below; 2. The radius of a circle is 20cm. find the angle
subtended at the center by an arc of length 8 .
l = ×2 r
r
l = rθ θ l Solution
r r = 20 , l = 8 and θ = ?
l = rθ
8 = 20θ
Area of a Circular Sector
If θ is the radian measure of a central angle of a θ= =
circle of radius r and if A is the area of the
circular sector determined by θ, then: 3. The area of a sector of a circle of radius r is
A = r2 θ 30cm2. If the angle subtended at the center of
the circle by this sector is radians, calculate r.
r
θ
r Solution
A = 30 , θ = radians, r = ?

Baffour Ba Series, Further Mathematics for Schools Page 517


30 = r 2 B. 1. In each of the following diagrams
below, find:
30 = r 2
i. the area of the shaded region.
5r 2 = 30 × 6 ii. the perimeter of the shaded region.
r2 =
r 2 = 36 r r
r = 6cm θ θ
r r
Exercises 18.4
1. i. Find the radian and degree measures of the
central angle θ subtended by the given arc of 2. In each of the following, find the length of
length l on a circle of radius r the minor arc and the area corresponding to
ii. Find the area of the sector determined by θ. the minor sector.
a. l = 7cm , r = 4cm b. l = 3cm , r = 0
1200 8cm
2. Find the length of the arc that subtends the 9cm 450
given central angle θ on a circle of diameter d.
ii. Find the area of the sector determined by θ.
a. θ = 500 , d = 16 m b. θ = 2.2 , d = 120cm

3. If a circular arc of length l subtends the Trigonometric functions of Angles


central angle θ on a circle, find the radius of the Trigonometric functions originated historically
circle given; as ratios of the sides of a right triangle. A
i. l = 10cm, θ = 4 ii. l = 3km , θ = 200 triangle is a right triangle if one of its angle is a
right angle (900).
4. The radius of a circle is 12cm. Find the angle
subtended at the center by an arc of length 16 Consider the right triangle below with θ as one
cm. of its acute angle

5. Find the area of the sector of a circle of c


radius 20 cm if the arc of the sector subtends an b

angle of . θ
a
6. The area of a sector of a circle, or radius r is Six ratios that can be obtained using the sides
24 cm2. If the angle subtended at the center of a, b and c are ;
the circle by this sector is , calculate r, the , , , , ,
radius of the circle.

Baffour Ba Series, Further Mathematics for Schools Page 518


These ratios depend on θ, but not the size of the Below is the summary.
triangle sin θ = cos θ = tan θ =

For each θ, the six ratios are uniquely csc θ = sec θ = cot θ =
determined and hence are functions of θ. They
are called Trigonometric functions and are Trigonometric Ratios of 300 and 600
identified as sine, cosine, tangent, cotangent, , The trigonometric ratio of 300 and 600 are
secant and cosecant functions, abbreviated as derived from an equilateral triangle of
sin, cos, tan, cot, sec, and csc respectively. dimensions 2 – units as shown below;
From the above right triangle; H
1. sin θ = = 4. csc θ = =
0 0
30 30
2. cos θ = = 5. sec θ = = 2 2
3. tan θ = = 6. cot θ = = 0 0
60 60 Fig. II
1 A 1
Note By Pythagoras theorem,
1. The domain of each of the six trigonometric 22 = | |2 + 12
functions is the set of all acute angles. | |2 = 22 – 12
2. The values of the six trigonometric functions | |2 = 3
are positive for every acute angle θ. This is
| |=√
because , the sides of a triangle are positive real
The dimensions of fig. II can be shown below:
numbers.
3. The hypotenuse is always greater than H
adjacent or opposite side an hence sin θ < 1,
0 0
cos θ < 1, csc θ > 1, and sec θ > 1 for every 30 30
acute angle θ. 2

2
0 0
60 60 Fig. II
Reciprocal Identities
1 A 1
Consider the following pair of ratios;
1. sin θ = and csc θ = From this figure, the trigonometric ratios of 300
and 600 are obtained as follows:
2. cos θ = and sec θ =

sin 300 = , cos 300 = tan 300 = ,
3. tan θ = and cot θ = √

sin 600 = cos 600 = tan 600 = √
It can be seen that sin θ and csc θ, cos θ and sec csc 300 = 2 sec 300 = √
cot 300 = √
θ and tan θ and cot θ are reciprocals of each √
csc 600 = sec 600 = 2 cot 600 = √
other.

Baffour Ba Series, Further Mathematics for Schools Page 519


Trigonometric Ratios of 450 and 900 Summary Table:
C Ratios of 00, 300, 450, 600 and 900
By Pythagoras 450
theorem, 00 300 450 600 900
| |2 = 12 + 12 1 sin 0 √ √ 1
| |2 = 2
450
cos 1 √ √ 0
| |=√
A 1 B tan 0 √ 1 √ –
The dimension of the figure is shown below; sec 1 √ 2 –

C csc – 2 √ √ 1

450 cot – √ 1 √ 0

1

450 Fig. III Trigonometry Ratios for Any Angle


A 1 B The Unit Circle
From this fig. III, the trigonometric ratios of The unit circle has its center at the origin(0, 0)
450 are obtained as follows; and the length of the radius is 1.
√ Take any point p(x, y), on the circle, making an
cos 450 = = csc 450 = √ angle of Ɵ, from the center.

√ y
sin 450 = = sec 450 = √

·
P ( x, y)
0 0
tan 45 = cot 45 = 1
1 y
0 θ
The trigonometric ratios of 90 are obtained as x x
follows
cos 900 = = 0, csc 900 = 1

√ √
sin 900 = = 1, sec 900 = = undefined

θ
√ cosθ = = x, sinθ = = y, tanθ = , tan θ =
tan 900 = = undefined cot 900 = =0 θ

This very important results indicates that the


These ratios can be used instead of a calculator,
coordinates of any point on the unit circle can
especially when answers are required in the
be represented by p(cos θ, sin θ), where θ is
form of surds.
any angle.

Baffour Ba Series, Further Mathematics for Schools Page 520


y
Both diagrams below represent the unit circle
but using two different notations to describe

·
P ( cos Ɵ, sin Ɵ)
any point p on the circle.

··
0
1 Sin Ɵ ( cos 900, sin 90 )
P ( cos Ɵ, sin Ɵ)
Ɵ
Cos Ɵ
x

· ·
1 Sin Ɵ
0 0
( cos180 , sin 180 ) ( cos 00, sin 00)
θ
Cos Ɵ ( cos 3600, sin 3600)

· ( cos 2700, sin 2700)

··
As the point p rotates, θ changes. These (0, 1)
definitions of cos θ and sin θ in terms of the P (x, y)
1

·
coordinates of a point rotating around the unit y
(-1, 0) Ɵ (1, 0)
circle apply for all values of the angle θ0,
x
Memory Aid: (Christian name, Surname)
= ( cos θ , sin θ ) = ( x, y)
(0, -1)

Note: Using Pythagoras theorem: By comparing corresponding points on both


cos2θ + sin2θ = 1 unit circles, the values of sin, cos, and tan for

Values of Sin, Cos and Tan for 00, 900, 1800,


2700 and 3600
·
00, 900, 1800, 2700 and 3600, can be read
directly without the use of a calculator

(cos 00, sin 00) = (cos3600, sin 3600) = (1, 0) (cos 900, sin 900) = (0, 1)
cos 00 = cos 3600 = 1 cos 900 = 0
sin 00 = sin 3600 = 0 sin 900 = 1
tan 00 = tan 3600 = = 0 tan 900 = ( undefined)
(cos 1800, sin 1800) = (-1, 0) (cos 2700, sin 2700) = (0, -1)
cos 1800 = cos 3600 = -1 cos 2700 = 0
sin 1800 = 0 sin 2700 = -1
tan 1800 = =0 tan 2700 = tan 3600 = (undefined)

Baffour Ba Series, Further Mathematics for Schools Page 521


The x and y axes divide the plane into four
quadrants. Consider the unit circle below, ( cos Ɵ, sinƟ)
y
x y
Ɵ
-Ɵ x
2 1 -y
x- x+
y+ y+
x ( cos (-Ɵ), sin(-Ɵ)
3 4
x- x+ cos θ = x, sin θ = y and tan θ =
y- y-
cos (-θ) = x, sin (-θ) = - y, tan (-θ) = –

cos θ = x and sin θ = y Thus;


Ɵ cos (-θ) = cos θ
tan θ = =
sin (-θ) = - sin θ
By examining the signs of the four quadrants, tan (-θ) = - tan θ
the sign of sin θ, cos θ, tan θ for any value of θ
can be found. Worked Examples
Find without using tables or calculators
Summary of Signs 1. sin (- 600) 2. cos (- 300) 3. tan(- 2250)
y
Solutions
S A √
x 1. sin (- 600) = - sin 600= –
T C √
x 2. cos (-300) = cos 300 =
x 3. tan (- 2250) = - tan 2250 = - tan 450 = - 1
+ tan are ALL positive
1st quadrant: sin cos and
2nd quadrant: sin is positive, cos and tan are Exercises 18.5
y
negative. - 1. tan (-1500) 2. tan (- 2100)
3rd quadrant: tan is positive, sin and cos are 3. cos (-1200) 4. cos (-1350)
negative. - 4. sin (-2700) 6. sin (-1350)
4th quadrant: cos is positive, sin and tan are
y
negative. - Methods for Finding the Trigonometric Ratio
for Any Angle between 00 and 3600
A very useful memory aid, CAST, in the Method 1
diagram above shows the ratios that are I. Draw a rough diagram for the angle in the x –
positive for the angles between 00and 3600. y plane, taking measurement in the
anticlockwise direction from the positive x -
Negative Angles axis
Consider the unit circle showing angles θ and –θ II. Determine in which quadrant the angle lies

Baffour Ba Series, Further Mathematics for Schools Page 522


III. Find its related acute angle to the nearest = – cos 300
horizontal line (x - axis) = –

IV. Use the trigonometric ratios of the related
c. tan (θ – 1800) = tan θ
angle
Eg. tan 2100 = tan ( 2100 – 1800 )
V. Use the CAST diagram below to find the
= – tan 300
signs of the ratios
S A =–
y √

T C
4. In the fourth quadrant, (2700 < θ < 2700),
Note: sin2 A = ( ) etc subtract the given angle , from 3600 to obtain
the related angle. That is, Related angle = (3600
Method 2 –θ). Only cos is positive. That is explained
If θ is the general angle, then below:
1. In the first quadrant, (00 < θ < 900), all the a. sin ( 3600 - θ) = - sin θ
trigonometry ratios are positive e.g. Sin 3000 = sin (3600 – 3000)
2. In the second quadrant, (900< θ < 1800), only = – sin 600= –

sin is positive. That is:


a. sin (180 – θ) = sin θ b. cos ( 3600 – θ) = cos θ
eg. sin 1200 = sin (1800 – 1200) E.g. cos 3000 = cos (3600 – 3000)
= sin 600 = cos 600 =

b. cos (1800 – θ) = – cos θ c. tan ( 3600– θ) = – tan θ


eg.cos 1200 = cos (1800 – 1200 ) E.g. tan 3000 = tan (3600 – 3000)
= – cos 600
= – tan 600= – √
c. tan (1800 – θ) = – tan θ
Summary
eg. tan 1200 = tan (1800 – 1200 )
= – tan 600 900
=√
Sin 1800 - x All
0 0
x
3. In the third quadrant, (180 < θ < 270 ), only
1800 00 3600
tan is positive. That is
a. sin ( θ – 1800) = – sin θ x + 1800 3600 - x
Tan
e.g. sin 2100 = sin (2100 – 1800) Cos
= – sin 300 2700
= –
Worked Examples
1. Find cos 2100, leaving your answer in surd
b. cos ( θ – 1800) = – cosθ
form:
e.g.cos 2100 = cos (2100 – 1800)

Baffour Ba Series, Further Mathematics for Schools Page 523


Solution Method 2
Method 1 Let θ = 3150
I. The diagram for the angle is shown below; 3150 is in the 4th quadrant, where only cos is
positive
Related angle = (3600– θ)
= (3600– 3150) = 450
300 ⇒ 3150 is related to 450
sin 3150 = - sin 450 = –

Related angle Start
0
II. 210 is in the 3rd quadrant 3. Find without using tables, tan 1200,
cos is negative in the 3rd quadrant
III. Related angle is 300 Solution
√ Method 1
IV.  cos 2100 = - cos 300 = –
1200 lies in the 2nd quadrant

Method 2
tan is negative in 300
Let θ = 2100 0
the 2nd quadrant 60
2100is in the 3rd quadrant where only tan is
tan 1200 = - tan 60
positive
=-√
Related angle = (θ – 1800)
= (2100 – 1800) = 300 Related angle
Method 2
⇒ 2100 is related to 300
Let θ = 1200

sin 2100 = – cos 300 = – 1200 is in the 2nd quadrant, where only sin is
positive
2.Find sin 3150, leaving your answer in surd Related angle = (1800– θ)
form: = (1800 – 1200) = 600
⇒ 1200 is related to 600
Solution √
sin 1200 = sin 600 = ,
Method 1
3150 lies in the 4th quadrant as shown
Simplifying and Evaluating Trigonometric
in the diagram below; Expressions
Replace each angle with it‟s ratio and perform
the included operation. If answer is required as
450 Related angle
0 a surd, do so by rationalizing the denominator
45

Worked Examples
1. Evaluate sin 300 + 2 tan 450 without using
sin is negative in the fourth quadrant
calculators or tables
sin 3150 = – sin 450 = –

Baffour Ba Series, Further Mathematics for Schools Page 524


Solution C. Without using calculators, simplify each.
Sin 300 + 2 tan 450 1. tan 600 + tan 2250
= + 2 (1) 2. tan 2850 ( use: 600 + 2250)
3. cos 1350 – cos 600
+2=2 =
4. cos 750 (use 1350 – 600)
5. tan2 2250 – 2 cos 2400
2. Without using tables or calculator evaluate 6. sin2 600 + cos2 450
cos 450 + sin 300.
Inverse of Trigonometric Ratios
Solution It is the reverse process of determining the
cos 450 + sin 300 angle given the value of one of the
√ √ trigonometric ratios. The inverse of a
+ =
trigonometric ratio is written as ( - 1 ). Thus,
3. Without using tables or calculators, simplify sin is pronounced “inverse sine x” or “sine
sin 1350 – sin 3150 inverse x” or “arc sine x”

Consider the triangle below;


Solution
sin 1350 – sin 3150
√ √ √ √ √ 2
–.– /= + = =√ √
θ
2 0 0 1
4. cos 45 + sin 30
√ Ratio Angle
=. / + = + =1 √ √
sin θ = θ = sin
cos θ = θ = cos
Exercises 18.6
A. Find the reference angle or related angle tan θ = √ θ=t n √
given the following measure.
1. 2400 2. 3400 3. 1650 4. 2750 5. -1100 However, a problem arises with the sin-1x, cos-1
x and tan-1 x notation.
6. 7. 8. 9. 10 .
Consider the following equations;
B. Evaluate, leaving your answer in surd
If sin θ = , then θ = sin = …,-3300,
form where necessary.
1. cos 1200 + cos 2250 -2100, 300, 1500, 3900, 5100,…
2. sin2 600 + cos2 450 If tan θ = , then θ = t n 1 =…,-3150,
3. 1 + cos2 30 -1350, 450, 3150, 4050, 6750, …
4. sin 600cos 450 + cos 600 sin 450 Hence, if there is no restrictions on the value of
5. tan 2400– tan 3300 θ, then the equation sin θ = and tan θ = 1 have
6. tan2 300– sin2 600 an infinite number of solutions.

Baffour Ba Series, Further Mathematics for Schools Page 525


This problem is overcome by restricting the ⇒y = tan -1(1.732)
value of θ to the range - 900 ≤ θ ≤ 900. y = 2400 (Third quadrant)

The angles within this range are often called 3. If A and B are acute, tan A = and tan B = ,
“principal values”. Using these restrictions for
find the value of A + B .
θ, we often obtain a single value for our

answer. Thus,sin = 600, not also 1200, as Solution
0 0 0
120 is not in the range -90 to 90 tan A = and tan B = ,
⇒A= . / = 26.56500
Domain and Range of the Inverse
Trigonometric functions B= . / = 18.43500

Inverse ratio Domain Range A + B = 26.56500 + 18.43500


[-1, 1] 0 , 1 = 44.99990
[- , ] = 450
0 , 1
[-1, 1] , , -
4. Evaluate tan ( . /*

Worked Examples
1. Find the principal values of each of the Solution
following; tan ( . /*
i. sin ii. cos iii. t n . /
√ √ Let A = 13
5
⇒ Sin A = ⇒ A
Solution
a
i. sin = sin ( ) = 300 a=√ ( )
a=√
ii. cos = cos ( ) = 450 a = 12

iii. t n . /=-t n . / = - 300


√ √ cos A = (From the diagram )

√ But tan A = = =
2. Given that sin y = - and tan y = √ ⁄
Find y.
Exercises 18.7
Solution Evaluate each of the following,
The only quadrant in which sin is – ve and tan
1. tan . / 2. cos . /
is + ve is the 3rd quadrant.

tan y = √ = 1.732 3. tan . / 4. sin2 . /

Baffour Ba Series, Further Mathematics for Schools Page 526


5. tan2 . / 6. sin . / Worked Examples
1. Find the values of the six trigonometric
B. Evaluate each of the following; functions for the angle θ in the triangle below.
1. sin . / 17
8
2. tan . /
θ
3. cos . / 15

4. sin . / Solution
√ √
sin θ = and csc θ =
Challenge Problem cos θ = and sec θ =
Prove that;
tan θ = and cot θ =
1. sin . / = sin . /
2. sin ( )= 2. Find the exact values
√ 8 x
3. 2 x= of x and y.

Solution 600
Application to Right Triangles y
From the diagram,
Case 1
sin 600 =
Give three sides of the triangle, any of the six
ratios can be used to calculate the value of the x = 8 sin 600
interior angles. x = 6.9282
x=7
Case 2
Given two sides of the right triangle, the third cos 600 =
side can be calculated by using Pythagoras y = 8 cos 600
theorem. y=4

Case 3 Alternatively;
Given two sides and an interior angle of the By Pythagoras theorem,
right triangle, apply the appropriate functions 82 = x2+ y2
that links the two sides and the given angle. 82 = x2 + 42 But y = 4
x2 = 82 – 42
Case 4 x2 = 64 – 16
Given an interior angle, and two unknown x2 = 48
sides , apply the appropriate functions that x=√ 8
links the unknown sides and the given angle. x = 6.9282
x=7

Baffour Ba Series, Further Mathematics for Schools Page 527


Exercises 18.8 II. Given cos θ = , a is the side containing the
A. Find the values of the six trigonometric right angle and angle θ(adjacent) and b is the
functions for the angle θ in each . side that faces the right angle (hypotenuse)
1. 2.
5
a 2 b
θ θ
b θ
a
3. 4.
IV. Given tan θ = , a is the side that faces θ
17
3 θ (opposite) and b is the side containing the right
1 8
θ angle and angle θ (adjacent).
15
B. Find the exact values of x and y a
1. 2. θ
b
x x 3 V. Apply similar principles when given csc θ,
4
sec θ and cot θ
300 600 VI. Obtain the unknown side using Pythagoras
y y
theorem.
VI. Find the required trigonometric function,
3. 10 300
y not forgetting the CAST system.
Obtuse Acute
x
S A
Finding Trigonometric function value given
one trigonometric function
Draw a right – angled triangle and name one T C
acute angle as θ. Reflex
Reflex

Worked Examples
θ 1. In a right – angled triangle, if θ is an acute
Then label the dimensions of the triangle by angle sin θ = . Find the values of the
observing the following; trigonometric functions of θ and the perimeter
I. Given sin θ = , a is the side that faces of the triangle
θ (opposite) and b is the side that faces the right
angle (hypotenuse) Solution
( )
i. sin θ = = .
a b ( )
Ɵ O = 3 and H = 5, Adjacent (A) = ?

Baffour Ba Series, Further Mathematics for Schools Page 528


By Pythagoras theorem, cos θ = and sec θ =
5
3
sin θ = and csc θ =
θ
– = 16 tan θ = and cot θ =
A = √ = 4cm
Now O = 3cm and H = 5cm, A = 4cm ii. The perimeter of the triangle;
1. cos θ = = 4 sec θ = = = 6 + 8 + 10
= 24 units
2. tan θ = = 5. cot θ = =

3. csc θ = = iii. Let the interior angles be 900, θ and α


cos θ =
ii. Perimeter of the triangle; θ= . / = 36.8610 = 370 (Nearest degree)
P=O+A+H
5
P = (3 + 4 + 5)cm 3 α = 1800 – 900 – 370
P = 12cm θ
4 α = 530
The interior angles of the triangle are ;
2. If θ is an obtuse angle and cos θ = . 370, 530 and 900
i. Find the values of the trigonometric functions
of θ. 3. In a right triangle, tan A = where A is a
ii. Find the perimeter of the triangle. reflex angle,
iii. Find the interior angles of the triangle i. Determine the other trigonometric functions
of A.
Solution ii. What is the perimeter of the triangle?
y 10
cos θ = iii. What are the interior angles of the triangle?
θ
Since θ is obtuse, 8
it falls in the second quadrant. Solution
tan A = where A is a reflex angle
From the diagram, 10
6
102 = y2 + 82 Since θ is reflex, 12
θ θ
y2 = 102 - 82 it falls in the
8
y2 = 100 – 64 fourth quadrant. h
5
y=√
y=6 From the diagram,
12
h2 = 122 + 52
θ
In the second quadrant, only sin is + ve; h2 = 144 + 25 5
The values of the trigonometric function of θ h=√ 13
are; h = 13

Baffour Ba Series, Further Mathematics for Schools Page 529


In the fourth quadrant , only cos is + ve; y=√
The values of the trigonometric function of A y=7
are; In the third quadrant, only tan is + ve;
cos A = and sec A = The values of the trigonometric function of θ
are;
sin A = and csc A =
cos θ = and sec θ =
tan A = and cot A =
sin θ = and csc θ =
ii. The perimeter of the triangle; tan θ = and cot θ =
= 5 + 12 + 13
= 30 units Exercises 18.9
A. Find the exact values of the trigonometric
iii. Let the interior angles be 900, θ and α function for the acute angle θ.
tan A = 1. sin θ = 4. cot θ =
A= . / 2. cos θ = 5. sec θ =
A = 67.3801 3. tan θ = 6. csc θ = 4
A = 670 (Nearest degree)
The Fundamental Identities
α = 1800 – 900 – 670
1. The reciprocal identities
α = 230
The interior angles of the triangle are ; csc θ = sec θ = cot θ =
230, 670 and 900
2. The tangent and cotangent identities:
4. In a right triangle, sec θ = , where θ is tan θ = cot θ =
reflex. Find the values of the trigonometric
functions of θ . 3. The Pythagorean identities:
24 a. sin2 θ + cos2 = 1
θ
Solution b. 1 + tan2 θ = sec2 θ
7 c. 1+ cot2 θ = csc2 θ
If sec θ = , then 25
cos θ = Proofs
Since θ is reflex, Consider the diagram below;
it falls in the third quadrant.
24
From the diagram, c
θ b
252 = y2 + 242 y 25
y2 = 252 – 242 θ
a
y2 = 625 – 576
Baffour Ba Series, Further Mathematics for Schools Page 530
The tangent and cotangent identities: + =
From the diagram,
Sin θ = , cos θ = and tan θ = . / +. / =. / ………..(2)

1. = ⁄
=
But cot θ = and csc θ =
Substitute in eqn (2)
2. cot θ = = ⁄
= 1+ cot2 θ = csc2 θ

The Pythagorean identities: Application of fundamental identities


1. From the diagram; The method of proving trigonometric identities
a2 + b2 = c2 ……………..(1) is to take one side and convert to the other side.
It is more easier to start with the complex side
Divide through by c; to obtain the simple side

. / + . / = . / ……….(2)
Worked examples
But = cos θ and = sin θ 1. Let θ be an acute angle.
a. Express sin θ in terms of cos θ
Substitute in eqn (2) b. Express tan θ in terms of sin θ.
(cos ) + (sin ) = 1
cos2 θ + sin2 θ = 1 Solution
a. cos2 θ + sin2 θ = 1
sin2 θ = 1 – cos2 θ
2. From cos2 θ + sin2 θ = 1 ……………(1)
Divide through by cos2 θ
sin θ = √

=
b. tan θ = ………(1)
+ = From cos2 θ + sin2 θ = 1
cos θ = √
. / +. / =. / ………..(2)
Put cos θ = √ in eqn (1)
But tan θ = and sec θ = ⇒ tan θ = √
Substitute in eqn (2)
1 + tan2 θ = sec2 θ 2. Prove + = 2 sec2 A

3. From cos2 θ + sin2 θ = 1 ……………(1)


Solution
Divide through by sin2 θ
+ = 2 sec2 A
=
L.H.S. R.H.S.

Baffour Ba Series, Further Mathematics for Schools Page 531


Consider the L.H.S; = csc θ
( ) ( )
( )( ) L. H. S = R. H. S

5. Verify the identity sec A – cos A = sin A tan A

Solution
sec A – cos A = sin A tan A
L. H. S R. H. S

But cos2 A + sin2 A = 1 Considering the L. H. S


cos2 A = 1 – sin2 A sec A – cos A
= =2 = – cos A
= sec A
=
⇒ = 2 sec2 A
=
L.H.S = R. H. S
= sin A . /
4. Prove that cot θ + = csc θ = sin A tan A
LHS = R.H.S
Solution
6. Verify the identity sec θ = sin θ (tan θ + cot θ)
cot θ + = csc θ
L.H.S R. H. S Solution
sec θ = sin θ (tan θ + cot θ)
Consider the L.H.S; L.H.S R. H. S
cot θ +
Consider the R. H. S
cot θ = sin θ (tan θ + cot θ)

⇒ + = sin θ . /

( ) = sin θ . /
= ( )
= sin θ . /
= ( ) =
=
2 2
But cos A + sin A = 1
= sec θ
⇒ = L. H. S = R. H. S
( ) ( )

Baffour Ba Series, Further Mathematics for Schools Page 532


Type 2 =
This is where the denominator of one side is a
binomial and the other is a monomial. In this Now, transform the R. H. S to obtain L.H.S
case, we change the form of the fraction by expression
multiplying the numerator and the denominator = .
by the conjugate of the denominator and then
apply one of the Pythagorean identities. =

Worked Examples =
1. Verify the identity =
Exercises 18.10
Solution A. Prove each of the following identities;
= 1. = sec θ + cot θ
2. (sec θ + tan θ) (1 – sin θ) = cos θ
= . 3. sec A sin A = tan A
( ) 4. (1 – cos A) (1 + cos A) = sin2 A
= 5. (1 + tan2 θ) ( 1 – sin2 θ) = 1
=
( ) 6. tan θ √ = sin θ

( )
= B. Prove the following;
1. csc θ – sin θ = cot θ cos θ
= 2. sin x + cos x cot x = csc x
3. tan t + 2 cos t csc t = sec t csc t + cot t
L.H.S = R. H. S
4. tan2 A – sin2 A = tan2A sin2 A
5. = csc θ – sec θ
2. Show that (tan θ – sec θ)2 =
6. = cot2 θ
Solution
(tan θ – sec θ)2 = Compound Angles
L. H. S = R. H. S A compound angle is an angle which is written
as a sum of or difference of two or more angle.
Consider the L.H.S For example, (A + B) , ( A – B) and (A – B – C)
(tan θ – sec θ)2 are compound angles.
= (tan2 θ – 2 tan θ sec θ + sec2 θ )
Addition and Subtraction Formulas for
=. / –2. /. /+. / Cosine Sine and Tangent
1. cos (A + B) = cos A cos B – sin A sin B
= – + 2. cos (A – B) = cos A cos B + sin A sin B

Baffour Ba Series, Further Mathematics for Schools Page 533


3. sin (A + B) = sin A cos B + cos A sin B A equal to the co-function of the
4. sin (A – B) = sin A cos B – cos A sin B complementary angle . /
5. tan (A – B) =
cot A = = tan . /
6. tan (A + B) =
csc A = = sec . /

Proof of the cosine formula sec A = csc . /


Each of the formulas are derived based on the
formula: For instance, cos R = sin ( – R) but = 900
cos (A – B) = cos A cos B + sin A sin B
⇒ cos R = sin (900 – R). Therefore, the sine of
1. cos (A + B) = cos A cos B – sin A sin B
an angle is equal to the cosine of its
complementary angle and vice – versa. For
Given cos (A – B) = cos A cos B + sin A sin B
example:
Let B = - B on both sides
sin 750 = cos (900 – 750)
cos [A – (-B)] = cos A cos (-B) + sin A sin (-B)
sin 750 = cos 150
But cos (-B) = cos B
cos 300 = sin (900 – 300)
cos (A + B) = cos A cos B + sin A sin (-B)
cos 300 = sin 600
cos (A + B) = cos A cos B - sin A sin B
Worked Examples
Co-function Formulas 1. If sin (x + 300) = cos 400, find x where
Consider the diagram below;
00 < x <

–A Solution
c
a Method 1
Expressing cos 400 as a sin
A sin (x + 300) = cos 400…………..(1)
b
sin (900 – 400) = cos 400………..(2)
If A is the radian measure of an acute angle,
then the angle with – A is complementary to
eqn (1) = eqn (2)
A (add up to 900). From the above triangle, the sin (x + 300) = sin (900 – 400)
following ratios can be obtained; sin (x + 300) = sin 500
sin A = = cos . / x + 300 = 500
x = 500 – 300
cos A = = sin . /
x = 200,
tan A = = cot . /
Method 2
These 3 formulas and their analogues for sec A, Expressing sin (x + 300) as a cos
csc A and cot A state that the function value of sin (x + 300) = cos 400 ……………….(1)

Baffour Ba Series, Further Mathematics for Schools Page 534


sin (x + 300) = cos [900 – (x + 300)]…..(2) ⇒cos (118 – 5x) = cos (3x – 50)0
1180 – 5x = 3x – 500
eqn (1) = eqn (2) 1180 + 50 = 3x + 5x
cos [900 – (x + 300)] = cos 400 1680 = 8x
cos (900 – x – 300) = cos 400 x=
cos (60 – x)0 = cos 400
x = 210
600 – x = 400
x = 600 – 400
sin is positive in the first and second quadrants
x = 200
x = 210, x = 1800 – 210 = 1590
x = 210, x = 1590
2. Given that cos (2x – 23)0 = sin 470. Find the
value of x where - < x < . cos is positive in the first and fouth quadrants
x = 210, x = 3600 – 210 = 3390
Solution x = 210, x = 3390
cos (2x – 23)0 = sin 470
cos (2x – 23)0 = cos (90 – 470) Check in the equation;
⇒2x – 230 = 430 The solution set is x = 210, x = 3390
2x = 230 + 430
2x = 660 3. Without, using tables or calculators, find the
x= value of – .
0
x = 33
Solution
cos is positive in the first and fouth quadrants –
x = 330, x = 3600 – 330 = 3270
x = 330, 3270
But cos 500 = sin (900 – 500)
= sin 400
Check in the equation;
cos 220 = sin (900 – 220)
The solution set is x = 330
= sin 680

3. Given that sin (5x – 28)0 = cos (3x – 50)0,


⇒ – =2–1 =1
find the value of x where 00 < x < 3600

Solution Exercises 18.11


sin (5x – 28)0 = cos (3x – 50)0 A. 1. If sin 5x = cos 200, find the value of x.
But sin (5x – 28)0 = cos 900 – (5x – 28)0 2. If sin 250 = cos (y + 50)0, find the value of y
= cos 900 – 5x + 280 3. If cos 3y = sin 2y and 00 ≤ y ≤ 900, find the
= cos (900 + 28o– 5x)
value of y.
= cos (118 – 5x)

Baffour Ba Series, Further Mathematics for Schools Page 535


4. If sin(x – 10 )0 = cos (x + 10)0 and 00≤ x ≤ = sin A cos B + cos A sin B
0
90 , calculate the value of x. sin (A + B) = sin A cos B + cos A sin B

Subtraction formula for sine


B. Solve the values of x between 00 and 3600 sin (A – B) = sin A cos B – cos A sin B
in the following equations.
sin (A + B) = sin A cos B + cos A sin B…(1)
1. 2 sin x = cos (x + 600)
2. cos (x + 450) = cos x Let B = (-B)
3. sin (x – 300 ) = cos x
eqn (1) becomes;
sin (A + (-B) = sin A cos (-B) + cos A sin (-B)
C. From the following equations, find the
sin (A – B) = sin A cos(-B) + cos A sin(-B)
value of tan x
1. sin (x + 300) = cos (x + 300) sin (A – B) = sin A cos(-B) – cos A sin B
2. 2 sin (x – 450) = cos (x + 450)
3. sin (x + 450) = 2 cos (x + 450) But cos (-B) = cos B put in;
sin (A – B) = sin A cos (-B) – cos A sin B
Proofs of Addition and Subtraction formulas ⇒ sin (A – B) = sin A cos B – cos A sin B
for Sine and Tangent
Addition formula for Tangent
1. sin (A + B) = sin A cos B + cos A sin B ( )
2. sin (A – B) = sin A cos B – cos A sin B tan (A + B) = ( )
3. tan (A + B) = =
4. tan (A – B) = If cos A cos B ≠ 0, then divide each numerator
and denominator by cos A cos B to obtain;
Addition formula for sine . /. / . /. /
tan (A + B) =
sin (A + B) = sin A cos B + cos A sin B . /. / . /. /
Using the co – function formula,
tan (A + B) =
sin (A + B) = cos . ( )/
= cos . / Subtraction formula for tangent
tan (A – B) =
= cos 0( ) 1
tan (A + B) = ………(1)
From cos (A – B) = cos A cos B + sin A sin B Let B = (-B) and substitute in eqn (1)
( )
cos 0( ) 1 tan (A +(- B) = ( )

= cos ( ) cos B + sin ( ) sin B tan (A – B) =

But cos ( ) = sin A and sin ( ) = cos A Worked Examples


By substitution; Type 1
= cos ( ) cos B + sin ( ) sin B 1. Express sin 1350 in surd form.

Baffour Ba Series, Further Mathematics for Schools Page 536


Solution sec 150 = = ( )
sin 1350 = sin (90 + 45)0
But sin (a + b) = sin a cosb + cosa sin b cos (45 – 30)0
⇒ sin (90 + 45)0 = cos 450 cos 300 + sin 450 sin 300
= sin 900cos450 + cos900 sin 450 √ √ √
√ √
= . / . /+ . / . /
= (1) . / + (0) . / √ √

= +
= √ √
=
2. Express cos 1500 in surd form.
sec 150 = = √ √
=
( ) √ √
Solution
cos 1500 = cos (90 + 60)0
5. Express tan 1050 in surd form
But cos (x + y)0 = cos x cos y – sin x sin y
⇒ cos (90 + 60)0
Solution
= cos 900cos 600 – sin 900 sin 600
tan 1050 = tan ( 600 + 450)

= ( ). / – ( ). / =

=0– √
= (√ ) ( )

=– √
=

3. Express cot 150 in surd form
6. Without using tables or calculator, find the
Solution value of sin ( 1200 + 450 )
cot 150 = cot (45 – 30)0
But cot 150 = = Solution
( )
sin (120 + 45)0
Now,
But sin (a + b) = sin a cos b + cos a sin b
tan(45 – 30)0 = ⇒ sin (120 + 45)0
√ √
√ = sin 1200 cos 450 + cos1200 sin 450
= √
= √
= √ √ √

( ) =. /. / +. /. /
√ √
0 √
= –
cot 15 = = √
=
( ) √

7. Simplify cos (45 – 30)0
4. Express sec 150 in surd form
Solution
Solution cos (α – β) = cos α cos β + sin α sin β
cos (45 – 30)0 = cos 450 cos 300 + sin 450 sin 300

Baffour Ba Series, Further Mathematics for Schools Page 537


√ √ √
= . / . /+ . / . / sin . /

=

+

=
√ √ sin (A – B) = sin A cos B – cos A sin B
= . /. / – . / . /
8. Find the value of tan A when tan (A – 45) = . = –

Solution =
tan (A – 45) = .
Execise 18. 12
=
A. Express the following as a trigonometric
= = ratio of one angle.
3 (tan A – 1) = 1 + tan A 1. cos 480 cos 230 + sin 480 sin 230
3 tan A – 3 = 1 + tan A 2. cos 130 cos 500 – sin 130 sin 500
3tan A – tan A = 1 + 3 3. cos 100 sin 0 – sin 100 cos 50
(3 – 1) tan A = 4 4. sin 570 cos 40 + cos 570 sin 40
2 tan A = 4 5. cos 30 sin (-2) – cos 20 sin 30
tan A = 2 6. sin (-5) cos 20 + cos 50 sin (-2)

9. Evaluate sin . / B. Express the following in surd form


1. cos 750 2. sin 1350 3. cosec 3300
4. tan 3000 5. cosec 1200 6. cot1500
Solution
and are angles Type 2
Let A = Worked Examples
17
h 1. Find the exact value of cos using the fact
⇒ cos A =
that = +
8
From the diagram
h=√ 8 17 Solution
15
h=√ cos = cos . /
A
h = 15
8 = cos cos – sin sin
8c
√ √ √
Let B = 13 m = . /. / – . / . /
12
⇒ sin B = √ √
B =
h
8c
From the diagram 13 Exercises 18.13
m
h=√ 12 Find the exact values:
h=√ A
1. cos + cos 4. cos (use + )
5
h=5 8c
m
Baffour Ba Series, Further Mathematics for Schools Page 538
2. sin + sin 5. cos (use + ) i. sin (A – B) = sin A cos B – cos A sin B
sin (A – B) = . / . / – . /. /
3. tan tan 6. tan cos
sin (A – B) = + =
Type 3
Worked Examples ii. cos (A – B) = cos A cos B + sin A sin B
1. a. If sin A = and cos B = , where A is =. /. /+ . /. /
obtuse and B is acute, find the exact value of ; = + =
i. sin (A – B) ii. cos (A – B) iii. cot (A + B)
iii. cot (A + B) =
Solution ( )

sin A = (obtuse) and But tan (A + B) =


5
cos B = (acute) 4 Substitute tan A = and tan B =
A
A h tan (A + B) =
c . /. /
From sin A =
m
Let h be the unknown side; tan (A + B) = = = × =
8c
h2 = 52 - 42
m ⇒ cot (A + B) = = =
h=√ 5 ( ) ⁄
4
h=3
A
3 4. If cos A = and tan B = , where A and B
If A is obtuse, then; 8c are both reflex angles, find the values of ;
cos A = and tan A = m i. sin (A – B) ii. tan (A – B) iii. cos (A + B)

From cos B = k
13 Solution
B cos A = (reflex) and tan B = ( reflex)
Let the unknown side be k; 12
k2 = 132 – 122 From cos A = 3
A
k=√ 13 h
5
k=5 5
B
Let h be the unknown side;
h2 = 52 – 32
If B is acute, then 12
h=√ 3
sin B = and tan B = A
h=√ 4
h=4 5
sin A = , cos A = and tan A =
If A is reflex, then;
cos B = , sin B = and tan B =

Baffour Ba Series, Further Mathematics for Schools Page 539


sin A = and tan A = are acute, find the exact value of ;
i. sin (A + B) ii. cos (A + B) iii. cot (A + B)
5
B
From tan B =
12 2. Suppose sin = and cos = where
is acute and is obtuse. Find the exact values
Let the unknown side be k;
of:
k2 = 122 + 52 5
B
i. sin ( + ) ii. tan ( + )
k=√
12 iii. find the quadrant containing ( + )
k = 13 13

4. A, B, C and D are acute angles such that tan


If B is reflex, then
A = , tan B = , tan C = and tan D = .
sin B = and cos B =
Evaluate i. tan (A + B) ii. tan (C + D)
ii. Hence, or otherwise, find the value of tan (A
sin A = , cos A = and tan A = + B + C + D)
iii. Show that A + B + C + D = 450
tan B = , sin B = and cos B =
5. Find the values of the following;
i. sin (A – B) = sin A cos B – cos A sin B i. sin (30 + 45) ii. cos (120 + 45)
ii. cos (450 – 300) iv.
sin (A – B) = . /. /-. /. /
sin (A – B) = + = 6. If tan (A + B) = 3 and tan B = 2, find the
value of tan A.
ii. tan (A – B) =
7. If sin A = and cos B = , where A is obtuse
Substitute tan A = and tan B =
and B is acute, find the exact value of sin (A +
B)
tan (A + B) =
. /. /
8. If α and β and acute angles such that csc α =
tan (A + B) = = = × = and cot β = , find
i. sin (α + β) ii. tan (α + β)
iii. cos (A + B) = cos A cos B – sin A sin B iii. the quadrant containing α + β
=. /. /–. /. /
9. If α and β and acute angles such that cos α =
= – =
and tan β = , find
i. sin (α + β) ii. cos (α + β)
Exercises 18 .14
iii. the quadrant containing α + β
1. If sin A = and cos B = , where A and B

Baffour Ba Series, Further Mathematics for Schools Page 540


10. If sin α = and sec β = where α and β ⇒ =5
are reflex and acute angles respectively.
i. sin (α + β) ii. tan (α + β) Substitute tan A = 4
iii. the quadrant containing α + β =5
4 + tan B = 5(1 – 4 tan B)
11. If α and β are reflex angles such that cos α
4 + tan B = 5 – 20 tan B
= and cos β = , find: 4 – 5 = - tan B – 20 tan B
i. sin (α – β) ii. cos (α – β) -1 = tan B (-1 – 20 )
iii. the quadrant containing α + β -1 = -21 tan B
tan B =
12. If α and β are obtuse angles such that sin α
= and cos β = , find:
2. Find the value of cot B, when cot A = and
i. sin (α + β) ii. tan (α + β)
cot (A – B ) = 8 ans
iii. the quadrant containing α + β h
4

B. 1. A and B are acute angles such that sin A = Solution A

and cos B = . Without using tables or cot A = and cot (A – B ) = 8 1

calculator, find the value of; From the diagram,


i. cos A v. sin (A + B) Cot A = =
ii. tan A vi. cos (A + B)
iii. sin B vii. tan (A + B) Let the unknown side be h
iv. tan B h2 = 42 + 12
h=√ 4 √
2. If tan α = and cot β = where α is h=√ A
obtuse and β is in the third quadrant, find:
1
i. sin (α + β) ii. cos (α + β) iii. tan (α + β) i. From the diagram tan A = 4
sin (α – β) ii. cos (α – β) iii. tan (α – β) cot (A – B) = =8
( )

Type 4 =
Worked Examples
⇒ cot (A – B ) = = =8
1. A and B are acute angles such that tan A = 4 ( )
and tan (A + B) = 5. Find tan B. = =8

Solution =8
tan A = 4, tan (A + B) = 5, tan B = ? 1 + 4 tan B = 8(4 – tan B)
tan (A + B) = 1 + 4 tan B = 32 – 8 tan B
4 tan B + 8 tan B = 32 – 1
Baffour Ba Series, Further Mathematics for Schools Page 541
tan B (4 + 8) = 31 written as a fraction of a single angle. For
12 tan B = 31 example, , , are sub – multiple angles.
tan B =
Double Angle Formulas
But cot B = = = 1. sin 2A = 2 sin A cos A

2. i. cos 2A = cos2 A – sin2 A
ii. cos 2A = 1 – 2sin2 A
Exercises 18.15
iii. cos 2A = 2 cos2 A – 1
1. If A and B are acute, such that tan A = and
3. tan 2A =
0
tan B = , find (A + B) ans 45
Proofs
2. If tan A = , 1800 ≤ θ ≤ 3600 and tan B = , 1. sin (A + B) = sin A cos B + cos A sin B
Now , let B = A
-900 ≤ θ ≤ 900, find A – B ans 2250
sin (A + A) = sin A cos A + cos A sin A
sin (2 A) = sin A cos A + cos A sin A
3. If tan A = , and tan B = , where A is
obtuse and B is acute, find the value of A – B By factorization
0
ans 135 sin (2A) = sin A cos A (1 + 1)
sin 2A = 2 sin A cos A
4. sin θ = and θ is obtuse, find the value of 2. i. cos (A + B) = cos A cos B - sin A sin B
Let B = A
.
cos (A + A) = cos A cos A – sin A sin A
cos 2A = cos A cos A – sin A sin A
5. If sin x = , where 00 x 900, find the cos 2A = cos2 A – sin2 A
value of :
i. 1 – cos2 x, ii. 1 + cos2 x, ii. cos 2A = cos2 A – sin2 A …...(1)
iii. ( ) , iv. ( ) But sin2A + cos2A = 1…………..(2)

6. If cos x = 0.8, find the value of the following From eqn (2);
for 00 x 900: cos2 A = 1 – sin2 A
i. cos2x – sin2x ii. – Substitute in eqn (1);
cos 2A = (1 – sin2 A) – sin2 A
Multiple Angles cos 2A = 1 – 2 sin2 A
A multiple angle is an angle which is written as
a multiple of a single angle. For example 2A, iii. cos 2A = cos2 A – sin2 A …...(1)
2B , 3θ are multiple angles. But sin2A + cos2A = 1……….…..(2)

A sub-multiple angle is an angle which is From eqn (2);

Baffour Ba Series, Further Mathematics for Schools Page 542


sin2 A = 1 – cos2 A Multiply top and down by cos2 A
=
Substitute in eqn (1)
cos 2A = cos2 A - (1 – cos2 A) = sin 2A
cos 2A = cos2 A - 1 + cos2 A
cos 2A = 2 cos2 A – 1 Worked Examples
1. If sin A = and A is acute, find the exact
3. tan (A + B) = value of sin 2A and cos 2A
Let B = A
tan (A + A) = Solution
5
sin A = = 4
A
tan 2A = l
Let the unknown side be l;
4
52 = 42 + l2
Other Proofs
l2 = 52 - 32 5
1. tan 6θ = 4
l=√
A
l=3 3
tan 2A = cos A =

Let A = 3θ at both sides; sin 2A = 2 sin A cos A


tan 2(3θ) = sin 2A = 2 . / . / =
tan 6θ =
cos 2A = cos2 A – sin2 A

2. cos 2A = cos 2A = . / – . / = – =

= 2. Express cos 3θ in terms of cos

= Solution
cos 3θ = cos (2θ + θ)
= (Multiply top and down by cos2 A) Using the addition formula;
= cos 2A cos (A + B) = cos A cos B - sin A sin B
cos 3θ = cos (2θ + θ)
3. sin 2A = cos (2θ + θ) = cos2θ cos θ – sin 2θ sin θ
= (2 cos2 θ – 1 ) cos θ – (2 sin θ cos θ ) sin θ
= Double angle formula
= 2 cos θ – cos θ – 2 cos θ sin2 θ (expand)
3

= = 2 cos3 θ – cos θ – 2 cos θ (1 - cos2 θ)

Baffour Ba Series, Further Mathematics for Schools Page 543


= 2 cos3 θ – cos θ – 2 cos θ + 2cos3 θ ⇒ tan A = or tan A = 7
= 4 cos3 θ – 3 cos θ

3. Show that sin 3A = 3 sin A – 4 sin3 A When tan A =


Let the unknown side be h;
Solution h2 = 12 + 72
sin 3A = sin (2A + A) √
h=√ 1
sin (2A + A) = sin 2A cos A + cos 2A sin A 1 A
= (2 sin A cos A) cos A + (cos2 A – sin2 A) sin A Since A is acute, 7
= 2 sin A cos2 A + sin a cos2 A – sin3 A
sin A = and cos A =
= 3 sin A cos2 A – sin3 A √ √

= 3 sin A (1 – sin2 A) – sin3 A


= 3 sin A – 3sin3 A – sin3 A h
7
= 3 sin A – 4 sin3 A When tan A = 7 =
A
1
4. If sin 2A = , 0 ≤ A ≤ , find tan A, sin A
,
and cos A. Let the unknown side be h;
h2 = 12 + 72 √
7
Solution h=√
A
1
sin 2A = , Since A is acute,
sin 2A = sin A = and cos A =
√ √
⇒ =
,
2. If tan θ = , 0 < θ < , without using tables
Now, let tan A = t or calculator find the value of:
,
= i. sin 2θ ii. cos 2θ iii. tan 2θ
7(1 + t2) = 2t × 25
7 + 7t 2 = 50t Solution
7t 2 – 50t + 7 = 0 tan θ = , 0 < θ < , 20 h

tan θ = = θ
Solving by factorization;
21
(7t – 1) (t – 7) = 0
Let the unknown side be h;
7t – 1 = 0 or t – 7 = 0
h 2 = 202 + 212
t = or t = 7
h h = √8
1 29
h = 29 20
But t = tan A A
θ
7
21

Baffour Ba Series, Further Mathematics for Schools Page 544


Since θ is acute; without using tables or calculator.
sin θ = and cos θ =
5. Find the value of tan 2θ when:
1. sin 2θ = 2 sin θ cos θ a. tan θ = b. cos θ =
=2. /. /=
6. Find the value of sin 2θ and cos 2θ given sin

ii. cos 2 θ = 1 – 2 sin2 θ θ=
=1–2. / =1– =
Challenge Problems
Prove each of the following:
iii. tan 2A = 1. 2 cos2 A – cos 2A – 1 = 0
2. sin(A + B) sin(A – B) = cos2 B + sin2 A – 1
. / ⁄
tan 2θ = = = × = 3. = tan A
. / ⁄
4. = tan2 A
Exercises 18.16 5. cos 3 A = 4 cos3 A – 3 cos A
A. Without using tables or calculator, find 6. tan 3θ =
the exact values of sin 2θ, cos 2θ and tan 2θ
for the given value of θ. Half – Angle Identities
1. cos θ = 00 < θ < 900 1. sin2 A =
2. cot θ = 1800 < θ < 2700 2. cos2 A =
3. sec θ = - 3 900 < θ < 1800
0 0
3. tan2 A =
4. sin θ = 270 < θ < 360

Proofs
B. 1. If cos x = , find the value of cos 2x 1. cos 2A = 1 – 2sin2 A (double angle formula)

without using tables or calculator. 2 sin2 A = 1 – cos 2A
sin2 A =
2. A and B are acute angles such that tan B =
and tan (A – B) = 2. Find: 2. cos 2A = 2 cos2 A – 1
i. tan A ii. sin 2A 1 + cos 2A = 2 cos2 A
cos2 A =
3. If cos 2A = , find the two values of cos A
3. tan2 A = (tan A)2 = . / =
without using tables or calculator.
Substitute identities (1) and (2) to obtain;
4. If cos 2A = , find the two values of tan A tan2 A =

Baffour Ba Series, Further Mathematics for Schools Page 545


Half – Angle Formulas Let h be the unknown side,
In the half angle identities, substitute h2 = 42 + 32
3
A = , to obtain the following; h=√ A
h=5 4
5
Solution
1. sin2 A =
From the diagram, if A is reflex,
Then sin A = and cos A =
sin2 = =

2. cos2 = = Using the half angle formula for tangent;


tan = =
2
3. tan = = ⁄
tan = = =

OR
Taking square root of both sides of each ⁄ ⁄
equation, we obtain the following formulas tan = = = =

called half – angle formulas.
2. Without using mathematical tables or
1. sin = √
calculator, find in surd form, (radicals) the
value of tan 22.50
2. cos = √
Solution
3. i. tan = √ tan =
ii. tan = tan = tan 22.50

iii. tan = tan =


√ ⁄
tan = √ ⁄
Worked Examples
1. If tan A = and A is reflex, find tan . tan = –√ ⁄ ÷√ ⁄


Solution = ×

Diagram ( √ )
=

3 √ √
A = ×
√ √
4 √ ( √ )
h = ( √ )( √ )

=

Baffour Ba Series, Further Mathematics for Schools Page 546


=

– L. H. S = R H S
sin (A + B) + sin (A – B) = 2 sin A cos B
=√ –1
sin A cos B = [sin (A + B) + sin (A - B)]
Exercises 18.17
Formula 2
A. Find the exact values of sin , cos and
Formula 2 is obtained by subtraction;
tan for the given conditions eqn (1) – eqn ( 2)
1. sec θ = 00 < θ < 900 √
,

,
L.H.S.
sin (A + B) – sin (A – B)
2. csc θ = -900 < θ < 00
3. tan θ = 1 -1800 < θ < -900 R.H.S.
4. sec θ = - 4 1800 < θ < 2700 cos A sin B – ( - cos A sin B)
cos A sin B + cos A sin B
B. Verify the following: = 2 cos A sin B .
1. sin 10θ = 2 sin 5θ cos 5θ
2. cos2 3x – sin2 3x = cos 6x L.H.S. = R.H.S.
3. 4 sin cos = 2 sin x sin (A + B) – sin (A – B) = 2 cos A sin B
cos A sin B = [sin (A + B) – sin (A – B)]
4. = 4 – 4 sin2 A
5. (sin t + cos t)2 = 1 + sin 2t
Formulas 3 and 4 are derived in a similar
function using addition and subtraction
Product to Sums and Differences Formulas
formulas for the cosine function.
1. 2 cos A cos B = cos (A + B) + cos (A – B)
2. 2 sin A cos B = sin (A + B) + sin (A – B)
Try
3. 2 sin A sin B = cos (A – B) – cos (A + B)
Prove that :
4. 2 cos A sin B = sin (A + B) – sin (A – B)
1. 2 sin A sin B = cos (A – B) – cos (A + B)
2. 2 cos A sin B = sin (A + B) – sin (A – B)
Proofs
Formula 1
sin (A + B) = sin A cos B + cos A sin B….(1) Worked Examples
sin (A – B) = sin A cos B – cos A sin B…….(2) 1. Express sin 4θ cos3θ as a sum
eqn (1) + eqn (2);
Solution
L.H.S Method 1
sin (A + B) + sin (A – B) . sin 4θ cos3θ
Let 4θ = A and 3θ = B
R. H. S sin (A + B) + sin (A - B) = 2 sin A cos B
= sin A cos B + sin A cos B sin (4θ + 3θ) + sin (4θ - 3θ) = 2 sin 4θ cos 3θ
= 2 sin A cos B sin 7θ + sin θ = 2 sin 4θ cos 3θ

Baffour Ba Series, Further Mathematics for Schools Page 547


sin 4θ cos 3θ = (sin 7θ + sin θ) 2 cos 3θ cos θ a sum or product
Let 3θ = A and θ = B
2 cos A cos B = cos (A + B) + cos (A – B)
Method 2
2 cos 3θ cos θ = cos (3θ + θ) + cos (3θ – θ)
sin 4θ cos3θ
2 cos 3θ cos θ = cos 4θ + cos 2θ
Let 4θ = A and 3θ = B
2 sin A cos B = sin (A + B) + sin (A – B)
Exercises 18.18
2 sin 4θ cos 3θ = sin (4θ + 3θ) + sin (4θ – 3θ)
A. Express as a sum or difference.
2 sin 4θ cos 3θ = sin 7θ + sin θ
1. sin 7t sin 3 t 5. 2 sin 9θ cos 3θ
sin 4θ cos 3θ = (sin 7θ + sin θ)
2. sin (- 4 x) cos 8x 6. 2 sin 7θ sin 5θ
3. cos 6u cos (- 4u) 7. 3 cos x sin 2x
2. Express sin 3x sin x as a sum or difference of 4. cos 4t sin 6t 8. 5 cos u cos 5u
two trigonometric functions
B. Express the following as a sum or
Solution difference of two trigonometric functions.
sin 3x sin x 1. 2 sin 6θ cos 2θ 5. - 2 sin 4θ sin θ
Let 3x = A and x = B 2. 2 cos 3θ cos θ 6. 2 cos 7θ sin 6θ
2 sin A sin B = cos (A – B) – cos (A + B) 3. 2 cos 4θ cos θ 7. cos x sin 5x
2 sin 3x sin x = cos (3x – x) – cos (3x + x) 4. 2 cos 6θ sin 3θ 8. sin 2A sin A
2 sin 3x sin x = cos 2x – cos 4x
sin 3x sin x = (cos 2x – cos 4x ) Sums and Differences to Product Formulas
The product to sum formulas are used to
express a sum or difference as a product.
3. Express cos 3θ sin 5θ as a sum or difference
Now, let A + B = m and A – B = n. Then :
of two trigonometric functions.
i. (A + B) + (A – B) = m + n
⇒A+B+A–B=m+n
Solution
A+A+B–B=m+n
cos 3θ sin 5θ
2A = m + n
3θ = A and B = 5θ
2 cos A sin B = sin (A + B) – sin (A – B) A=
2 cos 3θ sin 5θ = sin (3θ + 5θ) – sin (3θ – 5θ)
2 cos 3θ sin 5θ = sin 8θ – sin ( –2θ) ii. (A + B) – (A – B) = m – n
2 cos 3θ sin 5θ = sin 8θ + sin 2θ ⇒A+B –A+B=m–n
A–A+B+B=m–n
cos 3θ sin 5θ = (sin 8θ + sin 2θ)
2B = m – n
B=
4. Express as 2 cos 3θ cos θ a sum or
difference.
By substituting in the product to sum formulas,
the following is obtained as sum and
Solution
differences to product formulas.

Baffour Ba Series, Further Mathematics for Schools Page 548


1. 2 cos A cos B = cos (A + B) + cos (A – B) Solution
2 cos cos = cos m + cos n sin 6θ + sin 4θ
Let A = 6θ and B = 4θ
cos m + cos n = 2 cos cos
sin A + sin B = 2 sin cos
⇒ cos A + cos B = 2 cos cos
sin 6θ + sin 4θ = 2 sin cos
2. 2 sin A cos B = sin (A + B) + sin (A – B) = 2 sin cos
2 sin cos = sin m + sin n = 2 sin 5θ cos θ
sin m + sin n = 2 sin cos
2. Express sin 5x + sin 3x as the product of two
⇒sin A + sin B = 2 sin cos trigonometric functions.

3. 2 sin A sin B = cos (A – B) – cos (A + B) Solution


2 sin sin = cos n – cos m sin 5x + sin 3x
cos n – cos m = 2 sin sin Let A = 5x and B = 3x
Multiply though by -1; sin A + sin B = 2 sin cos
- cos n + cos m = - 2 sin sin sin 5x + sin 3x = 2 sin cos
cos m – cos n = - 2 sin sin = 2 sin cos
⇒ cos A – cos B = - 2 sin sin = 2 sin 4x cos x

4. 2 cos A sin B = sin (A + B) – sin (A – B) 3. Find the exact value of sin 1050 – sin 150
2 cos sin = sin m – sin n
Solution
sin m – sin n = 2 cos sin sin 1050 – sin 150
Let A = 1050 and B = 150
⇒ sin A – sin B = 2 cos sin
sin A - sin B = 2 cos sin
Summary sin 1050 – sin 150 = 2 cos sin
Sums and Differences to Product Formulas
sin 1050 – sin 150 = 2 cos sin
1. cos A + cos B = 2 cos cos
= 2 cos 60 sin 450
0

2. sin A + sin B = 2 sin cos √ √


=2× × =
3. cos A – cos B = - 2 sin sin
4. sin A – sin B = 2 cos sin 3. Verify the identity = cot t

Worked Examples Solution


1. Express sin 6θ + sin 4θ as the product of two = cot t
trigonometric functions.
Baffour Ba Series, Further Mathematics for Schools Page 549
L.H.S = R. H. S. A. Express each of the following as a
product of two trigonometric functions.
Consider the L.H.S; 1. sin 4θ + sin 2θ 6. cos θ + cos 7θ
2. cos 7θ + cos 5θ 7. sin θ + sin 3θ
=
3. cos 8θ – cos 2θ 8. cos θ + cos 5θ
4. sin 5θ – sin 3θ 9. sin 2θ – sin 8θ
= 5. cos 6θ + cos 2θ 10. cos θ + cos 2θ

( )
= B. Express as a product.
( )
1. sin 6θ + sin 2θ 4. cos 5t + cos 6t
( )
= ( )
2. sin 4θ – sin 8θ 5. sin 3t – sin 7t
3. cos 5x – cos 3x 6. cos θ – cos 5θ
=
= cot t C. Find the exact value of the following.
1. sin 750 – sin 150 4. 2 sin 750 sin 1050
4. Prove that = tan 2θ 2. cos 1050 – cos 150 5. 2 cos 750 cos 150
3. sin 2550 – sin 150 6. cos 37 0
sin7 0

Solution
= tan 2θ D. 1. If sin θ = 0.65, find the following for
00 x 900;
L.H. S R.H.S
i. θ ii. tan θ iii. iv. . /
Consider the L.H.S;

E. Verify the identity.


1. = cot t
=
2. = tan 2θ
=
3. = cot t
=
4. = tan (u + v)
( )
= ( ) 5. = - cot (u + v)
= 6. = -2 sin 2A
= tan 2θ
L.H.S = R.H.S. Challenge Problems
1. 3 sin θ + sin θ = 4 sin θ cos2 θ
Exercises 18.19 2. cos 3θ + cos θ = 4 cos3 θ – 2 cos θ

Baffour Ba Series, Further Mathematics for Schools Page 550


3. 4 cos x cos 2x sin 3x = sin 2x + sin 4x + sin 6x Solution
sin2 θ = 0≤θ≤2
4. = cot 2θ
sin θ = √ 0≤θ≤2
5. = - tan (u + v) tan (u - v)
sin θ =

sin θ = or sin θ = –
√ √
Trigonometric Equations 1
θ= . / or θ = . /
To solve trigonometric equations: √ √
0 0 0 0
1. Ignore the sign and calculate the related or θ = 45 , 135 , 225 , 315 0 ≤ θ ≤ 3600
reference angle.
2. From the sign of the given ratio , decide in Exercises 18.20
which quadrant the angle falls. A. Solve the following for 00 ≤ θ ≤ 3600
3. On a rough diagram, state the angles
between 00 and 3600. 1. cos θ = - 1 4. 4 cos2 θ = 1

A
2. tan θ = √ 5. tan2 θ =
S
3. sin θ = 0 6. 2 sin2 θ – 1 = 0
T C

B. Find all solutions of the equations for


00 ≤ θ ≤ 3600
Maximum and Minimum values for simple
1. 4 cos θ – 2 = 0 4. 4 sin2 x – 3 = 0
Trigonometric equations
If sin θ = A, then -1 ≤ A≤ 1, min = -1 max = 1 2. √ + 2 sin = 0 5. cot2 x – 3 = 0
If cos θ = A, then -1 ≤ A≤ 1, min = -1 max = 1 3. 3 – tan2 β = 0 6. sec2 x – 4 = 0
If tan θ = A, then A R, min = - max =
Type 2
1. Given that tan (x + 250) = 5.145, where
Worked Examples 00 ≤ θ ≤ 3600, find to one decimal place the
Type 1 value of x.
1. Solve cos θ = ,0≤θ≤2

Solution
tan (x + 250) = 5.145
Solution
x + 250 = t n (5.145)
cos θ = ,
√ x + 250 = 790
θ= . / x = 790 – 250

θ = 1350, 2250 ( 0 ≤ θ ≤ 2 ) x = 540
cos is – ve in second and third quadrants
For the given interval, 00 < x < 3600
Tan is + ve in the third quadrant,
2. Solve sin2 θ = , 00 ≤ θ ≤ 3600
x – 1800 = 540
x = 1800 + 540

Baffour Ba Series, Further Mathematics for Schools Page 551


x = 2340 8 sin x = 3
The solution set is x = 540, 2340 sin x =
x = sin . /
2. Find θ if cos (θ + 600) = 0.0872, where 00
x = 220
θ 2
For the given interval, 00< x < 2600
Solution
sin is + ve in the second quadrant,
cos (θ + 600) = 0.0872
= 1800 – x ,
θ + 600= cos (0.0872)
= 1800 – 220 ,
θ + 600 = 850
= 1580
θ = 850 – 600
The truth set is x = 220, 1580,
θ = 250
Exercises 18.21
For the given interval, 00< θ < 2 Solve the following for 00 ≤ θ ≤ 3600
Cos is + ve in the fourth quadrant
1. sin (θ + 600) = 4. sin (2θ + 300) =
⇒3600 – θ ,

= 3600 – 850 = 2750 2. cos (θ – 450) = 5. cos (3θ + 150) =
θ + 600 = 2750 3. tan (θ + 300) = - √ 6. tan . /=

θ = 2750 – 600
θ = 2150 B. 1. If 10 sin (x – 45)0 = 5, where 00< x < 900,
find the value of x.
The truth set is θ = 250 , 3350
2. Given that 2 cos x = √ , where 00 < x < 900,
3. If 1.5 cos x = 0.75, find x if 00 < x < 900 find the value of x.

Solution 3. Solve the equation 2 sin2 θ = sin θ, for the


1.5 cos x = 0.75 values of θ from 00 to 1800.
cos x =
4. Solve the equation 2 cos2 θ = cos θ, for the
x = cos . / values of θ from 00 to 1800.
0
x = 60
For the given interval, - <x< C. solve for the values of θ from 00 to 1800.
x = 600, 1. (cos θ – 1) (sin θ + 1) = 0
2. ( 2 sin θ + 1) (2 cos θ + 3) = 0
4. If 8 sin x + 2 = 5, find x correct to the nearest 3. (2 sin θ – 1) (cos θ – √ ) = 0
degree for 00 < x < 3600
Type 3
Solution Worked Examples
8 sin x + 2 = 5 √
1. Solve sin . /= ,0≤θ≤2
8 sin x = 5 – 2
Baffour Ba Series, Further Mathematics for Schools Page 552
Solution Worked Example
sin . /=

,0≤θ≤2 1. Solve cos 3A = 0 ≤ A≤ 3600

Let . /= θ
Solution

sin θ = cos 3 A = 0 ≤ A≤ 3600

θ= . / A=
θ = - 600 A = 600
sin is – ve in the third and fourth quadrants. cos is + in the 1st and 4th quadrants
Thus θ = 2400, 3000
⇒. / = 2400 or . / = 3000 In the first quadrant;
Given 0 ≤ A ≤ 3600 0
x + 300 = 2400 or x + 300 = 3000 0 60
⇒ 0 ≤ 3A ≤ 1080
x = 2400 – 300 or x = 3000 – 300
Thus, we go as far as A + 10800
x = 2100 or x = 2700

Therefore, in the first quadrant;


Exercises 18.22
3A = 600, 600 + 3600, 600 + 7200, 600 + 10800
Find the solution of the equation that are in
3A = 600, 4200, 7800, 11400
the interval [0, 2 ]
1. cos . /=0 4. cos . /=-1 A = 200, 1400, 2600, 3800
2. sin . /=1 5. sin . /=
In the fourth quadrant;

3. sin . /= 6. cos . /= 3A = 3000, 3000 + 3600, 3000 + 7200,
3000 + 10800
Involving Multiples and Sub-multiples 0
60
1. Find the value of the angle θ by using the 3A = 3000, 6600 , 10200 , 13800
inverse. A = 1000, 2200, 3400, 4600
2. Identify the quadrant in which the
trigonometric function falls using the CAST Given 0 ≤ A≤ 3600
method. A = 200, 1000, 1400, 2200, 2600, 3400
2. Given kθ, the number of angles required is (k
+ 1) angles. For example, 3θ means 4 angles 4. Solve cos x =

0 ≤ x ≤ 3600
are required.
3. Add the upper range value consecutively to
Solution
the first (related) angle to obtain the other

angles. cos x =
4. Select the angles within the given range as √
x= . /
the answer.
x = 1350

Baffour Ba Series, Further Mathematics for Schools Page 553


cos is – ve in the second and third quadrants Solution
sin θ tan θ = sin θ
In the second quadrants, given the interval sin θ tan θ – sin θ = 0
0 ≤ x ≤ 3600 sin θ (tan θ – 1 ) = 0
⇒ 0 ≤ ≤ 900 sin θ = 0 or tan θ – 1 = 0
sin θ = 0 or tan θ = 1
x = 1350 , 3600 – 1350, 2700 – 1350 , 1800 –
θ = 0 or θ = 450
1350 , 900 – 1350
x = 1350 , 2250, 450, - 450 For sin θ = 0, sin is + ve in the first and second
0 0 0
x = 4(135 ) , 4(225 ), 4(45 ), 4(-45 ) 0 quadrants. θ = 0, 1800
x = 5400 , 9000, 1800, - 1800
For tan θ = 1, tan is positive in the first and
In the third quadrants, third quadrants
θ = 450, 2250
x = 2250, 3600 – 2250, 2700 – 2250,
Thus, θ = 0, 450, 1800, 2250
1800 – 2250, 900 – 2250
x = 2250 , 1350 , 450 , - 450 , - 1350 2. Find the solution of 4 sin2 x tan x – tan x = 0
x = 4(2250) , 4(3150), 4(450) , 4(- 450) , 4(1350) that are in the interval [0, 2 ]
x = 9000 , 12600, 1800, -1800, 5400
Solution
For the given interval, 4 sin2 x tan x – tan x = 0
x = 1800 tan x ( 4 sin2 x – 1 ) = 0
tan x = 0 or 4 sin2 x – 1 = 0
Exercises 18.23 tan x = 0 or 4 sin2 x = 1
A. Solve the following for 00 ≤ θ ≤ 3600 tan x = 0 or sin2 x =

1. tan 3θ = 1 4. cos x =
tan x = 0 or sin x = √
2. sin 2θ = 5. 2 sin 3θ + √ = 0
√ tan x = 0 or sin x =
3. cos = 6. 2 cos 2θ - √ = 0
For tan x = 0, tan is + ve in the first and third
Trigonometric Equations 2
quadrants.
In more complicated trigonometric equations,
x = 0, 1800
reduce the given equation to one or more
simple trigonometric equation by factorization
For sin x = , sin is + ve in the first and second
and rearranging.
quadrants.
Worked Examples x = 300, 1500
1. Solve the equation sin θ tan θ = sin θ For sin x = , sin is - ve in the third and
fourth quadrants.

Baffour Ba Series, Further Mathematics for Schools Page 554


x = 2100, 3300 – 2 sin2 A + 3 sin A – 1 = 0
Thus, x = 0, 300, 1500, 1800, 2100, 3300 2 sin2 A – 3 sin A + 1 = 0

3. Approximate to the nearest degree the Now, let sin A = x


solution of the following equation in the 2x2 – 3x + 1 = 0
interval [0, 3600]; (x – 1) = 0 or (x - ) = 0 (By factorization)
5 sin θ tan θ – 10 tan θ + 3 sin θ – 6 = 0
x = 1 or x =
Solution ⇒ sin A = 1 or sin A =
5 sin θ tan θ – 10 tan θ + 3 sin θ – 6 = 0
A= (1) or A = . /
(5 sin θ tan θ – 10 tan θ) + (3 sin θ – 6 ) = 0
5 tan θ (sin θ – 2 ) + 3 (sin θ – 2 ) = 0 A = 900 or A = 300
(5 tan θ + 3 ) (sin θ – 2 ) = 0
For sin A = 1 , sin is + ve in the first and second
(5 tan θ + 3 ) = 0 or (sin θ – 2 ) = 0 quadrants.
A = 900,
tan θ = or sin θ = 2
θ= . / or θ = (2) For sin A = , sin is positive in the first and
θ= (2) has no solution because for every second quadrants
θ, -1 ≤ sin θ ≤ 1
A = 300, 1500
θ= . / Therefore, A = 300, 900, 1500 for the given
range
θ = 30 . 9638
θ = 310 (neareast degree)
2. Solve the equation 2 sin2 t – cos t – 1 = 0

tan is negative in the second and fourth


Solution
quadrants.
2 sin2 t – cos t – 1 = 0
(1800 – θ) , (3600 – θ )
But sin2 t + cos2 t = 1
1800 – 310 and 3600 – 310
sin2 t = 1 – cos2 t
θ = 1490 and 3290
2 (1 – cos2 t ) – cos t – 1 = 0
2 – 2 cos2 t – cos t – 1 = 0
Involving Quadratic Equations
-2 cos2 t – cos t – 1 + 2 = 0
1. Solve cos 2A + 3 sin A – 2 = 0 , [0, 2 ]
- 2 cos2 t – cos t + 1 = 0
Solution 2 cos2 t + cos t – 1 = 0
cos 2A + 3 sin A – 2 = 0 ,
Let cos t = x
But cos 2A = 1 – 2 sin2 A
2x2 + x – 1 = 0
(1 – 2 sin2 A) + 3 sin A – 2 = 0 ,
– 2 sin2 A + 3 sin A – 2 + 1 = 0 , (x + 1) = 0 or (x – ) (By factorization)

Baffour Ba Series, Further Mathematics for Schools Page 555


x = -1 or x = C. Solve each of the following for 0 ≤ θ ≤ 3600
1. cos 2θ + sin θ = 0 3. sin 2θ + sin θ = 0
⇒ cos x = -1 or cos x = 2. cos 4θ = cos 2θ 4. cos 2θ = 1 – 2sin θ
x = 1800 or x = 600
For cos x = -1 D. Approximate to the nearest 10‟ the solutions
cos is – ve in the second and third quadrants. of the equation in the interval [00 , 3600]
x = 1800 1. sin2 t – 4 sin t + 1 = 0
2. cos2 t – 4 cos t + 2 = 0
cos x = , cos is + ve in the first and fourth 3. tan2 t + 3 tan t + 2 = 0
quadrants. 4. 2 tan2 t – 3 tan t – 1 = 0
x = 600, 3000 5. 12 sin2 t – 5 sin t – 2 = 0
Thus x = 600, 1800, 3000 6. 5 cos2 t + 3 cos t – 2 = 0

Exercises 18.24 Challenge Problems


A. Find the solution of the equations that are Solve each of the following for 0 ≤ θ ≤ 3600
in the interval [0, 2 ] 1. tan θ + sec θ = 1
1. 2 sin2 u = 1 – sin u 2. 2 tan θ – sec2 θ = 0
2. 2 cos2 t + 3 cos t + 1 = 0 3. cot θ + tan θ = csc θ sec θ
3. tan2 x sin x = sin x 4. sec θ csc θ = 2 csc θ
4. sin2 θ + sin θ – 6 = 0 5. tan2 θ = 1 + sec θ
5. 2 sin2 θ + sin θ – 6 = 0 6. 2 sin θ + 1 = csc θ
6. 2 cos2 A + cos A = 0
B. Solve each for 0 ≤ θ ≤ 3600
B. Find the solution of the equations that are 1. tan2 + 4 tan = 0
in the interval [0, 2 ] 2. 6 sin2 – 5 sin + 1 = 0
1. 2 – 8 cos2 t = 0 3. cot2 – 3 cot + 2 = 0
2. cot2 θ – cot θ = 0 4. sec2 + 4 sec – 5 = 0
3. 1 – sin t = √ cos t
4. √ sin t + cos t = 1 Domain and Range of Trigonometric
5. cos α + sin α = 1 Function.
6. cos θ – sin θ = 1 Function Domain Range Period
f(x) = sin x All real 2
C. Solve each of the following for 0 ≤ θ ≤ 3600 numbers [-1, 1]
1. sin2 θ + 3 cos2 θ – 2 = 0 f(x) = cos x All real [-1, 1] 2
2. 3 – 3 cos θ = 2 sin2 θ numbers
f(x) = tan x All real All real
3. sin θ (2 sin θ – 1 ) = 0
numbers numbers
4. √ sin θ = 2 cos2 θ – 2
except +
5. √ sin θ cos θ – cos θ = 0
k , where k
6. (2 cos θ – 1) (cos θ + 1) = 0

Baffour Ba Series, Further Mathematics for Schools Page 556


is an When cos x = 1, y=1–3= -2
integer. The range is – 4 ≤ y ≤ -2
f(x) = cot x All real All real
numbers numbers 2. What is the range of the equation :
except k , y = 6 – 6 sin 3x
where k is
an integer.
Solution
f(x) = sec x All real (- , - 1] 2
y = 6 – 6 sin 3x
numbers ∪ [1, +
When sin 3x = 1
except )
y = 6 – 6 (1) = 0
f(x) = csc x All real (- , - 1] 2
numbers ∪ [1, + When sin 3x = -1
except k , ) y = 6 – 6 (-1) = 12
where k is
The range is 0 ≤ y ≤ 12
an integer

Application to Other Triangles


Domain and Range of Inverse Trigonometric
Notation
Function.
The diagram below shows the usual notation
for a triangle in trigonometry.
Function Domain Range
f(x) =
Angles: A, B, C
(x) {-1 ≤ x ≤ 1} 2 3 Length of sides: a, b, c
f(x) = {-1 ≤ x ≤ 1} * +
(x) A
f(x) = {-1 ≤ x ≤ 1} 2 3
c b
(x)
y 0 B C
f(x) = {-1 ≤ x ≤ 1} * + a
(x) y
The length of the sides are donated by a lower
f(x) = All real 2 3
numbers
case letters, and named after the angle they are
(x)
f(x) = All real * + opposite. i. e. a is opposite to angle A, b is
(x) numbers opposite to angle B and c is opposite to angle C

Worked Examples Sine and Cosine Rule, Area of a Triangle


1. Find the domain and range of y = cos (x ) – 3
A
Solution c b
Domain : x R C
B
a
Range : Sine Rule:
When cos x = -1, y = -1 – 3 = - 4 = =

Baffour Ba Series, Further Mathematics for Schools Page 557


Alternatively, triangle, but with right – angled triangles the
basic trigonometric ratios are used.

Cosine Rule (for length of sides): 4. The largest angle of a triangle is opposite the
= + – 2bc cos A largest side and the smallest angle is opposite
= + – 2ac cos B the shortest side. There can be only one obtuse
angle in a triangle.
= + – 2ab cos C
Tackling Problems in Trigonometry
Alternatively (for size of angles);
I. If not given, draw a diagram, and put in as
cos A = much information as possible.
II. If two, or more, triangles are linked redraw
cos B =
the triangles separately.
III. Watch for common sides which link the
cos C =
triangles (i.e. carry common values from one
triangle to another triangle).
Area of ∆ abc IV. Use the sine or cosine rule as needed.
A = ab sin C = ac sin B = bc sin A
Worked Examples
1. In ∆ abc, /ab/ = 3, /ac/ = 5 and /bc/ = 7.
When to Use the Sine Rule
Calculate:
Use the sine rule if you know:
i. the measure of the greatest angle of the
1. Two angles and one side.
triangle.
2. Two sides and an angle opposite one of these
ii. the area of ∆ abc, leaving your answer in the
sides.

form , where bis a prime.
When to Use the Cosine Rule
Use the cosine rule if you know: Solution
1. Two sides and an included angle. i. The largest angle is opposite the largest side
2. The lengths of the three sides. Using the cosine rule,
= + – 2bc cos A b
Note: = 52 + 32 – 2(5)(3) cos A
1. As a general rule, if you cannot use the sine 49 = 25 + 9 – 30 cos A 7
3
rule then use the cosine rule. 30 cos A = 25 + 9 – 49
A
30 cos A = - 15
2. If two angles are given, workout the third c 5 a
angle straight away, as the three angles in a cos A = =–
triangle add up to 1800. A= . /
3. The sine and cosine rules and the area of the
A = 1200
triangle formulas also apply to a right – angled

Baffour Ba Series, Further Mathematics for Schools Page 558


ii. Area of ∆ abc and one side, use the sine rule to find /ps/,
= bc sin A =
= (5) (3) sin 1200
r=
√ √
= (5) (3). / =
r = 8.82849883
Thus /ps/ = 8.83cm (2 d. p)
2. In the diagram below, /pq/ = 4cm, /pr/ =
5cm, /qr/ = 6cm and / <psr / = 220
p 3. In the figure below, /AB/ = 24cm, /BD/ =
17cm, AD = 13cm and < ACD = 900. Find /CD/
4 5 A
0
22
q r s
6
24cm
Find /ps/, correct to two places of decimals 13cm

B 17cm D C
Solution p
Work on the two angles 4 Solution
5
triangles separately Let x represent /CD/ A
R
q 6 r
I. Consider ∆ pqr, 24cm
Given three sides, 13cm
use the cosine rule;
B 17cm D x C
cos R =

cos R = = = Consider ∆ ABD,


( )( )
Using the cosine rule,
R= . / = + – 2(17)(13) cos D
R = 41.410 (2 d. p) 576 = 289 + 169 – 442 cos D A
 / <prs / = 1800 – 41.410 576 = 458 – 442 cos D
= 138.590 442cosD = 458 – 576 24
p
442cosD = -118 13

4 5 cos D = D
B 17 D
138.590
41.41 0 220
q r s A
6 p D= . / 0
r 0 15
II. Consider ∆ prs, D = 105 24cm
5 13cm
Having two angles 138.590 0 0
0 105 75
r
22 tan 150 = B
s 17cm D x C

Baffour Ba Series, Further Mathematics for Schools Page 559


tan 150= 3. In an acute – angled triangle ABC, the
x = 13tan 150 = 3.5 lengths of AB, AC are 10.3 cm , 15.7 cm
Hence /CD/ = 3.5cm respectively, and the angle C is 400, calculate to
the nearest tenth of a degree, the angle A .
5. Approximate the area of triangle ABC if a =
4. In ∆XYZ, XY = 17.2 cm, YZ = 21.3 cm, ZX
5cm, b = 3 cm and < A = 370
= 16.0 cm. Find the three angles and the area
of the triangle.
Solution
Applying the sin rule, 370
c 3 5. In ∆ABC, a = 9.5 cm, b = 8.4 cm, C = 730.
= Calculate;
B C i. the area of the triangle,
= =
5
ii. the side c,
iii. angles A and B.
Sin B =

B= . / B. 1. Calculate the three angles in triangle pqr


B = 210 below, correct to 2 places of decimals
p

C = 1800 – 370 – 210 2 3

C = 1220
q 4 r
Area of ∆ABC;
2. In the triangle below, cos A = . Without
= ab sin 1220
using tables or calculator, find the area of the
0
= (5) (3) sin 122 triangle.
6.4cm2
9cm
Exercises 18.25 A
Unless otherwise stated, where necessary, 10cm
give the lengths of sides and areas correct to
2 decimal places and give angles correct to 3. The area of the triangle below is 6cm2.
one place of decimals Find the value of x.
A. 1. In ∆ pqr, p = 7 cm, P = 300, Q = 840. Find
R, q and r
x

2. In ∆ abc,b = 8 cm, c = 10cm, and A = 600.


300
Find a (x + 2) cm

Baffour Ba Series, Further Mathematics for Schools Page 560


4. In the diagram below, From the diagram, ∆ABC is not a right – angled
p
triangle. Therefore, use the cosine rule to find
the value of the unknown side and unknown
4 5 the angles.
220 ⇒y2 = 662 + 482 – 2 (66)(48) cos 350
q r s
6 y 2 = 1469.85
/pq/ = 4cm, /pr/ = 5cm, /qr/ = 6cm and / <psr / y=√ 8
= 220. Find /ps/, correct to two places of y = 38 km
decimals.
From the diagram, < C
5. In ∆ ABC, A = 51, B = 680, and c = 4.5m. 482 = 662 + 382 – 2 (66)(38) cos C
Calculate; 482 – 662 + 382 = - 2 (66)(38) cos C
i. side b, ii. the area of ∆ABC. -3496 = - 5016 cos C
cos C =
Trigonometry and Bearings
I. Draw a diagram to represent the problem. C = cos –1. /
II. If the triangle formed is not a right angled C = 460(nearest degree)
triangle, use cosine rule or sine rule to
determine the value of an unknown side and the 350 + 450 + C + = 1800
values of unknown angles. 350 + 450 + 460 + = 1800
III. To find the bearing of say A from B, is to = 1800 – 1260 = 540
find the total angle turned through in the
clockwise direction from the north pole of B to But + θ = 900
the direction of A. 54 + θ = 90
θ = 90 – 54
Worked Examples θ = 360
1. B is 48km south – west of A, and C is 66km
from B on a bearing of 0800. Find, the distance Bearing of C from A;
and bearing of C from A. = 90 + θ
= 90 + 360 = 1260
Solution ⃗⃗⃗⃗⃗ = (38km, 1260)
A
0
y 2. A ship sails on a bearing of 1380 from a port
45
48km C P for a distance of 15km to a port Q and then
0
sails on a bearing of 0120 to a port R. If the
45 350 66km
bearing of R from P is 0530, find:
B i. the distance from Q to R;
ii. the ships final distance from P.
Let the distance of A to C be y

Baffour Ba Series, Further Mathematics for Schools Page 561


Solution Exercises 18.26
i. Let the distance from Q to R be ⃗⃗⃗⃗⃗⃗⃗⃗⃗⃗⃗ 1. An object Q is 6 km from P on a bearing
From the diagram, N200E, and object R is 7.5 km from P on a
(42 + 12)0 + (37 + 48)0 + θ = 1800 bearing N750E. Calculate the distance and
θ = 1800 – 540 - 850 bearing of Q from R.
θ = 410 R
θ 2. A ship sails due east at a steady speed of 24
Using sine rule, 0 km/h while a fishing boat is towed north – east
P 53 370 at a steady speed of 10km/h from the same
⃗⃗⃗⃗⃗⃗⃗⃗⃗⃗⃗ 0
= 48
( ) point. They start at the same time;
⃗⃗⃗⃗⃗⃗⃗⃗⃗⃗⃗ 15km 0
a. find the bearing of the ship from an observer
42 12
0
=
on the boat at the end of 2 hours.
⃗⃗⃗⃗⃗⃗⃗⃗⃗⃗ = Q
b. find the distance between the ship and the
boat at that time (2 hours after the starting time.
⃗⃗⃗⃗⃗⃗⃗⃗⃗⃗ = 22.7768km
⃗⃗⃗⃗⃗⃗⃗⃗⃗⃗ = 23km (nearest kilometer) 3. Town Q is 20km due north of P and the
bearing of town R from Q is 1400. If R is 8km
ii. Let the ships final distance from P be ⃗⃗⃗⃗⃗⃗⃗⃗⃗⃗ from Q, calculate;
⃗⃗⃗⃗⃗⃗⃗⃗⃗⃗ a. the bearing of R from P, to the nearest
= degree.
( ) R
⃗⃗⃗⃗⃗⃗⃗⃗⃗⃗ θ b. how far north of P, R is, correct to two
= significant figures.
0
53 370
P
⃗⃗⃗⃗⃗⃗⃗⃗⃗⃗ = 48
0
4. Kaya rides a bicycle to school every day.
⃗⃗⃗⃗⃗⃗⃗⃗⃗⃗ = 18.4972km 23km From home, he rides for 3.5km on a bearing of
15km
0370 and then 1.4 km on a bearing of 3350
0
0 12
⃗⃗⃗⃗⃗⃗⃗⃗⃗⃗ = 18km 42
before arriving at school.
Q a. What is the total distance Kaya rides from
home to school?
Alternatively; b. How far (i) north (ii) east is Kaya‟s school
Using the cosine rule; from home?
⃗⃗⃗⃗⃗⃗⃗⃗⃗⃗ = 152 + 22.7768)2 – 2(15)(22.7768) cos 540 c. find the bearing and distance of Kaya‟s
⃗⃗⃗⃗⃗⃗⃗⃗⃗⃗ = 743.7826 – 401.6360 school from his home.
⃗⃗⃗⃗⃗⃗⃗⃗⃗⃗ = 342.1466
5. Two ships leave port at the same time, one
⃗⃗⃗⃗⃗⃗⃗⃗⃗⃗ = √ sailing in the direction N 230 E at a speed of
⃗⃗⃗⃗⃗⃗⃗⃗⃗⃗ = 18.4972 11mi/hr and the second ship sailing in the
⃗⃗⃗⃗⃗⃗⃗⃗⃗⃗ = 18km direction S 670 E at 15mi/hr. Approximate the
bearing from the second ship to the first, one
hour later.
Baffour Ba Series, Further Mathematics for Schools Page 562
Challenge Problems Worked Examples
1. An aircraft flies 100 km from port A to port 1. Copy and complete the table of values for y
B on a course 0750, then 200 km from port B to = 1 – 4 cos x, to one decimal place for 00 x
port C on a course 3430. How far north and 3000
how far east are ports B and port C from port A.
X 00 300 600 900 1200 1500
Graphs of Trigonometric Functions Y -3.0 1.0
X 1800 2100 2400 2700 3000
Here is the graph of y = sin :
Y 4.5 -1.0

c. Use the graph


i. to solve the equation 1 – 4 cos x = 0
ii. find the value of y when x = 1050
iii. find x when y = 1.5

The graph of y = cos Solution


In y = 1 – 4 cos x
When x = 300, y = 1 – 4 cos 300
y = - 2.5
When x = 60 , y = 1 – 4 cos 600
0

y = - 1.0
When x = 120 , y = 1 – 4 cos 1200
0

y = 3.0
When x = 150 , y = 1 – 4 cos 1500
0

y = 4.5
The graph of y = cos x is the graph of y = When x = 1800, y = 1 – 4 cos 300
sin shifted, or translated y = 5.0
The graph of y = tan When x = 240 , y = 1 – 4 cos 2400
0

y = 3.0
When x = 2700, y = 1 – 4 cos 2700
y = - 1.0

X 00 300 600 900 1200 1500


Y -3.0 -2.5 -1.0 1.0 3.0 4.5
X 1800 2100 2400 2700 3000
Y 5.0 4.5 3.0 1.0 -1.0

Baffour Ba Series, Further Mathematics for Schools Page 563


b. d. Find, correct to one decimal place, the value
6 of y for which x = 720

5
Solution
4 a. When x = 00, y = 9 cos 00 + 5 sin 00
y= 9.0
3
When x = 600, y = 9 cos 600 + 5 sin 600
2 y = 8.8
1
When x = 900, y = 9 cos 900 + 5 sin 900
y = 5.0
0 When x = 00, y = 9 cos 1500 + 5 sin 1500
0 30 60 90 120 150 180 210 240 270 300
-1 y = -5.3

-2
X 00 300 600 900 1200 1500
-3 Y 9.0 10.3 8.8 5.0 -0.2 -5.3
-4
b.
Comparing 1 – 4 cos x = 0 to y = 1 – 4 cos x,
12
y=0
10
From the graph;
8
when y = 0, x = 750 and x = 2850
6
ii. From the graph, when x = 1050, y = 1.9
4
2
iii. From the graph, when y = 1.5, x = 970 and
0
x = 2640
-2 0 30 60 90 120 150 180

-4
2. a. Copy and complete the table of values for
-6
y = 9 cos x + 5 sin x to one decimal place
c. i. comparing 9 cos x + 5 sin x = 0 with y = 9
x 00 300 600 900 1200 1500 cos x + 5 sin x , y = 0
y 10.3 -0.2 From the graph, when y = 0, x = 1190

b. Using a scale of 2 cm to 300 on the x – axis ii. comparing 9 cos x + 5 sin x = 3.5 with y = 9
and a scale of 2 cm to 2 units on the y – axis, cos x + 5 sin x , y = 3.5
draw the graph of y = 9 cos x + 5 sin x for 00 ≤ From the graph, when y = 3.5, x = 990
x≤ 1500
c. Use your graph to solve the equations d. From the graph, when x = 720, y = 7.6
i. 9 cos x + 5 sin x = 0
ii. 9 cos x + 5 sin x = 3.5 3. Using a scale of 1 cm to on the x – axis and

Baffour Ba Series, Further Mathematics for Schools Page 564


2 cm to 1 unit on the vertical axis, draw the Hence, find the maximum and minimum values
graph of the function f defined by f(x) = 3 sin x of the function f in the given interval.
+ 2 cos x for 0 ≤ x≤ 2

Solution
Table of values for 3 sin x + 2 cos x

x 0 2
sin x 0 0.50 0.87 1 0.87 0.50 0 -0.50 -0.87 -1 -0.87 -0.5 0
sos x 1 0.87 0.50 0 -0.50 -0.87 -1 -0.87 -0.50 0 0.50 0.87 1
3sin x 0 1.50 2.61 3 2.61 1.50 0 -1.5 -2.6 -3 -2.6 -1.5 0
2 cos x 2 1.7 1.0 0 -1.0 -1.7 -2 -1.7 -1.0 0 1 1.7 2
f(x) 2 3.2 3.6 3 1.6 -0.2 -2 -3.2 -3.6 -3 -1.6 0.2 2

0
0 30 60 90 120 150 180 210 240 270 300 330 360 390
-1

-2

-3

-4

From the graph,


the maximum value is 3.6
the minimum value is – 3.6

Baffour Ba Series, Further Mathematics for Schools Page 565


Exercises 18.27 2. The graph has a period
A. 1. Construct a table of values for y = 3 sin x
+ 2 cos x for 00 x 3600
Worked Examples
b. Using a scale of 2 cm to 500 on the x –axis
1. Find the amplitude and the period of the
and 2 cm t 1 unit on the y – axis, draw the
graph y = 3 sin 2x
graph of for y = 3 sin x + 2 cos x for 00 x
3600
Solution
c. Use your graph to solve the equations;
y = 3 sin 2x compared to y = a sin bx
i. 3 sin x + 2 cos x = -1
a = 3 and b = 2
ii. 3 sin x + 2 cos x < 1.5
Amplitude : /a / = /3/ = 3

2. a. Copy and complete the table for y = sin x


+ 2 cos x, correct to 1 decimal place Period : =| |= =

2. Find the amplitude and period of the graph:


0 0 0 0
x 0 30 60 90 y = 2 sin x
y 2.2

x 1200 1500 1800 2100 2400 Solution


y -1.2 -2.0 y = 2 sin x compared to y = a sin bx
a = 2 and b =
b. Using a scale of 2cm to 300 on the x – axis
and 2 cm to 0.5 units on the y axis, draw the Amplitude : /a / = /2/ = 2
graph of y = sin x + 2 cos x for 00 x 2400.
c. Use your graph to solve the equations; Period : = = =4
| |
i. sin x + 2 cos x = 0 ii. sin x = 2.1 – 2 cos x
0
d. Find y when x = 171 . 3. Find the amplitude and period of the the
equation y = 2 sin (-3x)
B. Solve the following equations graphically
for the values of x in the – 1800 ≤ x ≤ 1800, Solution
using a suitable scale y = 2 sin (-3x)
1. 4 sin 2x + 3 cos 2x = 0 y = -2 sin 3x
2. 4 sin 2x + 3 cos 2x – 1 = 0
3. 2 sin 2x + 1.5 cos 2x – 1 = 0 y = -2 sin 3x compared to y = a sin bx
a = - 2 and b = 3
Finding an Amplitude and a Period Amplitude : /a / = /-2/ = 2
If y = a sin bx or y = a cos bx for a non zero
real number a and b, then :
Period : =| |= =
1. The graph has an amplitude /a/ ;

Baffour Ba Series, Further Mathematics for Schools Page 566


4. Find the amplitude and period of the the a = R cos α ………………(1)
equation y = 4 cos x b = R sin α……………… (2)

Solution These can be used to find the values for R and


y = 4 cos x compared to y = a cos bx α by squaring each of equations (1) and (2)
a = 4 and b = and adding them.
Amplitude : /a / = /4/ = 4 a2 + b2 = R2 cos2 α + R2 sin2 α
= R2 (cos2 α + sin2 α)
Period : =| |=2 = R2

since cos2 α + sin2 α = 1.


Exercises 18.28
Hence, R = √
A. Find the amplitude and period of the
following equations. It is conventional to choose only the positive
1. y = sin x 4. y = sin x square root, and hence R will always be
2. y = - 4 sin x 5. y = sin (- 4x) positive.
3. y = 2 sin x 6. y = sin x
The value of α can be found by dividing
equation (2) by equation (1) to give;
B. Find the amplitude and period of the
following equations. = ⇒ =
1. y = 3 cos x 4. y = cos x
Hence, tan α =
2. y = cos 3x 5. y = cos (- 3x)
α =
3. y = 2 cos x 6. y = cos x

Therefore, any expression of the form:


The expression Rcos (x − α)
a cos x + b sin x can be expressed in the form
Using the addition formula for cosine to obtain
R cos(x − α).
the expansion of R cos (x – α):
R cos (x − α) = R (cos x cos α + sin x sin α) Note
= R cos x cos α + R sin x sin α 1. a cos x + b sin x can be written as
Rcos (x − α) where :
Re – arranging the expression follows:
Rcos (x − α) = (R cos α) cos x + (R sin α) sin x R=√ and tan α =
2. When using tan α = to find the value of α,
Thus, to write an expression of the form:
care must always be taken to ensure that α lies
a cos x + b sin x in the form, R cos (x − α);
in the correct quadrant. In particular, if either or
compare a cos x + b sin x with the expression
both of a and b are negatives care must be
(R cos α) cos x + (R sin α) sin x to observe that;
taken.

Baffour Ba Series, Further Mathematics for Schools Page 567


Worked Examples α < . In each case, determine the values of
Each of the following expressions can be R and α (in radians) correct to 3 decimal
written in the form R cos (x − α) with − < α < places.
. In each case determine the values of R and α 1. 3 cos x − sin x 2. 6 cos x + 5 sin x
(in degrees) correct to the nearest degree. 3. - 5 cos x + 12 sin x 4. 4 cos x − sin x
1. 5 cos x + 12 sin x 2. 3 cos x – sin x 5. - 2 cos x − 3 sin x 6. - cos x + 3 sin x
7. cos x + sin x 8. cos x − sin x
Solution 9. sin x − cos x) − (cos x + sin x)
1. 5 cos x + 12 sin x compared to a cos x + 12
sin b Application to solving an equation
a = 5 and b = 12 Worked Example
But R = √ 1. Solve the trigonometric equation:
R=√ √ cos x + sin x = 1 for values of x in the
R=√ interval − < α < .
R = 13
Solution
tan α = √ cos x + sin x = 1 and a cos x + b sin x
compared, then a = √ and b = 1
α=
α= But R = √
0
α = 67
R = √(√ )
2. 3 cos x – sin x compared to a cos x + 12 sin b R=√
a = 3 and b = -1
But R = √ Also,
R=√ ( ) Rcos α = a and Rsin α = b
R=√ Therefore, cos α = and sin α =

cos α = and sin α =
tan α = √ √

α= The fact that sin α and cos α (and therefore tan


α= α) are all positive mean that α is an angle in the
first quadrant.
α = 180 (2nd quadrant)
α = 1800 – 180 = 1620 tan α =
α=
Exercises 18.29
Each of the following expressions can be α=

0
written in the form R cos (x − α) with − < α = 35

Baffour Ba Series, Further Mathematics for Schools Page 568


Change 350 to radians so, with the known values of R, a and b,
350 × = 0.615 radians (3 d.p.) cos α = and sin α = −

Therefore, Rcos(x − α) = √ cos (x – 350). The fact that sin α is negative and cos α is
√ cos x + sin x = 1⇔ √ cos (x – 350 ) = 1 positive mean that α is an angle in the fourth
quadrant.
From √ cos (x – 350 ) = 1 tan α =
cos (x – 350) = α=

0
x – 35 = √
√ α=
0 0
x – 35 = 55 α = - 600
x = 550 + 350
x = 900 Therefore, R cos (x − α) = 2 cos (x − (− 60◦)
= 2 cos (x + 60◦).
Now if x lies in the interval − < α < , then
So the equation cos x − √ sin x = 2 becomes;
x = - 600, 1200
2 cos (x + 60◦) = 2
x − 0.615 = − 0.955, 0.955 cos (x + 60◦) = 1
x = − 0.955 + 0.615, 0.955 + 0.615 x + 60 = 1
x = − 0.340, 1.570 x + 60 = 0
x = - 60
2. Solve the equation cos x − √ sin x = 2 for
cos is positive in the first and fouth quadrants.
values of x in the interval 00 ≤ x ≤ 3600.
In the interval 0◦ < x < 360◦, x is – 600 in the
Solution first quadrant.
First express cos x − √ sin x = 2 in the form R
In the fourth quadrant , the only angle in the
cos (x − α).
interval with cosine equal to 1 is 360◦. It
Comparing cos x − √ sin x = 2 with follows that;
a cos x + b sin x , we see that; x + 60◦ = 360◦
a = 1 and b = − √ x = 300◦
Thus x = - 600 , 3000.
R=√ The only solution lying in the given interval is
x = 300◦.
R=√ ( √ )
R=√ Exercises 18.30
R=2 Solve the following equations for 0 < x < 2
1. 2 cos x + sin x = 1 4. 2 cos x − sin x = 1
But R cos α = a and R sin α = b 2. - 2 cos x − sin x = 1 5. cos x − 2 sin x = 1

Baffour Ba Series, Further Mathematics for Schools Page 569


3. cos x + 2 sin x = 1 6. - cos x + 2 sin x = 1 Solution
f(x) = 3 cos x + 4 sin x
Finding maximum and minimum values of a compared to a cos x + b sin x
trigonometric function a = 3 and b = 4
Type 1 R=√
Given f(x) = a cos x + b sin x R=√
1. Express the given function in the form R=5
R cos (x – α):
2. Since the maximum value of the cosine Now, the maximum value of the cosine
function is 1, it follows that cos (x – α) = 1. function is 1
Hence, the maximum value of f(x) = R(1) = R ⇒ cos (x – α) = 1
and this occurs when the angle x – α = 0
3. The minimum value of the cosine function is Maximum value of f(x);
- 1 it follows that cos (x – α) = - 1. Hence, the = R cos (x – α )
minimm value of f(x) = R (-1) = - R and this = 5 (1)
occurs when x − α = , i.e. when x = + α. =5

Type 2 Angle at which the maximum value occurs is;


Given f(x) = a cos x + b sin x k x − α = 0, i.e. when x = α.
1. Express the given function in the form Now,
R cos (x – α) k: tan α =
2. Since the maximum value of the cosine
function is 1, it follows that cos (x – α) = 1. α=
Hence, the maximum value of f(x) = R(1) k = α=
R k and this occurs when the angle x – α = 0.
α = 530
3. The minimum value of the cosine function is
- 1 it follows that cos (x – α) = - 1. Hence, the Therefore, the maximum value of the f(x) is 5
minimm value of f(x) = R (-1) k=-R k and it occurs at x = 530
and this occurs when x − α = , i.e. when
x = + α. The minimum value of the cosine function is
−1 and this occurs when x − α = , i.e. when
Worked Examples x = + α.
1. Determine the maximum values of f(x) = 3
cos x + 4 sin x and the smallest positive angle So the minimum value of f(x) ;
= R cos (x − α) , if cos (x – α) = -1
(to the nearest degree) at which the maximum
=5×-1
value occurs. Find the minimum value and the =-5
angle at which it occurs. Therefore, the minimum value of the f(x) is -5
and it occurs at x = 1800 + 530 = 2330

Baffour Ba Series, Further Mathematics for Schools Page 570


2. Given f(x) = 4 cos x + 3 sin x − 3. Find the Therefore, the minimum value of the f(x) is -8
maximum and minimum values of the function. and it occurs at x = 1800 + 370 = 2170

Solution Exercises 18.31


f(x) = 4 cos x + 3 sin x – 3 For each of the following functions determine
compared to a cos x + b sin x the maximum value and the smallest positive
a = 4 and b = 3 angle (in radians, to three decimal places) at
R=√ which the maximum value occurs.
1. f(x) = 6 + 3 cos x + 4 sin x
R=√
2. f (x) = 3 − 4 cos x + 3 sin x
R=5
3. f (x) = 1 − 3 cos x − 4 sin x
4. f (x) = 2 + cos x − sin x
⇒ R cos (x – α) – 3
= 5 cos (x − α) − 3
Summary
A. 1. f(x) = a sin x b cos x
tan α =
2. f(x) = a sin x b sin x
α= 3. f(x) = a cos x b cos x
α= Max value = √
α = 370 Min value = - √

Now the maximum value of the cosine function B. f(x) = (sin x cos x)n
is 1 and this occurs when the angle Min value = . /
x − α = 0, i.e. when x = α. Max value can go up to infinity
So the maximum value of f(x)
= 5 cos (x − α) − 3 , if cos (x – α) = 1 Worked Examples
=5×1−3 1. Find the maximum and minimum values of
= 2. f(x) = 3 sin x 4 cos x
Therefore, the maximum value of the f(x) is 2
and it occurs at x = 370 Solution
Max value = √ =5
The minimum value of the cosine function is
Min value = - √ =-5
−1 and this occurs when x − α = , i.e. when
x = + α.
2. Find the minimum value of f(x) = sin4x cos4 x

Hence, the minimum value of f(x) ;


Solution
= 5 cos (x − α) − 3 , if cos (x – α) = -1
f(x) = sin4x cos4 x = (sin x cos x)4
=5×-1−3
= - 8. Min value = . / =

Baffour Ba Series, Further Mathematics for Schools Page 571


C. f(x) = a sin2 x b cos2 x 2. f(x) = a cos2 x b sec2 x
If a > b, max value = a and min value = b 3. f(x) = a tan2 x b cot2 x
If a < b, max value = b and min value = a Minimum value = 2√( )
Maximum value can go up to infinity
Note:
If the function is of tis type f(x) = a sin2 x b Note.
cos2 x, then the maximum value is the largest In each function, each term is the reciprocal of
of (a, b) and the minimum is the smallest of (a, the other .
b)
Worked Examples
Worked Example Find the minimum value of f(x) = 4 cos2 x 9
Find the maximum and minimum values of f(x) sec2 x
= 3 sin2 x + 5 cos2 x
Solution
Solution f(x) = 4 cos2 x 9 sec2 x
f(x) = 3 sin2 x + 5 cos2 x a = 4 and b = 9
a = 3 and b = 5
Minimum value = 2√( )
a<b
Maximum value = 5 Minimum value = 2√( )
Minimum value = 3 Minimum value = 2√( )
Minimum value = 2 (6) = 12
D. 1. f(x) = a sin2 x b cosec2 x

Baffour Ba Series, Further Mathematics for Schools Page 572


19 LIMITS OF FUNCTIONS Baffour – Ba Series

Limits of Functions at Infinity Solution


Definition lim . /
Let f be a function such that f(x) is defined for
sufficiently large x. Suppose L is a real number = lim – lim . /
satisfying the following condition: = 1 – 2. lim
= 1 – 2 (0)
f(x) is arbitrary close to L if x is sufficiently
=1
large. This condition means that if we let x
increase without bound, then the value f(x)
2. Find lim ( ), if it exists.
approaches L. Thus, we say that L is the limit
of f at infinity, expressed as lim ( )=L
Solution
lim ( )
Rules for Limits of Functions at Infinity
= lim + lim
1. lim = k , where k is a constant
= lim +3
2. lim = 0, where P is a positive constant =0+3
=3
3. lim = 0, where k is a constant greater
Type 2
than 1.
When the limits of the numerator and the
denominator do not exist, rule 7 cannot be
4. lim ( ( ) ( )) = lim ( )
applied. In this case, divide the numerator and
lim ( ) This result is valid for sums and denominator by the highest power of x, so that
difference of finitely many functions. limits at infinity of the new numerator and
denominator exist.
5. lim ( ( ) ( )) = lim ( )
lim ( ). This result is valid for product In general, for all rational functions, limits at
of finitely many functions. infinity can be found by dividing the numerator
and the denominator by a suitable power of x
6. lim ( ( )) = k . lim ( ), (usually the highest power).
( ) ( )
7. lim = , where Worked Examples
( ) ( )
lim ( ) ≠0 1. Evaluate lim

Worked Examples Solution


1. Find lim . / if it exists. lim

Baffour Ba Series, Further Mathematics for Schools Page 573


Divide numberator and denorminator by the = =
hihest power of x

lim ⁄ ⁄
Exercises 19.1
A. For each of the following , find the limit if
= lim ⁄ it exists.
= = lim ⁄
= =3 1. lim 6. lim

2. lim 7. lim
2. Find the lim , if it exists.
3. lim
4. lim √
Solution 5. lim ( )
= lim
B. Evaluate each of the following limits.
1. lim 4. lim
= lim ,
2. lim 5. lim
. /
= ,
. /
3. lim 6. lim
=
Challenge Problem
= =0 The concentration C of a drug in a patient‟s
bloodstream t hours after it was injected is
3. Find the lim , if it exists. given by C(t) =
a. Find lim ( )
Solution b. Interpret the results in (a)
= lim
Rules for Limits of Functions at a Point
1. lim = k , where k is a constant, k R.
= lim ,
2. lim = , where a R and n is a
( * positive integer.
= ,
. /

. / 3. lim √ = √ , where a R and n is an odd


= , positive integer.
. /

= lim √ = √ , where 0 < a R and n is an


even positive integer.

Baffour Ba Series, Further Mathematics for Schools Page 574


4. lim = , where a R and b is a positive Solution
real number. ( )
lim = ( )

5. lim ( ( ) ( )) = lim ( ) = =
lim ( ). The result is valid for sums and
difference of finitely many functions. 3. Evaluate lim

5. lim ( ( ) ( )) = lim ( ) Solution


lim ( ) where k is a constant. The result
lim
is valid for product of finitely many functions.
Put in 3 for x;
6. lim ( ( )) = k . lim ( ), where k = = =4
is a constant.
4. Evaluate lim
( ) ( )
7. lim = , provided that
( ) ( )
lim ( ) ≠ 0. Solution
lim
Note: Put in 1 for x;
1. Let p(x) be a polynomial and let a be a real ( )
= = =
number, then lim ( ) = p(a). Hence, the
limit of polynomial functions at any real
number can be found by substitution. Indeterminate Forms of Limit
If a rational function is undefined at x = a, it
2. Let p(x) and q(x) be polynomials and let a be means that a does not belong to the domain of f.
a real number. Suppose that q(a) ≠ 0. Then we Thus, when we substitute x = a in the
( ) ( ) numerator and denominator, the result is . The
have lim = .
( ) ( )
limit is said to be in the indeterminate form .
Worked Examples For example in lim , when x =1 in
1. Find lim ( ) , if it exists.
both the numerator and denominator is
Solution obtained. In this case , it is advisable to either
lim ( ) , if it exists. factorized the numerator or denominator first,
= lim + lim then simplify and take the limit at the given
= 1 + 42 point.
= 17
Worked Examples
2. Find lim , if it exists. 1. Find lim , if it exists.

Baffour Ba Series, Further Mathematics for Schools Page 575


Solution Factorize numerator and denominator
The rational function is undefined at x = 1. This lim
means that 1 does not belong to the domain of f.
( )( )
When x = 1 in both the numerator and lim ( )( )
denominator, we get .
lim
Factorizing the denominator gives
f (x) = ( )(
Put in 2 for x;
)
( )
lim = lim ( )( = = =3
)

= lim
= = 4. Evaluate lim

2. Evaluate lim Solution

lim
Solution
lim Put in x = 1; = (indeterminate form)
Put in 5 for x; Factorizing numerator and denominator will not
= (indeterminate form) work. Thus, use long divison to find the
quotient
Thus, factorize numerator and denominator
( )( ) x2 + x + 1
lim
x – 1 x3 + 0x2 + 0x – 1
lim x3 – x2

Put in 5 for x x2 + 0x – 1
x2 – x
= 5 + 5 = 10
x–1
x–1
3. Evaluate lim
= x2 + x + 1
Solution
lim
lim = lim ( )
Put in 2 for x; = 12 + 1 + 1
= (indeterminate)
=3

Baffour Ba Series, Further Mathematics for Schools Page 576


Exercises 19.2 ( )( √ )
= lim ( )
A. Find the limit if it exists.
1. lim ( ) = lim
( )( √ )

2. lim ( 8)
√ ( )( √ )
3. lim . / 4. lim = lim
( )( √ )
5. lim 6. lim = lim ( )

B. Find the limit if it exists. lim ( √ )=1+√


=1+1=2
1. lim 4. lim

2. lim 5. lim 2. Evaluate lim

3. lim 6. . lim Solution



lim
Evaluating Limits that Contain Surds
If the limit is initially indeterminate, multiply By substitution;

the numerator and denominator by the lim =
conjugate surd and then divide the numerator
and the denominator by the common factor. As Rationalize the numerator;
you proceed however, do not multiply out the (√ )(√ )
surd and the non-surd parts, as this will prevent lim ( )(√ )
you from finding the common factor. √
lim
( )(√ )
Worked Examples lim ( )(√ )
1. Evaluate lim
√ lim

Solution
By substitution;
lim
√ = =

lim = (indeterminate)

Multiply top and bottom by 1 + √ (the conjugate of Exercises 19.3
the numerator) Evaluate each of the following limits:
√ √
lim = lim . 1. lim 4. lim
√ √ √ √
( )( √ )
= lim 2. lim

5. lim
( √ )( √ ) √
( )( √ ) √
= lim 3. lim 6. lim
(√ ) √
( )

Baffour Ba Series, Further Mathematics for Schools Page 577


Equations Involving Limits lim
Worked Examples Substituting 0 for x, you find that
( )
If lim = 4, find lim ( ) cos x approaches 1 and sin x − 3 approaches - 3;
hence,
Solution lim =
( )
lim = 4,
( ) 2: Evaluate lim
=4
( )
=4
Solution
( )
=4 lim
= lim ( *
2 lim ( ) – 5 = 20
2 lim ( ) = 20 + 5 = (lim ) . lim
2 lim ( ) = 25 =4.1
lim ( ) = =4
lim ( ) = 12.5 lim =4

Limit of Simple Trigonometry Functions 3: Evaluate lim


The trigonometric functions sine and cosine
have four important limit properties:
Solution
lim sin = sin c
lim cos = cos c lim
lim =1 But sec x = ,
lim =0
= lim
These properties can be used to evaluate many
= lim
limit problems involving the six basic
trigonometric functions. = lim . /.. /
= 0lim 1 . 0lim 1.
Worked Examples
1: Evaluate lim =-1.0
lim =0
Solution

Baffour Ba Series, Further Mathematics for Schools Page 578


20 DIFFERENTIATION Baffour Ba Series

Differentiation from the first Principles of a line will give the rate of change of y with
Differentiation or differential calculus, is the respect to (the change in) x.
branch of mathematic measuring rate of
change. Slope of a Curve
Consider the curve below and the tangents that
Slope of a Line are drawn on it.
Below is the graph of line y = 3x
y
 

9 

6 The slope of the curve at a point is equal to the


slope of the tangent at that point. As we move
3
along the curve, the slope of each tangent
x
1 2 3 changes. In other words, the rate of change of y
There is a relationship between x and y. For with respect to x changes. We need to find a
every increase in x, there is three times increase method of finding the slope of the tangent at
in y. each point on the curve. The method of finding
the slope of a tangent to a curve at any point on
Rate of change of y = 3 times rate of change the curve is called differentiation.
of x.
Notation
From coordinate geometry, the slope of y = 3x We will now develop a method for finding the
is 3. slope of the tangent to the curve y = f(x) at any
Slope = 3 point (x, f(x)) on the curve.
⇒ Rate of change = slope
The graph shown below represent the function
The key word here is slope. The slope of a line y = f(x).
will give the rate of change of the variable on y
the vertical axis with respect to the variable on
the horizontal axis. Therefore, to find the rate (x + h, f(x + h)
of change is to find the slope.
f(x + h) – f(x)

Note: (x, f(x)


The y – axis is usually the vertical axis and the h
x
x – axis is the horizontal. Therefore, the slope

Baffour Ba Series, Further Mathematics for Schools Page 579


(x, f(x)) is a point on the curve and (x + h, f(x+ respect to x. The is an operator. means
h)) is a point further along the curve.
S is a line through these points. “the derivative with respect to x ”. Thus, is
T is a tangent to the curve at the point (x, f(x)). usually written as (y).

Slope = Definition
The slope of a tangent to the curve y = f(x) at
( ) ( )
= ( ) any point on the curve is given by :
( ) ( )
( ) ( ) = f’(x) = lim
=

This would be a good approximation of the Differentiation from the first principles
slope of the tangent T, at the point (x, f(x)), if (x involves four steps:
+ h, f(x + h)) is very close to (x, f(x)). By letting Find:
h get smaller, the point (x + h, f(x + h) moves 1. f(x + h)
closer to (x, f(x)). 2. f(x + h) – f(x)
( ) ( )
3.
The result is that the slope of S gets closer to ( ) ( )
the slope of T. In other words, as h approaches 4.
0, the slope of S approaches the slope of T.
Mathematically speaking, we say that the slope There are six functions, x2, x3, √ , , sin x, cos
of T is equal to the limit of the slope of S as h x on our course that we can be asked to
approaches 0. It is important to realize that h differentiate from the first principles.
approaches 0, but never actually becomes equal
to zero. Worked Examples
( ) ( )
This is usually written lim Differentiate from the first principles with
The process of finding this limiting value is respect to x :
called “differentiation”. For neatness, this i. x2, ii. x3, iii. √ , iv. , v. sin x, vi. cos x
limit is written as (pronounces as dee y, dee
x) of f’(x), pronounce “f dash of x” of “f prime Solution
of x”. f(x) = x2
f(x + h) = (x + h)2
or f’(x) is called the “differential coefficient
= x2 + 2xh + h2 – x2
“ or “first derivative of y with respect to x” = 2xh + h2
( ) ( )
= 2x + h
Note:
( ) ( )
It is important to understand that does not lim = lim
mean dy dx. It means the derivative of y with = 2x
Thus (x) = 2x

Baffour Ba Series, Further Mathematics for Schools Page 580


ii. f(x) = x3 = ( )
f(x + h) = (x + h)3
( )– ( )
f(x + h) – f(x) = (x + h)3 – f(x) = ( )
= x3 + 3x2h + 3xh2 + h3 – x3 ( )– ( )
lim = lim
= 3x2h + 3xh2 + h3 ( )

=-
( ) ( ) 2 2 2
= 3x h + 3xh + h Thus (x) = -
( ) ( )
= lim
= lim ( ) v. f(x) = sin x
= 3x2 f(x + h) = sin (x + h)
Thus, (x) = 3x2 f(x + h) – f(x) = sin (x + h) – sin x
= 2 cos . / sin . /
iii. f(x) = √
= 2 cos . / sin . /
f(x + h) = √
( )– ( ) . /
f(x + h) – f(x) = √ -√ = 2 cos . /
√ √ √ √
= × ( )– ( )
√ √
lim =
(rationalise the numerator)
. /
=

lim cos . /

= = (cos x) (1)
√ √
( ) ( ) = cos x
= h
√ √ Thus, (x) = cos x
=
√ √
( ) ( ) vi. f(x) = cos x
lim = lim
√ √ f(x + h) = cos (x + h)
= f(x + h) – f(x) = cos (x + h) – cos x
√ √
= = -2 sin . / sin . /

Thus (x) = = -2 sin . / sin . /

( )– ( ) . /
= -2 sin . /
iv. f(x) =
. /
f(x + h) = = -sin . /
f(x + h) – f(x) = - ( )– ( )
( )
lim =
= ( )

Baffour Ba Series, Further Mathematics for Schools Page 581


. / ( )– ( )
lim sin . / =

= (- sin x) (1) = + +
= -sin x = 2x + h + 4
Thus, (x) = - sin x ( )– ( )
lim = lim
( )– ( )
Let f(x) = x2 + 3 . Find lim
= 2x + 4

Solution 2. Find from the first principles the derivative


( )– ( )
=
(( ) ) ( ) of x3 + x.
( ) ( )
= Solution
= Let f(x) = x3 + x.
( )
f(x + h) = (x + h)3 + (x + h)
= = x3 + 3x2 h + 3xh2 + h3 + x + h
= 2x + h
f(x + h) – f(x)
( )– ( ) = x3 + 3x2 h + 3xh2 + h3 + x + h – (x3 + x)
lim = lim
= x3 + 3x2 h + 3xh2 + h3 + x + h – x3 – x
= 2x + 0 = x3 – x3 + 3x2 h + 3xh2 + h3 + x – x + h
= 2x = 3x2 h + 3xh2 + h3 + h

Some Solved Past Questions ( )– ( )


1. Find from the first principles the derivative =
of f : x → (x + 2)2 = + + +
= 3x2 + 3xh + h2
Solution
f : x → (x + 2)2
( )– ( )
f(x) = (x + 2)2 lim = lim
f(x) = (x + 2) (x + 2) = 3x2
f(x) = x2 + 4x + 4
3. Find from the first principles, the derivative
f(x + h) = (x + h)2 + 4 (x+ h) + 4 of ( ) , where x ≠ 0.
f(x + h) = x2 + 2xh + h2 + 4x+ 4h + 4
f(x + h) – f(x) Solution
= x2 + 2xh + h2 + 4x+ 4h + 4 – (x2 + 4x + 4) Let f(x) = ( )
= x2 + 2xh + h2 + 4x+ 4h + 4 – x2 – 4x – 4
f(x) = +
= x2 – x2 + 2xh + h2 + 4x – 4x + 4h + 4 – 4
= 2xh + h2 + 4h f(x) = 3x + x23

Baffour Ba Series, Further Mathematics for Schools Page 582


f(x + h) = 3(x + h)3 + (x + h)2 Differentiation by Rule
= 3[x3 + 3x2 h + 3xh2 + h3] + x2 + 2xh + h2 Differentiation from first principles can be
= 3x3 + 9x2 h + 9xh2 + 3h3 + x2 + 2xh + h2 tedious and difficult. Fortunately, it is not
= 3x3 + x2 + 9x2 h + 9xh2 + 2xh + 3h3 + h2 always necessary to use first principles. There
are few rules (which can be derived from first
f(x + h) – f (x) principles) which enables us to write down the
= 3x3 + x2 + 9x2 h + 9xh2 + 2xh + 3h3 + h2 – derivative of a function quite easily.
[3x3 + x2]
= 3x3 + x2 + 9x2 h + 9xh2 + 2xh + 3h3 + h2 – Rule 1 : General Rule
3x3 – x2 1. If y = xn, then =n
= 9x2 h + 9xh2 + 2xh + 3h3 + h2
2. If y = axn, then = na
( )– ( )
= In words, multiply the power and reduce the
= 9x2 + 9xh + 2x + 3h2 + h power by 1.

( )– ( )
lim = 9x2 + 2x Worked Examples
Differentiate with respect to x.
Exercises 20.1 1. y = x5 5. y = 6 √
A. For each of the following f, find 2. y = -3x2 6. y =

( )– ( )
lim 3. y = 5x 7. y = ⁄
2
1. f(x) = 4x – 13 4. f(x) = 2x – 4x = 1 4. y = 8. y = 7
2. f(x) = x4 5. f(x) = 1 – 3x2
3. f(x) = 3x2 - 4x A 6x – 4 6. f(x) = √ Solution
1. y = x5 =5 = 5x4
B. Find the derivatives of each of the
following functions by the first principles. 2. y = -3x2 = 2 × -3 = - 6x
1. f(x) = 2x + 5 4. f(x) = 3x – x2
2. f(x) = (x – 1)2 + 1 5. f(x) = 6x2 – 3x – 1 3. y = 5x =1×5 = 5x0 = 5
3. f(x) = x3 + 3x2 + 5 6. f(x) = - 3x2 + 4x
4. y = = -2 × 8 = -16
C. For each of the following, find the =-
derivative using first principles.
⁄ ⁄
1. f(x) = + 3 4. f(x) = +x 5. y = 6 √ = × 6 =3

2. f(x) = = ⁄ =

2
3. f(x) = 5. f(x) = x 6. y = = × 2 ⁄

Baffour Ba Series, Further Mathematics for Schools Page 583



= -1 = ⁄ = lim

7. y = = × 6 ⁄ = lim

= lim

= -2 = ⁄ =1
8. y = 7 =0×7 =0 Geometrically, this means that the graph of the
identity function is the line given by y = x. At
Derivative of a Constant every point on the line, the slope is 1.
The derivative of a constant is zero. Take for
instance, the line y = 7. This line is a horizontal Power Rule for Differentiation
line with slope 0. Therefore, its derivative (also (Positive Integer Version)
known as slope) equals 0. Let n be a positive integer, then the power
function xn differentiable on R is given by
In general, the derivative of a constant is =n , where x is the identity function.
always 0. That is:
= 0, where c is a constant Worked Examples
Let y = x123. Find
Proof
Let f(x) = c Solution
( ) ( )
= lim y = x123
= lim =

= lim = 123

= lim = 123
=0
Constant Multiple Rule for Differentiation
Geometrically, this means that the graph of a Let f be a function and k be a constant. Suppose
constant function c is the horizontal line given f is differentiable, then the function kf is also
by y = c. At every point on the line, the slope is 0. differentiable. Moreover, we have;
(kf) (x) = k ( ),
Derivative of Identity Function
The derivative of an identity function is the
Proof
constant function 1, that is;
Let f(x) = x
= 1, where x is the identity function. ( )( ) ( )( )
( ) ( ) = lim
Let f(x) = x ( ) ( )
( ) ( ) ( ) ( ) = lim
= lim

Baffour Ba Series, Further Mathematics for Schools Page 584


( ) )( )
( ) ( ) = lim . /
( ) ( )
Proof
( ) ( ) = k.lim (f + g)1 (x)
( ) ( ) = k. (x), = lim
( )( ) ( )( )

, ( ) ( )- , ( ) ( )-
Worked Examples = lim
( ) ( ) ( ) ( )
Let y = 3x4. Find = lim
( ) )( ) ( ) ( )
= lim . /
Solution ( ) )( ) ( ) ( )
y = 3x4. = lim + lim
= 3x4 = (x) + (x)

= 3. x4 2. The results is also true for difference of two


= 3. ( ) functions, that is;
= 12x3 (f - g) (x) = ( )– ( )

Exercises 20.2 This result for difference can be proved similar


Differentiate each of the following with to that for sum.
respect to x.
Term by term differentiation can be applied to
1. x3 2. 3x4 3. -5x2 4. 3x 5. -2x
sums and difference of finitely many terms.
6. 5 7. -3 8. 9. 10.
Worked Examples
B. Differentiate each of the following with 1. Find (x) for each of the following:
respect to x. a . f(x) = x + b. f(x) = – +5
⁄ √
1. 6 2. 3. √ 4. 5. ⁄

Solution
Derivative of Sum or Difference 1. a. f(x) = x +
If the expression to be differentiated contains
f (x) = x +
more than one term, just differentiate,
(x) = 1 –
separately, each term in the expression.
(x) = 1 –
1. Let f and g be functions with the same
denominator. Suppose that f and g are b. f(x) = – +5

differentiable. Then the function f + g is also f (x) = 2 ⁄
– +5
differentiable. Moreover, we have; ⁄
=- –
(f + g) (x) = ( )+ ( )
= ⁄ +

Baffour Ba Series, Further Mathematics for Schools Page 585


2. Let y = x2 + 3. Find B. For each of the following, find
1. y = x2 – 5. y = – + ⁄
Solution √

y = x2 + 3 2. y = + + ⁄ 6. y = x4 –
= (x2 + 3) 3. y = 2x2 – 7. y = 6 √ –

2
= x + 3 4. y = + 8. y = 2√ –
= 2x + 0
Evaluating Derivatives
= 2x
1. Differentiate the equation.
2. Substitute the value of the given variable in
3. Let f(x) = x5 – 6x7. Find (x) the differential equation.

Solution Worked Examples


f(x) = x5 – 6x7 1. If s = 3t2 + 5t – 7, find the value of when
(x) = (x5 – 6x7)
t = 2.
5 7
= x – 6x
= 5x – 6 (7x6)
4 Solution
= 5x4 – 42x6 s = 3t2 + 5t – 7,
= 6t + 5
3 2
4. Differnetiate y = 5x – 4x + 12x – 8
At t = 2,
Solution = 6(2) + 5
y = 5x3 – 4x2 + 12x - 8
( ) = 12 + 5 = 17
= (5x3) – (4x2) + (12x) -
( )
=5 (x3) – 4 (x2) + 12 (x) -
2. If f(x) = √ + 3x, evaluate (4)
2
= (3) (5) x – (2) (4) x + 12 - 0
Solution
= 15x2 – 8x + 12
f(x) = √ + 3x
f(x) = ⁄ + 3x
Exercises 20.3

A. Differentiate each of the following with = +3
respect to x. = +3= +3=3

1. y = x3 – 5x 4. y = x2 + 5x – 7
2. y = 1 – x2 5. y = x (x – 1)
5 4 3 3. Let f(x) = 2x(x2 – 5x + 7). Find the derivative
3. y = x + x – 5x 6. y = 2x9 + 3x
of f at 2.

Baffour Ba Series, Further Mathematics for Schools Page 586


Solution by and is called the „second derivative of y
f(x) = 2x(x2 – 5x + 7).
with respect to x‟.
f(x) = 2x3 – 10x2 + 14x
(x) = (2x3 – 10x2 + 14x)
is pronounced „dee two y, dee x two‟.
= 2x3 – 10x2 + 14x
= 2(3x2) –10 (2x) + 14(1) The derivative of ( ) is denoted by (x)
= 6x2 – 20x + 14 and is called the „second derivative of x‟.

The derivative of f at 2;
Worked Examples
(2) = 6(2)2 – 20(2) + 14
1. If f(x) = x + , find (x) and (2).
(2) = 24 – 40 + 14
(2) = - 2
Solution
Exercises 20.4 f(x) = x + ,
A. 1. If A = 3r2 – 5r, find the value of , when f(x) = x +
r=3 (x) = 1 –
(x) = 2
2. If V = 3h – h2 – 3h3 , find , when h = 1.
=
3. If A = r2, find , when = 1
4. If V = r3, find , when 2r – 5 = 0 (2) = =
5. f(x) = 3x – 4x . If
2
(k) = 8, find the value of k.
3 2
6. If s = t – 2t , evaluate at t = 3.
2. If h = 10 + 30t 2 – 4t3, evaluate when
2 3
7. If θ = 3t – t , evaluate at t = 2. t = 3.
8. If V = r3 , evaluate at r = 5.
Solution
h = 10 + 30t 2 – 4t3,
B. For each of the following f, find (a) for
= 60t – 12t2
the given a.
1. f(x) = x3 – 4x . a=1 = 60 – 24t
⁄ ⁄
2. f(x) = 3 - a = 27
3. f(x) = + a=2 At t = 3
4. f(x) = r2 – 2√ a=4 = 60 – 24(3) = - 12

Second Derivatives 3. If y = ( , find


)
The derivative of , that is . / is denoted

Baffour Ba Series, Further Mathematics for Schools Page 587


Solution 2. If y = 4x3 – 6x2 , show that x2 – 2x –
y=( , 12x = 0. Find the values of x for which :
)
y = 10( ) i. =0 ii. =0
= - 50 (4x) ( )
3. If y = , show that y + . / – 10y3
= - 200x ( )
= ( )
Product Rule
Suppose u and v are functions of x, if y = uv;
then;
4. If y = x4, show that . /–. / =0
=u + v

Solution
y = x4 In words,
First by the derivative of the second + second
= 4x3
by the derivative of the first.
= 12x2
Worked Examples
1. If y = (x2 – 3x + 2) (x2 – 2), find
Substitute in . /–. /

= ( )-( ) Solution
= 16 x6 – 16x6 y = (x2 – 3x + 2) (x2 – 2),
=0 Let u = (x2 – 3x + 2) and v = (x2 – 2),
= 2x – 3 and = 2x
Exercises 20.5
1. If f(x) = 3x2 – 4x – 7 , evaluate ; =u +v
i. (2) ii. (-1)
= (x2 – 3x + 2) (2x) + (x2 – 2) (2x – 3)
2. If f(x) = - 4√ , find . (9) = 2x3 – 6x2 + 4x + 2x3 – 3x 2 – 4x + 6
= 4x3 – 9x2 + 6
3. If s = 3t – 2t2, find the value of:
i. ii. when t = 2
3. Let y = √ (x + 1). Find

4. f(x) = x3 + 1. If (x) = 18, find the value of Solution


x, x R. y = √ (x + 1).
= (x + 1) √ +√ (x + 1).
Challenge Problems
1. If y = 3x2 + 2x , show that y – 3x – 6x = 0 = (x + 1). + √ . (1)

Baffour Ba Series, Further Mathematics for Schools Page 588


= (x + 1).

+√ (Rationalize denorminator)
In applying the quotient rule, it is usual to
simplify the top but not the bottom.
√ √
= . +√
√ √ Worked Examples
= + +√
1. If y = , find
Exercises 20.6
A. Find in each of the following Solution

1. y = (2x + 3) (x – 4) y=
2. y = (x + 5) (x2 – 3x + 2) Let u = x2 and let v = x – 2
3. y = (3x – 4) (x 2 – 2x + 3) = 2x and =x
4. y = (x + 3) (x 2 – 6x + 8)
5. y = (5x 2 – 3x) (x2 – 5x) =
6. y = (3x 3 – 2x2 + 4) ( 2x – 1) ( )( ) ( )( )
= ( )

B. Use product rule to find = ( )


1. y = (2x – 3) (5 – 6x )
=(
2. y = (5x – 2) (2x + 3) )

3. y = (x – 2) (x + 1)
( )
4. y = (x3 – √ ) (x3 – √ ) 2. Differentiate and simplify y = ( )
5. y = √ (√ + 1)
6. y = . / . / + 3p2, where p is a Solution
( )
constant y= ( )

C. 1. If f(x) = x3 – 5 and g(x) = x2 + 1, find Let u = (x – 3)2 and v = (x + 2)2


the following:
y=
1. f(g(x)) 2. D, ( ( ))- 3. D, ( ) ( )-

Quotient Rule = 2(x – 3) (1) and = 2(x + 2) (1)


Suppose u and v are functions of x.
=
If y = ; then;
( ) ( ) ( ) ( )
= ( )
( )( ) ( )( )
= ( )
In words; ( )( ) ( )( )
= –
( ) ( )
(Bottom by the derivative of the top – Top by
( ) ( )
the derivative of the bottom) ÷ (Bottom)2 = –

Baffour Ba Series, Further Mathematics for Schools Page 589


( ) Worked Examples
=
( ) A. For each of the following, find
=
1. y = (x2 – 3x )4
= 2. y = (3x + 2)4
3. y = (x2 + 3x)7
=

Solutions
Exercises 20.7 1. y = (x2 – 3x) 4
A. Find in each of the following. Let u = (x2 – 3x), n = 4, n – 1 = 3 , = 2x – 3
1. y = 4. y = Substitute in ;
2. y = 5. y = =

3. y = 6. y = = 4( – ) (2x – 3)
= 4(2x – 3) ( – )
B. Use quotient rule to find . = ( 8x – 12) ( – )

1. y = 5. y =
2. y = (3x + 2)4
2. y = 6. y = Let u = (3x + 2), n = 4, n – 1 = 3 , =3

3. y = 7. y = Substitute in ;
√ =
4. y = 8. y =
= 4( ) (3)
Function of a Function = 12( )
Consider the example, y = (x + 5)3, here we say
y is a function of x. 3. y = (x2 + 3x)7
Let u = (x + 5)…………(1)
Let u = (x2 + 3x), n = 7, n – 1 = 6, = 2x + 3
⇒ y = u3………………(2)
Substitute in ;
From eqn (1) and eqn (2), we say y is a function =
of u, and u is a function of x. The variable u, is = 7( ) (2x + 3)
a link between the two expressions.
= 7(2x + 3)( )
Chain Rule = (14x + 21)( )
Suppose u is a function of x.
If y = , then = B. Differentiate the following;
The chain rule should be done in one step. 1. y = √

Baffour Ba Series, Further Mathematics for Schools Page 590


2. y = Substitute in ;
=
3. y = . /
= 3( ) (2x - )
Solution =3. / . /
1. y = √

y=( )
4. If y = (x2 + 3x)2 and = 2, find when
Let u = (4x – 3), n = , n – 1 = - , =4
x=2
Substitute in ;
= Solution

= ( – ) (4) y = (x2 + 3x )2 , =2⇒ =


= 2(2x + 3)(x2 + 3x)
= (4) ( – )
= (4x + 6 ) (x2 + 3x)
= 2( – )
= Using chain rule;
( )
= ×
=

By substitution;
2. y = (4x + 6 ) (x2 + 3x) = ×
y = 3( ) 2(4x + 6 ) (x2 + 3x) =
Let u = (2x + 5), n = - 1 , n – 1 = - 2 , =2
= 2(4x + 6 ) (x2 + 3x)
Substitute in ;
= At x = 2;
= 3 (-1) ( ) (2) = 2[(4(2) + 6) (22 + 3(2)]
= - 3(2) ( ) = 2 [(8 + 6) (4 + 6)]
= -6( ) = 2 [(14) (10)] = 280
=( )
Exercises 20.8
A. Find
3. y = . /
1. y = (3x + 2)4 5. y = (x2 + 5)3
y=( )
2. y = (x2 + 2x)3 6. y = (5 – 2x)2
Let u = ( ), n = 3, n – 1 = 2 ,
3. y = (2x2 + 1)5 7. y = 2(3x + 4)4
= 2x - 4. y = (1 – x3)2 8. y = ( )

Baffour Ba Series, Further Mathematics for Schools Page 591


B. Find y = uv

1. y = (2x + 5) – 1 6. y = ( ) ⁄ =u +v (product rule)

2. y = (3 – 4x) – 3 7. y = ( ) ⁄
= 2x and = 3(x + 1)2 (1)
3. y = (3x – 4) – 2 8. y = ( ) ⁄

4. y = (6x3 – 4x) – 2 9. y = ( ) ⁄ By substitution;


5. y = (x2 + 5) 3 10. y = ( √ ) = (x2 )3(x + 1)2 + (x + 1)3 (2x)
= 3x2 (x + 1)2 + (2x) (x + 1)3
C. Use the chain rule to find = x(x + 1)2 {3x + 2(x + 1)}
1. y = 2. y = = x(x + 1)2 {3x + 2x + 2)}

= x(x + 1)2 {(5x + 2)}
2. y = 6. y =

2. Find if y = x(x2 + 1)4.
3. y = ( 7. y = (
) ) ⁄

4. y = ( 8. y = √ Solution
) ⁄
y = x(x2 + 1)4.
D. Differentiate the following; Let u = x and v = (x2 + 4)4
y = uv
1. y = √ 4. y = x2 √
=u +v (product rule)
2. y = (3√ )3 5. y = x √
= 1 and = 4(x2 + 1)3 (2x) = 8x (x2 + 1)3
3. y = √ 6. y = . /

By substituition;
Challenge Problems
= (x)8x(x2 + 1)3 + (x2 + 4)4 (1)
1. If f(x) = √ , find the value of (1)
= 8x2 (x2 + 1)3 + (x2 + 4)4

2. If f(x) = √ , find the value of . /


3. Differentiate the expression:
y = (x2 – 3) (x + 1)2 and simplify the results
Combination of Product, Quotient and Chain
Rules Solution
y = (x2 – 3) (x + 1)2
Worked Examples Let u = (x2 – 3) and v = (x + 1)2
1. Differentiate f(x) = x2(x + 1)3 with respect to x. y = uv

Solution = 2x and = 2(x + 1) (1)


f(x) = x2(x + 1)3 =u +v
Let f(x) = y, u = x2 and v = (x + 1)3

Baffour Ba Series, Further Mathematics for Schools Page 592


= (x2 – 3) 2(x + 1) + (x + 1)2 (2x) 6. Differentite (x2 + 1)3 (x3 + 1)2

= 2(x2 – 3) (x + 1) + (2x) (x + 1)2 Solution


2
= 2 (x + 1) {(x – 3) + x (x + 1)} y = (x2 + 1)3 (x3 + 1)2
u = (x2 + 1)3 and v = (x3 + 1)2
= 2 (x + 1) {(x2 – 3 + x2 + x)} Then y = uv
= 2 (x + 1) {(2x2 + x – 3)} = 3(x2 + 1)2 2x and = 2(x3 + 1) 3x2
= 2 (x + 1) {(2x + 3) (x – 1)} = 6x(x2 + 1)2 and = 6x2(x3 + 1)

4. Find for y = x√ =v +u
= (x3 + 1)2 6x(x2 + 1)2 + (x2 + 1)3 6x2(x3 + 1)
Solution
= 6x(x3 + 1)2 (x2 + 1)2 + 6x2(x2 + 1)3 (x3 + 1)
y = x√
y=x( ) ⁄ = 6x(x3 + 1) (x2 + 1)2 {(x3 + 1) + x (x2 + 1)}
= (x) . ( ) ⁄
(-2x) + ( ) ⁄
(1) = 6x(x3 + 1)(x2 + 1)2{(x3 + 1) + (2x3 + x + 1)}
(chain rule here)
⁄ ⁄
= -x ( ) + ( ) 7. Differentiate y =
√( )
=√ +√
Solution
y=
5. Given y = √ , find thedrivative of y √( )

y=
( ) ⁄
Solution y=x( )
y=√ Let u = x and v = ( )
⁄ = 1 and = (2x) ( )
y =. /
⁄ ( )( ) ( )( ) = 2x( )
= . / 0 1
( )

=u +v (Product rule)
= . / 0 1
( )
↑ By substitution;
(Chain rule followed by quotient rule) = x (-2x)( ) +( )

( ) ⁄ = -2x2( ) +( )
= ( ) ⁄ – ( )
= +
=( ) ⁄ ⁄ ( ) ( )
( )

Baffour Ba Series, Further Mathematics for Schools Page 593


Exercises 20.9 2. Bring all terms with to the left and bring
A. Find if ; all other terms to the right.
2 4 3
1. y = x (x + 3) 2. y = 3x (x + 2) 3. Make the subject of the equation.
3. y = 3 x2 (2x + 3)2 4. y = x2 √

5. y = x √ 6.y=√ Note:
Remember that, by the chain rule;
( ) = 2y and ( ) = 3y2 as y is
B. Differentiate the following functions with
respect to x considered as a function of x.
1. 2. ( 3. √
) In general,
4. (1 + x2)2 (1 – x2) 5. (1 – x2)2 (1 – x3)
( )=n . /
2 2 √
6. y = -3x (2x – 3) 7. y =

Worked Examples
C. 1. If f(x) = √ , find the value of (1) For each of the following curves, express in
terms of x and y.
2. If f(x) = √ , find the value of . / 1. x2 + y2 = 4 2. x2 + 2y – y2 = 5

3. Given f(x) = √ (x + 2), calculate . / Solution


1. x2 + y2 = 4
Implicit Differentiation ( ) ( ) ( )
+ =
If y = f(x), then the variable y is given explicitly
in terms of x. For example, y = x3 – 2x2 - 5x – 4 2x + 2y =0
is an explicit function.
2y = - 2x

Some curves are defined by implicit functions, =


that is functions which cannot be expressed in =
the form y = f(x). For example, x2 + xy + y3 = 7
is an implicit function because y = f(x, y), hence 2. x2 + 2y – y2 = 5
( ) ( ) ( ) ( )
it cannot be expressed in the form y = f(x) + – =
2x + 2 – 2y =0
An implicit function involving x and y can be
differentiated with respect to x as it, stands 2 – 2y = - 2x
using the chain rule.
(2 – 2y) = - 2x
How to Differentiate Implicit Functions =
1. Differentiate term by term, on both sides
=
with respect to x. ( )

Baffour Ba Series, Further Mathematics for Schools Page 594


= Solution
y2 + x3 – y3 + 6 = 3y
= ( ) ( ) ( ) ( ) ( )
+ – + =

3. Find the derivative of x3 + y3 = xy 2y + 3x2 – 3y2 =3


3x2 = 3 – 2y + 3y2
Solution
3x2 = (3 – 2y + 3y2 )
x3 + y3 = xy
(x3 + y3 ) = (xy) =

x3 + (y3) = x +y
Solved Past Questions
3x2 + 3y2 = x . (1) + y (1) 1. If x2 + y2 = 2pxy, where p is a constant, find
3x2 – y = x – 3y2 .
3x2 – y = (x – 3y2)
Solution
= x2 + y2 = 2pxy
2x + 2y = 2p . /
4. Find the derivative of x3 + 4xy2 – 7 = y3
2x + 2y = 2px + 2py

Solution 2x + 2y = 2px + 2py


x3 + 4xy2 – 7 = y3 2x – 2py = 2px – 2y
(x3 + 4xy2 – 7) = (y3)
2x – 2py = (2px – 2y)
3 2 2
x + (4xy ) – 7 = 3y –
=
(apply product rule)
( – )
= ( )
3x2 + 4x (y2) + y2 (4x) – 0 = 3y2

=
3x2 + 4x . 2y + y2. 4 – 0 = 3y2
3x2 + 8xy + 4y2 = 3y2 2. If (1 + x2) (1 + y2) – px2 = 0, where p is a
3x2 + 4y2 = 3y2 – 8xy constant, show that xy (1 + x2) = (1 + y2)
3x2 + 4y2 = (3y2 – 8xy)
Solution
= Let (1 + x2) (1 + y2) – px2 = 0…………..(1)
1 + y2 + x2 + x2y2 – px2 = 0
2y + 2x2y = 2px – 2x – 2xy2
5. Find the derivative of the function:
y2 + x3 – y3 + 6 = 3y y + x2y = px – x – xy2

Baffour Ba Series, Further Mathematics for Schools Page 595


y + x2y = px – x – xy2 C. Determine in each of the following:
2 4 2
y(1 + x2) = px – x – xy2 1. x y = 3x – 3x + 12
2. √ y = x – 3
3. x2y = x2 – 2
xy(1 + x2) = px2 – x2 – x2 y2 Multiply through by x
4. xy = √ y + x2 – 5
xy(1 + x2) = px2 – x2 (1 + y2) …..(2) 5. xy2 + x2 = 2

From eqn (1); Evaluating Implicit Functions at a Point


px2 = (1 + x2) (1 + y2) Steps:
Put px2 = (1 + x2) (1 + y2) in eqn (2); 1. Find by implicit differentiation.

xy(1 + x2) = (1 + x2) (1 + y2) – x2 (1 + y2) 2. Evaluate at the given point (x, y) by
substituting the values of (x, y) in .
xy(1 + x2) = (1 + y2) [ (1 + x2) – x2 ]

xy(1 + x2) = (1 + y2) (1) Worked Examples


1. Find the slope of the tangent to the circle
xy(1 + x2) = (1 + y2) as required. x2 + y2 = 4 at the point (√ , √ )

Exercises 20.10 Solution


A. For the following equations; find x2 + y2 = 4
1. xy2 – x2 + y = 0 (x2 + y2) = (4)
2. x3 + y3 – 6xy = 0
3. x5 + 4xy3 – 3y3 = 1 x2 + y2 = (4)
4. x2 + 2xy + y2 = 3 2x + 2y =0
5. x2 – 3xy + y2 – 2y + 4x = 0
6. 3x2 – 4xy = 7 2y = - 2x
=
B. For each of the following curves, express
in terms of x and y =
1. x2 + y2 – 6xy + 3x – 2y + 5 = 0
2. x2+ y2 – 4x – 6y + 9 = 0 At (√ , √ );
3. x2 + 3xy + 2y2 = 6 √
=
4. x2 + xy + y2 = 13 √

5. x2y + xy2 = 2 =-1


6. x2 – 6y3 + y = 0 The slope of the tangent at (√ , √ ) is – 1.

Baffour Ba Series, Further Mathematics for Schools Page 596


2. Given that 2x3 + 3xy2 – y3 + 6 = 0, evaluate Finding a Point on Curve given the Slope
at a the point (-1, 1). 1. Identify the given function y = f(x)
2. Find .
Solution 3. Set = m ( the given gradient), and solve
2x3 + 3xy2 – y3 + 6 = 0,
for x.
(2x3 + 3xy2 – y3 + 6) = (0) 4. Substitute the value of x in y = f(x) to obtain
(2x3) + (3xy2 ) – (y3) + (6) = (0) the ordered pair (x, y).

(2)3x2 + 3x. 2y + y2. 3 – 3y2 =0 Worked Examples


2 2 2
6x + 6xy + 3y – 3y =0 1. Find the coordinates of the points on the
curve y = 5 + 9x – 7x2 where the gradient is
6x2 + 3y2 = 3y2 – 6xy
zero.
6x2 + 3y2 = (3y2 – 6xy)
Divide through by 3 Solution
2
2x + y = 2 2
(y – 3xy) y = 5 + 9x – 7x2
= 9 – 14x
=
=0
At the point (-1, 1) 0 = 9 – 14x
( ) ( ) 14x = 9
=( = = =1
) ( )( )
x=

Exercise 20.11
A. For each of the following curves, find the Put x = in y = 5 + 9x – 7x2
slope at the indicated point.
y=5+9. /–7. /
1. 2y3 + y2 – x = 0 (3, 1)
2 3
2. xy – 3y + 8 = 0 (4, 2) y=5+ –7. /
2 2
3. x – 3xy + 2y – 2x = 4 (1, -1)
2 2
4. 2x + 3y = 14 (1, -2) y=5+ –
2 2
5. y + 3xy +2x = 6 (2, 1)
y=

B. Find the value of at the specified point. At the point . , /, the gradient is zero.
2 2
1. x + y = 25 (3, - 4)
2. x2 + 4xy – 2y2 – 8 = 0 (0, 2)
2. Find the coordinates of the points on the
3. x2 + xy + 2y2 = 28 (2, - 4)
curve y = x2 where the gradient is 8.
4. x3 + y2 + 3x2y = 21 (2, 1)

Baffour Ba Series, Further Mathematics for Schools Page 597


Solution Worked Examples
y = x2 1. Find the gradient of the tangent to the curve
= 2x y = x3 + 2x + 4 at x = 2

=8 Solution
8 = 2x y = x3 + 2x + 4
x=4 = 3x2 + 2

Put x = 4 in y = x2 At x = 2
y = (4)2
= 3(2)2 + 2
y = 16
At the point (4, 16), the gradient of the curve is 8. = 14
The gradient of the tangent at x = 2 is m = 14
Exercises 20.12
1. Find the coordinates of the points on the 2. Determine the gradient of the tangent of the
following curves at which the gradient has graph of f(x) at x = -2, if f(x) = x(3 – 2x)2
the given values;
1. y = x3, m = 12 Solution
2
2. y = x – 3x + 1 , m=0 f(x) = x(3 – 2x)2
3 2
3. y = x – 6x + 4, m = -12 f(x) = x(9 – 12x + 4x2)
4 3
4. y = x – 2x + 1, m=0 f(x) = 9x – 12x2 + 4x3
2
5. y = x(x – 3) m=0
6. y = x(2 – x) m=2 = 9 – 24 + 12x2
(-2) = 9 – 24 + 12(-2)2
Challenge Problems (-2) = 105
A curve is given by the equation x2 + 4xy = 2y2
– 8. Find the coordinates of the points on the Equation of a Tangent to a Curve at a Point
on the Curve.
curve at which =1
= the slope of a tangent to a curve at any
Application of Differentiation point on the curve.
Evaluating Gradient at a Point To find the equation of the tangent to a curve at
To find the gradient of a curve at a point (x1, y1) a point (x1, y1) on the curve;
on the curve; 1. Find (using the appropriate rule)
1. Find (using the appropriate rule) 2. Evaluate at x = . This gives m, the slope
2. Evaluate at x = . This gives m, the slope of the tangent.
of the tangent. 3. If the equation of the curve is given
implicitly, use at x = and y = .

Baffour Ba Series, Further Mathematics for Schools Page 598


4. Use m obtained in step (2) and the given y + 1 = 4x – 8
point ( , ) in the equation: (y – y1) = m (x – x1) 4x – y – 8 – 1 = 0
4x – y – 9 = 0
Worked Examples
1. Find the equation of the tangent to the curve 3. Find the equation of the tangent to the curve
y = 3 + 2x – x2 at (2, 3). y = (2x + 3)3 at (-1, 1).

Solution Solution
y = 3 + 2x – x2 y = (2x + 3)3 at (-1, 1)
= x – 2x Let u = 2x + 3, n = 3, n – 1 = 2 =2
Substitute in ;
At (2, 3) =
= 2 – 2 (2)
= 3( ) 2
=-2
= 6( )
m = -2
At (-1, 1)
Equation of tangent;
= 6( ( ) )
y – 3 = -2(x – 2)
y – 3 = -2x + 4 = 6( )
2x + y – 3 – 4 = 0
=6
2x + y – 7 = 0
m=6
2. Find the equation of the tangent to the curve
y = 4x2 – 12x + 3 at the point (2 , -1) Equation of tangent;
y – 1 = 6 (x + 1)
Solution y – 1 = 6x + 6
y = 4x2 – 12x + 3 6x – y + 6 + 1 = 0
6x – y + 7 = 0
= 8x – 12
At x = 2, 4. Find the equation of the tangent to the curve
= 8(2) – 12 y= at . , /
= 16 – 12
Solution
=4
y= at . , /
m = 4, (2 , -1)

Equation of tangent; Let u = 6x – 3 , v = 4x + 2


y + 1 = 4(x – 2) = 6 and =4

Baffour Ba Series, Further Mathematics for Schools Page 599


= 27 – 12 = 15
=

By substitution; When x = 3, y = 33 – 12(3)


( )( ) ( )( ) y = 27 – 36
= ( ) y = -9
= The gradient occurs at (3, - 9)
( )

=( m = 15, (3, - 9)
)
Equation of tangent;
At . , /; y – (-9) = 15 (x – 3)
y + 9 = 15 (x – 3)
=, y + 9 = 15x – 45
( ) -

= 15x – y – 45 – 9 = 0
15x – y – 54 = 0
=
m= Equation of a Tangent to a Curve at the point
where the Curve meets the line y = mx + c
i. Solve the two equations simulatanously to
Equation of tangent;
obtain the values of x and y.
y – = (x – 1)
ii. Find to obtain the gradient m, of the curve
3y – = 2 (x – 1) at x.
6y – 3 = - 4 (x – 1) iii. Put the values of m and (x, y), in the
6y – 3 = - 4x + 4 equation: (y – y1) = m (x – x1), where (x, y) =
4x + 6y – 3 – 4 = 0 ( , )
4x + 6y – 7 = 0
The equation of the tangent is 4x + 6y – 7 = 0 Worked Examples
1. Find the equations of the tangents to the
5. Find the equation of the tangent to the curve curve y = x2 – x – 2 at the point where the curve
y = x (x2 – 12) at the point of where x = 3 . meets the line y = x + 1

Solution Solution
y = x (x2 – 12) y = x2 – x – 2 ………………..(1)
y = x3 – 12x y = x – 1…………………….(2)
= 3x2 – 12
eqn (1) = eqn (2);
x2 – x – 2 = x + 1
At x = 3
x2 – x – 2 – x – 1 = 0
= 3(3)2 – 12 x2 – 2x + 3 = 0

Baffour Ba Series, Further Mathematics for Schools Page 600


(x + 1) ( x – 3) = 0 intersection of the curve and the line y = x – 1
x = -1 or x = 3
Solution
From eqn (2); y = 3x2 + 5x – 1…………….(1)
When x = -1, y = -1 + 1 = 0 y = x – 1…………………….(2)
When x = 3, y = 3 + 1 = 4
The points of intersection of the curve and the eqn (1) = eqn (2);
line are ( , ) and ( , ) 3x2 + 5x – 1 = x – 1
3x2 + 5x – x – 1 + 1 = 0
Now, gradient of the curve; 3x2 + 4x = 0
= (x2 – x – 2) x(3x + 4) = 0
x = 0 or 3x + 4 = 0
= 2x – 1 x = 0 or 3x = - 4
At x = -1, x = 0 or x =
= 2(-1) – 1
= -3 ⇒m = -3 Put x = 0 in eqn (2);
y = -1
(0, -1)
Equation of the tangent with m = -3 passing
through (-1, 0)
Put x = in eqn (2);
y - 0 = -3 (x – (-1))
y = -3 (x + 1) y= –1
y = -3x – 3
y=
3x + y + 3 = 0
. , /
At x = 3,
= 2(3) – 1 Now, gradient of the curve;

= 5 ⇒m = 5 = (3x2 + 5x – 1)
= 6x + 5
Equation of the tangent with m = 5 passing
through (3, 4) At x = 0,
y – 4 = 5 (x – 3) = 6(0) + 5
y – 4 = 5x – 15
5x – y + 4 – 15 = 0 = 5 ⇒m = 5
5x – y – 11 = 0
Equation of the tangent with m = 5 passing
2. Find the equations of the tangents to the through (0, - 1)
curve y = 3x2 + 5x – 1 at the point of y + 1 = 5(x – 0)

Baffour Ba Series, Further Mathematics for Schools Page 601


y + 1 = 5x 2. Set the derivative to zero and solve for the
5x – y – 1 = 0 value of the involving variable.
3. Use the solution(s) of the variable, find its
At x = , corresponding y – coordinate using the original
equation to obtain the point (x, y).
= 6( ) + 5
4. The corresponding equation of the tangent
= -3 line is given by; y – y1 = m(x – x1)
m = -3 For a tangent line parallel to the x – axis, its
equation is y – y1 = 0
Equation of the tangent with m = -3 passing
Note;
through . , /
If the tangent to the curve represented by y =
y + = -3 (x + )
f(x) at P(x, y) is parallel to y – axis, then
y + = -3x – which represents the slope =
3y + 7 = -9x – 12
9x + 3y + 7 + 12 = 0 Worked Examples
9x + 3y + 19 = 0 1. Find the values of x for which the gradient
The equations are y = 5x – 1 and 9x + 3y + 19 = 0 function of the curve y = 2x3 + 3x2 – 12x + 3 is
zero. Hence find the equation of the tangent to
Exercises 20.13 the curve which are parallel to x – axis.
A. Find the equation of the tangent to the
curve at the indicated points; Solution
1. x3 – 2x2 – 4x + 1 (-1, 2) y = 2x3 + 3x2 – 12x + 3
2 2
2. x + y – 10y = 0 (4, 2) = 6x2 + 6x – 12
3 3
3. y – xy – 6x (1, 2)

Equation of the Tangent Parallel to the x or At = 0,


y - axis ⇒ 6x2 + 6x – 12 = 0
Note that the x – axis is horizontal and the y – x2 + x – 2 = 0
axis is vertical. (x + 2) ( x – 1) = 0 (By factorization)
A line parallel to the x – axis will have a x + 2 = 0 or x – 1 = 0
gradient m = 0 and a line parallel to the y – axis x = - 2 or x = 1
will have the gradient m = 1.
When x = -2,
To find the equation of the tangent parallel to y = 2(-2)3 + 3(-2)2 – 12(-2) + 3
the x – axis; y = - 16 + 12 + 24 + 3
1. Take the derivative of the equation of the y = 23
given curve. (-2, 23) is a point of tangency

Baffour Ba Series, Further Mathematics for Schools Page 602


Equation of the tangent parallel to the x – axis Equation of the tangent at the point (0, 6) which
implies that gradient m = 0; parallel to the x- axis has slope m = 0
y – 23 = 0 (x + 2) y – 6 = 0 (x - 0)
y – 23 = 0 y–6=0

When x = 1; When x = √ ,
y = 2(1)3 + 3(1)2 – 12(1) + 3 y = (√ )4 – 4(√ )2 + 6
y = 2 + 3 – 12 + 3
y=4–8+6
y=-4
y=2
(1, - 4) is a point of tangency.
(√ , 2) is a point of tangency.
Equation of the tangent parallel to the x – axis
at (1, - 4); Equation of the tangent at the point (√ , 2)
y + 4 = 0 (x + 2) (at the x- axis m = 0) which is parallel to the x- axis;
y+4=0 y – 2 = 0 (x – √ )
y–2=0
2. Find the equation of the tangent to the curve
y = x4 – 4x2 + 6 parallel to x – axis. When x = - √ ,
y = ( √ )4 – 4( √ )2 + 6
Solution y=4–8+6
y = x4 – 4x2 + 6 y=2
= 4x3 – 8x ( √ , 2) is a point of tangency.

Equation of the tangent at the point ( √ , 2)


=0
which is parallel to the x- axis;
4x3 – 8x = 0
y – 2 = 0 (x + √ )
4x(x2 – 2) = 0
y–2=0
4x = 0 or (x2 – 2) = 0
3. Find the equation of the tangent line to the
Solving (x2 – 2) = 0
graph of f(x) = √ at the point (-1. 2)
x = √ or x = √
⇒4x(x2 – 2) = 0
Solution
x = 0 or x = √ or x = √
f(x) = √
When x = 0, f(x) = ( )
y = (0)4 – 4(0)2 + 6 (x) = ( ) . 2x
y=6
(x) = √
(0, 6) is a point of tangency.

Baffour Ba Series, Further Mathematics for Schools Page 603


At the point (-1, 2) ( ) ( )
= ( )
= =-1
( )
(-1) =
√( ) At the point (1, 1) , m = -1
(x) =

Equation of the tangent at the point (1, 1);
(x) = y – 1 = - 1 (x – 1)
y–1=-x+1
The equation of the tangent; x+y–1–1=0
y – y1 = m(x – x1) x+y–2=0
y–2= (x + 1)
2(y – 2 ) = - (x + 1) Exercises 20.15
2y – 4 = - x – 1 A. 1. Find the equation of the tangent to the
x + 2y – 4 + 1 = 0 curve y = x2 + 5x – 2 at the point where this
x + 2y – 3 = 0 curve cuts the line x = 4.
2. Find the equation of the tangent to the curve
Exercises 20.14 y = (2x – 1) (x + 1) at the point where the curve
A. Find the equation of the tangent to the cuts the x - axis. Find the point of intersection
following curves at the indicated points; of these tangents.
1. y = x2 at x=2
2 3. Find the equation of the two tangents to the
2. y = 3x + 2x at x=4
2 curve y2 + 3xy + 4x2 = 14 at the point where x = 1.
3. y = 3x – x + 1 at x=0
2
4. y = 3 – 4x – 2x at x=1
2 4. Find the coordinates of the point on y = x2 at
5. y = 9x – x at x = -3
which the gradient is 2. Hence find the equation
of the tangent to y = x2 whose gradient is 2.
Type 2: Involving Implicit Functions
Worked Examples
5. Find the coordinates of the point on y = x2 –
1. Find the equation of the tangent to the curve
5 at which the gradient is 3. Hence, find the
x2 + xy + y2 = 3.
value of c for which the line y = 3x + c is a
tangent to y = x2 – 5
Solution
x2 + xy + y2 = 3 (Apply implicit differentiation)
6. Find the equation of the tangent to y = 2x2 –
2x + x + y(1) + 2y =0 (use product rule on xy)
3x which has gradient of 1 Ans y = x – 2
x + 2y = - 2x – y
Challenge Problems
(x + 2y) = -2x – y
1. Find the coordinates of the point on the curve
= y= at which the tangents to the curve are
parallel to the line x – y + 8 = 0 . Find the
At (x, y) = (1, 1); equations of the tangents at these points.

Baffour Ba Series, Further Mathematics for Schools Page 604


2. Find the slope of the tangent to the curve Solution
y2 + 3xy + 2x2 = 6 at the point (1, 1). Let y = px2 + 1
= 2px
Finding Unknown Coefficients given the
value of At (1, q);
Given the y function; =6
1. find in terms of the involving variables ⇒ = 2p(1)
2. equate in (1) to the given value of p= =3
3. solve the equation to obtain the value (s) of
the unknown variables. Put p = 3 in y = px2 + 1
y = 3x2 + 1
Worked Examples
1. The slope of the tangent to the curve y = ax3 At(1, q)
+ bx + 4 is 21 at the point (2, 14) on the curve. ⇒ when x = 1, y = q
Find the value of a and the value of b. q = 3(1)2 + 1
q=4
Solution Therefore, p = 3 and q = 4
y = ax3 + bx + 4
3. The equation of the tangent to the curve f(x)
= 3ax2 + b = 21
= ax3 + bx at x = -1 is y – x – 4 = 0. Calculate
the values of a and b.
At x = 2,
3a(2)2 + b = 21 Solution
12a + b = 21………….(1) f(x) = ax3 + bx
(x) = 3ax2 + b
Given (2, 14) on the curve; (-1) = 3a(-1)2 + b
14 = a(2)3 + b(2) + 4 (-1) = 3a + b
14 = 8a + 2b + 4
8a + 2b = 14 – 4 Equation of tangent;
8a + 2b = 10 …………….(2) y–x–4=0
y=x+4
Solving eqn (1) and (2) simultaneously;
a = 2 and b = -3 Now, at x = -1
y = -1 + 4
2. The slope of the tangent to the curve px2 + 1 y =3
at the point (1, q) is 6. Find the value of p and Point of tangency = (-1, 3)
the value of q.

Baffour Ba Series, Further Mathematics for Schools Page 605


Gradient of tangent, =1 5. Determine the value of p if the line y = 3x +
Gradient of tangent = gradient of curve p is a tangent to the curve f(x) = 2x2 - 3x – 1 .
⇒ 3a + b……………………(1)
6. Given f(x) = ax3 + bx2 + cx – 5. The gradient
Substitute (-1, 3) in f(x) = ax3 + bx at any point (x, f(x)) is given by 6x2 – 24. Find
f(-1) = a(-1) 3 + b(-1) the values of a, b and c.
f(-1) = - a – b
Equation of Normal to a Curve
But f(-1) = 3 A normal is a straight line that is perpendicular
3=-a–b to the tangent at the same point of contact with
-3 = a + b ………………………...(2) the curve. That is the tangent and normal will
have the same point of contact on the curve, as
eqn (1) – eqn( 2); shown in the diagram below.
4 = 2a
a=2
Tangent
Put a = 2 in eqn (1)
1 = 3(2) + b
1=6+b
1–6=b Normal
b = -5

Exercises 20.16 Worked Examples


1. The slope of the tangent to the curve y = ax3 1. Find the equation of the tangent and normal
+ bx + 6 at the point (2, 4) is 3. Find the value to the curve y = x4- 3x3 + 6x + 2 at the point (2,
of a and the value of b. 6)

2. The slope of the tangent to the curve y = x4 – 1


Solution
at the point p is 32. Find the coordinates of p.
y = x4- 3x3 + 6x + 2 at the point (2, 6)
= 4x3 – 9x2 + 6
3. The curve y = , p, q R, x ≠ 0, x ≠ - 2 ,
( )
has zero slope at the point (1, -2). Find the
At x = 2;
value of p and q.
= 4(2)3 – 9(2)2 + 6
4. Determine the value of k for which the = 32 – 36 + 6
equation of the tangent to f(x) = 3x2 + kx + 4 at
=2
the point where x = 2 is y = 8x – 8
The gradient of the tangent is =2

Baffour Ba Series, Further Mathematics for Schools Page 606


Equation of tangent ; 2x = -5
y – y1 = (x – x1) x=
y – 6 = 2(x – 2)
The point at which the tangent is parallel to the
y – 6 = 2x – 4
given line is x =
2x – y – 4 + 6 = 0
2x – y + 2 = 0
At x =
Now, Let be the gradient of the
normal.Then = -1 for gradient of y=. / + 3. /+2
tangent and gradient of normal which are y= – +2
perpendicular at (2, 6).
y=
2 = -1
=- The coordinates of Q is . , /

Equation of normal; 3. Find the equation of the normal to the curve


y – y1 = (x – x1) y = (x2 + x + 1)(x – 3) at the point where it cuts
y–6= (x – 2) the x – axis.

2(y – 6) = -1 (x – 2)
Solution
2y – 12 = - x + 2
y = (x2 + x + 1)(x – 3)
x + 2y – 12 – 2 = 0
when y = 0
x + 2y – 14 = 0
⇒ (x2 + x + 1)(x – 3) = 0
But (x2 + x + 1) has no real roots
2. Find the coordinates of the point Q where the
So (x – 3) = 0
tangent to the curve y = x2 + 3x + 2 is parallel to
x=3
the line 2x + y + 2 = 0.

Solution At the point where it cuts the x – axis, y = 0


The gradient of the tangent to the curve y = x2 + The point where it cuts the x – axis is (3, 0)
3x + 2 which is parallel to the line 2x + y + 2 = y = (x2 + x + 1)(x – 3)
0 is the same as the gradient of the 2x + y + 2 = 0. y = x3 –2x2 – 2x – 3
From 2x + y + 2 = 0 = 3x2 – 4x – 2
y = - 2x – 2
⇒ m = -2 At x = 3,
= 3(3)2 – 4(3) – 2
y = x2 + 3x + 2
= 2x + 3 = 27 – 12 – 2
2x + 3 = - 2 = 13
2x = -2 – 3

Baffour Ba Series, Further Mathematics for Schools Page 607


The gradient of the tangent is 13. Therefore, the 2. y = x2 + 3x – 1 at x = 0
gradient of the normal is 3. y = at x = -1

Equation of normal; 4. y = (x – 2) (x2 + 1) at x = -1


2
y–0= (x – 3) 5. y = x – 5x + 2 at x = 3
13(y – 0) = -1(x – 3) 6. y = x2 – 2 where x=0
13y = -x + 3
x + 13y – 3 = 0 B. 1. Find the equation of the normal to the
curve y = x2 + 3x – 2 at the point where the
4. Find the equation of the normal line to the curve cuts the y – axis .
graph of f(x) = √ at the point (-1, 2)
2. Find the equation of the normals to the curve
Solution y = x2 – 5x + 6 at the point where the curve cuts
f(x) = √ the x – axis .
3. Find the equation of the normal to y = x2 – 3x
f(x) = ( )
+ 2 which has a gradient of .
(x) = ( ) . 2x
(x) = √ 4. Find the value of k for which y = 2x + k is a
normal to y = 2x2 – 3
At the point (-1, 2)
5. Find the equation of the tangent to y = (x – 5)
(-1) =
√( ) (2x + 1) which is parallel to the x – axis.
(x) =

6. Find the equation of the tangent and normal
(x) =
to the curve y = x2(x – 3) at the point where it
cuts the x – axis.
The equation of the normal;
y – y1 = 2(x – x1) Rates of Change 1 (Linear Kinematics)
y – 2 = 2(x + 1) Displacement (Position), Velocity and
(y – 2) = 2(x + 1) Acceleration
y – 2 = 2x + 2
The derivative is called the rate of change of
2x – y + 2 + 2 = 0
2x – y + 4 = 0 y with respect to x. It shows how changes in y
are related to changes in x. For instance, if =
Exercises 20.17 3, then y is increasing three times as fast as x
A. Find the equation of the normal to the increases. Similarly, if = - 5, then y
given curve at the given point on the curve.
decreases five times as fast as x.
1. y = x2 at x = 1

Baffour Ba Series, Further Mathematics for Schools Page 608


In mechanics, letters other than x and y are a = 6. / – 18
used. If S denotes the displacement (position)
of a particle from a fixed point, at a time t, then; a= – 18

1. Velocity , v = , the rate of change of a = 21 – 18


a = 3m/s2
position with respect to time.
iv. a = 6m/s2
2. Acceleration , a = =
a= = = 6t – 18 = 6
Worked Examples
6t = 6 + 24
1. A particle moves along a straight line such
6t = 24
that, after t seconds, the distance moved s, s
t=4
meters, is given by s = t3 – 9t2 + 15t – 3. Find:
After 4second, the acceleration is 6m/s2
i. the velocity and acceleration of the particle ,
in terms of t,
Velocity after 4 seconds;
ii. the value of t when its velocity is zero.
v= = 3t2 – 18t + 15
iii. the acceleration after 3 seconds
iv. the time at which the acceleration is 6m/s2,
At t = 4,
and the velocity at this time.
v = 3(4)2 – 18(t) + 15
Solution v = 42 – 72 + 15
i. s = t3 – 9t2 + 15t – 3 v = -9m/s
This means that after 4 seconds, the particle
v= = 3t2 – 18t + 15
moves in the opposite direction to where it
a= = = 6t - 18 started.

ii. At v = 0, =0 2. A car starts from rest and moves a distance s

⇒ 3t 2 – 18t + 15 = 0 meters in t seconds, where s = t3 + t2.


t 2 – 6t + 5 = 0 i. What is the initial acceleration;
t = 1 or t = 5 Solving by factorization; ii. What is the acceleration at the end of 2
This means that the particle is stopped after 1 seconds?
second and again after 5 seconds.
Solution
iii. a = = = 6t - 18 i. s = t3 + t2

At t = 3 v= = (3) t2 + (2) t

a = 6. / – 18 v = t2 + t

Baffour Ba Series, Further Mathematics for Schools Page 609


a= = (2) t + 4. A stone is thrown vertically upwards, and
after t seconds its height is h meters, were
a=t+
h = 10.5t – 4.9t2. Determine with particular
attention to the signs, the height, velocity and
Initial acceleration means t = 0 acceleration of the stone;
At t = 0 i. when t = 1 and t = 3
a= 0+ ii. State whether the stone is going up or down
a= = 0.5 m/s2 at an increasing or decreasing speed.
ii. At t = 2; Solution
a = 2 + = 2.5m/s2 h = 10.5t – 4.9t2.
When t = 1,
3. The relation s = 12t2 – 5t3 connects the h = 10.5(1) – 4.9(1)2.
displacement s meters and the time t seconds of h = 10.5 – 4.9
a moving particle. Find; h = 6m
i. the velocity of the particle at time t seconds, The stone is going up at an increasing speed.
ii. the velocity of the particle at 5 seconds
iii. the acceleration of the particle at time 5 When t = 3,
seconds. h = 10.5(3) – 4.9(3)2.
h = 31.5 – 44.1
Solution h = -12.6m
i. s = 12t2 – 5t3 The stone is going down at an increasing speed.
v= = 24t – 15t2
v = 24t – 15t2 ii. v = = 10.5 – (2) (4.9)t
v = 10.5 – 9.8t
2
ii. v = 24t – 15t At t = 1, v = 10.5 – 9.8 (1) = 0.7m/s
At t = 5; At t = 3, v = 10.5 – 9.8 (3) = - 18.9m/s
v = 24(5) – 15(5)2 a= = -9.8
v = 120 – 375
The stone is going up at a decreasing speed.
v = - 255 m/s
5. A ball is thrown vertically upwards and its
iii. a = = 24 – 30t height after t seconds in s meters where
a = 24 – 30t s = 25.2t – 4.9t2. Find;
i. its height and velocity after 3 s,
At t = 5; ii when it is momentarily at rest,
a = 24 – 30(5) iii. the greatest height reached,
a = -126 m/s2 iv. the acceleration when t = 2 .

Baffour Ba Series, Further Mathematics for Schools Page 610


Solution i. s = 10t + 16t3
i. s = 25.2t – 4.9t2. v= = 10 + 48t2
At t = 3
v = 10 + 48t2
s = 25.2(3) – 4.9(3)2
s = 75.6 – 44.1 At t = 15
s = 31.5 m v = 10 + 48(15)2
v = 10,810m/s
v= = 25.2 - 2(4.9t)
v = 25.2 – 9.8t ii. a = = 2(48)t
a = 96t
At t = 3,
v = 25.2 – 9.8(3) At t = 15
v = 25.2 – 29.4 a = 96 × 15
v = - 4.2 m/s a = 1440 m/s

ii. When it is momentarily at rest, v = 0 7. The displacement s meters and the time t
0 = 25.2 – 9.8t
9.8t = 25.2 seconds of a particle is given by the relation
t= s = ut + gt2. Find in terms of u and g ;
t = 2.6s i. the velocity of the particle at time t seconds,
ii. the velocity of the particle at 10 seconds;
iii. At the greatest height reached, t = 2.6s iii. the acceleration of the particle at time t
s = 25.2(2.6) – 4.9(2.6)2 seconds.
s = 65.52 – 33.124
s = 32.4 m Solution
i. s = ut + gt2
iv. At t = 2 . v= = u + gt
a= v = u + gt
a = - 9.8 m/s
ii. v = u + gt
6. The relation the displacement s meters and At t = 10
the t seconds of a particle is given by the v = u + 10g
relation s = 10t + 16t3. Find at a time t = 15s;
iii. v = u + gt
i. the velocity of the particle,
ii. the acceleration of the particle. a= =g ( u is a constant)
a =g
Solution

Baffour Ba Series, Further Mathematics for Schools Page 611


7. It is estimated that x months from now,the reach its maximum height and determine the
population of a certain town will be P(x) = x2 + maximum height.
20x + 8000.
a. At what rate will the population be changing 2. A body is thrown in the air such that after
with respect to time? time t, the distance in meters travelled is given
b. By how much will the population actually by s = 144t – 16t2. Find ;
actually change during the 16th month ? i. the velocity at t = 0
ii. the time the velocity is zero
Solution iii. the distance the body has travelled at the
Rae of change of population with respect to time in question.
time is the derivative of the population
function. 3. A stone is thrown up from the top of a 15
Rate of change = P1(x) meters coconut tree. If the relation connecting
= 2x + 20 displacement s meters and time t seconds is
s = 15 + 80t + 16t2. Find;
The rate of change of the population 15 months i. the velocity with which the stone was thrown
from now will be: up;
ii. the time the stone has zero velocity ( that is
P1(15) = 2(15) + 20 reached its highest point in the air);
iii. the velocity of the stone at twice the time in (ii);
= 50 per month
iv. the time the stone reached the bottom of the
b. Change in population; coconut tree;
= P(16) – P(15) v. the velocity of the stone at the time in (iv);
But vi. the greatest height the stone reached.
P(16) = 162 + 20(16) + 8000
= 8,576 4. A particle is moving in a straight line. It‟s
distance, s meters from a fixed point O after t
P(15) = 152 + 20(15) + 8000 seconds is given by s = t3 – 9t2 + 15t + 2.
= 8,525 Calculate;
i. the velocity at any time t;
P(16) – P(15) ii. its velocity after 6 seconds;
= 8,576 – 8,525 iii. the distance of the particle from o when it is
= 51 people instantly at rest;
iv. its acceleration after 4 seconds.
Exercises 20.18
1. Suppose a ball is thrown up into the air and 5. A car , starting at t = 0 seconds, travels a
its height after t seconds is s = 5 + 24t – 16t2 distance of s meters in t seconds where s = 30t
feet. Determine how long it will take the ball to – t2 .

Baffour Ba Series, Further Mathematics for Schools Page 612


i. Find the speed of the car after 2 seconds. (15.4t – 4.9t2) m. Find the greatest height it
ii. After how many seconds is the speed of the reaches, and the time it takes to get there.
car equal to zero.
iii. Find the distance travelled by the car up to Rate of Change 2
the time its speed is zero. Small Change and Percentage Error
Consider a function defined by y = f(x). If x is
6. The air resistance R to a body moving with increased by a small amount ∆x to x + ∆x , then
speed in meters per seconds is given by : as ∆x → 0, and .
R= .
i. Find the rate of change of the air resistance Therefore, if ∆x is small,
with respect to the speed.
ii. Calculate the rate of change when v = 16m/s ∆y ≈ ∆x or ∆y ≈ f ′(x ) ∆x

7. A parachutist jumps out of an aero plane. This result can be used to find solutions to
The distance h meters through which she falls problems of the type:
after t seconds is given by h = . Find; 1. If the radius of a circle is increased by a%
i. the distance she falls in the first second; what will be the corresponding change in the
ii. her velocity after 2 seconds . area?

8. A particle moves in a straight line so that its 2. If the radius of a circle is measured to be
distance s meters from a fixed point o at a time 10.0 ± 0.2 what will be the maximum error in
t is given by s = 1.5t3 – 10.5t2 – 4t + 10. calculating the area?
i. If the velocity after k seconds is 3.5m/s, find
the value of k. For problems involving rate of change;
ii. If its acceleration after q seconds is 6 m/s2, Identify the given rate, the given side (length,
find the value of q. height or radius) and the unknown rate. Set out
the formula for the unknown rate and
9. A particle moves along the x – axis in such a
way that the velocity v m/s after t seconds is differentiate the formula implicitly. Here, two
given by v = 2t3+ 9t2 – 5t . If the particle is at conditions arise.
the point (4, 0) at a time t = 0, find;
i. the position when t = 2. A. If the problem involves area (A) and radius
ii. the average velocity during the third seconds. (r), then it is likely that after implicity
iii. the acceleration when t = 3. differentiation of the formula of the unknown
rate, the given rate and the given side will
10. A ball is thrown vertically upwards from remain. In this case, substitute their values to
ground level and its height after t seconds is complete the work.

Baffour Ba Series, Further Mathematics for Schools Page 613


B. If it involves volume (V) and surface area (A), = 30
i. identify the formula of the known rate and
The area is increasing at 30 cm2 per second
differentiate it implicitly and equate to its given
value.
Method 2
ii. Observe the new rate . / that appears in the A = r2
diferential equation, and make it the subject. =3 =? =2 r
Name this as eqn (1).
iii. Identify the formula for the known rate.
Setting up the chain rule;
iv. Differentiate the unknown rate implicitly
and name it eqn (2). = .
vi. Observe the re – appearance of the same = 2 r (3)
new rate . / in equation (2) and make it the
subject. Name this as eqn (1) At r = 5
vii. Substitute eqn (1) in vi and the value of the = 2 (5) (3)
given side in eqn (2). = 30
viii. Simplify, where possible to complete the
The area is increasing at 30 cm2 per second.
work.

Worked Examples 2. The radius of a circle is 12 cm . Find leaving


1. The radius of a circle is increasing at a rate the answer in terms of , the rate at which the
of 3 cm per second. Find the rate of change of area is increasing when the radius is increasing
the area inside the circle when the radius is at a rate of 0.2
5cm.
Solution
Solution r = 12cm, = ? and = 0.2
= 3, = ? and r = 5
Area of a circle;
The area A and the radius r of the circle is A=
related by A = r2 =2 r

Differentiate both sides with respect to time, At r = 12 and = 0.2;


2
= ( r)
= 2 (12) (0.2)
=2 r (implicit)
= 4.8 cm2/s

At r = 5 and =3
Method 2
= 2 (5) (3) r = 12cm, =? = 0.2

Baffour Ba Series, Further Mathematics for Schools Page 614


A= Put eqn (1) in eqn (2);
=2 = 12 . /
= 2 (12) But r = 12 = 12. /
= 24 = 12. /

Using Chain rule, At l = 10


= × = 12. /
= 24 × 0.2 = 3.6cm3/s
= 4.8
4. The volume of a cube is increasing at a rate
3. The volume of a cube is increasing at a rate of 2 . Find the rate of change of a side
of 9 cm3/s. How fast is the surface area of the base when its length is 2cm.
increasing when the legth of an edge is 10 cm?
Solution
Solution Let l represent side of the cube
Let V be the volume of the cube;
L be the length of a side, = 2 = 2.5 , = ? and l = 2
V be the volume,
A be the surface area.
V=
=3 (implicit diff)
= 9, = ? and l = 10
⇒3 = 2.5
Volume of a cube; =
V=
=3 At l = 2
= ( )
But =9 = 0.2083
⇒3 =9
= ………………...(1) 5. The volume of a sphere is increasing at a rate
of 3 cm3/s. Find the rate of increase of its
surface area, when the radius is 2 cm.
Surface area of a cube;
A=6
Solution
= 12 …………...(2)
=3, = ? and r = 2

Baffour Ba Series, Further Mathematics for Schools Page 615


Volume of a sphere; =3. / (implicit diff)
V=
=4
=3. / (implicit diff)
⇒4 = 0.25
=4
=
⇒4 =3
= ……………..(1)
=
= ……………..…(1) Since the value of r is not given, determine it
from the given volume.
Surface area of a sphere; V= = 10
A=4 = 10
= (2) 4 =
=8 …………….(2)
r= √

Put eqn (1) in eqn eqn (2);


r= √
=8 . /
=2. / r= √

=
Put r = √ in eqn (1);
At r = 2;
=
= 4√ 5

=3 = 0.0163

6. The volume of a spherical balloon is


7. The radius of a right circular cylinder is
increasing at a constant rate of 0.25 m3 .
decreasing at arate of 4ft/min, while the height
Find the rate at which the radius is increasing at
is increasing at a rate of 2 ft/min. Find the rate
the instant when the volume is 10 m3.
of change in the volume when the radius is 2
feet and the height is 6 feet
Solution
= 0.25 , = ? and V = 10 Solution
=2, = - 4, r = 2 and h = 6
Volume of a sphere; Volume of a cylinder ;
V= V= h

Baffour Ba Series, Further Mathematics for Schools Page 616


= 0 1 7. If the radius of a sphere is increasing at a rate
of 0.4cm/s, how fast is the volume increasing
when the diameter is 80 cm?
= , ( ) ( )-
At r = 2 and h = 6 8. The volume of a spherical ballon is
= ,( ) ( ) ( )( )( )- increasing at the rate of 20cm3/s. Find the rate
of change of its surface area at the instant when
= ,8 - its radius is 8 cm?
= 88
9. The volume of a cube is increasing at a rate
The volume is decreasing at a rate of
of 8cm3/s. How fast is the surface area
88 /min
increasing when the length of an edge is 12cm?
Exercises 20.19
1. If the volume of a sphere is increasing at a 10. The volume of a sphere is decreasing at a
rate of 9 m3 . Find the rate of change of its constant rate of 3cm3/s. At the instant when the
radius of the sphere is decreasing at the rate of
surface area when its volume is cm3.
0.25 cm/s, what is the radius of the sphere?

2. If the radius of a sphere starts to increase at a 11. The volume of a cube is increasing at a rate
rate of 0.1 cm/s, what will be the rate of change of 300cm3/min at the instant when the edge is
of volume, at an instant when the radius is 10 20cm. Find the rate at which the edge is
cm? changing.

3. The edge of a cube is increasing at a rate of


12. The radius of a circle is increasing at rate of
10cm/s. How fast the volume of the cube will
7cm/min. Find the rate of change of the area
increase when the edge is 15 cm long?
wnhe radius is 12cm.
4. The volume of a cube is increasing at arate of
13. If the volume of a sphere is increasing at a
9cm3/s. How fast is the surface area increasing
rate of 8 m3 . Find the rate of change of its
whent the length of an edge is 10cm?
surface area when its volume is cm3.
5. Find the rate of change of the area of a circle
with respect to its radius when radius is 5cm. For problems involving small change:
i. Identify the figure in question and state its
6. The radius of a circle is increasing at rate of area, surface area, perimeter or whatever, as
0.7cm/sec. stated in the question.
i. What is the rate of increases of its ii. Differentiate the formula with respect to the
circumference when radius is 10cm. length of a side or radius.
ii. What is the rate of increases of its area when iii. Set out the small change formula as:
radius is 10cm.

Baffour Ba Series, Further Mathematics for Schools Page 617


Exercises 20.20
1. If the radius of a sphere is increased from 10
iv. Substitute the known values and work out
cm to 10.1 cm what is the approximate increase
for the unknown.
in surface area?
Worked Examples
2. The height of a cylinder is 10 cm and its
1. The side of a square is 5cm. How much will
radius is 4cm. Find the approximate increase in
the area of the square increase when the side
volume when the radius increases to 4.02 cm.
expands by 0.01cm?
Percentage Error
Solution
Percentage error is calculated by the formula:
Let the area of the square be A and the length of ( )
a side be x cm. % error = × 100%

Then A = x2 ⇒ = 2x and x = 5
Note that whenever an error occurs, there is a
change. Therefore, the small change formula is
From = 2x; helpful here.
set out the small change formula as A change in volume is indicated as ∆v;
A change in surface area is indicated as ∆s;
where ∆A is the change in area
A change in area is indicated as ∆a;
∆x is the change in side
A change in radius is indicated as ∆r and so on.
∆A = ?, ∆x = 0.01 , = 2x and x = 5
, , indicate % error in volume, surface
∆A ≈ ∆x area and radius respectively;
∆x = (2) (5) (0.01) = 0.1
The increase in area ≈ 0.1 cm2 Worked Examples
1. A 2% error is made in measuring the radius
Method 2 of a sphere. Find the percentage error in the
A = x2 =? volume.

= (x2) Solution
= 2x Let the radius be r and the volume be V .
V= r3
But x = 5, and = 0.01 = 4πr2
= 2(5) (0.01) Error in radius ∆r = 2% r = = 0.02r
= 0.1 Error in volume ∆v = ?
The increase in area ≈ 0.1 cm2
Set out the small change formula as;

Baffour Ba Series, Further Mathematics for Schools Page 618


( )
= = × 100%
4πr2 = ≈ × 100 %
2
∆v = 4π r 0.02 r
= × 100%
= 0.08π r3
= × 100%
The percentage error in the volume; =4%
( ) The percentage error in the volume ≈ 4%
= × 100%

≈ × 100 % 3. An error of 9% is made in measuring the


volume of a sphere. Find the corresponding
= × 100%
percentage error made in measuring its radius.
= × 100%
Solution
=6% Let V be the volume of the sphere and r be the
The percentage error in the volume ≈ 6% radius of the sphere,
V=
2. A 2% error is made in measuring the radius
of a sphere. Find the percentage error in surface =3×
area. =4

Solution
% error in volume = 9% = 0.09
Let the surface area be S and the radius be r,
S = 4 r2 Error in radius ∆r = ?
=8 r
Set up the small change formula as;
=
Error in radius, ∆r = 2% r = 0.02r
Error in surface area ∆s = ? ∆v = × ∆r
∆v = 4 × ∆r
Set out the small change formula as ;
= =

8 r= =
∆s ≈ 8 r (0.02r)
Percentage error in Volume, = 9%
∆s ≈
∆s ≈ 0.16 9% =
=
The percentage error in surface area;
= 3%

Baffour Ba Series, Further Mathematics for Schools Page 619


4. The volume of a sphere is increased by 6%. ∆A = 8 ∆r
Calculate the corresponding percentage error
=
increase in its area.
=
Solution
V=
But = 2%
=4
⇒ = 2(2%)
= 4%
% error in volume, = 6%,
Percentage increase in area is 4%
% error in radius =?
But Exercises 20.21
1. An error of 3% is made in measuring the
∆v = × ∆r
radius of a sphere. Find the percentage error in
∆v = 4 × ∆r the volume of the sphere.
=

2. An error of 2.5% is made in the measurement
= of the area of a circle. What is the percentage
error in :
Percentage error in volume, = 6% a. the radius of the circle?
b. the circumference of the circle?
6% =
3. The error in the measurement of the radius of
= a sphere is 2%. What will be the error in the
= 2% calculation of its volume?

% error in radius = 2% 4. If the percentage error in the side of a cube is


3%, what is the percentage error in its volume?
Let A be the area of the sphere,
A= 4 5. What is the maximum percentage error in the
=8 measurement of a side of a cube, if its
percentage error in the measurement of its
% error in radius, = 2%
volume is 6%.
% error in area, =?
6. If 2% is the percentage error of measurement
= of a side of a square, then find the percentage
error in the measurement of its:
∆A = × ∆r i. area; ii. volume.

Baffour Ba Series, Further Mathematics for Schools Page 620


Challenge problems Worked Examples
1. If there is 2% of error in measuring the 1. i. Find the coordinate of the point on the
volume of a cube, then what will be the curve y = 4x – 2x2 where the gradient is zero.
percentage error in measuring a side of a cube ii. What is the greatest or least value.

2. An error in the measurement of the radius of Solution


a sphere is 2%. What will be the error in the y = 4x – 2x2
calculation of its volume. At the greatest point, =0
(4 x – 2x2 ) = 0
3. The kinetic energy K of a body of mass m
4–4x=0
moving with speed v is given by K = mv2 . If a
4(1 – x ) = 0
body‟s speed is increased by 1.5% what is the 1–x=0
approximate percentage change in the kinetic x=1
energy? When x = 1, y = 4(1) – 2(1)2
y=4–2
Greatest and Least Values y=2
Curves of the form ax2 + bx + c = 0, produce a At the point (1, 2) the gradient is zero.
∪ or ∩ shape depending on the nature of the
coefficient of x2. When it is positive, we have a ii. The greatest value is y = 2
minimum curve ∪ and when it negative, we
have a maximum curve ∩. 2. i. At what point on the curve y = 4x2 – 6x + 2
is the gradient zero?
A ∪ – shaped curve decreases from left to the ii. Find the greatest or least value
minimum or least point where its gradient is
zero and starts increasing to the right from the Solution
minimum point. At the least point, the gradient y = 4x2 – 6x + 2
is zero.
At the least point =0

Likewise, an ∩ – shaped curve increases from (4 x2 – 6x + 2) = 0


right to the maximum or greatest point where 8x – 6 = 0
its gradient is zero and starts decreasing to the 8x = 6
right from the greatest point. At the maximum x= =
point, the gradient is zero.

At the maximum or minimum point, the When x = , y = 4. / – 6 . / + 2 =


gradient is zero, meaning =0 Therefore, at the point . , / the gradient is
zero.

Baffour Ba Series, Further Mathematics for Schools Page 621


ii. The least value is y = At the turning point;
=0
3. Let x + y = 13 where x, y > 0. If A = 2x + 3y ⇒ 2ax + 12 = 0
+ xy , write A as aquadratic in x. Calculate the
maximum value of A. At x = 2,
2a(2) + 12 = 0
Solution 4a = -12
x + y = 13……………..(1) a = -3
A = 2x + 3y + xy ………(2)
ii. Put a = -3 in y = ax2 + 12x + 1
From eqn (1), y = -3x2 + 12x + 1
y = 13 – x The curve is maximum because the co-efficient
Put y = 13 – x in eqn (2) of x2 is negative.
A = 2x + 3(13 – x) + x(13 – x)
A = 2x + 39 – 3x + 13x – x2 At x = 2,
A = -x2 + 12x + 39 (max curve ) y = -3(2)2 + 12(2) + 1
y = 13
At the maximum point; The curve y = -3x2 + 12x + 1 is maximum at the
=0 point (2, 13)
= - 2x + 12
Exercises 20.22
- 2x + 12 = 0 A. Find the turning points on the following
- 2x = - 12 curves and determine whether y has a
x=6 maximum or minimum value at each;
1. y = x3 – 12x 2. y = x2 (3 – x )
Maximum value of A;
3. y = x(x – 8) (x – 15)
A = - x2 +12x + 39
A = - (6)2 + 12(6) + 39 4. y = x3 – 5x2 + 3x + 2 5. y = 4x2 +
A = 75 (maximum curve)

4. i. Given that the curve, y = ax2 + 12x + 1 has B. Find the maximum or minimum values.
a turning point at x = 2, calculate the value of a. 1. f (x) = 4x – 3x3
ii. Show whether the point is maximum or 2. f (x) = 2x3 – 3x2 – 12x – 7
minimum? 3. f (x) = x + 4. f (x) = x2 (x – 4)

Solution C. 1. Let x + y = 12 where x, y > 0. If A = x2 +


i. y = ax2 + 12x + 1 y2 , write A as aquadratic in x. Calculate the
= 2ax + 12 minimum value of A.

Baffour Ba Series, Further Mathematics for Schools Page 622


2. The curve px2 + qx + r has a maximum Worked Examples
turning point at (2, 18). If (0, 10) is a point on 1. Find the location of the stationary points of
the curve, find the values of p , q and r. the function y = x3 – 3x + 2

3. Find two positive real numbers x and y Solution


satisfying 2x + y = 15 such that x2 + y2 is y = x3 – 3x + 2
maximized. = 3x2 – 3
=0
Maxima and Minima
Stationary Points and their Nature ⇒ 3x2 – 3 = 0
At any point on a curve when the gradient is 3(x2 – 1) = 0
zero, y is said to have a stationary value. In 3(x – 1) (x + 1) = 0
other words, stationary points are located where x = 1 or x = - 1
the gradient of the curve is equal zero. The
At x = 1, y = 13 – 3 (1) + 2
three types of stationary points are:
y=1–3+2
1. a maximum
y=0
2. a minimum
Stationary point (1, 0)
3. a point of inflection

At x = -1, y = (-1)3 – 3 (-1) + 2


Local max A
y=-1+3+2
y=4
C Stationary point (-1, 4)
Therefore, the function y = x3 – 3x + 2 has two
stationary points at (1, 0) and (-1, 4)
Local min B 3. For what values of x will the function f : x →
x3 – 6x2 + 5 have turning points?
Note
At all turning points = 0, but not all
Solution
stationary points are turning points. f : x → x3 – 6x2 + 5
(x) = 3x2 – 12x
Finding the Stationary Point of a Function
1. Differentiate the function to obtain At the turning point,
2. Set = 0, and solve for the value(s) of x. 3x2 – 12x = 0
3x (x – 4) = 0
3. Substitute the value of x in the original
3x = 0 or x – 4 = 0
function to obtain the corresponding y – value.
x = 0 or x = 4

Baffour Ba Series, Further Mathematics for Schools Page 623


4. Find the turning points of the function 1. y = 3x2 – 4 4. y = 4x4
( )
y= 2. y = x2 – x
3. y = 12x3 – 4 5. y = x4 – x3 + x2
Solution
( ) Using the Second Derivative to Determine the
y=
Nature of the Turning Points
Let u = (x – 1)2
The second derivative of a function can be used
= 2(x – 1) (1) to classify the type of stationary point.
= 2x - 2 Mathematically, the second derivative is
written as ; = . /
Let v = x
=1 The left- hand side is the usual way of writing
the second derivative.

By substitution, Observations
( ) ( ) ( ) 1. If the second derivative is positive, then the
gradient of the graph is always increasing. The
( )
stationary point is a minimum. That is > 0.
( )
2. If the second derivative is negative, then the
) gradient of the graph is always decreasing. The
stationary point is a maximum. That is <0

( )( )
3. If the second derivative is zero, then the
gradient of the graph is neither increasing nor
decreasing. Further investigation is required at
At the turning point, =0
( )( ) this stage. That is =0
0=
(x – 1) (x + 1) = 0 If = 0, pick a point on either side of the
x = 1 or x = - 1 stationary point and observe if any of the
following conditions exist;
( ) a. Both y – coordinates less than that of
When x = 1, y= =0
( ) the stationary point ⇒ maximum
When x = -1, y = =4
The stationary points occur at (1, 0) and (-1, 4) b. Both y – coordinates greater than that of
the stationary point ⇒ minimum
Exercises 20.23
Find the coordinates of the stationary points.

Baffour Ba Series, Further Mathematics for Schools Page 624


c. One coordinates less than and another Now, find the value of second derivative at
coordinate greater than that of the each stationary point(s)
stationary point ⇒ point of inflection At x = 1, = 6(1) = 6 (positive)
The point (1, 0) is a local minimum point.
Worked Examples
1. Classify the stationary points of the function
y = x3 – 3x + 2 At x = -1, = 6(-1) = - 6 (negative)
The point (-1, 4) is a local maximum point.
Solution
y = x3 – 3x + 2 2. Find the location and nature of the stationary
2
= 3x – 3 points of y = 3x3 – 5

=0
Solution
⇒ 3x2 – 3 = 0 y = 3x3 – 5
3(x2 – 1) = 0 = 9x2
3(x – 1) (x + 1) = 0
x = 1 or x = - 1 =0
This means that we have an x – coordinate of 1 ⇒ 9x2 = 0
and another x – coordinate of -1. x=0
At x = 0, y = 3(0)2 – 5
y = -5
At x = 1, y = 13 – 3 (1) + 2 The stationary point is (0, -5)
y=1–3+2
y=0 To classify the stationary point,
Stationary point (1, 0) = (9x2 )

At x = -1, y = (-1)3 – 3 (-1) + 2 = 18x


y=-1+3+2
At x = 0, = 18(0) = 0 (positive)
y=4
Stationary point (-1, 4) Therefore, the point (0, -5) is neither increasing
nor decreasing.
Therefore, the function y = x3 – 3x + 2 has two
Further investigation;
stationary points at (1, 0) and (-1, 4)
Pick values of x, either side of 0, for example
x = -1 and x = 1
To classify the stationary points,
= (3x2 – 3 ) x -1 0 +1
y = 3x – 52
-8 -5 -2
= 6x

Baffour Ba Series, Further Mathematics for Schools Page 625


Observation Solution
One value is larger and the other smaller than i. y3 + 3xy2 – x3 = 3.
the value of y at the stationary point . 3y2 +3. ( ( ) / – 3x2 = 0

Conclusion 3y2 + 6xy + 3y2 – 3x2 = 0


Therefore, the function has a rising point of 3y2 + 6xy = 3x2 - 3y2
inflection.
(3y2+ 6xy) = 3x2 - 3y2
2. Investigate the stationary points of y = 3x4 – =
4x3
ii. At the stationary point,
Solution
=0
y = 3x4 – 4x3
= 12x3 – 12x2 ⇒ =0
At the stationary point, =0 3x2 – 3y2 = 0
3x2 = 3y2
12x3 – 12x2 = 0
x=y
12x2 (x – 1) = 0
x = 0 or x = 1
iii. At (1, -1)
When x = 0 , y = 0 =
When x = 1 , y = -1 ( ) ( )
=
The stationary points are (0, 0) and (1, -1) ( ) ( )( )

=0
To investigate the stationary point;
y + 1 = 0 (x – 1)
= 36x2 – 24x y+1=0
The equation of the normal is y + 1 = 0
At x = 0, =0
The point (0, 0) is a point of inflection 4. Let y = x3 – x2 – 9x + 9 be the equation of a
curve.
At x = 1, = 36 – 24 = 12 i. determine the intercepts the coordinates make
The point (1, -1) is a local minimum point. with the x – axis.
ii. find the turning point on the curve,
3. A function is given as y3 + 3xy2 – x3 = 3. distinguishing between them.
i. Find an expression for .
Solution
ii. Find the stationary point.
y = x3 – x2 – 9x + 9
iii. Find the equation of the normal at the point
(1, -1) On the y – axis, x = 0

Baffour Ba Series, Further Mathematics for Schools Page 626


y = 03 – 02 – 9(0) + 9 To classify the stationary point, use the second
y=9 derivative test,
(0, 9) = 6x – 2

On the x – axis, y = 0
When x = 2.1,
0 = x3 – x2 – 9x + 9
= 6 (2.1) – 2 = 10.6 > 0
Using factor theorem, Hence, (2.1, -5.05) is a minimum point.
x3 – x2 – 9x + 9 = 0
(x – 1) (x – 3) ( x + 3) = 0 When x = -1.4,
x = 1 or x = 3 or x = - 3 . = 6 (-1.4) – 2 = 10.4 < 0
Hence, and (-1.4, 16.9) is a maximum point.
Intercept on the x – axis are the points (1, 0),
(3, 0) and (-3, 0) Exercises 20.24
A. Classify the stationary points of the
ii. y = x3 – x2 – 9x + 9 following curves.
= 3x2 – 2x – 9 1. y = x9 4. y = 10x – x2
2
2. y = x + 3x + 4 5. y = 12(x + 1)3 + 6
At the turning point ; 3. y = x2 – 2x + 5 6. y = 3x2 + 6x – 5
=0
3x2 – 2x – 9 = 0 B. Locate the position and nature of the
a = 3, b = -2 and c = - 9 turning points of the following functions.
1. y = x2 – 2x 4. y = -3x2 + 3x + 1

x=
( ) √( ) ( )( ) 2. y = x2 + 4x + 1 5. y = x4 + 2
( ) 3. y = 12x – 2x2 6. y = 7 – 2x4
( ) √
x=

C. Locate the position and nature of the
x= turning points of the following functions.
x=

or x =
√ 1. y = 2x3 – 9x2 + 12x
2. y = 4x3– 6x2 –72x + 1
x = 2.1 or x = -1.4
3. y = - 4x3 + 30x2 – 48x – 1
When x = 2.1, y = (2.1)3 – (2.1)2 – 9(2.1) + 9
y = -5.05 Appliction of Maxima and Minima
Steps
When x = - 1.4, y = (-1.4)3 – (-1.4)2 – 9(-1.4) + 9 1. Read the problem carefully.
y = 16.9 2. If possible, make a sketch of the problem.
The turning points are (2.1, -5.05) and 3. Define variables to be used and carefully
(-1.4, 16.9) label the diagram with the variables.

Baffour Ba Series, Further Mathematics for Schools Page 627


4. Write an expression for the quantity that is When x = 3, P = 3(9 – 3)2
going to be maximized or minimized. Eliminate P = 3 (6)2
some of the variables. Form an equation for this P = 108
quantity in terms of a single independent
variable. When x = 9, P = 3(9 – 9)2
5. If y = f(x) is the quantity to be maximized or P = 3 (0)2
mininmized, find the value of x for which =0 P=0
The maximum value of the produc P is 108,
6. Test each for which = 0 to determine if it
occurring at x = 3 and y = 6
a local maximum, or local minimum or neither.
7. If the y = f(x) is defined on an interval [a, b] 2. An open rectangular box with square base is
or (0, ]. Check the value of the function at the to be made from 48 cm2 of material. What
end points. dimensions will result in a box with the largest
possible volume.
Worked Examples
1. Find two non negative numbers whose sum Solution
is 9 and so that the product of one number and Let variable x be the length of one edge of the
the square of the other number is a maximum. square base and variable y be the height of the
box.
Solution
Let x and y represent the two nonnegative
numbers. The sum of the two numbers is given
to be:
9=x+y x y
y=9–x
x

Maximizng the product; Surface area of the box = 48


P = xy2 48 = (area of base) + 4(area of one side)
P = x(9 – x)2 48 = x2 + 4xy
= x . 2(9 – x) (- 1) + (9 – x)2 (1) 4xy = 48 – x2
= -2x (9 – x) + (9 – x)2 y=
= - 18x + 2x2 + 81 – 18x + x2
= 3x2 – 36x + 81 y= –
= x2 – 12x + 27 y= –
(x – 9) (x – 3) = 0
x = 9 or x = 3 To maximize the volume;
V = LWH
Hence the possible values for P to be maximize V = (x) (x) (y)
are, x = 9 and x = 3
Baffour Ba Series, Further Mathematics for Schools Page 628
V = x2y Solution
V=x .2
/ Let the legth of the rectangle be x and the and
the width be y cm, and the area be A m2
V = 12x –
V = 12x – (x3)
y

= 12 – x2 x

=0 The perimeter of the rectangle = 1000m


0 = 12 – x2 2x + 2y = 1000
x + y = 500
0 = 48 – 3x2
y = 500 – x
48 – 3x2 = 0
3(16 – x2) = 0
From the diagram,
(16 – x2) = 0
A = xy
(42 – x2) = 0
A = x(500 – x)
4 + x = 0 or 4 – x = 0
A = 500x – x2
x = - 4 or x = 4 (ignore negative answer)

Because x and y are lengths it implies that 0 < x


Since the base is a square and there are 48cm2
≤ 500. The problem now reduces to finding the
of material, it follows that 0 < x ≤ √ 8 value of x in [0, 500] for which A is maximum.
Hence the possible values for V to be maximize
are x = 4 and x = √ 8 = 6 At the maximum point ;
( )
When x = 4, y= ( )
=2 (4, 2) = 500 – 2x
( )
When x = 6, y= = 0.5 (6, 0.5) When =0
( )
⇒ 0 = 500 – 2x
2
Substitute (4, 2) and (6, 0.5) in V = x y 500 – 2x = 0
At (4, 2); 500 = 2x
V = (4)2 (2) = 32 cm3 x = 250

At (6, 0.5); Hence the possible values for A to be maximize


V = (6)2 (0.5) = 18 cm3 are, x = 250 and x = 500
The maximum volume of the rectangle is Now;
32cm2, occurring at x = 4 cm and y = 2 cm When x = 250, A = 500(250) – (250)2 = 62500
When x = 250, A = 500(500) – (500)2 = 0
3. Find the dimension of a rectangle with
perimeter 1,000m so that the area of the The maximum area of the rectangle is 62,500
rectangle is a maximum. m2, occurring at x = 250m and y = 250 m

Baffour Ba Series, Further Mathematics for Schools Page 629


This means that the rectangle is a square. =0

4. A sheet of card board 3cm by 4 cm will be 0 = 12x2 – 28x + 12


made into a box by cutting equal sized squares 4(3x2 – 7x + 3) = 0
from each corner and folding up the four edges.
What will be the dimensions of the box with Using the quadratic formula;
largest volume? ( ) √( ) ( )( )
x= ( )
Solution √
x=
Let x be the length of one edge of the square cut
√ √
from each corner of sheet of cardboard. x= or x =
x = 1.77 or x = 0.56
x x
x x
When x = 1.77 ,
3 – 2x V = [4 – 2(1.77)] [3 – 2(1.77)]) (1.77)
x x V = (0.46) (-0.54) (1.77)
x 4 – 2x x V = - 0.44
4 Therefore, x ≠ 1.77, since volume cannot be
negative.
After removing the corners and folding up the
flaps, we obtain an ordinary rectangular box. When x = 0.56 ,
V = [4 – 2(0.56)] [3 – 2(0.56)]) (0.56)
V = (2.88) (1.88) (0.56)
x
V = 3.03
Therefore, the largest possible volume of the
3 – 2x box is 3.03cm3 which occurs at x = 0.56 cm
4 – 2x

Maximizing the volume of the box; 5. A container in the shape of a right circular
V = LWH cylinder with no top has surface area 3 ft 2.
V = (4 – 2x) (3 – 2x) (x) What height h and base radius r will maximise
V = (4 – 2x) (3x – 2x2) the volume of the cylinder?
V = 4 (3x – 2x2) – 2x (3x – 2x2)
V = 12x – 8x2 – 6x2 + 4x3 Solution
V = 4x3 – 14x2 + 12x Let r be the radius of the circular base and h be
the height of the cylinder.
= 12x2 – 28x + 12
Total surface area of the cylinder = 3
At the maximum point; (area of base) + (area of curved side) = 3

Baffour Ba Series, Further Mathematics for Schools Page 630


r2 + (2 rh) = 3 6. A square sheet of card board with each side a
2 rh = 3 – r2 cm is to be used to make an open - top by
h= cutting a small square from each of the cornes
h and bending up the sides. What is the side
h= – r length of the small squares if the box is to have
h= – as large a volume as possible?

Solution
Maximizing the volume of the cylinder ; x x
V = r2 h x x

But h = – a – 2x
V = r2 . / x x
3 x a – 2x x
V= r– r
a
= – r2
Let the side length of the small squares be x cm.
= (1– r2)
The side length of the open box is (a – 2x) cm
and the height is x cm. Here a is a constant, and
At the maximum point; x is a variable to work with.
=0
0 = (1– r2) From (a – 2x) = 0;
a = 2x
3 (1– r2) = 0
x=
3 (1– r) (1 + r) = 0
(1– r) = 0 or (1 + r) = 0 0≤x≤
r = 1 or r = -1 (ignore negative answer)
r=1 The volume of the box is given by ;
Since the base of the box is a circle and there V = x(a – 2x)2
are 3 cm2 of material, it follows that 0 < r ≤ V = x(a2 – 4ax + 4x2)
√ . Hence the possible values for V to be V = 4x3 – 4ax2 + a2x
maximize are r = 1 and r = √ = 12x2 – 8ax + a2
= (2x – a) (6x – a)
When r = 1, V = (1)2 . /=
( )
√ =0
When r = √ , V = (√ )2 . /=
(√ ) 0 = (2x – a) (6x – a)
(2x – a) (6x – a) = 0
The maximum or largest possible volume of the 2x – a = 0 or (6x – a) = 0
cylinder is ft2, occurring at r = 1ft 2x = a or 6x = a

Baffour Ba Series, Further Mathematics for Schools Page 631


x = or x = b. Find an expression for the volume V in terms
of x.
V. / = . / 0 – . /1 c. Find the maximum possible volume of the
V=0 box.

Using the second derivative test, Solution


= 24x – 8a
At x = h

= 24. / – 8a x
x
= 4a – 8a Surface area, A
= - 4a < 0 (maximum) A = x2 + 4xh (open top)
1200 = x2 + 4xh
Hence , x = is a local maximum point. 4xh = 1200 – x2
This means that the maximum value of the
h=
function is at x = .
h=. / cm
Put x = in V = x(a – 2x)2
b. V = x2h
V = . / 0 – . /1
But h =
V=. /0 – 1
V = x2 . /
V=. /0 – 1 V = 1200x – x3
V=. /0 1

V=. / . / iii. Maximizing the volume;


V = x2h
V= =
= 1200 – 3x2
The maximum volume is V =
=0
2
7. A 1,200 cm of materisl is available to make
a rectangular box with a square base and an ⇒1200 – 3x2 = 0
open top . Assume that all the material is used 3(400 – x2) = 0
to make the box. 400 – x2 = 0
a. Given that the square base has side length x x2 = 400
cm , find an expression for the height h of the x = 20
box in terms of x.

Baffour Ba Series, Further Mathematics for Schools Page 632


Substitute x = 20 in h = ii. Maximum possible volume;
( ) = 12a2 – 32a + 15
h= = 10
( )
=0

Substitute x = 20 and h = 10 in ⇒12a2 – 32a + 15 = 0


V = x2h
( ) √( ) ( )( )
V = (20)2 (10) = 4,000cm3 a= (Quadratic formula)
( )

Solved Past Question a=
1. A welder used a rectangular sheet of metel a=

or a =

with dimensions 5m by 3m to make an open
a = 2.06 or a = 0.61
rectangular box by removing equal squares of
sides a m from each corner of the sheet.
When a = 2 .06 ,
i. Show that the volume, Vcm3 of the box is
V = 4(2.06)3 – 16(2.06)2 + 15(2.06)
given by V = 4a3 – 16a2 + 15a.
V = 34.9673 – 67.8976 + 30.9
ii. Find, correct to the nearest whole number,
V = - 2.0303
the maximum possible volume of the box.
V = - 2m3
Therefore, x ≠ 2.06, since volume cannot be
Solution
negative.
a a
a a When x = 0.61 ,
V = 4(0.61)3 – 16(0.61)2 + 15(0.61)
3 – 2a
V = 0.9079 – 5.9536 + 9.15
a a V = 4.1043
a 5 – 2a a
V = 4m3
5 Therefore, the largest possible volume of the
(5 – 2a )m box is 4m3 which occurs at a = 0.61m
(3 – 2a )m
2. A chemical industry manufactures closed
cylindrical metal containers to hold 50 cm3 of a
a
chemical. The total surface area, S cm2 of the
container is S = . r / cm2, where r is

Volume of the box; the radius of the cylinder. Find the value of r
V = lbh that will give the minimum surface area.
V = (5 – 2a) (3 – 2a) (a)
Solution
V = (15 – 16a + 4a2) (a)
V = 4a3 – 16a2 + 15a S=. r / cm2

Baffour Ba Series, Further Mathematics for Schools Page 633


S=2 + 100 4. A farmer has 100m of metal railing with
which to form two adjacent sides of a
Minimizing the surface area; rectangular enclosure, the other two sides being
= 4 r – 100 two existing walls of the yard, meeting at right
angles. What dimensions will give him the
=4 r – maximum possible area?
=0
5. A rectangular sheep pen is to be made out of
⇒4 r – =0 1000m of fencing, using an existing straight
4 = 100 hedge for one of the sides. Find the maximum
3 area possible and the dimensions necessary to
r =
achieve this.
r= √
6. Find the dimensions of the rectangle that has
r = 1.9965 maximium area if its perimeter is 20 cm.
r = 2 cm (nearest whole number )
7. Find the dimensions of the rectangle of area
Exercises 20.25 100 square units that has the least perimeter.
A. 1. The figure below represents a rectangular
sheet of metal 8 cm × 5cm. 8. An open top box is to have a square base and
a volume of 13500 cm3. Find the dimension of
the box that minimize the amount of material
used.

9. A rectangular box without lid is to made


Equal squares of side x cm are removed from from a square cardboard of sides 18cm by
each corner, and the edges are then turned up to cutting equal squares from each corner and then
make an open box of volume, V cm3. Show that folding up the sides. Find the length of the side
V = 40x – 26x2 + 4x3. Hence find the maximum of the square that must be cut off if the volume
possible volume, and the corresponding value is to be maximized. What is the maximum
of x. volume?

2. Find two positive real numbers whose sum is 10. Build a rectangular pen with three parallel
50 and whose product is a maximum. partitions using 500 ft of fencing. What
dimensions will maximize the total area of he
3. Find two non negative numbers whose sum pen?
is 10 and that the sum of their squares is a
maximum. B. 1. A storage container is to be made in the

Baffour Ba Series, Further Mathematics for Schools Page 634


form of a right circular cylinder and have a a. Express c as a function of x and find its
volume of 28 m3. Material for the top of the domain.
container cost Ghȼ5 per square meter adnd the b. Find the dimension of the box so that the cost
material for the side and base coat Ghȼ2 per of material is minimized. What is the minimum
square meter. What dimenstions will minimized cost?
the total cost of the container?
2. A cylindrical soup cans are to be
2. An opened top cylindrical pot is to have manufactured to contain a given volume V. No
volume 250 cm3. The material for the bottom of waste is involved in cutting the material for the
the pot cost Ghȼ4 per cm2; that for its curved vertical side of each can, but each top and
sides cost Ghȼ2 per cm2. What dimensions will bottom which are circles of radius r, are cut
minimized the total cost of this pot? from a square that measures 2r units on each
side. Thus the material used to manufacture
3. A hollow plastic cylinder with a circular base each soup can has an area of A = 2 rh + 8r2
and open top is to be made and 10 m2 plastic is square units.
available. Find the dimension of the cylinder a. How much material is wasted in making each
that give the maximum volume and find the soup can?
value of the maximum volume. b. Find the ratio of the height to diameter for
the most economical can (that is requiring the
4. A chemical factory wishes to make a least amount of material for manufacture.
cylindrical container, of thin metal, to hold c. Use either the first or second derivative test
10cm3, using the least possible area of metal. If to verify that you haveminimized the amount of
the outside surface is S cm2, and the radius is r material used r making each can.
cm, show that S = 2 r2 + and hence find the
required radius and height for the container. 3. A closed rectangular box is made of very
(leave in the answer) thin sheet metal, and its length is three times its
width. If the volume of the box is 288 cm3,
5. A cylinder with open top with radius, r and show that its surface area is equal to:
height h has a surface area of 8 cm2. Find the . / cm3, where x is the width of the
largest possible volume of such a cylinder.
box. Find by differentiating the dimensions of
Challenge Problems the box of least surface area.
1. An open top box is to have a square base and
a volume of 10m3. The cost per square meter of Curve Sketching
material is Ghȼ5 for the bottom and Ghȼ2 for The follwing steps are involed in curve
the four sides. Let x and y be the lengths of the sketching.
box‟s width and height respectively. Let C be 1. Find the intercept on x and y axis. On the x –
the total cost of materaila required to make the axis, let y = 0 and solve for x. On the y – axis,
box. let x = 0 and solve for y.

Baffour Ba Series, Further Mathematics for Schools Page 635


2. Solve for x at the turning point, where =0 f(2) = 22 – 4(2) – 5 = - 9
and substitute the value of x obtained in the =2>0
original function y = f(x) to obtain the ordered
pair (x, y). The point (2, -9) is a minimum point
3. Test to find the nature of the turning points,
y
whether maximum or minimum, using the
second derivative test, at the value of x obtained
in (2). -1 5 x
-5
1. If > 0, the turning point is minimum. 
2, -9
2. If < 0, the turning point is maximum.
Make a sketch of the curve y = x3 – 3x + 2
Worked Examples
1. Sketch the curve y = x2 – 4x – 5 Solution
y = x3 – 3x + 2
Solution = 3x2 – 3
y = x2 – 4x – 5
At the x – intercept, y = 0
0 = x2 – 4x – 5 At the stationary point , =0
(x – 5) (x + 1) = 0 (factorization) 3x2 – 3 = 0
x = 5 or x = -1 3(x2 – 1) = 0
The points (5, 0) and (-1, 0) are the intercept on 3(x + 1) (x – 1) = 0
the y – axis. x = 1 or x = -1

At the y – intercept, x = 0 Put x = 1 or x = -1 in y = x3 – 3x + 2


y = (0)2 – 4(0) – 5 When x = 1, y = (1)3 – 3(1) + 2
y = -5 y=0
The point (0, - 5) is the intercept on the y – Stationary point is (1, 0)
axis.
When x = -1, y = (-1)3 – 3(-1) + 2
At the turning point, y=4
= 0 Stationary point is (-1, 4)
y = x2 – 4x – 5
The stationary points occurs at (1, 0) and (-1, 4)
= 2x – 4
To find the maximum or minimum point;
⇒2x – 4 = 0
= 6x
2x = 4
x=2
At x = 1

Baffour Ba Series, Further Mathematics for Schools Page 636


= 6 (1) = - 6 < 0 (maximum point)

= 6 > 0 (minimum point)


Exercises 20.26
A. Find the coordinates of the local
maximum point, the local minimum point
4 and the point of inflection of each of the
following. Draw a rough graph of the
function in each case.
-2 -1 1 2 1. y = 27x – x3
2. y = 12x – x3
3. y = x3 – 9x2 + 15x + 10
4. y = 2 – 3x2 – x3
At x = -1
5. y = x3 – 6x2
= 6 (-1) 6. y = x3 + x + 1

Baffour Ba Series, Further Mathematics for Schools Page 637


21 INTEGRATION Baffour Ba Series

Definition ∫ . / dx = y + c or ∫ (x) dx = f(x) + c


Integration is the inverse of differentiation. In
where c is an arbitrary constant or
other words, if the process of differentiation is
reversed, we arrive at integration.
With regard to the above example,
∫ 2x dx = x2 + c, where c can be 0, 3 or -5.
The symbol for integration is ∫, an elongated
c shows uncertainty; it can take any value
s.Thus ∫2x is read as “the integral of 2x dx” or
which is not certain at the time of integration.
“the integral of 2x with respect to x”.
Therefore, the result is called indefinite
integral.
The following example shows it:
y = x2 Formula for integration:
⇒ = 2x
∫ xn dx = +c
So,
∫ dx = ∫ 2x dx = x2 In words, increase the power by one and divide
by the new power.
∫ and dx go hand in hand and indicate the
integration of the function with respective to x. If a is a constant,
Likewise, ∫ s dt indicate the integration of s ∫ axn dx = a ∫ xn dx = + c, (n ≠1). This
with respect to t. The outcome of the
means that aconstant factor of the integrand can
integration is called integral.
be taken outside the symbol of integration.

Now look at the following three examples:


Worked Examples
y = x2 ⇒ = 2x A. Perform the following:
y = x2 + 3 ⇒ = 2x 1. ∫x dx 2. ∫x2 dx 3. ∫a dx 4. ∫ dx
2
y=x -5⇒ = 2x
Solution
1. ∫x dx
From the examples above, an issue arises when
it comes to integration: The surerity and = +c
certainty about the exact solution of ∫ 2x dx
cannot be acertained; it can be any of the above = +c
three:
y = x2 or y = x2 + 3 or y = x2 – 5 2. ∫x2 dx

In order to deal with the uncertainty, the basic = +c


integration is denoted as follows: = +c

Baffour Ba Series, Further Mathematics for Schools Page 638


3. ∫a dx Exercises 21.1
= ∫a (1) dx A. Integrate with respect to x .
= a ∫ x0 dx (x0 = 1) 1. 3x2 2. 3x4 3. 4. 5.
=a +c
= ax + c B. Integrate with respect to x
1. x2 2. – a 3. 4. 5.
4. ∫ dx
Integration of Sum or Difference
= +c To integrate a sum or difference, add or
subtract the individual integrals
= ⁄
+c
Worked Examples
= +c 1. Perform ∫ . / dx

B. Find the following: Solution


1. ∫ dx 2. ∫ dx 3. ∫

dx ∫. / dx


= ∫ xdx - ∫ 11dx + ∫ dx
Solution ⁄
= - 11x + 3 ∫ dx
1. ∫ dx

= +c = - 11x + 3 . +c

= x3 + c = x2 – 11x + 3 (2) ⁄
+c

= x2 – 11x + 6 +c
2. ∫ dx
= x2 – 11x + 6 √ + c
=∫ +c
= +c 2. Find ∫(2x – 3) (x2 + 1) dx

3. ∫ dx Solution

∫(2x – 3) (x2 + 1) dx
∫ ⁄ dx = ∫ 2x (x2 + 1) – 3(x2 + 1) dx
∫ dx = ∫ (2x3 + 2x – 3x2 – 3) dx
= ∫ (2x3 – 3x2 + 2x – 3) dx
= +c = ∫ 2x3 dx – ∫ 3x2 dx + ∫ 2x dx – ∫ 3 dx
= 2∫ x3 dx – 3 ∫ x2 dx + 2 ∫ x dx –3x
= +c =2. –3. +2. – 3x + c


= +c = – x3 + x2 – 3x + c

Baffour Ba Series, Further Mathematics for Schools Page 639


3. Perform ∫(x + 2)2 dx = ∫( ⁄
( ) ⁄
) dx
⁄ ⁄
= ∫( ) dx
Solution ⁄ ⁄

∫(x + 2)2 dx = + ⁄
+c

∫ (x2 + 4x + 4) dx ⁄

=6 + +c
= + + 4x + c

= + 2x2 + 4x + c 7. Find f(x) , if (x) is given y the following :


i. (x) = 7 + 6x ii. (x) = √
4. Find ∫ . / dx
√ Solution
i. (x) = 7 + 6x
Solution (x) = 7x0 + 6x
∫. / f(x) = + +c

∫( + ) dx
√ √ f(x) = 7x+ +c
∫ (x . +2 dx

=∫ +2 dx ii. (x) = √
=

+2

+c (x) = ⁄
⁄ ⁄

= 2 +4 ⁄
+c f(x) = +c

5. Integrate (x + 1)( x – 2) with respect to x . f(x) = ⁄


+c

Solution f(x) = +c
(x + 1)( x – 2)
Let y = (x + 1)( x – 2) Exercises 21.2
y = x2 – 2x + x – 2 A. Perfom the following integration:
y = x2 – x – 2 1. ∫ 2x5 dx
∫ =∫ dx 2. ∫ x7 – 3x + 2 ) dx
= – – 2x + c 3. ∫ (x2 – √ + 3) dx
4. ∫ (x2 – 5x + 1) (2 – 3x ) dx
6. Find ∫ dx 5. ∫ dx

Solution B. Integrate the following functions.


1. f(x) = 3x2 + 2 2. f(x) = 2x – 1
∫ √ 3. f(x) = x – 3x + 5 4. f(x) = x4 + 2 +
3 2

=∫ . / dx 5. f(x) = x5 + 5 6. f(x) = x – x2
√ √

Baffour Ba Series, Further Mathematics for Schools Page 640


C. Integrate the following functions. = 4,
2
1. f(x) = 2x + x 2. f(x) = 3x – dy = 4 dx
3. f(x) = √ + √ 4. f(x) = 1 – 2√ ∫ = ∫( ) dx
5. f(x) = 3 – 6. f(x) = y = 4x + c
√ √

When y = 10, x = - 2
The General Solution 10 = 4(-2) + c
If = xn , then dy = (xn ) dx 10 = - 8 + c
Integrating both sides; 10 + 8 = c
∫ = ∫( ) dx c = 18
y = ∫( ) dx ⇒ y = 4x + 18

y= +c ii. The solution represents a straight line

y= +c The Equation of a Curve


Given the gradient function and a point where
it occurs on a line or a curve, the actual
This is called the general solution
equation of the line or the curve can be
= xn is called differential equation obtained.

Steps:
Worked Examples
1. Find all the possible formula for y , given 1. Set = (x)
that = x3 – 3x dy = (x) dx
∫ y = ∫ (x) dx
y = ∫ (x) dx
Solution
= x3 – 3x
2. Obtain y = f(x) + c
dy = (x3 – 3x) dx 3. Substitute the values of the given point (x, y)
y = ∫( ) dx in y = f(x) + c to obtain the value of c
y= – +c (consatant of integration)
4. Substitute the value of c in y = f(x) + c to
obtain the equation of the line or curve.
2. i. If = 4, find y in terms of x, given that
y = 10 when x = -2. Worked Examples
ii. What does the solution represents 1. The gradient of a curve y = f(x) at any point
graphically? is x – x2. If the curve passes through the point
(3, 5), find its equation.
Solution

Baffour Ba Series, Further Mathematics for Schools Page 641


Solution y = x2 + 5x – 25
= x – x2 The equation of the curve is y = x2 + 5x – 25
dy = x – x2 dx
3. What is the gradient function of a straight
∫ =∫ dx
line passing through (- 4, 5) and (2, 6). Find its
y= – + c ………………..(1) equation.

At (3, 5) = (x, y) Solution


( ) ( ) (- 4, 5) and (2, 6).
5= – +c
m= = (gradient of a line joining two points)
5= – +c ( )

y–6= (x – 2 )
5– + =c
6 (y – 6) = x – 2
c= 6y – 36 = x – 2
6y = x – 2 + 36
Put c = in eqn (1) 6y = x + 34

y= – + y= +

The equation of the curve is y = – + Exercises 21.3


A. 1. A curve passes through the point (2, 0)
2. A curve passes through the point (3, -1) and and its gradient function is 3x2 – . Find its
its gradient function is 2x + 5. Find its
equation.
equation.
2. The gradient of a curve at the point (x, y) is
Solution
8x – 3x2 and it passes through the origin. Find
= 2x + 5
where it cuts the x – axis and find the equation
dy = 2x + 5 dx of the tangent parallel to the x – axis .
∫ =∫ dx
y= + 5x + c 3. Find y in terms of x if = 3x – , given that
y = x2 + 5x + c…………….(1) y = when x = 1

At (3, -1) = (x, y) ( )( )


-1 = (3)2 + 5(3) + c 4. Find y in terms of x if = .
-1 = 9 + 15 + c What is the value of y when x = 2, if y = 0
-1 – 9 – 15 = c when x = 1?
c = - 25
B. Solve the following ;
Put c = - 25 in equation (1) 1. = 3ax2 2. = 3t3 3. = u + ax

Baffour Ba Series, Further Mathematics for Schools Page 642


4. = 5. = 6. = At (1, 7). = (x, y)

7 = 5(1) – (1)2 + c
7=5–1+c
C. Find an expression for y if is given by 7=4+c
the following. c=7–4
1. 2x – 5 2. (x + 2) (x2 + 5) c=3
3. x3 + 7x2- 3x + 5 4. 2x2. /
Therefore, the function y = f(x) = 5x – x2 + 3
5. . /
Definite Integral
Finding the value of the Constant of The general integration gives a constant to
Integration signify the uncertainty of the numerical value
Worked Examples that could be added or taken away from the
1. Find the constant of integration , given that result. In definite integral, there is no room for
∫( ) = 6 when x = 2. the constant, as the integration is performed
between a certain range of the variable.
Solution ∫ (x) dx = , ( ) -
∫( ) =6
= (f(b) + c) – (f(a) + c)
+x+c=6
= f(b) – f(a)
At x = 2,
+ 2+ c = 6 The constant disappears; this is the definite

+2+c=6 integral as it gives a definite answer.

+c=6
Note
c=6– 1. The dx indicates that limits a and b are x
c= limits.
2. The constant a is called the lower limit of the
2. Find the function y = f(x), given that (x) = integral .
5 – 2x and that the graph of y = f(x) passes 3. The constant b is called the upper limit of the
through the point (1, 7). integral.
4. There is no constant of integration when
Solution definite integrals are evaluated, as they cancel
Let y = f(x), out each other.
(x) = 5 – 2x
Worked Examples
f(x) = 5x – +c
1. Find ∫ dx
f(x) = 5x – x2 + c

Baffour Ba Series, Further Mathematics for Schools Page 643


Solution
=0 1
∫ dx
( ) ( ) ( ) ( )
=0 1–0 1
=0 1
=( )–( )
=, - = 72
= 43 – 23
= 64 – 8
5. Express in the form (ax + b) and,
= 56
hence, evaluate ∫ dx
2. Find ∫ ( ) dx
Solution
( ) ( )( )
Solution = = = (2x + 3)
( ) ( )
∫ ( ) dx
∫ dx = ∫ ( ) dx
= 0 1
( ) ( ) ∫ ( ) dx
= 0 1– 0 1
= 0 1 –, - = 0 1
= 8.6 = ,( )-
= ,( ( )- – ,( ( )-
3. Evaluate ∫ ( )dx
= (, -–, -)
Solution = (10 – 0)
∫ ( )dx =5

=0 1 Exercises 21.4
Evaluate each of the following:
=0 1
1. ∫ dx 2. ∫ dx 3. ∫ dx
( ) ( )
=0 ( ) ( )1 – 0 ( ) ( )1
4. ∫ √ d dx 5. ∫ dx 6. ∫ dx
=. /– . / =. /=9
B. Evaluate each of the following:
4. Evaluate ∫ ( )dx 1. ∫ ( )dx
2. ∫ ( )dx
Solution
3. ∫ ( )dx
∫ ( )dx
4. ∫ ( ) dx
∫ ( )dx

Baffour Ba Series, Further Mathematics for Schools Page 644


C. Evaluate each of the following: 0 1=9
1. ∫ dx 2. ∫ dx k3 = 3 × 9
3. ∫ . / dx 4. ∫ . / dx k3 = 27

k=3
5. ∫ . / dx 6. ∫ . / dx

Exercises 21.5
D. Evaluate In each of the following, fing the value of k,
√ 1. ∫ ( ) dx = 6 2. ∫ dx = 30
1. ∫ . / dx 4. ∫ . / dx √


2. ∫ . / dx 5. ∫ . / dx
√ Integration by Substitution
3. ∫ . / dx Steps:

1. Define a new variable u = g(x) where g(x) is
chosen in such a way that g‟(x) “ is a factor” of
The Negative Integral
the integrand and that when written in terms of
It is an established fact that :
u, the integrand is simpler than when written
∫ ( ) dx = , ( )- = F(b) – F(a) interms of x.
2. Transform the integral with respect to x into
Let us consider; an integral with respect to u by replacing g(x)
∫ ( ) dx = , ( )- everywhere by u and g’(x) dx by du.
= [ ( ) – ( )] 3. Integrate the resulting function of u.
= F(a) – F(b) 4. Substiute back u = g(x) to express the results
in trems of x.
= ∫ ( ) dx

Worked Examples
∫ ( ) dx = ∫ ( ) dx 1. Find ∫ ( ) dx

Finding an Unknown Limit Solution


Worked Examples 1. ∫ ( ) dx
2
Let u = x + 1
1. Find the value of k > 0 , if ∫ dx = 9
= 2x
Solution du = 2x dx
∫ dx = 9 dx=

0 1 =9
Substitute dx= in ∫ ( ) dx
⇒0 1–0 1=9 ⇒∫ ( )
0 1 –0=9 = ∫( ) du

Baffour Ba Series, Further Mathematics for Schools Page 645


=∫ du
= +c
= +c (substitute u = x2 + 1)

( ) = +c
= +c ⁄


= +c
4
2. Find ∫ (2x + 3) dx = ⁄
+c (substitute u = 1 - x2)

Solution = ( ) +c
∫ (2x + 3)4 dx
Let u = 2x + 3 4. Find ∫ . / dx
=2
dx = Solution
∫. / dx
4
Put u = 2x + 3 and dx = in ∫ (2x + 3) dx ∫( ) dx
4
= ∫ (u)
Let u =
=∫
=
= +c du = dx
( )
= +c du = dx

3. Perform ∫ √ dx Substitute dx = du in ∫( ) dx

Solution ⇒∫( ) du

∫ √ dx =∫ ( ) du
2
Let u = 1 – x =∫ ( ) du
= -2x
= +c (substitute u = )
du = - 2x dx
dx = = +c
( )
( )
= +c
( )
Put u = 1 - x2 and dx = in∫ √ dx
⇒∫ √ ( ) Exercises 21.6
A. Find each of the following:
⇒∫ √ du
⁄ 1. ∫( ) dx 2. ∫ ( ) dx
= ∫ du
3. ∫ ( ) dx 4. ∫ ( ) dx

Baffour Ba Series, Further Mathematics for Schools Page 646


5. ∫ ( ) dx 6. ∫ ( ) dx =4
du = 4 dx
B. Perfom the following :
dx = du
1. ∫( ) dx 2. ∫ ( ) dx
3. ∫ dx 4. ∫ √ dx
√ Now, From eqn (1);
5. ∫ ( dx When x = 0, u = 4(0) + 1 = 1
)
When x = 2, u = 4(2) + 1 = 9
Change of Limits
When a u – substitution is used to evaluate a Old bounds for were 0 and 2 but new bounds
definite integral, each part of the integral for u is 1 and 9.
changes to use a new variable, usually u instead
of x. Observe the following; Put u = 4x + 1 and dx = du in the integral
1. The function itself changes from x to an ) dx
∫ √(
expression involving u.
2. The differential dx changes to an expression =∫ √ du
involving du. ⁄
=
3. The bounds of integration changes forn from

values of x into values of u. =0 1

=0 1
To change the bounds,
1. Use the expression that relate x and u. =0 ⁄
1
2. Put in the original lower bound for x and
⁄ ⁄
solve for u to obtain new lower bound. =0 ( ) 1– 0 ( ) 1
3. Put in the original upper bound for x and = ( 27 – 1 )
solve for u to obtain new upper bound.
=
Note:
Changing the limit or bound to that of u is 2. Evaluate ∫ dx

matter of choice, but that makes the
substitution easier. Solution

Worked Examples ∫ √

1. Evaluate ∫ √( ) dx Let u = x – 1 ……….(1)


=1
Solution dx = du
∫ √( ) dx
From eqn (1) ;
Let u = 4x + 1…………..(1) x=u+1

Baffour Ba Series, Further Mathematics for Schools Page 647


From equation (1); ∫ sec sec x + c
When x = 1, u = 1 – 1 = 0 ∫ csc cot - csc x + c
When x = 3, u = 3 – 1 = 2
Substitute x = u + 1, u = x – 1 and The next four idefinate integrals result from
dx = du in the integral ∫ dx trigonometric identities and u – substitution.

=∫ du
√ Integral Rule
=∫ ⁄ du ∫t n - ln |cos | + c
=∫ . / apply indices ∫t n ln |sec | + c
⁄ ⁄

⁄ ⁄ ∫ cot ln |sin | + c
=[ ⁄
] ∫ sec ln |sec t n |+c

∫ csc ln |csc cot | + c
⁄ ⁄
=0 1

=0 ( ) ⁄
( ) ⁄
1-0 ( ) ⁄
( ) ⁄
1 Worked Examples
Compute the following:
⁄ ⁄
=0 ( ) 1 1. ∫ sin
= 4.7140
2. ∫ t n

Exercises 21.7 3. ∫ cos ( )


Evaluate the following:
1. ∫ ( ) dx 2. ∫ ( ) dx Solutions
3. ∫ ( ) dx 4. ∫ ( ) dx 1. ∫ sin
Let u = 3x

5. ∫ √ dx 6. =3
dx = du
B. Evaluate:

1. ∫ √ dx 2. ∫ √
dx
By substitution;

3. ∫ √
dx 4. ∫ dx ∫ sin = ∫ sin . /
( )

= ∫ sin
Integration of Simple Trigonometric Function
= ( cos ) + c
Integral General Rule =– cos u + c
∫ cos sin x + c
=– cos 3x + c
∫ sin - cos x + c
∫ tan x + c
2. ∫ t n
∫ csc - cot x + c
Let u = 5x

Baffour Ba Series, Further Mathematics for Schools Page 648


=5 Type 1: (Presence of cosine only)
Use any of the relation where applicable.
dx = du
cos 2A = cos2 A − sin2 A
= 2 cos2 A − 1
By substitution; = 1 − 2 sin2 A
∫t n = ∫t n . /
Worked Examples
= ∫t n
Evaluate the integral of cos2 x.
= ln |sec | + c
= ln |sec |+c Solution
∫ x
From
Solution
cos 2A = 2 cos2 A − 1
3. ∫ cos ( )
2 cos2 A = cos 2A + 1
Let u = 2x – 6
cos2 A = (cos 2A + 1)
=2
cos2 A = (1 + cos 2A)
dx = du
If x = A;
cos2 x = (1 + cos 2x)
∫ cos( ) = ∫ cos
= ∫ cos
∫ x= ∫( cos ) dx
= sin u + c
= [∫ cos ∫ ( ) ]
= sin (2x – 6) + c
= 0 sin( )1 + c

Integrals requiring the use of trigonometric = x + sin (2x) + c


identities
The trigonometric identities that are required Type 2 : (Presence of sine only)
are summarised below: 1. If the exponent on the sine is odd, strip out
1. 2 sin A cos B = sin (A + B) + sin (A − B) one of the sines. The exponent on the
2. 2 cos A cos B = cos (A − B) + cos (A + B) remaining sines will then be even.
3. 2 sin A sin B = cos (A − B) – cos (A + B) 2. Convert the remaining sines to cosines using
4. sin2 A + cos2 A = 1 the identity, cos2 x + sin2 x = 1
5. cos 2A = cos2 A − sin2 A 3. Apply the substitution method to complete
= 2 cos2 A − 1 integration.
= 1 − 2 sin2 A
6. sin 2A = 2 sin A cos A Worked Examples
7. 1 + tan2 A = sec2 A 1. Find ∫ sin2 x dx.

Baffour Ba Series, Further Mathematics for Schools Page 649


Solution Now, using the substutition method,
The strategy is to use a trigonometric identity let cos x = u
to rewrite the integrand in an alternative form
which does not include powers of sin x. The ∫ x dx = – ∫ ( ) du
trigonometric identity applicable here is one of = ∫ 1 – 2u2 + u4 du
the „double angle‟ formulae: =-. /+c
cos 2A = 1 − 2 sin2 A……………….(1)
= - cos x + x– x+c
From eqn (1);
sin2 A = (1 − cos 2A) Note
If the exponent on the sines is even, this would
be difficult to do. We could strip out a sine,
Notice that by using this identity we can
but the remaining sines would then have an odd
convert an expression involving sin2 A into one
exponent and while we could convert them to
which has no powers in x. Therefore, our
cosines, the resulting integral would often be
integral can be written as :
even more difficult than the original integral in
∫ sin2 x dx = ∫ (1 − cos 2x) dx most cases.
= ∫ (1 − cos 2x)dx
Type 3(Combination of sine and cosine)
= 0. sin /1 + c Worked Examples
= x – . sin /+c Find ∫ sin 3x cos 2x dx.

= x – ( sin cos ) + c
Solution
= x – sin x cos x + c ∫ sin 3x cos 2x dx.
Use the identity;
2. Evaluate ∫ x dx 2 sin A cos B = sin (A + B) + sin (A − B) to
express the integrand as the sum of two sine
Solution functions.
∫ x dx sin A cos B = [sin (A + B) + sin (A− B)]
Substitution wont work because this integral ∫sin A cos B = ∫ [sin (A + B) + sin (A− B)]
do not have the cosine
∫sin A cos B = ∫ [sin (A + B) + sin (A− B)]
∫ x dx
=∫ x sin x dx (strip out sin x)
) sin x dx……………(1) Let A = 3x and B = 2x we have
= ∫(
∫ sin 3x cos 2x dx
From trigonometry, cos2 x + sin2 x = 1 = ∫ [sin (3x + 2x ) + sin (3x - 2x)] dx
sin2 x = 1 – cos2 x = ∫ (sin 5x + sin x)dx
= ∫( – ) sin x dx……………(1) = . cos cos / + c

Baffour Ba Series, Further Mathematics for Schools Page 650


= cos 5x – cos x + c At this stage the substitution u = cos x,
du = − sin x dx enables us to rapidly complete
the solution:
Exercises 21.8
Use the trigonometric identities to find the
∫ sin x (1 − cos2 x) cos2 x dx
following integrals.
= − ∫ (1 − u2) u2 du
1. ∫ cos2 x dx
⁄ = ∫ (u4 − u2 ) du
2. ∫
= − +c
3. ∫ sin 2x cos 2x dx
⁄ = − +c
4. ∫ ⁄ cos cos
5. ∫ (sin2 t + cos2 t) dt
6. ∫ sin 7t sin 4t dt. 2. Evaluate ∫ cos x dx

3. Integrals involving products of sines and Solution


cosines (integrals of the form ∫sinm x cosn x dx). ∫ cos x dx
Let u = sin x
Type 1: Integrals in which m is odd. ∫ cos x dx
In this integral, if the exponent on the sines (m) = ∫ du
is odd ,strip out one sine, convert the rest to = +c
cosines using the identity cos2 x + sin2 x =
1 and then use the substitution , u = cos x
Type 2: Integrals in which m is even and n is
.
odd.
Likewise, if the exponent on the cosines (n) is
In the case when m is even and n is odd, use the
odd strip out one cosine and convert the rest to
identity cos2 A = 1 − sin2 A and the substitution
sines and the use the substitution u = sin x
u = sin x.
.
Worked Examples Worked Examples
1. Find ∫ sin3 x cos2 x dx. 1. Find ∫ sin 4 x cos 3x dx

Solution Solution
∫ sin3 x cos2 x dx. ∫ sin4 x cos3 x dx
Rewrite the term sin3 x as sin x sin 2 x, and use ∫ sin4 x (cos2 x ·cos x) dx.
the identity sin2 x = 1 − cos2 x.
Using the identity cos2 x = 1− sin2 x
The reason for doing this will become apparent. ∫ sin4 x (cos2 x cos x) dx
∫ sin3 x cos2 x dx = ∫ sin4 x (1 − sin2 x) cos x dx
= ∫ (sin x· sin2 x) cos2 x dx = ∫ (sin4 x cos x − sin6 x cos x) dx
= ∫ sin x (1 − cos2 x) cos2 x dx

Baffour Ba Series, Further Mathematics for Schools Page 651


Then the substitution u = sin x, du = cos x dx . ∫ = ∫ . cos ( / dx
(sin4 x cos x − sin6 x cos x) dx = ∫(u4 − u6) du
= ∫(u4 − u6) du = ∫ . sin ( /+c

= − +c = sin ( ) +c

= − +c
Exercises 21.9
A. Find the following integrals.
2. Find the integral ∫ dx 1. ∫ cos3 x dx
2. ∫ cos5 x dx
Solution 3. ∫ sin5 x cos2 x dx.
∫ dx
=∫ dx B. 1. Evaluate ∫ sin2 x cos2 x dx by using the
=∫ ( ) dx double angle formulae
sin2 x = , cos2 x =
Let u = sin x
=∫ ( ) du 2. Using the double angle formulae twice find
= ∫( ) du ∫ sin4 x cos2 x dx.
= sin x – sin x + c
Kinematics
Type 3 : Integrals in which m and n are even A. Motion in a Straight Line
In the case when both m and n are even, try Under differentiation, when the displacement s
using the double angle formulae: is differentiated the velocity v, is obtained.
When velocity v, is futher differentiated, the
cos2 x = (1 + cos 2x);
acceleration a , is obtained. That is:
sin2 x = (1 – cos 2x); If s denotes the displacement (position) of a
particle from a fixed point, at a time t, then;
Worked Examples 1. Velocity , v = , the rate of change of
1. Evaluate ∫ sin 2 x cos 2 x dx
position with respect to time.
Solution
∫ sin 2 x cos 2 x dx 2. Acceleration , a = =
= ∫ (1 − cos 2x) (1 + cos 2x) dx
Since integration is the reverses of
= ∫ (1 − (2x) dx differentiation, we integrate acceleration a, to
obtain the velocity v, and v is further integrated
Using the half – angle formular for cos2 to obtain displacement s.
That is: cos2 2x = (1 + cos 2x)
If a particle moving in a straight line with
= ∫ (1 − (1 + cos (4x) dx acceleration a, is a function of t , then:

Baffour Ba Series, Further Mathematics for Schools Page 652


1. a = f(t) By substitution,
2. v = ∫ a dt, when v = 0, the particle is at rest 0 = 03 + 02 + c
and v < 0 (v is negative), then the particle is c=0
moving in the opposite direction. v = t3 + t2
3. s = ∫ v dt
When t = 3,
From differentiation, v = 33 + 32
1. a = v = 27 + 9
a dt = dv v = 36 .
⇒ dv = a dt
2. A heavy truck start to travels fromToase to
Integrating both sides, Abuakwa with acceleration, a given by a
∫dv = ∫a dt = 3 + 2t. Find:
v = ∫a dt i. how long it takes the truck to attain a speed
of 18 m/s.
2. v = ii. the distance it has covered within that time.

v dt = ds
Solution
⇒ ds = v dt
a = 3 + 2t
Integrating both sides, a= = 3 + 2t
∫ds = ∫v dt = 3 + 2t
s = ∫v dt
dv = (3 + 2t) dt
v = ∫( ) dt
Worked Examples
1. A particle P is moving along a staright line v = 3t + +c
2
from a fixed point O on it. The acceleration of v = 3t + t + c
P after t seconds is given by 3t2 + 2t .
Find an expression for the velocity of P at any Before the truck starts from rest;
time t and find the velocity after 3 seconds. t = 0, and v = 0
⇒ 0 = 3(0) + (0)2 + c
Solution c=0
a= = 3t2 + 2t
2 v = 3t + t2
dv = (3t + 2t) dt
At v = 18,
v = ∫ (3t2 + 2t)
18 = 3t + t2
v= + +c t2 + 3t – 18 = 0
v = t3 + t2 + c (t – 3) (t + 6) = 0
t = 3 or t = - 6 (ignore – ve answer)
At rest, v = 0 and t = 0; t = 3s
Baffour Ba Series, Further Mathematics for Schools Page 653
ii. v = = 3t + t2 48t where S is measured in meters and t in
seconds.
= 3t + t2 i. Find the velocity of the particle at time t.
ds = (3t + t2) dt ii. The particle stops moving twice, once when
ds = ∫ (3t + t2) dt time A and again when t = B, where A< B.
s= + +c Determine A and B.
iii. What is the position of the particle at time
12s?
when t = 0, s = 0
( ) ( ) iv. What is the total distance the particle travels
0= + +c between time 0 and time 12?
c=0
4. A particle moves along a staright line such
s= +
that its displacement at any time is given by
S = t3 – 6t2 + 3t + 4. Find the velocity when the
At t = 3
( ) ( )
acceleration is zero.
s= +
s= + B. Motion under Constant Acceleration
Consider a body moving under a constant
s = 22.5m
acceleration, a. If the initial velocity is given
Exercises 21.10 as u, the velocity after time t is v and the
1. A particle moves along a straight line so that displacement from the initial point after time t
at the end of t second the its acceleration a m/s2 is s. Then
is give by the relation a = t + 2. If the particle a=
started from rest , find its velocity at the end of a dt = dv
3 seconds and the distance covered during the dv = a dt
next second. v = ∫ a dt
v = at + c
2. A train runs between two consecutive
stations with a velocity given by . / At t = 0;
where t seconds is the time after passing v=c
through the first station. The velocity is Let u = v = c
measured in kilometer per second. Find: Then v = at + u
i. the time taken between the two stations. v = u + at ……………..(1)
ii. the maximum velocity attained.
Also,
iii. the distance between the two stations.
v=
3. A particle moves along a straight line and the v dt = ds
position at time t is given by S = 2t3 – 18t2 + ds = v dt

Baffour Ba Series, Further Mathematics for Schools Page 654


s = ∫ v dt But t =
s = ∫ (u + at ) dt ⇒ 2s = t(v + u)
s = ut + at2 + c 2s = t(u + v)
When t = 0 and s = 0; s = (u + v)………………..(4)
c=0
⇒ s = ut + at2 ……………..(2) Note:
For motion under constant acceleration, a = g
From eqn (1) and the value of g is approximately 9.8m/s2
v = u + at
t= Worked Examples
1. A boy rides on a bicycle downhill with a
constant acceleration and increases his speed
Put t = in eqn 2;
from 6m/s to 9m/s while travelling 150m. How
s = u. /+ a. /2 long does it takes and what is his acceleration?
s = (v – u) + ( )
Solution
( ) ( )
s= + Initial velocity u = 6m/s
Final velocity v = 9m/s
Factorize out Time t = ?

s=. /0 . /1 Distance S = 150m


s=. /0 1 s = (u + v)
s=. /0 1 150 = (6 + 9)
( )( ) 150 × 2 = t(6 + 9)
s= (difference of two squares)
300 = 15t
s= t = 20 s
2as = v2 – u2
2as + u2 = v2 Now, u = 6 m/s, v = 9 m/s, t = 20 s and a = ?
v2 = u2 + 2as………………(3) v = u + at
a= (make a the subject)
Again, from equation (3):
a=
v2 = u2 + 2as
v2 – u2 = 2as a= m/s2
= 2s
Alternatively, v2 = u2 + 2as can be used
2s =
( )( ) 2. A stone is dropped from the window of a
2s =
storey building and it reaches the ground with a
Baffour Ba Series, Further Mathematics for Schools Page 655
speed of 14 m/s2. If the acceleration g is 4. A stone is projected vertically upwards from
9.8m/s2, find: a height 5m above the ground. It takes 3.2s for
i. the height above the ground of the point from it to hit the ground. Calculate the average speed
which the stone is dropped. and the average velocity of the stone.
ii. the time taken for the stone to hit the
ground. 5. A car travels in a straight road under
constant acceleration. In the first 20 seconds,
Solution the distance travelled is 125 metres and the
i. u = 0, v = 14, a = g = 9.8 , s = ? displacement is 100m east. Find the
From v2 = u2 + 2as acceleration and initial velocity of the car.
s=
Area under a Curve
( ) ( )
s= ( )
s = 10m

ii. u = 0, v = 14, a = g = 9.8, s = 10, t = ?


From v = u + at
t=
t=
When calculating the area under a curve f(x),
t= seconds follow the steps below:
1. Sketch the area.
Exercises 21.11 2. Determine the boundaries a and b,
1. A stone is projected vertically upwards from 3. Set up the definite integral,
a height 5m above the ground. It takes 3.2s for 4. Integrate.
it to return to the point of projection. Find the
total distance covered. A. Area between a curve and the x – axis
The area between a curve and the x-axis is the
2. A car travels in a straight line under a definite integral of the function of the curve
constant acceleration. In the 5th second, the car within the given range of x.
tarvels 15m, and in the 6th second, it travels y
12m. Find the speed of the car at t = 0. f(x)

3. A racing van increased its speed to 30m/s


after a displacement of 80 m in 4 seconds.
Calculate its acceleration and the change in its x
0 a b
velocity during the 4 s period.

Baffour Ba Series, Further Mathematics for Schools Page 656


For instance, in the above diagram, the area A, If the curve cuts the x – axis between the limits,
of the region bounded by the curve y = f(x), the then:
x – axis and the lines x = a, and x = b is given 1. find the areas to the left and right the y – axis
by: separately;
Area = ∫ dx ii. add these two values together.
y
This is positive if the area is above the x – axis b
and negative if the area is below the x – axis.
A>0

If the curve cuts the x – axis between the limits,


then: c
A<0
1. find the areas above and below the x – axis
a
separately;
x
ii. add these two values together.

y
Worked Examples
1. Find the area under the curve, f(x) = x2, for
-1 ≤ x ≤ 2.
A>0
x
a b A<0 c

B. Area between a curve and the y – axis


The area A, of the region bounded by the curve
y = f(x), the y – axis and the lines y = a, and y =
b is given by: Area = ∫ dy Solution
y Area = ∫ dx
b
A = 0 1
a ( )
=0 1– 0 1
x = –
= 3 square units
In this case, the y must be expressed as a
function of y before integration. This is positive 2. Find the area bounded by the curve y = x2 –
if the area is to the right of the y – axis and 2x – 8, the x – axis and the line x = 1 to x = 5.
negative if the area is to the left of the y – axis.

Baffour Ba Series, Further Mathematics for Schools Page 657


Solution A = A1 + A2
y = x2 – 2x – 8, A = 18 +
At the x = axis, y = 0
A= square units
x2 – 2x – 8 = 0
(x – 4) (x + 2) = 0
x = 4 or x = -2 3. Find the area bounded by the following
curves: y = x2 – 4, y = 0 and x = 4
A1
Solution
1 4 5
A1 y = x2 – 4, y = 0 and x = 4
When x = 0, y = - 4

The required area is: When y = 0 , finding the boundaries:


A = A1 + A2 (x – 2) (x + 2) = 0
x = 2 or x = - 2
A1 = 1 ≤ x ≤ 4 , the curve below the x- axis
From the graph it is seen that x = 2 is our
A2 = 4 ≤ x ≤ 5 , the curve above the x- axis boundary at a. The value x = -2 is a solution
to the equation above but it is not bounding the
area.
A1 = ∫ ( – 8) dx

A1 = 0 8 1
( ) ( ) ( ) ( ) ab
A1 = 0 8( )1 – 0 8( )1
-4
A1 = 0 1–0 81

A1 = –. / Set up the integral:

A1 = - 18 (area must be positive)


A=∫ dx
A1 = 18 square units A=∫ ( ) dx

A=0 1
A2 = ∫ ( – 8) dx
( ) ( )
A=0 ( )1 – 0 ( )1
A2 = 0 8 1
( ) ( ) ( ) ( ) A=0 1 –0 81
A2 = 0 8( )1 – 0 8( )1
A= 8
A2 = 0 1–0 1
 square units.
A2 = –. /
A2 = square units 3. Find the under the curve, f(x) = x(x – 2) (x +
2), for -1 ≤ x ≤ 1.

Baffour Ba Series, Further Mathematics for Schools Page 658


The total area under the curve;
A = A1 + A2
= +
= sq. units

4. Find the area of the combined region


bounded by the curve y = x3 – 5x2 + 6x

Solution
y = x3 – 5x2 + 6x
Solution
Solving for the x – coordinates of the
To avoid the error, we have to integrate it in
intersection points of the curve and the x- axis
two parts:
x3 – 5x2 + 6x = 0
from x = -1 to x = 0 and x = 0 to x = 1.
x(x2 – 5x + 6) = 0
Area of the left part A1 x(x – 2) (x – 3) = 0
x = 0, or x = 2 or x = 3
=∫ x(x – 2)(x + 2) dx
x1 = 0, or x2 = 2 or x3 = 3
=∫ x3 – 4x dx

=0 1
y = x3 – 5x2 + 6x
=0 1
( ) ( )
=0 ( ) 1–0 ( ) 1 A1

= [0 – . /] 0 2 A2 3
=
The required area is:
Area of the right part A2 ; A = A1 + A2
= ∫ x(x – 2)(x + 2) dx A1 = 0 ≤ x ≤ 2 , the curve above the x- axis
= ∫ x3 – 4x dx A2 = 2 ≤ x ≤ 3 , the curve below the x- axis

=0 1 A1 = ∫ ( – ) dx

=0 1 A1 = 0 1
( ) ( ) ( ) ( ) ( )
=0 ( ) 1–0 ( ) 1 A1 = 0 1-, -
= A1 =
Since area cannot be negative, the value is A2 = ∫ ( – ) dx

Baffour Ba Series, Further Mathematics for Schools Page 659


A2 = 0 1 B =∫ ( )

A2 = 0
( ) ( ) ( )
1–0
( ) ( ) ( )
1 B =0 1

A2 = . / B=0 1
A2 = ( ) ( )
B=0 ( ) ( ) 1 – 0 ( ) ( ) 1
B= square units
A = A1 + A2
A= + B = square units (ignore –ve sign)

A= square units
A = A+ B
A= +
5. Calculate the areas of the segments
contained between the x-axis and the curve y = A = square units
x(x − 1)(x − 2).‟
4. Find the area between the curve y = x(x − 3)
Solution and the ordinates x = 0 and x = 5.
y = x(x − 1)(x − 2)
Solution
On the x – axis, y = 0
x(x − 3) = 0,
x = 0 or x = 3.
The curve cuts the x-axis at x = 0 and x = 3.
A
0 1 B 2 When x = 0, y = 0
When x = 3, y = 0
Area A, is given by the integral from x = 0 to x
= 1 of the curve y = x(x − 1)(x − 2) = x3 − 3x2 + Intercepts = (0, 0) and (0, 3)
2x; The x2 term is positive, and so the curve forms
A=∫ a U-shape as shown below. y = x(x − 3)
A =∫ ( ) A=∫
A =∫ ( )
A=0 1
A=0 1
A=0 1
( ) ( ) ( ) ( )
( ) ( ) A=0 1–0 1
A=0 ( ) ( ) 1–0 ( ) ( ) 1
A=. /–0
A1 = square units
A=

Baffour Ba Series, Further Mathematics for Schools Page 660


A= square units y x2 + x + 4 = 0,

For this quadratic;


B=∫ y = x(x − 3) b2 − 4ac
B =∫ ( ) = 1 − 16
B
x = −15 ( there are no real roots).
B=0 1 A 3 5 This means that the curve does not cross the x-
B=0
( ) ( )
1–0
( ) ( )
1 axis. The curve is U-shaped (coefficient of x2 is
positive)
B=. /
B= square units When x = 0, y = 4

Total area = A + B
y = x2 + x + 4
Total area = +
Total area = square units
A
5. Find the area of the region that is bounded 1 3
by the curve y = √ , the line x = 1 and the x –
axis. A=∫
A =∫ ( )
Solution
y=√ , y=√ , A=0 1

A=∫ dx ( ) ( ) ( ) ( )
A=0 ( )1 - 0 ( )1
⁄ A
A= [ ⁄
] A=0 1–0 1
⁄ 1 A= square units
A=
( ) ⁄ ( ) ⁄
A= 0 1–0 1 C. Area between two curves
1. Find the point of intersection of the two
A= 0 1 – , -
curves.
A = square units 2. Subtract the areas under the curves between
the points of intersection.
6. Find the area bounded by the curve y = x2+ x
+ 4, the x-axis and the ordinates x = 1 and x = 3. This can be done by the evaluation of one
integral using the x – coordinates of the point
Solution of intersection as the limits.
y = 0 to obtain the quadratic equation:

Baffour Ba Series, Further Mathematics for Schools Page 661


Worked Examples Area A3 under the curve between x = 2 and x = 3
1. Find the area of the blue region, between the A3 = ∫ x2 - 2x dx
curve, y = x(x - 2) and the line, y = x.
A3 = 0 1
( ) ( )
A3 = 0 ( ) 1–0 ( ) 1
A3 = , - – 0 1
A3 =

First, find the point of intersection of the curve So, the area of the shaded region;
and the line. A = A1 - A2 + A3
A= – +
At the point of intersection,
x(x – 2) = x A= square units
x2 – 2x –x = 0
x2 – 3x = 0 2. Find the area of the region bounded by the
x(x – 3) = 0 parabola y = x2 and the line y = x + 2.
x = 0 or x = 3
Solution
Area below the x-axis A1; y = x2 and y = x + 2
Solve for the x – coordinates point of
A1 = ∫ (x2 – 2x) dx
intersection of the curve and the x – axis.
A1 = 0 1 x2 = x + 2
x2 - x + 2 = 0
A1 = 0 1 (x – 2) (x + 1) = 0
A1 = 0 1 x = 2 or x = -1
Now, let x1 = -1 and x2 = 2
A1 = y=x+2
A1 =

Area A2 under the line between x = 0 and x = 3 A


A2 = ∫ x dx y = x2x

A2 = 0 1 -1 2

A2 = 0 1 The region under consideration lies below the


line y = x + 2 and the curve y = x2, and between
A2 =
the vertical lines x = -1 and x = 2

Baffour Ba Series, Further Mathematics for Schools Page 662


Area of the region; =∫
A = ∫ ,( ) -dx
=∫ ( )
A=0 1 =∫ ( )
( ) ( ) ( ) ( )
A =0 ( ) 1–0 ( 1 =0 1
A=. /–. / =0
( ) ( )
1–0
( ) ( )
1
A= –. / =0 1
A = square units = square units

3. Calculate the area of the segment cut from The area under the straight line is
the curve y = x(3 − x) by the line y = x.
=∫
Solution =∫

=0 1
y=x
P
( ) ( )
=0 1–0 1
=, -
y = x(3 – x)
= 2 square units

The shaded area is;


0 2 A= –
At P, the y co-ordinates of both curves are A= –2
equal. Hence: A=
x(3 − x) = x

3x − x2 = x Area of a region bounded by the horizontal


2x − x2 = 0 lines and the y – axis.
x(2 − x) = 0 In certain problems it is easier to rewrite the
x = 0, the origin, or else x = 2, the x co-ordinate function in terms of y and calculate the area
of the point P. using horizontal elements. In this case the
From the diagram, the shaded region is the area formula for the area would be: A= ∫ ( ) dy
under the curve of y = x(3 − x), between the x-
axis and the ordinates x = 0 and x = 2 minus the When calculating the area under a curve, or in
area under the, the line y = x, and between the this case to the left of the curve g(y), follow
x-axis and the ordinates x = 0 and x = 2.
the steps below:
1. Sketch the area.
The area under the curve is;

Baffour Ba Series, Further Mathematics for Schools Page 663


2. Determine the boundaries c and d, ⁄
A=∫ ( ) du
3. Set up the definite integral,

4. Integrate. A=[ ]

Worked Examples ⁄
A=0 1
1. Find the first quadrant area bounded by the
following curves: y = x2 + 2 , y = 4 and x = 0 . ( ) ⁄ ( ) ⁄
A=0 1–0 1
Solution A = 1.8856 square units
A=∫ ( ) dy
2
y = x + 2 , y = 4 and x = 0 . Area of a Rectangle
y
Determine the boundary values of c and d.
On the y – axis, x = 0 y=a
a
y = (0)2 + 2
y=2
0 b x
The boundary values of c = 2 and d = 4
From y = x2 + 2, express x in trems of y The equation of the line is y = a
x2 = y – 2
So, the area under the line that forms the
x= √ (ignore negative in the first quadrant)
rectangle = ∫ a dx
d
Area = , -
A Area = ab
Area = length × width
c
Area of a Triangle
x= √ y

h
A = ∫ (√ ) dy y = mx

A=∫ ( ) dy
0 b x
Using u – substitution;
Let u = y – 2 The equation of the line is y = mx, where m is
=1 the gradient.
So, the area under the line that forms the
dy = du
triangle = ∫ mx dx

Baffour Ba Series, Further Mathematics for Schools Page 664


Area = [ ]0b Exercises 21.12
1. Find the area enclosed by the given curve,
( )
Area = - the x-axis, and the given ordinates.
a. The curve y = x, from x = 1 to x = 3.
Area =
b. The curve y = x2 + 3x, from x = 1 to x = 3
Since m, the gradient, m = c. The curve y = x2 − 4 from x = −2 to x = 2
d. The curve y = x − x2 from x = 0 to x = 2
Area = ×
Area = h × b 2. Find the area contained by the curve y = x(x
Area = × height × base − 1)(x + 1) and the x-axis.

3. Calculate the value of ∫ ( )( ) dx


Area of a Trapezoid
y
4. Calculate the value of ∫ ( )dx.
Explain your answer.
y = mx + a
b

a B. 1. Find the area contained between the line


y = x and the curve y = x2. Ans
0 h x
2. Find the area contained between the two
The equation of the line is y = mx + a where m
curves y = 3x − x2 and y = x + x2. Ans
is the gradient and a is the y-intercept.

3. Find the area of the region enclosed by the


Area under the line that forms the trapezoid;
following curves: and y = x2, y = x + 6 , x = 0,
A=∫ dx x = 5 Ans square units
A= 0 1
4. Find the area of the region enclosed by the
A= following curves:
A= i. f(x) = 2x + 1, a = 0, b = 2
ii. f(x) = x2 – 4 , a = 2, b = 4
A = [mh + 2a]
( ) iii. f(x) = 3 – x2 , a = -1, b = 1
A= 0 1
iv. f(x) = 2x - x2 , a=0, b=2
A = [b - a + 2a]
A = [b + a] C. 1. The diagram opposite shows the graph of
Area = [sum of the two parallel sides] y = x2 – 5x.

Baffour Ba Series, Further Mathematics for Schools Page 665


y = 2x2 – 18.

P Q

a. Calculate the coordinates of P and Q.


Calculate the shaded area. b. Find the shaded area.

2. The diagram shows the graph of y = 4x – x2. 5. The diagram shows part of the graph of
y = 3 – 3x2.
y = 4x – x2.
y = 3 – 3x2

Calculate the area between the curve and the x- Calculate the shaded area.
axis.
6. The diagram shows the graph of
3. The diagram shows part of the graph y = x2 – 2x – 12.
of y = 6x + 2x2.

y = x2 – 2x – 12.
2
y = 6x + 2x

A 1 4

a. Find the coordinates of A.


b. Calculate the shaded area. Calculate the shaded area.

4. The dagram shows part of the graph of 7. The diagram shows part of the graph of
y = 2x2 – 18. y = 10 – 4x – 3x2

Baffour Ba Series, Further Mathematics for Schools Page 666


Calculate the shaded area.
a. Find the coordinates of A and B.
b. Calculate the shaded area.
8. The diagram shows the graph of
y = 8 – 2x – x2.
11. The diagram shows the graph of
y = x3 – 4x2 – 7x + 10.

a. Find the coordinates of A and B.


b. Calculate the shaded area.
a. Find the coordinates of A and B.
b. Calculate the shaded area.
9. The diagram opposite shows part of the
graph of y = x3 – 3x2 + 2x.
12. The diagram shows the graph of
y = x4 – 5x2 + 4.

a. Find the coordinates of P and Q. Calculate the shaded area.


b. Calculate the shaded area.
Challenge Problems
10. The diagram shows the graph of Find the area of the region bounded by the
y = x3 – 3x2 – 10x. given curves. Decide whether to integrate
with respect to x or y.

Baffour Ba Series, Further Mathematics for Schools Page 667


1. y = 3x2 , y = 0, and x = 3 2. Determine the volume of the solid obtained
2. y2 = x , y = 2 - x , x = 0 and x = 0 by rotating the region bounded by y = x2 – 4x +
5, x = 1, x = 4 and the x – axis about the x –
Volumes of Revolution axis .
If an area is rotated about the x or y – axis , the
three dimensional object formed is called a Solution
“solid of revolution” and its volume is a y = x2 – 4x + 5, a = 1 and b = 4
“volume of revolution” Volume = ∫ dx
Volume = ∫ ( ) dx
Rotation about the x-axis
Integration can be used to find the area of a Volume = ∫ ( – 8x3 + 26x2 – 40x + 25) dx
region bounded by a curve whose equation is
known. For example, to find the area under the = 0 1
curve y = x2 between x = 0 and x = 5, for
= 0 – 1
example, simply integrate x2 with limits 0 and 5.
= . /
2
Now imagine that a curve, for example y = x ,
=
is rotated around the x-axis so that a solid is
formed. The volume of the shape that is formed =
can be found using the formula:
Volume = ∫ dx Rotation about the y-axis
If the body is rotated about the y-axis rather
Worked Examples than the x-axis, then we use the formula:
1. The graph of y = x2 between x = 1 and x = 3 Volume = ∫ dy
is rotated completely about the x – axis. Find
the volume generated. Worked Examples
Solution 1. Find the volume of a solid generated by
y = x2 , a = 1 and b = 3 rotating about the y – axis the area bounded by
Volume = ∫ dx the y – axis , the line y = 2x + 2 and the lines
y = 4 and y = 7.
Volume = ∫ ( ) dx (substitute y = x2)
Volume = ∫ dx Solution
y = 2x + 2, a = 4 , b = 7
=0 1
Volume = ∫ dy
( ) ( )
=0 1–0 1
From y = 2x + 2,
= – 2x = y – 2
= x=

Baffour Ba Series, Further Mathematics for Schools Page 668


y-axis. Find the volume of the solid of
x2 = . /
revolution formed.

(y – 2)2 = y2 – 4y + 4 4. The area between the curve y = x2, the y-axis


22 = 4 and the lines y = 0 and y = 2 is rotated about the
y-axis. Find the volume of the solid of
x2 = . / revolution formed.

x2 =
5. Find the volume of the solid generated by
2 2
x = (y – 4y + 4) rotating about the y – axis the area bounded by
the y – axis, the line 2x – y – 2 = 0 and the lines
Volume = ∫ dy y = 2 and y = 6.

Volume = ∫ ( – ) dy
6. A conical funnel is formed by rotating the
= ∫ ( – ) dy curve y = about the y-axis. The radius of the

= 0 1 rim of the funnel is to 6 cm. Find the depth of


the funnel and its volume.
= 0 1
7. Sketch the curve y2 = x(x − 4)2 and find the
= 0. 8 8/ – . /1 volume of the solid of revolution formed when
= . / the closed loop of the curve is rotated about the
x-axis.
= ( )
= B. Find the volume of the solid of revolution
generated when the area described is rotated
about the x-axis.
Exercises 21.13
1. The area between the curve y = x and the
A. 1. The equation of the line k is 3x + 2y – 6 =
ordinates x = 0 and x = 4.
0. Find the volume of the cone generated by
2. The area between the curve y = x3/2 and the
rotating the areas bounded by k and the axes
ordinates x = 1 and x = 3.
about:
3. The area between the curve x2 + y2 = 16 and
i. the x – axis ii. the y – axis
the ordinates x = −1 and x = 1.
4. The area between the curve x2 − y2 = 9 and
2. The area cut off by the x-axis and the curve
the ordinates x = − 4 and x = −3.
y = x2 − 3x is rotated about the x-axis. Find the
5. The area between the curve y = (2 + x)2 and
volume of the solid of revolution formed.
the ordinates x = 0 and x = 1.

3. The area between the curve y = , the y-axis The Trapezium Rule
and the lines y = 1 and y = 2 is rotated about the To find the area under a curve,

Baffour Ba Series, Further Mathematics for Schools Page 669


1. Split the space into very thin strips Worked Examples
2. Look and consider them individually to 1. Find the value of ∫ dx by the trapezium
realize that each strip is approximately the
rule using 8 strips.
same shape as a trapezium,
3. Use the trapezium formula , A = (a + b) h Solution
estimate the area under the curve. A=∫ dx, n = 8, h = = 0.25
4. To obtain a more accurate approximation,
divide the interval a ≤ x ≤ b into n equal parts Table of values
each of length h, where h =
x y=
Consider the figure below; 2 y0 = 0.5000
y 2.25 y1 = 0.4444
2.5 y2 = 0.4000
2.75 y3 = 0.3636
3 y4 = 0.3333
yn – 2 yn – 1 yn 3.25 y5 = 0.3077
y0 y1 y2
3.5 y6 = 0.2857
a b x 3.75 y7 = 0.2667
4 y8 = 0.2500
If we looked at the total area of the trapeziums,
you would get: Write out the formula.
A= (yfirst + y1)h + (y1 + y2)h + (y2 + y3)h + (y4 A = × h (yfirst + ylast + 2(sum of the rest )
+ ylast)h
yfirst + ylast = 0.5000 + 0.2500
Factorise the common factors, to obtain = 0.7500
A = × h (yfirst + y1 + y1 + y2 + y2 + y3 + y3 + y4 + y4 (sum of the rest )
+ ylast = 0.4444 + 0.4000 + 0.3636 + 0.3333 + 0.3077
+ 0.2857 + 0.2667
This can s further be simplified to obtain the = 2.4014
formula:
A = × h (yfirst + ylast + 2(y1 + y2 + y3 + y4 ) Substitute in
A = × h (yfirst + ylast + 2(sum of the rest )
This formula can be made more general for any
A = × (0.25) (0.7500 + 2(2.4014)
amount of trapeziums.
A = × (0.25) (0.7500 + 4.8028)
A = × h (yfirst + ylast + 2(sum of the rest )
A = × (0.25) (5.5528)
A = 0.6941 square units (4 decimal places)
This is called the trapezium rule:

Baffour Ba Series, Further Mathematics for Schools Page 670


2. Given that y = f(x) and for the given values 3.6 10.6480
of x , the corresponding y values are shown in 3.8 17.5760
the table below. Use trapezium rule to find the 4 27.0000
approximate value of ∫ dx
A1 = ∫ ( – ) dx, n = 5, h = = 0.2
x 1 2 3 4 5 A1 = × h (yfirst + ylast + 2(sum of the rest )
y 0 4 5 7 4
yfirst + ylast = 1.0000 + 27.0000
Solution = 28.0000
A=∫ dx, n = 4, , h = =1
(sum of the rest )
A = × h (yfirst + ylast + 2(sum of the rest )
= 2.7440 + 5.8320 + 10.6480 + 17.5760
yfirst + ylast = 0 + 4 = 4
= 36.80

(sum of the rest )


A1 = × (0.2) (28.00 + 2 (36.80)
= 4 + 5 + 7 = 16
A1 = × (0.2) (28.00 + 73.60)
Substitute in
A1 = × (0.2) (101.60)
A = × h (yfirst + ylast + 2(sum of the rest )
A1 = 10.16 (2 decimal places)
A = × (1) (4 + 2(16)
y= =( )
( )
A = × (1) (4 + 32)
A = × 36 x y=( )
A = 18 square units 3 1.0000
3.2 0.3644
3.4 0.1715
3. Use the trapezium rule with ordinates at x = 3.6 0.0939
3, 3.2, 3.4, 3.6, 3.8, and 4.0 to find, correct to 3.8 0.0569
two decimal places, the finite area enclosed by 4 0.0370
the curves: y = (2x – 5)3, y = and the
( )
lines x = 3 and x = 4. A1 = ∫ ( ) dx, n = 5, h = = 0.2

Solution A1 = × h (yfirst + ylast + 2(sum of the rest )


y = (2x – 5)3
yfirst + ylast = 1.0000 + 0.0370
x y = (2x – 5)3 = 1.0370
3 1.0000
3.2 2.7440 (sum of the rest )
3.4 5.8320 = 3.6440 + 0.1715 + 0.0939 + 0.0569 = 0.6867

Baffour Ba Series, Further Mathematics for Schools Page 671


A2 = × (0.2) (1.0370 + 2 (0.6867) A = × (1) (0 + 70)
A2 = × (0.2) (1.0370 + 1.3734) A = × (70)
A2 = × (0.2) (2.4104) A = 35 square units
A2 = 0.24104
4. Use trapezium rule to calculate the integral
A2 = 0.24 (2 decimal places)
∫ √ ) dx with seven ordinates.
A = A1 – A2
A = 10.16 – 0.24 Solution
A = 9.92 square units (2 decimal places) A = ∫ (√ ) dx, n = 6, h = = 0.3

4. Use the trapezium rule to estimate the area


x y=√
cut – off between the parabola y = 6x – x2 and
1 1.4142
the x – axis. 1.3 1.6401
1.6 1.8868
Solution 1.9 2.1471
y = 6x – x2 2.3 2.5080
At the x – axis , y = 0 2.6 2.7857
6x – x2 = 0 2.9 3.0676
x (6 – x ) = 0
x = 0 or 6 – x = 0 A = × h (yfirst + ylast + 2(sum of the rest )
x = 0 or x = 6
A=∫ ( – ) dx, n = 6, h = =1 yfirst + ylast = 1.4142 + 3.0676
= 4.4818
x 0 1 2 3 4 5 6 (sum of the rest )
y 0 5 8 9 8 5 0 = 1.6401 +1.8868 + 2.1471 + 2.5080 + 2.7857
= 10.9677

A=∫ ( – ) dx, n = 6, h = =1
A = × (0.3) (4.4818 + 2 (10.9677)
A = × h (yfirst + ylast + 2(sum of the rest )
A = × (0.3) (4.4818 + 21.9354)

yfirst + ylast = 0 + 0 = 0 A = × (0.3) (26.4171)


A = 3.9005 square units
(sum of the rest )
= 5 + 8 + 9 + 8 + 5 = 35 Exercises 21.14
A. 1. Find the approximate value for the
A = × (1) (0 + 2 (35) definite integral ∫ dx using the trapezium
ruke with 5 ordinates.

Baffour Ba Series, Further Mathematics for Schools Page 672


2. Use trapezium rule to estimate the area of 2. a. Draw a curve from the data below, using a
the quarter – circle bounded by the x – axis, the scale of 1 cm to 1 unit on both axes:
y – axis and the curve x2 + y2 = 25 using
ordinats at x = 0, 1, 2, 3, 4, 5 x 1 3 4 5 7 9 10
y 6.2 5.4 5.2 5.4 6 6.8 6.8
3. Use the trapezium rule to estimate the area
under the curve given by : b. Draw the ordinates at x = 1, 2, 3…10 so
dividing the area under the curve into 9 strips.
x 0 4 8 12 16 20 Use trapezium rule to estimate the area under
y 0 16 25 20 24 12 the curve.

4. Use the trapezium rule to estimate the area 3. a. Draw the graph of y = 4x2 from x = 0 to x
unser the curve given by : = 5, using a scale of 2cm to 1 unit on the x –
axis and 1cm to 10 units on the y – axis.
x 0 5 10 15 20 25 30 b. Draw ordinates at x = 1, 2, 3, 4, 5. Use
y 6 12 18 15 30 25 10 trapezium rule to estimate the area under the
curve.
5. Copy and complete the table below for the
parabola y = 4x – x2 Challenge Problems
1. a. Copy and complete the table below, giving
x 0 0.5 1 1.5 2 2.5 3 3.5 4 values for √( ) to 3 decimal places.
y 0 1.8 3.0
X 0 0.5 1 1.5 2 2.5 3
Use trapezium rule to estimate the area cut off
Y 1.414 1.957 3.000
between parabola abd the x – axis.
y
B. 1. A curve passes through the points given
by the following table:

x 0 2 4 6 8 10
y 4 4 3.5 4 5 6.5 R

0 3 x
a. Construct the curve on a graph sheet taking a
scale of 1 cm to 1 unit on each axis. Shade the The figure above shows the region R , bounded
area enclosed by the curve, the x – axis and the by the curve with equation
two end ordinates. y = √( ), the x – axis and the line x = 0
b. Estimate the area by the trapezium rule, and x = 3. Use trapezium rule with all the
taking 5 strips. values from your table to find an
approximation for the area of R

Baffour Ba Series, Further Mathematics for Schools Page 673


2. a. Given y = √( ) . copy and complete y = x√( ) 0 ≤ x ≤ 2. Copy and complete
the table below to three decimal places. the table below given the values of y to 3
decimal places at x = 1 and x = 1.5.
x 0 0.5 1 1.5 2
y 2.464 3.630 x 0 0.5 1 1.5 2
y 0 0.530 6
b. Use trapezium rule, with all the values from
your table to find an approximation for the Use the trapezium ruke with all the y values
from your table to find an approximation for
value of ∫ √( ) dx
∫ √( ) dx giving your answer to 3
3. The curve c has the equation; decimal places.

Baffour Ba Series, Further Mathematics for Schools Page 674


22 PERMUTATION AND COMBINATION Baffour Ba Series

Permutations (Arrangements) Cynthia has 5 tops, 6 skirts and 4 caps from


The act of arranging a set of objects in a which to choose an outfit. In how many ways
particular order is called permutation. For can she select one top, one skirt and one cap?
example, using only two different objects , A
and B, there are two different arrangements or Solution
permutation; AB, BA. For three different objects 5 tops, 6 skirts and 4 caps
A, B and C, the arrangements are ABC, ACB, a = 5, b = 6 and c = 4
BAC, BCA, CAB, CBA. Here, there a choice of Ways = 5 × 6 × 4
3 letters for the first place, then there is a choice = 120 ways
of 2 letters for the second place and there is
only 1 choice for the third place. Thus, the three Repetition of an Event
operartions can be peformed in 3 × 2 × 1 = 6 If one event with a outcomes occurs b times
ways with repetitions allowed, then the number of
ordered arrangements is ab.
Now, out of four or more different objects,
listing all the permutations could be a bit Worked Examples
tedious. Thus, the fundamental counting 1. Find the number of arrangements if a die is
principle for calculating the number of rolled;
permutation is used. a. 2 times b . 4 times c. r times
For four different objects ABCD;
= (4)(3)(2)(1) Solution
= 24 Outcomes on a die = 6
a. 2 times = 62 = 6 × 6 = 36
For five different objects ABCDE; b. 4 times = 64 = 6 × 6 × 6 × 6 = 1,296
= (5) (4)(3)(2)(1) c. r times =
= 120
2. a. How many car number plates are possible
In an arrangement or permutation, the order of with three letters followed by 3 digits?
the object chosen is important. b. How many of these number plates begin with
ABC?
The Multiplication Rule
If one event can occur in a ways, a second Solution
event in b ways and a third event in c ways, a. Number of English alphabets = 26
then the three events can occur in a × b × c. Number of digits ranges from 0 – 9
= 10 numbers
Worked Examples
Car number plates possible with three letters

Baffour Ba Series, Further Mathematics for Schools Page 675


= and followed by 3 digits = ; Rule 2 : Arrangements with Repetitions
= × The number of distinct permutation of n objects
= 26 × 26 × 26 × 10 × 10 × 10 when x are alike, y are another kind that are
= 17576 × 1000 alike, z are alike of another type…… then the
= 17,576,000 number of ordered selection or permutation is
given by;
b. ×
= 1 × 1 × 1 × 10 × 10 × 10
Worked Examples
= 1,000
1. Find the number of permutation of the letters
of the word “KAAK”
Rule 1: Arrangements without Repetitions
In general, the number of permutation of n
Solution
distinct objects taken all together is, n!
In the word “KAAK”, number of word = 4
Number of permutation = 4!
Worked Examples
K = 2! A = 2!
1. Using factorial notation, express the number ( )( )( )( )
of arrangements of the letters in the following Number of permutation = =( )( )( )( )
=6
words:
a. Maths b. Length c. Objects 2. Find the number of permutation of the letters
of the word “ILLINOIS”
Solution
a. Maths = 5! Solution
b. Length = 6! In “ILLINOIS”, number of words = 8
c. Objects = 7! Number of permutations = 8!
I = 3! L = 2!
2. In how many ways can 6 people be arranged Number of permutation;
in a row? ( )( )( )( )( )( )( )( )
= = ( )( )( )( )( )
= 3,360

Solution
3. How many different arrangement of the word
Number of ways = (6) (5) (4) (3) (2) (1)
PARRAMATTA are possible?
= 6!
= 720 ways
Solution
PARRAMATTA = 10 letters
Exercise 22.1
Number of permutation = 10!
Using factorial notation, express the number
A = 4! R = 2! T=2
of arrangements of the letters in the
following words: Number of permutation;
1. HISTOGRAM 2. RHOMBUS
=
3. FACTORS 4. CARGO
Baffour Ba Series, Further Mathematics for Schools Page 676
( )( )( )( )( )( )( )( )( )( ) 1. The first position (President) is being
= ( )( )( )( )( )( )( )( )
( )( )( )( )( )( )
contested by 6 candidates.
= ( )( )( )( ) 2. If one is elected president, then there is 5
candidates to contest for the vice president.
=
3. After the vice – president is elected; there are
= 37,800
four candidates to contest for the secretary.
4. After the secretary is elected, there are three
Exercises 22.2
people to contest for the treasure.
A. Find the number of different permutation
of the letters of the word
The number of ways of filling these four
1. MATHEMATICS
positions can be described as:
2. STATISTICS
(6) (5) (4) (3) = 360 ways.
3. ALABAMA
The notation 6P4 is used to denote the
4. HIPOPOTAMUS
permutation of 6 objects taken 4 at a time.
5. PARALLELOGRAM
6. ERROR
To evaluate 6P4,
1. Find 6!
B. How many distinguishable arrangements
2. Then divide by the part of 6! that were not
can you contain from each of the following
counted, that is 2!
sets; ( )( )( )( )( )( )
1. {1, 8, 8, 6, } ⇒ 6P4 = = ( )( )
= 360 ways
2. (2, 2, 3, 5, 5, 9}
3. {4, 5, 6, 6, 7, 7, 7, 8, 9} Worked Examples
1. Find the number of ways to seat 3 people in
Rule : 3 a row of three chairs selected from a group of
The number of permutation of r objects 15 people.
chosen from n objects where 0 ≤ r ≤ n is :
n
Pr = ( Solution
)
n = 15 , r = 3
15
When n = r, P3 = ( = = (15) (14) (13) = 2,730
)
n
Pr = ( =( = = n! (0! = 1)
) )
2. Consider 8 participants in a contest with
four winning positions. In how many ways
Explanation
could there be four winning positions?
Take for instance, four positions, namely;
President, Vice – president, Secretary and
Solution
Treasure are to be contested among six
n = 8 and r = 4
candidates. Observe the following; 8
P4 = ( = = (8) (7) (6) (5) = 1,680
)

Baffour Ba Series, Further Mathematics for Schools Page 677


Therefore, the four winners could be arranged b. n = 4 and r = 2
in 1680 ways 4
P2 = ( = = (4) (3) = 12 ways
)

3. How many ordered choices of 3 objects can


Arrangements with Restrictions
be made from 10 distinct objects?
a. Number of permutation of „n‟ things taken
„r‟ at a time, when a particular thing is to be
Solution
always included in each arrangement is given
n = 10 and r = 3
10
by :
P3 = ( = = (10) (9) (8) = 720
)
Therefore, the number of ordered choices of 3 b. Number of permutation of „n‟ things taken
objects from 10 is 720 „r‟ at a time, when a particular thing is fixed is
given by : n – 1
4. a. How many ways can 6 people be arranged
in a row? c. Number of permutation of „n‟ things taken
b. How many arrangements are possible if only „r‟ at a time, when a particular thing is never
3 of them are chosen? taken is given by: n – 1

Solution d. Number of permutation of „n‟ things taken


a. Number of ways 6 people be arranged in a „r‟ at a time, when m specified things always
row = (6) (5) (4) (3) (2) (1) come together = m! × (n – m + 1) !
= 6!
= 720 e. Number of permutation of „n‟ things taken all
at a time, when m specified things always come
b. Number of arrangements possible if only 3 of together = n! – [ m! × (n – m + 1)!]
them are chosen;
6 Worked Examples
P3 = ( = = (6) (5) (4) = 120 ways
)
1. How many words can be formed with the
letters of the word „OMEGA‟ when;
5. An English debating team consist of 4
i. „O‟ and „A‟ occupying end places.
speakers.
ii. „E‟ being always in the middle.
a. How many ways can all 4 speakers be
iii. vowels occupying odd places.
arranged in a row for a photo?
iv. vowels never together.
b. How many ways can the captain and the
vice – captain be chosen?
Solution
i. When „O‟ and „A‟ occupying end places.
Solution
M, E, G, (OA)
a. n = 4 and r = 4
4
Here (OA) are fixed, hence M, E, G can be
P4 = ( = = 4! = 24 ways arranged in 3! ways.
)

Baffour Ba Series, Further Mathematics for Schools Page 678


But (O, A) can be arranged in 2! ways Solution
i. There are no restriction.
Total number of ways = 3! × 2! = 12 ways ENZYME = 6 letters
Number of arrangements = 6P6 = 6! = 720
ii. When „E‟ is fixed in the middle.
O, M, (E), G, A. ii. They begin with EN;
Hence, four letters can be arranged in 4! If each arrangement begins with EN, then the
= 24 ways remaining letters to be reshuffled = 4
EN _ _ _ _ = 4P4 = 4! = 24
iii. Three vowels can be arranged in the odd
places( 1st , 3rd and 5th ) = 3! ways iii. They do not begin with EN;
Total – the arrangements that begin with EN
Two consonants (M, G) can be arranged in the = 6! - 4!
even place (2nd and 4th ) = 2! = 696

Total number of ways = 3! × 2! = 12 ways 3. How many arrangement of the letters of the
word ENZYME are possible if;
iv. Total number of letters = 5! = 120 i. they have EN together in order?
If all the vowels come together, then we have: ii. they have ENZ together in any order?
(O, E, A), M, G. These can be arranged in 3! iii. E, Z and M are not to be together?
ways
But (O, E, A) can be arranged in 3! ways. Solution
i. If they have EN together in order;
Number of ways when vowels come together; (EN)_ _ _ _ = 5P5 = 5! = 120
= 3! × 3!
= 36 ways ii. If they have ENZ together in any order;
(ENZ) _ _ _ = 4P4 or 3P3 = 3! × 4! = 144
Number of ways, when vowels being never
together; iii. E, Z and M are not to be together;
= 120 – 36 Total – ENZ together in any order
= 84 ways = 6! – 144
= 576
2. How many arrangements of the letters of the
4. There are six boys who enter a boat with 8
word ENZYME are possible if:
seats, 4 on each side. In how many ways can
i. there are no restriction.
i. they sit anywhere?
ii. they begin with EN?
ii. two boys A and B sits on the port side and
another boy C, sit on the starboard side.
iii. They do not begin with EN?

Baffour Ba Series, Further Mathematics for Schools Page 679


Solution iv. Abi and Joe wish to stay together.
i. The number of ways in which they can sit (AJ) _ _ _ _ _ _ _ or (JA) _ _ _ _ _ _ _
anywhere = 8P6 = = 8! + 8! = 2(8!) = 80640
ii. A and B sits on the port side = 4P2
C sits on the starboard side = 4P1 6. In how many ways can the letters of the word
The others will have an arrangement of 5P3 OPTICAL be arranged so that the vowels
Total = AB and C and others always come together.
Total = 4P2 × 4P1 × 5P3 A. 610 B. 720 C. 825 D. 920
Total = 12 × 4 × 60
= 2,880 Solution
OPTICAL = 7 letters
5. In how many ways can 5 boys and 4 girls be Vowels = OIA = 3 letters
arranged on a bench if; If these three vowels should always come
i. there are no restrictions. together, then they can be grouped and
ii. boys and girls alternate considered as a single letter. That is
iii. boys and girls are in separate groups PTCL(OIA)
iv. Abi and Joe wish to stay together.
PTCL(OIA) = 5 letters
Solution Number of ways of arranging 5 letters;
i. Total number of boys and girls; 5 ! = 120
= 5 boys + 4 girls = 9
Number of ways of arranging the vowels (OIA)
If there are no restrictions; = 3! = 3 × 2 × 1 = 6
n = 9 and r = 9
9
P9 = ( = = 9! Required number of ways;
)
= 5! × 3!
ii. Boys and Girls alternate; = 120 × 6
BGBGBGBGB = 720 different ways.
= 5P5 × 4P4
7. In how many different ways can the letters of
iii. boys and girls are in separate groups; the word “CORPORATION” be arranged so
Boys and girls in separate groups or girls and that the vowels always come together?
boys separate groups; A. 47200 B. 48000 C. 42000 D. 50400
Boys and girls or girls and boys;
= 5P5 × 4P4 + 4P4 × 5P5 Solution
= 5! × 4! + 4! × 5! “CORPORATION” = 11 letters
= 120 × 24 + 24 × 120 Vowels = OOAIO and these five vowels shoud
= 2,880 + 2,880 come together. Hence they can be grouped and
= 5,760 considered as a single letter, giving us

Baffour Ba Series, Further Mathematics for Schools Page 680


CRPRTN(OOAIO). Hence we assume the total Number of ways of arranging the 2 vowels
letters are 7. But in these 7 letters, “R” occurs taking one at a time
two times and the rest of the letters are 2
= = = 2 ways
( )
different.
Number of ways of arranging these letters;
Number of ways of arranging the 8 letters
7
= = 2520 taking 3 at a time
8
= = = 336 ways
( )
In the 5 vowels (OOAIO), “O” occurs 3 and the
rest of the vowels are different.
Required number of ways;
Number of ways of arranging the vowels
=2 ×8
= 20 = 2 × 336
= 673 ways
Hence, total number of ways;
= 2520 × 20 iii. The letters K, G, N, S = 4 letters
= 50,400 different ways Number of ways of arranging so that it ends
with any of the letters K, G, N, S arranging the
Solved Past Question four letters taking one at a time.
4
1. A pupil tried to arrange the letters of the =( = = 4ways
)
word DUCKLINGS; taking four letters at a
time. Find the number of ways of the
Number of ways of arranging the 8 letters
arrangement if :
taking 3 at a time
a. any of the letters can be used; 8
=( = = 336 ways
b. it must begin with a vowel; )
c. it must end with any of the letters K, N, G
and S. Required number of ways;
=4 ×8
Solution = 4 × 336
a. DUCKLINGS = 9 letters = 1,344 ways
If any of the letters can be used:
Number of ways of arranging 9 letters taking 4 Exercises 22.4
at a time 1. A, B, C, D, E and F are six students. In how
9 many ways can they be seated in a row if:
=( = = 3,024 ways
) i. there are no restrictions on the seating.
ii. A and B must sit beside each other;
b. If it must begin with a vowel: iii. A and B must not sit beside each other,
(UI) = 2 letters iv. D, E and F must sit beside each other,
(UI)DCKLNGS = 8 v. A and F must sit at the end of each row?

Baffour Ba Series, Further Mathematics for Schools Page 681


2. How many different arrangements can be 7. i. In how many ways can three girls and two
made from the letters PQRST if no letter can be boys be seated in a row of 5 seats?
repeated and taking: ii. In how many ways can this be done if the
i. five letters boys must sit together?
ii. four letters iii. In how many ways can this be done if the
iii. three letters boys must not sit together?
iv. two letters, at a time?
8. How many arrangements of the letters of the
3. Ten horses run a race. In how many ways word FORMULAS are possible if:
can the first , second and third places be filled if i. all letters are used in the arrangements.
there are no dead heats? ii. the three vowels must come together in the
arrangement?
4. How many arrangements can be made using iii. the three vowels must not all come together
all the arrangements of the word DUBLIN? in the arrangement?
i. How many arrangements begin with the letter
D? Application to Forming Digits
ii. How many arrangements begin with B and A number of digits can be formed from a given
end in L? set of digits with some conditions likely to be
iii. How many arrangements begin with a “repetition of digits” or “without repetition of
vowel? digits”
iv. How many arrangements end with LIN?
v. How many arrangements begin with D and Find possibilities for first position digit, second
end in LIN? position digit, third position digits etc, not
forgetting the conditions or restrictions.
5. Taking all the letters of the word ALGEBRA,
in how many arrangements are the two As Multiply possibilities for each first and
together? subsequent digits to obtain the required total
number of possibilities and the total digit
6. Six children can be seated in a row on a numbers that can be formed
bench.
i. How many arrangements are possible? With Repetitions of Digits
ii. How many arrangemets are possible if the Worked Examples
youngest child sit at the left-hand end and the 1. How many four digit numbers greater than
oldest child must sit at the right-hand end? 6,000 can be formed using the digits (1, 2, 4, 5,
iii. If two of the children are twins, in how 6, 7) if repetitions are allowed?
many ways can the children be arranged if:
a. the twins are together, Solution
b. the twins are not together? (1, 2, 4, 5, 6, 7)
ABCD > 6,000

Baffour Ba Series, Further Mathematics for Schools Page 682


Any one of the two digits (6, 7) can take the Worked Examples
first position = 2 possibilities 1. How many 5 – digit numbers can be formed
from the set {1, 2, 3, 4, 5}, if each digit is to be
Any one of the six digits (1, 2, 4, 5, 6, 7) can used once.
take the second position = 6 possibilities
Any one of the six digits (1, 2, 4, 5, 6, 7) can Solution
take the third position = 6 possibilities {1, 2, 3, 4, 5}
Each of the five digits can occupy the first
Any one of the six digits (1, 2, 4, 5, 6, 7) can position giving 5 possibilities
take the fourth position = 6 possibilities
Number of digits remaining is 4.
Number of four digit numbers greater than Each of the remaining four digits can occupy
6,000 = 2 × 6 × 6 × 6 the second position giving 4 possibilities
= 432
Number of digits remaining is 3.
2. How many 4 digit numbers greater than 4300 Each of the remaining three digits can occupy
can be formed from the digits (1, 2, 3, 4, the third position giving 3 possibilities
5, 6, 7) if repetition of the digits is allowed?
Number of digits remaining is 2.
Solution Each of the remaining two digits can occupy
Let the 4 digits be ABCD, the fourth position giving 2 possibilities
ABCD > 4300 Number of digits remaing is 1.
A ≥ 4 (4, 5, 6, 7) = 4 possibilities The remaining digits can occupy the fifth
position giving 1 possibility
B ≥ 3 (3, 4, 5, 6, 7) = 5 possibilities
C ≥ 1 (1, 2, 3, 4, 5, 6, 7) = 7 possibilities
Total five digit numbers that can be formed
D ≥ 1 (1, 2, 3, 4, 5, 6, 7) = 7 possibilities
= 5 × 4 × 3× 2 × 1
= 120 different five digit numbers
Total possibilities = 4 × 5 × 7 × 7
= 980 possibilities
Type 2 :
Here, the digits to be formed are limited to a
Without Repetition of Digits
given value.
The use of the the word “distinct” means
without repetition. In this case, make sure each
Worked Examples
digit is used once.
1. How many 4 digit numbers greater than
Type 1: 5,000 can be formed using the digits (3, 4, 5, 6,
Here, the digits to be formed are not limited to 7) without repetition of the digits?
any given value.

Baffour Ba Series, Further Mathematics for Schools Page 683


Solution Remaining digits = 3
(3, 4, 5, 6, 7) Any one of the remaining two digits of (1, 2, 4
Let the 4 digits be ABCD, 5) or one of the two of (6 ,7) can take the fourth
ABCD > 5,000 position = 2 + 1 = 3 possibilities
Any one of the three digits (5, 6, 7) can take the
first position = 3 possibilities Number of four digit numbers greater than
6,000 = 2 × 5 × 4 × 3 = 120
Remaining digits = 4
Any one of the four remaining digits can take Solved Past Questions
the second position = 4 possibilities 1. How many number can formed between
Remaining digits = 3 3,000 and 7,000 using digits 1, 3, 6, 8 without
Any one of the three remaining digits can take repetition?
the third position = 3 possibilities
Solution
Remaining digits = 2 1, 3, 6, 8
Any one of the two remaining digits can take Number formed must be between 3,000 and
the fourth position = 2 possibilities 7,000. This means that:
First position can be occupied by any one of (3
Total possibilities = 3 × 4 × 3 × 2 and 6) = 2 possibilities
= 72 possibilities Remaining digits = 3 digits

2. How many four digit numbers greater than Second position can be occupied by any one of
6,000 can be formed using the digits (1, 2, 4, 5, (3 or 6) or any one of (1, 8)
6, 7) if no digit can be repeated? = 3 possibilities
Remaining digits = 2 digits
Solution
ABCD > 6,000 Third position can be occupied by any one of (3
Any one of the two digits (6, 7) can take the or 6) or any one of (1, 8) = 2 possibilities
first position = 2 possibilities Remaining digits = 1 digits

Remaining digits = 5 Fourth position can be occupied by any one of


Any one of the four digits (1, 2, 4, 5) or one of (3 or 6) or any one of (1, 8) = 1 possibility
the two of (6, 7) can take the second position Remaining digits = 0 digits
4 + 1 = 5 possibilities
Thus the number of numbers that can be
Remaining digits = 4 formed between 3000 and 7000
Any one of the the remaining three digits of (1, =2×3×2×1
2, 4 5) or one of the two digits of (6 ,7) can take = 12 numbers
the third position = 3 + 1 = 4 possibilities

Baffour Ba Series, Further Mathematics for Schools Page 684


2. How many four digit numbers greater than 1, 2, 3, 4, 6, 7, 8
2,000 can be formed using the digits: 0, 1, 2, 3, 4. Let the 4 digits be ABCD,
ABCD < 5000
Solution Any one of the four digits (1, 2, 3, 4) can take
0, 1, 2, 3, 4. the first position = 4 possibilities
Without repetition means each digit will be
used ones. Since the number should b egreater Remaining digits = 7
than 2,000; Any one of the seven remaining digits can take
the second position = 7 possibilities
The first digit can take any one of the following Remaining digits = 7
(2, 3, or 4) = 3 possibilites Any one of the seven remaining digits can take
Remaining digits = 4 the third position = 7 possibilities

The second digit can take any one of the Remaining digits = 7
remaining 4 digits = 4 possibilites Any one of the seven remaining digits can take
Remaining digits = 3 the fourth position = 7 possibilities

The third digit can take any one of the Total possibilities = 4 × 7 × 7 × 7
remaining 3 digits = 3 possibilites = 1,372 possibilities
Remaining digits = 2
ii. No repetition allowed
The fourth digit can take any one of the
1, 2, 3, 4, 6, 7, 8
remaining 2 digits = 2 possibilites
Let the 4 digits be ABCD,
Remaining digits = 1
ABCD < 5000
Any one of the four digits (1, 2, 3, 4) can take
The four digit numbers greater than 2,000 that
the first position = 4 possibilities
can be formed = 3 × 4 × 3 × 2 = 72

Covering Repetition and without Repitition Remaining digits = 6


When restrictions or conditions as to whether Any one of the six remaining digits can take the
the digits are formed with or without repetitions second position = 6 possibilities
are not given, then it is advisable to cover both
paths. Remaining digits = 5
Any one of the five remaining digits can take
Worked Examples the third position = 5 possibilities
1. How many 4 digit numbers less than 5,000
can be formed from 1, 2, 3, 4, 6, 7 and 8? Remaining digits = 4
Any one of the four remaining digits can take
Solution the fourth position = 4 possibilities
i. Repetition allowed

Baffour Ba Series, Further Mathematics for Schools Page 685


Total possibilities = 4 × 6 × 5 × 4 Remaining digits = 3
= 480 possibilities Any one of the three remaining digits can take
the third position = 3 possibilities
2. Use the set {1, 2, 3, 4, 5} to answer the
following questions: Remaining digits = 2
i. How many distinct five digit numbers can Any one of the two remaining digits can take
you form? the fourth position = 2 possibilities
ii. How many distinct four digit numbers can
you form? Remaining digits = 1
iii. How many distinct three digit numbers can The remaining digit can take the fifth position
you form? = 1 possibilities
iv. How many five digit numbers are greater
than 20,000? Total possibilities = 4 × 4 × 3 × 2 × 1
v. How many five digit numbers are less than = 96 possibilities
3,000?
ii. Repetition allowed
Solution 1, 2, 3, 4, 5
{1, 2, 3, 4, 5} Let the 4 digits be ABCDE,
i. 5 × 4 × 3 × 2 × 1 ABCDE > 20,000
= 120 different five digit numbers. Any one of the four digits (2, 3, 4, 5) can take
the first position = 4 possibilities
ii. 5 × 4 × 3 × 2 × 1
= 120 different five digit numbers. Remaining digits = 5
Any one of the five remaining digits can take
iii. 5 × 4 × 3 the second position = 5 possibilities
= 60 different five digit numbers.
Remaining digits = 5
iv. five digit numbers greater than 20,000 Any one of the five remaining digits can take
{1, 2, 3, 4, 5} the third position = 5 possibilities

i. No repetition allowed Remaining digits = 5


1, 2, 3, 4, 5 Any one of the five remaining digits can take
Let the digits be ABCDE, the fourth position = 5 possibilities
ABCDE >20000
Any one of the four digits (2, 3, 4, 5) can take Remaining digits = 5
the first position = 4 possibilities Any one of the five remaining digits can take
Remaining digits = 4 the fifth position = 5 possibilities
Any one of the four remaining digits can take
the second position = 4 possibilities Total possibilities = 4 × 5 × 5 × 5 × 5

Baffour Ba Series, Further Mathematics for Schools Page 686


= 2,500 possibilities Any one of the five remaining digits can take
the third position = 5 possibilities
iv. Five digit numbers are less than 3,000
ABCD < 3,000 Remaining digits = 5
Any one of the five remaining digits can take
i. No repetition allowed the fourth position = 5 possibilities
1, 2, 3, 4, 5
Let the digits be ABCDE, Total possibilities = 4 × 5 × 5 × 5
ABCDE < 3,000 = 500 possibilities
Any one of the two digits (1, 2) can take the
first position = 2 possibilities Forming Even, Odd and Multiples of Numbers
1. To form an even number, ensure the last
Remaining digits = 4 (unit) digit of the number is an even number.
Any one of the four remaining digits can take 2. To form an odd numbers, ensure the unit
the second position = 4 possibilities (last) digit of the number is an odd number.
3. To form mutiples of a number, ensure the
Remaining digits = 3 last (unit) digit of the number is a multiple of
Any one of the three remaining digits can take that number. For example, for all multiples of
the third position = 3 possibilities 5, the last digit must be 0 or 5. Likewise, for
multiples of 2, the last digit must be 0 or 2.
Remaining digits = 2
Any one of the two remaining digits can take It is always advisable to check the multiples
the fourth position = 2 possibilities being formed so as to choose a correct last or
units digit. Here, it important to fill in the last
Total possibilities = 4 × 4 × 3 × 2 digit first.
= 96 possibilities
Worked Examples
ii. Repetition allowed 1. How many different five digit numbers can
1, 2, 3, 4, 5 be formed from the digits 1, 2, 3, 4 and 5 if:
Let the 4 digits be ABCD, i. the number is odd and no repetitions are
ABCDE < 3,000 allowed;
Any one of the two digits (1, 2) can take the ii. the number is even and repetitions are
first position = 2 possibilities allowed;

Remaining digits = 5 Solution


Any one of the five remaining digits can take i. 1, 2, 3, 4 and 5
the second position = 5 possibilities Odd and no repetitions allowed;
Any one of the two digits (1, 3 , 5) can take the
Remaining digits = 5 last position = 3 possibilities

Baffour Ba Series, Further Mathematics for Schools Page 687


Remaining digits = 4 2. From the digits 2, 3, 4, 5, 6
Any one of the four remaining digits can take a. How many digits greater than 4,000 can be
the first position = 4 possibilities formed without repetition?
b. How many four digit numbers would be
Remaining digits = 3 even, without repetitions?
Any one of the three remaining digits can take
the second position = 3 possibilities Solution
2, 3, 4, 5, 6
Remaining digits = 2 a. ABCD > 4,000
Any one of the two remaining digits can take Any one of the three digits (4, 5, 6) can take the
the third position = 2 possibilities first position = 3 possibilities
Remaining digit = 1
The one remaining digits can take the fourth Remaining digits = 4
position = 1 possibilities Any one of the four remaining digits can take
the second position = 4 possibilities
Total possibilities = 4 × 3 × 2 × 1 × 3
= 24 possibilities
Remaining digits = 3
Any one of the three remaining digits can take
ii. Even and repetitions are allowed;
the third position = 3 possibilities
Any one of the two digits (2, 4) can take the
last position = 2 possibilities
Remaining digits = 2
Any one of the two remaining digits can take
Remaining digits = 5
the fourth position = 2 possibilities
Any one of the five remaining digits can take
the first position = 5 possibilities
Total possibilities = 3 × 4 × 3 × 2
Remaining digits = 5 = 24 possibilities
Any one of the five remaining digits can take
the first position = 5 possibilities b. Four digit numbers that would be even;
Even ends with 2, 4, 6 = 3 digits
Remaining digits = 5
Any one of the five remaining digits can take ii. Even without repetitions;
the first position = 5 possibilities Digits = 2, 3, 4, 5, 6
Any one of the two digits (2, 4, 6) can take the
Remaining digits = 5 last position = 3 possibilities
Any one of the five remaining digits can take
the first position = 5 possibilities Exercises 22.5
1. How many three different three digit
Total possibilities = 5 × 5 × 5 × 5 × 2 numbers can be formed using the digits 0, 1, 2,
= 1,250 possibilities 3, 4 if 0 cannot be the first digit and if;

Baffour Ba Series, Further Mathematics for Schools Page 688


i. no digit may be repeated; i. If one of the digit must be 7, how many
ii. repitions are allowed different numbers can be formed?
ii. if the digits 2 and 5 cannot be used together,
2. How many four digit numbers can be formed how many different numbers can be formed?
from the digits 1, 2, 3, 5, 6 and 8 if :
i. there are no restrictions and repetitions are Circular Arrangements
allowed; Circular arrangements are permutations in
ii. the number is odd and no repetitions are which objects are arranged in a circle.
allowed;
iii. the number is divisible by 5 and repetitions Consider arranging 4 persons; A, B, C, D
are allowed; around a circular table.
iv. the number is greater than 5,000, divisible A
by 5 and no repetitions are allowed. ABCD
D B BCDA
3. How many numbers between 2,000 and CDAB
4,000 can be made with the digits1, 2, 3, 4 if no DABC
C
digit may be repeated?
Shifting ABCD one position in anti – clockwise
4. How many numbers between 100 and 1,000 direction we obtain the following
can be made with the digits 1, 2, 3, 4, 5 and 6 if B A D
no digit may be repeated? C A B D A C
i. repetitions are allowed
D C B
ii. repetitions are not allowed.
If 4 persons are sitting at a round table, then
they can be shifted four times, but these four
5. i. How many different numbers each with 3 – arrangements will be the same, because the
digits or fewer n be formed from the digits 2, 3, sequence ABCD is the same.
4, 5, 6? Each digit can be used once in each
number. Linear Arrangements;
ii. How many of the above numbers are odd? If ABCD are sitting in a row, and they are
shifted, then the four linear aranegemnts will be
6. How many odd numbers between 4,000 and different as shown below;
6,000 can be made from the digits 3, 4, 5, 6, 7 ABCD, DABC, CDAB, BCDA
and 8, if no digit may be repeated?
There are two cases of circular permutations
7. How many three – digit numbers can be a. If clockwise and anti – clockwise orders are
formed using the digit 0 to 9 inclusive, if no different, then total number of circular
digit can be used more than once and 0 cannot permutations is given by (n – 1) !
be the first digit?

Baffour Ba Series, Further Mathematics for Schools Page 689


Proof a. If clockwise and anticlockwise orders are
If ABCD are sitting in a row, and they are taken as different, then total number of circular
shifted, then the four linear arrangements will permutations =
be different as shown below;
b. If clockwise and anticlockwise orders are
ABCD, DABC, CDAB, BCDA
taken as not different, then total number of
Hence, if we have four things, then for each
circular permutations =
circular arrangement number of linear
arrangement = 4.
Likewise, if we have n things, then for each Worked examples
circular agreement, number of linear 1. At a dinner party, 6 men and 6 women sit at
arrangement = n a round table. In how many ways can they sit if:
a. there are no restrictions
Let the total circular arrangement = p b. men and women alternate
Total number of linear arrangement = np
⇒ Total number of linear arrangement Solution
= n (number of circular arrangement) a. there are no restrictions
6 men + 6 women = 12
Number of circular arrangement n = 12
= 1 (number of linear arrangement)
Ways = (n – 1) !
n=
Ways = (12 – 1) !
Circular permutation = (n – 1) !
b. men and women alternate;
b. If clockwise and anti – clockwise orders sre Arbitrarily fix one man and alternate the rest of
taken as not different, then total number of the group
( – )
circular permutations is given by (6 – 1)! × 6!
= 5! × 6 !
Proof = 120 × 720
When clockwise and anti – clockwise = 86400
arrangements are not different, then
observations can be made from both sides , and 2. How many necklace of 12 beads each can be
this will be the same. Here two permutations made from 18 beads of different colors?
will be counted as one . So total permutations
Solution
will be half, hence in this case;
Here clockwise and anti - clockwise
( – )
Circular permutations = arrangements are the same.

Note Total number of circular permutations:


Number of circular permutation of n different = = = =
( )
things, taken r at a time:

Baffour Ba Series, Further Mathematics for Schools Page 690


Combination Use results 3 if r is greater than .
A combination is a selection of a number of
For example, ( ) = ( ) =( )
objects in any order.

In making a selection of a number of elements Explanation of ( ) = ( )


from a given sets, only the contents of the Assuming there are 13 soccer players from
group selected are important, not the order in which only 11 players must be picked to play.
which the items are selected. For example, AB The number of ways of choosing 11 from 13 is
and BA represent the same selection. However, given by ( )
AB and BA represent different arrangements. ( )= = 78

The ( ) Notation On the other hand, when ever 11 is chosen to


( ) Gives the number of ways of choosing r play, 2 is chosen not to play. Thus:
objects from n different objects. Its value can ( ) =( )= = 78
be calculated in two different ways.
Notice that 11 + 2 = 13
1. ( ) (definition)
( )
( )( ) ( ) Similarly ( ) = ( ) as 17 + 3 = 20 and
2. ( ) (in practice)
( ) =( ) as 98 + 2 = 100
Both give the same result; however, the second
is easier to use in practical questions. If r is large, the calculator may not be able to
For example: calculate. Thus use ( ) = ( )
1. ( ) = = = = 15
( )
( ) = is sometimes written as nCr or nCr
,
2. ( ) = ⇒ ,
= 15 In general :
Number of combination of n different things
Memory Aid: taken r at a time is given by :
n
( )= = Cr =
( ) ( ) ( )

( ) is pronounced as “n-c-r” or “n-choose-r”


Note
1. Use permutation if a problem calls for the
Other Results
number of arrangements of objects and
1. ( ) = 1
different orders are to be counted.
2. ( ) = 1 2. Use combinations if a problem calls for the
3. ( ) = ( ) number of ways of selecting objects and the
order of selection is not to be counted.

Baffour Ba Series, Further Mathematics for Schools Page 691


Some Important Results taken r at a time , when p particular things are
Let n and r be positive integers such that r ≤ n. always to be excluded.
Then; = n - p Cr
(i) nCr = nC n – r
(ii) nCr + nC r – 1 = n + 1Cr Worked Examples
(iii) n n – 1 C r – 1 = (n – r + 1) nC r – 1 1. i. In how many ways can a committee of 4
people be chosen from a panel of 10 people?
Worked Examples ii. If a certain person must be on the committee,
1. Ten people take part in a chess competition. in how many ways can the commitee be
How many games will be played if every chosen.
person must play each of the other. iii. If a cetain person must not be on the
committee, how many ways can the committee
Solution be chosen?
n = 10 people, r = 2 ( two play each game)
Number of games ; Solution
( )= = 45 i. n = 10, r = 4
( )= = 210
2. In a class, there are 27 boys and 14 girls. The Therefore, from a panel of 10, 210 different
teacher wants to select 1 boy and 1 girl to committees of 4 people can be formed.
represent the class for a function. In how many
ways can the teacher make this selection? ii. One particular person must be on the
committee.
Solution n = 9 r = 3 ( one person chosen already)
Here the teacher is to perform two operations: ( )= = 84
i. Selecting a boy from among the 27 boys and
Therefore, from a panel of 10 people, 84
ii. Selecting a girl from among 14 girls.
different committee of 4 people can be formed
The first of these can be done in 27 ways and if one particular person of the 10 must be on
second can be performed in 14 ways. By the every committee.
fundamental principle of counting, the required
number of ways is 27 × 14 = 378 ways iii. one particular person must not be on the
committee;
Restricted Combination n = 9 (one person cannot be chosen)
a. Number of combination of n different things r = 4 (4 people required on the committee)
taken r at a time , when p particular things are ( )= = 126
always included Therefore, from a panel of 10 people, 126
= n - p Cr – p different committee of 4 people can be formed
if one particular person of the 10 must not be
b. Number of combination of n different things on the committee.
Baffour Ba Series, Further Mathematics for Schools Page 692
2. A committee of 5 persons is to be formed Number of ways of selecting any doctor;
from 5 men and 4 women. Find the number of = 7 C3
ways of forming the committee if: Number of ways of selecting any nurse;
a. there must be only two men. = 5 C2
b. one particular woman must be included.
Number of ways of selecting any doctor and
Solution any nurse ;
Number of men = 5 = 7C3 × 5C2
Number of women = 4 = 35 × 10
n=9 = 350 ways
Number of ways of selecting only 3 women
= 4 C3 ii. One particular doctor must be on the
committee.
Number of ways of selecting only 2 men This means one doctor is already chosen, so
= 5 C2 remaining 6 doctors.
Number of ways of selecting 2 doctors from 6
Number of ways of forming the committee; doctors = 6C2
= Only 2 men + Only 3 men
= 4 C3 × 5 C2 Number of ways of selecting 2 nurses from 5
= 4 × 10 nurses = 5C2
= 40 ways
One particular doctor must be on the committee
b. If one particular woman is included;
= 6C2 × 5C2
(one person is already chosen)
= 15 × 10
n=8,r=3
= 150 ways
= 8 C3
= 56 ways
4. How many ways can a student answer 8 out
of 10 questions, if he must answer the first five
3. A committee of 3 doctors and 2 nurses is to
questions.
be formed from 7 doctors and 5 nurses. In how
many ways can this be done if :
Solution
i. any doctor and any nurse can be included;
Number of ways of answering the first five
ii. One particular doctor must be on the
questions = 5C5
committee.
Number of ways of answering 3 out of the
Solution
remaining five questions = 5C3
i. Number of doctors = 7
Number of nurses = 5
Number of ways a student can answer 8 out of
Total n = 12
10 questions, if he must answer the first five

Baffour Ba Series, Further Mathematics for Schools Page 693


questions = 5C5 × 5C3 5. A student is to answer 10 out of 13 questions
= 1 × 10 in an examination. In how many ways can this
= 10 ways be done if he must answer :
a. the first two questions;
4. A student is to answer 6 out of 8 questions in b. the first or the second question but not both;
an examination. How many choices has he if: c. exactly three of the first five questions;
a. he must answer the first two questions d. at least three of the first five questions.
b. he must answer at least three of the first four
questions. Solution
a. Total number of questions = 13
Solution Number of ways of answering the first two
a. Number of ways of answering the first two questions = 2C2
questions = 2C2 Remaining questions = 11

Number of ways of answering 4 questions out Number of ways of answering 8 out of 11


of the remaining 6 questions = 6C4 questions = 11C8

Number of choices if he must answer the first Number of ways = 2C2 × 11C8
two questions = 2C2 × 6C4 = 1 × 165
= 1 × 15 = 165
= 15 choices
b. Number of ways of answering the first
b. To answer at least three of the first four question (neglecting the second question)
questions, the possible combinations are: = 1 C1
(3 from first four questions and three from the Remaining questions = 11
remaining four) or (4 from first four questions
Number of ways of answering 9 out of 11
and two from the remaining four )
question = 11C9
(3 from first four questions and three from the
remaining four) = (4C3 × 4C3) Number of ways of answering the first question
(neglecting the second question) and 9 out of
(4 from first four questions and two from the 11 question = 1C1 × 11C9
remaining four) = (4C4 × 4C2) = 1 × 55 = 55

Total number of choices; Number of ways of answering the second


= (4C3 × 4C3) + (4C4 × 4C2) (or means “ +”) question (neglecting the first question) = 1C1
= (4 × 4) + (1 × 6) Remaining questions = 11
= 16 + 6
= 22 choices Number of ways of answering 9 out of 11
question = 11C9
Baffour Ba Series, Further Mathematics for Schools Page 694
Number of ways of answering the second Solution
question (neglecting the first question) and 9 Number of ways of selecting 3 consonants from
out of 11 question = 1C1 × 9C8 = 1 × 55 = 55 7 = 7 C3
Number of ways of answering 10 out of 13 if Number of ways of selecting 2 vowels from 4
he must answer the first or the second question = 4 C2
but not both = 55 + 55 (or means “+”)
= 110 Number of ways of selecting 3 consonants from
7 and 2 vowels from 4 = 7C3 × 4C2 = 210
c. Number of ways of answering exactly 3 out This means we can have 210 groups where
of the first 5 questions = 5C3 each group contains a total of 5 letters (3
Remaining = 8 questions (last) consonants and 2 vowels)

Number of ways of answering 7 out of the 8 Number of ways of arranging 5 letters among
questions = 8C7 themselves = 5! = 5 × 4 × 3 × 2 × 1 = 120
Hence required number of ways = 210 × 120
Number of ways of answering exactly 3 out of = 25200

the first 5 questions and 7 out of the remaining 2. In a group of 6 boys and 4 girls, four children
8 questions = 5C3 × 8C7 are to be selected. In how many different ways
= 10 × 8 can they be selected such that at least one boy
= 80 should be there?
A. 159 B. 209 C. 201 D. 212
d. The possible combinations for answering
atleast three of the first five questions are : Solution
(3 from first five, 7 from remaining 8) OR Four children are to be selected, such that at
(4 from first five, 6 from remaining 8) OR least one boy should be there?
(5 from first five, 5 from remaining 8) We have 4 options listed below:
Therefore, number of ways; 1. Selecting 4 boys = 6C4
= (5C3 × 8C7) + (5C4 × 8C6) + (5C5 × 8C5) 2. Selecting 3 boys and 1 girl = 6C3 × 4C1
= ( 10 × 8) + ( 5 × 28) + (1 × 56) 3. Selecting 2 boys and 2 girls = 6C2 × 4C2
= 80 + 140 + 56 4. Selecting 1 boy and 3 girls = 6C1 × 4C3
= 276
Total number of ways
Combination Solved Examples = 6C4 + 6C3 × 4C1 + 6C2 × 4C2 + 6C1 × 4C3
1. Out of 7 consonants and 4 vowels, how many = 15 + 80 + 90 + 24
words of 3 consonants and 2 vowels can be = 209
formed?
A. 24400 B . 21300 C. 210 D. 25200 3. From a group of 7 men and 6 women, five
persons are to be selected to form a committee

Baffour Ba Series, Further Mathematics for Schools Page 695


so that at least 3 men are there in the in 4P3 ways 3 consonents can be arranged in 3
committee. In how many ways can it be done? ways. Hence, the required number of ways
A. 624 B. 702 C. 756 D. 812 = 3! × 4P3 = 144.

Solution 6. Ten different letters of alphabet are given.


We have 4 options listed below: Words with five letters are formed from these
1. Selecting 5 men = 7C5 given letters. Then the number of words which
2. Selecting 4 men and 1 woman = 7C4 × 6C1 have atleast one letter repeated is:
3. Selecting 3 men and 2 woman = 7C3 × 6C2 A. 69760 B. 30240 C. 99748 D. 99784

Total number of ways; Solution


= 7C5 + (7C4 × 6C1) + (7C3 × 6C2) Number of 5 letters words (with the condition
= 21 + 210 + 525 that a letter can be repeated) = 105. Again
= 756 number of words using 5 different letters is
10
P5.
4. In how many ways a committee consisting of Therefore, required number of letters
3 men and 2 women, can be chosen from 7 men = Total number of words – Total number of
and 5 women? words in which no letter is repeated
A 45 B. 350 C. 4200 D. 230 = 105 – 10P5 = 69760. (A) is correct choice.

Solution 7. The number of signals that can be sent by 6


(B) is the correct choice. Out of 7 men, 3 men flags of different colours taking one or more at
can be chosen in 7C3 ways and out of 5 women, a time is
2 women can be chosen in 5C2 ways. Hence, the A. 63 B. 1956 C. 720 D. 21
committee can be chosen in
7
C3 × 5C2 = 350 ways. Solution
The correct answer is B.
5. All the letters of the word „EAMCOT‟ are Number of signals using one flag = 6P1
arranged in different possible ways. The =6
number of such arrangements in which no two Number of signals using two flags = 6P2
vowels are adjacent to each other is = 30
A. 360 B. 144 C. 72 D. 54 Number of signals using three flags = 6P3
= 120
Solution Number of signals using four flags = 6P4
(B) is the correct choice. We note that there are = 360
3 consonants and 3 vowels E, A and O. Since Number of signals using five flags = 6P5
no two vowels have to be together, the possible = 720
choice for vowels are the places marked as „X‟. Number of signals using all six flags = 6P6
X M X C X T X, these volwels can be arranged = 720

Baffour Ba Series, Further Mathematics for Schools Page 696


Therefore, the total number of signals using one iii. how many teams can now be selected, if the
or more flags at a time is: captain must still be on every team?
= 6 + 30 + 120 + 360 + 720 + 720
= 1956 (Using addition principle). 3. In how many ways can a party of 6 children
can be chosen from a a group of 10 children if :
8. In an examination there are three multiple i. any child may be selected?
choice questions and each question has 4 ii. the oldest child must not be selected?
choices. Number of ways in which a student iii. the youngest child must be selected?
can fail to get all answer correct is iv. the youngest child and the oldest child must
A. 11 B. 12 C. 27 D. 63 both be selected?

Solution 4. A fifth year student has to choose 4 subjects


The correct choice is (D). There are three from the following list: Accounting, Biology,
multiple choice question, each Chemistry, Physics, French, Applied Maths and
has four possible answers. Therefore, the total Classical studies.
number of possible answers will be i. How many different choices are possible?
4 × 4 × 4 = 64. Out of these possible answer ii. How many choices include French?
only one will be correct and hence the iii. How many choices do not include French?
number of ways in which a student can fail to iv. How many choices include Accounting and
get correct answer is 64 – 1 = 63. Biology?
v. How many choices include Applied Maths
Exercises 22.7 but not Chemistry?
1. In how many ways can a committee of 4
people be chosen from 7 people?
5. Three delegates to form a committee are to
be selected from 8 members of a club. How
2. There are 15 pupils in a class. How many
many different committees can be formed if:
teams of 11 can be selected from the class?
i. three are no restrictions?
i. If one person in the class is made captain, and
ii. a certain member must be on each
must always be included in each team,
committee?
ii. how many teams can now be selected if 2
iii. two particular members cannot both be on
puplis in the class refuse to play,
the committee?

Baffour Ba Series, Further Mathematics for Schools Page 697


Special Types
Worked Examples
1. i. In how many ways can a group of 5 be selected from 9 people?
ii. How many groups can be selected if two particular people from the nine cannot be in the same
group?

Solution
i. n = 9, r = 5
( )= = 126

ii. If two particular people cannot be included, then calculate the number of ways of selecting 5
people with this particular two always included. That is 7 to chose 3 (because two are already
selected)
n = 7, r = 3
( )= = 35

( ,= : ;+( ,

= 126 – 35
= 91

2. How many ways may 10 people be divided into three groups of 5, 3 and 2 people?

Solution
The first group of 5 can be selected in ( ) = 252 ways
After the first selection, 5 people are left.

The second group of 3 can be selected in ( ) = 10 ways


After the second selection, 2 people are left.

The third group of 2 can be selected in ( ) = 10 ways

The total number of ways 10 people can be divided into groups of 5, 3 and 2;
=( )×( )×( )
= 252 × 10 × 1
= 2,520

Baffour Ba Series, Further Mathematics for Schools Page 698


3. Four letters are selected from the word SECTIONAL.
i. How many different selections are possible?
ii. How many of these selections contain at least one vowel?

Solution
i. n = 9, r = 4
( ) = = 126

ii. Vowels = E, I, O, A Consonants = S, C, T, N, L


„At least one vowel‟ means, one vowel, two vowels, three vowels or four vowels.
It is easier to calculate the number of selection containing no vowels (four consonats) and subtract
this from the number of ways of selecting four letters without any restrictions.

Number of selections containing no vowels = Number of selection containing four consonants


=( ) = =5
(There are 5 consonats and we want to select 4, thus n = 5, r = 4)
In every other selection, there must be at least one vowel.
Thus the number of selections containing at least one vowel ;
= (total number of selections) – (number of selections containing no vowel)
= 126 – 5
= 121

Choosing Objects from Two Different Groups


This involves choosing a number of objects from one group and then choosing a number of objects
from another group.

There are two key words when applying the fundamental principle of counting
1. „AND‟ is understood to mean „multiply‟. Thus, and = (×)
2. „OR‟ is understood to mean „add‟. Thus, or = (+)

Worked Examples
There are 5 women and 4 men in a club. A team of 4 has to be chosen. How many different teams
can be chosen if there must be either exactly one woman or exactly two women in the team .

Solution
AND means “ ×” OR means “ +”
5 women and 4 men. These are always the upper numbers in the combination bracket. A team must
consit of 4 people. Thus, exactly one womwn on the team means „1 woman and 3 men; and exactly

Baffour Ba Series, Further Mathematics for Schools Page 699


two women on the team means „2 women and 2 men‟. Thus, the choice is 1 woman and 3 men or 2
women and 2 men.

Let W stand for women and M stand for men

1 W and 3 M or 2W and 2M (lower numbers in each case)


↓ ↓ ↓ ↓ ↓ ↓ ↓
( ) × ( ) + ( ) × ( )
= 5 × 4 + 10 × 6
= 20 + 60
= 80
Thus, 80 teams can have either one woman or two men on the team.

2. How many bundles of 5 different books can be nade from 8 maths books and 6 physics books, if
the number of maths books must always be greater than the number of physics books?

Solution
We have 8 maths books and 6 physics books and these are always the upper numbers in the
combination bracket. A bundle must consist of 5 books. We need to nave more maths books than
physics books. Therefore, the choice is:
(5 Maths and 0 Physics books) or (4 Maths and 1 Physics books) or (3 Maths and 2 Physics books)

Let M stands for Maths books and P stands fro Physics books.
(5 M and 0P) or (4 M and 1 P) or (3 M and 2 P)
↓ ↓ ↓ ↓ ↓ ↓ ↓ ↓ ↓ ↓ ↓
( ) × ( ) + ( ) × ( ) + ( ) × ( )
= 56 × 1 + 70 × 6 + 56 × 15
= 56 × 420 × 840
= 1.316

Exercises 22.8 of 4. If each nation has to be represented on


1. From a set of six different coins, in how each committee, in how many ways can the
many ways can four or more coins be selected? committee be selected?

2. In how many ways can 12 different objects 4. i. Find the number of different selections of 4
be divided into groups of 6, 4 and 2? letters that can be nade from the letters of the
word SPHERICAL.
3. 5 Irishman, 3 frenchmen and 4 Germans are ii. How many of these selections cannot contain
available for selection to a European committee a vowel?
Baffour Ba Series, Further Mathematics for Schools Page 700
ii. How many of these selections contain at 11. A team of 5 players is to be chosen from 6
least one vowel? boys and 5 girls. If there must be more boys
than girls, how many different teams can be
5. Find the number of different selection of 5 formed.
letters that can be made from the letters of the
word CHEMISTRY. How many of these 12. A group consist of 5 men and 7 women. A
selections contain at least one vowel? commitee of 4 must be chosen from the group.
How many commitees can be chosen in which
6. A team of 6 players is to be chosen from a there are an odd number of men?
group of 10 players . One of the 6 is then to be
elected as captain and another as vice – captain. 13. A club has only 5 women and 4 men as
In how many ways can this be done? (Hint: members. A team of 3 is to be chosen to
select and arrange) represent the club. In how many ways can this
be done if :
7. In how many ways can a committee of 7 i. there are no restrictions.
people be selected from 4 men and 6 women, if ii. the club captain must be on the team?
the committee must have at least 4 women on iii. there must be atleast one woman on the
it? team?
iv. there must be more women than men on the
8. A committee of six is to be formed from team?
eight students and five teachers. How many
different commitees can be formed if there are Challenge Problems
to be more teachers than students? Twelve distinct points are taken on the
circumference of a circle.
9. An examination consist of ten questions, four i. a. Calculate the number of different chords
in section A and the rest in section B. A that can be formed using these points as end
candidate must attempt 5 questions, at least two points.
of which must be from each section. How many b. How many different triangles can be formed
different ways may the candidate select the five using these points as vertices?
questions? ii. a. Calculate the number of different
quadrilaterals that can be formed using these
10. In how many ways can a committee of six points as vertices.
be selected from five men and four women, if b. Two of the vertices are labeled x and y
each committee consist of : respectively.
i. an equal number of men and woman? c. How many of the above quadrilaterals have
ii. at least three men? x and y as vertices?

Baffour Ba Series, Further Mathematics for Schools Page 701


23 PROBABILITY II Baffour Ba Series

The Binomial Probability Distribution “goodness” here. Thus, for example, when
A binomial experiment is one that possesses the looking at births, the statistician might label the
following properties: birth of a boy as a “success” and the birth of a
1. The experiment consists of n repeated trials; girl as a “failure,” but it wouldn‟t necessarily
Each trial results in an outcome that may be mean the same thing to the parents. The usual
classified as a success or a failure (hence the notation is :
name, binomial); p = probability of success,
q = probability of failure = 1 – p.
The probability of a success, denoted by p, Note that p + q = 1.
remains constant from trial to trial and repeated
trials are independent. A Bernoulli trial
The number of successes X in n trials of a In statistical terms, a Bernoulli trial is each
binomial experiment is called a binomial repetition of an experiment involving only 2
random variable. outcomes.

The probability distribution of the random Most atimes, the interest is in the result of
variable X is called a binomial distribution, and independent, repeated bernoulli trials, i.e. the
is given by the formula: nCx q n − x px. number of successes in repeated trials.
1. Independent - the result of one trial does not
where affect the result of another trial.
n = the number of trials 2. Repeated - conditions are the same for each
x = 0, 1, 2, ... n trial, i.e. p and q remain constant across trials.
p = the probability of success in a single trial This is refers to as a stationary process. If p
q = the probability of failure in a single trial and q can change from trial to trial, the process
(i.e. q = 1 − p) is nonstationary. The term identically
distributed is also often used in this case.
P(X) gives the probability of successes
in n binomial trials. A binomial distribution gives the probabilities
associated with independent, repeated Bernoulli
For example, many experiments share the trials.
common element that their outcomes can be
classified into one of two events, e.g. a coin can In a binomial distribution, the probabilities of
come up heads or tails; a child can be male or interest are those of receiving a certain number
female; a person can die or not die; a person of successes, x, in n independent trials each
can be employed or unemployed. These having only two possible outcomes and the
outcomes are often labeled as “success” or same probability, p, of success. For example,
“failure.” Note that there is no connotation of using a binosmial distribution, the probability
Baffour Ba Series, Further Mathematics for Schools Page 702
of getting 4 heads in 10 coin tosses can be are carried out (that is, independent events).
determine as follows:. Then what is the probability that, in n
First, determine the probability of one possible successive trials, exactly x will be successful ?
way the event can occur, and then determine
the number of different ways the event can General Analysis of the problem
occur. That is, P(Event) Build up the solution in simple stages:
= (Number of ways event can occur) × P(One a. If p is the probability of success in a single
occurrence). trial, then the probability of failure is q = 1− p
b. In the following table, let S stand for success
In trying to find the probability of getting 4 and let F stand for failure. The table shows the
heads in 10 tosses, call getting a heads a possible results of one, two or three trials and
“success.” Also, in this case, n = 10, the their corresponding probabilities:
number of successes is x = 4, and the number of
failures (tails) is n – x = 10 – 4 = 6. No. of Possible Results Respective
Trials Prob.
One way this can occur is if the first 4 tosses 1 F,S q, p
are heads and the last 6 are tails, i.e. the
binomial distribution is S S S S F F F F F F 2 FF,FS,SF,SS q2, qp, pq, p2
The likelihood of this occurring is :
P(S) × P(S) ×P(S) × P(S) × P(F) × P(F) × P(F) 3 FFF, FFS, FSF, q3, q2p, q2p, qp2,
FSS, SFF, SFS, q2p, qp2, qp2, p3
× P(F) × P(F) × P(F)
SSF, SSS
More generally, if p = probability of success
and q = 1 – p = probability of failure, the
Summary
probability of a specific sequence of outcomes
i. In one trial, the probabilities that there will
where there are r successes and n – x failures is
be exactly 0 or exactly 1 successes are the
respective terms of the expression q + p.
In this particular instance, p = q = 0.5, x = 4,
ii. In two trials, the probabilities that there will
n – x = 6, so the probability of 4 straight heads
be exactly 0, exactly 1 or exactly 2 successes
followed by 6 straight tails is:
4 are the respective terms of the expression : q2 +
0.5 0.56 = 0.0009765625 (or 1 out of 1024). 2qp + p2 = (q + p)2.

Finding the probability that in n successive iii. In three trials, the probabilities that there
trials, exactly x will be succesful will be exactly 0, exactly 1, exactly 2 or exactly
Statement of the problem 3 successes are the respective terms of the
Suppose that the probability of success in a expression:
single trial is unaffected when successive trials
q3 + 3q2p + 3qp2 + p3 = (q + p)3.

Baffour Ba Series, Further Mathematics for Schools Page 703


iv. In any number, n, of trials, the probabilities Solution
that there will be exactly 0, exactly 1, exactly 2, Method 1
exactly 3, ...... or exactly n successes are the p= = 0.1,
respective terms in the binomial expansion of
q = 1 – 0.1 = 0.9,
the expression (q + p)n.
n = 12,
x=9
Main Result:
The probability that, in n trials, there will be
P(X = x) = nCx
exactly x successes, is the term containing
P (at least 9 are defective);
in the binomial expansion of (q + p)n. = P(X = 9) + P(X = 10) + P(X = 11) + P(X = 11)
It can be shown that this is the value of = 12
C9 ( ) ( ) + 12C10 ( ) ( ) +
n
Cx 12
C11 ( ) ( ) + 12C12 ( ) ( )
= 1.658 ×
Worked Examples
1. Determine the probability that, in 6 tosses of
Method 2
a coin, there will be exactly 4 heads.
p= = 0.1, q = 1 – 0.1 = 0.9, n = 12,
Solution The required probabilities (added together) of
q = 0.5, p = 0.5, n = 6, x = 4. exactly 9, 10, 11 or 12 defective items and
P(exacty 4 heads); these are the last four terms in the expansion of
= P(X = 4) (q + p)n. That is,
= 6 C4 ( ) ( ) = 12C9.(0.1)9.(0.9)3 + 12C10.(0.1)10.(0.9)2 + 12C11.
(0.1)11.(0.9) + (0.1)12
=
= 1.658 ×

2. Determine the probability of obtaining 4. A fair die is thrown 5 times. Calculate to


exactly 2 fives in 7 throws of a die. three decimal places, the probability of
obtaining:
Solution
i. at least 3 sixes;
q = , p = , n = 7, x = 2. ii. exactly 2 sixes.
P(exacty 2 fives);
= P(X = 2) Solution
= 7 C2 . . / . . / i. P( 6 in a single throw) = p =

= 0.234 P( not 6 in a single throw) = q =


n=5
3. It is known that 10% of certain components P(X = x) = nCx
manufactured are defective. If a random sample
of 12 such components is taken, what is the P (at least 3 sixes);
probability that at least 9 are defective ? = P(X = 3) + P(X = 4) + P(X = 5)

Baffour Ba Series, Further Mathematics for Schools Page 704


= 5 C3 . / . / + 5 C4 . / . / + 5 C5 . / . / . / = . / + 6. / . /+ 15. / . /
= + + = = 0.036 = 0.3349 + 0.4019 + 0.2009
= 0.9377
ii. P(exactly 2 sixes);
6. The probability that a pupil will be caned
= P(X = 2)
daily in school is . If a school week consist of
= 5 C2 . / . /
5 days, find correct to 3 decimal places, the
= probability that the pupil will:
= 0.161 i. be caned in 2 days only;
ii. not be caned in the week;
5. Determine the probability of throwing at iii. by all means be caned during the week.
most 2 sixes in 6 throws of a die.
Solution
Solution i. P(X = x) = nCx , where x is the
Method 1 number of times a student is caned.
The phrase “at most 2 sixes” means exactly 0, p= ,q=1– = ,n=5
or exactly 1, or exactly 2.
p = and q = P(caned in 2 days);
n = 6, x = 2. = P(X = 2)
P(X = x) = nCx = 5 C2 . / . /
= 0.346
P (at most 2 sixes);
= P(X = 0) + P(X = 1) + P(X = 2)
ii. P(no caning);
6 6 6
= C0 . / . / + C1 . / . / + C2 . / . / = P(X = 0)
= 0.3349 + 0.4019 + 0.2009 = 5 C0 . / . /
= 0.9377
= 0.078
Method 2
The phrase “at most 2 sixes” means exactly 0, iii. P(by all means caned);
or exactly 1, or exactly 2. Hence, add together = 1 – P(no caning)
the first three terms in the expansion of: = 1 – 0.078
= 0.922
(q + p)6, where q = and p =
Note:
It may be shown that
The use of the “binomial distribution” becomes
(q + p)6 = q6 + 6q5p + 15q4p2 + ......
very tedious when the number of trials is large
By substituting for q and p, the sum of the first
and two other standard distributions (called the
three terms turns out to be :
Baffour Ba Series, Further Mathematics for Schools Page 705
“normal distribution” and the “Poisson c. at least 2, students buy text books.
distribution”) can sometimes be used.
Solution
Solved Past Questions a. p = = 0.2
1. In an examination, 90% of the candidates
q = 1 – 0.2 = 0.8, n = 5
passed. If 8 of the candidates are selected at
Using P(X = x) = nCx , where x is the
random, calculate the probability that;
number of students;
a. exactly 5 passed;
b. at most 3 failed.
P(exacty 3);
= P(X = 3)
Solution
= 5C3 ( ) ( 8)
a. Probability of candidates who passed;
= 0.0512
= 0.9
= 0.051 (3 d.p)

Probability of candidates who failed; b. P( less than 4 students)


= 0.1 = P(X ≤ 4)
= 1 – P(X ≥ 4)
Using P(X = x) = nCx = 1 – [P(X = 4) + P(X = 5) ]
For p = 0.9, q = 0.1, n = 8 = 1 – [5C4 ( ) ( 8) + 5C5 ( ) ( 8) ]
= 1 – [0.0064 + 0.00032]
P(exacty 5 passed) = 1 – 0.00672
= P(X = 5) = 0.993 (3 d. p)
= 8 C5 ( ) ( )
= 0.00331 c. P(at least 2 students);
= P(X = 2) + P(X = 3) + P(X = 4) + P(X = 5)
b. P( at most 3 failed); = 5C2 ( ) ( 8) + 5C3 ( ) ( 8) + 5C4
= P(X = 0) + P(X = 1) + P(X = 2) + P(X = 3) ( ) ( 8) + 5C5 ( ) ( 8)
= 8C0 ( ) ( ) + 8C1 ( ) ( ) + 8C2 = 0.2048 + 0.0512 + 0.0064 + 0.00032
( ) ( ) + 8 C3 ( ) ( ) = 0.26272
= 0.43047 + 0.38264 + 0.14880 + 0.033067 = 0.263 (3 d. p)
= 0.995
Method 2
2. A survey carried out among a group of P(at least 2 students);
secondary school students showed that 2 out of = P(X ≥ 2 )
every 10 students buy text books. If 5 students = 1 – P(X < 2)
are randomly selected from the group, find, = 1 – [P(X = 0) + P(X = 1)]
correct to 3 decimal places, the probability that: = 1 – [5C0 ( ) ( 8) + 5C1 ( ) ( 8) ]
a. exactly 3; = 1 – [0.32768 + 0.4096]
b. less than 4; = 1 – 0.073728

Baffour Ba Series, Further Mathematics for Schools Page 706


= 0.263 (3 d.p) n=5
Using P(X = x) = nCx ,
3. Two out of every three teachers in a
secondary school are females. If 5 teachers are P(a red ball was picked 3 times);
chosen at random from the school, what is the = P(X = 3)
probability that:
= 5 C3 . / . /
a. exactly 3 are females;
b. at least 4 are females. = 10 × ×
= 0.2710
Solution
a. P(male) = P(a black ball was picked at most two times);
= P(X ≤ 2)
P(female) =
= P(X = 0) + P(X = 1) + P(X = 2)
n=5
P(X = x) = nCx = 5 C0 . / . / + 5 C1 . / . / + 5 C2 . / . /
= 0.0173 + 0.1084 + 0.2710
P(exactly 3 females) = 0.3967
= P(X = 3)
= 5 C3 . / . / 5. For every 10 persons in the city, one is left
handed. If 6 persons are selected at random
= = 0.3292 from the city, find the probability that;
i. exactly three;
P (at least 4 females); ii. more than half;
= P(X = 4) + P(X = 5) iii. at least 2.
= 5 C4 . / . / + 5 C 5 . / . /
Solution
= + = i. P (left handed) = = 0.1
= 0.4609
P (right handed) = = 0.9
4. A bag contains 4 red and 5 black identical p = 0.1, q = 0.9 and n = 6
balls. If 5 balls are selected at random from the
bag, one after the other without replacement, P(exactly three are left handed)
find the probability that; = P(X = x) = nCx ,
6
i. a red ball was picked three times; = C3 ( ) ( )
ii. a black ball was picked at most two times. = = 0.0146

Solution ii. P(more than half are left handed)


i. n (R) = 4, n (B) = 5, n (S) = 4 + 5 = 9 = P(X > 3)
P(R) = , P(B) = = P(X = 4) + P(X = 5) + P(X = 6)

Baffour Ba Series, Further Mathematics for Schools Page 707


= 6 C4 ( ) ( ) + 6 C5 ( ) ( ) + 6 C6 c. exactly three will die;
( ) ( ) d. a particular three will die and the others
= 0.001215 + 0.000054 + 0.000001 survive.
= 0.00127
6. A particular student can solve, on average,
iii. P(at least two are left handed; half of the problems given to him. In order to
P(X ≥ 2) pass the course, he is required to solve seven
= P(X = 2) + P(X = 3) + P(X = 4) + P(X = 5) + out of ten problems on an examination paper.
P(X = 6) What is the probability that he will pass?
= 6C2 ( ) ( ) + 6C3 ( ) ( ) + 6C4
( ) ( ) + 6 C5 ( ) ( ) 7. At an election, 40% of the voters favoured
= 0.098415 + 0.01458 + 0.001215 + 0.000054 the Environmental party. Eight voters were
+ 0.000001 interviewed at random. Find the probability that
= 0.1143 a. exactly 4 voters favoured the environmental
party.
Exercises 23.1 b. a majority of those interviewed favoured the
1. A coin is tossed six times. What is the environmental party.
probability of getting exactly four heads? c. at most 3 of the people interviewed favoured
the environmental party.
2. What is the probability of throwing at least
four 7‟s in five throws of a pair of dice ? 8. In the long run, Thomas wins two out of
every 3 games. If Thomas plays five games,
3. In a roll of five dice, what is the probability
find the probability that he will win:
of getting exactly four faces alike ?
a. exactly 4 games.
b. at most 4 games.
4. If three dice are thrown, determine the
c. no more than 2 games.
probability that:
d. all 5 games.
a. all three will show the number 4;
b. all three will be alike;
9. A tenis player finds out that he wins 3 out of
c. two will show the number 4 and the third,
7 games he plays. If he plays 7 games straight,
something else;
find the probability that he will win:
d. all three will be different;
a. exactly 3 games.
e. only two will be alike.
b. at most three games.
5. Hospital records show that 10% of the cases c. all 7 games.
of a certain disease are fatal. If five patients d. no more than 5 games.
suffer from this disease, determine the
probability that: 10. The births of males amd females are
a. all will recover; assumed to be equally likely, find the
b. at least three will die; probability that in a family of 6 children;

Baffour Ba Series, Further Mathematics for Schools Page 708


a. there are exactly 3 girls. type are alike, r of a third type are alike, etc is:
b. there are no girls.
c. the girls are in the majority.

Permutations and Combinations Worked Example


Arranging Objects In how many ways can the letters in the word:
Rule 1 STATISTICS be arranged?
The number of ways of arranging n unlike
objects in a line is n! (pronounced „n factorial‟). Solution
n! = n × (n – 1) × (n – 2) ×…× 3 × 2 × 1 STATISTICS
n = 10
Worked Examples There are 3 S‟s, 2 I‟s and 3 T‟s in this word,
How many different ways can the letters P, Q, therefore, the number of ways of arranging the
R, S be arranged? letters are:
= 50400
Solution
P, Q, R, S Rings and Roundabouts
n=4 1. The number of ways of arranging n unlike
There are four spaces to be filled: objects in a ring when clockwise and
, -, , -, , -, , - = 4! = 24. anticlockwise arrangements are different is:
(n – 1)!
Explanation
The first space can be filled by any one of the 2. When clockwise and anti-clockwise
four letters. = 4 possibilities arrangements are the same, the number of ways
is : (n – 1)!
The second space can be filled by any of the
remaining 3 letters = 3 possibilities Worked Examples
1. Ten people go to a party. How many
The third space can be filled by any of the 2 different ways can they be seated?
remaining letters = 2 possibilities
Solution
The final space must be filled by the one Anti-clockwise and clockwise arrangements are
remaining letter = 1 possibility the same. Therefore, the total number of ways
The total number of possible arrangements is is (10 – 1)! = 181 440
4 × 3 × 2 × 1 = 4!
Permutations
Rule 2 A permutation is an ordered arrangement. The
The number of ways of arranging n objects, of number of ordered arrangements of r objects
which p of one type are alike, q of a second taken from n unlike objects is:
Baffour Ba Series, Further Mathematics for Schools Page 709
n
Pr = ( 3. Suppose we have 12 adults and 10 kids as an
)
audience of a certain show. Find the number of
ways the host can select three persons from the
Worked Example
audiences to volunteer. The choice must
1. In the Match of the Day‟s goal of the month
contain two kids and one adult.
competition, you had to pick the top 3 goals out
of 10. Since the order is important, it is the
Solution:
permutation formula which we use.
As order does not matter here so choosing 2
10
P3 = = 720 kids and an adult,
There are therefore 720 different ways of = 10C2 × 12C1
picking the top three goals. = ×
( ) ( )
= 45 × 12
Combinations
= 540
The number of ways of selecting r objects from
There are 540 ways in which the host can
n unlike objects is:
choose the volunteers containing two kids and
( ) = nCr = ( ) an adult.

Worked Examples Probability


1. There are 10 balls in a bag numbered from 1 The above facts can be used to help solve
to 10. Three balls are selected at random. How problems in probability.
many different ways are there of selecting the
three balls? Worked Examples
1. In the National Lottery, 6 numbers are
Solution chosen from 49. You win if the 6 balls you pick
n = 10 and r = 3 match the six balls selected by the machine.
10 What is the probability of winning the National
C3 = = = 120
( )
Lottery?
2. In a lucky draw, chits of ten names are out
Solution
in a box out of which three are to be taken out.
The number of ways of choosing 6 numbers
Find the number of ways in which those three
from 49 is 49C6 = 13 983 816 .
names can be taken out.
Therefore the probability of winning the lottery
Solution is = 0.000 000 0715 (3sf),
n = 10 and r = 3 which is about a 1 in 14 million chance.
10
C3 = = = 120
( )
Combination Probability Formula
So there are 120 different ways of choosing When selecting more than one item, it is
three names out of the ten from the box. important to use combination formula to find

Baffour Ba Series, Further Mathematics for Schools Page 710


probability. Total number of outcomes = 9 + 6 = 15 balls,
P(3W) = selecting 3 balls from the 15 balls.
The combination formula for selecting r items This can be done in 15C3 ways.
from n is : nCr = P(3W) = = = = 0.1846
( )

Worked Examples iii. Let the probability of selecting 4 black balls


1. A bag contains 9 white and 6 black balls. be P(4R)
What is the probability of selecting
i. 2 white balls Favourable outcome = 4 balls from 6 black
ii. 3 white balls balls. This can be done in 6C4 ways.
iii. 4 black balls
iv. 1 white and 3 black balls Total number of outcomes = 9 + 6 = 15 balls,
v. 4 white and 5 black balls = selecting 4 balls from the 15 balls. This can
be done in 15C4 ways.
Solution:
The combination formula for selecting r items P(4R) = = = 0.0110
n
from n is : Cr = ( )
iv. Let the probability of selecting 1 white and
Also the probability of an event A is : 3 black balls be P(1W∩3B)

P(A) =
Favorable cases = 1 white from the 9 white
i. Let the probability of selecting 2 white balls balls and 3 black balls from the 6 black balls.
= P(2W) This can be done in 9C1 × 6C3 ways.

Favourable outcomes = 2 balls from 9 white Total outcomes 9 + 6 = 15 balls, the total
balls. This can be done in 9C2 ways. P(1W∩3B) = selecting 4 balls from the 15
balls. This can be done in 15C4 ways.
Total number of outcomes = 9 + 6 = 15 balls,
P(2W) = selecting 2 balls from the 15 balls.
P(1W∩3B) = = = 0.1319
This can be done in 15C2 ways.

v. Let the probability of selecting 4 white and


P(2W) = = = 0.3429
5 black balls be P(4W ∩ 5B)

ii. Let the probability of selecting 3 white balls Favorable cases = select 4 white from the 9
be P(3W) white balls and 3 black balls from the 5 black
Favourable outcomes = selecting 3 balls from balls. This can be done in 9C4 × 6C5 ways.
9 white balls. This can be done in 9C3 ways.

Baffour Ba Series, Further Mathematics for Schools Page 711


Total out comes 9 + 6 = 15 balls, Total outcome = 4 + 5 = 9 people,
P(4W ∩ 5B) = selecting 9 balls from the 15 P(R1) = selecting 3 people from the 9 people.
balls. This can be done in 15C9 ways. This can be done in 9C3 ways.

P(4W ∩ 5B) = = = 0.1510 P(R1) = = = 0.0476

2. There are 4 men and 5 women. Find the iii. Let the probability of selecting at least one
probability of selecting 3 of which: woman P(At least 1W)
i. exactly two are women;
ii. no woman; Since we have to select three people, at least
iii. at least one women; one women means 1 women or 2 women or 3
iv. at most one women; women. That is 1 woman and 2 men or 2
v. no men. women and 1 man or all three woman.

Solution - Favorable cases;


i. Let the probability of selecting exactly two = 5C1 × 4C2 + 5C2 × 4C1 + 5C3 ways.
women be P(2W)
Total outcome = 4 + 5 = 9 people,
The choice is to select three people. Exactly P(Atleast 1W) = selecting 3 people from the 9
two women means two women and one man people. This can be done in 9C3 ways.
Favorable cases = selecting 2 women from the
P(Atleast 1W)
5 women and 1 man from the 4 men. ( )
This can be done in 5C2 × 4C1 ways. = = = 0.9524

Total outcome = 4 + 5 = 9 people, iv. Let the probability of selecting at most one
P(2W) = selecting 3 people from the 9 people. woman be P(At most 1W)
This can be done in 9C3 ways.
P(2W) = = = 0.4762 To select three people, at most one women
means 0 women or 1 women. That is 0 woman
and 3 men or 1 woman and 2 men.
ii. Let the probability of selecting no woman
4
ne P(R1) Favorable cases = C3 + 5C1 × 4C2 ways.

Since we have to select three people, no woman Total outcome = 4 + 5 = 9 people,


means all three are men. P(At least 1W) = selecting 3 people from the 9
people. This can be done in 9C3 ways.
Favorable cases = selecting 3 men from the 4
( )
men. This can be done in 4C3 ways. P(At least 1W) = = = 0.4048

Baffour Ba Series, Further Mathematics for Schools Page 712


v. Let the probability of selecting no men be iii. P(none of them is white)
P(M1). = P(RRR) or P(BBB) or P(RBB) or P(BRR)
7 8 7 8 8 7
To select three people, with no men means all = + + +
20 20 20 20
three are women.
= + + + = =
Favorable cases = select 3 women from the 5
women. This can be done in 5C3ways.
4. A committee of 5 people including at least
Total outcome 4 + 5 = 9 people, one woman is to be selected from a group of 5
P(M1) = selecting 3 people from the 9 people. men and 4 women. Calculate correct to three
This can be done in 9C3 ways. decimal places, the probability that there will
P(M1) = = = 0.1190 be few women than men on the committee.

Solution
3. A bag contains 20 small identical objects, 8
n(W) = 4, n(M) = 5, n(S) = 9
of them are balck, 7 are red and the rest are
The probability that there will be few women
white. If three of the objects are selected at
than men on the committee.
random the bag at once, find the probability
P(1W and 4M) or P(2W and 3M)
that :
4 5 4 5
i. one is black, one is blue and the remaining is +
= 9 9
white.
ii. exactly two are red.
= + = = 0.635
iii. none of them is white.

Solution 5. Two students are chosen at random from a


i. n(S) = 20, n(B) = 8, n (R) = 7 and n (W) = 5 class consisting of twelve girls and eight boys.
What is the probability that the two students
P(1 black, 1 blue and the remaining is white) selected are :
= P(1R ∩ 1B ∩ 1W) a. both boys;
b. a boy and a girl.
8 7 5
= 20
= =
Solution
n(G) = 12, n (B) = 8, n(S) = 20
ii. P(exactly two are red); P(Both boys) = P(BB)
= P(RRB) or P(RRW);
8
8 7 5 = = =
= 7 +
20
20 20

= + = =
b. P(a boy and a girl) = P(BG)

Baffour Ba Series, Further Mathematics for Schools Page 713


iii. at most two are good apples.
8 12
= 20
= = =
3. Consider a family of 4 children. What is the
probability of having:
Solved Past Question
i. two boys and two girls;
1. A committee consist of 6 Nigerians and 4
ii. at least one boy ;
Ghanaians. If a chairman and a secretary are
iii. no boys;
selected at random, what is the probability that
iv. at most two boys.
they are:
a. of same nationality; 4. A box contains 100 bulbs, 20 of which are
b. of different nationality. defective. 10 of the bulbs are selected for
inspection. Find the probability that:
Solution i. all 10 are defective;
i. n(N ) = 6, n (G) = 4, n (S) = 10 ii. all 10 are good;
P(same nationality) = P(NN) or P(GG) iii. at least one is defective;
6 4 iv. at most three are defective.
= 10 +
10
5. A coin is tossed 16 times. What is the
= + = = probability of obtaining?
i. exactly five heads;
b. P(Different nationality) ; ii. 10 or more tails
= P(1 N, 1 G)
=6 4 Selection Without Replacement
10 “Selection without replacement” means
selecting an item from a lot without putting it
= + =
back from where it was taken.

Exercises 23.2 Generally, if a selection is done without


1. The probability that a player scores a goal in replacement,
a football match is . What is the probability 1. The number of balls or items in the various
that out of 5 matches he may score a goal in: groups reduces.
i. exactly 2 matches; 2. The total number of items in the lot (Sample
ii. none of the matches? space) reduces.
3. The probability of subsequent draws depends
2. A basket contains 70 good apples and 30 are on the previous selection.
spoiled. Three apples are at random from the
basket. What is the probability that of the 3 Worked Examples
apples: 1. A bag contains 15 small identical balls
i. exactly two are good apples; except for colour, 6 of them are black, 5 are
ii. at least one is good apples; white and the rest are red. Three balls are

Baffour Ba Series, Further Mathematics for Schools Page 714


selected at random, one after the other, without other without a replacement. Find the
replacement from the bag. Find the probability probability that they are all of the same color?
that:
i. one of each colour is selected; Solution
ii. no red ball is selected. n(B) = 5, n (Y) = 4, n(W) = 3, n(S) = 12
P(all three are the same color);
Solution = P(BBB) or P(YYY) or P(WWW)
i. n(S) = 15, n(B) = 6, n(W) = 5 and n(R) = 4 5 4 3
P(one of each colour is selected) =12 + +
12 12
8 7 5
= = + + =
20

= = Exercises 23.3
1. A bag contains 4 yellow, 5 green and 6
ii. P(no ball is red); orange candies. Two candies are drawn one
There are 11 none red ball (6B + 5W) out of after the other without replacement. Find the
which 3 is to be selected. probability that both candies are green.

11
= 11 = = 2. A jar contains 10 blue balls and 11 red balls.
15 Two balls are drawn at random one after the
11
other without replacement. What is the
2. A 1box contains 5 black, 4 yellow and 3 white
probability of drawing two red balls?
balls. Three are picked at random one after the

Baffour Ba Series, Further Mathematics for Schools Page 715


24 STATISTICS Baffour Ba Series

Meaning of Statistics might actually be 9.812. Age is also another


It is a branch of Mathematics which deals with example of continuous data. Although the
the collection and study of numbers in order to values are usually taken as whole numbers like
get facts or information. 15, 18, 26 years, they can assume intermediate
values like 15 or 18 years, 2 months…..
Data: It is information collected from various
sources through observation or from results of
Exercises 10.1
an experiment. These are raw data, array and
Determine whether the following data is
grouped data.
continuous or discrete.
1. The number of children in the various
Types of Data
families in the community .
Raw Data:It is a data which is in the form in
2. The monthly income of workers in a factory.
which it was collected from source.
3. The marks obtained by a student in an
examination.
Array: It refers to data which has been arranged
4. The distance travelled by a day student to
in ascending or descending order of magnitude.
school each school day.
5. The amount of pocket money given to
Grouped Data: It is a data which has been
students.
arranged into classes like 0 – 9, 10 – 19 etc.
Sources and Collection of Data
Discrete data: It is data that can be counted, for
A data or information can be collected from
example the number of people, cattle, sheep,
many sources such as schools, hospitals and
table, chair etc. The count of discrete data is
business.
whole numbers, since we cannot get half a
person, or 2 sheep on a farm. Similarly, the In schools, records such as attendance, personal
number which appears on a die when thrown records, results of an examination and others
can only be 1, 2, 3, 4, 5 or 6. Other examples of are kept. At the hospitals, records of
discrete data are vehicles passing a station, and attendance, diseases treated number of patients
the grade obtained by a student in a class. treated etc are also collected and kept.
In order to collect data, a survey must be
Continuous data: It is a data which result from conducted. The result of a survey is called a
measurement, for example, the height of a raw data. For example, 20 students in
person, the height of a tree or the, the time Asuofua D/A J.H.S. one were asked to mention
taken to complete an activity. The count of the days of the week in which they were born
continuous data is not a whole number but and the following results were obtained;
rounded off. For instance, a tree of height 9.8m Thursday, Thursday, Wednesday, Tuesday,

Baffour Ba Series, Further Mathematics for Schools Page 716


Friday, Wednesday, Monday, Tuesday, but a sight value which must be carefully
Wednesday, Friday, Thursday, Saturday, identified by sight.
Sunday, Friday, Thursday, Monday, Sunday,
Tuesday, Sunday, Saturday. Worked Example
The ages of 10 K.G pupils are recorded as; 4, 4,
When data are collected from a source as 5, 5, 6, 5, 6, 6, 6, 6. Find the modal age.
above, it is called a raw data.
Solution
Raw data is not in organized form. There is The mode or the modal age is 6 because it‟s the
therefore the need to process raw data in an number that appears most.
organized form called information to enhance
understanding. From the above example, Median
information can be obtained in a tabular form The median of a set of numbers is the middle
as follows: value or the arithmetic mean of the two middle
values when they are ordered in ascending or
Day Tally No. of pupils descending order of magnitude.
Sunday /// 3
Monday // 2 Steps in finding the median of a raw data with
Tuesday /// 3 “n” numbers;
Wednesday /// 3 Method I
Thursday //// 4 I. Re – arrange the numbers in either ascending
Friday /// 3
or descending order.
Saturday // 2
II. Count to locate the middle value as the
In this case, one can easily know the number of median.
student born on each day of the week. III. If two numbers, a and b, appear as the
middle values, find the sum of the two and
Measure of Central Tendencies divide by 2 to obtain the median. That is:
(Mode, Median and Mean) median = .
The mode, median, and mean are the three most
common examples of measures of central Method II
tendencies. They are also called averages. They I. Arrange the numbers in order of magnitude
refer to a single digit which enables one to and count to ensure that they are up to “n”
assess the position in which a group is with numbers.
respect to others. II. If n is odd, then the median is the middle
term. That is; ( ) term or position
Mode
III. If n is even, then the median is the
The mode is the most commonly occurring
arithmetic mean of the two middle terms. That
number or the number that repeats itself most in
a set of data.The mode is not a calculated value is: ( ) and ( ) terms or positions

Baffour Ba Series, Further Mathematics for Schools Page 717


Worked Examples Total number of entries, n = 10 (even) Median
1. Find the median of the following numbers: = (n) and (n) terms
47, 30, 56, 31, 55, 43 and 44
( ) = 5th and ( ) + 1 = 6th
Solution
Method I The median positions are 5th and 6th
Re-arrange the numbers in ascending order to 4, 4, 4, 5, 5, 5, 6, 6, 7, 7
get: 30, 31, 43, 44, 47, 55, 56,
The middle number is 44. ⇒median = 44 6thPosition number
th
5 Position number
Method II Median = =5
Re-arrange the numbers in ascending order to
get: 30, 31, 43, 44, 47, 55, 56,
Mean
Total number of observations is 7
The mean is the ratio of the sum of all the
Therefore, n = 7 and n is odd
values in a raw data to the number of entries.
Median = ( ) term or position
= ( ) term or position The mean is denoted by ̅. Thus, given the set
of values: x1, x2, x3, x4, ..xn,
= (8) = 4th position
the mean,( ̅ )= =
30, 31,43, 44, 47, 55, 56, Where n is the position of the last number of
the entries.
4th Position number
Worked Examples
Therefore, the median is 44.
1. Find the mode, median and mean of the
following numbers: 4, 24, 10, 17, 19, 21, 10
2. Find the median of the following numbers;
4, 5, 4, 6, 7, 5, 7, 4, 6, 5 Solution
Mode = 10 (Most frequently number)
Solution
Method I Median: Ordering the numbers in ascending
Re-arrange the numbers in ascending order to order, 4, 10, 10, 17, 19, 21, 24
get: 4, 4, 4, 5, 5, 5, 6, 6, 7, 7 The middle number is 17. ⇒median = 17
The middle numbers are 5 and 5. Therefore,
median = =5 ̅= = = 15

Method II 2. The masses of 12 boxes measured to the


Re-arrange the numbers in ascending order to nearest kg are: 26, 23, 27, 28, 29, 22, 23, 27,
get: 4, 4, 4, 5, 5, 5, 6, 6, 7, 7 20, 20, 24, and 25. Find the median.

Baffour Ba Series, Further Mathematics for Schools Page 718


Solution 3. The ages in years of 8 boys are: 14, 14, 15,
In ascending order: 20, 20, 22, 23, 23, 24, 25, 15, 12, 11 13, 10.What is the average age?
26, 27, 27, 28, 29
The middle numbers are 24 and 25. Properties of Mean
Median = = = 24.5 1. If the mean of a set of numbers is p, and each
number is increased by q, the new mean is p +
q. That is ;
Mean ( ̅ ) =
New Mean = old mean + increment
Where n = number of entries = 11
̅= = = 5.36(2 d.p.) 2. If the mean of a set of numbers is p, and
each number is decreased by q, the new mean is
3. Find the mean, median and modal heights of p – q. That is;
the following distribution: 170cm, 171cm, New Mean = old mean – decrement
173cm, 174cm, 177cm, 177, 184cm, 186cm,
3. If the mean of a set of numbers is p, and each
Solution number is multiplied by q, the new mean is qp.
i. Mean ( ̅ ) = That is;
Where n = number of entries = 8 New Mean = Multiplier × old mean
̅=
Worked Examples
̅= = 176.50 cm 1. The mean of 12 numbers is 18. If each of the
numbers is increased by 4. What is the new
ii. To find the median, arrange the numbers in mean?
ascending order to obtain: 170cm, 171cm,
173cm, 174cm, 177cm, 177, 184cm, 186cm. Solution
New Mean = old mean + increment
The two middle values are 174 and 177. = 18 + 4 = 22
Median = = 175. 50 cm
Frequency and Frequency Diagrams
iii. The modal height = 77cm The number of times an event scores in a given
data is called its frequency.
Exercises 24.1
1. The marks scored by 8 pupils in a science Frequency Diagrams for Ungrouped Data
test are 3, 7, 8, 8, 5, 8, 4, 8.What is the median
mark? It is a table that shows the event and the
number of times each event occurs. It is usually
2. The scores of 10 students in an examination divided into four (4) sections as shown below:
are given as follows; 45, 12, 75, 81, 54, 51, 24, Range(x) Tally Frequency(f) fx
67, 19 and 39. What is the median and mean
score? ∑fx=

Baffour Ba Series, Further Mathematics for Schools Page 719


Column 1(x) : is made up of the events usually 5 4 3 4 4 5 4 4 4 3
arranged downward in increasing order. It is 3 3 5 4 3 6 3 2 4 5
also called the range. 3 3 4 3 3 6 5 2 5 5
Column 2 (Tally): consist of slight tilted
strokes like /, that represents the number of Grouped Frequency Table
times an event occurs in a data. That is / for 1, When the values of the entries are put into well
// for 2, /// for 3 and //// for 4. However, at organized groups and presented in a tabular
every fifth successive occurrence, a horizontal form, a grouped frequency table is obtained.
line is drawn to across the first 4 strokes. Thus The groups into which the entries are put are
//// for 5 and //////// // for 12. referred to as classes.

Column 3 (frequency): is made up of the The grouped frequency table is usually used for
digital representation of the tally. data‟s with many entries. Below is an example
of a grouped frequency table
Column 4 (fx): consist of the product of each
event and its corresponding frequency. Class Marks (x) Frequency (f)
is the sum of all the frequencies. 25 – 29 3
is the sum of the product of frequencies 30 – 34 4
and their corresponding events. 35 – 39 7
40 – 44 6
45 – 49 2
Worked Examples
1. The ages 20 school children were recorded as
On the above table, column 1 represents the
follows;
group put into the classes: 25 – 29, 30 – 34,
13 9 15 17 13 17 15 11 9 9
35 – 39, 40 – 44, 45 – 49 and column 2
9 11 9 11 15 11 15 11 11 11
represents the respective frequencies. If the
Make a frequency table for the data.
class 25 – 29 has a frequency of 3, it means that
Solution there are three different or same numbers
within 25 and 29
Age (x) Tally Frequency (f) fx
9 //// 5 45 Class Limits
11 //// // 7 77 Consider the table below;
13 // 2 26
15 //// 4 60
Class Marks (x) Frequency (f)
17 // 2 34
25 – 29 3
= 20 =242
30 – 34 4
35 – 39 7
2. The table below gives family sizes recorded 40 – 44 6
45 – 49 2
by 30 respondents. Construct a frequency table
for the data.
Baffour Ba Series, Further Mathematics for Schools Page 720
It is seen that the classes contain a range of I. Find the difference between the upper class
values i. e. 25 – 29, 30 – 34, 35 – 39, 40 – 44, limit of a particular class interval and the lower
45 – 49. This means that each class has a class limit of the next class and divide by 2. For
smallest value and a largest value. These two example, the class boundary of 25 – 29, 30 –
values are called Class limits. Thus, we have 34… is determined as (30 – 29) = 0.5
the lower class limit representing the smallest
II. Subtract the answer obtained from the lower
number and the upper class limit representing
class limit of each class and add the answer to
the largest number. For example in the class 25
the upper class limit of each class to obtain the
– 29, the lower class limit is 25 and the upper
class boundaries
class limit is 29
Lower class limit 25 – 29 upper class limit Worked Examples
Copy and complete the table below;
Class Boundaries
It is the point where one class separates itself
Marks Class
from the other. It is also explained as the (x) Boundaries
smallest and largest values an item in that class 5–9
can take. For grouped frequency distribution, 10 – 14
each class boundary is usually half way 15 – 19
between the upper limit of one class and the 20 – 24
lower limit of the next class. That is: 25 – 29
= (upper limit of one class + lower limit of the
Solution
next class)
Method 1

For instance, to determine the class boundary of ( )= 9.5


the class 25 – 29 and the next class 30 – 34, ( ) = 14.5
I. Identify the upper class limit of 25 – 29 as 29
( ) 19.5
and the lower class limit of 30 – 34 as 34
II. Find the sum of the limits identified in I as ( ) = 24.5
29 + 30 = 59 Marks Class
III. Find half of the sum obtained in II as: (x) Boundaries
( ) = 29.5 5–9 4.5 – 9.5
10 – 14 9.5 – 14.5
IV. Recognize 29.5 as the upper class boundary 15 – 19 14.5 – 19.5
of the class 24 – 29 and the lower class 20 – 24 19.5 – 24.5
boundary of the next class 30 – 34 25 – 29 24.5 – 29.5

The Class Boundary of a Grouped Data Method 2


To find the class boundary of a grouped data, Consider 5 – 9 and 10 – 14
follow the steps below; ( – ) = 0.5

Baffour Ba Series, Further Mathematics for Schools Page 721


Subtract 0.5 obtained from the lower class limit
of each class and add 0.5 to the upper class Marks (x) Class Mark
limit of each class 5–9
10 – 14
Marks Class 15 – 19
(x) Boundaries 20 – 24
5–9 4.5 – 9.5 25 – 29
10 – 14 9.5 – 14.5
15 – 19 14.5 – 19.5 Solution
20 – 24 19.5 – 24.5 (Calculations are not necessary)
25 – 29 24.5 – 29.5
Marks (x) Class Mark
Class Mid – point/Class Mark 5–9 7
It is the average of the lower and upper limits 10 – 14 12
of a class. This is achieved by adding the lower 15 – 19 17
and upper class limits and dividing the sum by 20 – 24 22
two to get the mid – point of the class. The 25 – 29 27
midpoint is often called class mark. For
example, the class mark of the class 5 – 9 is Class Size/Width
determine as : = =7 The class width or class size or class interval is
the difference between the upper and lower
Worked Example class boundaries of a class. For instance, the
Copy and complete the table below: class width or class size of a class with
boundaries 4.5 and 9.5 = 9.5 – 4.5 = 5

Baffour Ba Series, Further Mathematics for Schools Page 722


Format of a Grouped Frequency Table

Intervals Class Boundary Class Marks (x) Frequency (f) fx

= =

Worked Examples
Below is a record of the marks (out of 50) 39 36 37 42 45 40 41 44 46 44 46
obtained by 22 students in a test. Prepare a group frequency table for the data
14 12 8 19 13 27 22 21 32 30 35 using the intervals 1 – 10 , 11 – 20 …

Marks Class Boundaries Class Marks Frequency (f) fx


1 - 10 0.5 – 10.5 5.5 1 5.5
11 – 20 10.5 – 20.5 15.5 4 62
21 – 30 20.5 – 30.5 25.5 4 102
31 – 40 30.5 – 40.5 35.5 6 213
41 – 50 40.5 – 50.5 45.5 7 318.5
= 22 701

Mean of a Grouped Data III. Substitute the values of


To calculate the mean from a grouped and into the formula below to obtain
frequency table, make sure the table is of the the mean value of the data : Mean ( ̅ ) =
format shown below:

Class Class Mode of a Grouped Data


Interval Mark(x) f fx For a grouped data, the mid – point of the class
with the highest frequency may be taken as the
crude estimate of the mode. The class in which
= the mode occurs is called the modal class.

II. Carefully complete the table and accurately The mode can also be calculated by the
compute the values of and formula;
Mode = L1 + . / C, where;

Baffour Ba Series, Further Mathematics for Schools Page 723


L1 = Lower class boundary of modal class Worked Examples
∆1 = excess frequency of modal class over the 1.The table below shows the distribution of the
frequency of the next lower class. masses of parcels in a local post office
∆2 = excess frequency of modal class over the
frequency of the next higher class. Mass (Kg) Frequency
C = size of modal class. 20 – 24 2
25 – 29 3
When using the formula, create a column for 30 – 34 7
class boundary. 35 – 39 26
40 – 44 29
45 – 49 25
Median of a Grouped Data 50 – 54 6
For a grouped data, it is a bit complicated to 55 – 59 2
determine the median but not the class within
which the median value falls. These are few Find from the table:
hints: i. the mean ii. the mode
I. Find half of the total frequency and identify iii. the modal class iv. the median class
the value. v. the median.
II. Count from the top of the table to obtain a
sum of frequencies that equals the value Solution
obtained.
III. Identify the class within which the values Marks Class
falls as the median class. (kg) Mark(x) f fx
20 – 24 22 2 44
A good estimate of the median is best obtained 25 – 29 27 3 81
from a cumulative frequency curve 30 – 34 32 7 224
35 – 39 37 26 962
The median can also be estimated by the 40 – 44 42 29 1218
formula: 45 – 49 47 25 1175
50 – 54 52 6 312
Median = L1 + 4 5C 55 – 59 57 2 114
= =
Where; 100 4130
L1 = the lower class boundary of the median
class;
Mean ( ̅ ) = = = 41.30 kg
N = total frequency;
= sum of frequencies of all classes lower
than the median class; ii. The mode is the mid point of the highest
= frequency of the median class;
C = the size of the median class. frequency = 42

Baffour Ba Series, Further Mathematics for Schools Page 724


Method 2 (using the formula) v. The median;

Median = L1 + 4 5C
Marks Class Class
(kg) boundary Mark(x) f
20 – 24 19.5 – 24.5 22 2 Median class = 40 – 44
25 – 29 24.5 – 29.5 27 3
L1 = 39.5
30 – 34 29.5 – 34. 5 32 7
N = 100
35 – 39 34.5 – 39.5 37 26
= 26 + 7 + 3 + 2 = 38
40 – 44 39.5 – 44.5 42 29
= 29
45 – 49 44.5 – 49.5 47 25
50 – 54 49.5 – 54.5 52 6 C = 44.5 – 39.5 = 5
55 – 59 54.5 – 59 .5 57 2 ( )
Median = 39.5 + 4 5×5

The modal class is 40 – 44; Median = 39.5 + . / × 5


L1 = 39.5
∆1 = 29 – 26 = 3 Median = 39.5 + . /
∆2 = 29 – 25 = 4 Median = 39.91 (2 d.p)
C = 44.5 – 39. 5 = 5
2. From the table below, find the estimated
Mode = L1 + . / C, values of the mode, median amd mean, suing
formulas.
= 39.5 + . / × 5,
= 39.5 + Time (S) Frequency
51 – 55 2
= 39.5 + 2.1429
56 – 60 7
= 41.6429
= 42 (nearest whole number) 61 – 65 8
66 – 70 4
iii. From the table, the highest frequency is 29
and falls within the class 40 – 44. Therefore, Solution
the modal class is 40 – 44
T Class Class
(s) boundary Mark(x) f fx
iv. The median class 51 – 55 50.5 – 55.5 53 2 106
= × 56 – 60 55.5 – 60.5 58 7 406
61 – 65 60.5 – 65. 5 63 8 504
= × 100 = 50 66 – 70 65.5 – 70.5 68 4 272

Adding frequencies from the top, 50 can be


= =
located in the class 40 – 44. Therefore, the 21 1288
median class = 40 – 44

Baffour Ba Series, Further Mathematics for Schools Page 725


Mode = L1 + . / C, Exercises 24.2
1. The table below shows the distribution of
marks scored by a group of 30 students. Find:
Modal class = 61 – 65
the mode and mean. Ans = 27.4
L1 = 60.5
∆1 = 8 – 7 = 1
Marks 10 – 15 – 20 – 25 – 30 – 35 –
∆2 = 8 – 4 = 4 14 19 24 29 34 39
C = 65.5 – 60.5 = 5 No of
Students 1 3 4 6 7 4

Mode = 60.5 + . / × 5, 2. The table below is a record of the mass of


Mode = 60.5 + . / × 5, eggs recorded at a certain farm.

Mode = 60.5 + . / Mass of egg (g) Frequency


Mode = 60.7 45 – 49 7
50 – 54 16
55 – 59 49
Median = L1 + 4 5C 60 – 64 86
65 – 69 30
Median Class = ( ) 70 – 74 8
75 – 79 4
= ( )
= 11 th position From the table, determine the mode and mean.

Median Class = 61 – 65
3. A gardener dig up his carrots and measure
L1 = 60.5
their lengths ( to the nearest mm) and grouped
N = 21;
the results as shown below:
=7+2=9
=8
Length (mm) Frequency
C = 65.5 – 60.5 = 5 150 – 154 5
( ) 155 – 159 2
Median = 60.5 + 4 5×5
160 – 164 6
165 – 169 8
Median = 60.5 + . /×5
170 – 174 9
Median = 60.5 + . / × 5 175 – 179 11
180 – 184 6
Median = 60.5 + . / 185 – 189 3
Median = 60.69
Estimate the mode, median and mean using the
Mean ( ̅ ) = = = 61.33 involving formulas. Ans : 170.6, 171.7, 175.9

Baffour Ba Series, Further Mathematics for Schools Page 726


The Histogram b. Using the class marks, place the class mark
A histogram is a graph showing the number of at the center of the bars and ensure that each
occurrences of items of data, and is drawn bar has the same width corresponding to the
using vertical bars, similar to that of a bar class width
chart. Group data is usually represented
graphically by histograms. IV. Use the symbol to indicate that the
graph does not start from zero on the horizontal
The bars used for histograms are usually of axis. The vertical axis should start from zero
standard size. The width of each bar is V. Give the graph a title
equivalent to the class interval representing it.
Also, the area of the bars is proportional to the Histogram for Ungrouped Data
total frequency of the items in the class they Worked Examples
represent 1. A die was tossed 50 times and the following
were recorded
Steps in Drawing a Histogram 3 2 6 4 6 2 5 1 4 5
I. Choose a suitable scale for both vertical and 2 5 4 1 2 6 1 3 2 6
horizontal axes, when no scale is given. The 3 1 3 5 6 3 2 4 6 4
scale must be large enough to ensure that the 2 1 1 6 1 2 4 2 5 6
histogram covers at least half of the graph 1 6 6 2 1 3 4 2 5 1
sheet. Usually, scales using multiples of 5 or 10 Use the data to draw a histogram
are the best
Solution
II. Draw two axes, the vertical and horizontal
axes. Label the vertical axes for the frequencies Score Tally Frequency
and the horizontal according to the given data. 1 //////// 10
E.g. Ages (years), Marks Height (cm), not 2 //////// / 11
forgetting the units where applicable 3 //// / 6
4 //// // 7
5 //// / 6
III. The bars can be drawn in two ways
6 //////// 10
explained as follows:
a. Using the class boundaries, draw bars using
the lower and upper class boundaries

Baffour Ba Series, Further Mathematics for Schools Page 727


Frequency
16
14
12
10
8
6
4
2
0
1 2 3 4 5 6

Marks (%)

Histogram for a Grouped Data


Worked Examples
1. Below is a record of the marks (out of 50) obtained by 22 students in a test.
14 12 8 19 13 27 22 21 32 30 35
39 36 37 42 45 40 41 44 46 44 46
i. Prepare a group frequency table for the data using the intervals 5 – 9 , 10 – 14 …
ii. From the table, determine the mean mark.
iii. Draw a histogram to represent the data.

Solution
i.

Marks Class Boundaries Class Marks (x) Frequency (f) fx


5–9 4.5 – 9.5 7 1 7
10 – 14 9.5 – 14.5 12 3 36
15 – 19 14.5 – 19.5 17 1 17
20 – 24 19.5 – 24.5 22 2 44
25 – 29 24.5 – 29.5 27 1 27
30 – 34 29.5 – 34.5 32 2 64
35 – 39 34.5 – 39.5 37 4 148
40 – 44 39.5 – 44.5 42 5 210
45 – 49 44.5 – 49.5 47 3 141
= 22 = 694

Baffour Ba Series, Further Mathematics for Schools Page 728


ii. Mean ( ̅ ) = = = 31.55

frequency

4.5 9.5 14.5 19.5 24.5 29.5 34.5 39.5 44.5 49.5
Marks

2. Copy and complete the table below and use it Solution


to draw a histogram using: Using class boundaries
i. the class boundaries;
ii. the class marks.
Ages Class
(yrs) boundaries Frequency
Ages (years) Frequency 5–9 4.5 – 9.5 3
5–9 3 10 – 14 9.5 – 14.5 2
10 – 14 2 15 – 19 14.5 – 19.5 4
15 – 19 4 20 – 24 19.5 – 24.5 5
20 – 24 5 25 – 29 24.5 – 29.5 3
25 – 29 3 30 – 34 29.5 – 34.5 1
30 – 34 1

Baffour Ba Series, Further Mathematics for Schools Page 729


5

Frequency
4

0
4.5 9.5 14.5 19.5 24.5 29.5 34.5 Marks (%)

ii. Using the class mark

Ages
(yrs) Class Mark Frequency
5–9 7 3
10 – 14 12 2
15 – 19 17 4
20 – 24 22 5
25 – 29 27 3
30 – 34 32 1

Baffour Ba Series, Further Mathematics for Schools Page 730


5

Frequency
4

0
7 12 17 22 27 32 Marks (%)

Estimating the Mode from a Histogram b. Draw another straight line from the top left
The class with the highest frequency is called corner of the tallest bar to the left corner of the
the modal class. The actual mode is contained bar to the right.
in the modal class.
III. Draw a perpendicular from the point of
The mode of can be estimated from the intersection of the diagonals onto the horizontal
histogram by following the steps below: axis.
I. Draw the histogram for the data.
IV. Read off the value of the point where the
II. Draw two diagonals from the top corners of perpendicular touches the horizontal axis, as
the adjacent bars the mode.
a. Carefully draw a straight line from the top
right corner of the tallest bar to the top right Worked Examples
corner of the bar on the left 1. Determine the mode of the histogram below:

Baffour Ba Series, Further Mathematics for Schools Page 731


Frequency
40
35
30
25
20
15
10
5
0
35.5 45.5 55.5 65.5 75.5 85.5 95.5

Marks (%)

Solution Estimating the Median from a Histogram


The modal class has the tallest bar The Median can be estimated from the
= (55.5 – 65.5) histogram by carefully following the steps
below:
I. Draw the histogram for the distribution
Frequency

II. Find the sum of the areas of all the


40 rectangles
35
30 III. Calculate half of the sum of the areas of all
25 the rectangles and locate it vertically on the
20 histogram
15
10 IV. If the vertical line falls exactly on a
boundary, then read off the boundary value as
0 the median. However, if the vertical line does
35.5 45.5 55.5 65.5 75.5 85.5 95.5
not fall on a boundary, then calculate the area
Marks (%) of the additional strip and add it to the area up
to the boundary value just on the left, to obtain
From the histogram, the modal mark = 55.9 the median.

Baffour Ba Series, Further Mathematics for Schools Page 732


Worked Examples
Determine the median of the histogram.

Frequency
5

0
4.5 9.5 14.5 19.5 24.5 29.5 34.5 Marks (%)

Solution
Area of each bar = L × B,
B = (9.5 – 4.5) = 5
A1 = 3 × 5 = 15 A4 = 5 × 5 = 25
A2 = 2 × 5 = 10 A5 = 3 × 5 = 15
A3 = 4 × 5 = 20 A6 = 1 × 5 = 5
Total area = 15 + 10 + 20 + 25 +15 + 5 = 90
Half of total area = = 45
Median class = (14.5 – 19.5) and (19.5 – 24.5)

Baffour Ba Series, Further Mathematics for Schools Page 733


Frequency
5

A1 A2 A3 A4 A5 A6
0
4.5 9.5 14.5 19.5 24.5 29.5 34.5 Marks (%)

Median = 19.5 (shown on the graph)

2. What is the median of the histogram below?


Frequency

16
14
12
10
8
6
4
2
0
0.5 5.5 10.5 15.5 20.5 25.5 30.5 35.5 40.5

Marks (%)

Baffour Ba Series, Further Mathematics for Schools Page 734


Solution
Area of each bar = L × B,
B = (5.5 – 0.5) = 5
A1 = 2 × 5= 10 A5 = 12 × 5 = 60
A2 = 3 × 5 =15 A6 = 7 × 5 = 35
A3 = 6 × 5 = 30 A7 = 4 × 5 = 20
A4 = 15 × 5 = 75 A8 = 2 × 5 = 10
Total area = 10 + 15 + 30 + 70 + 60 + 35 + 20 + 10 = 255
Half of total area = = 127.5
Median class = (15.5 – 20.5) meaning after 15.5 but not up to 20.5
Frequency

16
14
12
10
8
6
4
2
x
0
0.5 5.5 10.5 15.5 20.5 25.5 35.5 40.5

Marks (%)

From the histogram, area of the shaded region or rectangle = 15 × x = 15x


Half the area = A1 + A2 + A3 + area of shaded region = 127.5
10 + 15 + 30 + 15x = 127.5
15x = 127.5 – 10 – 15 – 30
15x = 72.5
x= = 4.8
Estimated median = 15.5 + 4.8 = 20.3

Baffour Ba Series, Further Mathematics for Schools Page 735


2. What is the median of the histogram below? B = (8 – 3 ) = 5
A1 = 2 × 5 = 10 A5 = 12 × 5 = 60
A2 = 3 × 5 =15 A6 = 7 × 5 = 35
A3 = 6 × 5 = 30 A7 = 4 × 5 = 20
Frequency

A4 = 15 × 5 = 75 A8 = 2 × 5 = 10

16 Total area
14
= 10 + 15 + 30 + 70 + 60 + 35 + 20 + 10
= 255
12
10
Half of total area = = 127.5
8
6
Median class = (18)
4
2
0
Class boundary = = 2.5
3 8 13 18 23 28 33 38

Marks (%) Lower class boundary of the median class;


= 13 + 2.5
Solution = 15. 5

From the histogram, area of the shaded region


Frequency

or rectangle = 15 × x = 15x

Half the area = A1 + A2 + A3 + area of shaded


16 region = 127.5
14 10 + 15 + 30 + 15x = 127.5
12 15x = 127.5 – 10 – 15 – 30
10 15x = 72.5
8 x=
6 x = 4.8
4 Estimated median = Lower class boundary + x
2 = 15.5 + 4.8
0 x = 20.3
3 8 13 18 23 28 33 38

Marks (%) Exercises 10.15


1. The table below show the weight of potatoes
Area of each bar = L × B, obtained from a number of plants.

Baffour Ba Series, Further Mathematics for Schools Page 736


Weight per plant Frequency 4. The following table shows the marks
0–2 3 distribution of 25 students in a mathematics
2–4 8 examination.
4–6 21
6–8 27
Marks Frequency
8 – 10 19
0–9 4
10 – 12 12
10 – 19 5
12 – 14 8
20 – 29 2
14 – 16 2
30 – 39 3
40 – 49 6
i. Draw a histogram of this distribution. 50 – 59 3
ii. Determine the mode and state the modal 60 – 69 2
class.
Draw a histogram for the distribution.
2. In a traffic survey, the number of vehicles Use your histogram to estimate;
passing along a given road in a period of 5 i. the mode
minutes was recorded on a number of different ii. the median of the distribution.
occasions. The results were as follows; iii. iii. the upper and lower quartiles.

No. of 5. A group of students were asked to say the 12


vehicles 8 9 10 11 12 13 14 – times table as fast as possible. The time taken
6 12 20 16 7 4 2 by each student was recorded. The resulst are
shown in the table below:
i. Draw a histogram of this distribution and
calculate the mean. Time (s) Frequency
ii. Draw the cumulative frequency curve and 0 < t ≤ 10 1
10 < t ≤ 20 2
from it, obtain the median of the distribution.
20 < t ≤ 30 8
30 < t ≤ 40 12
3. The table below gives the distribution of 120 40 < t ≤ 50 6
students in an institution. 50 < t ≤ 60 3

Age 16 - 19 – 22 – 25 – 28 – Draw a histogram for these data.


(years) 18 21 24 27 30
No. of 12 36 40 24 8 6. The marks obtained by candidates in a
students mathematics examination were first grouped
0 – 9, 10 – 19, 20 – 29, 30 – 39 and so on.
a. Draw a histogram for the distribution. The midpoint of the mark of each group was
b. Use your histogram to estimate correct to the taken as the mark representing the group. The
nearest whole number the mode and the following table gives the distribution of the
median. marks

Baffour Ba Series, Further Mathematics for Schools Page 737


Midpoint 4 .5 14.5 24.5 34.5 44.5
Freq. 18 19 x 12 9

Cumm. Freq.
5
Midpoint 54.5 64.5 74.5 84.5
Freq. 5 2 2 1 4

If the mean mark for the candidate was found


3
to be 26.06;
i. determine the value of x
ii. draw a histogram for the distribution 2

iii. find the probability that a candidate chosen


at random obtained 55 marks or more 1

Preparing a Frequency Distribution Table


4.5 9.5 14.5 19.5 24.5 29.5 Ages (yrs)
for a Given Histogram with Equal Interval
A. Class Boundary Histogram
I. On the vertical axis, identify the height of Solution
each bar as the frequency. I. Frequencies = Height of bars = 3, 5, 2, 4, 1
II. Class boundaries = 4.5 – 9.5, 9.5 – 14.5,
II. For each bar, identify the first and second 14.5 – 19.5, 19.5 – 24.5, 24.5 – 29.5
numbers on the horizontal axis as the lower III. Add 0.5 to lower class boundaries and
class boundary and upper class boundary subtract 0.5 from upper class boundaries of the
respectively, and by this technique, generate all same group to generate the class size.
the class boundaries. ⇒5 – 9, 10 – 14, 15 – 19, 20 – 24, 25 – 29

III. For each class boundaries, add 0.5 to the Class Class
lower class boundary and subtract 0.5 from the Size Boundary Frequencies
upper class boundaries to generate each class 5–9 4.5 – 9.5 3
width or intervals. 10 – 14 9.5 – 14.5 5
15 – 19 14.5 – 19.5 2
Worked Examples 20 – 24 19.5 – 24.5 4
Prepare a frequency distribution table for the 25 – 29 24.5 – 29.5 1
histogram below;

Baffour Ba Series, Further Mathematics for Schools Page 738


B. Class Mark/Mid-point Histogram Solution
I. On the vertical axis, identify the height of I. Frequencies = Height of bars = 3, 5, 2, 4, 1
each bar as the frequency II. Difference between the first and second
class marks = 12 – 7 = 5, the = 2.5
II. On the horizontal axis, find the common
III. Subtract and add 2.5 from and to each class
difference (d) between the class marks, and
mark to obtain the class boundaries
divide by two to obtain an answer, say A.
7 – 2.5 = 4.5 and 7 + 2.5 = 9.5
⇒ =A (4.5 – 9.5)
12 – 2.5 = 9.5 and 12 + 2.5 = 14.5
III. Subtract and add the answer obtained in act (9.5 – 14.5)
II, that is A, from and to each class mark to 17 – 2.5 = 14.5 and 17 + 2.5 = 19.5
obtain the lower class boundary and the upper (14.5 – 19.5)
class boundary respectively of that class 22 – 2.5 = 19.5 and 22 + 2.5 = 24.5
(19.5 – 24.5)
IV. For each class boundaries, add 0.5 to the 27 – 2.5 = 24.5 and 27 + 2.5 = 29.5
lower class boundary and subtract 0.5 from the (24.5 – 29.5)
upper class boundaries to generate each class
width or intervals. IV. Add 0.5 to the lower class boundary and
subtract 0.5 from the upper class boundaries
Worked Example (5 – 9) (10 – 14), (15 – 19), (20 – 24)
(25 – 29) to obtain class size/width

Class Class
Cumm. Freq.

5
Size Boundary Frequencies
5–9 4.5 – 9.5 3
4
10 – 14 9.5 – 14.5 5
15 – 19 14.5 – 19.5 2
3
20 – 24 19.5 – 24.5 4
25 – 29 24.5 – 29.5 1
2

7 12 17 22 27 Ages (yrs)

Baffour Ba Series, Further Mathematics for Schools Page 739


Frequency Distribution Table for a Given Histogram with Unequal Interval
Given a data and unequal class interval, complete a frequency table as shown below:

Interval Class Class Width Frequency Frequency


boundaries Density

The formula for finding the frequency density The ages of children entering a playground in a
is given by: 1 hour period is recorded as shown below;
Frequency Density = =
0–3 4 – 10 11 – >
Age 18 18
The class frequency of a histogram is basically Frequency 12 14 48 0
the width of the group.
When drawing the histogram for unequal Draw a histogram to represent the data.
intervals:
1. The frequency density goes along the y – Solution
axis. The class boundaries are; 0, 4, 11 and 19
2. The class width goes along the x – axis.
3. The area of each bar represents the Class width;
frequency. 4 – 0 = 4, 11 – 4 = 7 , 19 – 11 = 8

Worked Examples Frequency densities;


= 3, = 2, = 6.

Intervals Class Class Width Frequency Frequency


boundaries (c) (f) Densities . /
0–3 0 4 12 3
4 – 10 4 7 14 2
11 – 18 11 8 48 6
> 18 19 0 0 0

Baffour Ba Series, Further Mathematics for Schools Page 740


6

Frequency Density
5

2 4 6 8 10 12 14 16 18 20
Ages

2. The following table shows the ages of 25 children on a school bus.

Age (years) 5 – 10 11 – 15 16 – 17 > 17


Frequency 6 15 4 0

i. Draw a histogram to represent the data.


ii. How many children are between 16 and 17 years.

Solution

Intervals Class Class Width Frequency Frequency


Boundaries (c) (f) Densities . /
5 – 10 5 6 6 1
11 – 15 11 5 15 3
16 – 17 16 2 4 2
> 17 18 0 0 0

Baffour Ba Series, Further Mathematics for Schools Page 741


3

Frequency Density 2

4 6 8 10 12 14 16 18 20
Ages

ii. Area = frequency density × class width


Area = 2 × 2
Area = 4.
Therefore, there are 4 children between the ages 16 and 17

3. The table below shows the mark distribution in a test.

Marks 11 – 20 21 – 35 36 – 40 41 – 50 51 – 60
No of candidates 5 7 10 16 12

Draw a histogram for the distribution.

Solution

Marks Class Class size Frequncy Frequency density


boundaries (c) (f) . /
11 – 20 10.5 – 20.5 10 5 0.5
21 – 35 20.5 – 35.5 15 7 0.47
36 – 40 35.5 – 40.5 5 10 2.0
41 – 50 40.5 – 50.5 10 16 1.6
51 – 60 50.5 – 60.5 10 12 1.2

Baffour Ba Series, Further Mathematics for Schools Page 742


Frequency density

2.0

1.5

1.0

0.5

10.5 20.5 30.5 40.5 50.5 60.5


Marks

Exercises 24.4 The results are shown in the frqeuncy table


1. The following table below shows the below.
distribution of the monthly incomes of 100
workers. Distance (cm ) Frequency
0<d≤5 3
Monthly Number of 5<d≤7 5
income workers 7<d≤8 4
125 – 129 10 8<d≤9 6
130 – 139 32 9 < d ≤ 10 3
140 – 144 18 10 < d ≤ 15 6
145 – 154 28 15 < d ≤ 25 6
155 – 159 12
Constuct a histogram to display this
Draw a histogram for the distribution. information.

2. Mr. Brown conducted an experiment to see 3. The length of 62 different songs are shown
how far 33 snails would move in 10 minutes. in the frequency table below:

Baffour Ba Series, Further Mathematics for Schools Page 743


Length (s ) Frequency a. Draw a histogram to display these data.
0 < L ≤ 100 5 b. Use your histogram as a guide to draw a
100 < L ≤ 180 8 frequency polygon for these data.
180 < L ≤ 210 12
210 < L ≤ 240 15 Preparing a Frequency Distribution Table
240 < L ≤ 300 12 for a Given Histogram with Unequal Interval
300 < L ≤ 500 10 I. On the vertical axis, identify the height of
each bar as the frequency density.
Draw a histogram to display these data.
II. For each bar, identify the first and second
4. The height in centimeters of 30 students numbers on the horizontal axis as the lower
were recorded and shown in the table below: class size and upper class sizes respectively,
and by this technique, generate the size of each
Height (cm ) Frequency class.
140.5 < h ≤ 150.5 5
150.5< h ≤ 155.5 8 III. For each class size subtract 0.5 from the
lower class size and add 0.5 to the upper class
155.5 < h ≤ 160.5 6
size to generate each class boundary.
160.5 < h ≤ 165.5 3
165.5 < h ≤ 180.5 6
IV. From the relation;
Frequency = frequency density × class size,
Draw a histogram to display these data.
calculate the frequency of each bar/ class.
5. The exact ages (in years) of guests at a
V. Copy and complete the table with the
fancy dress party were recorded as shown in
required details
the table below;
class Class Class F F. D
Age (years) Frequency
boundaries Width
15 < A≤ 20 5
20 < A ≤ 23 15
23 < A ≤ 25 20
25 < A ≤ 30 20
30 < A ≤ 40 30

Baffour Ba Series, Further Mathematics for Schools Page 744


Worked Example
The histogram below represent the marks scored by some studnts in a test.

Frequency Density

10 30 40 50 60 70 80 90 20
Marks

Use the histogram to:


i. construct a frequency table.
ii. estimate the total number of students who took the test.

Baffour Ba Series, Further Mathematics for Schools Page 745


Solution
i. From the given histogram;

Class = 0 – 30 , 30 – 40 , 40 – 50 , 50 – 60 , 60 – 65, 65 – 75, 75 – 90


Class width = 30, 10, 10, 10, 5, 10, 15
Frequency density = 0.1, 1.5, 4.2, 5.1 , 8.6, 4.5, 1.2

But frequency density =


Frequency = frequency density × class size
= 0.1 × 30 = 3 = 8.6 × 5 = 43
= 1.5 × 10 = 15 = 4.5 × 10 = 45
= 4.2 × 10 = 42 = 1.2 × 15 = 18
= 5.1 × 10 = 51

Marks Class Class size Frequency Frequency density


boundaries (c) (f) . /
0 – 30 0.5 – 30.5 30 3 0.1
30 – 40 30.5 – 40.5 10 15 1.5
40 – 50 40.5 – 50.5 10 42 4.2
40 – 60 40.5 – 60.5 10 51 5.1
60 – 65 60.5 – 65.5 5 43 8.6
65 – 75 65.5 – 75.5 10 45 4.5
75 – 90 75.5 – 90.5 15 18 1.2

ii. Total number of students who took the test


= 3 + 15 + 42 + 51 + 43 + 45 + 18
= 217

Baffour Ba Series, Further Mathematics for Schools Page 746


Exercises 24.5
The histogram below shows the time in seconds taken for some people to climb up four flight of
stairs.
i. Draw a frquenct table for the histogram.
ii. How many people climb up the stairs.
iii. Estimate how many people took between 55 and 65 seconds to climb up the stairs.
Frequency density

10
14
12
10
8
6
4
2

20 40 60 80 100 120 140


Time (seconds)

Measures of Positions values for 25%, 50% and 75% of the data. Q2
Measures of positions are techniques that coincides with the median.
divide a set of data into equal groups. The
different measures of positions are quartiles, Determining Quartiles
deciles and percentiles I. Order or arrange the data from smallest to
largest
To determine measurement of position, the data II. Find the place that occupies every quartile,
must be sorted from the lowest to the highest. using the expression, , where k = 1, 2, 3, for
Q1, Q2, and Q3 respectively
Quartiles
The quartiles are the three values of the
Quartiles of Odd Number of Data
variable that divide an ordered data set into
Quartiles of odd number of data are determined
four equal parts.Q1(lower quartile), Q2(middle ( )
quartile) and Q3(upper quartile) determine the by the formula, , where k = 1, 2, 3 for

Baffour Ba Series, Further Mathematics for Schools Page 747


Q1, Q2, and Q3 respectively II. Identify the average of e and f as the median
( ) ( ) ( ) or Q2.
Q1= , Q2 = , Q3 = ,
a, b, c, d, e, f, g, h, k, l
Worked Example
Find the lower, middle and upper quartiles of
a, b, c, d, e f g, h, k, l
the data 2, 5, 9, 3, 2, 6, 4

Solution Q2 =
Let the lower quartile be Q1, the median be Q2
and the upper quartile be Q3 III. If the data from a to e is an odd number,
Ordering the data: 2, 2, 3, 4, 5, 6, 9 identify the middle number of a to e as the first
n=7 quartile, Q1
( )
Q1= = = 2nd position
a, b, c, d, e, f, g, h, k, l
2, 2, 3, 4, 5, 6, 9

Q1
Q1 = 2
IV. If the data from f to l is an odd number,
( ) th
Q2 = = = 4 position identify the middle number of f to l as the third
2, 2, 3, 4, 5, 6, 9 quartile, Q3

Q2 = 4 a, b, c, d, e, f, g, h, k, l

( ) Q1 Q2
Q3 = = = 6th position
2, 2, 3, 4, 5, 6, 9 Case 2
↓ Given a, b, c, e, f, g, h, k, if the data from a to e
Q3 = 6 and f to k are even numbers, identify the
average of the two middle numbers from a to e
Quartiles of Even Number of Data as Q1 and the average of the two middle
Quartiles of even number of data, for example numbers from f to k as Q3.
a, b, c, d, e, f, g, h, k, l are determined by the
following procedure:
a , b, c, e, f, g, h, k,

Case 1
I. Order the data and divide the data into two Q1 = Q2 = Q1 =
equal parts as shown below:

Baffour Ba Series, Further Mathematics for Schools Page 748


Using the Formula = 6th and 7th positions
For even number of data, two positions are Q2 = = 5.5
required for each quartile. Thus:
Q1 = (n)th and (n)th + 1
Q3 = (12)th and (12)th + 1
Q2 = (n)th and (n)th + 1 = 9th and 10th positions

Q3 = (n)th and (n)th + 1


Q3= = 7.5

Use the positions to identify the two numbers 2. Find Q1, Q2 and Q3 of the data; 2, 7, 6, 14,
and divide the sum by two to obtain the value 7, 11, 9, 3, 4, 4, 8
of the quartile.
Solution
Worked Examples 2, 3, 4, 4, 6, 7, 8, 9, 11, 14
1. Determine the quartiles of the data; 2, 2, 5, 6, n = 10
7, 7, 9, 3, 3, 4, 8, 9 Q1 position = (10)th and (10)th + 1
Q1 position = 2.5th and 3.5thpositions
Solution
Method 1 Q1 position = . /
Ascending order: = 3rd position
2 2 3 3 4 5 6 7 7 8 99 Q1 = 4

Q2 position = (10)th and (10)th + 1


Q1 Q2 Q3
= 5th and 6th positions
Q1 = =3
= 6 and 7
Q2 = = 5.5 Q2 = = 6.5
Q3 = = 7.5
Q3 position = (10)th and (10)th + 1
Method 2
Ascending order : Q3 position = 7.5th and 8.5th
2 2 3 3 4 5 6 7 7 8 99
Q3 position = . /
n = 12
Q1 = (12)th and (12)th + 1 Q3 position = 8th position
Q3 = 9
= 3rd and 4th positions
Q1= =3 Deciles
The deciles (D) are the nine values of the
Q2 = (12)th and (12)th + 1 variable that divide an ordered set of data into

Baffour Ba Series, Further Mathematics for Schools Page 749


ten equal parts. The deciles determine the 2. From the data; 4, 1, 3, 12, 3, 2, 11, 6, 11, 10,
values for 10%, 20%, 30%...90%. The median 9, 10, 8, 7, determine the values of D3, D5 and
corresponds with D5. D8

Calculating Deciles of Raw Data Solution


I. Re- arrange the data in ascending order. 1, 2, 3, 3, 4, 6, 7, 8, 9, 10, 10, 11, 11, 12
II. Identify the number of entries as N. N = 14
III. Find the place or position that occupies D3 =
( )
= = 4.5th position
( )
every decile, using the relation, D = , ⇒D3 lies between the 4th and 5th positions
where k = 1, 2, 3…9 for D1, D2, D3 ….D9 D3 = = 3.5
respectively.
( )
Worked Examples D5 = = = 7.5th position
1. Find D4, D5 and D7 of the data : 6, 4, 7, 6, ⇒D5 lies between the 7th and 8th positions
11, 8, 5, 3, 2, 9, 10, 12, 3 D5 = = 7.5

Solution
( )
2, 3, 3, 4, 5, 6, 6, 7, 8, 9, 10, 11, 12 D8 = = = 12th position
N = 13 ⇒D8 = 11
( )
D4 = = = 5.6th position
Percentiles
It implies that D4 lies between the 5th and 6th
Percentiles are the 99 values of the variable that
positions. But 5thposition = 5 and 6th position =
divide an ordered data set into 100 equal parts
6
⇒ 4 = = 5.5 The percentiles determine the values for 1%,
2%, 3%...99% of the data. D1 = P10, D2 = P20…
( ) The median corresponds to P50
D5 = = = 7th position
⇒D5 = 6
Determining Percentiles
( )
I. Re- arrange the data in ascending order
D7 = = = 9.8th position II. Identify the number of entries as n
III. Find the place or position that occupies
It implies that D7 lies between the 9th and 10th
every percentile, using the relation, P =
positions. But 9th position = 7 and 10th position
=8 ( )
, where k =1, 2, 3…99 for P1, P2, P3 …

P99 respectively.
⇒D7 = = 8.5

Baffour Ba Series, Further Mathematics for Schools Page 750


Worked Examples Cumulative Frequency Table
1. Find P15 and P84 of the data : 6, 4, 7, 6, 11, 8, Consider the table below:
5, 3, 2, 9, 10, 12, 3
Marks (x) Frequency (f)
Solution 11 – 20 7
21 – 30 3
2, 3, 3, 4, 5, 6, 6, 7, 8, 9, 10, 11, 12
31 – 40 4
N = 13 41 – 50 6
( )
P15 = = 2.1st position 51 – 60 5

To draw up a cumulative frequency table,


⇒P15 lies between the second and third
identify the upper class boundaries of the class
positions. But second position = 3 and third
intervals. Thus, for the class interval 11 – 20,
position = 3
with frequency 7, it means that 7 students had a
P15 = =3 mark of 20.5(upper class boundary) or less. For
the class interval 21 – 30, consider students
( ) who had marks less than 30.5, and this is found
P84 = = 11.76 th position
by adding the frequency of the previous class to
⇒P84 lies on the 11th and 12th positions
that of itself. i.e. 7 + 3 = 10

P84 = = 10.5 The cumulative frequency table for the above


information is drawn as shown below:
Exercises 24.6
A. Find Q1, Q2 and Q3 from the data: Marks Frequency Cumulative
1. 2, 5, 8, 8, 3, 4, 5, 1 less than (f) frequency (f)
2. 8, 9, 1, 2, 7, 3, 2, 4, 7, 8, 20.5 7 7
30.5 3 7 + 3 = 10
3. 7, 11, 9, 2, 4, 2, 10, 4, 6
40.5 4 10 + 4 = 14
4. 10, 11, 14, 11, 18, 19, 11, 12, 13, 15, 16
50.5 6 14 + 6 = 20
60.5 5 20 + 5 = 25
B. Find D3, D5 and D8 of the following data:
1. 2, 5, 8, 8, 3, 4, 5, 1 Worked Examples
2. 7, 11, 9, 2, 4, 2, 10, 4, 6 1. The heights of 100 girls were measured to
3. 10, 11, 14, 11, 18, 19, 11, 12, 13, 15, 16 the nearest centimeter. The results were as
follows
C. Find P35, P50, and P70
1. 2, 5, 8, 8, 3, 4, 5, 1
Height (Cm) Frequency
2. 8, 9, 1, 2, 7, 3, 2, 4, 7, 8, 130 – 134 4
3. 7, 11, 9, 2, 4, 2, 10, 4, 6 135 – 139 7
4. 10, 11, 14, 11, 18, 19, 11, 12, 13, 15, 16 140 – 144 14
145 – 149 20

Baffour Ba Series, Further Mathematics for Schools Page 751


150 – 154 24 c. How many students obtained 69.5% or
155 – 159 16 more?
160 – 164 9
165 – 169 6 Solution
a.
a. Draw up a cumulative frequency table
Marks less Frequency Cumulative
b. How many girls were 149.5cm tall or more? than Frequency
29.5 1
Solution 39.5 4
49.5 7
Height Frequency Cumulative 59.5 22
less than Frequency 69.5 26
134.5 4 4 79.5 31
139.5 7 11 89.5 32
144.5 14 25
149.5 20 45 b. Students who obtained 49.5% or less?
154.5 24 69 =7
159.5 16 85 c. Students who obtained 69.5% or more?
164.5 9 94
32 – 26 = 6 students
169.5 6 100

Exercises 24.7
b. Number of girls who were 149.5cm tall or
1. The following table gives the masses of 100
more clients of a company to the nearest kilogram.
= Total girls – Number of girls who were not
149.5cm tall or more Mass Mass
(Nearest kg) Freq (Nearest kg) Freq
= 100 – 25 55 – 59 3 85 – 89 11
60 – 64 5 90 – 94 7
= 75
65 – 69 10 95 – 99 4
70 – 74 18 100 – 104 2
2. The following is the records of marks (%) 75 – 79 20 105 – 109 2
obtained by some students in a test 80 – 84 17 110 – 114 1
59 59 64 50 74 79 33 57
57 53 67 49 80 57 32 57 a. Draw up a cumulative frequency table.
76 48 74 24 56 50 56 50 b. Using the frequency table, find how many
60 39 52 57 30 61 73 40 clients had mass more than 79.5kg.
a. Using the class intervals of 20 – 29, 30 – 39,
40 – 49 …construct a cumulative frequency 2. The following table gives the masses of 160
table for the data babies born in a maternity home to the nearest
b. How many students obtained 49.5% or less? tenth of a kilometre.

Baffour Ba Series, Further Mathematics for Schools Page 752


Mass Frequency Worked Examples
1.5 – 1.9 5 1.The table below gives the distribution of
2.0 – 2.4 15 marks obtained by 33 candidates in an
2.5 – 2.9 60 examination.
3.0 – 3.4 54
3.5 – 3.9 25
Marks Frequency
4.0 – 4.4 1
11 – 20 1
21 - 30 1
a. Draw up a cumulative frequency table
31 – 40 2
b. Using the cumulative frequency table, find
41 – 50 4
how many babies had mass of less than 29.5kg 51 – 60 6
at birth. 61 – 70 7
c. What proportion of the total number of births 71 – 80 7
is this? 81 – 90 4
91 – 100 1
Cumulative Frequency Curve (Ogive)
Steps to follow when drawing the cumulative Draw a cumulative frequency curve for the
frequency curve: data.
I. Prepare a cumulative frequency table.
II. Draw two perpendicular axes on a graph Solution
sheet.
III. Label the two axes, (horizontal axis for Marks
upper class boundaries and vertical axis for Marks Less Frequency Cumulative
than Frequency
cumulative frequencies) 11 – 20 20.5 1 1
IV. Choose a suitable/ given scale and mark of 21 - 30 30.5 1 2
the values. 31 – 40 40.5 2 4
V. Plot the points from the cumulative 41 – 50 50.5 4 8
frequency table drawn up in step I 51 – 60 60.5 6 14
61 – 70 70.5 7 21
VI. Join the points with a smooth curve, but not 71 – 80 80.5 7 28
a ruler. 81 – 90 90.5 4 32
91 – 100 100.5 1 33

Baffour Ba Series, Further Mathematics for Schools Page 753


35

30

Cumulative frequency 25

20

15

10

0
10.5 20.5 30.5 40.5 50.5 60.5 70.5 80.5 90.5 100.5 110.5
Marks
2. A class of students obtained the following marks (%) in a test
59 59 64 50 74 79 33 57
57 53 67 49 80 57 52 57
76 48 74 24 56 50 56 50
60 39 52 57 30 61 73 40
Use this information to draw a cumulative frequency table and a cumulative frequency curve, using
the intervals 20 – 29, 30 – 39…

Solution

Marks Marks less than Frequency Cumulative frequency


20 – 29 29.5 1 1
30 – 39 39.5 3 4
40 – 49 49.5 3 7
50 – 59 59.5 15 22
60 – 69 69.5 4 26
70 – 79 79.5 5 31
80 – 89 89.5 1 32

Baffour Ba Series, Further Mathematics for Schools Page 754


35

30

frequency14N1 25

20
Cumulative

15

10

0
19.5 29.5 39.5 49.5 59.5 69.5 79.5 89.5
Marks

Exercises 26.7 Estimating the Quartiles from a Cumulative


1. Plot the cumulative frequency curve from the Frequency Curve
table below. The positions of quartiles on a cumulative
frequency curve involving N observations are
Area (cm2) Cum. frequency determined as follows:
A ≤ 10 19 Q1 = , Q2 = and Q3 =
A ≤ 20 39
A ≤ 30 47
A ≤ 40 48 The median Q2 is estimated on the cumulative
A ≤ 50 48 frequency curve as follows:
I. Find . /
2. a. Complete the cumulative frequency table
below and plot the curve. II. Look for this value on the vertical axis (the
cumulative frequency axis)
III. Draw a line perpendicular to the vertical
Mass (kg) Frequency
50 < m ≤ 60 1 axis at mark and extend it till it meets the
60 < m ≤ 70 11 cumulative frequency curve
70 < m ≤ 80 28 IV. Draw a line from the point of intersection
80 < m ≤ 90 39 with the curve perpendicular to the horizontal
90 < m ≤ 100 21
axis

Baffour Ba Series, Further Mathematics for Schools Page 755


V. Read off the corresponding value on the 46 33 09 75 59 46 71 05
horizontal axis as Q2 63 19 50 47 42 47 28 21
This same procedure is used to estimate the a. Construct a cumulative frequency table for
values of Q1 and Q3 the distribution using the class intervals 1 – 10,
Note that Q1 corresponds to the 25th percentile 11 – 20, 21 – 30 …
and Q3 corresponds to the 75th percentile b. Draw a cumulative frequency curve for the
distribution
Worked Examples c. Use your graph to find:
1. The following are the marks obtained by i. the median
some students in an achievement test ii. the lower quartile
90 25 31 35 52 50 48 15 ii. the upper quartile
19 40 60 83 23 38 40 70 iv. the marks obtained by a student who was
55 65 43 68 58 57 62 83 15th in the class

Solution
a.
Marks Marks less than Frequency Cumulative frequency
1 – 10 10.5 2 2
11 – 20 20.5 3 5
21 – 30 30.5 4 9
31 – 40 40.5 6 15
41 – 50 50.5 9 24
51 – 60 60.5 6 30
61 – 70 70.5 5 35
71 – 80 80.5 2 37
81 – 90 90.5 3 40

Baffour Ba Series, Further Mathematics for Schools Page 756


b.
45

40

35
Cumulative frequency

30

25

20

Upper quartile
15

10 Lower quartile

Median
5

0
0.5 10.5 20.5 30.5 40.5 50.5 60.5 70.5 80.5 90.5 100.5
Marks

c. From the cumulative frequency curve, Exercises


1. The table below shows the distribution of
i. Median (Q2) = . /
marks obtained by 100 students in a test.
But n = 40
Q1 = . / = 20th position = 46.5 Marks 0 – 9 10 – 20 – 30 – 40 –
19 29 39 49
Frequency 3 7 12 26 19
ii. Lower quartile (Q1) = . / Marks 50 – 60 – 70 – 80 – 90 –
59 69 79 89 99
But n = 40 Frequency 13 8 7 3 2
Q1 = . / = 10th position = 32.5
a. Construct a cumulative frequency table and
draw and use it to draw a cumulative frequency
iii. Upper quartile (Q3) = . / curve.
But n = 40 b. Use your graph to find:
( ) i. the pass mark if 40% of the students failed
Q1 = . / = 30th position = 60.5
the test
ii. the semi – interquatile range.
iv. From the graph, the marks obtained by a iii. the probability of selecting a candidate who
student who was 15th in the class is 40.5 scorrd between 43 and 54 marks;

Baffour Ba Series, Further Mathematics for Schools Page 757


2. The table below shows the distribution of Marks 0 – 9 10 – 20 – 30 – 40 –
marks obtained by some students in a test. 19 29 39 49
Frequency 4 6 10 10 13
Marks 0 – 9 10 – 20 – 30 – 40 – Marks 50 – 60 – 70 – 80 – 90 –
19 29 39 49 59 69 79 89 99
Frequency 1 3 5 3 10 Frequency 17 20 15 4 1
Marks 50 – 60 – 70 – 80 – 90 –
59 69 79 89 99 a. Construct a cumulative frequency curve of
Frequency 20 25 18 10 5 the distribution.
b. From your curve, estimate:
a. Construct a cumulative frequency table and i. the 80% percentile;
draw ana ogive. ii. the median;
b. Use your graph to determine; iii. the interquartile range;
i. the semi – interquatile range.
ii. the number of students that failed, if the pass 5. The following table gives the distribution of
mak for the test is 42; marks obtained by 150 candidates in an
iii. the probability that a student selected at examination in mathematics.
random scored between 20 % and 60 %.
Marks Frequency
3. The table below shows the division by age of 0 – 24 15
a village‟s population. 25 – 49 22
50 – 74 41
75 – 99 38
Age Frequency
100 – 124 24
Under 10 65
125 – 149 9
10 - 19 65
150 – 199 1
20 – 29 61
30 – 39 60
40 – 49 42 Draw a cumulative frequency curve for the
50 – 59 21 distribution and use it to estimate ;
60 and over 8 i. the median
ii. the number of candidates who
i. Form a cumulative frequency table and draw passed if the pass mark was 76
the ogive. iii. the number of candidates who
ii. Use your graph to estimate the median and obtained distinction if the minimum
the quartiles. mark for this grade is 145

4. The tablebelow gives the distribution of 6. The table below shows the distribution of
marksobtained by some students in an marks of candidates in an examination.
examination.
Baffour Ba Series, Further Mathematics for Schools Page 758
21 – 30 27
Marks Marks Freq. Cumulative 31 – 40 37
less than frequency 41 – 50 63
0–9 9.5 4
51 – 60 78
10 – 19 19.5 7
20 – 29 29.5 5
61 – 70 108
30 – 39 10 71 – 80 78
40 – 49 13 81 – 90 56
50 – 59 20 91 – 80 27
60 – 69 15
70 – 79 13
i. Draw a cumulative frequency diagram for the
80 – 89 3
90 – 99 1 distribution.
ii. Use your diagram to estimate
a. Copy and complete the table. a. the number of students who scored
b. Draw a cumulative frequency curve for the more than 65 marks.
distribution. b. the pass mark, if 60% of the candidates
passed
7. The table below show the weight of potatoes c. the interquartile range
obtained from a number of plants.
8. A class of students obtained the following
Weight per plant Frequency marks (%) in a test;
0–2 3 28 35 41 47 51 54 56 57 58 60 61
2–4 8 62 62 64 64 66 67 68 69 70 70 72
4–6 21 73 74 75 76 77 78 80 81 83 88 90
6–8 27 94
8 – 10 19 i. Form a grouped frequency table for
10 – 12 12 this data and draw the ogive. Use
12 – 14 8
the classes 20 – 29, 30 – 39 …
14 – 16 2
ii. Estimate the number of students
who failed if the pass mark was
i. Plot the cumulative frequency curve and from
45%
it obtain the median of the distribution.

Mean Deviation of a Raw Data


8. The marks of 500 candidates in a test
The mean deviation or average deviation is the
showed the following distribution;
arithmetic mean of the absolute deviations and
it is denoted by ̅ . Thus:
Marks Number of ̅ ̅ ̅
Candidates ̅ =
0 – 10 8
11 – 20 18

Baffour Ba Series, Further Mathematics for Schools Page 759


The steps involved in finding the mean 2. Find the mean deviation of the following set of
deviation about the mean are as follows: numbers: 3, 4, 4, 4, 5, 8, 8, 9
I. Find the mean of the given data
II. Subtract the mean from each of the 3. For the set of numbers 11, 14, 14, 13, 15, 20,
observations and record your results. 17, determine the mean deviation
III. Find absolute values of the deviations
obtained. Mean Deviation of a Grouped Data
IV. Add the absolute values obtained. To find the mean deviation ( ̅ ) of a grouped
V. Divide the results of the last step by the data,
number of observations to obtain the mean I. Prepare and complete a table such as the one
deviation of the data. below:

Worked Examples Class x f fx d=| ̅| fd


Find the mean deviation about the mean for the
following set of data: 6, 7, 10, 12, 13, 4, 8, 12

Solution
Data = 6, 7, 10, 12, 13, 4, 8, 12 = = =
Mean = = =9

II. From the table, determine the mean by the


Deviations from the mean;
formula:
= 6 – 9, 7 – 9 , 10 – 9 , 12 – 9 , 13 – 9 , 4 – 9 , 8 –
9 , 12 – 9 Mean ( ̅ ) =
= -3, -2, 1, 3, 4 -5, -1, 3 III. Calculate the mean deviation by the
formula: ̅ =
Absolute deviations;
= /-3/, /-2/, /1/, /3/, /4/, /-5/, /-1/, / 3/ IV. Leave the answer to two decimal places
= 3, 2, 1, 3, 4, 5, 1, 3 unless stated

Sum of absolute deviations; Worked Example


= 3 + 2 + 1 + 3 + 4 + 5 + 1 + 3 = 22 The marks obtained by 20 students in a test
(marked out of 20) were recorded as follows;
Mean deviation about the mean = = 2.75 12 9 14 13 2 9 14 5 12 7
18 13 6 10 8 10 15 3 12 16
Exercises 24.8 i. Construct a group frequency table for the
1. Calculate the mean deviation of the following data, using the class intervals 1 – 5, 6 – 10, 11
distribution: 9, 3, 8, 8, 9, 8, 9, 18 – 15 …
ii. Use your table to calculate the mean
deviation of the distribution.
Solution

Baffour Ba Series, Further Mathematics for Schools Page 760


Class x f fx d=| ̅| fd
1–5 3 3 9 7.25 21.75
6 – 10 8 7 56 2.25 15.75
11 – 15 13 8 104 2.75 22
16 – 20 18 2 36 7.75 15.5
= 20 = 205 75

( ̅)= = = 10.25

Mean deviation, ̅ = = = 3.75(2.d.p)

Exercises 24.9 Interval 20 30 40


1. The following data represents the age Frequency 8 12 10 8
distribution of 100 people covered by health Class 40 – 50 – 60 –
insurance. Interval 50 60 70
Frequency 3 2 7
Age 25 – 34 35 – 4 4 45 – 54 55 – 64
Number 23 29 28 20 4. The following table shows the distribution of
the number of hours worked each week (on
i. Find the mean deviation about the mean average) for a sample of 100 community
ii. What is the probability of selecting a student college students.
whose age is more that 54?
Hour per Number of
2. From the table below, calculate the mean week students
deviation from the mean. 5, 0–9 24
10 – 19 14
Class 2–4 4–6 6–8 8 – 10 20 – 29 39
Interval 30 – 39 18
Frequqncy 5 6 3 1 40 – 49 5

3. Find the mean deviation about the mean Calculate the mean deviation of the
from the data below. MD = 16 distribution.
5. The heights of 25 boys are measured to the
Class 0 – 10 10 – 20 – 30 – nearest cm and are then grouped as follows:

Baffour Ba Series, Further Mathematics for Schools Page 761


Height(cm) Frequency interquartile range, divide the inter quartile
151 - 155 4 range by 2. i.e.
156 – 160 8
161 – 165 7
166 – 170 5 Worked Examples
171 – 175 1 The height of 15 boys to the nearest cm is
given as: 138, 136, 143, 143, 147, 158, 146,
Calculate the mean deviation from the mean. 145, 137, 135, 143, 140, 129, 156, 149
From these data, find:
Measures of Dispersion i. the range
Measures of dispersion indicate the degree of ii. the quartiles,
„spread‟ of the data. The most common iii. the inter quartile range,
statistics used as measures of dispersion are the iv. the semi – interquartile range.
range, the interquartile range, variance and
standard deviation Solution
i. Arranging the data in order of size,
The Range 129, 135, 136, 137, 138, 140, 143, 143, 143,
The range of the numbers in a group of data is 145, 146, 147, 149, 156, 158
the difference between the greatest number in The range 158 – 129 = 30
the data and the least number in the data. For
example, given the list 11, 10, 5, 13, 21, the ii. n = 15
range of the numbers is 21 – 5 = 16 Q1 =
( )
= = 4th position = 137cm
( )
Worked Examples Q2 = = = 8th position = 143cm
The marks of 8 students in a test are 10, 4, 5, 3, ( )
Q3 = = = 12th position = 147cm
14, 13, 16 and 7. Find the range of the marks.

Solution iii. Interquartile range


Range = Biggest – smallest = Q3 – Q1 = 147 – 137 = 10
Range = 16 – 3 = 13
iv. Semi – interquartile range
The Interquartile Range = = =5
The interquartile range is defined as the
difference between the third quartile and the Exercises 24.10
first quartile. i. e. Q3 – Q1. Thus, the 1. For the data:11, 11, 13, 15, 17, 19, 23, 31,
interquartile range measures the spread of the 45, 47, 49, find the range and the inter quartile
middle half of the data range
Semi - Interquartile Range 2. Find the quartiles and the semi –
The semi – interquartile range is another interquartile range of the data, 5, 16, 5, 12, 8, 1,
measure of dispersion. To find the semi – 4, 16, 4, 11, 4, 6
Baffour Ba Series, Further Mathematics for Schools Page 762
1. Find the variance of the following scores in a
3. Find the range, the quartiles and the test: 17, 12, 14, 13, 19
interquartile range of the data30, 64, 49, 45, 30,
55, 47, 49 Solution
17, 12, 14, 13, 19
4. Find: n=5
i. the interquartile range
Mean ( ̅ ) = = = 15
ii. Semi – interquartile range of the following
data:
Variance
a. 4, 11, 19, 5, 6, 3, 8, 3, 5, 4, 10, 6, 19, 7, 26 ( ) ( ) ( ) ( ) ( )
b. 43, 34, 45, 31, 40, 22, 30, 17, 25, 18, 27 (σ2) =
( ) ( ) ( ) ( ) ( )
σ2 =
Variance
The variance is defined as a measure of how σ2 = = = 6.8
the data distributes itself about the mean. It is
computed as the average of the squared Exercises 24.11
differences from the mean. Determine the variance:
1. 9, 3, 8, 9, 8, 9, 18
To calculate the variance, follow the steps 2. 30, 64, 49, 45, 30,
below: 3.22, 30, 17, 25, 18, 27
I. Work out the mean.
II. Then for each number subtract the mean and Properties of Variance
square the results (the squared difference). 1. The variance is always positive or in the
III. Then work out the averages of those event that the values are equal, the variance is
squared differences. zero.
2. If all the values of the variables are added or
Note: reduced by the same number, the variance does
When you have n data values that are: not change.
1. The population: divide by n when 3. If all the values of the variables are
( ̅) multiplied by the same number, the variance is
calculating the variance. That is σ2 = ,
multiplied by the square of that number.
where ̅ is the mean and n is the number of
scores. Standard Deviation
2. A sample (a selection taken from a bigger Standard deviation is the measure of the extent
population): divide by n – 1 when calculating to which the given values or data are spread or
( ̅)
the variance. That is: σ2 = dispersed around the mean. It is the square root
In this work, all data is taken as the population of the variance.

The steps to follow in computing the standard


Worked Examples
deviation are as follows:
Baffour Ba Series, Further Mathematics for Schools Page 763
I. Find the mean ( ̅ ) of the distribution. 21 -3 9
II. For each given value, find the deviation 21 -3 9
from the mean. i.e. x - ̅ . 23 -1 1
III. Find the square of each deviation. 23 -1 1
24 0 0
i. e. ( ̅)
24 0 0
IV. Find the sum of all the squared deviations. 25 1 1
i. e. ( ̅) 25 1 1
V. Divide this sum by the number of values in 27 3 9
( ̅) 27 3 9
the data. i. e.
( ̅ ) = 40
VI. Find the positive square root of the answer
to obtain the standard deviation. ( ̅)
( ̅)
But =√ , where ( ̅ ) = 40 and
i. e. =√
n = 10
=√
Standard Deviation of a Raw Data
I. Find the mean ( ̅ ) of the distribution by the = 2.00 (2 d. p)
formula: ̅= = Exercises 24.12
II. Prepare and complete a table of values as 1. The marks scored by 8 pupils in a science
shown below: test are 3, 7, 8, 8, 5, 8, 4, 8.What is the standard
deviation?
x x- ̅ ( ̅)
2. The scores of 10 students in an examination
( ̅) = are given as follows; 45, 12, 75, 81, 54, 51, 24,
67, 19 and 39. What is the standard deviation
II. Carefully identify the values of n and of the score?
( ̅ ) and substitute in the formula:
( ̅) 3. The price of 12 commodities in cedi are
=√ to obtain the standard deviation
recorded as: 5, 2, 5, 3, 3, 5, 3, 2, 4, 3, 3, 2
of the data. Calculate the standard deviation.

Worked Examples
4. The ages in years of 8 boys are: 14, 14, 15,
1. The scores obtained by students in a test are:
15, 12, 11 13, 10. Calculate the standard
21, 25, 27, 25, 27, 21, 24, 23, 23 and 24.
deviation.
Calculate the standard deviation.
Solution
Properties of Standard Deviation
̅= = 24 1. If a certain number (constant) is added or
subtracted from each data value, the standard
x x- ̅ ( ̅) deviation remains unchanged.

Baffour Ba Series, Further Mathematics for Schools Page 764


2. If the variance of a set of numbers is p and 3, 4, 6, k, (k + 3), 9 compared to 6, 8, 12, 2k,
each number is multiplied by a constant k, then (2k + 6), 18, the first set is multiplied by 2.
the standard deviation is kp. Variance of 3, 4, 6, k, (k + 3), 9 = 21.6,
3. If all the data values are the same, the Standard deviation of 3, 4, 6, k, (k + 3), 9
standard deviation is zero (because each value =√ = 4.6
is equal to the mean).
4. Standard deviation is never negative. Standard deviation of 6, 8, 12, 2k, (2k + 6), 18
5. A low standard deviation indicates that the = 2 × 4.6
data values tend to be closer to the mean, = 9.2
whereas a high standard deviation indicates that
the data points are spread out over a large range 4. The variance of the numbers 6, 5, 7, 8, 9, and
of values. 7 is 1 . Find the standard deviation of 9, 8, 10,
11, 12 and 10
Worked Examples
1. The standard deviation of a set of ten Solution
numbers is p. If each number is increased by 2, Variance of { 6, 5, 7, 8, 9, 7} = 1
what will be the new standard deviation?
Variance of {9, 8, 10, 11, 12, 10 } = 1

Solution
Standard deviation of {9, 8, 10, 11, 12, 10 }
When each number is increased by a constant,
the standard deviation remains unchanged. = √ =√
Therefore, the standard deviation is p.

Standard Deviation of Ungrouped Data


2. If the standard deviation of a set of numbers
To find the standard deviation of an ungrouped
{1, 3, 5, 7 } is 2.24, find the standard equation
data, prepare and complete a table
of the set {6, 8, 10, 12}
as shown below:
Solution
x f fx fx2
{1, 3, 5, 7 } compared to {6, 8, 10, 12}, it can
be seen that each number is increased by 5. The
standard deviation remains the same.
The standard deviation of a set of the numbers
{6, 8, 10, 12} is 2.24 II. From the table, calculate the mean by the
formula: ̅ =
3. The variance of 3, 4, 6, k, (k + 3) and 9 is
III. Carefully substitute the values of ,
21.6. What is the standard deviation of 6, 8, 12,
2k, (2k + 6) and 18? and ̅ in = √ . / and

Solution

Baffour Ba Series, Further Mathematics for Schools Page 765


simplify to obtain the value of the standard 3 32 96
deviation. 4 40 160
5 52 260
Worked Examples 6 80 480
1.The marks scored by 15 pupils in a test are as 7 59 413
follows; 14, 14, 11, 13, 17, 14, 11, 13, 20, 19, 8 56 448
9 21 189
17, 11, 20, 14, 17. Find:
10 16 160
a. the mean mark,
b. the standard deviation, correct to 2 decimal Find correct to one decimal place;
places. a. the mean mark of the distribution
b. the standard deviation of the distribution
Solution
Solution
x f fx fx2
11 3 33 363
Marks Frequency fx
13 2 26 338
(x) (f)
14 4 56 784
1 14 14 14
17 3 51 867
2 30 60 120
19 1 19 361
3 32 96 288
20 2 40 800
4 40 160 640
15
5 52 260 1300
225 3513
6 80 480 2880
7 59 413 2891
From the table, 8 56 448 3584
̅= = = 15 9 21 189 1701
10 16 160 1600
Substitute the values of ̅, and in 400 = =
1880 15018
δ=√ ( ̅)

δ=√ ( ) a. ̅ = = = 4.7

δ = 3.03 (2d.p)
b. Substitute the values of ̅, and in
2. The table below gives the frequency =√ ( ̅)

distribution of marks scored by 400 students in =√ ( )


a test
. 5 (2 d. p)
Marks(x) Frequency (f) fx
1 14 14
2 30 60
Baffour Ba Series, Further Mathematics for Schools Page 766
3. The table below gives the distribution of the ⇒ =5
number of letters per word in the first 50
words of an easy. = 50 × 5
= 250
No. of 3x + 6y = 250 – 190
letters 1 2 3 4 5 6 7 8 9 3x + 6y = 60
x + 2y = 20……….(2)
No. of
words 1 3 x 10 12 y 6 4 1
eqn (2) – eqn(1)
a. If the average number of letters per word is (x – x) + (2y – y) = (20 – 13)
5, find the values of x and y y=7
b. Calculate correct to two decimal places, the
standard deviation of the distribution Put y = 7 in eqn (1)
x + 7= 13……….(1)
Solution x = 13 – 7 = 6
(x, y) = (6, 7)
x f fx fx2
1 1 1 1 ii. = (190 + 3x + 6y)
2 3 6 12 = 190 + 3(6) + 6(7)
3 x 3x 9x = 250
4 10 40 160
5 12 60 300 (1104 + 9x + 36y)
6 y 6y 36y = 1104 + 9(6) + 36(7)
7 6 42 294 = 1410
8 4 32 256
Substitute the values of ̅, and in in
9 1 9 81
= = =√ ( ̅)
(37 + x (190 + 3x (1104 +
+ y) + 6y) 9x + 36y) =√ ( )

a. From the table, =√


= (37 + x + y) =50
37 + x + y = 50 =√ 8
x + y = 50 – 37 = 1.79 (2 d. p)
x + y = 13……….(1)
Exercises 24.13
1. Calculate the standard deviation of the
̅= = frequency distribution below:
But ̅ = 5 Marks 1 2 3 4 5 6

Baffour Ba Series, Further Mathematics for Schools Page 767


Freq 4 2 1 2 1 2 ii. mode and median.

2. The following is a record of scores obtained Standard Deviation of Grouped data


by 30 J.H.S. 2 pupils in a test marked out of 5: For a grouped data, the standard deviation is
5 3 2 4 5 2 4 3 1 3 calculated as follows:
3 4 2 3 4 5 3 4 3 2 I. Prepare a table frequency as shown below:
4 3 1 2 3 3 2 4 2 1
Construct a frequency table for the data and class x f fx fx2
calculate the standard deviation. a–c
d–f
3. The marks obtained by 16 pupils in a test
were recorded as follows: 4, 3, 8, 7, 7, 6, 6, 4, = = =
5, 1, 4, 7, 8, 4, 3, 2.
i. Construct a frequency distribution table for
II. From the table, calculate the mean by the
this data.
ii. Calculate the mean mark. formula: ̅ =
iii. Calculate the standard deviation. III. Carefully substitute the values of ,

4. The table below shows the marks obtained and ̅ in = √ . / and


by some students in a test. simplify to obtain the value of the standard
deviation
Marks 5 6 7 8 9 10
No. of student Note: x in this case, is the class mid-point of
5 4 2 3 7 7 each class

i. How many students took part in the test? Worked Examples


ii. Find from the table, the median score. 1. Calculate the standard deviation of the
iii. Calculate the mean score. frequency distribution below:
iv. What is the standard deviation of the
distribution? Age 1–3 4–6 7–9 10 –12 13 – 15
Freq 4 10 20 11 5
5. The table gives the frequency distribution of
marks obtained by a number of students in a
test.
Solution
Marks 3 4 5 6 7 8
Frequency 5 9 4 1 class x f fx fx2
1–3 2 4 8 16
If the mean mark is 5, calculate the; 4–6 5 10 50 250
i. value of ;
Baffour Ba Series, Further Mathematics for Schools Page 768
7–9 8 20 160 1280 40 – 59 49.5 36 1782 1280
10 – 12 11 11 121 1331 60 – 79 69.5 17 1181.5 1331
13 – 15 14 5 70 980 80 – 99 89.5 7 626.5 980
=
= = = 91 4364.5 =
50 409 = 3857 247912.75

From the table, From the table,


= 50, = 3857 and = 409 = 91, = 247912.75 and =
By substitution; 4364.5

=√ . / ̅= = = 47.9616
̅ = 48 (Nearest whole number)
=√ . /
=√ ii. Standard deviation;
=√
=√ . /
= 3.20 (2d. p)
=√ . /
2. The table below shos the distribution of
marks in percentages , scored by some students =√
in a test. =√
= 20.5914 (2d. p)
Marks 0 – 20 – 40 – 60 – 80 – = 21 (nearest whole number)
(%) 19 39 59 79 99
No of 7 24 36 17 7 b. number of students who scored between
students 19% and 60% 24 + 26 = 50
P (a student who scored between 19% and 60%
a. Calculate correct ot the nearest whole
=
number, the:
i. mean mark of the distribution;standard
deviation of the distribution. Exercises 24.14
b. Find, correct to two decimal places, the 1. Find the standard deviation for the data
probability of selecting a student who scored below: Ans ̅ = 8.24 S = 3.27
between 19% and 60%.

Solution Age 1–3 4–6 7–9 10 – 12 13 – 15


Freq 2 5 10 5 3
i.
class x f fx fx2
0 – 19 9.5 7 66.5 16 2. Calculate the standard deviation of the
20 – 39 29.5 24 708 250 frequency distribution below: Ans = 6.87
Baffour Ba Series, Further Mathematics for Schools Page 769
These values are obtained by completing the
Age 1– 6– 11 – 16 – 21 – group frequency table such as the one below:
5 10 15 20 25
Freq 2 5 10 5 3 class x d=x-A Freq. fd Fd2
(f)
3. The following table gives the distribution
of height (in cm) of 60 boys.

Height (cm) Frequency The variance is calculated by the formula:


144 – 153 4
= . /
154 – 163 15
164 – 173 16
174 – 183 11 The standard deviation is calculated by the
184 – 193 14 formula:

=√ . /
Calculate the mean and the standard deviation
of the distribution of heights
For ungrouped frequency, complete the table
below:
The Assumed Mean
The assumed mean as the name suggests, is an
x f d= x–A fd fd2
assumption or a guess of the mean. The
assumed mean is usually denoted by the letter
“A”. It doesn‟t need to be correct or even closer
=
to the actual mean and the choice of the
= =
assumed mean is usually at one‟s own
discretion (usually any of the class marks
Worked Examples
estimated to be closer to the mean) except
1. The student body of a certain school were
where the question explicitly states a certain
polled to find out what their hobbies were. The
assumed mean value
number of hobbies of each student was
recorded and the data obtained were grouped
The assumed mean is used to calculate:
into classes as shown on the table below,
1. the actual mean,
2. the variance,
Number of hobbies Frequency
3. the standard deviation. 0–4 45
Given the assumed mean, the actual mean 5–9 58
is calculated by the formula: ̅ = A + 10 – 14 27
15 – 19 30
20 – 24 19
25 – 29 11

Baffour Ba Series, Further Mathematics for Schools Page 770


30 – 34 8
35 – 39 2 =√ . /

Using an assumed mean of 17, find: =√ . /


i. the mean; = 8.81
ii. the variance;
iii. the standard deviation of the number of 2. The table below shows the masses, to the
hobbies of the students in the school. nrearest kg, of 100 workers in a factory.

Solution Mass Frequnecy


i. Assumed mean, A = 17 50 – 59 10
60 – 69 8
Class x d= x f fd fd2 70 – 79 18
–A 80 – 89 30
0–4 2 -15 45 -675 10125 90 – 99 20
5–9 7 -10 58 -580 5800 100 – 109 9
10 – 14 12 -5 27 -135 675 110 – 119 5
15 – 19 17 0 30 0 0
20 – 24 22 5 19 95 475 i. using an assumed mean of 84.5 kg, calculate,
25 – 29 27 10 11 110 1100 correct to one decimal place;
30 – 34 32 15 8 120 1800 α. the mean of the distribution;
35 – 39 37 20 2 40 800 β. the standard deviation of the distribution.
ii. If a worker is selected at random for a blood
= = test, what is the probability that his mass ia
-1025 20775 between 70 kg and 99 kg.

̅ =A+ Solution
i. Assumed mean, A = 84.5
But A = 17, = 200 and = - 1025
̅ = 17 + Class x f d= x fd fd2
̅ = 17 – 5.125 –A
̅ = 11.88 50 – 59 54.5 10 -30 -300 9000
60 – 69 64.6 8 -20 -160 3200
ii. The variance: 70 – 79 74.5 18 -10 -180 1800
80 – 89 84.5 30 0 0 0
= . / 90 – 99 94.5 20 10 200 2000
100 – 109 104.5 9 20 180 3600
= . / 110 – 119 114.5 5 30 150 4500
= 77.61
= = =
iii. The standard deviation;
Baffour Ba Series, Further Mathematics for Schools Page 771
100 -110 24100 44 1 4 4 16
45 1 5 5 25
. ̅ =A+
= = 13 = 67
̅ = 84.5 + 15
̅ = 84.5 – 1.1
̅ = 83.4 kg ̅ =A+

β. The standard deviation;


But A = 40, = 15 and = 13
=√ . / ̅ = 40 +
̅ = 40 + 0.8667
=√ . / ̅ = 40.8667
= 15.5 kg ̅ = 40.87 (2 d. p)

ii. Nmber of workers with mass between 70 kg ii. The standard deviation;
and 99 kg = 18 + 30 + 20 =√ . /
= 68
=√ . /
Total number of workers = 100
=√
P(mass between 70 kg and 99 kg) = = 0.68
̅ = 1.93 kg (2 d. p)

3. The masses of 15 eggs in grammes are ; 42,


4. The table below shows the marks obtained
43, 40, 44, 39, 41, 39, 38, 40, 42, 45, 40, 41, 39 by students in an examination.
and 40. Using an assumed mean of 40
grammes, correct to two decimal places: Marks 0 – 9 10 – 20 – 30 – 40 –
i. the mean of the data; 19 29 39 49
ii. the standard deviation of the data. Freq 2 3 15 10 10

Solution Assuming the mean is 15, calculate the actual


i. A = 40 mean mark.
x f d= x fd fd2
–A Solution
38 1 -2 -2 4 A = 15
39 3 -1 -3 3 class x d= x–A f fd
40 4 0 0 0 0–9 4.5 -10.5 2 -21
41 2 1 2 2 10 – 19 14.5 -0.5 3 -1.5
20 – 29 24.5 9.5 15 142.5
42 2 2 4 8
30 – 39 34.5 19.5 10 195
43 1 3 3 9 40 – 49 44.5 29.5 10 295
Baffour Ba Series, Further Mathematics for Schools Page 772
=
= 610 ii. The standard deviation;
40
=√ . /
̅ =A+
But A = 15, = 40 and = 610 =√ . /
̅ = 15 +
=√
̅ = 15 + 15.25 = 30.25
=√
5. The deviations of a six numbers from 12 are
- 4, 3, 2, 0, 1 and – 3 . Calculate correct ot three = 2.54 (3 s. f)
significant figures, the:
a. mean; 6. The table below shows the frequency
b. standard deviation of the numbers. distribution of marks obtained by 50 students in
a test.
Solution
a. A = 12
d d2 Marks 1 – 11 – 21 – 31 – 41 –
-4 16 10 20 30 40 50
3 9 Frequency 2 3 5 x 10
2 4 Marks 51 – 61 – 71 – 81 – 91 –
0 0 60 70 80 90 100
1 1
Frequency 9 6 4 y 1
-3 9
= -1 = 39
If the mean of the distribution is 47.7, find
using an asummed mean of 45.5;
̅ =A+
a. the values of the constants x and y.
̅ = 12 + b. the variance of the distribution.
̅ = 11.8 (3 s. f)

Solution
a. A = 45. 5
Marks x f d=x - ̅ fd f
Baffour Ba Series, Further Mathematics for Schools Page 773
1 – 10 5.5 2 - 40 - 80 3200
11 – 20 15.5 3 - 30 - 90 2700
21 – 30 25.5 5 - 20 - 100 2000
31 – 40 35.5 x - 10 - 10x 100x
41 – 50 45.5 10 0 0 0
51 – 60 55.5 9 10 90 900
61 – 70 65.5 6 20 120 2400
71 – 80 75.5 4 30 120 3600
81 – 90 85.5 y 40 40y 1600y
91 – 100 95.5 1 50 50 2500
40 + x + y 110 + (-10x) + 40y 17300 + 100x + 1600y

Solving eqn(1) and (2) simultaneously,


Since 50 students took the test, x + y = 10 ……………………(1)
= 50 10x – 40y = 0 ………………..(2)
⇒ 40 + x + y = 50
x + y = 50 – 40 eqn (1) × 10;
x + y = 10 ……………………(1) 10x + 10y = 100 ……………(3)

̅ =A+ eqn (3) – eqn (2);


50 y = 100
y=2
By substitution,
( )
47.7 = 45.5 + Put y = 2 in eqn (1);
x + 2 = 10
47.7 – 45.5 = x = 10 – 2
x=8
2.2 =
b. The variance:
But x + y = 10 = . /
2.2 = = 40 + x + y = 50
= 40 + 8 + 2 = 50
2.2 = = 110 + (-10x) + 40y
= 110 – 10 ( 8) + 40 (2)
110 = 110 – 10x + 40y
= 110
110 – 110 = 40y – 10x
= 17300 + 100x + 1600y
= 17300 + 100 (8) + 1600 (2)
10x – 40y = 0 ………………(2) = 21300

Baffour Ba Series, Further Mathematics for Schools Page 774


By substitution; assumed mean method, given the assumed
mean as 30.5;
= . /
= 426 – 4.84 Class Frequency
= 421.16 10 – 19 11
20 – 29 5
Exercises 24.15 30 – 39 13
A. 1. Consider the test results of 42 form three 40 – 49 4
students in Mathematics. 50 – 59 3
60 – 69 4
marks 40 – 50 – 60 – 70 – 80 –
49 59 69 79 89 5. The table below shows the distribution of the
Freq. 1 4 19 15 3 ages in years of a group of participants at a
seminar;
Taking the assumed mean as 64.5, find :
i. the actual mean of the distribution;
ii. the varaiance of the distribution. Age (years) 23 – 27 28 – 32 33 – 37
iii. the standard deviation of the distribution. No of 8 19 25
participants
2. The test results of 50 students were recorded Age (years) 38 – 42 43 – 47 48 – 52
as follows: No of 23 16 4
participants
Marks 50 – 60 – 70 – 80 – 90 –
59 69 79 89 99 a. Using an assumed mean of 35, calculate:
Freq. 6 11 19 9 5 i. the mean,
ii. the standard deviation of the distribution.
Assuming the mean is 74.5, calculate: b. If a participant is selected at random from
i. the mean mark; the group, what is the probability that she is at
ii. the standard deviation of the distribution. least 38 years old.
3. The table below shows the marks obtained B. 1. Consider the test results of a form three
by students in an examination.
class in Mathematics
Marks 0 – 9 10 – 20 – 30 – 40 –
19 29 39 49 marks 40 – 50 – 60 – 70 – 80 –
Freq 2 3 15 10 10 49 59 69 79 89
Assuming the mean is 15, calculate: Freq. 1 4 19 15 3
i. the actual mean mark of the distribution. Taking the assumed mean as 64.5, find the
ii. the variance of the distribution. standard deviation of the distribution.

4. Calculate the mean, variance and standard 2. The test results of 50 students were recorded
deviation of the following data using the as follows:

Baffour Ba Series, Further Mathematics for Schools Page 775


Marks 50 – 60 – 70 – 80 – 90 – 1. Finding the difference between the mid
59 69 79 89 99 points of the highest class and the lowest class
Freq. 6 11 19 9 5
2. Finding the difference between the highest
Choose a suitable working mean and use it to value and the lowest value
calculate the standard deviation
Worked Example
3. Calculate the standard deviation of the From the frequency distribution table below,
following data using the assumed mean find the range
method.
Marks 1 – 11 – 21 – 31 – 41 – 51 –
10 20 30 40 50 60
class Frequency
10 – 20 4 Freq. 2 6 7 14 20 35
21 – 30 5
31 – 40 4 Solution
41 – 50 4 Method 1
51 – 60 3 Midpoint of the highest class
61 – 70 4
= = 55.5

4. The table below shows the distribution of Midpoint of the lowest class
marks of 40 students in a test. = = 5.5
Range = 55.5 – 5.5 = 50
Marks 21 – 31 – 41 – 51 –
30 40 50 60 Method 2
No of 2 7 m 7
From the table, the highest value = 60 and the
students
Marks 61 – 71 – 81 – 91 – lowest value = 1
70 80 90 100 Range = 60 – 1 = 59
No of 8 5 n 2
students Exercise 24.16
Determine the range of the frequency
If the mean of the distribution is 57.5, find distribution table below:
using an asummed mean of 65.5, the
a. values of the constants m and n. 6 and 3 Marks 11 – 21 – 31 – 41 – 51 – 61 –
b. standard deviation of the distribution. 18.51 20 30 40 50 60 70
Frequency 5 21 15 10 7 3
The Range and Interquartile Range from
Frequency Distribution The Interquartile Range
The Range The inter quartile range for data in a frequency
The range can be determined from the distribution is still the difference between the
frequency distribution table in two ways third quartile (Q3) and the first quartile (Q1). i.
e. Q3 – Q1

Baffour Ba Series, Further Mathematics for Schools Page 776


But the first quartile is determined by the f is the frequency in which the first quartile
falls;
formula: Q1 = L + i, where:
L is the lower boundary of the class containing The Third quartile Q3= L + i where all
the first quartile; variable carry the same meaning except L, CF,
n is the number of frequencies; f and i which refer to the values needed from
CF is the cumulative frequency of all the the third quartile.
classes preceding the class in which the first
quartile lies;

Worked Example
The following table shows the distribution of the masses of parcels in a local post Office

Marks (kg)20 – 24 25 – 29 30 – 34 35 – 39 40 – 44 45 – 49 50 – 54 55 – 59
Frequency 2 3 7 26 29 25 6 2

Find the interquartile range for the data.

Solution

Classes Class boundaries Frequency Cumulative frequency


20 – 24 19 .5 – 24.5 2 2
25 – 29 24.5 – 29.5 3 5
30 – 34 29.5 – 34.5 7 12
35 – 39 34.5 – 39.5 26 38
40 – 44 39.5 – 44.5 29 67
45 – 49 44.5 – 49 .5 25 92
50 – 54 49.5 – 54.5 6 98
55 – 59 54.5 – 59 .5 2 100
Cumulative frequencies of all the classes
Determine the first quartile as follows: preceding the class containing Q1 is 12, so CF
=12
Q1 = L + i
The frequency of the class containing Q1 is 26.
But = = 25
Therefore f = 26
25 > 12 but 25 < 38 ⇒ Q1 lies in fourth class.
Therefore L = 34.5 The class interval of the class containing Q1 is
39.5 – 34.5 = 5. Therefore i = 5
In all, there are 100 observations, so n = 100

Baffour Ba Series, Further Mathematics for Schools Page 777


By substitution,
The class interval of the class containing Q1 is
Q1 = 34.5 + ×5
49.5 – 44.5 = 5. Therefore i = 5
= 34.5 + ×5
In all, there are 100 observations, so n = 100
= 34.5 + ×5
By substitution,
= 34.5 +
Q3= L + i
= 37.0
By substitution,
( )
Determine the third quartile as follows:
Q3 = 44.5 + ×5
Q3= L + i = 44.5 + ×5
( )
But = = 75 = 44.5 + ×5
75 > 67 but 75 < 92⇒ Q3 lies in sixth class.
= 44.5 +
Therefore L = 44.5
= 46.1
Cumulative frequencies of all the classes
The interquartile range;
preceding the class containing Q3 is 67, so CF
= Q3 – Q1
= 67 = 46 .1 – 37.0
The frequency of the class containing Q3 is 25. = 9.1
Therefore f = 25

Exercises 24.17
Find the range and interquartile range of each of the data below;
1
Weight 116 – 118 119 – 121 122 – 124 125 – 127 128 – 130
Frequency 7 19 28 16 5

2.
Marks 40 – 44 45 – 49 50 – 54 55 – 59 60 – 64 65 – 69 70 – 74
Frequency 4 11 20 31 19 11 4

Baffour Ba Series, Further Mathematics for Schools Page 778


25 MATRICES Baffour Ba Series

In mathematics, matrix (plural matrices) is a The Zero Matrix


rectangular array of numbers, symbols or It is a matrix which has each entry as zero. For
expressions arranged in rows and columns.
Examples of matrices are shown below: example, A = 0 1, B = [ ]

1. 0 1 2. 0 1 3. 0 1
Addition and Subtraction of Matrices
The horizontal arrays are called rows and the Two matrices can be added only if they have
vertical arrays are called columns. the same size. To add two matrices, add the
elements in the corresponding positions in each
ROWS
0 1 matrix. For example, given that A = 0 1
COLUMNS
B=[ ] and C = [ ] , then;
Describing a Matrix 1. A + B
A matrix is described by stating the
=0 1 + [ ]
dimensions. For example, , - is a 1 × 2

(read one – by – two), 0 1 is a 2 × 1 (read two – =[ ]

by – one), 0 1 is a 2 × 2 (read two – by – 2. A + C


= 0 1 +[ ]
two) and [ ] is 2 × 3 (read two – by –

three ) all because of their respective number of =[ ]

rows and columns.


3. A + B + C
The individual items in the matrix are called the =0 1 +[ ] + [ ]
elements or entries.
=[ ]
A Square Matrix
If a matrix has the same number of rows and Worked Examples
the same number of columns, it is called a
square matrix. 1. Given that P = [ ] and Q =

0 1 [ ] [ ], find;

2×2 3×3

Baffour Ba Series, Further Mathematics for Schools Page 779


i. P + Q and Q + P A+B
ii. What can you say about P + Q and Q + P =0 1 +0 1

Solution ( ) ( )
=[ ]
P+Q ( )

=0 1
= [ ] +[ ]
A + B is a zero matrix.

=[ ( )] Subtraction of Matrices
( ) Two matrices can be subtracted only if they
have the same size. To subtract two matrices,
=[ ] subtract the elements in the corresponding
8 positions in each matrix. For example, given
that A = 0 1 B=[ ] then;
Q+P
1. A – B = 0 1 – [ ]
=[ ]+[ ]
=[ ]
( )
=[ ]
( ) 2. B – A = [ ]–0 1

– –
=[ ] =6 7
8 – –

Worked Examples
ii. P + Q = Q + P = [ ]. Therefore
8 1. Given that A = [ ] and B =
P + Q and Q + P are commutative.

2. Find the sum of A and B if A and B are [ ] , find ;

0 1 and 0 1 respectively. What i. A – B and B – A


type of matrix is A + B? ii. What can you say about A - B and B – A?

Solution Solution

A=0 1 and B = 0 1 A=[ ] and B = [ ]

Baffour Ba Series, Further Mathematics for Schools Page 780


A–B
4. [ ] + [8 ]
A – B =[ ] – [ ]
8
5. [ ] +[ ]+[ ]

A–B=[ ( )]
( )
Multiplication of Matrices
A. Scale factor
A–B=[ 8]
All the entries of a matrix can be multiplied by
a common factor called scale factor, k, in a
B–A= process called scalar multiplication to obtain a
( ) scalar product. That is if A = 0 1 , then kA
B–A=[ ]
( ) = k0 1=0 1

B–A=[ 8] Likewise, a matrix with a common factor can


be expressed in the form k0 1 where k is the
A–B≠B–A scale factor or common factor. For example, A
So A – B and B – A are not commutative.
=0 1= 30 1, where k = 3.
8
Exercises 25.1
A. Find A + B and A – B in each Worked Examples
1. A = 0 1 B=0 1 1. If P = 0 1 and Q = 0 1 find ;

2. A = 0 1 B=0 1 i. -5P ii. Q

3. A = [ ] B=[ ] Solution
i. P = 0 1
8
4. A = 0 1 B= 0 1
-5P = -5 0 1
B. Simplify the following:
-5P = 0 1
1. 0 1+0 1
-5P = 0 1
2. 0 1+0 1
8
8
3. 0 1+0 1+0 1 ii. Q = 0 1

Baffour Ba Series, Further Mathematics for Schools Page 781


Q= 0 1 =0 1

⁄ ⁄ =0 1
Q =[ ]
⁄ ⁄

Q =0 1 4. Let P = 0 1 and Q = 0 1 find


P2 – Q2 .
2. Given that A = 0 1 and B = 0 1
Solution
find 2A + 3B.
P=0 1 and Q = 0 1
Solution P2 – Q2
A=0 1 and B = 0 1,
=0 10 1–0 10 1
8
2A = 2 0 1=0 1, 8
=0 1 –0 1
8
3B = 3 0 1=0 1
=0 1–0 1
8 8
8 =0 1
2A + 3B = 0 1+0 1 8
8 =0 1
=0 1

=0 1,
8 5. If A = 0 1 and A – 3B = 0 1 find
B.
3. Given that A = 0 1 and B = 0 1, find
B2 – A. Solution
A=0 1 and A – 3B = 0 1
Solution
A – 3B = 0 1
A=0 1 and B = 0 1, find

0 1 – 3B = 0 1
2
B –A=0 10 1 0 1
3B = 0 1–0 1
=0 1–0 1
8
3B = 0 1
=0 1–0 1

Baffour Ba Series, Further Mathematics for Schools Page 782


3B = 0 1 A=0 1 and B = [ ]
Then A × B written as AB;
B= 0 1
= 0 1[ ]=[ ]

5. Given that A = [ ] find xA.


For 3 × 3 matrices,

Solution A=[ ] and B = [ ]then


xA = x [ ]
AB
( ) ( )
= [ ]
( ) ( ) =< =

=[ ]
Note:
Exercises 25.2 If A = 0 1 , then
A. In each of the following, find 2A and -3B
1. A2 = 0 10 1
1. A = 0 1 B=0 1
2. 2A = 20 1
2. A = [ ] B=[ ]
⇒ A2 2A

3. A = 0 1 B =[ ] Worked Examples
1. Find the matrix product AB given that
8
B. Given that A = 0 1 and A=[ ] and B = [ ]
8
B=0 1 evaluate;
Solution
1. 3A 2. 2B 3. 3A + 2B 4. 3A – 2B 8
AB = [ ] [ ]
5. Given that A = 0 1 and 8
=
B=0 1, find 5A + 4B 8
[ 8 ]
8 8 8 8
B. Square Matrices
It involves the multiplication of two or more 8 8
[ 8 8 ]
matrices with same number of rows and
8 8
columns. Thus, given the matrices:

Baffour Ba Series, Further Mathematics for Schools Page 783


AB = [ 8 8] Size of A Size of B
8 m×n n×p

equal
Exercises 25.3
A. Find the product AB. Size of AB is m × p

1. A= 0 1 B=0 1 Size of Matrices in product

2. A = 0 1 B=0 1 Size of A Size of B Size of AB


2×3 3×5 2×5
3. A = 0 1 B=0 1 4×2 2×3 4×3
3×1 1×3 3×3
4. A = 0 1 B=0 1
1×3 3×1 1×5
5×3 3×5 5×5
5. A = 0 1 B=0 1
5×3 5×3 AB is not defined

B. Find the product AB. Note :


The product of a 1 × n row matrix and an
1. A = [ ] B=[ ] n × 1 column matrix is a 1 × 1 marix given by

2. A = [ ] B=[ ]
( , ) =( )

3. A = [ ] B=[ ] ( )
8
1. The number in the row matrix and a column
4. A = [ ] B=[ ] matrix must be equal, else the matrix product
AB is not defined.

2. If A is an m × p matrix and B is a p × n
5. A = [ ] B=[ ]
matrix, then AB is an m × n matrix whose
elements in the ith row and jth column is the
real number obtained from the product of ith
C. Multiplication of m × n matrices by n × p row of A and the jth column of B. If the number
For multiplication, A and B may have different of columns of A does not equal the number of
sizes; however, the number of columns of A rows in B, then the matrix product AB is not
must be the same as the number of rows of B. defined. Hence, it is important to check sizes
Thus, if A is m × n, then B must be n × p. before starting any matrix multiplication.

Baffour Ba Series, Further Mathematics for Schools Page 784


Worked Examples Solution
A=0 1 and B = 0 1
1. Given A = (2, 3, 0) and B = ( + . Find AB
AB = 0 1 0 1
Solution AB = 0 1
AB = (2, 3, 0) ( + AB = 0 1
AB = (2) (-5) + (3) (2) + (0) (2) AB = 0 1
AB = - 10 + 6 + 0 8
AB = 4
2. Given A = (2, 3, 0) and B = ( + . Find AB
2. Find the product of AB if A = 0 1

and B = [ ] Solution

AB = (2, 3, 0) ( +
Solution
AB = (2) (-5) + (3) (2) + (0) (2)
A=0 1 and B = [ ] AB = - 10 + 6 + 0
AB = 4

AB = 0 1[ ] Exercises 25.4
Find the matrix product.
AB = 0 1 1. 0 10 1 4. 0 10 1
8

AB = 0 1 2. 0 10 1 5. 0 10 1
8
8 8
AB = 0 1 3. 0 10 1 6. 0 10 1

C. Multiplication of row and a column matrix Matrix Algebra


Given A = 0 1 and B = 0 1 , then AB = If A, B and C are a pair of m × n matrices, then:
1. A + B = B + A: so matrix addition is
0 10 1 = [ ] commutative.
2. If 0 is an m × n zero matrix, then A + 0 =
0+A=A
Worked Examples
3. A – B ≠ B – A; so matrix subtraction is not
1. Find AB given that A = 0 1 and B = 0 1 commutative

Baffour Ba Series, Further Mathematics for Schools Page 785


4. (A + B) + C = A + (B + C); so matrix Solution
addition is associative.
[ ] = 2I,
5. AB ≠ BA, so matrix multiplication is not
commutative. [ ] = 20 1
6. If I = 0 1, then IA = AI = A, where I is
[ ]=0 1
the unit matrix.
7. If A and B are n × n matrices, and AB = BA = Equating the entries;
I, where I is a unit matrix, then A is said to be x = 2 and y = 2
the inverse of B and B is the inverse of A. √
=√
Exercises 25.5 = √8
Use matrices to prove all the properties.
=2√

The Unit (Identity) Matrix


2. Given that A = 0 1, Find A2 – 2I , where I
A unit matrix denoted by I, is a matrix that
multiplies another matrix A, such that the result is the identity matrix.
is A.
Solution

A unit matrix is given by I = 0 1. For A=0 1,

instance, if A = 0 1 , then; A2 – 2I = 0 10 1 – 20 1

AI = 0 1 0. 1 =0
8
1–0 1

AI = 0 1 =0 1–0 1

AI = 0 1 =0 1

In general, given A = 0 1 and =0 1

I= 0 1, then AI = A
Matrix Equality
⇒0 .
1 0 1=0 1 Two matrices A and B are said to be equal if
they have the same dimension and the same
entries. For instance, given A = 0 1 and
Solving Problems Involving Identity
1. Given that [ ] = 2I, wher I is the identity B=[ ], then A = B implies that:

matrix, find the value of √ a = a1, b = b1, c = c1 and d = d1

Baffour Ba Series, Further Mathematics for Schools Page 786


Worked Examples 3c = 4
1. The matrices A = 0 1 and B = 0 1 are c=
such that AB = A + B. Find the values of a, b Therefore a = , b = 0 and c =
and c.
2. Given that the following matrices are equal,
Solution find the values of x, y and z.
A=0 1 and B = 0 1 0 1=0 1

AB = 0 10 1
Solution
AB = 0 1 0 1=0 1
Equate corresponding elements and solve for
AB = 0 1 involving variables
x+3=6
x=6–3
A+B=0 1+0 1 x=3

A+B=0 1 y = -1

A+B=0 1 z–3=4
z=4+3
z=7
AB = A + B Therefore, x = 3, y = -1 and z = 7

⇒0 1= 0 1
3. If 0 1=0 1, find the values
⇒ 3a = 3 + a
of a and b.
3a – a = 3
2a = 3 Solution
a= 0 1=0 1,
⇒ a – b = 2 ………(1)
3b = b
a + b = 6……….(2)
3b – b = 0
2b = 0
eqn (1) + eqn (2);
b=0
2a = 8
a=
4c = 4 + c
4c – c = 4 a=4

Baffour Ba Series, Further Mathematics for Schools Page 787


Put a = 4 in eqn (1) 5. Given that M = 0 1, N = 0 1 and
4–b=2
-b=2–4 P satisfies the equation PM2 + MN = 2I, where
- b = -2 I is a 2 × 2 unit matrix, find:
b=2 a. b. P

4. Find a 2 × 2 matrix A and B such that A + 2B Solution

=0 1 and 2A + 3B = 0 1 a. M = 0 1 and N = 0 1

=0 10 1
Solution
=0 1
A + 2B = 0 1 and 2A + 3B = 0 1
= 0 1
A + 2B = 0 1 ……………(1)

2A + 3B = 0 1…………….(2) b. Let P = 0 1

eqn (1) × 2; P + MN = 2I
2A + (2)2B = (2) 0 1 0 10 1+0 10 1=20 1

2A + 4B = 0 1 ……………(3) 0 1+0 1=0 1

0 1+0 1=0 1
eqn (3) – eqn (2);
B= 0 1–0 1 0 1 =0 1– 0 1
B=0 1 0 1 =0 1

Put B = 0 1 in eqn (1); Comparing corresponding entries;


- a + 2b = 0………………......(1)
A+20 1=0 1 - 4a – b = 4…………………..(2)
- c + 2d = - 5 ………………...(3)
A+0 1=0 1 - 4c – d = 3…………………..(4)

A=0 1–0 1 Solving eqn (1) and eqn (2);


- a + 2b = 0………………......(1)
A=0 1 - 4a – b = 4…………………..(2)

Baffour Ba Series, Further Mathematics for Schools Page 788


eqn (1) × 4; Exercises 25.6
- 4a + 8b = 0…………………..(5) A. 1. Determine the values of p, q and r from

eqn (5) eqn (2); the relation 0 1=6 7



9b = - 4
b= 2. Given that P = 0 1 and Q =

Put b = in eqn (1) 0 1 find the values of a, b and c if P is


equal to Q.
- a + 2. / =0
-a– =0 3. Given that the following matrices are equal,
find the values of x, y and z.
a=

A=[ ] B=< =
Solving eqn (3) and eqn (4); ⁄
- c + 2d = - 5 ………………...(3)
- 4c – d = 3…………………..(4) 4. Find the value of hfor which

eqn (3) × 4; 0 1 0 1 = h0 1
- 4c + 8d = -20………………(6)
5. Find the matrix B such that:
eqn (6) – eqn (4);
B0 1 =0 1
9d = - 23
d=
B. Find the values of x, y and z.

Put d = in eqn (3); 1. 0 10 1 = 0 1


8
- c + 2. / =-5 2. 0 1[ ]=0 1
-c– =-5
8
3. 0 10 1=0 1
-c=-5+
c= 4. [ ]0 1=0 1
8

P=< = 8
5. 0 1[ ]=0 1

8
P= 0 1 6. [ ]6 7=6 7

Baffour Ba Series, Further Mathematics for Schools Page 789


C. Solve the matrix equations given that /A/ = 5 – (-6)
A=0 1 and B = 0 1 /A/ = 11

i. A + X = B ii. 2A + X = B
2. Given that B = 0 1, find /B/
iii. A – 3X = B iv. 2X – 3A = 5B

D. Find the values of a b, c, d in the Solution


following: B=0 1,
1. 0 1+0 1=0 1 Determinant of B,
/B/ = | |
2. 0 1+0 1=0 1
/B/ = (2) (12) – (6) (4)
3. 0 1+0 1=0 1 /B/ = 24 – 24
/B/ = 0
8
4. 0 1+0 1=0 1
8
2. Given that A = 0 1, find det A.
5. 0 1+0 1=0 1
8
6. 0 1+0 1=0 1 Solution
8 8
A=0 1,
Determinants of a 2 × 2 matrices det A = (8) (3) – (5) -2)
Given the matrix A = 0 1, the determinant = 24 + 10
= 34
of A, denoted by /A/ is determine as follows;
i. Find the cross product of the matrix. That is;
Exercises 25.7
ad and bc.
A. Find the determinant of the matrix.
ii. Obtain ad – bc as the determinant of the
1. 0 1 2. 0 1 3. 0 1
matrix. That is: /A/ = | | = ad – bc
4. 0 1 5. 0 1 6. 0 1
8
Worked Examples
Find the determinant of 0 1 B. Evaluate the determinant of each;
1. [ ], p=2
Solution
i. Let A = 0 1 2. 0 1, n = -4
Determinant of A,
3. 0 1 , r=4
/A/ = | |
/A = (5)(1) – (-2)(3) 4. 0 1 „ y = -2

Baffour Ba Series, Further Mathematics for Schools Page 790


5. 0 1 , x = -3 3. 0 1 4. 0 1

6. 0 1, x=6 Minors
To each element of a 3 × 3 matrix, there
C. Evaluate the following determinants corresponds a 2 × 2 matrix that is obtained by
1. | | 2. | | 3. | | deleting the row and column of that element.
The determinant of the existing 2 × 2 matrix is
called the minors of that element.
Challenge Problem
1. Take A = | | and B = | |, prove that Worked Examples
Find the minors for the elements 2, 3 and - 6 of
det (AB) = det A det B
8
the 3 × 3 matrix[ ].
The Adjoint of a Matrix
If A = 0 1, then the adjoint matrix of A, Solution
To find the minors for 2, delete the first row
written as A* is determined as A* = 0 1 and the first column of the matrix.
8
Worked Examples [ ]
1. Find the adjoint of the matrix 0 1

Now, find the determinant of 0 1;


Solution
Let A = 0 1 | | = (-2)(7) – (-6 )(3)
= -14 + 18
A* = 0 1
=4
Therefore, the minor for 2 is 4.
2. Find the adjoint of the matrix 0 1
8 To find the minors for 3, delete the second row
and the third column of the matrix.
Solution
8
Let A = 0 1 [ ]
8
A* = 0 1
8 Now, find the determinant of 0 1

Exercises 25.8 | | = (2) (-6) – (4 ) (-1)


Find the adjoint of the following:
= -12 + 4
8
1. 0 1 2. 0 1 = -8

Baffour Ba Series, Further Mathematics for Schools Page 791


Therefore, the minor for 3 is -8. The determinant following a1, a2 and a3 are the
minors for a1, a2, and a3 respectively. Writing
To find the minors for - 6, delete the third row the determinants of a 3 × 3 matrix in terms of
and the second column of the matrix. minors is called expansion by minors.
8
[ ] It is always advisable to expand about a column
or row that contains zero(s).
8
Now, find the determinant of 0 1;
Method 2 (using the sign array)
| | = (2)(3) – (0)(3) In expanding about any row or column, the
signs of the coefficients of the minor alternate
=6–0
in accordance with the following sign array:
=6
Therefore, the minor for - 6 is 6.
[ ]
Exercises 25.9
Find all the minors and cofactors of the
elements in the matrix. The sign is easily remembered by observing
that there is a “+” sign in the upper left position
1. [ ] 4. [ ] and then alternating signs for all the remaining
positions.
It is equally advisable to expand about a
2. [ ] 5. [ ]
column or row that contains zero(s).

Worked Examples
3. [ ] 6. [ ]
1. Find the determinant of the matrix by
expanding by minors about the first column.
Determinant of a 3 × 3 Matrix
[ ]
Method I: (Expansion by minors)

Given the 3 × 3 matrix, A = [ ], Solution


Method 1
the determinant is defined as:
Let A = [ ]
/A/ = | |

/A/ = | |

/A/ = a1| | – a2 | | + a3| |


/A/ = 1| | – (-2)| | + 0| |

Baffour Ba Series, Further Mathematics for Schools Page 792


/A/ = 1[(4)(9) – (6)(-7)] + 2[(3)(9) – (-5)(-7)] /A/ = 0 | |–0| |+2| |
+ 0[(3)(6) – (4)(-5)] /A/ = 0[(4)(9) – (7)(-6)] – 0[(3)(9) – (7)(-5)] +
/A/ = 1[36 + 42] + 2[(27 – 35] + 0[18 + 20] 2[(3)(-6) – (4)(-5)]
/A/ = (1)(78) + (2)(-8) + (0)(38)
/A/ = 78 – 16 /A/ = 0(36 + 42) – 0(27 +35) + 2(-18 + 20)
/A/ = 62 /A/ = 0(78) – 0(62) + 2(2)
/A/ = 4
Method 2
Expanding by minors about the second row;
3. Evaluate | | along the rows.
Observe that the second row has the sign:
(– , + , – )
Solution
/A/ = | |
Let /A/ = | |
/A/= - (-2)| |+ 4| |– 6 | |
Evaluate along the third row because it contains
/A/ = 2[(3)(9) – (-5)(-7)] + 4[(1)(9) – (0)(-5)] - zero. The signs of the third row are (+, – , +)
6[(1)(-7) – (0)(3)]
| |=0| |–1| |+2| |
/A/ = 2(27 – 35) + 4(9 – 0) – 6(-7 – 0)
/A/ = 2(-8) + 4(9) – 6(-7) = 0[(1)(2) – (3)(2)] – 1 [(4)(2) –
/A/ = -16 + 36 + 42 (1) (2)] + 2[(4)(3) – (1)(1)]
/A/ = 62 = 0(2 – 6) – 1(8 – 2) + 2(12 – 1)
= 0 (- 4) – 1(6) + 2(11)
2. Find the determinant of the matrix
= - 6 + 22
[ ] = 16

Exercises 25.10
Solution A. Find the determinant of the matrix

Let A = [ ] 1. [ ] 4. [ ]

Expand by minors about the third column


because it contains two zeros. The signs of the 2. [ ] 5. [ ]
third column are (+ , –, +).

/A/ = | | 3. [ ] 6. [ ]

Baffour Ba Series, Further Mathematics for Schools Page 793


B. Evaluate the determinant of each; 2. If A = 0 1, find the value of x if the
1. [ ], x=3 determinant of A = 32.

Solution
2. [ ], x=4
A=0 1 and /A/= 32
But /A/ = (4)(5) – (3x)(-2)
3. [ ] x=3 = 20 + 6x
⇒ 20 + 6x = 32
6x = 32 – 20
4. [ ], g = -2 6x = 12
x=2

5. [ ], b=2
3. Given | | = 0, determine the
value of x.
Equations Involving Determinants
If a given matrix involving a variable is equated Solution
to a real number, the value of the variable can | |=0
be determined as follows;
I. Obtain the determinant of the matrix in terms (x – 2) (x – 3) – (1) (2) = 0
of the variable. x2 – 3x – 2x + 6 – 2 = 0
II. Equate the determinant to the given real x2 – 5x + 4 = 0
number. x = 1 or x = 4 factorization

III. Solve to obtain the value of the variable.


4. If | | = 22, find the value of x.
Worked Examples
Solve the following equations; Solution
1. Find the value of x if | | =2 | | = 22

Solution (2x + 3) (x + 1) – (3) (-4) = 22

| | =2 2x2 + 2x + 3x + 3 + 12 = 22
(x) (3x) – (5) (x) = 2 2x2 + 5x + 3 + 12 – 22 = 0
3x2 – 5x = 2 2x2 + 5x – 7 = 0
3x2 – 5x – 2 = 0 ( x – 1) . /=0
(3x +1) (x -2) = 0 Solving by factorization;
x = 1 or x =
⇒ x = – or x = 2

Baffour Ba Series, Further Mathematics for Schools Page 794


5. If | | = - 3, find the value of x. (x – 2) (x – 3) – 2 = 0
x2 – 3x – 2x + 6 – 2 = 0
Solution x2 – 5x + 4 = 0
| |=-3 (x – 4) (x – 1) = 0
x = 4 or x = 1
( 1 + 2x) (3 – x ) – (6) (- 1) = - 3
3 - x + 6x – 2x2 + 6 = -3 Exercises 25.11
- 2x2 + 5x + 3 + 6 + 3 = 0 1. If | |+| | = 8, find the value of
- 2x2 + 5x + 12 = 0 x , where x > 0. Ans 3
2x2 – 5x – 12 = 0
2. Solve for x | | = - 24
( x – 4) . /=0
x = 4 or x =
3. Solve for x | |=8

6. Find the value of x for which


Type 2 : Equations involving 3 × 3 matrix
| | has no inverse.
Worked Examples

Solution 1. Solve | | =0
| |

If | | has no inverse; Solution

⇒| |=0 | |=0

(x – 3) (x – 4) – 6 = 0
0 | | – (-1) | | + (1)
x2 – 4x – 3x + 12 – 6 = 0
x2 – 7x + 6 = 0 | |=0
(x – 6) (x – 1) = 0
0[(x – 3)(x + 4) – (1) (-1)] + (1) [(5)(x + 4) –
x = 6 or x = 1
(1)(6)] + (1)[(5)(-1) – (6)(x – 3)] = 0
7. Find the value of x such that | | 0[(x – 3)(x + 4) – (1) (-1)] + (1) [(5)(x + 4) –
=0 (1)(6)] + (1)[(5)(-1) – (6)(x – 3)]
5x + 20 – 6 + (-5) – 6x + 18 = 0
Solution 5x + 20 – 6 – 5 – 6x + 18 = 0
| |=0 5x – 6x + 27 = 0

Baffour Ba Series, Further Mathematics for Schools Page 795


- x + 27 = 0 M =I
x = 27 0 10 1 =0 1

2. If [ ] = 13, find the value of x.


⇒2 3

Solution Solve the top lines simultaneously for p and q


and the lower lines simultaneously for r and s
[ ] = 13 to obtain the following;
p= , q= , r= ,s=
4| | – (2) | | + (1) | | = 13
4[(0)(3) – (k) (-1)] – (2) [(5)(3) – (6) (k)] +
The inverse matrix is therefore obtained as:
(1)[(5)(-1) – (6)(0)] = 13
4[( 0 + k] – (2) [15 – 6k] + (1)[-5 – 0] =13 =[ ]
4k – 30 + 12k – 5 = 13
4k + 12k – 35 = 13
Where ad – bc is the determinant of M written
4k + 12k = 13 + 35
16k = 42 as | |.
k=3
By factorization,
Exercises 25.12
Solve for the involving variable. = 0 1
1. | | = 54 2. | | =0
Where 0 1 is the adjoint matrix of

The Inverse of a Matrix 0 1.


If a square matrix M has an inverse, we say A is
invertible. If a matrix is not square, it cannot
Worked Examples
have an inverse.
1. If M = 0 1, find
Inverse of 2 × 2 Matrixes
The inverse of a matrix M is denoted by . Solution
Thus, if M = 0 1 , then its inverse, , is Let M =0 1=0 1,
such that M= I, where I is the unit matrix, I a = 4, b = 5, c = 2 and d = 3
= 0 1. For instance, if M = 0 1 and ⇒ ad – bc = (4)(3) – (5)(2)
= 12 – 10
=0 1 , it implies that: =2

Baffour Ba Series, Further Mathematics for Schools Page 796


Substitute in = 0 1 ii. What is the relationship between ( ) and
?
= 0 1
Solution

2. Find if A = 0 1 A=0 1 and B = 0 1

AB = 0 10 1
Solution
A=0 1 AB = 0 1
8 8
=( )(
0 1 AB = 0 1
) ( )( )

= 0 1
Determinant of AB;
= 0 1 /AB/ = (14 × -14) – (-11 × 32)
/AB/ = -196 + 352
/AB/ = 156
=< ==< =
( ) = 0 1

3. Find the inverse of the matrix 0 1 Determinant of A;


/A/ = (1 × 4) – (6 × 5)
/A/ = 4 - 30
Solution
/A/ = - 26
Let A = 0 1
= 0 1
=( )( ) ( )( )
0 1
Determinant of B;
= 0 1 /B/ = (4 × -2) – (2 × -1)
/B/ = -8 + 2
= 0 1 /B/ = - 6
= 0 1
=< ==< =
= 0 1× 0 1

Solved Past Question = × 0 10 1

Given that A = 0 1 and B = 0 1 8


= 0 1
8
Find:
i. ( ) and = 0 1

Baffour Ba Series, Further Mathematics for Schools Page 797


ii. ( ) = = 0 1 Let M = 0 1= 0 1,
a = 3, b = 2, c = 6 and d = 4
Exercises 25.13
A. Find the inverse of each matrices. ⇒ ad – bc = (4)(3) – (6)(2)
1. 0 1 2. 0 1 3. 0 1 = 12 – 12
=0
8
4. 0 1 5. 0 1 6. 0 1 The matrix M has no inverse. Therefore, it is a
8
singular matrix.

B. Find the inverse of each, if it exist. 2. Find the value of k for which the matrix
1. . / 2. . /
A=0 1 has no inverse.
8
3. . / 4. . /
Solution
C. Find the inverses of A, B and C Let A = 0 1=0 1

A= 0 1 B= 6 7 C= 0 1 If A has no inverse, then ad = bc


⇒ k(k + 1) = (1) (6)
k2 + k = 6
Challenge Problems k2 + k – 6 = 0
Write down the inverse matrix
cos sin Solving by factorization,
1. 0 1 2. 0 1
sin cos k = 2 or k = -3
3. Given X = 0 1, show that x2 – 3x + 8I =
Exercises 25.14
Z. Deduce that = ( ) Identify the singular matrices.
1. 0 1 2. 0 1 3. 0 1
Singular Matrix
It is a matrix that has no inverse. This occurs 4. 0 1 5. 0 1 6. 0 1
when the determinant of the matrix is zero. In
this case, ad = bc. This makes ad – bc = 0 Transpose of a Matrix
The transpose of a matrix, M, is written as .
Worked Examples If the first column of M is the first row of
1. Show whether or not M = 0 1 is a and the second column of M is the second row
singular matrix. of , then is said to be the transpose of M.
Consider M = 0 1, then the M transpose is
Solution
given by =0 1.
M=0 1

Baffour Ba Series, Further Mathematics for Schools Page 798


II. Find the cofactors of the minors to obtain the
Likewise, if K = [ ] then K transpose
matrix of cofactors. ( +

is given by =6 7 III. Find the transpose of the matrix of


cofactors.
IV. Find the determinant of the given (original
Worked Examples
matrix)
1. Write down the transpose of 0 1 V. Divide the transpose matrix by the
determinant to obtain the inverse of the matrix.
Solution
Worked Examples
Let M = 0 1
1. Find if A = [ ]
⇒ =0 1

2. Write down the transpose of B = 0 1 and Solution

show that (B1)1 = B A=[ ]

Solution
Minors of A,
If B = 0 1, then B1 = 0 1
( )( ) ( )( ) ( )( ) ( )( ) ( ) ( )( )
⇒ (B1)1 = 0 1 <( )( ) ( )( ) ( )( ) ( )( ) ( )( ) ( )( )=
( )( ) ( )( ) ( )( ) ( )( ) ( )( ) ( )( )

Therefore (B1)1 = B = 0 1
=[ ]
Exercises 25.15
Find the transpose of the matrices:
=[ ]
8
1. 0 1 4. 0 1

8
2. 0 1 5. 0 1 Matrix of cofactor;

3. 0 1 6. 0 1 [ ]

Inverse of a 3 × 3 Matrix Transpose matrix,


The steps in finding the Inverse of 3×3 matrixes
are as follows: [ ]
I. Replace every entry by its minor.

Baffour Ba Series, Further Mathematics for Schools Page 799


Points were awarded as follows:
Determinant of given matrix [ ] 2 for a win (W), 1 for a draw(D and 0 for a
lose (L). How many points did each team
= -10 1–20 1+30 1 obtain?
= -1,( )( ) ( )( )- 2,( )( )
b. The results of the awy matches are shown
( )( )- + 3,( )( ) ( )( )-
below:
= -1(-10 -1) – 2 (-6 – 1) + 3(0 – 1)
W D L
= -1(-11) – 2 (-7) + 3(-1)
Black 1 1 1
= 11 + 14 – 3 Red 2 0 1
= 22 Green 0 2 1

Divide transpose matrix by determinant of the What is the total number of points (home and
matrix to obtain the inverse of the matrix. away) scored by each team

[ ] Solution
The number of goals scored in home matches .

Exercises 25.16 [ ][ ]=[ ]=[ ]


Find the inverse of the following matrices:

1. [ ] 4. [ ] So the three teams obtained the following home


points ;
Black 5
2. [ ] 5. [ ] Red 4
Green 2

3. [ ] 6. [ ] b. Total number of points scored;


= Home points + Away points

Word Problems Involving Matrices Home Points


Worked Examples Black 5
a. The table below shows the results of the Red 4
home natches of four teams in a world cup. Green 2

Away Points
W D L
Black 2 1 0 The number of goals scored in away matches .
Red 2 0 1
Green 0 2 1 [ ][ ]=[ ]=[ ]

Baffour Ba Series, Further Mathematics for Schools Page 800


So the three teams obtained the following awy The prices of the bundles are :
points ; Ghȼ per bundle cost
Black 3 Linen 50
Red 4 Blankets 100
Green 2 Towels 30
a. Make a 3 × 3 matrix showing the number of
Total points scored by each team bundles delivered.
= Home points + Away points b. Make up a 3 × 1 matrix giving the prices per
8 bundle.
= [ ] + [ ] = [8] c. Make up a 3 × 1 matrix showing the total
money spent by each shop on these items.
Black 8
Red 8 Matrix Equation
Green 4 Matrix equation is solved in a similar manner
as a linear equation. In matrix equation, there is
Method 2 a missing entry represented by a variable that
Results for home and away matches makes the matrix equation true. The task of the
student is to find the missing matrix represented
[ ]+[ ]=[ ]
by a variable in the equation.

Total number of points scored by each team; Steps


8 1. Group available like terms and perform the
[ ] [ ] = [8 ] = [8] included operation.
2. Ensure that the variable stands alone by
addition, subtraction, multiplication or division.
Total points obtained by each team
Black 8
Worked Examples
Red 8
Involving 2 × 2
Green 4
1. Find p and q in each of the following;
Exercises 25.17 a. ( *=. /
1. A firm supplies linen, towels and balankets b. ( )=( )
to three shops as follows:
Solution
Number of bundles
a. ( *=. /
Linen Blankets Towels
⇒ 4p = p + 9
Shop A 6 10 5
Shop B 7 8 6 4p – p = 9
Shop C 11 13 10 3p = 9

Baffour Ba Series, Further Mathematics for Schools Page 801


p=3
3. Given that [ ]0 1 = 0 1, find the value
8
10q = 100 of y.
q = 10
Solution
b. ( )=( ) [ ]0 1 = 0 1,
⇒ p + 5 = 40 8
p = 40 – 5 [ ]=0 1
p = 35 8

[ ]=0 1
2=q+2 8
q=2–2
[ ]=0 1
q=0 8
12 – 2y = 8
2y = 12 – 8
2. Given that A = 0 1 and B = 0 1
2y = 4
Find the values of m and n such that: y=2
AB = 20 1+0 1
4. Given 0 10 1=0 1
Solution Find the values of x.
A= 0 1 and B = 0 1
Solution
AB = 20 1+0 1 0 10 1=0 1

0 10 1 = 20 1+0 1 0 1=0 1
0 1=0 1+0 1
8 8 0 1=0 1
0 1=0 1 4+x=3
x=3–4
Equating the entries; x = -1
2m – 2 = 14
2m = 16 5. Given that [ ] = 0 1,
8
m=8
find the values of x and y.
- 6 + n = - 14
n = -14 + 6
Solution

n=-8 [ ]=0 1
8

Baffour Ba Series, Further Mathematics for Schools Page 802


Equating the entries,
3x + 1 = 10 2. A + [ 8] = [ ]
3x = 10 – 1
3x = 9
A=[ ] – [ 8] = [ ]
x=3

2x + y = 8
2(3) + y = 8
6+y=8 3. [ ]=Y– [ ]
y=8–6
y=2
Y=[ ]+[ ]=[ 8 ]
More Worked Examples
Solve each of the following:
4. 0 1 =0 1 + 4X
1. 5B = [ ]

4X = 0 1–0 1
2. A + [ 8] = [ ] ( )
4X = [ ]
( )
8
3. [ ]=Y– [ ] 4X = 0 1

8
X= 0 1
4. 0 1 =0 1 + 4X

8⁄ ⁄
5. [ ] = - 2B + [ ] X=[ ]
8 ⁄ ⁄

X=0 1
Solution 8 8

1. 5B = [ ]
5. [ ] = - 2B + [ ]
8

B= [ ] = ⁄ =[ ]
⁄ ] 2B = [ ]+[ ] =[ ]
[ 8 8
8

Baffour Ba Series, Further Mathematics for Schools Page 803


Involving 3 × 3 and Other Types
B= [ ]= [ ]
8 1. Given that [ ] = 58, find the
8
value of k.
Solved Past Question
Solution
If P = 0 1 and Q = 0 1 and PQ = QP,
find the values of c and d. [ ] = 58

Solution
20 1 – (-5)0 1 + (-1)0 1 = 58
P=0 1 and Q = 0 1
= 2(3 + 2k) + 5 (9 + 2) – 1(3k – 1) = 58
PQ = 0 10 1 6 + 4k + 45 + 10 – 3k + 1 = 58
8 8 k + 62 = 58
PQ = 0 1
k = 58 – 62
8 k=-4
PQ = 0 1

Exercises 25.18
A. Solve the following;
QP = 0 10 1
1. 2X = [ ]
QP = 0 1
8
8 2. 0 1 + 4Q = 0 1
QP = 0 1
8

Given that PQ = QP 3. -3X – [ ]=[ ]


8 8
0 1=0 1
8
4. 0 1 – 5P = 0 1
Comparing corresponding terms; 8
32 = 24 + 2c
5. 0 1 – 3G = 0 1
2c = 32 – 24
2c = 8
c=4 B. Solve the following:

8 + 2d = 18 1. 3 0 1 + 50 1 = -40 1
2d = 18 – 8
2d = 10 2. 2 0 1–40 1 = 30 1
d=5
Baffour Ba Series, Further Mathematics for Schools Page 804
3. Find all the values of k for which the matrix ⁄
0 1=[ ]

M=0 1 is singular.
0 1=0 1

Solving Simultaneous Equations


2. Write the simultaneous equations:
Given the equations x + y = and x +
7x + 9y = 1 and 10x + 13y = 2 in matrix form
y= ,
and solve them.
I. Express the equations as a single matrix
equation;
Solution
[ ] 0 1 = 0 1…………….(1) 7x + 9y = 1 and 10x + 13y = 2

II. Find the inverse of [ ] as [ ] 0 10 1 0 1

III. Multiply the inverse matrix on the left of


Inverse of 0 1=0 1
each side of equation (1)
IV. Simplify and equate to obtain the values of 0 10 10 1 0 10 1
x and y.
8
0 10 1 = 0 1
Worked Examples
1. Solve 4x + 5y = 3 and 2x + 3y = 1 0 10 1 = 0 1
simultaneously;
0 1=0 1
Solution
4x + 5y = 3 and 2x + 3y = 1 as a single matrix
3. Solve the equations:
equation is; 3x + 4y = 12 and 2x + 6y = 23
0 10 1 0 1
Solution
3x + 4y = 12 and2x + 6y = 23
Inverse of 0 1=0 1
0 10 1 0 1
0 10 10 1 0 10 1
Inverse of 0 1=0 1
0 10 1 = 0 1
8 8 0 10 10 1 0 10 1
0 10 1 = 0 1
8 8
0 10 1 = 0 1
8 8
[ ]=0 1
0 10 1 = 0 1

Baffour Ba Series, Further Mathematics for Schools Page 805


[ ]=0 1 0 1=0 1
8

0 1=0 1 Exercises 25.19


A. Solve for x and y using the matrix
4. Determine the values of x and y such that method.
8
0 10 1 = 0 1 1.0 1 0 1= 0 1 2. 0 1 0 1= 0 1

3. 0 1 0 1= 0 1 4. 0 1 0 1= 0 1
Solution
0 10 1 = 0 1 5. 0 1 0 1= 0 1 6. 0 1 0 1= 0 1

Invserse of 0 1=0 1
B. Write down the following pair of
0 10 10 1 = 0 10 1 equations in matrix form and solve for x and
y in each case.
8 8 1. 2x + 3y = - 4, 5x + y = 3
0 10 1 = 0 1
8 2. x – y = -7, 3x + 2y = 4
0 10 1 = 0 1 3.7x – 2y = 9, 2x – 4y = 2
8 4. 2x + 8y = 33, y – 3x =-4
[ ]=0
1 5. 3x + 2y = 10, 2x – 7y =10
8
⁄ C. Solve giving x, y correct to 2 significant
0 1=[ ]
8⁄ figures.
8
1. 0 10 1 = 0 1
5. Solve the equations:
8 2. 0 10 1 = 0 1
0 10 1 = 0 1

3. 0 1 0 1= 0 1
Solution
8
0 10 1 = 0 1 4.0 1 0 1= 0 1
8 8 8
Inverse of 0 1=0 1 5. 0 1 0 1= 0 1
8 8 8
0 10 10 1 = 0 10 1
D. 1. What is the inverse of 0 1? Use it to
0 10 1 = 0 1
solve the simultaneous equation:
[ ]=0 1 9x + 16y = - 2 and 5x + 9y = -1

Baffour Ba Series, Further Mathematics for Schools Page 806


= (3)(3) – (1)(5)
2. By finding 0 1 solve th equations:
=9–5
3x + 7y = 9 and x + 6y = 5 =4
x= = =2
Cramer‟s Rule
The solution to the system;
x + y = and x + y = is given by =| |=| |
x= and y = , where D = | | = (4)(1) – (3)(2)
=4–6
=| | and =| |, D ≠ 0 = -2
y= = = -1
Note: (x, y) = (2, -1)
1. That D is the determinant made up of the
original coefficient of x and y. 2. Use Cramer‟s rule to solve the system;
2. That D is used in the denominator for both x 3x – 2y = 4 and 2x + y = -3.
and y.
3. That is obtained by replacing the first (or Solution
x) column of D by the constants and . 3x – 2y = 4 and 2x + y = -3.
4. That is obtained by replacing the second
0 10 1 = 0 1
(or y) column of D by the constants and .
D=| |
Worked examples
D = (1)(3) – (-2) (2)
1. Use Cramer‟s rule to solve:
D = 3 – (-4)
4x + 5y = 3 and x + 3y = 1
D=7

Solution
From 4x + 5y = 3 and 2x + 3y = 1; =| |
= 4, = 5, = 3 =2 =3, =1 = (4)(1) – (-2)(-3)
=4–6
0 10 1 = 0 1 = -2
x= = =
D=| |=| |

=| |
D = (4)(3) – (5) (2)
D = 12 – 10 = (3)(-3) – (4)(2)
D=2 = -9 – 8
= -17
=| |=| |

Baffour Ba Series, Further Mathematics for Schools Page 807


y= = = -1 ii. Using the result in i. find, correct to two
dedcimal places, the value of x in the systems
(x , y) = . , / of equations:
2x – y + 2z + 5 = 0
3. Use Cramers rule to solve the equations : x + y + 4z – 1 = 0
4x – 2y = 10 and 3x – 5y = 11 x + 2y + z + 2 = 0

Solution Solution
4x – 2y = 10 and 3x – 5y = 11
| |
0 10 1 = 0 1
= 2| | - (-1) | | + 2| |
D=| | = 2(3 – 8) + 1(1 – 4) + 2(2 – 3)
D = (4)(-5) – (-2) (3) = 2(-5) + 1(-3) + 2(-1)
D = -20 + 6 = -10 – 3 – 2
D = - 14 = -15

ii. 2x – y + 2z + 5 = 0
=| | x + y + 4z – 1 = 0
= (-5)(10) – (-2)(11) x + 2y + z + 2 = 0
= -50 + 22
= - 28 2x – y + 2z = -5
x= = =2 x + y + 4z = 1
x + 2y + z = -2

=| |
| | =| | = -15
= (4)(11) – (3)(10)
= 44 – 30 | |= | |
= 14
= -5| | – (-1) | | + 2| |
y= = = -1
(x , y) = ( , ) = -5(3 – 8) + 1(1 + 8) + 2(2 + 6)
= -5 (-5) + 1(9) + 2(8)
Solved Past Question
= 25 + 9 + 16
i. Evaluate | | = 50
x= = = = - 3.33

Baffour Ba Series, Further Mathematics for Schools Page 808


Exercises 25.20 Challenge Problems
A. Solve each system using Cramer‟s rule. 1. What is the inverse of 0 1?
1. 2x – y = 5 and 3x + 2y = -3
2. 3x + y = -1 and x + 2y = 8 Use it to solve the simultaneous equation:
3. 3x – 5y = -2 and 2x + 3y = 5 13a – 20b = -1 and 7a – 9b = 3
4. x – y = 1 and 3x – 2y = 0
5. 4x – 3y = 5 and 2x + 5y = 7 2. Calculate [ ] [ ]

B. Use Cramer‟s rule to solve; and use your answer to solve the following
1. 0.5x + 0.2y = 8 and 0.4x – 0.6y = -5 x+y+z=2
2. 0.6x + 0.5y = 18 and 0.5x – 0.2y = 7 2x + 3y + 5z = -1
3x + y – 4z = 18
3. x + y = 5 and x – y = -1
3. Use matrices to solve the following :
4. x + y = 4 and x + y = -2.
a. 8p – 15q = 2 and 4p – 7q = 2
b. 13a – 20b = -1 and 7a – 9b = 3
C. Use Cramer‟s rule to solve each:
4. Use cramers rule to solve the system of
1. - 8x – y = 19 and -5x – 6y = 28 linear equations for x.
2. – x – 8y = 82 and 2x + 8y = - 76 - x + 2y – 3z = 1
3. 9x – 2y = 83 and -9x – 8y = 17 2x + z = 0
3x – 4y + 4z = 2
4. 8x + 5y = - 10 and 4x + 8y = - 60

Baffour Ba Series, Further Mathematics for Schools Page 809


26 MATRICES AND TRANSFORMATION Baffour Ba Series

Matrices and Transformations


P:0 1=[ ]
Here, we consider what happens to a point in a
plane when it is subjected to one or more of the P:0 1=0 10 1
transformations: mapping, translation,
P=0 1
reflection, rotations and enlargement.
The matrix of transformation is 0 1
Points or lines which maps onto themselves are
said to be invariant.
Image of a point under a given Linear
Transformation
The image of a point under transformation is
A. Mapping
generally denoted by A1; expressed as A → A1.
A position vector is a 2 × 1 matrix and can be
The transformation T can also be represented
multiplied on the left by a 2 × 2 matrix. For
by using functional notation: T(A) = A1.
example, 0 10 1 = 0 1. The 2 × 2 matrix
Given the linear transformation, is regarded as a mapping. In this case, we say
x1 = ax + by and y1 = cx + dy, the matrix of the the image of the point (-1, 2) is (4, 5) under the
linear transformation is expressed as:
mapping represented by M = 0 1.
M=0 1
To find the image of a point under a given
Worked Examples linear transformation;
1. Find the matrix of the linear transformation, I. Identify the given point or the position
x1 = 3x + y and y1 = 2x – 3y
vector, 0 1.

Solution II. Identify the mapping represented by the 2 ×


x1 = 3x + y 2 matrix 0 1.
y1 = 2x – 3y
III. Multiply the position vector on the left by
[ ]=0 10 1 the mapping, to obtain the image of the point.
0 10 1 = [ ]
The matrix of transformation is 0 1

2. Find the matrix of the linear transformation, Worked Examples


P : (x, y) = (-2y, - x + 2y) 1. Find the image of A(1, 0), B(2, 0) and C(2, 2)
under the mapping represented by the matrix
Solution 0 1.
P : (x, y) = (-2y, - x + 2y)

Baffour Ba Series, Further Mathematics for Schools Page 810


Solution R(-2, 0) under the mapping 0 1
A(1, 0), B(2, 0) and C(2, 2).
Mapping 0 1. Solution
P(-1, 2), Q(-1, - ) and R(-2, 0)
Image of A(1, 0);
⇒0 10 1 = 0 1 0 16 7

=0 1=0 1
( )
P1 = 0 1= 0 1= 0 1
A1 = (1, 2)
P1 = (-2, -1)
Image of B(2, 0);
( )
⇒0 10 1 = 0 1 Q1 = [ ]= 6 7= 6 7
=0 1=0 1
1
Q1 = . , /
B = (2, 4)

Image of C(2, 2); R1 = 0 1= 0 1=0 1


1
⇒0 10 1 = 0 1 R = (0, -2)
P1 = (-2, -1) Q1 = . , / and R1 = (0, -2)
=0 1=0 1
1
C = (2, 6)
3. Find the image of the matrix where the point is
A1= (1, 2), B1 = (2, 4) and C1 = (2, 6) (-2, 3) and the matrix B(x, y) →(3x + 5y 2x + y)

Method 2
Solution
A1 = 0 1=0 1=0 1 B(x, y) →(3x + 5y 2x + y)
Object = (-2, 3)
A1 = (1, 2)

=0 1=0 1=0 1 Matrix × Object = Image


0 10 1=0 1
= (2, 4)
=0 1=0 1
= 0 1=0 1=0 1

= (2, 6) 4. A linear transformation T is defined by T :


= (1, 2), = (2, 4) and = (2, 6) (x, y) (2x – y , 3x + 2y). Find the image of (2,
- 4) under T.
2. Find the images of P(-1, 2), Q(-1, - ) and

Baffour Ba Series, Further Mathematics for Schools Page 811


Solution Exercises 26.1
T : (x, y) (2x – y , 3x + 2y) 1. Find the image of A(-2, 2), B(2, 1) and C(-1,
Object = (2, -4) 0) under M = 0 1
Matrix × Object = Image
( )
0 10 1=0 1
2. A transformation T is given by T = 0 1.
=0 1=0 1 If Q(5, 6) is the image of the point P under
8
transformation T, find the coordinates of P.
5. A point A has (9, 8) under the linear
3. A linear transformation is given by P : (x, y)
transformation 0 1. Find A. → (2x + y, 3x + 2y). Find the point whose
image under P is (1, 3).
Solution
4. The point (a, b) is transformed by the matrix
Mapping 0 1 and Image = (9, 8)
0 1. If the image is (10, -1), find the
Image = Matrix × Point
values of a and b.
0 1=0 10 1
8
0 10 1=0 1 Finding the Transformation given the Matrix
8
Given the matrix 0 1, the linear
[ ] =0 1
8 transformation T, represented by the matrix is
T : (x, y) → (ax + by, cx + dy)
x + 4y = 9……………………….(1)
2x + 3y = 8...……………………(2) Worked Examples
1. Write down the linear transformation defined
eqn (1) × 2;
by the matrix 0 1
2x + 8y = 18………..………….(3)

eqn (3) – eqn (2); Solution


5y = 10 Let the matrix be M =0 1
y=2
The linear transformation, T, defined by M is T
: (x, y) → (-x + 2y, 3x – 4y)
Put y = 2 in eqn (1);
x + 4 (2) = 9
Translation
x+8=9
Under translation, every point of the plane
x=9–8
moves the same distance in the same direction;
x=1
for all points P, OP1 = OP + T, where T is the
A is the point (1, 2)
translation vector.

Baffour Ba Series, Further Mathematics for Schools Page 812


If OP = ( ) and T = . /, then : ii. Image of Q(-4, 0) under the
transformation.
O =( ) +. / = . / Image = Object + Tv
Image = Object + Tv
[ ]=0 1+ 0 1= 0 1 =0 1
Properties of Translation
1. It requires a translation vector, T. 2. In a translation in the plane, the image of
2. All points P map unto their images P1 so that P(5, 2) is (7, 3). Find the image of Q(-3, 0)
PP1 = T. under the translation.
3. The vector gives the magnitude and direction
of the translation. Solution
4. There is no invariant point. Image = Object + Tv
A1
P (7, 3) = P(5, 2) + Tv
A
0 1 = 0 1 + Tv
B1
B Tv = 0 1=0 1
1
P+T=P
Image = Object + Tv
Worked Examples
1. The image of P(5, 4) under a translation in [ ]=0 1 + 0 1= 0 1=0 1
the xy plane is P = (8, -2).
i. Find the matrix equation of the B. Reflection
transformation. Here, an image is produced by reflecting an
ii. Find the image of Q(-4, 0) under the object or point in either the x – axis or the y –
transformation. axis. Thus, we have:
1. Reflection in the x – axis (or line y = 0).
Solution 2. Reflection in the y – axis (or line x = 0).
Image = Object + Tv 3. Reflection in a given line.
P (8, -2) = P(5, 4) + Tv
8 1. Reflection in the x – axis (or line y = 0).
0 1 = 0 1 + Tv
8 This is represented by the matrix 0 1
Tv = 0 1–0 1=0 1
Image = 0 1 × object
Image = Object + Tv
[ ]=0 1+ 0 1=[ ] Worked Examples
1. Find the image of the point A(-2, 5) in the
x – axis.
The matrix equation is:
=x+3 =y–6

Baffour Ba Series, Further Mathematics for Schools Page 813


Solution (2, 5)
A(-2, 5)
Reflection in the x – axis is represented by the 3. Reflection in the line y = x or y – x = 0
matrix 0 1 This is represented by the matrix 0 1

A1 = 0 10 1 Image = 0 1 × object

A1 = 0 1= 0 1= 0 1
Worked Examples
A1 = (-2, -5) 1. Find the image of the point C(5, 7) under
reflection in the line y = x.
2. Find the image of the point P(5, 3) under the
linear transformation M = . / Solution
C(5, 7)
Reflection in the y = x is represented by the
Solution
Image = Matrix × Point matrix 0 1
=. /. /
C1 = 0 10 1
=. /=. /=. /
(5, -3) C1 = 0 1=0 1=0 1

C1 (7, 5)
2. Reflection in the y – axis (line x = 0)
This is represented by the matrix 0 1
4. Reflection in the line y = - x or y + x = 0
Image = 0 1× object This is represented by the matrix 0 1

Image = 0 1 × object
Worked Examples
1. Find the image of the point B(-2, 5) in the
y – axis. Worked Examples
1. Find the image of the point D(6, 11) under
Solution reflection in the line y = - x
B(-2, 5)
Reflection in the y – axis is represented by the Solution
D(6, 11)
matrix 0 1 Reflection in the y = -x is represented by the
matrix 0 1
B1 = 0 10 1
=0 10 1
=0 1= 0 1=0 1

Baffour Ba Series, Further Mathematics for Schools Page 814


=0 1 √
=: ;
√ . /
=0 1=0 1
=( √ *
(-11, -6) √
=( √ , √ )
Solved Past Question
Write down the matrices A and B representing Properties of Reflection
the following transformations respectively: 1. There is a single invariant line, the axis, A (x,
i. a reflection in the line x + y = 0 y ) of reflection.
ii. (x, y ) →. √ , √ / 2. All other points P map onto their images P1
b. Find the image of the point (2, 4) under the so that (x, y) axis is the mediator of PP1. PP1
transformation BA. are equidistance from any point M of A, and
PM, P1M are equally inclined to A.
Solution
a. i. Reflection in the line x + y = 0
⇒ a reflection in the line y = -x
⇒ A(x, y) → (-y, -x)
A=. /
A

ii. The matrix B of the linear transformation


√ C. Rotation
B= : ; Rotation is the turning of an object or point

about or around a fixed point (where the fixed
point could be the origin). Rotation can be
√ clockwise or anticlockwise through 900, 1800,
b. BA = : ;. / 2700 and other angles, represented by .

√ A. Rotation through an angle θ, anticlockwise


=: ; about the origin:

This is represented by the matrix
identity:. /
The image of (2, 4) under the transformation
BA ;
Worked Example

: ;. / Given that R(θ) = . /, find

0
R(45 )

Baffour Ba Series, Further Mathematics for Schools Page 815


Solution Image of P;

R(θ) = . /,
P1 = : ;. /

R(450) = . /,

√ √
P=: ;
0 √
R(45 ) = : ;
√ √
√ √
P= . , /

2. Find the point, , of the point P .√ , /


√ 4. Find the image of the point (-3, 5) when it is
when it is rotated through 450 about the origin. rotated through an angle of 450 in the clockwise
direction.
Solution
R(θ) = . /, Solution
R(θ) = . /,
R(450) = . /,
√ √ R(450) = . /,
0
R(45 ) = : ; √ √
√ √
0
R(60 ) = : ;
√ √

Image of P;
√ √
√ Image of (-3, 5);
P1 = : ;4 5 = : ; =: ; √ √
√ √

=: ;. /
√ √

3. The point P(-1, 3) undergoes an √ √ √


anticlockwise rotation through 600 about the
=< ==: ;=( √ *
origin. Find the coordinates of its image. √ √ √ √

Solution = ( √ , √ )

R(θ) = . /, B. Anticlockwise Rotation through an Angle 900or


Clockwise Rotation through 2700about the origin.
R(600) = . /,
This is represented by the matrix . /

0
R(60 ) = : ; Worked Example

1. Find the image of P(- 4, 9) under
anticlockwise rotation through 900 about the
origin.
Baffour Ba Series, Further Mathematics for Schools Page 816
Solution Solution
P(- 4, 9) under anticlockwise rotation through R(-6, 4) under anticlockwise rotation through
900 about the origin is given by: 2700 about the origin:
=. /. / =0 10 1

=. /= . /=. / =0 1=0 1=0 1


= (-9, -4)
2. Determine the image of the point (q, - q)
C. Anti clockwise Rotation through 1800 or under a clockwise rotation through 900, about
Clockwise Rotation through 1800 about the the origin.
origin.
This is represented by the matrix identity Solution
0 1 (q, - q) under a clockwise rotation through 900,
about the origin.

Worked Example =0 10 1
1. Find the image of Q(7, 10) under
=[ ] =0 1
anticlockwise rotation through 1800 about the
origin. = (- q , - q )

Solution Properties of Rotation


Q(7, 10) under anticlockwise rotation through 1. There is a single invariant point, O of the
1800 about the origin is given by: rotation.
2. All other points P map onto their images P1
0 10 1
so that PO = P1O, < POP1 = θ
=0 1 =0 1=0 1 3. The angle θ, gives the magnitude of the
Q1 = (-7, -10) rotation, the sign convention being positive for
clockwise and negative for anticlockwise.
D. Anti-clockwise Rotation through 2700 or 4. The angle between any line and its image,
Clockwise Rotation through 900about the origin. example PQ and P1Q1 is equal to θ.
This is represented by the matrix identity Q1
0 1 O
θ
θ

Worked Examples
1. Find the image of R(-6, 4) under Q
0
P
anticlockwise rotation through 270 about the P1
origin.

Baffour Ba Series, Further Mathematics for Schools Page 817


Exercises 26.2 Worked Examples
1. Find the image of the point (1, 1) when it is 1. Find the image of A(3, -10) under
rotated 400 anti clockwise about the origin. enlargement from the origin with scale factor 2.
2. A rotation about O maps P(3, 5) → (5, -3),
Q → . If Q is (-2, -4), find the coordinates of Solution
and the matrix which represents the rotation. A(3, -10) under enlargement with scale factor 2:
A1 = 0 10 1
Rotation about a Point
A rotation through θ anticlockwise about the A1= 0 1= 0 1= 0 1
point (a, b) can be build up in the following A1 = (6, 17)
way:
i. translate the plane by 0 1 so that the center 2. Find the image of the point P(8, -2) under an
of roataion is now at the origin. enlargement from the origin O, with scale
ii. rotate about this origin through θ anti factor 2.
clockwise.
iii. translate the plane back by 0 1 to its original Solution
P(8, -2) under enlargement with scale factor 2:
position. 8
P1 = 0 10 1
[ ] =0 1 0 1 +0 1
8
P1= 0 1= 0 1= 0 1
OR 8
P1 = (16, - 4)
[ ] =0 1 0 1
Properties of Enlargement
Note: 1. There is a single invariant point, the center of
The second form is more instructive, since it enlargement.
maintains a straight forward rotation with the 2. All other points P map onto their images:P1
point (a, b) as center. so that OP1= kOP, where k is the scale factor of
enlargement.
D. Enlargement
Enlargement is the process of making an object Under enlargement, any figure is mapped onto
bigger than its original size. This is made a similar figure.
possible by the use of a scale factor, k. In some 1. If /k/ > 1, the image is larger than the original
cases, the object is made smaller than its figure but /k/ < 1, it is smaller. To avoid the
original size in a process called reduction or concept of an enlargement producing a smaller
dilatation. figure, the term dilatation is often used.
 If k < 0, O lies between P, P1 and the
Enlargement from the origin, with scale factor image figure is inverted.
k, is represented by the matrix identity 0 1

Baffour Ba Series, Further Mathematics for Schools Page 818


 A scale factor -1 is equal to a rotation 0 1=0 1
about the center O, of 1800.
a = 4 and c = 2
 With a scale factor , k, similar figures
have lengths in the ratio k : 1 and areas
C(0,1) → C(-2, 4)
in the ratio k2 : 1
0 10 1 = 0 1

∆2 0 1=0 1
O
0 1=0 1
∆1
∆ b = -2 and d = 4

Substitute a = 4, b = -2, c = 2 and d = 4


0 1=0 1

Isometrics The matrix of the transformation is 0 1


Under isometrics, any figure maps onto a
congruent figure. Translation and rotation ii. The coordinate of the image of B(1, 1).
produce directly congruent figures, reflection is
inversely congruent. B1 = 0 10 1 = 0 1

B1=0 1=0 1=0 1


More Worked Examples
1. A linear transformation maps A(1, 0) → A1 B1 = (2, 6)
(4, 2) and C(0,1) to C(-2, 4)
i. Find the matrix which represents the iii. The determinant of the matrix 0 1
transformation.
ii. Find the coordinate of the image of B(1, 1). | | = (4)(4) – (-2) (2)
iii. Find the determinant of the matrix. = 16 + 4 = 20
iv. Illustrate the information with a sketch of
OABC and its image. iv.

Solution
i. Let the transformation matrix be 0 1
A(1, 0) → A1 (4, 2)
C B
0 10 1 = 0 1

A
0 1=0 1

Baffour Ba Series, Further Mathematics for Schools Page 819


2. The unit square OABC above, is stretched The matrix of transformation is 0 1.
parallel to the line y = 2x. O is fixed, A → (2, 2).
i. Find the matrix which represents the
transformation. ii. Image of B;
ii. Find the image of B, and that of C. B1 = 0 1 0 1
iii. Find the value of the determinant of the
matrix. B1= 0 11= 0 1=0 1
iv. What can you say about the areas of the unit B1 (4, 7)
square and its image?
Image of C;
Solution
C1 = 0 1 0 1
Let 0 1 is the matrix of transformation;

0 10 1 = 0 1 C1 = 0 1=0 1=0 1

0 1=0 1 C1= (2, 5)

0 1=0 1 iii. Determinant of 0 1


a = 2 and c = 2
| | = (2)(5) – (2)(2)
Now, since O is fixed, and the stretched is = 10 – 4
along the line y = 2x, all points in the line y = =6
- x are fixed.
iv. Since the determinant is 6, the area of the
0 10 1=0 1 image is 6 times the area of OABC.
0 1=0 1
Reflection in a Line through the Origin
0 1=0 1 Consider the point P(x, y), a distance of d from
the line y = x tan θ (where θ is the angle
4–b=2 between line and the positive x – axis), and its
b=4–2 image P1( , ) after reflection in this line.
b=2 y
( , )
 dθ
4 – d = -1
d
d=4+1  θ
d=5 (x, y)

Substitute a = 2, b = 2, c = 2 and d = 5 θ
x
0 1=0 1

Baffour Ba Series, Further Mathematics for Schools Page 820


cos sin √
[ ] =0 1 0 1
sin cos The matrix < = has the form

Worked Examples
1. Find the matrix which represent a cos sin
0 1
transformation in the line y = 2x sin cos

⇒ cos θ = and sin θ =
Solution

y = 2x θ = . / and θ = . /
From y = 2x, the gradient of the line is 2 θ = and θ =
⇒ tan θ = 2
tan θ = 2 √ The matrix represents a roataion about O
through 2100 anti clockwise.
sin θ = 1

Exercises 26.3
tan θ = A. Write down the matrices representing the

following transformations.
Now, from trigonometry, 1. Reflection in the line through the origin
cos 2θ = 2 cos2 θ – 1 which makes an angle of 600 with the positive x
cos 2θ = 2 . / – 1 – axis.

2. Rotation through 1350 clockwise about O.
cos 2θ =
3. Rotation through . / clockwise about O.
4. Reflection in the line y = -3x
sin 2θ = 2 sin θ cos θ
sin 2θ = 2 . / . / = B. Decribe geometrically, the plane
√ √
transformation with the following matrices;
By substitution, the required matrix is : √
√ √ √ √
cos sin 1. < = 2. < = 3. < =

0 1 =< =
sin cos √ √ √ √

8 8
4. 0 1 5. 0 1
8 8
2. A plane transformation has matrix

Inverse of a Linear Transformation
< = . Describe this transformation
√ The inverse transformation of a linear
transformation:
geometrically.
= ax + by = cx + dy is
Solution x= ( )
y= ( ),

Baffour Ba Series, Further Mathematics for Schools Page 821


provided K = ad – bc ≠ 0 T:. /→. /0 1
The inverse is also linear.
T=. /
Worked Examples
1. Find the inverse of the linear transformation
ii. Let a = 4, b = 1 , c = 6 and d = 3
= 2x + 3y = 4x + 5y
Det T = ad – bc
Det T = (4) (3) – (1) (6)
Solution
= 12 – 6
= 2x + 3y = 4x + 5y
=6
Let a = 2, b = 3, c = 4 and d = 5
K = ad – bc
= . / =: ;
K = (2) (5) – (3) (4)
K = 10 – 12
K = -2
iii. Let the coordinates of P be (x, y)

x= ( )
: ; . /=. /
y= ( ),

By substitution; : ;=. /
x= ( )
y= ( ), ⇒ = -2 …………….(1)
= 1 ……………….(2)
x= +
y=2 Reducing the equation;
( ) ( ) = -2 (6)
2. A transformation is defined by T : (x, y) →
(4x + y, 6x + 3y). 3x – y = - 12 ………………..(3)
i. Write down the matrix T of the
transformation. ( ) ( ) = (3)1
ii. Find the inverse , of the matrix T. -3x + 2y = 3………….…….(4)
iii. If the point P is transformed by the matrix
and the imagae is (-2, 1), find the eqn (4) + (3)
coordinates of P. y = -9

Solution Put y = -9 in eqn (3);


i. T : (x, y) → (4x + y, 6x + 3y). 3x – (-9) = - 12
T:. /→. / 3x + 9 = - 12
3x = - 12 – 9
Baffour Ba Series, Further Mathematics for Schools Page 822
3x = - 21 In general, if a matrix P is followed by a matrix
x=-7 Q, the composite matrix „P followed by Q‟ is
P(-7, -9) written as QP. In functional notation, if P(M) =
M1, Q( ) = ( ), then Q(P(M)) = ( ) or
Exercises 26.4 simply put, QP(M) = ( )
Find the inverse linear transformation of the
following: Worked Examples
1. = 2x + 3y = x – 2y
1. Given the matrices A = 0 1 and B =
2. = - 4x + 5y = 3x + 2y
3. = 3x - 2y = 4x + 6y 0 1, find the matrix of the transformation
A , followed by B.
Composition of Linear Transformation
This occurs when an object undergoes two or
Solution
more transformations, one after the other. Thus,
A=0 1 and B = 0 1,
AB . / is defined as the transformation for the
matrix B, followed by the transformation for
the matrix A. This is calculated as follows: A followed by B;
I. Write the matrix A and B of the linear A○ B = BA
transformation. A○ B = 0 10 1
II. Find the matrix product AB. 8
III. Write the linear transformation for AB. That A○ B = 0 1=0 1
8 8
is AB. /.
IV. Notice that the matrix which represents the 2. Given that M = 0 1 and N = 0 1,
first transformation A is placed on the right and
find the image of the point (2, 3) under the
the second transformation B is placed on the
transformation M ○ N.
left in the matrix product. This is illustrated
below:
Solution
Given A : [ ] = [ ] 0 1 and M=0 1 and N = 0 1,
B : [ ] = [ ] 0 1, if A is applied first, M ○ N = MN
followed by B, we obtain: M○N= 0 10 1,
AB. /: [ ]=[ ][ ]0 1
M○N=0 1= 0 1
Thus, the matrix which represents the
composite transformation A followed by B is
Image of (2, 3) under transformation M ○ N
the matrix product [ ][ ].
=0 1 0 1= 0 1= 0 1

Baffour Ba Series, Further Mathematics for Schools Page 823


3. Let S : (x, y) →(-x, y) and T : (x, y) → (- y, x) A : (x, y) →(x + y, 2x – y)
be two linear transformations in the x – y plane. A=0 1
Find a single matrix representing the composite
transformation TS.
When a point is transformded under A,
Solution followed by B it remains unchanged.
⇒ B○A = BA = I
S : (x, y) →(-x, y) ⇒ S = 0 1 This means that B is the inverse of A
T : (x, y) →(- y, x) ⇒ T = 0 1 B=
Det A = (1) (-1) – (2) (1)
TS = ⇒ S = 0 10 1 = -1–2
=-3
TS = 0 1=0 1
= 0 1

4. Two linear transformations are defined by:


A : (x, y) →(2x – y, 3x + 2y) = < =
B : (x, y) →(3x + 2y, 4y)
Find the matrix representing A○B.
Since B =
Solution
A : (x, y) →(2x – y, 3x + 2y) B=< =

A=0 1
As a linear transformation;
B : (x, y) → . /
B : (x, y) →(3x + 2y, 4y)
B : (x, y) → . /
B=0 1

Exercises 26.5
A ○ B = AB
1. Find the image of the point P(2, 5) under a
=0 10 1= 0 1=0 1 reflection in the y – axis , followed by an anti –
8
clockwise rotation of 1800 about the origin.
Solved Past Question Describe the combined linear transformation.
1. The linear transformations A and B are
defined by A : (x, y) →(x + y, 2x – y) and B : 2. Find the image of the point P(4, -5)
(x, y) →( , ). When a point is transformded under: a rotation through 600 anti clockwise ,
under A, followed by B it remains unchanged. followed by another rotation through 750 anti
Find the point ( , ). clockwise, followed by a reflection in the line
y = -x
Solution

Baffour Ba Series, Further Mathematics for Schools Page 824


27 LOGIC Baffour Ba Series

Statements 6. 22 is a composite number.


A statement is a sentence which is either true or
false, but not both. Notation of Statements
In mathematics, statements are denoted by
Consider the following sentences: letters such as P, Q and R. For example, P: 5 +
1. Please sit down. 6 = 13, simply means P is the statement 5 + 6 =
2. How old are you? 13. P is obviously a false statement because 5 +
3. Mr. Osei is a hard working teacher. 6 13.
4. Kumasi is the capital town of Ghana
5. A triangle has four sides. Negation of a Statement
6. The baby is either a boy or a girl. Consider the statement P: Linda is a girl. The
statement “Linda is not a girl” is called the
Sentences 1 and 2 are not statements because negation of P and it is denoted by P, where
they are neither true nor false. the symbol means “not” and P is read as
Sentences 3, 4 and 5 are statements because “not P”. Hence P:“Linda is not a girl”.
they can either be true or false, depending on Obviously, when P is true, the negation of P is
the context. The sixth sentence, is obviously a false and vice – versa.
true statement.
Worked Examples
Exercises 27.1 Write down the negation of the statements
A. Which of the sentences are statements? 1. P : 5 {primes} 2. 7 ∉ {even integers}.
1. Ghana is in Africa. 3. P : x < 2 4. P : 3 < x < 4
2. All primes are odd numbers.
3. Will you go to school tomorrow? Solution
4. 5 = 3 mod 3. 1. P : 5 ∉ {primes}
5. 3 is an even number. 2. P : 7 {even integers}
6. 10 is a multiple of 5. 3. P : x 2, or P: x 2
4. P : x 3, or x 4
B. Which of the following sentences are
statements? If they are statements, state Exercises 27.2
whether they are true or false Write down the negation of each of the
1. Do your mathematics exercises. following statements and state whether the
2. There are 52 weeks in a year. negation is true or false;
3. Drive with care. 1. P : 3 {primes} 2. P : 4 {factors of 10}
4. sin 300 = sin 600. 3. P : 5 < 3 4. P : x > 3
5. All prime numbers can only be divided by 1 5. P : 4 = 2 6. P : - 4 x 5
and itself.
Baffour Ba Series, Further Mathematics for Schools Page 825
Statements and Venn Diagrams 3.R : Every science student studies mathematics
Similar to sets, statements can be represented in but not history.
Venn diagrams. Identify the following from the 4. S : All my friends are intelligent people.
given statements: 5. T : Every multiple of 10 is also a multiple of 5.
1. Universal set.
2. Subsets. B. Illustrate on a Venn diagram.
3. Complement. 1. P : All football players play volley ball.
Q : A volley ball player does not play hockey
Worked Examples Blay plays volleyball but not football.
Represent each of the following statements on a
Venn diagram. 2. P : All my friends are intelligent.
i. P : Kofi is a good boy. Q : Intelligent people are quite people.
ii. Q : 4 is a factor of 100 but 3 is not. R : Dufie is a quite person but not intelligent.

Solution 3. P : A vegetarian does not eat meat.


i. U = {boys} Q : All my friends are vegetarians.
A = {Good boys} R : Jonathan is not a vegetarian but does not eat
k = Kofi meat.
k {Good boys} S : Rosemond eats meat.
U T : Brown is a vegetarian but he is not my
A friend.
K
4. P : Rich men live in big houses.
Q : Mr. Owusu is a rich man.
R : Mr. Owusu lives in a big house.
ii. U = {Integers}
A = {factors of 100} 5. P : All boys play football.
4 *factors of 100} Q : Danny plays football.
U
A1 = {3} R : Danny is a boy.
A

4 Compound Statements
 A compound statement is a combination of two
3
or more statements in a single statement. They
are usually formed by the conjunction “and”
Exercises 27.3
and “or”. For example, P : x > – 4, Q : x < 3.
Represent each of the following statements
The compound statement P and Q is the
on a Venn diagram.
statement “x > – 4 ” and “x < 3”written as P
1. P : A square is a quadrilateral.
and Q : - 4 < x < 3
2. Q : Rich people are hardworking people.

Baffour Ba Series, Further Mathematics for Schools Page 826


In general, P and Q is denoted by P˄Q, where “ a compound statement using “and” and
˄” means “and” determine whether the compound statement is
true or false.
If P is a statement about a subset A and Q is a 1. P : A square has four sides, Q : A triangle
statement about subset B, then P˄Q,is a has three sides
statement about the subset A ∩ B represented in 2. P : 6 + 7 = 13 , Q : 5 + 4 > 8
the diagram below: 3. P : 4 {primes}, Q : 4 {even numbers }

A B U
B. From the following pair of statements, form
a compound statement using “or” and
determine whether the compound statement is
true or false.
A∩ 1. P : 4 {primes}, Q : 4 {even numbers}
The compound statement “P and Q” is true 2. P : 3 {factors of 27}, Q : 5 {multiples of
only if both P and Q are true and false only if 10}
either P or Q is false. 3. P : 2 {odd numbers}, Q : 2 {factors of 15}

Similarly, the compound statement P or Q is the Solution


statement “x > - 4 ” or “x < 3” written as: A. 1. P˄Q : A square has four sides, and a
P and Q : - 4 < x < 3 triangle has three sides. P˄Q is true, because P
and Q are both true.
In general, P or Q is denoted by P˅Q, where “
˅ ” means “or” 2. P˄Q : 6 + 7 = 13 and 5 + 4 > 8. P˄Q is true,
If P is a statement about a subset A and Q is a because both P and Q are true.
statement about subset B, then P˅Q, is a
statement about the set A ∪ B. 3. P˄Q: 4 {primes}∩ {even numbers }
P ˄Q is false since P is false.
A B U
B. 1. P˅Q: 4 {primes}∪{even numbers}
⇒ 4 is either a prime or an even number
P ˅Q is true, since P is true
A∪B
2. P˅Q : 3 is a factors of 27 or 5 is a multiples
The compound statements “P or Q” is true only of 10.
if either P or Q is true and false if P is false and P ˅Q is true since Q is true
Q is also false.
3. P˅Q: 2 {odd numbers} ∪ {factors of 15}
Worked Examples ⇒ 2 is either an odd number or 2 is a factor of 15
A. From the following pair of statements, form P ˅ Q is false since P and Q are false.

Baffour Ba Series, Further Mathematics for Schools Page 827


Exercises 27.4 Let us, consider the “if clause” as statement P
A. From the following pair of statements, and the “then clause” as statement Q. It is
form a compound statement using “and” observed that in each case, if statement P is
and determine whether the compound true, then it implies that statement Q is also
statement is true or false. true. This is expressed as P implies Q,
1. P: 2 {primes}, Q : 2 { even number} symbolically P ⇒ Q Sentences of the type P ⇒
2. P: 3 {factors of 6}, Q :3 { factors of 9} Q are referred to as Implication.

3. P : 2 + 5, Q : –12 + 16 > 3 Statements 1, 2 and 3 may appear as follows:


4. P : 6∉{primes}, Q : 6 { composite} 1. It is a dog ⇒ It is an animal with 4 legs
2. Two triangles are congruent ⇒ Two triangles
5. P : 10 {even}, Q : 5 { primes}
are similar
3. a = b, ⇒a2 = b2
B. From the following pair of statements,
form a compound statement using “or” and
Note that the arrow “⇒” always points at
determine whether the compound statement
the“then clause”
is true or false
1. P : {integers}, Q : 2 { prime numbers} Worked Examples
2. P : 5 {factors of 14}, Q : 5 {multiples of Which implications are true or false;
10}. 1. x2 > 4, then x > 2
3. P : 3 , Q: 2 3 2. If a number is a perfect square, then it is
4. P : 15 {primes}, Q : 5 { prime} positive

5. P : 7 {perfect squares}, Q : 8 { perfect Solution


cubes} 1. False, (-3)2 > 4, (But -3 < 2)
2. True
Implication
Consider the following statements; Converse of an Implication
1. If it is a dog, then it is an animal with four For all implications, the arrow “⇒” always
legs. points at the “then clause” as shown in the
2. If two triangles are congruent then the two following implications:
triangles are similar. 1. It is a dog⇒It is an animal with 4 legs.
3. If a = b, then a2 = b2. 2. Two triangles are congruent ⇒ Two triangles
4. If he has passed the examination, then he is are similar.
promoted. 3. a = b, ⇒a2 = b2

Each of these sentences consist of two clauses – If the arrow is reversed like this “⇐”, the
the “if clause” and the “then clause”. Each of implications appear as follows:
the two clauses is a statement in itself. a. It is a dog ⇐ It is an animal with four legs.

Baffour Ba Series, Further Mathematics for Schools Page 828


b. Two triangles are congruent ⇐ Two triangles Note:
are similar. P ⇒ Q means, P implies Q.
c. a = b⇐a2 = b2 P ⇐ Q means, P is implied by Q.
Implications a, b and c are the respective P ⇔ Q means, P implies and is implied by B.
converses of the implications 1, 2 and 3
Worked Examples
For converse (a); 1. Two statements P and Q are defined by;
a. It is a dog ⇐ Ithas four legs P : an angle is 900
This means that “if it has four legs, then it is a Q : an angle is a right angle
dog” Write down the following implication in full
and determine whether P and Q are equivalent
The implication is false, because not only dogs a. P ⇒Q b. Q ⇒ P c. P ⇔ Q
have four legs. Since this implication is not
true, it is written as: Solution
It is a dog ⇐ It has four legs a. P ⇒Q : If an angle is 900, then it is a right
angle
Equivalent Statements b. Q ⇒P : If an angle is a right angle, then it is 900
Consider the statement below: c. P ⇔Q : If an angle is 900, then it is a right
„If he passed the examination, then he is angle and vice – versa.
promoted‟ Since P ⇒ Q and Q ⇒P are true, P ⇔Q is also
1. The implication is: true and hence P and Q are equivalent.
„He has passed the examination⇒ He is
promoted‟. This is a true statement. 2. Two statements P and Q are defined by
P : x {even numbers}
2. The converse is ; Q : 4x {even numbers }
He has passed the examination⇐ He is Determine whether P and Q are equivalent
promoted. This is also a true statement.
Solution
This implies that the implication as well as its
P ⇒Q : If x is even , then 4x is even
converse is true. The implication and its
converse can be simplified by the use of the Q ⇒P : If 4x is even , then x is even
arrow “⇔”. Thus, the implication appears as: P ⇒Q is true, but Q ⇒ P is false since 4 × 1
„He has passed the examination⇔ He is {even numbers} but 1 ∉ {even numbers}
promoted‟ Hence P ⇔ Q is false. Therefore P and Q are
not equivalent
Generally, when an implication P ⇒ Q and its
converse Q ⇒ P are both true, the statements P Exercises 27.5
and Q are said to be equivalent, expressed as A. For the following pair of statements P and
P⇔Q Q, determine whether P and Q are
equivalent

Baffour Ba Series, Further Mathematics for Schools Page 829


1. P : The sides of a triangle are equal C. Fill in the blanks of the following:
Q : The sum of two angles of a triangle is 1200 1. If he is a taxi driver, then he drives well
Paa is a taxi driver, therefore …..
2. P : ∆ ABC is isosceles triangle
Q : ∆ ABC is an equilateral triangle 2. If a triangle has two equal angles, then it is
an isosceles triangle
3. P : A number is divisible by 10 In ∆ABC, < A = < B. Therefore …
Q : The factors of a number include 2 and 5
3. If a quadrilateral is a parallelogram, then its
4. P : x2 – 3x + 2 = 0, Q : x = 1 or x = 2
diagonal bisects each other. A rhombus is a
parallelogram. Therefore ….
5. P : x + 2 > 3, Q : x + 1 > 3

B. State whether the following implications The Chain Rule of Implication


are true or false Consider the statements that P, Q and R are
1. x = 4 ⇒x2 = 16 2. x2 = 16 ⇒x = 4 defined by :
P : x > 7, Q : x > 5, R : x > 3 and the
3. x < 0 ⇒x2 > 0 4. x > 0 ⇒x2> 0
implications P ⇒Q : x > 7 ⇒x > 5
5. x2 > 0 ⇒x > 0
Q ⇒R : x > 5 ⇒ x > 3
6. A polygon has n sides ⇔It has n angles
This can be represented in a Venn diagram as
7. Each angle of a triangle is 600⇔It is an
shown below:
equilateral triangle
∪ = {numbers}
8. is the mediator of ⇐ is
A = {numbers > 7}
perpendicular to B = {numbers > 5}
C = {numbers > 3}
B. Fill in the blank space with ⇒, ⇐ or ⇔
where appropriate, if none is applicable, use C U
B
⇔ A

n(A) = n(B) A and B are


equivalent sets
(a + b)2 = 16 (ab)2 = 16
It is observed that A ⊂ B ⊂ C ⇒ A ⊂ C
x+y>3 x > 3 or y > 3
Area of a square The perimeter of Thus, if a number is greater 7, then it is also
is 25cm2 the square is 20cm greater than 3.
A= {x : 1< x < 6, A = {2, 3, 4, 5}
x is an integer} Generally, if P ⇒ Q and Q ⇒ R, the
It is a It is a square conclusively, P ⇒ R. This is called the chain
quadrilateral rule of implication, written as P ⇒ Q ⇒ R.
with 4 right
angles

Baffour Ba Series, Further Mathematics for Schools Page 830


The chain rule can be extended to more than R : x { integers }
three statements . i. Form valid implications from the statements
P, Q and R
Worked Examples ii. Form a chain of implication.
The following implications are true in a certain
college; 2. The following statements are true of
* If a student studies science subjects, he inhabitants of Toase;
studies mathematics S1 : only persons over 21 years pay basic rate.
* If a student studies mathematics, he does not S2 only persons who pay basic rate can register
study history. as voters.
i. Represent the implications symbolically and S3 : only persons who register as voters can
use the chain rule of implication to deduce a vote in an election.
valid implication. i. Express the statements S1, S2 and S3 as
ii. Illustrate in a diagram. implications.
ii. Construct a chain of implications.
Solution iii. Which of the following implications are
i. P : a student studies science subject valid deductions from S1, S2and S3?
Q : a student studies mathematics a. P1 : every voter in an election is over 21
R : a student does not study history years old.
The implications are: b. P2 : every person who pays basic rate can
P ⇒ Q; Q⇒R vote in an election.
The implication P ⇒ R is valid, because P ⇒R : c. P3 : every person who registers as a voter
If a student studies science subjects, he does not pays basic rate.
study history d. P4 : every person over 21 years of age can
register as a voter .
ii. U = {students in the college}
A = {Science students} Truth Tables
B = {mathematics students} A truth table is a table which gives all the truth
C = {non – history students} values of a compound statement. The table is
filled in by considering all possible
C U
B combinations of true and false for P and Q and
A then filling in the results for the various
connectors mentioned above.

The truth table for Negation Statement


Exercises 27.6
1. Consider the following statements: P P
P : x {natural numbers } T F
Q : x {rational numbers } F T

Baffour Ba Series, Further Mathematics for Schools Page 831


If P is true, then its negation P is false and Q : 2 is a factor of 15 = F
vice versa. The statement is false.

The “and” Truth Table P Q P˄Q


Below is the truth table for the conjunction of T F F
two simple statements p and q.
iv. 10 is not a multiple of 2 and 2 is a factor of 15.
P Q P˄Q P : 10 is not a multiple of 2 = F
T T T Q : 2 is a factor of 15 = F
T F F The statement is false.
F T F
F F F P Q P˄Q
F F F
Worked Examples
Use truth table to show whether each of the Exercises 27.7
following statement is true or false. Use truth table to show whether each of the
i. 10 is divisible by 5 and 10 is a multiple of 2. following statement is true or false.
ii. 2 is a factor of 15 and 10 is a multiple of 2. 1. 4 is a perfect square and four is the square
iii. 10 is a multiple of 2and 2 is a factor of 15 root of 16.
iv. 10 is not a multiple of 2 and 2 is a factor of 15 2. x is odd, and x2 is odd.
3. x is a prime number greater than 2 and x is
Solution odd.
i. 10 is divisible by 5 and 10 is a multiple of 2. 4. x is a prime factor of 30 and x is a prime
P : 10 is divisible by 5 = T factor of 40.
Q : 10 is a multiple of 2 = T 5. x is a prime factor of 18 and x is a prime
The statement is true. factor of 24.

P Q P˄Q The “or” Table


T T T Below is the truth table for the disjunction of
two simple statements p or q.
ii. 2 is a factor of 15 and 10 is a multiple of 2.
P : 2 is a factor of 15 = F
P Q P˅Q
Q : 10 is a multiple of 2 = T T T T
The statement is false. T F T
F T T
P Q P˄Q F F F
F T F
Worked Examples
iii. 10 is a multiple of 2 and 2 is a factor of 15. Use truth table to show whether each of the
P : 10 is a multiple of 2 = T following statement is true or false.

Baffour Ba Series, Further Mathematics for Schools Page 832


1. 2 is either an odd number or a factor of 15 If, Only if, and If and Only if Table

Solution P Q P ⇒Q P ⇐Q P⇔Q
2 is either an odd number or a factor of 15 T T T T T
P : 2 is an odd number = F T F F T F
Q : 2 is a factor of 15 = F F T T F F
F F T T T
P Q P˅Q
F F F Note that the P ⇒ Q column can be obtained
from (P⇒Q) ˄ (P ⇐ Q).
The statement is false
Worked Examples
2. A triangle has three sides, or a parallelogram Use truth table to show whether the statement is
has three sides. true or false.
“The product of two numbers is even if and
Solution only if both numbers are even”
P : A triangle has three sides = T
Q : A paralellogram has three sides = F Solution
P : the product of two numbers is even
P Q P˅Q Q : both numbers are even ,
T F T
P ⇒Q : If the product of two numbers is even
The statement is true then both numbers are even. T

Exercises 27.8
P ⇐Q : If two numbers are even, then their
Use truth table to show whether each of the
product is even. T
following statement is true or false.
1. An acute angle is 900 or a right angle is
P Q P ⇒Q P ⇐Q P⇔Q
between 900
T T T T T
2. The sum of interior angles of a triangle is
less than 1000, or an octagon has seven vertices The statement is true.
3. x is even or x2 is odd.
4. 6 is a perfect square or eight is the square 2. If 2 is an odd number then 2 is not a prime
root of 16 number.
5. The product of two numbers is even, or the
sum of both numbers is even. Solution
6. 83 is an odd number or 38 is a composite P : 2 is an odd number F
number. Q : 2 is not a prime number F
7. 169 is a perfect square or 255 is a perfect The statement is true
square.

Baffour Ba Series, Further Mathematics for Schools Page 833


3. If the number 21 is divisible by 10, then 10 is 3. If a rectangle is a parallelogram, then a
divisible by 21 square is parallelogram
4. If 6 is a factor of 36, then 6 is a perfect
Solution square
P : 21 is divisible by 10 F 5. If 14 is a prime number, then 97 is a an odd
Q : 10 is divisible by 21 F number
The statement is true 6. If a kite has congruent angles, then a
rectangle has congruent angles
4. If the number 121 is divisible by 11, then 5 is
divisible by 3 Negation and Contrapositive Table

Solution P Q ¬P ¬Q P⇒ Q ¬Q⇒ ¬P
P : 121 is divisible by 11 T T T F F T T
Q : 5 is divisible by 3 F T F F T F F
The statement is false F T T F T T
F F T T T T
5. If a triangle is a polygon, then a polygon is a
triangle Exercises 27.9B
A. In the following exercises, P, Q, R,
Solution and S will represent truth statements.
P : A triangle is a polygon T Construct the truth tables for the following
Q : A polygon is a triangle F statements, and give their converse and
The statement is false contrapositive:
1. ¬P ⇒Q
Exercises 27.9 A 2. P ⇒ ¬ Q
Use true or false for each 3. (P ˅ Q) ⇒ R
1. A whole number is divisible by ten if and 4. (P ˄ Q) ⇒ (R˅ S)
only if its decimal numeral ends with zero. 5. (P ⇒ Q) ⇐ (R⇒S)
2. Two lines are parallel if and only if they do
not meet, however far they are extended. B. Negate the following statements:
3. Two figures have the same area if they are 1. P ˄ Q
congruent. 2. (P ˅ Q) ˄ (R˄ S)
4. The product of two numbers is even if and 2. (P ˄ ¬ Q) ˅ (¬R ˅ S)
only if both numbers are even.
Valid Arguments
B.1. If two acute angles are congruent and A valid argument is a series of statements in
complementary then their measure i 450. T which each statement follows logically from
2. If 8 is divisible by 5, then 8 is a factor of 5. T the preceding ones. Thus, an argument begins

Baffour Ba Series, Further Mathematics for Schools Page 834


with a given statement and ends with a final From the diagram, since k A, we conclude
statement (conclusion). that k B. This confirms that Nana Amoah is a
good runner.
The student‟s duty is to determine whether the
argument is valid/logical or invalid/ illogical by 2. Obtain a valid conclusion from the following
analyzing the given statement to see if it statements:
matches with the conclusion. This is easily P : Every science student study Chemistry.
determined by representing the statement in a Q : Brown does not study chemistry.
Venn diagram.
Solution
In order to avoid making false deductions or U = {students}
conclusions, mathematicians assumes that all C = {chemistry students}
the premises (generalization, definitions..) S = {science students}
are true Brown = k U
C
k ∉C, S
Worked Examples
1. Determine whether the following statement
K
is valid or not:
If he is a good tennis player, he can run well.
Nana Amoah is a good tennis player. Therefore, From the Venn diagram, we conclude that
Nana Amoah can run well. „Brown is not a science student‟

Solution 3. i. Illustrate the statements P Q and R on a


Rewrite the statement as follows: Venn diagram
P : All good tennis players can run well. P : All my friends are religious
Q : Nana Amoah is a good tennis player. Q : Religious people are honest people
Therefore, Nana Amoah can run well. R : Tom is an honest person since he is my
friend
The statement P can be represented in the ii. Determine whether R is a valid conclusion
following Venn diagram: from P and Q
∪ = {people}A = {good tennis players}
B = {goodrunners} Solution
U i. U = {people}
Nana Amoah = k B
k A, A F = {my friends} R = {religious people}
K H = {honest people} U
b = Tom, b F, H
R
F⊂ R⊂ H F
A ⊂ B (because all good tennis players are
K
good runners)

Baffour Ba Series, Further Mathematics for Schools Page 835


Exercises 27.10
A. Study the arguments and state whether 2. P : In Tanokrom, every old man uses a
they are valid by using Venn diagrams to walking stick.
illustrate your answers; Q : Atiah is an oldman since he uses a walking
1. All odd numbers are whole numbers. stick.
3 is an odd number.
 3 is a whole number. 3. P : My friend are all clever people
Q :Amuzu is a clever person since he is my
2. All boys over 15 years acan wear long pants friend.
to school.
Ablo is 16 years old. 4. P : People who lives in glass house donot
 Ablo can wear long pants to school throw stones.
Q : David lives in a glass since he does not
3. All flowers are beautiful to look at. throw stones.
Roses are flowers.
 Roses are beautiful to look at 5. P : When it rains , there is no school
Q : Yesterday it rained since there was no
4. All locally manufactured goods are marked school.
“Made in Ghana”
The portable radio is marked, “Made in China” C. Each case, determine by drawing Venn
 the portable radio is not manufactured in diagrams whether the statement R is a valid
Ghana conclusion from the statements P and Q
1. P : Rich men live in big houses
5. All squares are rhombuses Q : Mr. Owusu lives in a big house
ABCD is not a rhombus R: Mr. Owusu is a rich man
 ABCD is not a square
2. P : Allboys like football
6. All triangles having at least two equal sides Q : Dzifa likes football
are isosceles triangles. R : Dzifa is a boy
An equilateral triangle has three equal sides.
 an equilateral triangle is an isosceles triangle. 3. P : Basketball people are tall people.
Q : Neos is a basketball player.
B. In each case,draw a Venn diagram to R : Neos is tall.
determine whether the statement, Q is a
valid conclusion from the statement P. 4. P : Vegetarians do not eat meat.
1. P : Lazy students fail their examination. Q : Hindus are vegetarians.
Q : Aku is lazy since she failed her R : Hindus do not eat meat.
examination.

Baffour Ba Series, Further Mathematics for Schools Page 836


28 CORRELATION AND REGRESSION Baffour Ba Series

Introduction to Correlation and Regression between the two variables. It is also known as
Analysis two-way frequency distribution. For example,
Correlation analysis is used to quantify the height and weight of students in a class being
association between two continuous variables collected. In this case one variable is labelled x
(e.g., between an independent and a dependent and the other y.
variable or between two independent variables).
The Scatter Diagram
Regression analysis is a related technique to The scatter diagram is used to graphically
assess the relationship between an outcome display bivariate distributions. One variable is
variable and one or more risk factors or along the x – axis and the other along the y –
confounding variables. The outcome variable is axis.
also called the response or dependent variable
and the risk factors and confounders are called The scatter diagram is a useful tool in
predictors or explanatory or independent examining relationships, especially between
variables. two variables.

In regression analysis, the dependent variable is A plot of the sample data on the graph gives a
denoted by "y" and the independent variables
visual indication of the degree of association
are denoted by "x".
between two variables, say x and y. That is, by
Univariate and Bivariate Distributions examininthe scatter diagram, we can :
Univariate Frequency Distribution
1. observe whether the vraiables are indeed
When data is classified on the basis of single
variable, the distribution is known as univariate related or not;
frequency distribution. 2. choose an appropriate regression model for
estimation.
It aims to make description about the particular
variable. It is also known as one-way frequency The pair of observations (x, y) represented by
distribution. For example height of students in a the plot are called scatter points.
class.

Forms of Correlation and Type of Scatter


Bivariate Frequency Distribution
When a data is classified on the basis of two Diagrams
variables, the distribution is known as The various types of association between x and
bivariate frequency distribution.
y are shown in the digrams below;

It aims to determine the empirical relationships

Baffour Ba Series, Further Mathematics for Schools Page 837


Positive linear association. x 12 24 30 34 47 58 68
y 32 44 47 58 73 72 88
y

Plot a scatter diagram for the data.

x Solution
y
Negative linear Association
y

90
80
70
60
x 50
40
Non – linear Association
30
y
20
10
x
10 20 30 40 50 60 70

2. The number of abscence for females (x) and


x the number of absence for males (y) for the first
10 days of the re- opening of a certain school
No Association
are shown in the table below.
y

x 5 0 3 1 2 2 5 5 3 4
y 1 2 1 3 3 4 3 0 4 2

Plot a scatter diagram for the data.


x
Worked Examples
Solution
1. The test scores of 7 students in English (x)
and Mathematics (y) are shown below:

Baffour Ba Series, Further Mathematics for Schools Page 838


y
y

4 ( ̅ , ̅)

3 x

2
Steps (If there is a fair dagree of scatter)
In this case, two regression lines A and B are
1
drawn:
1. A line of regression of x on y, which gives an
1 2 3 4 5 6 7 x extimate of x, given a value of y
2. A line of regression of y on x, which gives an
The Line of Best Fit extimate of y, given a value of x.
y
A line of best fit is a straight line that is the
best approximation of the given set of data. It is
used to study the nature of the relation between
two variables.

Linear regression consists of finding the best- ( ̅ , ̅)


fitting straight line through the points. The best- x
fitting line is called a regression line.

Equation of the Line of Best Fit A line of regression of x on y, which gives an


a. By Graphical Method extimate of x, given a value of y.
A line of best fit can be roughly determined In this case, it is assumed that the values of y
using an eyeball method by drawing a straight are accurate. The line of regression is drawn as
line on a scatter plot so that the number of follows:
points above the line and below the line is i. Cal culate M( ̅ , ̅) , the respective means of x
about equal (and the line passes through as and y values
many points as possible). ii. Draw a line parallel to the x – axis through
M( ̅ , ̅) to divide the points into two groups.
Steps (If there is little scatter) iii. Let MA be the mean of the numbers above
Draw the line through the point ( ̅ , ̅) where ( ̅ , the parallel line drawn in (ii) and hence find MA
̅) are the respective means of x and y – values. ( ̅ , ̅)

Baffour Ba Series, Further Mathematics for Schools Page 839


iv. Let MB be the mean of the numbers below ̅= = = 39
the parallel line drawn in (ii) and hence find MB
̅= = = 59.14 = 59
( ̅ , ̅)
iv. Draw a line through M( ̅ , ̅), MA ( ̅ , ̅) and M( ̅ , ̅) = (39, 59)
MB ( ̅ , ̅) as the line of best fit.
y

A line of regression of y on x, which gives an


extimate of y, given a value of x.
In this case, it is assumed that the values of x 90

are accurate. The line of regression is drawn as 80

follows: 70
i. Calculate M( ̅ , ̅), the respective means of x 60 (39, 59)
and y values. 50
ii. Draw a line parallel to the y – axis through 40
M( ̅ , ̅) to divide the points into two groups. 30
iii. Let ML be the mean of the numbers above 20
the parallel line drawn in (ii) and hence find ML 10
( ̅ , ̅)
10 20 30 50 60 x
iv. Let MR be the mean of the numbers below 40 70
the parallel line drawn in (ii) and hence find MR
( ̅ , ̅) 2. Given the table below.
iv. Draw a line through M( ̅ , ̅), ML ( ̅ , ̅) and
MR ( ̅ , ̅) as the line of best fit. x 1 3 4 6 8 9 11 14
y 1 2 4 4 5 7 8 9
Worked Examples
The following table shows the examination a. Draw a scatter diagram for the data.
marks of seven students in French (x) and b. Draw the regression line y on x and use it to
English (y). estimatae the value of y corresponding to x =
10.
F 12 24 30 34 47 58 68 b. Draw the regression line x on y and use it to
E 32 44 47 58 73 72 88 estimate the corresponding value of x when
y=6.
Draw a scatter diagram of the data and draw the
line of best fit. Solution
b. ̅= = =7
Solution
F (x) 12 24 30 34 47 58 68 ̅= = =5
E (y) 32 44 47 58 73 72 88 M(7, 5)

Baffour Ba Series, Further Mathematics for Schools Page 840


For regression line y on x draw a line through c. To draw the regression line x on y, draw a
M, parallel to the y – axis and identiy the points line through M(7, 5) and parallele to the x –
below M and the points above M. axis.
Points below (left) M
Points above the parallel line.
x y
1 1 x y
3 2 9 7
4 4 11 8
6 4 14 9
= 14 = 11 = 34 = 24

= = 3.5 = = 2.75 = = 11.33 = =8


Plot (3.5, 2.75) Plot (11.33, 8) (As shown on the graph)

Points above (right) M Points below the parallel line

x y x y
8 5 1 1
9 7 3 2
11 8 4 4
14 9 6 4
= 42 = 29 = 14 = 11

= = 10.5 = = 7.25 = = 3.5 = = 2.75


Plot (10.5, 7.25) Plot (3.5, 2.75)

Regression line is drawn through : Regression line x on y is passes through the


M(7, 5), (3.5, 2.75) and (10.5, 7.25) points:
M(7, 5), (11.33, 8) and (3.5, 2.75).
From the graph, when x = 10, y = 6.9

Baffour Ba Series, Further Mathematics for Schools Page 841


a.
y

x
1 2 3 4 5 6 7 8 9 10 11 12 13 14

From the graph, when y = 6, x = 8.6 3: Compute the y -intercept of the line by using
the formula:
b. The Least Square Method b= ̅–m ̅
A more accurate way of finding the line of best
fit is the least square method . 4. Use the slope m and the y -intercept b to
form the equation of the line.
The following steps are used to find the
equation of line of best fit for a set of ordered Worked Example
pairs (x1, y1), (x2, y2),...(xn, yn) . 1. Use the least square method to determine the
1. Calculate the mean of the x -values and the equation of line of best fit for the data. Then
mean of the y -values. plot the line using the data below.
̅= and ̅ =
x 8 2 11 6 5 4 12 9 6 1
y 3 10 3 6 8 12 1 4 9 14
2. The following formula gives the slope of the
line of best fit:
( ̅) ( ̅) Solution:
m= Plot the points on a coordinate plane .
( ̅)

Baffour Ba Series, Further Mathematics for Schools Page 842


y ̅= = 6.4

̅= =7

Now calculate xi − ̅ , yi − ̅, (xi − ̅ ) (yi − ̅),


14 and (xi − ̅ )2 for each i .
12
10
8
6
4
2

2 4 6 8 10 12 14 16 x

Calculate the means of the x -values and the y -


values.

i xi yi xi − ̅ yi − ̅ (xi − ̅ ) (yi − ̅) (xi − ̅ )2


1 8 3 1.6 -4 - 6.4 2.56
2 2 10 - 4.4 3 - 13.2 19.36
3 11 3 4.6 -4 - 18.4 21.16
4 6 6 - 0.4 -1 0.4 0.16
5 5 8 - 1.4 1 - 1.4 1.96
6 4 12 - 2.4 5 - 12 5.76
7 12 1 5.6 -6 - 33.6 31.36
8 9 4 2.6 -3 - 7.8 6.76
9 6 9 - 0.4 2 - 0.8 0.16
10 1 14 - 5.4 7 - 37.8 29.16
∑ (xi − ̅ )(yi − ̅)
∑ (xi − ̅ )2 = 118.4
= - 131

Baffour Ba Series, Further Mathematics for Schools Page 843


Calculate the slope. When x = 0, y = 14
( ̅) ( ̅) When y = 0, 0 = - 1.1x + 14
m=
( ̅) 1.1x = 14
m= x = 12.7
m ≈ −1.1
x 0 12.7
Calculate the y -intercept. y 14 0

Use the formula to compute the y -intercept.


b= ̅–m ̅ 2. The following table shows the marks
b = 7 − (−1.1 × 6.4) obtained by some students in the two papers of
b =7 + 7.04 a subject.
b ≈14.0
Paper 1 (x) 51 45 23 47 18
Paper 2 (y) 41 38 11 28 12
Paper 1(x) 31 27 30 21 23
Paper 2 (y) 14 14 17 15 17

a. Plot a scatter diagram to represent the


14
information.
12 b. Find ̅ , the mean of x and ̅, the mean of y,
10 and plot ( ̅ , ̅) on the diagram.
8
c. Draw the lline of best fit to pass through ( ̅ ,
̅)
6
d. From your graph, determine;
4 i. the relationship between x and y .
2 ii. the value of y when x = 25
x
2 4 6 8 10 12 14 16 Solution
-2
b. ̅=
̅= = 31.6
Use the slope m and y –intercept b to form the
equation of the line of best fit.
̅=
The slope of the line is m = −1.1 and the y - ̅= = 20.7
intercept is b = 14.0 .
Therefore, the equation is y = −1.1x +14.0 . d. The relationship between x and y;
Gradient , m = = = 0.83

Baffour Ba Series, Further Mathematics for Schools Page 844


Let m = 0.83, and ( ̅ , ̅) = (31.6, 20.7) a. Plot a scatter diagram of the data.
Substitute in y – = m (x – ) b. If ̅ is the mean of x and ̅ is the mean of y,
find :
y – 20.7 = 0.83 (x – 31.6) i. the mean values of and of x and y
y – 20.7 = 0.83x – 26.228 respectively for which x is graete than ̅ .
y = 0.83x – 26.228 + 20.7 ii. the mean values of and of x and y
y = 0.83x – 5.53 respectively for which x is graete than ̅ .
c. i. Plot the points ( ̅ , ̅) , ( ̅ , ̅ ) and ( ̅ , ̅ )
y on your scatter diagram.
ii. Form the equation of the line of best fit and
use it to predict possible mathematics mark of a
45
student whos scored 7 in English.
40
2. In a certain hospital, the mean weight of a
35
certain group of babies born on the same day
30 were observed over several month at intervals
of one month. The results over the first 7
25 months are in the table below;
20
Age in Months Mean weight
15 (A) in kg (W)
0 2.50
10 1 3.75
2 4.20
5
3 5.70
4 6.40
10 20 30 40 50 60 70 80 x
5 7.37
6 8.25
7 9.60
From the graph, when x = 25, y = 15
a. Draw a scatter diagram of the mean weight,
Exercises 28.1 W, against the age, A.
1. The table below shows the marks (out of 10) b. Draw the line of best fit of W on A, and
in a class test as english ans amths of a sample estimate its gradient.
of 10 students selected at random from a class. c. From your graph, estimate the mean weight
of a baby whose age is 5.5 months.
English (x) 3 3 4 5 6 4. The data in the table below shows the years
Maths (y) 4 6 5 4 6 of experience and the salar of 7 eorkers selected
English (x) 7 7 8 9 6 at random from an industry.
Maths (y) 8 9 7 10 9

Baffour Ba Series, Further Mathematics for Schools Page 845


Worker A B C D E F G x Rank y Rank d d2
Years of 3 7 4 9 11 16 8
experience (x)
Salary (Gh000) 13 15 14 17 15 19 17
(y)
Substitute the values of n and d2 in
a. Draw a scatter diagram of the salary y,
=1–
against the experience in years x, ( )

b. Find:
i. ̅ is the mean of x; Worked Examples
ii. ̅ is the mean of y; 1. The scores for nine students in physics and
c. Draw the line of best fit passing through ( ̅ , mathematics test are as follows:
̅) and find the equation of the line. Physics 35 23 47 17 10 43 09 06 28
d. From your graph estimate the expected Maths 30 33 45 23 08 49 12 04 31
salary of a worker who had 10 years of i. Compute the students rank in the two subjects
experience. and compute the Spearmans rank correlation
coefficient.
Spearman‟s Rank Correlation Coefficient ii. Interprete your results.
The spearmans correlation coefficient can
take values from +1 to – 1. of + 1 indicates Solution
a perfect association of ranks, of 0 indicates Physics Ranks Maths Ranks
35 3 30 3
no association between ranks and of - 1
23 5 33 5
indicates a perfect negative association of
47 1 45 2
ranks. The closer is to zero, the weaker the 17 6 23 6
association between the ranks. 10 7 8 8
43 2 49 1
The Spearman‟s rank correlation coefficient is 9 8 12 7
calculated by : 6 9 4 9
28 4 31 4
=1– ( )
Where n is the sample size and is the sum P Ranks M Ranks d d2
of the squares of the difference in ranks. 35 3 30 3 2 4
23 5 33 5 2 4
To compute this value, 47 1 45 2 1 1
i. Find the rank for each individual subject. 17 6 23 6 0 0
This is done by ordering the scores from 10 7 8 8 1 1
greatest to smallest; assign the rank 1 to the 43 2 49 1 1 1
highest score, 2, to the next highest and so on. 9 8 12 7 1 1
6 9 4 9 0 0
ii. Copy and complete the table below; 28 4 31 4 0 0

Baffour Ba Series, Further Mathematics for Schools Page 846


Sum up all d2 values; 3. In an examination, candidates were ranked
= 4 + 4 + 1 + 0 + 1 + 1 + 1 + 0 + 0 = 12 in Mathematics and English as shown in the
table below.
Since the ranks are not tied, substitute the
values in the formula : Maths 9 6 7 2 5 1 8 3 10 4
Eng 5 7 1 4 10 3 2 8 6 9
=1– ( )
( )
=1– i. Calculate the Spearmans rank correlation
( )
coefficient.
=1–
ii. Interprete your results.
= 0.9
Solution
ii. There is a prefect association between
physics and mathematics. Maths Eng d d2
9 5 4 16
2. Five finalist in a beauty pageant were ranked 6 7 -1 1
by two judges X and Y as shown in the table. 7 1 6 36
2 7 -2 4
Judges Ann Linda Suzy Rose Erica 5 10 -5 25
X 1 4 3 5 2 1 3 -2 4
Y 3 2 4 5 1 8 2 6 36
3 8 -5 25
10 6 4 16
Calculate the Spearmans rank correlation.
4 9 -5 25

Solution
= 188
( )
Rank of X Rank of Y d d2 =1– ( )
1 3 -2 4 =1–
4 2 2 4
3 4 -1 1 = 1 – 1.1394
5 5 0 0 = - 0. 1394
2 1 1 1
ii. There is a low negative correlation between
= 4 + 4 + 1 + 0 + 1 = 10 Mathematics and English.
=1– ( ) Exercises 28.2
( )
=1– 1. Use the data below to calculate the
( )
spearman‟s rank order correlation coefficient
=1–
and interprete your results.
= 0.5

Baffour Ba Series, Further Mathematics for Schools Page 847


English 56 75 45 71 62 Canada 7 6
Maths 66 70 40 60 65 USA 1 7
English 64 58 80 76 61 Finland 6 8
Korea 9 9
Maths 56 59 77 67 63
Spain 4 10

2. On a certain market day, the number of fish i. Calculate the Spearman‟s rank correlation
displayed (fish num) and the rating of fish coefficient.
quality (fishgood) on a 1 – 10 scale is shown in
ii. Interprete your results.
the table below. 4. Jack and Jill were asked to rank their
favourite vocation sports. The table below gives
Fishnum 32 41 31 38 21 17 their ranking.
Fish good 6 5 3 3 7 7
Fishnum 13 17 22 24 11 20 Soccer Hockey Racing Rugby Cricket
Fish good 9 9 8 6 9 8 Jack 2 4 1 5 3
Jill 3 2 1 5 4
i. Calculate the Spearman‟s rank correlation
coefficient. Calculate the Spearman‟s rank correlation
ii. Interprete your results. coefficient.

3. The following is a small selection of 5. Show that for the following data = -1
countries ranked according to the Human between the ranking of these two judges. Best
Development Index (HDI) and income per ice cream is given rank 1.
capital.
Country Income HDI Ice
Rank (x) Rank (y)
cream A B C D E F G H I J
Norway 3 1
Judge 1 1 2 3 4 5 6 7 8 9 10
Iceland 2 2
Judge 2 10 9 8 7 6 5 4 3 2 1
Sweden 10 3
Belgium 8 4
Japan 5 5

Baffour Ba Series, Further Mathematics for Schools Page 848


Answers to exercises
Exercises 1.2
A. 1. {3, 6} 2. {4, 16} B. 1. {0, 2, 4, 6, 8} 2. {2, 3, 5,6, 7, 9, √ - 3 3. 6√ 4. 98
11, 12, 13, 17…} 3. {-2, -1, 0, 1, 2, 3, 4, 5, 6} 4. {-3,-2, -1, 0, 1,
2, 3, 4, 8} C. i. 1, 2, 3, 4, 5, 6, 7, 8, 9, 10, 11, 12, 13, 14, 16,
Exercises 2.10
17, 18, ii. 2 iii. 2, 4, 6, 8, 10 iv. 2, 3,4, 5,6, 7, 8, 10, 12, 14, √ √ √ √
16, 18 v. 1, 4, 6, 8, 10 vi. {12, 14, 15, 16, 18} A. 1. 2. – 18 + 6 √ 3. 4. 5. - 4 –
√ √ √ √
2√ 6. √ √ 7. 8. 9. B. 1.
Exercises 1.3
A. 1. 2, 4, 6, 10 2. 3, 5, 7, 9 √ √ √
2. 1 + √ 3. 4 + 2 √ 4. 5. 6.
√ √
Exercises 1.4 7. 8. 9. - 6 + √ C. 1. - 2 – 2 √ 2. 12 -
2.a. 20 b. 12 c. 8 3. 7, 16 4. 9 5. a. 45, b. 14, c. 33, d. 55 6. a. √ √
√ 3. 3 + 2 √ 4. D. 1. 2. 4 – 2 √
10, b. 30 7… 8. ii. 9 9. i. 6 ii. 28. iii. 56 10. 13
√ √ √ √ √
3. 4. - 4 + 2 √ C.P. 1. 2. – √
Exercises 1.5 √ √ √ √ √ √
1. a. 24 b. 6. 2. a. 10 b. 33 3. 4. 5. 6. 7. +
√ √ √ √
+ + 8. 9. -7 – 3 √ B. 1. 2. -1 + 7
Exercises 1.6
1. b. 6 2. 3, 7, 3 3. 3.i. a. {18} b. {6, 12} c. {3, 15} 4. a. 47 √ √ √ √
√ 3. 4. C. 1. ; – 2 √ , 2.

b. 22 c. 18 5. 6 6. a. 9, b. 10, c. 12. d. 168, e. 159 7. 4, 14 8.
10% 9. 10 10. 50 12. 14. 10 + =√ 3. 2.036

Ex 2.1 Exercises 2.11


A. 1. 5√ 2. 32√ 3. 26√ 4. 28√ 5. 13 √ 6. 7√ 6. √ √ √
1. 2.
√ –√ 7. -18 √ B. 1. 9√ 2. 6√ B. 1. 13√ 2.
6√ 3. 25 √ 4. 18√ 5. 19√ 6. 4√ + √ Exercises 2.12
A. 1. 3 2. 52 3. 37 4. 5. 3 6. 7 B. 1. -1 2. 10 3. 5 4. - 6
Exercises 2.2
A. 1. 5 2. 8 3. 2 4. 3 √ 5. 6. 2 B. 1. 2. 3. 4. 5. 4 6. -1.65 C. 1. 3 2. 3. 3 or -5 4. 5 or – 4 5. 1 or -1 6.
5 or -2 7. -2 or -3 8. -3 or -9 9. 4
Exercises 2.3
1. 40√ 2. 16√ 3. 100√ 4. 600√ 5. 500 √ Exercises 2.12B
1. x = 0 2. x = 2.34 3. x = 2 4. x = 2 5. 2 6. 5
Exercises 2.4
Exercises 2.13
1. √ + √ 2. 3√ – √ 3. 10 + 6√ ) 4. 6√ + A. 1. x = 3 y = 2 2. x = -11 y = – 3 3. x = 0, y = -3 4. x = 15,
√ ) 5. 12 – 9 √
y = - 4 5. x = 6, y = 0 6. x = 0, y = 0 B. 1. 2, 2. . , /
Exercises 2.5 3. . , / 4. . , / c. 1. (3, 1) 2. (4, 5) 3. (5, 2) C.P ;
1. 14 + 7 √ 2. 26 + 8 √ 3. 10 - √ + 4√ + 4 √ 4. 1. 179.5555 2. 3.1463

(√ – 2 ) (√ – 5) 5. 17 – 7 √ 6. +
Exercises 2.14
Exercises 2.6 1. ( √ ) 2. ( √ ) 3. ( √ ) 4. ( √
1. – 11 2. – 4 3. 3 4. - 4 5. – 2 ) 5. ( √ ) 6. ( √ √ )

Ex 2.7 Exercises 3.2


1. 13 – 4 √ 2. a - 6√ – √ ) + 6 3. 77 + 30√ 4. 28 - 1. 4 2. 32, 2 3. i 39 ii. 55 iii. 5811 iv. 1 or -9 4. 25
12√ 5. 28 √ CP: p = 7 + 10n

Exercises 2.8 Exercises 3.7


√ A. 1. 2. - 1 3. e =
A. 1. 5 2. 5√ 3. √ 4. √ B. 1. 3 – √ 2. 1 +
√ √
3. 2 – √ 4. 3 + 5. – 6. –√ Exercises 4.7
1.onto 2. onto 3. Onto 4.one to one 5. Both 6. None
Exercises 2.9
√ √ √ √ Exercises 4.8
A. 1. 2. 3. 4. B. 1. √ 2. √ +
1. E 2. E 3. O 4. E 5. E 6. O

Baffour Ba Series, Further Mathematics for Schools Page 849


Exercises 4.10 Exercises 5.4
A. 1. {x : x R, x ≠ 9} 2. {x : x R, x ≠ 4} 3. {x : x R, x ≠ - 1. 7x + 3 2. 6x2 + 38x + 60 3. x2 + 19x + 60
6} 4. {x : x R,} B. 1. {x : x R} 2. {x : x R, x ≥ -2}
4. {x : x R, x ≥ 2} 5. {x : x R, x ≥ 3} C. 1. {x : x R} Exercises 5.5
2. {x : x R, x ≠ } 3. {x : x R } 4. {x : x R, x ≠ -1 or x B. 1. 2x2 + x + 6 ; 7 2. 3x2 – 16x + 63 ; -244 3. x2 – x + 17 ; -
56 4. 5x2 + 14x; 71
≠ 3} 5. {x : x R, x ≥ } 6. {x : x R, x ≥ -3 or x ≠ } 7. {x : x
R, x ≥ - or x ≠ 1 or x ≠ - 1 8. {x : x R, x ≠ 2 or x ≠ - 5 Exercises 5.6
A. 1. -4 2.10, 3.5, 4.- 29, 5. - 4 B. 1. 11 2. 7 3. -25 4. 11 5. -
4 6. -26
Exercises 4.11
A. 1. {y : y R, y ≥ -5} 2. {y : y R, y ≠ 1} 3. {y : y R, y ≠
Exercises 5.7
0} 4. {y : y R, y ≠ 3} 5. {y : y R, y ≠ 0} 6. {y : y R, y ≥ -
1. – 5 2.– 6 3. 6 4. 4 5. 15 6. a. -8 b. - 1
}
Exercises 5.8
Exercises 4.12 1. , 2. 3, 6 3. a. 5, 6 b. -14 4. ,
A. 1. i. -4x – 3 ii. 4x + 4h – 3 2. 25 + 10 h + h2. b. 10 + h
3. i. ii. 25a2 iii. 4.i. ii. iii. iv. 1.4 Exercises 5.9
A. 1. yes 2. Yes 3. yes 4. No B. 1. Yes 2. Yes 3. Yes 4. Yes C. 1.
B. 1. a. b. c. e. f. 2. a.
(x + 1), (x + 2) 2. (x + 3), (x + 5) 3. (x + 2), (3x - 1) 4. (x - 4) ,

b. c. d. -√ 2
3. i. (a + b ) + 3a + 3b + 4 ii. h – 1 - (2x – 3)

C. 1. i. 0 ii. undefined iii. . /. , / iv. 0 Exercises 5.10


v. undefined 1. k = 19, (2x + 3) , (3x – 1) 2. k = 7 ; (x + 1) , (x + 2) 3. 2 4.

Exercises 4.13 5. a. 7 b. c. – 17 C. P. 1. k = 2
A. 1.a. 45.1 and 38.9 2. a. 11. 5 and 17.25 B. 1. i. 25, 49 ii.
15, 14, 21, 22, 26 33. 2. i. 2 ii. 2 iii. 1 Exercises 5.11
1. 2, -10 2. 5, 19 3. , 4. x = 2, x = -3; p = 3 , q = 12 5. -
Exercises 4.17 5, 19 C.P 1. a = 2, b = -1 and c= -13
1. a. g(x) = 3x f(x) = 2x + 1 b. g(x) = x2, f(x) = 2x
2. If f (x) = √ , g(x) = x2 + 1 3. f(x) = √ and g(x) = x or f(x) = Exercises 5.12
1. (1, -5) 2. . , , / 3. a. (5, -1) b. (x + 2) (x - 1) (2x + 3)
√ and g(x) =
4. a. – 20 b. – 6 5. a. f(2) = 0 b. (3, 8) c. b 2 – 4ac < 0 6.
Exercises 4.18 Exercises 5.13
1. x2 + 6x + 7 2. 4x2 – 6x 1. x3 - 3x2 + 10x = 0. 2. x3 - 3x2 - 4x + 12 = 0.
3. x4 - 4x3 - 9x2 + 4x + 8 = 0. 4. x4 - 3x3 - 13x2 + 15x = 0.
Exercises 4.30
1. i. 2 ii. 5 iii. 10 iv . -6 v. -30 vi. -16 vii. – 22 viii. 52 Exercises 5.14
ix. x. xi. 5 xii. 2. i. 12 ii. 9 iii. 22 iv. – 75 vii. A. 1.;(x – 3) (x – 2)(x + 4); 3, 2, - 4 2. (2x – 1) (x – 1)(x + 1); ,
– 2 viii. 1, - 1 3. -1, 3 4. -3 2√ . 5. 7; 2, 3 6. a = 2, b = 9; k = 7. k =
4 or . 8. p = -1, q = 14, r = -8;1, 2, 4 B. 1. (x – 1) (x + 2) (x +
Exercises 5.2
A. 1. x3 + 9x – 7 2. x2 – x – 1 3. 3x2 – 7x – 4 4. (x2 – 12x – 5 4) 2. (2x – 3) (x - 2) (2x -1) 3. (x + 2) (2x - 1) (3x + 2) 4. 3x3 +
5. (x3 – 4x2 – 6x + 6 6. (9.6x2 – 1.93x – 1.49 B: 1. - x2 + 2x 2. 7x2 + x – 6 = 0. C. 1. -4, 3, 1 2. 2, -1 , -3 3. , 3, 2 4. -2, ,
x5 – x3 3. 2x + 13 4. + x3 + x2 – 2x + 2 5. – 8x + 7 C.P 1. –
5. 6, , - 4 6. - 2, 5, 4 C. P. 1. - 4p, -p, 2p
3m + 3 2. -13y 3. (-x3 + 3x2 – 6x + 12) 4. -14x2 – 13x + 10 5.
x4 – 3x2 + x + 6 2. a. (a – b)(a2 + ab + b2) b. (a + b)(a2 - ab + b2)

Exercises 5.1 Exercises 6.1


1. 71 2. -137 3. - 4. 97665 4. – 42.61 5. 6 6. -85 A. 1. {x : x R, x ≠ -2 or x ≠ 3} 2. {x : x R, x ≠ -2 or x ≠ 2}
3. {x : x R, x ≠ 1 or x ≠ 3} 4. {x : x R, x ≠ -5}
Exercises 5.3 5. {x : x R, x ≠ } 6. {x : x R, x ≠ -1 or x ≠ 1}
1. 14x3 - 40x2 + 12x + 24 2. 7x4 – 36x3 – 76x2 + 25
3. 7x4 – 52x3 – 28x2 + 67x + 42 4. 15x7 + 16x6 + 4x5 + 25x + 10 Exercises 6.2
5. 12x8 + 11x7 + 17x6 + 10x5 + 20x2 + 5x + 25
A. 1. 1 2. 1 3. 5 4. B. 1. 0 , 4 , -57.95, 3001.999

Baffour Ba Series, Further Mathematics for Schools Page 850


2. -0.6, 9, 401, -199 Exercises 9.4
1. 5 or 2. 1 3. 2 4. 5 C. P 1. (-29, 0) or (3, 0) 2. 1 =
Exercises 6.3
or and = 3 or
1. x – 1 2. 3. 4. 5. 6. 2
( ) ( )

Exercises 9.5
Exercises 6.4 A. 1. (1.5, 0) 2. (5.5, 7) 3. (3, 1) 4. (2, -4) 5. (3.5, 0.5) B. 1. (3,
2. 12x + 30 3. 2 4. ( )( )
5. 3) 2. (0, 1) 3. . , /

Exercise 6.5 Exercises 9.6


( )
1. 2 2. 3. 4. x2 – 1 5. 9x2 – 12x + 4 1. (3, 5) 2. (2, -1)
( ) ( )

Exercise 6.6 Exercises 9.7


A. B. A. 1. True 2. T 3. T 4. T 5. T B. 1. 3 or -6
Exercises 9.8
Exercises 6.7 1. i. . , / ii. (7, -1) 2. (10, 16) 3. (16, -19) 4. (2, 7) 5. (-2,
A. 1. - 2. + 3. + 4. + ) 6. (-18, 7) 7. ( -4, ) 8. (-7, 2) 9. i. (31, 14.5) ii. (13.5,
( ) ( )

12.25) C.P. 1. 3.: , , 4. ,


7. Binomial Theorem

Exercises 7.1 Exercises 9.9


1. A. 1. P6 + 6p5q + 15p4q2 + 20p3q3 + 15p2q4 + 16pq5 + q6 1. (8, - 8) 2. (5, 10) (1, - 6) 3. ( , and ) 4. √8 , √ , 5√
2. x7 – 7x6 y + 21x5y2 – 35x4y3 + 35x3y4 – 21x2y5 + 7xy6 – y7
3. x3 + 6ax2 + 12a2x + 8a3 4. a4 +12a3b +54a2b2 +36ab3 + 81b4 Exercises 9.10
5. x6 – 8x6 + 24x4 – 32x2 + 16 1. Int 2 :3 2. Ext 3 : 2 C.P. 1. 2: 3 (I), 3 : 2 (E) 2. Ans (-6, 2)
6. x10 + 5a3x8 + 10a4x6 + 10a6x4 + 5a8x2 + a10
Exercises 9.11
Ex 7.5 1. x = 7 2. y = 9
A.
Exercises 9.12
2. (a) 32x5 − 80x4 + 80x3 − 40x2 + 10x − 1; 1. y = 5x – 14 2. y = 4 3. y = -2x – 1 4. y = -3x + 4
(b) 81 + 54x + x2 + x3 + x4;
(c) x3 − 6x + - Exercises 9.13
1. 3x – y + 1 = 0 2. 2x + y – 4 = 0 3. 2x – 5y – 15 = 0
4. y + x = 0
Exercises 8.10
1. a. 2x + 4y ≤ 24, 4x + 2y ≤ 24, b. P = 3x + 5y c. (4, 4) = Exercises 9.14
Gh32 2. a. 12, b. 36, c. Gh3000 5. a. 20, b. 10, c. Gh12.50 6.
a. 3, b. 4 c. Gh5.50 7.a. 10, b. 22.5, c. Gh¢3250 8. 1, 9 9. x + y A.1. 5x + 3, 2. -2x – 1, 3. y = 3x, 4. y = x + 1,
≤ 70 , x + 2y ≤ 120, 3x + 2y ≤ 180, x ≥ 10 , y ≥ 20 10 … 11. i. 5. y = x+
5x + 2y ≤ 100 , x + 4y ≤ 64, x + y ≤ 25, ii. 30x + 70y , 1270 (x B.
= 12, y = 13) C. P 2. 2kg (A) , 14kg (B) 3. x + y 24, 2x + 5y Equation Gradient y - intercept
60 c. 4 tonnes of x, 20 tonnes of Y:Gh5600 d. Gh9600 4. i. x + y = 3x + 2 3 2
2y ≤ 180, x + y ≥ 105, y ≥ 40, 2y ≥ x ii. 30 ≤ x ≤ 90 y = 5x – 2 5 -2
y = -2x + 4 -2 4
Exercises 8.11
1. b. 22, 12 c. 28, 18 y= x- -

2 y – 10x = 8 5 4
Exercises 9.1
x+y+1=0 -1 -1
1. 5.39 2. 17 49 3. 15.26 4. 10 5. 16.28
y = 2x 2 0
Exercises 9.2
Exercises 9.16
B. 1. 2. PQ = PR = √ , . , / 3. i. AB = BC = √ 4. PQ = 1. x – 2y + 5 = 0 2. 2x + 7y – 11 = 0
PR 5. AB = 5, BC = √ AC = 5: isosceles, Right triangle
6. Isoscelles AB = AC = 5 Exercises 9.17
1. 3x – 2y = 1 2. x – y – 1 = 0 3. y = x – 1 4. 2x + y = 5
Exercises 9.3
A. 1. 7 3. 24 6. 12 B. 1. 10 2. 12 3. 7 4. 7 C. 1. 8 3. 10

Baffour Ba Series, Further Mathematics for Schools Page 851


Exercises 9.18 Exercises 10.10
1. A(-4, -2) 2. (1, 7) A. 1. 2. i. ii. 3. 4.

3. The points (1, 4) (6, 3) (10, 4) and (a, b) form the vertices of B. 1. i. 11 ii. 2. i. 3 ii. 3. a. i. ii. iii.
a kite. Find the possible values for a and b. b. i. ii. iii. 4. i. 10 ii. iii.

Exercises 9.19
Exercises 10.11
A. 1. . , / 2. . , / 3. . , / 1. HHH, HHT, HTH, HTT, THH, THT, TTH, TTT
B. 1. i. (2, -1) ii. 5x +2y – 8 = 0 iii. 5(4) +2(-6) – 8 = 0 2. i. 12 ii. 3. ii. . iii. 4. i. ii. 5. i. ii. iii. 6.
3. (-1, -1) 4. (5, 4)
Exercises 11.1
Exercises 9.21
1. ( ) 2. ( ) ( ) ( ) ( )
Exercises 10.1
1. mon, tue, wed….sun 2. Jan Feb…Dec Exercises 11.4
3. M, A. E 4. Boy and girl 5. Win , lose , draw 1. 6.4, 3210 2. 14.4, 450 3. 18, 1800 4. 1.4, 450 5. 4, 2100
6. Win or lose 7. 1, 2, 3, 4, 5, 6 6.9 .450 7. 5, 2700 8. 12, 1800

Exercises 10.2 Exercises 11.5


A. 1. 1 2. 0 3. (0 ), 4. 1. -8, 2 2. i. 6, -1 ii. 13 3. i. 2j ii. , 2. i. (6, -1) ii. √ 3.
i. j ii. . , / 4. a. (3, -1) b. √
Exercises 10.3
1. i. 18 ii. 30 2. i. 30 ii. 20 3. 20 4. 20 5. i. 75 ii. 20 iii. . 6. i. Exercise 11.6
1. 11N, 2210 2. 9.9N, 1070
56 ii. 52 7. x = 2 C. P: 1. i. ii. 2. i. ii.
Exercises 11.7
Exercises 10.4 a. i. ( ) ii. ( ) iii. ( ) iv. ( ) v. ( ),
1.a. b. c. 2. 0.99 3. 4. 5.. 6. a. b. c.
b. 2. ii. 1 iii. √ iv. √ v. √ c. ,-j

7. i. ii. 8. i. , ii. B. 1. a. 20 b. 2. i. 12 ii. 3. i.
a. b. c. 4. 12, , 5. i. ii. Exercises 11.8
A. i. same direction
Exercises 10.5 1. 2. 3. 4. 5. 6.
√ √ √ √ √
1. ii. a. b. c. 2. 3. a. b. c. 4. 7. 8. ii. opp dxn 1. 2. 3. 4.
√ √ √ √ √
5. ii. (6, 2) (5, 3) (4, 4) (3, 5), (2, 6) iii. . iv. (2, 2) (4, 2) (6, 2) 5. 6. 7. 8. B. 1. 2.
√ √ √ √ √
(2, 4) (4, 4), (6, 4) (2, 6), (4, 6) (6,6) B. 1. a. , b. , c. 2. ,
3. i. ii. 4. i. ii.
√ √ √
, , , 3. i. , i. 1 iii. 4. a. , b. , c. 5. a. ,b. ,c. 6. ,
, 0 7. 0 Exercises 11.9
A. 1. – 14, 1600 2. 0, 900 3. 37, 5404. 45, 220 5. – 29, 1760 B.
1. 630 2. 1150 3. i. p = 8.3 units 0160 ii. 1110, 4. 870.
Exercises 10.6
1. 2. 3. Exercises 11.10
A. 1. Parallel 2. Not 3. Not 4. Not 5. 6.
Exercises 10.7
1. i. P(Y) = ii. P(XUY) = 2. i. Not ii. yes 3. Exercises 11.11
4. 5. 10% 6. 0.03 7. a. b. c. d. A. 1. yes 2. Not 3. Yes 4. Not 5. Yes 6. Yes
B. 1. 2. 3. 4.
Exercises 10.8
1. 0.15 2. 0.13 3. 66.7% 4. Exercises 11.12
1. i. ( ) ii. ( ) iii. √ 2. i. ( ) ii. ( ) iii. ( ) iv. √
Exercises 10.9 3. a= ( ) b = ( ) ii. ( ) iii. 5 4. a. B = ( ) C= ( ) b. ( ) 5.
1. 2. a. b. c. 3. a. b. c. d. a. ⃗⃗⃗⃗⃗ = ( ) ⃗⃗⃗⃗⃗⃗ = ( ) D = ( ) b. ⃗⃗⃗⃗⃗⃗ = 5.1 6. i. Q= ( )
4.i. 9 ii. and S= ( ), ii. PR= √ 8 7. a.i. B= (2, 6) D = (1, -1) ii. ⃗⃗⃗⃗⃗ =

Baffour Ba Series, Further Mathematics for Schools Page 852


( )and ⃗⃗⃗⃗⃗ = ( ) b. equal vectors 8. i. ⃗⃗⃗⃗⃗ , ( ), ⃗⃗⃗⃗⃗ = 5.i. ( ) ( ) ii. (4.5m/s, 0500) iii. 3.5× 103 N, 2300) 6. 0.6
( ) and ⃗⃗⃗⃗⃗ ( ) ii./ ⃗⃗⃗⃗⃗ ⃗⃗⃗⃗⃗ = 10 9. (10, 2)
Exercises 13.8
Exercises 11.13 1. b. 36 2. 0.6 3. i. 10 ii. 50 4. i. 1.7 ii. 30.5
1. h = 2, k = -1 . 2. S = (2, -6) 3. C = (0, 4) , D = (-5, 3) 4.
Ex 14.1
( ) 5. a , a A. (x – 2)2 + (y – 2)2 = 52; (x + 3)2 + (y + 1)2 = 42
(x – 0)2 + (y + 4)2 = 1; (x + 1)2 + (y – 2)2 = 102 B. (x – 2)2 + (y
Exercises 11.15 + 6)2 = 62; (x + 3)2 + (y - 0)2 = 122 ; (x – 4)2 + (y + 4)2 = 7.52;
1. a. 21km b. 0630 2. i. 1252km ii. 1400 3. i. 23km ii. 2430 4. (x + 1)2 + (y – 2)2 = 5.52 C. 1. (x + 2.2)2 + (y + 4.4)2 = 2.
i. 121 km ii. 1020. 5. 13 km 6. 1.6, 2.2 7. 1680 238km/h 2 2 2 2
(x - 14) + (y – 10) = 100 3. (x + 5) + (y – 4) = 81 4. (x +
Exercises 12.1 16)2 + (y – 9)2 = 169 5. (x + 7)2 + (y – 5)2 = 18
A. 1. 102N 2. 10N 3. 7N, 4. 28N B. 1. 91N, 2070 2. 56N, 2580
3. 6N, 1290 4. 92N, 3440 Exercises 14.2
A. 1. x 2 + y 2 = 9 2. (x -1 )2 + (y – 2)2 = 25 3. (x - 3)2 + (y –
Exercises 12.2 2)2 = 4 4. (x + 5)2 + (y – 3)2 = 37 B. 1. 2. 3. 4.
1. 36.8N, 200 to the 13N force. 2. 15.7N, -1720 3. (-5√ + 6√ )i
Exercises 14.3
+ (6 + 5√ ) j, 75.70 4. F(17N, 0570 B. 1. 4N,570 2. 41N, 310 A. 1. x2 + y2 - 6 x - 10y + 25 = 0 2. x2 + y2 + 4x - 6y + 12 = 0
3. x2 + y2 – 2x + 6y + 6 = 0 4. x2 + y2 – 4x + 4y – 17 = 0 5. x2
Ex 12.2C + y2 - 10y + 9 = 0 B. 1., (1, 2), 5 2. (2, −3), 3 3. (−1, 0), 2 4.
1. 200 2. 120 3. 250 4. 1520
(−3, − 7 2 ), 6 5. (1, − 3 2 ), √
Exercises 12.4
1. 3.06N, 450 2.18.8N, 1500 3.53.5N, 1170 4. 34.6N, 100
Exercises 14.4
Exercises 12.5 1. (5, 1) ; 5 2. (-2, 3) ; 6 3. (-5, -3) ; 1 4. . , /;
1. 47N, 1010 2. 6N, 1010 5. (0, 0) ; 2

Exercises 12.5B Exercises 14.5


1. 8.06 and 5.86 2. 9.96kN and 7.77kN 3. 9N and 7.3N 5a. A. 1. ( 8, ); 8 2. ( , ); √ 3. ( , ); √
31.5N, 37.5N, 49N b. 113N, 56.6N, 98N 6. 0.946N, 0.695N √
4. ( , ); 5. ( , , ); 4 6. . , /; 2 B. 1. (-12 , -5) ; 3
2. (-13 , -14) ; 1 3. (3 , 16) ; 1 4. (3 , 5) ; √
Exercises 12.6
1. 62.5N
Exercises 14.6
A. 1. x2 + y 2 + 2x − 6y − 106 = 0 2. x2 + y 2 − 11x − 5y + 28 =
Exercises 13.2
0 3. x2 + y 2 - 6x − 10y = 0 4. x2 + y 2 + x − 9y − 124 = 0 5. x2
1. u = 12.247m/s 2. 3. A = 2.02s, 606m
+ y 2 − 4y − 94 = 0 B. 1. x2 + y 2 + 2x − 2y − 32 = 0 2. x2 + y 2 +
2x + 16y + 90 = 0, 3. x2 + y 2 + 26x + 28y + 324 = 0, C. P : x2
Exercises 13.3
+ y 2 + 2x − 6y − 26 = 0,
1. 45.9m 2. 20s 3. a. 2 b. 20 4. a. 29.39 b. 2.45 c. 2.45 d.
24m/s (conservation of energy)
Exercises 14.7
1. x2 + y 2 + 2x − 10y + 24 = 0 2. x2 + y 2 - 4x − y − 45 = 0,
Exercises 13.5
3. x2 + y 2 - 4x + 8y = 0 4. x2 + y 2 - 2x − 9 = 0 5. x2 + y 2 +
1. ii. 25 iii. 43.3 N 2. ii. / T / = 79N iii. F = 48.4N
2x + 6y + 5 = 0 6. 11x2 +11 y 2 - 43x − 104y + 356 = 0,
3. 110.3N 4. i. 102. 9N ii. 127.4N 5. b. i. 965N, ii. 170N iii.
965N iv. 965N c. 160
Exercises 14.8
Exercises 13.6 A. 1. 4 2. 10 3. √ 4. 5. B. 1. 25 2. 9
A. 1. 0.08Ns 2. 75Ns B. 1. 16000 kg m 2. 450 m 3. 3. 4 4.
2.4 kg m 4. 6500 N. 5. Three forces (-10i + 4j), (16i – 10j )
and 3i N in the x – y plane act on a body of mass 3kg. Find the Exercises 14.9
magnitude and direction of the acceleration of the body. A. 1. 3x + y – 8 = 0 2. 4x + y + 1 = 0 3. x - 4y + 17 = 0
4. 3x - y = 0 5. 9x + 4y – 5 = 0 6. 5x + 7y + 31 = 0
Exercises 13.7 B. 1. y – 4 = 0 2. 2x + y + 11 = 0 3. x - 2y + 4 = 0 4. 4x + 3y
2. 13m/s, 5m/s 3. 7.5m/s 4. (12.9m/s, 015 0) 5. 8000m/s 6. 60m/s – 32 = 0 5. 2x + 5y - 4 = 0 6. The points of intersection are (1,
C.P. 1. 2.49 m/s 2. i 8.4m/s, 392N; 6m/s 3. i. ( ) m/s ii. ( ) 3) and (−1, −1). The mid-point of these is (0, 1) which is the
m/s, ( ) m/s 4. i. 480g m/s. ii. (240 + 30v) gm/s b. 8m/s centre of the circle, and hence y = 2x + 1 is a diameter. The
tangents are 2y + x = 7 and 2y + x = 3 respectively. 7. The

Baffour Ba Series, Further Mathematics for Schools Page 853


points of intersection are (1, −2) and (2, −1). The tangents are y Exercises 16.1
= −2 and x = 2 respectively. They intersect at the point (2, −2) 1. 15, 22, 29, 36, 43 2. 2, -3, -8, -13, -18 3. – 12 4. 37
8. The points of intersection are (4, 2) and (2, 6). The tangents 5. i. 35, 62 ii. -19, -46
are 3y + x = 10 and y = 3x respectively. They intersect at the
point (1, 3). C. P: 4. -8, 4, -5 Ex 16.2
A. 1. 2n + 1 2. 3. 8 4. -5n + 3 5. 6n + 13
Exercises 14.10
1. √ 2. √ 3. √ 8 C. P. 1. 12 C. 1. 4n – 11 2. −11n + 2. 3.i. 6 – 2n ii. 4. 7n + 2; 137
7. = − 3n + 5; -22
Exercises 14.11
1. 3x + 4y – 43 = 0, 3x – 4y + 34 = 0 2. 2x – y + 5 = 0; x + 2y = Exercises 16.4
0 3. 4x + 5y + 29 = 0 ; 5x - 4y – 15 = 0 1. 1863 2. i. - 110 ii. - 4550 3. 1717 4. 52

Exercises 14.12 Exercises 16.5


B. 1. 26 2. i. (k, -2) ; r2 = – 11 – k2 ii. 47 3. 3, or -3 A. 1. a = 40, d = - 3, S20 = 230. B. 1. i. - ii. k - (n – 1)
4. r = 2, k = 11 or k = 9 5. i. a. 5k2+ 8k + 4 b. 5k2- 28k + 40
| | iii.
b. | | ii. k = 1

Exercises 14.13 Exercises 16.6


A. 1. (3, 1) (-3, -1) 2. (-1, 2) 3. (-1, 3) (3, 1) 4. (4, -1) 1. 4600. 2. 8200, 7400, 6600, 5800, 5000, 4200, 3400, 2600,
5. (2, 1) (1, 0) 6. (3, 1) (-1, 1) B. C. D. 1800, 1000

Exercises 14.15 Exercises 16.7


1. x2 + y2 – 6x + 2y – 26 = 0 2. x2 + y2 – 4x - 8y – 5 = 0 1. 1, 1, 1, 1, 1, 2. 1536 ; 1572864 3. 1458

Exercises 14.16 Exercise 16.10


A. 1. 3x + 2y – 24 = 0 2. 18x - 20y – 7 = 0 3. y – 6 = 0 4. 9x – 5y + 1. 93 2. 19946 3. 6 4. 2, 5. R = 2 ; 765; 1,761, 205, 026..
2 = 0 5. 10x – 8y – 4 = 0 6. (3, -7) 2y – x + 17 = 0 B. 1. 4x - 10y – 29
6. 248832
= 0 2. 11x + y – 8 = 0 3. y + 6 = 0 4. x – 4 = 0

Ex 14.17 Exercises 16.11


1. 3x2 + 3y2 + 4x – 14y + 7 = 0 2. x2 + y2 – 2x – 4y - 3 = 0 1. Number at year 6 = 5 × 46 = 20480; Total = 27305
3. 5x2 + 5y2 – 26x – 48y + 44 = 0 (circle) 2. Ghȼ3400 3. 3559.57 4. 5. i = 284.1.04n; ii. 374
6. 1,048, 575 C. P : 1. Total distance = 10 + = 190m
4. 5x2 - 3y2 – 46x – 88y + 205 = 0 5. . / →. / circle
centre (3, -3) r = √ Exercises 16.12
Exercises 14.19 1. 6 2. 3. A =

A. 1. 3x2 – y2 = 48 2. 3x2 + 4y2 = 48
Exercises 16.13
Exercises 14.20 1. 4, 7, 11 2. 2, -3 3. 5, 8, 11 4. 2, -3
1. x2 + y2 = 16 2. (4, 1), 3 3. x2 + y2 = 4
Exercises 16.14
Exercises 14.21 A. 2. 16 3. 4. 5. 6. B. 1. 3.
A. 1. 3x2 + 3y2 - 8x - 26y - 55 = 0 2. 3x2 + 3y2 + 36x - 38y +
159 = 0 3. 8x2 + 8y2 – 4x – 4 4. (4, 0), 2 Exercises 16.16
Exercises 15.1 1. (a) (a) −4, −15; 51 − 11n; (b) 8 , 9 ; (9 + 4n);
1. (x – 3)2 = 16(y – 1) 2. (y + 4)2 = -56 (x – 6) 3. x = 4 (c) 135, 405; 5(3)n−1; (d) 7.6, 6.8; 10.8 − 0.8n
4. y2 – 6y – 12x = 15 (e) −24, 16; (−1)n −1 2 n −1 3 5 − n; (f) , ,
5
Exercises 15.2 3. 6th term = (1.2) 2.488 and S10 25.96 8. n(n + 1)(4n + 5).
1. (y - 5)2 = 12(y + 2) 2. 4y = (x + 2)2 + 16 3. y2 – 2y – 10x +
76 = 0 4. x = (y + 2)2 – 6 5. (y + 2)2 = - 4(x – 1) Ex 17.1
A. 1. 15 2. 3. 9a4 4. 5. 6. ad2 B. 1.3 2.
Exercises 15.3 3. 288 4. 243 5. 36
1. y = -2 2. (9, 3) 3. (-2, -3) 4. x = 1 5. x= -2 6. i. (7, 2) (6.9,
-2) , x = 7.1 ii. (4, -14) (4, -13.8) y = -14 Exercises 17.2
0
1. 1 2. 3. 1 4. 4096 5. =1

Baffour Ba Series, Further Mathematics for Schools Page 854


Exercises 17.3 Exercises 17.19
B. 1. 27 2. 9 3. 8 4. – 2 5. 16 6. 4 7. 343 8. 243 9. 36 A. 1. 9 2. 5 3. 8 4. 5. 6. 7. 27 8. B. 1. 21 2.
1. 3 2.
13 3. 99 4. 5. 6. 7 C. 1. 9 2. 1.0718 3. 1.54 4.

Exercises 17.4
Exercises 17.20
B. 1. 9 2. 3. 4 4. 2 5. 6. C. 1. 2. 3.
A. 1. 1.67 2. 3 3. 1 4. 5. 8 B. 1. 2. 25.5 3. 307 4.
4. 5. -2.1 5. 9 C. 1. 10 2. 2 3. 6 4. 5.09 D. 1. 1.42 2. 8 3.
1.75 4. 0.75 5. 146.77 C. P . 1. a. 4.15 or 1.84 2. (a) 3
Exercises 17.5 (b) 3 3. (a) 4 (b) 1 (c) 3.71
A. 1. 5 2. 2 3. 2 or 4. 3 5. 6. 7 7. B. 1. 4 2.
Exercises 17.21
3. 4 4. 2 5. 6. 7 -1 8. 6 9. 10. -3 C. 1. 2 2. A. 1. 1 or 109 3. 1 or 103 4. 1 or 10 5. 21.5508 6. 1064
3. 4. 5. 3 6. -3 7. 0 8. B. 1. 2 3. 10 or 5. 1.88 C. 1. 3 2. 10 3. 20.98 4. 1 5.
10 or 104 6. D. 1. 16 2. 2 3. 21.5443 4. 3 or 9 5. 10
Exercises 17.8
A. 1. lg 4 2. log 2 6 3. log6 1296 4. log5 625 5. lg 2500 Exercises 17.22
A. 1. (3, 2) 2. . , / 3. (1, 1) (-2, 4) B. 1. (24.14, 4.14) or
Exercises 17.9
1. 2 + 3 log32 2. 2 + log 37 3. 2 + log 52 + log 53 4. 1 + log 83 + (- 4.14 , -24.14 ) 3. (17.78, 56.23) 4. . , /
log 85 5. + log 27 6. + log 2
Exercises 17.24
Exercises 17.10 1. 23 yrs 4. a = 0.4, b = 3.3 5. n =1.06, C = 65887
6. a = 1.38 × 10−3, b = 2.08 7. a = 183.95, n = 0.17.
A. 1. lg ₃ 75 2. 3. log₂ . / 4. log ₉ 5. log ₅ (-3) CP: a = 8.0, b = 0.99
6. log ₅ 15 B. 1. 3 + 5 log ₃ 5 2. log ₃ ( ) 3. log ₅ 4. log ₂
Exercises 18.7
Exercises 17.11 A. 1 ,2. , 3. √ , 4. , 5. ,6. B. 1. ,2 , 3. ,4.

A. 1. 5 2. 3. 5 4. 5. - 6. -3 B. 1. -2 C. P:
2. – 4 3. 2 log 4.- 5. – 3 6. 2 7. -3 8. 2 C. 1. 2 2. 1
Exercises 18.15
3. 4. 4 5. 6. – 2 7. 1 8. – 1 9. D. 1. 1.3 2. 0.85 1. 450 2. 2250 3. 1350 4. . 5. i. ii. , iii. iv.
3. 0.51 4. 1.5 E. 1. 8 2. 4 6. i. ii.

Exercises 17.12
Exercises 18.17
1. 2. 2 3. 2.7 4. 1 5. 0.3010 √ √
A. 1. , , 3. √ √ , √ √ , √ –1
Exercises 17.13
Exercises 18.26
1. 3; 2. – 2 3. 3 4. 1 5. 6.
5. 3290

Exercises 17.14 Exercises 20.3


A. 1. 0.632 2. 7.12 3. 4.05 4. 4.95 5. 3.88 6. 4.36 A. 1. 3x2 – x 2. – 2x 3. 5x4 + 4x3 – 15x2 4. 2x + 5
5. 2x – 1 6. y = 18x8 + 3
Exercises 17.15
A. 1. 2 or 1 2. -1 or 2 3. 1.53 4. 2.78 5. 2 Ex 20.6
A. 1. 4x + 5 3. 9x2 – 26x + 18 5. 110x3 – 56x2 6. 24x3- 21x2+
Exercises 17.16
A. 1. 0 or 1 2. 1.20 or 0.58 3. 1.77 4. – 0.16 B. 1. 1.13 2. 3. 4x + 8 B. 2. 20x + 11 4. 1 + C. a. (x2+ 1)3 – 5 b. 6x(x2
1.46 4. 0 + 1)2 c. 5x4 + 3x2 – 10x

Exercises 17.17 Ex 20.7


A. 1. 0.8891 2. 0.1761 3. 1.795 4. 0.6532 5. 6. 2.68 A. 1. 3. 5. B. 2. 4.
( ) ( ) ( ) ( ) ( )
7. -2.318 8. 1.1275 C. 1. 1.113 2. 2.0478 3. 1.2920 4. 1.5440
( )
5. 0.3413 6. - 0.2519 7. - 8614 8. 2.8614 9. 0.2519
6. 8.
Exercises 17.18
A. 1) 2 2) 4 3) – 2 4) 1 Exercises 20.10

Baffour Ba Series, Further Mathematics for Schools Page 855


A. 1. 2. 3. 4. - 1 5. Exercises 20.22
A. 1. (-2, 16) max, (2, -16) min 2. (0, 0) min, (2, 4) max
B. 1. 2. 3. 4. 5.
( )
3. . , / max, (12, -144) min 4. . , / max, (3, -7) min
6. C. 1. 2. 3. 4.
√ 5. . , / min C. 1. 72 2. -2, 8 10 3. (6, 3)
( )
5.
B. 1. min (2, -2) 2. Min (-2, -3) 3. Max (-3, -54) 4. Max
Exercise 20.11 . , / 5. Min (0, 2) 6. Max (0, 7) C. 1. Max(1, 5) , min (2, 4)
A. 1. 2. 3. 4. not defined 5. 2. Max (-2, 89) , min (3, -16) 3. Max (4, 31), min (1, -23) 4.
B. 1. 2. 1 3. 0 4. 4. Max (-1, 0), min (3, 8)

Exercises 20.12 Exercises 20.25


1. 1. (2, 8) (-2, 8) 2. (1.5, -1.25) 3. (2, -12 ) A. 1. 18cm3, x = 1 2. 25, 25 3. 5 and 5 4. 50m × 50m 5.
250m, 500m, 125000 m2 6. 5cm × 5cm 7. 10 × 10 sq units 8.
4. . , / or (0, 1) 5. (3, 0) or (1, 4) 6. (-2, 8) 30, 15. 9. 3; 432 cm3 10. (50, 125); 6250 ft2
C.P : (0, 2) (0, -2)
B. 1. 2.41, 4.86 2. 4.3, 4.3; 464.8cm2 3. 1.03, 1.09; 3.52m2 4.
Exercises 20.13
A. 1. 3x – y + 5 = 0 2. 4x – 3y – 10 = 0 3. 20x – 11y + 2 = 0 √ , 2 4√ 5 C. P 3. 4 × 12 × 6

Exercises 20.14
Exercises 21.1
A. 1. 4x – y – 4 = 0 2. 26x – y – 48 = 0 3. x + y = 0 4. 8x + y
– 5 = 0 5. 15x – y + 9 = 0 A. 1. x3 + c 2. x5 + c 3. + c 4. + c 5. -2
+ c B. 1. + c 2. a + c 3. +c
Exercises 20.15
A. 1. y = 13x – 18 2. 6x – 2y – 3 = 0 3x + y + 3 = 0 . , / Exercises 21.2
3. 2x + y – 4 = 0 ; x + y + 4 = 0 4. y = 2x – 1 5. c = 6. y =
A. 1. + c 2. – + 2x + c 3. – + 3x + c
x – 2 C.P: 1. (0, 0) (-2, 2) ; x – y = 0 x – y + 4 = 0 2.
4. + + + 2x + c 5. x – + c B. 1. x3 – 2 +c
2
Exercises 20.16 2. x – x + c 3. – + 5x + c 4. + 2x – +c
1. 2, -5 2. 2, 15 3. 2, -8 4. – 4 5 . – 5.5 6. 2, 0, - 24
5. – +c 6. – + c C. 1. + +c 2. +
Exercises 20.17
A. 1. x + 2y + 1 = 0 2. x + 3y + 3 = 0 3. y = x 4. x + 8y +49 = +c 3. + + c 4. x – + c 5. 3x – 8 +c
0 5. x + y + 1 = 0 6. x = 0 B. 1. x + 3y + 6 = 0 2. x – y – 2 = 0
and y + x – 3 = 0 3. x – 2y + 1 = 0 4. k = 5. y = Exercises 21.3
6. T: y + 3x = 0; y = 6x - 6√ N: x – 3y = 0; 6y = x + √ A 1. y = x3 + - 2. 4, 0 , 3. y = + – 8 C. 1. x2 – 5x
+ c 2. + + + 10x + c 3. + - + 5x + c 4. –
Exercises 20.18
a. 1. 0.75s, 14 ft 2. i. v = 144 ii. 4.5s iii. 324 3. i. v = 80 + 32t x2 + c 5.
ii. 2.5 iii. 240 4. i. 3t2 – 18t + 15, ii. 15m/s, iii. 9m and 23m
iv. 6m/s2 5. i. 21 ii. 6.67s iii. 21m/s, 300m 6. i , ii. 7. i. Exercises 21.4
2.5 ii. 8. i. k = 5 ii. q = 3 9.i. (26, 0) ii. 77m/s iii. 103m/s 2 A. 1. 7 2. 16 3. 4. B. 1. – 30 2. 3. 4. – 60
10. 12.1 m , 1.6 s C. 1. 2. 0 3. 4. -92 D. 1. -9 2. 5 3. 2.7 4.

Exercises 20.19 Exercises 21.6


1. 4.5 2. 40 3. 450 4. 1080 5. 40 6.. i. 1.4 ii. 14 7. 2560
A. 1. ( ) + c 2. ( ) + c 3. ( ) +c
8. 5cm2/s 9. 1152 10. 0.98 12. 527.8
4. ( ) + c 5. ( ) + c B. 1. 2. ( ) +
Exercises 20.20 ( ) + c 3. - 8( ) ⁄ ⁄
+ c 4. ( ) 5. +c
1. 25.13cm2 2. 5.03cm3 ( )

Exercises 20. 21 Exercises 21.7


1. 9% 2. a. 1.25% b. 1.25% 3. 4 4. 9 5. 2 C.P: 1. 2. 6% 3. 3% A. 1. 20 2. 3. 4. 5. B. 1. 2. 1 3. 2 4.

Baffour Ba Series, Further Mathematics for Schools Page 856


Exercises 21.10 ⁄
1. 10.5m/s , 13.2m 2. i. 120 ii. 30km/s iii. 2400 km C. i. 0 1 ii. 0 1 iii. [ ]
3. v = 6t2 – 36t + 48; A = 2 and B = 4; 1440m ; 1456m 4. -9 ⁄ ⁄

Exercises 21.12 Exercises 25.7


1. (a) 4 (b) c. d. 2. 3. 4.. B. 1. 2. 3. A. 1. 5 2. 2 3. 9 4. 43 5. 0 6. 42
B. 1. -6 2. 36 3. 32 4. 22 5. -52 6. 124
4. i. 6 ii. iii. iv. C. 1. 2. 3. a. -3 b. 21 4.
a. -3 and 3 b. 36 5. 6 6. 30 7. 18 8. a. -4 and 2 b. 9.a. Exercises 25.10
A. 1. 125 2. – 18 3. – 84 6. 0
1 and 2 b. 10. a. -2 and 5 b. 11. a. -2 and 5 b. 12. 8
C.P : 1. 27 2. B. 1. 57 3. -254 5. 6

Exercises 25.11
Exercises 21.13
1. 3 2. 3 or -2 3. 4 or -5
A. 1. i. 4 ii. 6 2. 3. 4. 2 5. 6. 24 7. 21
B. 1. 2. 20 3. 4. 5. Exercises 25.12
1. 3 2. -1.41, 1.41, 1
Exercises 21.14
Exercises 25.13
A. 1. 156 2. 19 3. 364 4. 540 5. 10.6 B. 1. b. 43.5
2. b. 52 3. b. 170 A. 1. 0 1 3. 0 1 5. 0 1

Exercises 23.1 Exercises 25.16


1. 0.47 2. 0.0032 3. 0.019 4. a. A = 0.0046 b. 0.069 c. 0.028 d.
0.56 e. 0.168 5. a. (0.9)5 b. 0.0086 c. 0.0081 d. 0.00081 6. 1. [ ] 3. [ 8 ]
0.17 8
8
Exercise 23.2 5. [ 8 8]
1. i. ii. 2. i. 0.441 ii. 0.973 iii. 0.657 3. i. ii. iii.
iv. Exercises 25.17

Exercises 23.3 1. a. ( 8 + b. ( + c.( +


1. 2.
Exercises 25.18
Exercises 25.2
B. 1. 0 1 2. 0 1 3. 0 1 1. [ ] 2. 0 1 3. [ ] 4. 0 1 6. 0 1
4. 0 1

Exercises 25.20
Exercises 25.3
8 A. 1. (1, -3) 2. (-2, 5) 3. (1, 1) 4. (-2, -3) 5. . , /
A. 1. 0 1 2. 0 1 3. 0 1 4. 0 1
8
B. 1. (10, 15) 2. . , / 3. . , / 4. . , /
B. 1. [ ] 2. [ ] 3. [ ] C. 1. (-2, -3) 2. (6, -11) 3. (7, -10) 4. (5, -10)
8 8
C.P : 2. [ ]6 7=[ ] 3. a. 4, 2 b.
Exercises 25.4
1. 0 1 2. 0 1 3. 0 1 4. 0 1
8 Exercises 27.1
2. b. 0.973 c. 7.95 kg 4. b. i. 8.3 ii. 15.7 d. 16500
Exercises 25.6
1. 4, - 6, 9 2. 3, -1, 7 3. 4, -6, 9 4. 0 1 5. 0 1 Exercises 28.1
1….. 2. b. 0.973 c. A = 7.95 kg 4. d. 16500
B. 1. (6, 3) 2. 0, 3, -6 3. -6, 9, - 17 4. -5, 4 -1
Exercises 28.2
1. 0.67 2. 3. i. 0.115 4. 0.7.

Baffour Ba Series, Further Mathematics for Schools Page 857


REFERENCES
George Humphrey, New Concise Mathematics 4, Mathematics Composition Setters Ltd. Salisbury ,
Wiltshire, 2003

V. W. Ferris and J. N. Busbridge, Modern Mathematics for Secondary Schools, Book 3, 4 & 5, Evans
Brothers Limited, Ibadan, 1978

Scottish Mathematics Group, Mordern Mathematic for Schools, Second Edition, Butler and Tanner Limited,
1974

New Mathematics for Secondary Schools, West African Editon 4, Afram Publications, Accra, 1981

P. J. F Horril, Pure Mathematic 1 & 2, Longman Group Limited, Essex, 2008

GRE Premier, Kaplan Publishing, New York, 2015

Teacher Education Division, Further Algebra, Ministry of Education Youth and Sports, Accra, 2007

Joint School Project Mathematics, Metric Edition 4S and 5S, Longman Group Limited, 1980

Ghana Senior Secondary School Mathematics, Books 1, 2 and 3, Ministry of Education, Accra,1993

Mathematical Association of Ghana, Mathematics for Senior Secondary Schools Books 1, 2 and 3, Addison
Wesley Longman Limited, 1997

Revise for GSCE Mathematics, Intermediate Tier, University Press, Cambridge 1997

James K. Morton, Richard J. Swinney, GCSE Mathematics, Intermediate Level Key Stage 4

Baffour Ba Series, Further Mathematics for Schools Page 858


Baffour Ba Series, Further Mathematics for Schools Page 859

You might also like